MCQ for PBS 1. a. b. c. d. e.
Cystic fibrosis of the pancreas is inherited as autosomal recessive islets of Langerhans are affected diagnosis is pssible by DNA analysis all the above a and c only
2. the following are clinical criteria of Bank ad Wise a. pulmonary b. neurological c. renal d. all the above e. a and c only 3. antibiotics in acute pancreatitis a. must be given to all cases b. should not be given c. given only to modarate & sever cases d. should cover anaerobic & Gam positive bactria e. none of the above 4. pancreatic carcinoma a. occure usually at the age of 50years b. usually is cystadenocarcinoma c. more favaorable in the body and tail d. less favourable if occure with back pain e. all the above 5. infected pancreatic necrosis a. is a collection of pus around the pancreas b. diagnosed by CT scan c. more serious than pancreatic abscess d. all the above e. none of the above contrast-enhanced computed tomography (CT) remains the gold standard in the diagnosis of pancreatic necrosis 6. secondarey survay in polytrauma a. should be done in the first hour b. done as soon as the patient is stable c. detects life threateing problems d. a and c e. none of the above 7. spontaneous ruptur of the spleen a. occure less in tropical countries b. the spleen may be enlarged c. malaria is a common cause d. all the above e. a and c 8. a. b. c. d. e.
1
spleic aretry aneurysm incidence is around 5% usually symptomless more conmmon in males all the above non of the above
9. wolf skin graft a. used to cover large area of burn b. success rate is less than split thckness graft c. both thighs are common donor areas d. usually contracts e. none of the above 10 . medullary carcinoma of the thyroid a. is a tumor of the G cells b . may occur in MEN type 1b c. familial in 70% of the cases d. diagosed by low level of calcitonin e. amyloid stroma is charactrestic 11.solitary thyroid nodule a. 70% are follicular adenoma b. 30% are malignant c. more favourable in young males d. all the above e. none of the above 12. rupture of the diaphragm a. is a life threatenig problem b. usually missed c. poly galctan suture is used for repair d. all the above e. noe of the above 13.
Varicose ulcer is: a.Usually very painful& shallow b.Deep with much necrotic tissue c.More common with primary varicose vein d.A & C only e.None of the above
14. Complication of vascular graft includes All the following except: a.Infection b.Aneurysm c.Graft failure d.Hemorrhage e.a & c only 15. Adequate urine out put for adult postoperative surgical patient is greater than: a.35 ml per hour regardless of body size b. 50 ml per hour regardless of body size c. o.5 ml per kg. per hour d. 1 ml per kg. per hour e.1.5 ml per kg. per hour 16. Gangrene: a. is necrosis of tissue b. The cause may be venous occlusion c. usually painful d.All the above e.None of the above
2
17. Lynphedema: a. may be Congenital b. should be bilateral c. may be pitting in early stage d.A & C only e. None of the above 18. Suprapubic cystostomy: a. indicated in case of bladder out flow obstruction b. indicated in case of urethral injury c. indicated in case of urethral stricture d. All the above e. B& C only 19.differential diagnosis of Acute scrotal swelling in young adult includes all the follow except: a. Incarcerated inguinal hernia . b. Torsion of testes c. Acute epididymo- orchitis d. Teratoma e.. Seminoma 20. Horizontal spread of infection across the external sphincter can result in which type of anorectal abscess: a. ischiorectal b. perianal c. supralevator d. intersphincteric e. intermuscular 21. All of the following statements are true of diffuse esophageal spasm EXCEPT: a. chest pain is frequently seen b. high amplitude esophageal contraction are present. c. it is best diagnosed with barium esophogram. d. usual surgical treatment is long esohagomyotomy. e. most patients do not have significant coronary artery disease. 22. Gastroesophageal reflux is BEST diagnosed with: a. radiography b. 24-hour pH monitoring of lower esophagus c. esophagoscopy d. documentation of a decrease in esophageal pH after HCI is placed in the stomach . e. acid-clearing swallowing test . . 23. The risk of esophageal cancer increases with all of the following EXCEPT: a. alcohol ingestion. b. smoking . c. chronic ingestion of hot beverages. d. aflatoxin. e. poor oral hygiene.
3
.24. The treatment of an esophageal burn with a caustic agent may include all of the following EXCEPT: a. expeditious administration of an antidote. b. induction of vomiting. c. steroids and antibiotics. d. bougienage. e. gastrectomy. 25. All of the following substances are irritating to the peritoneum EXCEPT: a. bile. b. meconium. c. blood. d. gastric content. e. pus. 26. Complications of truncal vagotomy and pyloroplasty include all of the following EXCEPT: a. dumping syndrome. b. recurrent ulcer. c. diarrhea. d. alkaline reflux gastritis. e. steatorrhea. 27. . Gastric polyps: a. are most commonly adenomatous. b. require gastrotomy and removal if greater than 2 cm and are pedunculated. c. are rarely multiple. d. are clearly premalignant. e. are more frequent in achlorhydric patients. Hyperplastic polyps are among the most commonly observed gastric polyps 28. . Vascular compression of the duodenum resulting in obstruction: a. is present primarily in patients who are overweight. b. should be given a trial of conservative management. c. is common in pediatric patients. d. is best diagnosed by identifying a "double bubble" sign on abdominal x-ray. e. includes as medical therapy lying in the supine position after meals 29. Conditions associated with gastric cancer include all of the following EXCEPT: a. higher socioeconomic groups. b. pernicious anemia. c. chronic atrophic gastritis. d. adenomatous polyps. e. a high intake of dietary nitrates 30. . Patients with morbid obesity have an increased incidence of all of the following EXCEPT: a. gastric carcinoma. b. diabetes. c. stroke d. gallbladder disease. e. joint deterioration.
4
31. All of the following contribute to malabsorption following truncal vagotomy and antrectomy EXCEPT: a. increased rate of gastric emptying. b. poor mixing of pancreatic secretions and bile salts with food. c. increased release of secretions and bile salts with food. d. decreased small intestinal transit time. e. malabsorption of fat and carbohydrates. 32. All of the following statements are true about patients with carcinoid tumors EXCEPT: a. they often have evidence of serotonin production. b. tumor growth is often slow. c. the majority have carcinoid syndrome. d. they have a much better prognosis if the tumors are less than 2 cm. e. the combination of streptozotocin and 5-fluorouracil can often result in objective response. 1. A pregnant woman in her 32nd wk of gestation is given magnesium sulfate for pre-eclampsia. The earliest clinical indication of hypermagnesemia is a. Loss of deep tendon reflexes b. Flaccid paralysis c. Respiratory arrest d. Hypotension 17. The chief surgical risk to which patients with polycythemia vera are exposed is that due to a. Anemic disturbances b. Hemorrhage c. Infection d. Renal dysfunction 28. The surgeon should be particularly concerned about which coagulation function in patients receiving anti-inflammatory or analgesic medications? a. APTT b. PT c. Reptilase time d. Bleeding time 29. The substrate depleted earliest in the postoperative period is d. Glycogen 26. Signs and symptoms of hemolytic transfusion reactions include a. Hypothermia b. Hypertension c. Polyuria d. Abnormal bleeding
5
27. A patient suspected of having a hemolytic transfusion reaction should be managed with b. Fluid restriction d. Steroids e. Fluids and mannitol 53. In a hemolytic reaction caused by an incompatible blood transfusion, the treatment that is most likely to be helpful is e. Stopping the transfusion immediately A 22-months-old child develops maculopapular rash, after 4 days of high fever, associated ?with posterior occipital lymphadenopathy. What might be the diagnosis A. Measles B. Rubella C. Roseola infantum D. Erythema infectiosum ?In which of the following layer of skin are melanocytes located A. Stratum corneum B. Stratum granulosum C. Stratum spinosum D. Stratum basalis :Perinaud syndrome occurs due to lesion in A. Superior colliculi B. Inferior colliculi C. Red nucleus D. Thalamus 33. The enteric fluid with an electrolyte (Na, K, C1−) content similar to that of Ringer’s lactate is a. Saliva b. Contents of small intestine c. Contents of right colon d. Pancreatic secretions Which of the following cerebral lobe is usually spared in Alzheimer disease? A. Frontal lobe B. Temporal lobe C. Parietal lobe D. Occipital lobe Which of the following disease is characterized by rapidly progressive dementia with myoclonus? A. Alzheimer disease B. Pick disease C. Creutzfeldt-Jakob disease D. Vascular dementia 34. Which of the following medications administered for hyperkalemia counteracts the myocardial effects of potassium without reducing the serum potassium level? b. Sodium bicarbonate
6
c. 50% dextrose d. Calcium gluconate e. Insulin
32. Hypocalcemia is associated with a. Acidosis b. Shortened QT interval c. Hypomagnesemia d. Myocardial irritability 36. Which of the following characteristics of this patient might increase the risk of a wound infection? a. History of colon surgery b. Hypertension c. Male sex d. Receipt of chemotherapy 38. A correct statement regarding 5% dextrose in 0.9% saline is which of the following? a. It contains the same concentration of sodium ions as does plasma c. It is isosmotic with plasma d. It has a pH of 7.4 e. It may cause a dilutional acidosis 39. Correct statements regarding lactated Ringer’s solution include which of the following? d. It has a pH of less than 7.0 Isotonic saline solutions contain 154 meq/L of both sodium and chloride ions. Each ion is in a substantially higher concentration than is found in the normal serum (Na=142 meq/L; Cl=103 meq/L). when isotonic solutions are given in large quantities, they overload the kidney’s ability to exerete chloride ion, which results in dilutional acidosis. They also may intensify preexisting acidosis by reducing the base bicarbonate/carbonic acid ration in the body. Isotonic saline solutions are particularly useful in hyponatremic or hypochloremic states and whenever a tendency to metabolic alkalosis is .present, as occurs with significant nasogastric suction losses or vomiting Administration of lactated Ringer’s solution is appropriate for replacing gastronistestinal losses and correcting extracelllular fluid deficits. Containing 130 meq/L sodium, lactated Ringer’s is hyposmolar with respect to sodium and provides approximately 150 mL of free water with each liter given. although ordinarily not a significant “physiological” to enable administration of large amounts without affecting the body’s acid-base balance significantly. It is worth noting that both isotonic saline and lactated Ringer’s are acidic with respect to the plasma: 0.9% NaCl/5%dextrose has a pH of 4.5; lactated .Ringer’s has a pH of 6.5 42. Prophylactic regimens of documented benefit in decreasing the risk of postoperative thromboembolism include
7
a. Early ambulation b. External pneumatic compression devices placed on the upper extremities c. Elastic stockings d. Leg elevation for 24 h postoperatively 43. Signs and symptoms associated with early sepsis include a. Respiratory acidosis b. Decreased cardiac output c. Hypoglycemia e. Cutaneous vasodilation 51. The most common physiologic cause of hypoxemia is a. Hypoventilation b. Incomplete alveolar oxygen diffusion c. Ventilation-perfusion inequality d. Pulmonary shunt flow 52. Generally accepted indications for mechanical ventilatory support include d. Respiratory rate greater than 35 breaths/min 54. Which of the following inhalation anesthetics accumulates in air-filled cavities during general anesthesia? a. Diethyl ether b. Nitrous oxide c. Halothane d. Methoxyflurane 55. Major alterations in pulmonary function associated with adult respiratory distress syndrome (ARDS) include a. Hypoxemia b. Increased pulmonary compliance c. Increased resting lung volume e. Decreased dead space ventilation Adult respiratory distress syndrome (ARDS) has been called “shock lung” or “traumatic wet lung” and occurs under a variety of circumstances. Clinically, its manifestations can range from minimal dysfunction to unrelenting pulmonary failure. Three major physiologic alterations include (1) hypoxemia usually unresponsive to elevations of inspired oxygen concentration; (2) decreased pulmonary compliance, as the lungs become progressively stiffer and harder to ventilate; and (3) decreased functional residual capacity. Progressive alveolar occurs owing to leakage of protein-rich fluid into the interstitium and the alveolar spaces with the subsequent Radiologic picture of diffuse fluffy infiltrates bilaterally. Ventilatory abnormalities develop that result in shunt formation, decreased resting lung volume and increased dead space ventilation. 58. Dopamine is a frequently used drug in critically ill patients because
8
a. At high doses it increases splanchnic flow b. At high doses it increases coronary flow c. At low doses it decreases heart rate d. At low doses it lowers peripheral resistance Dopamine has a variety of pharmacologic characteristics that make it useful in critically ill patients. In low doses [1 to 5 mg/(kg.min)], dopamine affects primarily the dopamonergic receptors. Activation of these receptors causes vasodilation of the renal and mesenteric vasculature and mild vasoconstriction of the peripheral bed, which thereby redirects blood flow to kidneys and bowel. At these low doses, the net effect on the overall vascular resistance may be slight. As the dose rises [2 to 10 mg/(kg.min)], β1-receptor activity predominates and the inotropic effect on the myocardium leads to increased cardiac output and blood pressure. Above 10 mg/(kg.min), α-receptor stimulation causes peripheral vasoconstriction, shifting of blood from extremities to organs, decreased kidney function and hypertension. At all doses, the diastolic blood pressure can be expected to rise; since coronary perfusion is largely a result of the head of pressure at the coronary ostia, .coronary blood flow should be increased 63. If end-diastolic pressure is held constant, increasing which of the following will increase the cardiac index? a. Peripheral vascular resistance b. Pulmonary wedge pressure c. Heart rate d. Systemic diastolic pressure 68. The preoperative characteristics of patients likely to experience postoperative ischemia after non cardiac surgery include a. Angina b. More than three premature ventricular contractions (PVCs) per minute c. Dyspnea on exertion d. Tricuspid regurgitation 70. Compensatory mechanisms during acute hemorrhage include a. Decreased cerebral and coronary blood flow b. Decreased myocardial contractility c. Renal and splanchnic vasodilation d. Increased respiratory rate e. Decreased renal sodium resorption 73. An 18-year-old woman develops urticaria and wheezing after an injection of penicillin. Her blood pressure is 120/60 mm Hg, rate is 155 beats/min, respiratory rate is 30 breaths/min. Immediate therapy should include a. Intubation b. Epinephrine c. Beta blockers e. Fluid challenge 74. During blood transfusion,
9
heart and
clotting of transfused blood is associated with a. ABO incompatibility b. Minor blood group incompatibility c. Rh incompatibility d. Transfusion through Ringer’s lactate 75. When an arterial blood gas determination of PCO2 40 kPa is obtained b. Alveolar ventilation is adequate
77. Among patients who require nutritional resuscitation in an intensive care unit, the best evidence that nutritional support is adequate is a. Urinary nitrogen excretion levels b. Total serum protein level c. Serum albumin level d. Serum transferrin levels 78. Paradoxical aciduria (the excretion of acid urine in the presence of metabolic alkalosis) may occur in the presence of a. Release of inappropriate antidiuretic hormone b. Severe crush injury c. Acute tubular necrosis d. Gastric outlet obstruction 79. If a patient suffered a pulmonary arterial air embolism during an open thoracotomy, the anesthesiologist’s most likely observation would be e. Falling end-tidal CO2 A. Unexpected systemic hypertension B. Rising right atrial filling pressures C. Reduced systemic arterial oxygen saturation D. Rising systemic CO2 partial pressures E. Falling end-tidal CO2 A 72-year-old man undergoes resection of an abdominal aneurysm. He arrives in the ICU with a core temperature of 33°C (91.4°F) and shivering. Which of the following is a physiologic consequence of the shivering? Rising mixed venous O2 saturation Increased production of CO2 Decreased consumption of O2 Rising base excess Decreased minute ventilation
10
Shivering is the physiologic effort of the body to generate heat to maintain the core temperature. In healthy persons, shivering increases the metabolic rate by 3 to 5 times and results in increased O2 consumption and CO2 production. In critically ill patients, these metabolic consequences are almost always counterproductive and should be prevented with other means employed to correct systemic hypothermia. In the presence of vigorous shivering, O2 debt in the muscles and lactic acidemia develop. 81. To prepare for operating on a patient with a bleeding history diagnosed as von Willebrand’s disease (recessive), you would give d. Cryoprecipitate von Willebrand disease is similar to true hemophilia in frequently of occurrence. It is being diagnosed more commonly today because of more reliable assays for factor VIII. This autosomal dominant disorder (recessive transmission can occur) is characterized by a diminution in factor VIII:C (procoagulant) activity. The reduction in activity is not as great as in classic hemophilia, and the clinical manifestations are more subtle. These manifestations are often overlooked until an episode of trauma or surgery makes them apparent. Treatment requires correcting the bleeding time and providing factor VIII R:WF (the von Willbrand factor). Only cryoprecipitate is reliably effective. High-purity factor VIII:C concentrates, effective in hemophilia, lack the von Willbrand factor and are consequently underdependable. 83. The accidental aspiration of gastric contents into the tracheobronchial tree should be initially treated by a. Tracheal intubation and suctioning b. Steroids c. Intravenous fluid bolus d. Cricothyroidotomy 84. In performing a tracheostomy, authorities agree that a. The strap muscles should be divided b. The thyroid isthmus should be preserved c. The trachea should be entered at the second or third cartilaginous ring d. Only horizontal incisions should be used Although tracheostomy is occasionally an emergency procedure, it can be more effectively performed in an operating room where hemostasis and antisepsis are readily achieved. Most authorities recommend a horizontal incision; however, limited direct midline incisions have the advantage of not opening any unnecessary tissue planes and perhaps reducing the incidence of bleeding complications. Both approaches have advocates. In either case, the skin incision is made just below the cricoid cartilage, the strap muscles are spared and retracted, the thyroid isthmus is divided if necessary, and the trachea is entered at the second tracheal ring. The second and third tracheal
11
rings are incised vertically, allowing placement of the tracheostomy tube. The .first tracheal ring and the cricoid cartilage must be left intact 85. If malignant hyperthermia is suspected intraoperatively a.Complete the procedure but pretreat with dantrolene prior to future elective surgery b.Administer inhalational anesthetic agents c.Administer succinylcholine d.Hyperventilate with 100% oxygen e.Acidify the urine to prevent myoglobin precipitation in the renal tubules 86. Central venous pressure (CVP) may be decreased by a. Pulmonary embolism b. Hypervolemia c. Positive-pressure ventilation d. Pneumothorax e. Gram-negative sepsis 88. Signs and symptoms of Addison’s disease include a. Hypothermia b. Hypokalemia c. Hyperglycemia d. Hyponatremia e. Hypervolemia 89. The etiologic factor implicated in the development of pulmonary insufficiency following major non thoracic trauma is a. Aspiration b. Atelectasis c. Fat embolism syndrome d. Fluid overload 90. For the severely traumatized patient requiring airway management d. Intubation should be performed in the emergency room if the patient is unstable 91. Treatment for clostridial myonecrosis (gas gangrene) Includes which of the following measures? a. Administration of an antifungal agent b. Administration of antitoxin c. Wide debridement d. Administration of hyperbaric oxygen
12
92. An abnormal ventilatioperfusion ratio (Qs/Qr) in the postoperative patient has been associated with a. Pulmonary thromboembolism b. Lower abdominal surgery d. The upright position e. Increased cardiac output Abnormalities of ventilation-perfusion ratio result from the shunting of blood to a hypoventilated .lung or from the ventilation of hypoperfused regions of lung tissue When this imbalance is extreme, as following massive pulmonary thromboembolism, the effect is life-threatening hypoxemia. Other common predisposing factors in the postoperative patient that contribute to this maldistribution include the assumption of a supine position, thoracic and upper abdominal incisions, obesity, atelectasis, and reduced cardiac output 93. Correct statements concerning drowning or near-drowning include which of the following? d. Renal damage may occur in affected persons as a result of hemoglobinuria 94. Spontaneous retroperitoneal hemorrhage during anticoagulant therapy a. Is best confirmed by bleeding scan b. Is equally likely with parenteral and oral anticoagulants c. May mimic an acute surgical abdomen d. Frequently requires laparotomy for ligation of the bleeding site 96. Indications for surgical intervention to remove smuggled drug packets that have been ingested include e. Signs of toxicity from leaking drug packets Burns 1- When determining the depth of a burn: A. A knowledge of the type of injury is important. B. The presence of blisters is of no clinical significance. C. Impairment of sensibility of the burned area denotes full thickness burn. D. The presence of severe pain denotes a full thickness skin loss. E. A&C only. 2- Estimation of the area of a burn: ( all correct except one) A. Is of very little clinical significance. B. Provides important prognostic information. C. Is an important factor in the estimation of the fluid required. D. Can be based on a formula which states that the adult trunk is 36 per cent of the whole body surface area. E. Should be recorded in each chart of burned patient.
13
3- Patients with major burns: A. Are in a negative nitrogen balance. B. Have normal calorie requirements. C. Do not generally become anaemic. D. Are resistant to septicaemia. E. All of the above. 4- The catabolic response to trauma and infection is characterized by: A. An increase in lean body mass. B. A positive nitrogen balance. C. Gluconeogenesis. D. A falling haemoglobin level. E. A&B only.
5- The catabolic response to trauma: A. Is related to the severity of the trauma. B. Is accompanied by decreased urinary losses of potassium and nitrogen. C. Can be prevented by parenteral nutrition. D. Does not occur in the adrenalectomised patient. E. C&D only. 6- Scalds: A. Are more frequent in children. B. Commonly cause full thickness skin loss. C. Should be skin grafted within 48 hours of the injury. D. Need routine antibiotic treatment. E. All of the above. 7- The dressing of a small burn should be: A. Occlusive. B. Non-absorbtive. C. Non-compressive. D. Changed daily as a routine. E. All of the above 8- A partial thickness burn: A. Usually requires grafting. B. May deteriorate into full thickness skin loss. C. Rarely causes severe physiological derangement of the patient. D. Heals within 7 days in the absence of infection. E. C&D only. 9-Fluid losses in a major burn: A. Are maximal between 12 and 24 hours after the injury. B. Are related to the age of the patient. C. Are not related to the weight of the patient. D. Are related to the area burnt. E. Are not related to the burnt duration. 10- The increased fluid requirements of a patient with a full thickness burn are due to: (all
14
correct except one) A. Increased evaporative water loss. B. Sequestration of fluid in the injured tissues. C. Serum exuding from the burned area. D. Destruction of blood in the skin vessels. E. Vomiting & paralytic ileus occasionally 11- 48 hours after a major burn and with satisfactory fluid therapy a patient: A. Has very few abnormal fluid losses. B. May need a blood transfusion. C. Is often hypornatraemic. D. Usually needs skin grafting. E. C&D only. 12- Major burns are sometimes complicated by: (all correct except one) A. Acute gastric and duodenal ulcers. B. Paralytic ileus. C. Cerebral oedema. D. Mesenteric thrombosis. E. Septicemia.
13- If burned patient, associated pulmonary injury: (all correct except one) A. Should be suspected in head and neck burns. B. Should be suspected when the nasal hairs are burnt. C. Does not appear clinically in the first 24 hours. D. Should be suspected if burns occurred in closed space. E. May require ICU care. 14- Secondary infection of burns: A. Is less common in partial than in full thickness skin loss. B. Is relatively more common in burns of more than 20 per cent body area. C. Is avoided by leaving the burn eschar intact. D. Is avoided by the immediate application of a sterile occlusive dressing. E. All of the above. 15- . The early management of a burn wound may include: A. Early excision. B. Occlusive dressings. C. Exposure treatment. D. Dressings with local antibiotics. E. All of the above. 16- Skin grafting of a burn wound: A. Should usually be with full thickness skin grafts. B. Is more likely to be successful if undertaken in the first week after injury. C. Will be unsuccessful unless the wound surface is sterile. D. Minimizes scar contracture. E. A&C only. 17- The prognosis of a burned patient is: A. Not related to the patient's age. B. Related to the area burnt.?????? C. Generally better below the age of 10 years.
15
D. Very poor in the patient with burns of over 40 per cent surface area. E. C&D only.
Palestinian Medical Council Final Written Exam (April,2011) Paper ( I ) Candidate’s Name:…………………………………………………………………………………………………………….................. 2. Anion gap is: A. The difference between measured anions and measured cations B.Is normal in lactic acidosis C.Is high in shock D. Normally is 20 mmol E.High in pancreatic fistula 1. Causes of hypovolaemic shock include all the following Except: A. burn injury. B. low output intestinal fistula C. spinal cord transaction. D. myocardial infarction. E. endotoxaemia. 2. A 53-year-old female has had a high fever with cough productive of yellowish sputum for the past 2 days. Auscultation of the chest reveals a few crackles in both lung bases. A chest radiograph reveals bilateral patchy pulmonary infiltrates. Which of the following inflammatory cell types will be seen in greatly increased numbers in a sputum specimen? A. Macrophages B. Neutrophils C. Mast cells D. Small lymphocytes E. Langhans giant cells 3. The most common breast lesion in women is A. fibroadenoma B. carcinoma of the breast C. fibrocystic disease D. ductal papilloma 4. The liver is held in position in the upper part of the abdominal cavity mainly by: A. Attachment of hepatic vein to inferior vena cava B. Peritoneal ligament C. Tone of Abdominal muscles D. Angular ligament
16
E. Lesser omentum 5. When compared with the late-evening values typically observed in normal subjects, plasma levels of both adrenocorticoiropic hormone and conisol would be expected to be higher in which of the following individuals? A. Normal subjects after waking in the morning B. Normal subjects administered dexamethasone C. Patients with Cushing's syndrome (adrenal adenoma) D. Patients with Addison's disease E. Patients with Conn's syndrome 6. A 23-year-old female receiving chronic corticosteroid therapy for an autoimmune disease underwent minor surgery for incision and drainage of an abscess on her upper outer right arm. The wound healed poorly over the next month. Which of the following aspects of wound healing is most likely to be deficient? A. Re-epithelization B. Fibroblast growth factor elaboration C. Collagen deposition D.Serine proteinase production E.Neutrophil infiltration 7. Regarding the blood supply of the stomach which of the following is not true A. Left gastric artery arise from hepatic artery B. Right gastric artery the lower right part of stomach C. The short gastric artery supply the fundus D. Left gastroepiploic artery arise from splenic artery at the hilum of stomach E. right gastroepiploic artery arise from gasrtoduodenal artery left gastric artery arises from the celiac artery 8. Precipitating physiological events in the metabolic response to injury include: A. haemorrhage. B. pain. C. ischaemia. D. Endotoxaemia E. all the above 9. Psychological stress is often associated with peptic ulcer disease. Which of the following is thought to be a contributing factor in stress-induced ulcer disease? A. Sympathetic stimulation decreases gastric acid secretion B. Sympathetic stimulation increases peristalsis, which decreases the transit time of ingested food C. Psychological stress increases mucous secretion
17
D.Sympathetic stimulation decreases the alkaline mucous secretions of Brunner's glands located in the first few centimeters of the duodenal wall 10. Low molecular weight heparin (LMWH) produces its primary effects because of its inhibition of which factor? A. IIa B. IXa C. Xa D. XIa E. XIIa
11. All of the following are potential outcome in laparoscopic surgery EXCEPT : A. Hypercarbia B. Alkalosis C. Decrease urine out put D. Increase intracranial pressure E. Increased cardiac work 12. Acute Epidural haematoma : A. due to meningeal artery tear B. can be treated conservatively C. classically there is contra lateral, dilated, fixed pupil D. needs operation within 24 hours E. a and c only 13. Which of the following best characterizes the secretions of the small intestine? (A) Hypotonic and slightly acidic (B) Hypotonic and slightly alkaline (C) Isotonic and slightly alkaline (D) Isotonic and slightly acidic 14. Management of hypovolaemic shock involves all except: A. achievement of venous access via peripheral vein catheterisation. B. bladder catheterisation. C. immediate administration of whole blood or red cell concentrate. D. effective analgesia. E. oxygen administration. 15. Concerning the use of a central venous catheter:
18
A. administration of TPN is safe and does not require repetitive haematological and biochemical monitoring. B. its insertion should have mandatory electrocardiographic monitoring. C. it provides a convenient portal for blood sampling and antibiotic administration. D. if tunnelled subcutaneously it has a higher incidence of infection with endogenous staphylococcus. E. if infected it is effectively treated by administration of antibiotics via the offending catheter. 16 .A 16-year old driver, without seat belt, is involved in a rollover head-on accident. Extensive damage to the vehicle . The patient was pulled out of the car, hemodynamically unstable and was transported to a trauma center. During transportation the patient became severely hypotensive , with distended neck veins, unresponsive, with impending respiratory distress. Which of the following is considered an immediate life-threatening injury in this patient? A. pneumothorax secondary to rib fractures B. aortic intimal tear C. diaphragm rupture D. tension pneumothorax E. myocardial contusion 17. Hypochlorhydria is a condition in which acid secretion by the stomach is greatly reduced. How does this condition affect the digestion and absorption of food? (A) Digestion and absorption of all food substances are nearly normal (B) Digestion and absorption of all foods substances are markedly reduced (C) Digestion and absorption of carbohydrates are greatly affected by this condition (D) Digestion and absorption of fats are greatly affected by this condition 18. In chest injury the following are indications of thoracotomy EXCEPT : A. When cardiac tamponade present. B. Massive air leak C. For all transmediastinal wounds D. Flail chest with sever lung contusion E.
Clotted hemothorax
19. Which of the following tends to decrease potassium secretion by the cortical collecting tubule? (A) Increased plasma potassium concentration (B) A diuretic that decreases proximal tubule sodium reabsorption (C) A diuretic that inhibits the action of aldostcrone (e.g., spironolactone) (D) Acute alkalosis (E) High sodium intake
19
20.The metabolic response to injury is characterised by a sequence of physiological events aimed at: A. increasing core body temperature. B. conservation of sodium and water. C. mobilising glucose from fat and protein stores. D. maintaining body weight. E. enhancing immune function. 21. A 50 year old man present with haematemsis, Endoscopy shows a bleeding peptic ulcer in the posterior wall of the first part of duodenum. Which artery is involved A. Right gastric artery B. Right gastroepiploic artery C. inferior pancreaticoduodenal artery D. gasrtoduodenal artery E. superior pancreaticoduodenal artery 22. Fasciotomy for extremity compartment syndrome should be performed at a compartment pressure exceeding : A. 20 mmHg B. 30 mmHg C. 40 mmHg D. 50 mmHg E. none of the above 23. All of the following are true about neurogenic shock except: a. there is a decrease in systemic vascular resistance and an increase in venous capacitance. b. tachycardia or bradycardia may be observed, along with hypotension. c. the use of an alpha agonist such as phenylephrine is the mainstay of treatment. d. Severe head injury, spinal cord injury, and high spinal anesthesia may all cause neurogenic shock. 24. The radiographic findings indicating a torn thoracic aorta include all Except : a. Widened mediastinum. b. Presence of an apical “pleural cap.” c. Tracheal deviation to the right. d. right hemothorax.(lt) The most common abnormality noted is a widening of the mediastinal shadow, although only 20% to 40% of patients with a wide mediastinum have aortic injury. In addition to the radiographic signs listed, other findings that may alert the physician to the possibility of an aortic tear include loss of aortic contour, elevation of the left mainstem bronchus, depression of the right mainstem bronchus, shift of the nasogastric tube to the left, and the presence of retrocardiac density. Aortography remains the “gold standard” diagnostic modality and is indicated if aortic injury is suspected on the basis of mechanism of injury and any of these suggested findings.
20
25. Which of the following changes tends to increase urinary Ca++ excretion? (A) Extracellular fluid volume expansion (B) Increased plasma parathyroid hormone concentration (C) Decreased blood pressure (D) Increased plasma phosphate concentration (E) Metabolic acidosis extracellular fluid volume expansion inhibits calciumabsorption and increases calcium excretion
26.Pseudomembranous enterocolitis is caused by which the following organisms: A) Clostridium sporogenes B) Clostridium defficile C) Streptococcus faecalis D) Penicillin sensitive staphylocci E) Pseudomonas aeruginos 27. A 60-year-old man is being treated for persistent diverticulitis with a prolonged course of antibiotic therapy. The antibiotic being used has broad-spectrum activity and over time has caused sterilization of the gut in this patient. Which of the following sequelae is likely to be observed in this patient? (A) Vitamin B!2 deficiency (B) Steatorrhea (C) Vitamin K deficiency (D) Bloating due to excess gas formation 28. The secretion of bile is important for the proper digestion of which of the following? (A) Complex carbohydrates (B) Lipids (C) Proteins (D) Monosaccharides
21
29. Carotid body tumour: A) is chromffin paragranuloma B) is usually bilateral C) is called chemodactoma or potato tumour D) is situated at the bifurcation of the carotid artery E) C and D are correct 30. A 35-year-old trauma victim requires hyperalimentation. The patient’s injuries include a stable undisplaced fracture of the third thoracic vertebra, a closed head injury, multiple upper and lower extremity fractures and bilateral pulmonary contusions requiring ventilatory support. Which of the following are the most appropriate site and type of venous access in this patient? A. bilateral antecubital fossae, 18G peripheral intravenous catheters B. femoral vein, central venous catheter C. dorsum of one foot, single 16G peripheral intravenous catheter D. subclavian vein, central venous catheter E. long saphenous vein cut-down, long catheter 31. Which of the following disorders is not associated with hypocalcemia? (A) sarcoidosis (B) vitamin D deficiency (C) renal insufficiency (D) hypoparathyroidism (E) pancreatitis 32.Diffuse axonal injury (DAI) results from what type of force acting on the brain? A. direct impact B. axial loading C. linear acceleration D. rotational acceleration E. accceleration deceleration injury 33. Which of the following will immediately delay or cancel an elective surgical case if not obtained appropriately preoperatively? A. CBC B. urinalysis C. CXR D. informed consent E. ECG
22
34.While performing total thyroidectomy the surgeon ligate the superior pedicle as close to thyroid as possible to avoid to preserve: A. External laryngeal nerve B. Recurrent laryngeal nerve C. Superior laryngeal nerve D. Internal laryngeal nerve 35. The greatest burst strength of the wound will be achieved by: A. 1 week B. 3 weeks C. 6 weeks D. 12 weeks E. 6 months
36. The intravenous fluid that a 60 kg., 30-year-old woman with an 80% burn should be given in the first 24 hours following burn injury is: A. 19.2 liters of 5% glucose in lactated Ringer's. B. 14.4 liters of lactated Ringer's. C. 9.6 liters of hypertonic salt solution (sodium concentration 200 mEq) per liter.. D. 7.2 liters of 5% albumin solution. E. 5.5 liters of the pentafraction component of hydroxyethyl starch. 37. With regards to cytokines, which of the following alternatives is correct? A. IL- is secreted mainly by lymphocytes and mediates inflammation. B. IL- 10involved in cell division and activation. C. IL-8 is secreted by macrophages and promotes chemotaxis. D. IL-2 is a major inhibitor of cell division. E. TNF-a is produced by T cells and is associated with a rise of immature neutrophils in the blood circulation. 38. A 63-year-old male with end-stage renal disease requiring hemodialysis three times per week presents with bone pain and several pathologic fractures of the extremities. Which is the most likely electrolyte abnormality in this patient? A. hypokalemia B. hypernatremia C. hyperphosphatemia
23
D. hypercalcemia E. hypochloremia 39.Tuberculous cervical lymphadenitis: A. is caused by bovine tuberculous bacillus rather than the human bacillus B. usually occurs through the tonsil of the corresponding side C. collar-stud abscess is never a feature of T. B adenitis D. is treated always by surgery E. biopsy is not necessary for diagnosis 40.Which of the following is not an action of angiotensin II? A. aldosterone secretion B. sodium absorption C. efferent arteriolar constriction D. arterial dilation E. nephrosclerosis in the kidney
41.Which of the following laboratory values correspond with acute DIC? A. slowly rising platelet count B. selective deficiency of vitamin K factors C. hypofibrigonemia D. prolonged bleeding time E. presence of fibrin split products 42.Which hormone is produced by the kidney? A. calcitonin B. erythropoietin C. 25 hydroxyvitamin D D. aldosterone E. antidiuretic hormone 43. A 46-year-old patient had respiratory failure from pneumonia and developed acute respiratory distress syndrome (ARDS). He remained on a ventilator for a prolonged period of time and ultimately underwent a tracheostomy at the level of tracheal ring 4. The patient is now in the rehabilitation unit of your hospital 2 weeks after his tracheostomy and you are called to see him about bright red blood around his tracheostomy site. The most common cause of this bleeding is
24
A. granulation tissue B. erosion of the inferior thyroid artery C. erosion of the innominate artery D. tracheal chondritis E. recurrence of pneumonia 44. Which of the following is not associated with increased likelihood of infection after major elective surgery? A. B. C. D. E.
Age over 70 years. Chronic malnutrition. Controlled diabetes mellitus. Long-term steroid use. Infection at a remote body site.
45. Advantages of epidural analgesia include: A. Earlier mobilization after surgery. B. Earlier return of bowel function. C. Shorter hospitalizations. D. Decreased stress response to surgery. E. All of the above.
46. Which breast lesion is most commonly bilateral A. intraductal carcinoma B. medullary carcinoma C. tubular carcinoma D. in situ lobular carcinoma 47. Damage to the pelvic nerves does which of the following? A. Has little, if any, effect on the defecation reflex B. Attenuates the defecation reflex C. Increases the strength of the defecation reflex D.Results in a continuous urge to defecate (rectal urgency 48. Diabetes insipidus is associated with a lack of A. glucocorticoids B. insulin C. thyroid hormone D. antidiuretic hormone
25
E. growth hormone 49. Regarding Acute subdural haematoma : A. It is due to meningeal artery tear B. has 30% mortality. C. may cause secondary brain injury D. Treated by early operation E. More urgent than epidural haematoma 50. The following structures form the walls of the inguinal canal except: A. The conjoint tendon B. The aponeurosis of external oblique muscle C. The internal oblique muscle D. The lacunar ligament E. the fascia transversalis
51. The cell type most characteristic of chronic inflammation is the: A. macrophage B. B cell C. natural killer cell D. neutrophil E. eosinophil 52.what is the least common position of appendix A. Preileal B. Postileal C. Pelvic D. Paracaecal E. Subcaecal Retrocecel 74% Post ileal 0.5% 53. Anaphylactic shock:all except A. is an immune-mediated reaction. B. results in mast cell activation and increased circulating histamine concentrations. C. produces microcirculatory changes similar to hypovolaemic shock . D. requires prompt treatment with parenteral adrenaline and hydrocortisone. E. may occur after ingestion of drugs
26
54. Which of the following is true regarding the Ureter: A. It is lie at the lateral edge of psoas major muscle B. It crosses into the pelvis at the level of L3 C. The internal iliac artery branches supply the middle part D. The middle part of each Ureter drain to lymph nodes associated with internal and external vessels E. The left Ureter is crossed by testicular and left colic vessels
F.The left ureter is crossed by thetesticular (or ovarian) and left colic vessels and then passes above the pelvic brim, The abdominal ureter lies on the medial edge of psoas major the inferior part of each ureter drains to lymph nodes associated with the external and internal iliac vessels 55. The following statement concerning the anal canal are correct except : A. It encircle the anal canal B. It is not attached to the anococcygeal body C. It is composed of striated muscle fibers D. It is not responsible for causing the anal canal and rectum to join at an acute angle E. It is innervated by the middle rectal nerve
56. 75% of the breast is drained by axillary lymph nodes, regarding the axillary lymph node which of the following is not true A. Anterior group lie indirect contact with the axillary tail of Spence B. Posterior group lie along to lateral thoracic vein C. The intercostobrachial nerve passes between the central lymph nodes D. The apical group receive most of lymph from upper part of breast E. Lateral group lie along upper part of Humerus 57. All true about the blood supply of thyroid gland except A. Superior thyroid artery is the first branch of ECA B. Inferior thyroid artery is a branch of thyrocervical trunk C. Thyroid ima arise from subclavian artery D. Superior and middle thyroid vein drain into IJV E. Inferior thyroid vein drain into brachiocephalic veins thyroid imaartery (L. thyroidea ima) usually arises from the bra- chiocephalic trunk; 58. The following statements concerning the parotid salivary gland are correct except i. The facial nerve passes through it, dividing the gland into superficial and deep parts. ii. The secretomotor nerve supply is derived from the facial nerve. iii. The parotid duct pierces the buccinator muscle and opens into the mouth. iv. The external carotid artery divides within its substance to form the superficial temporal and maxillary arteries. v. The retromandibular vein is formed within it by the union of the superficial temporal vein and the maxillary vein
27
59. Which of the following changes would you expect to find in a patient consuming a highsodium diet (200 mEq/day) compared with the same patient on a normal-sodium diet (100 mEq/day), assuming steady-state conditions? (A) Increased plasma aldosterone concentration (B) Increased urinary potassium excretion (C) Decreased plasma renin activity (D) Decreased plasma atrial natriuretic peptide (E) An increase in plasma sodium concentration of at least 5 mmol/L
60. The primary anatomic site of pressure regulation in the vascular system is A. aorta B. arteries C. arterioles D. capillaries E. heart 61. A 60 years old male present with dysphagia. Flexible endoscopy shows a growth at cardia. What is the distance of this growth from the incisor teeth A. 25 cm B. 30 cm C. 35 cm D. 40 cm E. 45 Cm F. 62. The pancreas lies across vertebral bodies A. T10-T12 B. T11-L1 C. T12-L2 D. L1-L2 E. L1-L3 63. The major cause of pulmonary thromboemboli is: A. hypertension B. heart failure C. atherosclerosis D. thrombophlebitis
28
E. varicose veins 64. Brown tumors of bone are the result of A. hyperparathyroidism B. trauma C. metastatic carcinoma D. occlusion of the nutrient artery E. vitamin deficiency 65. Which is the most common primary malignant neoplasm of bone in the age group 15—25 years A. giant cell tumor B. Ewing’s sarcoma C. chondroblastoma D. osteosarcoma E. chondrosarcoma
66.The fundus of gall-bladder is in the angle between lateral border of right recius abdominis and which costal cartilage? A. B. C. D. E.
6th 7th 8th 9th 10th
9) In myocardial infarction there is radiation of pain along the left upper limb. This is due to .......... nerve : One answer only. Left vagus Left phrenic Left recurrent laryngeal Left intercostobrachial 10) Oculomotor nerve leaves the skull through which of the following One answer only. Optic canal
29
Superior orbita fissure Inferior orbital fissure foramen magnum F. 67. Which of the following structures in the gastrointestinal tract is most at risk in a patient as a consequence of the circulatory shock? A.Submucosal glands B. Brunner's glands C. Tips of the villi D. Sphincter muscles 68. Preoperative investigation and preparation of a patient with obstructive jaundice should include all of the following except : A. measurement of coagulation status. B. measurement of 24-hour urinary output. C. measurement of serum urea and electrolytes. D. fluid restriction during the 24 hours preoperatively. E. administration of antibiotics during invasive diagnostic procedures (e.g. PTC, ERCP)
69.All the following about Rectus Sheath is true except A. Enclose the rectus abdominis and pyramidalis muscles B. Formed by the Aponeuroses of two abdominal muscles C. Contain anterior rami of lower 6 thoracic nerves D. Contain superior and inferior epigastric vessels E. Separated from its fellow on the opposite side by linea alba
70. Bleeding from the nipple in a 45-year-old woman, without a palpable breast mass should suggest A. fibroadenoma B. sclerosing adenosis C. fat necrosis D. intraductal papilloma E. chronic cystic mastitis 71. Nasotracheal intubation:
30
A. Is preferred for the unconscious patient without cervical spine injury. B. Is preferred for patients with suspected cervical spine injury . C. Maximizes neck manipulation. D. Is contraindicated in the patient who is breathing spontaneously. 72. Conditions associated with increased risk of breast cancer include all of the following except A. a previous mastectomy for cancer B. fibrocystic disease with severe epithelial hyperplasia C. a maternal history of breast cancer D. early pregnancies and breast feeding 73. Which of the following statements about extracellular fluid is true? A. The total extracellular fluid volume represents 40% of the body weight. B. The plasma volume constitutes 20%of the total extracellular fluid volume. C. Potassium is the principal cation in extracellular fluid. D. The protein content of the plasma produces a lower concentration of cations than in the interstitial fluid. E. The interstitial fluid equilibrates slowly with the other body compartments.
74. The Stein-Leventhal syndrome is characterized by each of the following except A. many corpora lutea present in the ovaries B. thick ovarian tunica albuginea C. obesity D. hirsutism 75. Regarding Shock, all are correct Except : A. it is defined as inadequate cellular perfusion. B. it may occur in the presence of normotension. C. it may occur following cardiomyopathy. D. it may follow gastrointestinal perforation. E. it invariably results in sympathomimetic activity in circulation. 76.The most common site of endometriosis is A. ovary B. wall of large intestine C. posterior serosa of uterus D. umbilicus
31
E. oviduct 77. Paget’s disease of the nipple presupposes the existence of A. ductal carcinoma B. simple eczema C. lobular carcinoma D. abnormal estrogen stimulation E. sclerosing adenosis 78. Each of the following applies to Hirschsprung’s disease except A. intestinal obstruction B. absence of ganglion cells in myenteric plexus C. treated by removal of distended segment of bowel D. toxic megacolon as complication E. narrow, more distal segment exhibits characteristic deficiency of development
79. An 8-month-old boy is reported by his mother to have had several episodes of “colic” and she has noticed blood in his stools. Your examination is negative except for slight abdominal tenderness and moderate anemia. Which of the following is most likely A. pyloric stenosis B. intestinal atresia C. Meckel’s diverticulum D. Hirschsprung’s disease 80. Carcinoid tumors (argentaffinomas) are characterized by all of the following except A. many are multiple especially in the ileum B. can occur in alimentary, biliary and respiratory tracts C. may produce systemic symptoms when accompanied by liver metastases D. appendix is the most common site E. appendiceal types have greatest malignant potential
32
81.Which of the following can’t be palpated in the anterior surface while doing PR( digital rectal examination) A. Prostate B. Bulb of penis C. Posterior surface of bladder D. Seminal vesicle E. Ischiorectal fossa 82. Reed-Sternberg cells are characteristic of : A. chronic lymphatic leukemia B. cat scratch disease C. Hodgkin’s disease D. histiocytic medullary reticulosis E. large cell lymphoma 83.All the following structures lie in the transpyloric plane EXCEPT: A. origin of the inferior mesenteric artery B. fundus of the gall-bladder C. termination of the spinal cord D. pancreatic neck E. duodenojejunal flexure
84. Stored whole blood used for transfusion: A. contains similar amounts of coagulation factors blood. B. contains a concentration of leucocytes similar to normal blood. C. can be stored for up to 100 days at 4±2°C. D. contains normal platelet count. E. is used when rapid volume transfusion is required for a patient who has suffered major trauma. 85. The most common site of peptic ulcer formation is shown at which point on the figure below?
(A) Point A (B) Point B
33
(C) Point C (D) Point D 86. Pathologic fractures most typically occur with which of the following A. pituitary adenoma B. adrenal adenoma C. thyroid adenoma D. parathyroid adenoma E. pheochromocytoma 87.
Which of the following is true regarding the blood supply of appendix: A. The Appendicular artery passes in front the terminal ileum B. Blood from appendix drained by inferior mesenteric vein C. The Appendicular artery is a branch of lower division of ileocolic artery D. In most of the people there is accessory Appendicular artery E. The Appendicular artery run toward the base of appendix
88. A 45-year-old man is found to have a condition in which the parietal cells of his stomach have been destroyed by an autoimmune mechanism. His diagnosis is chronic autoimmune gastritis. This condition is often associated with which of the following? (A) Pernicious anemia (B) Gastric ulceration (C) Steatorrhea (D) Protein deficiency Which of the following components of bile is critical for fat digestion? A. Calcium salts B. Lecithin C. Bilirubin D. Bicarbonate 89. The most common cause of aortic aneurysms is A. syphilis B. congenital weakness of the aortic wall C. atherosclerosis D. occlusion of the vasa vasorum E. bacterial arteritis 90. Which of the following changes would you expect to find in a patient who developed acute renal failure after ingesting poisonous mushrooms that caused renal tubular necrosis?
34
(A) Increased plasma bicarbonate concentration (B) Metabolic acidosis (C) Decreased plasma potassium concentration (D) Decreased blood urea nitrogen concentration (E) Decreased hydrostatic pressure in Bowman's capsule 91.Regarding the anatomy of gallbladder (GB) all the following is true except: A. GB lies in a fossa separating the right and quadrate lobes of liver B. The cystic duct is 3 cm length usually C. The intraduodenal part of common bile duct passes through the second part of duodenum D. Cystic artery is a branch of left hepatic artery E. Lymphatic vessels of GB drain into lymph node of Lund 92.What segment of the nephron is responsible for the majority of sodium absorption? A. proximal convoluted tubule B. loop of Henle C. distal convoluted tubule D. collecting tubule 93. Which of the following is not normally associated with activation the peristaltic reflex? (A) Distention of the gut wall (B) Sympathetic stimulation (C) Presence in the gut of a hypertonic solution (D) Irritation of the epithelium of the gut 94. Which of the following is true of mass movements? (A) They normally move colonic contents from the cecum to the transverse colon (B) They are strong peristaltic contractions of the small intestine in response to mucosal irritation (C) They normally occur approximately 9 to 12 times per minute (D) They are a modified type of peristalsis that occurs in the large intestine 95.The anatomical left & right lobes of the liver are separated on the diaphragmatic surface of the liver by which of the following structures A. fissure for the round ligament of the liver B. fissure for the ligamentum venosum C. falciform ligament D. porta hepatic E. lesser omentum
35
96. Which of the following would cause the greatest degree of hyperkalemia? (A) Increase in potassium intake from 60 to180mmol/day in a person with normal kidneys and a normal aldosterone system (B) Chronic treatment with a diuretic that inhibits the action of aldosterone (C) Decrease in sodium intake from 200 to l00mmol/day (D) Chronic treatment with a diuretic that inhibits loop of Henle Na +-2 Cl -- K+ co-transport (E) Chronic treatment with a diuretic that inhibits sodium reabsorption in the collecting ducts 97. Furosemide (Lasix) is a diuretic that also produces natriuresis. Which of the following is an undesirable side effect of furosemide due to its site of action on the renal tubule? (A) Edema (B) Hyperkalemia (C) Hypercalcemia (D) Decreased ability to concentrate the urine (E) Heart failure 98. The mesentery of small intestine, along its attachment to the posterior abdominal wall, crosses all of the following structures except A. Left gonadal vessels B. Third part of duodenum C. Aorta D. Right Ureter E. All the above 99.The structure in the free border of lesser omentum from anterior to posterior are A. Common bile duct (CBD), Hepatic artery (HA), portal Vein (PV). B. PV, HA, CBD C. HA, PV, CBD D. PV, CBD, HA 100. Which of the following changes would you expect to find in a patient consuming a highsodium diet (200 mEq/day) compared with the same patient on a normal-sodium diet (100 mEq/day), assuming steady-state conditions? (A) Increased plasma aldosterone concentration (B) Increased urinary potassium excretion (C) Decreased plasma renin activity (D) Decreased plasma atrial natriuretic peptide (E) An increase in plasma sodium concentration of at least 5 mmol/L
36
101. A 26-year-old man develops glomerulonephritis, and his glomerular filtration rate (GFR) decreases by 50 per cent and remains at that level. For which of the following substances would you expect to find the greatest increase in plasma concentration. (A) Creatinine (B) K+ (C) Glucose (D) Na+ (E) Phosphate (F) H+ 102. Over a period of several months, a 39-year-old woman has developed hyperpigmentation in association with an increase in blood pressure. Additionally, her blood glucose concentration has increased slightly. Which of the following is the most likely diagnosis? (A) Addison's disease (B) Conn's syndrome (C) Pituitary tumor secreting large amounts of adrenocorticotropic hormone (D) Adrenal tumor secreting large amounts of cortisol (E) Panhypopituitarism
46. Recurrent laryngeal nerve A.
Is a pure motor nerve
B.
Supply cricothyroid muscle
C.
Supply mucous membranes of larynx above vocal cords
D.
The rate of injury during thyroid surgery is about 10%
E.
None of the above
52. Level one Axillary clearance A. Means removing lymph nodes behind pectoralis minor B. Mean dissecting apical lymph nodes
37
C. Include removing anterior and posterior Axillary lymph nodes D. Include removing central Axillary nodes E. None of the above Third space loss is due to all except A. Burns crush syndrome. B.
Severe soft tissue infections
C.
Repeated vomiting
D.
Intestinal obstruction
E.
Site of major operative dissection
Regarding polyps of the colon A.
Adenomatous polyps are usually solitary
B.
Villous polyps are usually pedunculated
C.
Metaplastic polyps are not precancerous
D.
Villous polyps occur more proximal in colon
E.
Cancer risk is not related to size of polyp
Regarding nitrogen balance A. 60% of nitrogen is lost in urine B.
40% of nitrogen is lost in skin and stool
C.
10 gm nitrogen is present in 62.5 gm proteins
D.
Nitrogen requirement is urinary nitrogen +40%
E.
All of the above
89. Regarding K+ ions: A. Hypokalemia Is defined as K+ level less than 3.9 mmol per liter B.
Hypokalemia lead to wide QRS complex
C.
More than 80 % of K is present intracellulary
D.
K+ maintenance per day is around 3 mmol/ kg /day
E.
Hypokalemia cause metabolic alkalosis
Where does the arterial supply to the superior and inferior parathyroids originate from? F.
38
A. superior thyroid artery
G.
* B. inferior thyroid artery
H.
C. external carotid artery
I.
D. internal carotid artery
J.
E. common carotid artery
According to the tumor, node, metastasis (TNM) stag ing for colon cancer, which of the following is true? K.
*A. All perforated colon cancers are considered T4.
L.
B. N2 refers to involvement of greater than one regional lymph node.
M.
C. T5 grade involves direct carcinoma invasion into adjacent solid organs.
N.
D. MX indicates metastatic disease involvement of more than one additional organ system (e.g., liver, lung, brain(.
O.
E. Five-year survival for stage I colon cancer is approximately 75%.
To confirm endotracheal intubations in emergency room, all are true except A. Equal bilateral breath sounds. B. Pulse oximetry C. No noise over epigastrium. D. Chest x ray. E. CO 2 detector
Q1: about gastric lymphoma is true most common extranodal(true about 1/3 of GIT tumor 15%of gastric tumor Q2:one unit of FFPrepresent about (1:1,5 Q3:about ischemic ulcer all true except:ischemic ulcer had hyperkeratnizedmargin Q4:If your pateint had an intermediate cardiac risk and will do elective operation what wiil be done :intraoperative monitoring :Q5 most common cause of hypercalcemia parathyroid adenoma Q6:most sensitive test 4 localizing parathyroid sestambi scan Q7:SUBMANDIBULAR SURGERY what nerve suspected to injury fray's syndrom(true loss of sensation lip drop
39
hoarsness of voice :Q8 :what can indicate malnutrition (WBC increase(true transfertin <20mg pre albumin <30 mg Q9:pathological breast pain radiate to axilla unicentric(true bilateral Q10 :Life long treatment of achalasia calcium channel blocker dilation hellet eosphatomy(true antiacid Q11 :about LES all true except about 3cm distally B adrenergic increase LES pressure(true gastric acid increase pressure about 10-20mhg in rest Q12 :ABOUT ARDS all true except noncardiac pulmonary edema hypoxia hypocapnia(true Q13 what anti inflamatory mediatorxx IL 10 Q14 :Warthin tumor is true except pleomorphic adenoma benign salivary gland tumor need total parotid surgery usually present with fascial pulsy Q15 About thyroglossal cyst is false about 70%of congenital tumor 10%malignant(true usually in medline treated by operation Q16 :about femoral hernia diffivult to distinguish from L.N Q17 :MOST common symptom of inguinal hernia painless pulg painful pulg heaviness in groin Q18 :Narwost ring of hernia femoral :most common risk 4 strangulation narrow neck
40
Q20 :What most important question in lump when present Extrahepatic Biliary System Spleen And Pancreas 1- Acute cholecystitis: (all correct except one)
• • • • •
Is almost invariably related to the presence of gallstones.
• • • • •
Nasogastric suction and intravenous fluids.
• • • • •
Have an incidence which increases with age.
• • • • •
Is commonly associated with a long history of dyspepsia.
• • • • •
Are present in nearly 50 per cent of cases of cholecystitis.
• • •
Intravenous cholangiography.
Usually presents with biliary colic. Is often associated with jaundice. Is characterised by a pyrexia in the early hours of the disease..
May be complicated by peritonitis. 2- Acute cholecystitis should usually be treated by: (all correct except one) Antibiotic therapy. Urgent cholecystectomy. Cholecystostomy.
Analgesics and antispasmotics. 3- Gallstones: (all correct except one) Are more frequent in females. Usually contain a predominance of cholesterol. Are formed in bile which is supersaturated with bile acids.
Are formed in bile which is supersaturated with cholesterol. 4- The presence of stones in the common bile duct: ( all correct except one) Is usually associated with jaundice. Must be considered during every cholecystectomy. May requeres treatment by choledochoduodenostomy.
May cause liver abscess. 5- stones in the common bile duct: Often give rise to jaundice, fever and biliary colic. Are usually accompanied by progressive jaundice. Are usually associated with a distended gallbladder.
A&D only. 6- In severe jaundice diagnostic evidence of an extrahepatic obstrction of the biliary tract may be gained by: (all correct except one)
41
A barium meal. Endoscopic retrograde cholangiography.
• •
Percutaneous transhepatic cholangiography.
• • • • •
Most commonly arise as a consequence of cholecystitis.
• • • • •
Is usually a squamous cell neoplasm.
• • • • •
Results in anaemia, leucopenia and thrombocytopenia.
• • • • •
Has a higher incidence in alcoholics.
• • • • •
A history of gallstones
CT scan of the abdomen. 7- Internal biliary fistulae: (all correct except one) Most commonly occur between the gallbladder and the duodenum. Can result in intestinal obstruction. Are in most cases fatal complications.
May cause cholongitis. 8- Carcinoma of the gallbladder: Is more common in men. Is rarely associated with gallstones. Has a relatively poor prognosis.
Non of the above is correct. 9- hypersplenism: Only occurs in the presence of a large spleen. Frequently follows liver cirrhosis. May be diagnosed by bone marrow biopsy.
All of the above are correct. 10- Acute pancreatitis: (all correct except one) Is commonly associated with the presence of gallstones. Occurs most commonly in diabetics. Becomes less severe with each recurring episode.
May occurs as a complication of abdomenal trauma. 11- Acute pancratitis is characterised by: (all correct except one) Diffuse epigastric pain. Exaggerated bowel sounds. An elevated urinary amylase. An elevated serum amylase.
12-chronic pancreatitis:
• • • • 42
Is commonly associated with alcoholism. Is associated with diabetes. May be diagnosed by the analysis of pancreatic secretions. May be treated by surgical procedures which decompress the pancreatic duct.
•
All of the above are correct. 13-Pancreatic pseudocysts: (all correct except one)
• • • • •
Are developmental in origin.
1.
The initial maneuver to establish an airway in a patient with multiple injuries is: a. Oropharyngeal airway. b. Uncuffed endo-tracheal tube. c. Suctioning foreign debris and lifting up the mandible. d. Cuffed endo-tracheal tube. e. Tracheostomy.
2.
Which is the most commonly injured intra-abdominal organ in blunt trauma? a. Pancreas. b. Kidney. c. Spleen. d. Stomach. e. Colon.
Usually arise in the lesser peritoneal sac Produce a smooth epigastric mass which does not moves on respiration. May be effectively treated by internal drainage. May be associated by elevation of serum amylase.
3.
The most important principle in the management of severe hemorrhagic shock is to: a) Obtain blood for possible type-specific transfusion. b) Place CVP lines early for fluid resuscitation and monitoring. c) Rapidly infuse colloid fluids. d) Apply MAST garment. e) Secure the airway and adequate ventilation. 4. A 6-year-old child sustained the following injuries in an accident. Which one of the followings should be managed first? a. Extradural hematoma. b. Pneumothorax. c. Hollow viscus injury. d. Renal injury. e. Liver laceration. 5.
All the following are complications of massive blood transfusion except: a. Hypothermia. b. Hypocalcaemia. c. Hypokalaemia. d. Acidosis. e. DIC. 6.Severe limb pain of sudden onset can be caused by all the following conditions except: a. Acute ischaemia. b. Deep venous thrombosis. c. Muscle tear. d. Sciatica. e. Bone fracture.
43
?Which of the following is not a classic sign of a basal skull fracture .7 a. Battle sign b. racoon eyes c. hemotympanum d. Gray –Turner sign e. CSF rhinnorhea/ottorrhea :Tension pneumothorax is best diagnosed with.8 A, stat CT scan b. chest x-ray c. watch and wait d. clinical exam .e. none of the above :While doing thoracocentesis, it is advisable to introduce needle along .9 .a. Upper border of the rib .b. lower border of the rib .c. In the center of the inter-costal space .d. In anterior part of inter-costal space .e. any where The neurovascular bundle of each intercostal space is closely related to the lower border .of rib above To avoid damage to the intercostal nerves and vessels therefore,the needle should be introduced close to the upper border of lower rib 10. The most significant immediate complication associated with pelvic fracture is: a. Hemorrhage. b. Rectal or vaginal lacerations. c. Sciatic nerve injury. d. Infection. e. Myositis ossificans. What is the commonest complication of supracondyla fracture of humerus: A Malunion В Myositis ossificans С Stiffness of elbow D Volkmann's contracture Most serious E Non union. 11 - Which is not contributory to Glasgow Coma Scale? a. b. c. d. e.
obey commands localizes painful stimuli open eyes to calling incomprehensible sounds 5mm pupils
12. In role of nine extent of burn if entire trunk is burned it will be equal to: a. 9% body surface area. b. 18% body surface area. c. 36% body surface area. d. 27% body surface area. e. 45% body surface area.
44
13. A 54-year-old man presents with two episodes of hematemesis since yesterday. The most likely cause of this patient’s upper gastrointestinal bleeding is: a. Gastritis. b. Esophagitis. c. Esophageal varices. d. Peptic ulcer disease. e. Mallory-Weiss tear. 14. Adequate minimum urine output in a 70kg man during resuscitation is a) b) c) d) e)
35 ml/hr 20 ml/hr 50 ml/hr 45 ml /hr 60 ml/hr
15. A 28 year old lady complains of painful defecation associated with fresh per-rectal bleed. Possible diagnosis to consider: a) b) c) d) e)
hiradenitis suppurativa dermoid cyst pilonidal sinus anal fissure pruritis ani
16. Which is not a cause of pancreatitis? a) b) c) d) e)
hypercalcaemia hypokalaemia hyperlipidaemia obstruction at ampulla of Vater thiazide
17-All of the following can be treated conservatively in a stable trauma patient except: a. Lung contusion. b. Liver laceration. c. Kidney laceration. d. Splenic hematoma. e. Perforation of the small intestine. 18.What is the commonest presentation of a nephroblastoma? a) Abdominal pain. b) Haematuria. c) Fever. d) Abdominal mass. e) Loss of weight. 19. Which of the following are not found in peritonitis? a. Patient is lying still b. Guarding c. Rebound tenderness d. Hyperactive bowel sounds e. Rigid abdomen
45
,The following is an indication for thoracotomy in chest injury .20
a. Cardiac tamponade b. Uncontrolled pulmonary air leakage c. Perforation of thoracic esophagus d. Blood loss of 200ml/hr for 2-3 hrs via chest tube e. All of the above 1. The most ominous sign or symptom of urinary system disease is: A. Urinary frequency. B. Pyuria. C. Pneumaturia. D. Dysuria. E. Hematuria.
Answer: E
2. A patient with acute urinary tract infection (UTI) usually presents with: A. Chills and fever. B. Flank pain. C. Nausea and vomiting. D. 5 to 10 white blood cells E. Painful urination.
Answer: E
3. Renal adenocarcinomas: A. Are of transitional cell origin. B. Usually are associated with anemia. D. Are extremely radiosensitive. E. Frequently are signaled by gross hematuria.
Answer: E 4. Ureteral obstruction: A. Is associated with hematuria. B. Is associated with deterioration of renal function and rising blood urea nitrogen (BUN) and creatinine values. C. Is commonly caused by a urinary tract calculus. D. Usually requires open surgical relief of the obstruction. E. Is usually associated with infection behind the obstruction. Answer: C
5. Stress urinary incontinence: A. Is principally a disease of young females. B. Occurs only in males. C. Is associated with urinary frequency and urgency. E. Is a disease of aging produced by shortening of the urethra. Answer: E
6. Which of the following is/are true of blunt renal trauma? D. Blunt renal trauma requires exploration only when the patient exhibits .hemodynamic instability
46
7. Carcinoma of the bladder: .D. May mimic an acute UTI with irritability and hematuria
8. The major blood supply to the testes comes through the: A. Hypogastric arteries. B. Pudendal arteries. C. External spermatic arteries. .D. Internal spermatic arteries
Answer: D
9. Patients who have undergone operations for benign prostatic hypertrophy or hyperplasia: A. Require routine rectal examinations to detect the development of carcinoma of the prostate. B. Do not need routine prostate examinations. C. Have a lesser incidence of carcinoma of the prostate. D. Have a greater incidence of carcinoma of the prostate. Answer: A
11. To maximize fertility potential, orchidopexy for cryptorchidism should be done before: A. Age 15 years. B. Age 12 years. C. Marriage. D. Age 2 years. Answer: D
12. Within the age group 10 to 35 years, the incidence of carcinoma of the testis in males with intra-abdominal testes is: .D. Twenty times greater than that in the general population
13. The appropriate surgical treatment for suspected carcinoma of the testis is: A. Transscrotal percutaneous biopsy. B. Transscrotal open biopsy. C. Repeated examinations. D. Inguinal exploration, control of the spermatic cord, biopsy, and radical orchectomy if tumor is confirmed. Answer: D
14. If torsion of the testicle is suspected, surgical exploration: A. Can be delayed 24 hours and limited to the affected side. B. Can be delayed but should include the asymptomatic side. C. Should be immediate and limited to the affected side. D. Should be immediate and include the asymptomatic side.
Answer: D
47
15. Epididymitis, either unilateral or bilateral, in a prepubertal male: A. Is a frequent diagnosis.
B. Can be dealt with on an outpatient basis. C. Is a major scrotal problem in this age group. D. Is a rare phenomenon. Answer: D
16. Patients with prostatitis, especially acute suppurative prostatitis: A. Should have residual urine measured by intermittent catheterization. B. Should have bladder decompression by urethral catheter. C. Should have repeated prostatic massage. D. Should have no transurethral instrumentation if possible. Answer: D 17. Benign prostatic hypertrophy with bladder neck obstruction: A. Is always accompanied by significant symptoms. B. Is best diagnosed by endoscopy and urodynamic studies. C. Is easily diagnosed by the symptoms of frequency, hesitancy, and nocturia. D. Is always accompanied by residual urine volume greater than 100 ml. Answer: B
1- A fracture is said to be: Closed if an overlying skin laceration has been sutured. Comminuted if there has been associated damage to adjacent nerves or vessels. A fatigue fracture if it occurs through a diseased bone. Pathological if it occurs through a bony metastasis. Non of the above correct.
2- In a healing fracture: (All correct except one) • The haematoma is initially invaded by osteoblasts. • The tissue formed by the invading osteoblasts is termed osteoid. • Calcium salts are laid down in the osteoid tissue. • The final stage of repair is the remodelling of the callus. • The callus formation is related to the amount of stress at fracture side. 3- Non-union is often seen in:
• • 48
23. A 28-year-old white male presents with asymptomatic testicular enlargement. Which of the following statement(s) is/are true concerning his diagnosis and management c. The diagnosis of seminoma should be followed by postoperative radiation therapy
Fractures And Dislocations
• • • • •
Fractures of the 4th metatarsal. Fractures of the neck of the femur.
• • •
Fractures of the condyle of the mandible. Colles’ fractures. Oblique fracture line of femur.
4- Fractures of the clavicle:
• • • • •
Are usually of the greenstick variety in children under the age of 10 years. Are usually the result of direct violence. Can be recognised by the abnormal elevation of the distal fragment. Are usually treated by internal fixation. C&D only.
5- in fractures of the surgical neck of the humerus: (All correct except one)
• • • • •
The lesion is usually due to indirect violence.
• • • • •
The distal fragment is usually posteriorly angulated by the action of biceps.
The fragments are usually impacted. The proximal fragment is usually internally rotated. The distal fragment is usually adducted.
Early mobilisation is encouraged. 6- In a fracture of the distal third of the shaft of the humerus: The radial nerve is rarely damaged. Delayed radial nerve palsy is usually due to oedema. Late onset of radial nerve palsy is usually due the involvement of the nerve with callu s. Non of the above is correct.
7- A supracondylar fracture of the humerus: ( All correct except one) • Is a fracture commonly seen in childhood. • Is particularly subject to the complication of ischaemic muscle contracture. • Is held in the position of reduction by the tendon of brachioradialis. • When properly reduced has the index finger pointing approximately to the tip of the shoulder of the same side. • When reduced care must be taken to ensure that the radial pulse is present. 8- A transverse fracture of the scaphoid is:
• • • • •
Prone to infection. Usually seen in old men. Prone to avascular necrosis. Usually seen on an early scaphoid x-ray. All of the above is correct.
9- In a colles’ fracture the distal radial fragment:
49
• • • • •
Is dorsally angulated on the proximal radius. Is usually torn from the intra-articular triangular disc. Is deviated to the ulnar side. Is rarely impacted. Is ventrally displaced.
10- dislocations of the shoulder joint:
• • • • •
Most commonly occur in middle age. Usually occur when the arm is in the abducted position. Usually have the head of the humerus situated behind the glenoid fossa. Are often recurrent in the young. B&D only.
11- In pelvic fractures: • Avulsion injuries are usually treated by early mobilisation. • Undisplaced lesions of the ischial or pubic rami are usually treated by early dmbilisation. • Extraperitoneal urinary extravasation may be due to damage either to the membraneous urethra or to the base of the bladder. • Which are unstable are accompanied by extensive hemorrhage. • All of the above are correct. 12- Intracapsular fractures of the upper end of the femur are usually: (All correct except one)
• • • • •
Accompanied by shortening of the leg. Accompanied by external rotation of the leg. Accompanied by adduction of the leg. Treated by internal fixation. Accompanied with a vascular necrosis of the head of femur.
13- Extracapsular fractures of the upper end of the femur are usually:
• • • • •
Subtrochanteric in position. Subject t o avascular necrosis of the head of the femur. Accompanied by internal rotation of the leg. Treated by external fixation. Non of the above are correct.
14- In fractures of the mid shaft of the femur: ( all correct except one)
• • • • 50
The proximal fragment is usually flexed. The proximal fragment is usually abducted. The distal fragment is usually adducted. The common femoral vessels are usually damaged.
•
Hamstring and quadriceps produce some shortening of the leg.
15- In fractures of the middle third of the tibia and fibula: (All correct except one)
• • • • •
Delayed union is common. Indirect violence usually results in a spiral or oblique fracture line. Shortening and anterior angulation of the tibia are common. Comminuted fractures are usually treated external fixation. All of the above are correct.
16- In injuries of the ankle joint:
• • • • •
Eversion injuries are the most commonly encoutered. Inversion injuries are usually accomapanied by a tear of the deltoid ligament. There is frequently associated posterior tibial nerve damage . The joint is rendered unstable by rupture of the inferior tibio-fibular ligament. A&B only.
17- Dislocation of the hip joint: • Is most common when the hip is in a neutral position. • Is usually associated with a fracture of the acetabular rim. • Usually results in the femoral head coming to lie anteiorly over the pubis or obturator externus. • May be associated with injuries of the sciatic nerve. • B&D only. 18- Acute osteomyelitis in childhood:
• • • • •
Is usually the result of compound bony injuries. Is characterised by a constant bone pain. Characteristiclly produces necrosis of the periosteum overlying the infected bone. Is not demonstrable radiologically for the first week of the disease. All of the above are correct.
19- In osteoarthritis of the hip joint:
•
The articular cartilage undergoes initial hypertrophy and then becomes hardened and eburnated.
• • • •
The joint capsule becomes stretched and lax. The leg is usually adducted and externally rotated when the patient lies supine. A femoral osteotomy usually helps halt the progress of the disease process. C&D only.
20- In a case of congenital dislocation of the hip: • There is a defect of the posterior rim of the acetabulum. • On bilateral hip abduction with the knees extended there is often limited abduction on the diseased side.
51
• • •
Reduction is sometimes hindered by a tight gluteus minimus muscle. Splinting of the limbs following reduction should be maintained until the femoral epiphysis returns to its normal density on x-ray examination. Non of the above is correct
21- Osteognic sarcoma:
• • • • •
Are most frequent in the 10 to 25 year age group. Readily metastasise via the blood stream. Are frequently surrounded by non-malignant new bone formation. When treated by conventional methods have a 10 percent 5 years survival rate. All of the above are correct.
1-Which is the shortest phase of the normal cell cycle? a) G1 phase b) S phase c) G2 phase d) M phase e) All phases approximately equal in length 2- A 67 year-old women with rectal cancer is admitted to gereral surgical floor which of the following laboratory studies should be included in the surgeon’s initial nutritional assessment : a) Transferrin b) Prealbumin c) Albumin d) Glutamine e) All of above 3- In which of the following conditions is the entral route appropriate for nutrition : a) Upper gastrointestinal obstruction b) Complete small bowel obstruction c) Acute flare-up of Crohn’s disease d) Low out put colonic fistula e) Non of the above 4-5-Which is the most commonly cultured hospital acquired organism in critical care with aspiration pneumonia: a) Staphylococcus aureus b) Streptococcus pneumonia c) Anaerobic species d) Pseudomonas aeroginosa e) Haemophlus influenzae 5-Which is the most appropriate single agent for empiric coverage of the above patient : a) Metranidazole b) Clindamycin c) Pipracillin_tazobactam d) Vancomycin e) First generation penicillin 6-All of the following are true about neurogenic shock except: a) There is a decrease in systemic vascular resistance and an increase in venous capacitance. b) Tachycardia or bradycardia may be observed, along with hypotension.
52
c) The use of an alpha agonist such as phenylephrine is the mainstay of treatment . d) Severe head injury, spinal cord injury, and high spinal anesthesia may all cause neurogenic shock. e) A and B 7-Which of the following statements about head injury and concomitant hyponatremia are true? a) b) c) d) e)
There are no primary alterations in cardiovascular signs . Signs of increased intracranial pressure may be masked by the hyponatremia. Oliguric renal failure is an unlikely complication. Rapid correction of the hyponatremia may prevent central pontine injury. This patient is best treated by restriction of water intake
8 Regarding Cushing's Syndrome a) 20% of cases are due to pituitary adenomas (Cushing's Disease) b) Most ACTH secreting pituitary adenomas are more than 2 cm in diameter c) Is characterised by loss of the diurnal rhythm of cortisol secretion d) Cortisol production is suppressed by low-dose dexamethasone e) Adrenal carcinomas are more common than adrenal adenomas 9-Which of the following statements about extracellular fluid are true? a) b) c) d)
The total extracellular fluid volume represents 40% of the body weight. The plasma volume constitutes one fourth of the total extracellular fluid volume . Potassium is the principal cation in extracellular fluid. The protein content of the plasma produces a lower concentration of cations than in the interstitial fluid. e) The interstitial fluid equilibrates slowly with the other body compartments. 10-In patients receiving massive blood transfusion for acute blood loss, which of the following is/are correct? a) Packed red blood cells and crystalloid solution should be infused to restore oxygencarrying capacity and intravascular volume. b) Two units of FFP should be given with every 5 units of packed red blood cells in most cases. c) A “six pack” of platelets should be administered with every 10 units of packed red blood cells in most cases. d) One to two ampules of sodium bicarbonate should be administered with every 5 units of packed red blood cells to avoid acidosis. e) One ampule of calcium chloride should be administered with every 5 units of packed red blood cells to avoid hypocalcemia.
11- Which of the following statements about the presence of gallstones in diabetes patients is/are correct? a) Gallstones occur with the same frequency in diabetes patients as in the healthy population.
53
b) The presence of gallstones, regardless of the presence of symptoms, is an indication for cholecystectomy in a diabetes patient. c) Diabetes patients with gallstones and chronic biliary pain should be managed nonoperatively with chemical dissolution and/or lithotripsy because of severe complicating medical conditions and a high operative risk. d) The presence of diabetes and gallstones places the patient at high risk for pancreatic cancer. e) Diabetes patients with symptomatic gallstones should have prompt elective cholecystectomy, to avoid the complications of acute cholecystitis and gallbladder necrosis. 12. concerning Tetanus all true except: a. Is due to an infection with a gram-negative spore forming rod b. The organism produces a powerful exdotoxin c. The toxin prevents the release of inhibitory neurotransmitter d. Clostridium tetani is sensitive to penicillin e. Risus sardonicus is the typical facial spasm Clostridium is a genus of Gram-positive bacteri 13-when should parentral antibiotics be given perioperatively? a) b) c) d) e)
The night before 6 hr prior to surgery 30 minutes prior to incision . at the time of incision 30 minutes after incision
14. Which of the following statements about esophageal anatomy is correct? a) The esophagus has a poor blood supply, which is segmental in distribution and accounts for the high incidence of anastomotic leakage. b) The esophageal serosa consists of a thin layer of fibroareolar tissue. c) The esophagus has two distinct muscle layers, an outer, longitudinal one and an inner, circular one, which are striated in the upper third and smooth in the distal two thirds . d) Injury to the recurrent laryngeal nerve results in vocal cord dysfunction but does not affect swallowing. e) The lymphatic drainage of the esophagus is relatively sparse, localized primarily to adjacent paraesophageal lymph nodes. 15 Wich of the following medication should be given in preparation of a pation with pheochromocytoma? a) b) c) d) e)
Phnoxybenzamine Nifedipine Linsinopril Hydrochlorothiazide Propranolol
16-Which of the following statement(s) is true concerning excessive scarring processes? a) Keloids occur randomly regardless of gender or race
54
b) Hypertrophic scars and keloid are histologically different c) Keloids tend to develop early and hypertrophic scars late after the surgical injury d) Simple reexcision and closure of a hypertrophic scar can be useful in certain situations such as a wound closed by secondary intention e) Non of the above 17-A 22-year-old man sustains a single stab wound to the left chest and presents to the emergency room with hypotension. Which of the following statement(s) is true concerning his diagnosis and management? a) The patient likely is suffering from hypovolemic shock and should respond quickly to fluid resuscitation b) Beck’s triad will likely be an obvious indication of compressive cardiogenic shock due to pericardial tamponade c) Echocardiography is the most sensitive noninvasive approach for diagnosis of pericardial tamponade d) The placement of bilateral chest tubes will likely resolve the proble 18-Which of the following statement(s) is/are true concerning septic shock? a) The clinical picture of gram negative septic shock is specifically different than shock associated with other infectious agents b) The circulatory derangements of septic shock precede the development of metabolic abnormalities c) Splanchnic vascular resistance falls in similar fashion to overall systemic vascular resistance d) Despite normal mechanisms of intrinsic expansion of the circulating blood volume, exogenous volume resuscitation is necessary 19- During surgery on the submandibular gland a) An incision on the lower border of the mandible is safe b) The submandibular gland is seen to wrap around the posterior border of mylohyoid c) The facial artery and vein are divided as they course through the deep part of the gland d) The hypoglossal nerve is seen to loop under the submandibular duct e) Damage to the lingual nerve will cause loss of sensation to the posterior third of the tongue 20- Regarding benign breast diseas ex a) Cyclical mastalgia is the commonest reason for referral to the breast clinic b) Fibroadenomas are derived from the breast lobule c) Lactational breast abscesses are usually due to Staph aureus d) Duct ectasia is more common in smokers e) Atypical lobular hyperplasia is associated with an decreased risk of breast cancer
21-. Which of the following statements regarding unusual hernias is incorrect?
55
a) An obturator hernia may produce nerve compression diagnosed by a positive HowshipRomberg sign. b) Grynfeltt's hernia appears through the superior lumbar triangle, whereas Petit's hernia occurs through the inferior lumbar triangle. c) Sciatic hernias usually present with a painful groin mass below the inguinal ligament . d) Littre's hernia is defined by a Meckel's diverticulum presenting as the sole component of the hernia sac. e) Richter's hernia involves the antimesenteric surface of the intestine within the hernia sac and may present with partial intestinal obstruction 22- Staples may safely be placed during laparoscopic hernia repair in each of the following structures except: a) b) c) d) e)
Cooper's ligament. Tissues superior to the lateral iliopubic tract. The transversus abdominis aponeurotic arch. Tissues inferior to the lateral iliopubic tract . The iliopubic tract at its insertion onto Cooper's ligament.
23-The following statements about the repair of inguinal hernias are true except: a) The conjoined tendon is sutured to Cooper's ligament in the Bassini hernia repair . b) The McVay repair is a suitable option for the repair of femoral hernias. c) The Shouldice repair involves a multilayer, imbricated repair of the floor of the inguinal canal. d) The Lichtenstein repair is accomplished by prosthetic mesh repair of the inguinal canal floor in a tension-free manner. e) The laparoscopic transabdominal preperitoneal (TAPP) and totally extraperitoneal approach (TEPA) repairs are based on the preperitoneal repairs of Cheattle, Henry, Nyhus, and Stoppa. 24-A number of special circumstances exist in the repair of inguinal hernias. The following statement(s) is correct. a) Simultaneous repair of bilateral direct inguinal hernias can be performed with no significant increased risk of recurrence b) The preperitoneal approach may be appropriate for repair of a multiple recurrent hernia c) A femoral hernia repair can best be accomplished using a Bassini or Shouldice repair d) Management of an incarcerated inguinal hernia with obstruction is best approached via laparotomy incision e) All are correct. 25- Which of the following statement(s) is true about benign lesions of the liver? a) Adenomas are true neoplasms with a predisposition for complications and should usually be resected. b) Focal nodular hyperplasia (FNH) is a neoplasm related to birth control pills (BCPs) and usually requires resection. c) Hemangiomas are the most common benign lesions of the liver that come to the surgeon's attention. d) Nodular regenerative hyperplasia does not usually accompany cirrhosis 26. Ligation of all of the following arteries usually causes significant hepatic enzyme abnormalities except:
56
a) b) c) d)
Ligation of the right hepatic artery. Ligation of the left hepatic artery. Ligation of the hepatic artery distal to the gastro-duodenal branch. Ligation of the hepatic artery proximal to the gastroduodenal artery
27- Which of the following is the most effective definitive therapy for both prevention of recurrent variceal hemorrhage and control of ascites? a) b) c) d) e)
Endoscopic sclerotherapy. Distal splenorenal shunt. Esophagogastric devascularization (Sugiura procedure). Side-to-side portacaval shunt. End-to-side portacaval shunt.
28-which of the following is associated with best prognosis for patient with breast cancer? a) b) c) d) e)
Male sex Estrogen receptor positive Patient age <35 years Pregnant patient Tumor with overexpression of HER/ner.
29-A 49-year-old women has a palpable breast mass in the upper outer quadrant. The size of the mass has increased over the last month . exicisional biopsy reveals cystic carcinoma with invasion .appropriate management now would be : a) b) c) d) e)
Re-excision with wide margins Axillary node dissection and hormonal therapy Simple mastectomy Modified radical mastectomy Bilateral mastectomies
30-A contraindication to stereotactic core biopsy of the breast is the mammographic presence of: a) b) c) d) e)
Microcacification A radial scar A nonpalpable mass lesion Lesions<8 mm in diameter Mutifocal lesions.
31-. Which of the following statements about epiphrenic diverticula of the esophagus is/are correct? a) b) c) d) e)
They are traction diverticula that arise close to the tracheobronchial tree. They characteristically arise proximal to an esophageal reflux stricture. The degree of dysphagia correlates with the size of the pouch. They are best approached surgically through a right thoracotomy. The operation of choice is a stapled diverticulectomy, long esophagomyotomy, and partial fundoplication.
32- Which of the following statements about Schatzki's ring is correct?
57
a) b) c) d) e)
The ring represents a panmural fibrotic stricture resulting from gastroesophageal reflux. Dysphagia occurs when the ring diameter is 13 mm. or less . The ring occurs within 1 to 2 cm. of the squamocolumnar epithelial junction. Schatzki's ring indicates reflux esophagitis. Schatzki's ring signifies the need for an antireflux operation.
33. Which of the following statements about pathology encountered at esophagoscopy is/are correct? a) Reflux esophagitis should be graded as mild, moderate, or severe, to promote consistency among different observers. b) An esophageal reflux stricture with a 2-mm. lumen is not dilatable and is best treated with resection. c) A newly diagnosed radiographic distal esophageal stricture warrants dilation and antireflux medical therapy. d) In patients with Barrett's mucosa, the squamocolumnar epithelial junction occurs 3 cm. or more proximal to the anatomic esophagogastric junction. e) After fasting at least 12 hours, a patient with megaesophagus of achalasia can safely undergo flexible fiberoptic esophagoscopy. 34. Infantile hypertrophic pyloric stenosis a) Occurs with a female : male ratio of 4:1. b) Sons of affected mothers have a 20% risk of developing the lesion c) Invariably presents between six and eights months of age d) Typically presents with bile stained projectile vomiting e) Surgical treatment is by Heller's Cardiomyotomy 35-. Which of the following statements regarding the pathology of esophageal carcinoma is/are correct? a) Worldwide, adenocarcinoma is the most common esophageal malignancy. b) Squamous cell carcinoma is most common in the distal esophagus, whereas adenocarcinoma predominates in the middle third. c) Patients with Barrett's metaplasia are 40 times more likely than the general population to develop adenocarcinoma. d) Metastases from esophageal carcinoma are characteristically localized to regional mediastinal lymph nodes adjacent to the tumor. e) Achalasia, radiation esophagitis, caustic esophageal stricture, Barrett's mucosa, and Plummer-Vinson syndrome are all premalignant esophageal lesions that predispose to the development of squamous cell carcinoma. 36-45-year-old with isolated 6-cm colorectal metastasis in the liver 2 years after colectomy, otherwise healthy pest treatment would be: a) b) c) d) e)
Radiofrequency ablation Systemic chemotherapy Hepatic lobectomy Liver transplantation Cryosurgical ablation
37- Oesophageal atresia all true except: a) Is often associated with a distal trachea-oesophageal fistula
58
b) Polyhydramnios is often present early late in pregnancy c) 50% have other associated congenital abnormalities d) Contrast X-ray studies are necessary to confirm the diagnosis e) Post-operatively over 30% develop oesophageal strictures 38-All are true about the dumping syndrome except: a) b) c) d)
Symptoms can be controlled with a somatostatin analog. Diarrhea is always part of the dumping syndrome. Flushing and tachycardia are common features of the syndrome. Separating solids and liquids in the patient's oral intake alleviates some of the symptoms of the syndrome. e) Early postoperative dumping after vagotomy often resolves spontaneously. 39-Which of the following statements about gastric polyps is/are true? a) Like their colonic counterparts, gastric epithelial polyps are common tumors. b) They are analogous to colorectal polyps in natural history. c) Endoscopy can uniformly predict the histology of a polyp based on location and appearance. d) In a given patient, multiple polyps are generally of a multiple histologic type. e) Gastric adenomatous polyps greater than 2 cm. in diameter should be excised because of the risk of malignant transformation. 40-All of the following statements about surgical management of gastric lymphomas are true except: a) Stage I gastric lymphomas (small lesions confined to the stomach wall) can be cured completely with surgical therapy alone. b) Extensive gastric lymphomas that initially are treated with radiation and/or chemotherapy occasionally perforate during treatment and require secondary resection. c) Patients explored with a presumptive diagnosis of gastric lymphoma should undergo an attempt at curative resection when this is safe and feasible. d) Without a preoperative diagnosis resection for gastric mass should not be attempted unless lymphoma can be excluded. e) Appropriate staging for primary gastric lymphoma includes bone marrow biopsy. 41-the most accurate test to confirm diagnosis of infected necrotizing pancreases is: a) b) c) d) e)
Abdominal ultrasound study Indium-labeled leeukocte scan Cimputed tomographic scan Elevated serum level of interleukain 6 and 8 Percutaneous needle aspiration
42- Which of the following variables best predicts prognosis for patients with a recent diagnosis of cutaneous melanoma and no clinical evidence of metastatic disease?
59
a) b) c) d) e)
Breslow thickness. Clark's level. Ulceration. Gender. Celtic complexion.
43-the following are true about intracranial tumors except: a) The most common location of brain tumors of childhood is the posterior cranial fossa. b) With few exceptions, examination of the CSF is of no value in the diagnosis of an intracranial tumor. c) Even the most malignant of primary brain tumors seldom spread outside the confines of the central nervous system (CNS). d) The majority of astrocytomas can be cured surgically. e) Primary neoplasms of astrocytic, oligodendroglial, or ependymal origin represent gradations of a spectrum from slowly growing to rapidly growing neoplasms. 44 A right-sided disc herniation at the L5–S1 level typically may cause: a) b) c) d) e)
Low back pain and left sciatica. Weakness of dorsiflexion of the right foot. A diminished or absent right ankle jerk. Diminution of sensation over the medial aspect of the right foot, including the great toe. Weakness of dorsiflexion of the left foot.
45-. The preferred operation for initial management of a thyroid nodule that is considered suspicious for malignancy by FNAB is: a) b) c) d) e)
Excision. Partial lobectomy. Total lobectomy and isthmusectomy. Total thyroidectomy. All methods are correct
46-the most common presentation of Meckel,s diverticulum in an adult is: a) b) c) d) e)
GIT bleeding GIT obstruction Intussuception Litter,s hernia Diverticulitis
47-Optimal front-line treatment of squamous cell carcinoma of the rectum includes: a) b) c) d) e)
Abdominal perineal resection. Low anterior resection when technically feasible. Radiation therapy. Chemotherapy. Combined radiation and chemotherapy.
48 -65-year-old man presents with complaints of mucous discharge and perianal discomfort. Physical examination reveals a fistulous opening lateral to the anus. Anoscopic examination permits passage of a probe through the fistula tract. The fistula traverses the internal anal sphincter, the
60
intersphincteric plane, and a portion of the external anal sphincter. The fistula is categorized as which type? a) Intersphincteric b) Transsphincteric c) Suprasphincteric d) Extrasphincteric e) Non of the above 49-Warthin's tumor: a) b) c) d) e)
Is a pleomorphic adenoma of salivary gland Should be treated by total paritidectomy Is considered a benign salivary gland neoplasia Respond well to preoperative radiotherapy Often present with facial nerve compression
50-A 38 year old woman presents with right upper quadrant pain and bouts of vomiting. She is known to have gallstones and has had similar episodes in the past. Which of the following might support a diagnosis of acute cholecystitis rather than biliary colic a) duration of symptoms b) Severity of vomiting c) Presence of Murphy's sign d) Presence of gas under right hemidiaphragm on erect CXR. 1. In the workup on a patient for possible appendicitis, CT scanning should be performed: a) Before consulting the surgeon, by the emergency physician b) In patients with equivocal physical findings c) Routinely, in all patients with right lower quadrant pain d) With equal frequency in men and women e) Never 2. a. b. c. d. e.
Acute appendicitis Occurs most commonly in the second and third decades of life Can be cured readily by antibiotics Is most commonly caused by a fecalith Carries an overall mortality rate of 7% Induces leukocytosis in 90% of patients
3. The gastric mucosal cell that secretes intrinsic factor is the a) G cell b) Parietal cell c) D cell d) Enterochromaffin-like cell d) Chief cell b— 4. Hypertrophic pyloric stenosis is likeliest to occur in a(n) a) Firstborn child b) African-American infant c) Child 6 to 9 months of age
61
d) Female infant e) Infant born prematurely a— 5. Overwhelming postsplenectomy infection (OPSI) a) Occurs more frequently after resection for trauma than hematologic disease b) Occurs with equal frequency in children and adults c) Is most frequently caused by Streptococcus pneumoniae d) Usually occurs within 2 years after splenectomy e) Generally has an identifiable site of infection c— 6. The characteristic feature of Crohn's colitis that best distinguishes the clinical entity from ulcerative colitis is a) Perianal disease b) Rectal bleeding c) Risk of malignancy d) Obstructive symptoms e) Pseudopolyps a— 7. Type 1 gastric ulcers are a) At the incisura b) Along the greater curvature c) Prepyloric d) Associated with simultaneous duodenal ulcers e) Close to the esophagogastric junction a— 8. Radiation enteritis a) Usually presents with perforation b) Is caused by thrombosis of mucosal vessels c) Occurs after 3,000 cGy of abdominal radiation d) Routinely requires operative therapy e) Is likely in patients who have undergone laparotomy e– 9. The small bowel tumor with the greatest propensity for bleeding is a) Carcinoid b) Lymphoma c) Adenocarcinoma d) Hamartoma e) Leiomyoma e— 10.
a—
62
A grade III laceration of the spleen is characterized by a. Intraparenchymal hematoma > 5 cm b. Capsular tear 1-3 cm deep c. Hilar vessel disruption d. Subcapsular hematoma involving 10%-50% of surface area e. Active hemorrhage
11. The spleen filters all of the following particles/cells EXCEPT a. Malformed erythrocytes b. T lymphocytes c. Malarial parasites d. Streptococcus pneumoniae e. Platelets b— 12. which of the following is the strongest of all other risk factors in the development of Gastric carcinoma : A. B. C. D. E. B
Helicobacter pylori. Atrophic gastritis. Blood group A. Pernicious anemia. Low socioeconomic class.
13. whilst performing a small bowel resection for strictures following crohn”s disease, you realize that on inspection, there are marked differences between jejunal and ileal anatomy. Such differences include all the following Except: A. Wider lumen in the jejunum. B. Less lymphatics in the jejunal mesentry compared to ileal. C. More prominent and multiple arcades of vessels in the ileum. D. Thicker wall of the jejunum. E. Thicker and more fat-laden mesentry increasing towards the ileum. D 14. A 54 years old woman is referred to your surgical team with a diagnosis of small bowel obstruction. Which one of the following clinical signs would you look for in trying to identify the commonest cause of this condition : A. B. C. D. E.
surgery scar. Lump in the groin above & medial to the pubic tubercle. Lump in the groin below & lateral to the pubic tubercle. Cachexia & nodule at the umbilicus. Circumoral pigmentation & a family history of previous obstruction.
A 15. True statements regarding appendiceal neoplasms include which of the following? A. Carcinoid tumors of the tip of the appendix less than 1.5 cm are adequately treated by simple appendectomy B. Appendiceal carcinoma is associated with secondary tumors of the GI tract in up to 60% of patients C. Survival following right colectomy for a Dukes’ stage C appendiceal carcinoma is markedly better than that for a similarly staged colon cancer at 5 years
63
D. Mucinous cystadenocarcinoma of the appendix is adequately treated by simple appendectomy, even in patients with rupture and mucinous ascites E. Up to 50% of patients with appendiceal carcinoma have metastatic disease, with the liver as the most common site of spread :A 16. All of the following statements about carcinoma of the gallbladder are correct Except : A. The neoplasm usually starts in the cystic duct and neck of the gallbladder. B. It is found more commonly in women than men. C. It is associated with the presence of gallstones in > 85% of cases. D. Prognosis is generally poor with < 1 year survival with local invasion. E. Chemotherapy and radiotherapy do not alter disease progression. A 17. All of the following statements are true regarding diverticular disease Except : A. It is found more commonly in the developed world. B. Surgical treatment is usually unnecessary in acute uncomplicated cases. C. Diverticulae are more commonly found in the descending colon. D. Perforation and fistula formation can result from an attack of acute diverticulitis. E. Resolution of the diverticulae can occur with high fiber diet and adequate hydration. E 18. All of the following arteries are branches of the superior mesenteric artery,except: a. Ileocolic b. Replaced left hepatic c. Inferior pancreaticoduodenal d. Jejunal e. Replaced right hepatic b— 19. Mesenteric cysts a. Occur primarily in the mesocolon b. Are discovered mainly in children c. Can contain chyle or serous fluid d. Require total enucleation for a cure e. Are neoplastic c— 20. Decompression for abdominal compartment syndrome should be performed a. If the urine output falls to 30 mL/hour
64
b. When the patient's respiratory rate increases to 24 breaths per minute c. Based purely on physical findings d. When bladder pressure exceeds 35 mm Hg e. If the patient becomes hypoxemic d— 21. In laparoscopic surgery, the vessel most likely to be punctured during trochar placement is the a. Aorta b. Right common iliac artery c. Vena cava d. Left common iliac artery e. Right common iliac vein b— 22. Following resolution of acute appendicitis, an interval appendectomy a. In 90% of interval appendectomies no pathologic abnormality could be found. b. Always reveals luminal occlusion of the appendix c. Should be performed because of the high incidence of recurrent appendicitis d. Is not indicated in patients older than 40 years e. Needs to be performed open because of the fibrosis induced by appendicitis a 23. True statements concerning the diagnosis and management of retroperitoneal fibrosis include all of the following except: A. Most patients present with dull, non-colicky back, flank, or abdominal pain B. Evidence of impaired renal function with an elevated blood urea nitrogen is common C. The diagnosis is most commonly suggested by intravenous pyelography although contrast studies with CT scan or MRI are useful in further defining the disease D. Most patients will need operative intervention. E. The prognosis for nonmalignant retroperitoneal fibrosis is grim with progression of disease until death occurring in most patients :E 24. Lynch Syndrome all are true except? a) It is due to mutation in MMR gene b) Associated with APC gene c) It has Autosomal Dominant Inheritence
65
d) Adenomas in patients with Lynch syndrome display high grade dysplasia than adenoma in patients with sporadic colorectal cancer? e) It occurs predominantly on right side, has increased incidence of synchronous and metachronous disease. B 25. Which of the folowing about Pancreatic Ascites is not true a) Conservative treatment effective in only 1/4th of patients b) ERCP should be done before surgery c) It is exudative d) Metaplastic cells are present e) Pancreatic fluid has high amylase and high albumin A 26. Tetanus all are true except? a. Is due to an infection with a gram-negative spore forming rod b. The organism produces a powerful exotoxin c. The toxin prevents the release of inhibitory neurotransmitter d. Clostridium tetani is sensitive to penicillin e. Risus sardonicus is the typical facial spasm A 27. Which of the following statements about treatment of esophageal carcinoma is/are correct?
the
surgical
A. The finding of severe dysphagia in association with Barrett's mucosa is an indication for an antireflux operation to prevent subsequent development of carcinoma. B. Long-term survival is improved by radical en bloc resection of the esophagus with its contained tumor, adjacent mediastinal tissues, and regional lymph nodes. C. The morbidity and mortality rates for cervical esophagogastric anastomotic leak are substantially less than those associated with intrathoracic esophagogastric anastomotic leak. D. The leading complications of transthoracic esophagectomy and intrathoracic esophagogastric anastomosis are bleeding and wound infection. E. Transhiatal esophagectomy without thoracotomy achieves better long-term survival than transthoracic esophagectomy. Answer: C 28. The most effective therapy for morbid obesity, in terms of weight control, is:
66
A. Intensive dieting with behavior modification. B. A multidrug protocol with fenfluramine, phenylpropanolamine, and mazindol. C. A gastric bypass with a 40-ml. pouch, a 10- to 20-cm. Roux-en-Y gastroenterostomy. D. A gastric bypass with a 15-ml. pouch, a 40- to 60-cm. Roux-en-Y gastroenterostomy. E. Daily exercise with strong emphasis on utilizing all four limbs. Answer: D 29. All of the following statements about the embryology of Meckel's diverticulum are true except: A. Meckel's diverticulum usually arises from the ileum within 90 cm. of the ileocecal valve. B. Meckel's diverticulum results from the failure of the vitelline duct to obliterate. C. The incidence of Meckel's diverticulum in the general population is 5%. D. Meckel's diverticulum is a true diverticulum possessing all layers of the intestinal wall. E. Gastric mucosa is the most common ectopic tissue found within a Meckel's diverticulum. Answer: C 299. What is the most common serious complication of an end colostomy? a. Bleeding b. Skin breakdown c. Parastomal hernia d. Colonic perforation during irrigation e. Stomal prolapse 310. Which of the following colonic pathologies is thought to have no malignant potential? a. Ulcerative colitis b. Villous adenomas c. Familial polyposis d. Peutz-Jeghers syndrome e. Crohn’s colitis 308. Which of the following hernias follows the path of the spermatic cord within the cremaster muscle? a. Femoral b. Direct inguinal c. Indirect inguinal d. Spigelian 313. Spontaneous closure of which of the following congenital abnormalities of the abdominal wall generally occurs by the age of 4? a. Umbilical hernia
67
b. Patent urachus c. Patent omphalomesenteric duct d. Omphalocele 314. Laparoscopic cholecystectomy is indicated for symptomatic gallstones in which of the following conditions? a. Cirrhosis b. Prior upper abdominal surgery c. Suspected carcinoma of the gallbladder d. Morbid obesity e. Coagulopathy 318. In determining the proper treatment for a sliding hiatal hernia, the most useful step would be a. Barium swallow with cinefluoroscopy during Valsalva maneuver b. Flexible endoscopy c. 24-h monitoring of esophageal pH d. Measuring the size of the hernia surgical treatment for sliding esophageal hernias should only be considered in symptomatic patients with objectively documented esophagitis or stenosis. The overwhelming majority of sliding hiatal hernias are totally asymptomatic, even many of those with demonstrable reflux. Even in the presence of reflux, esophageal inflammation rarely develops because the esophagus is so efficient at clearing the refluxed acid. Symptomatic hernias should be treated vigorously by the variety of medical measures that have been found helpful. Patients who do have symptoms of episodic reflux and who remain untreated can expect their disease to progress to intolerable esophagitis or fibrosis and stenosis. Neither the presence of the hernia nor its size is important in deciding on surgical therapy. Once esophagitis has been documented to persist under adequate medical therapy, manometric or pH studies may help .determine the optimum surgical treatment 320. A previously healthy 9-year old child comes to the emergency room because of fulminant upper gastrointestinal bleeding. The hemorrhage is most likely to be the result of a. Esophageal varices b. Mallory-Weiss syndrome c. Gastritis d. A gastric ulcer Massive hematemesis in children is almost always due to variceal bleeding. The varices usually result from extrahepatic portal vein obstruction consequent to bacterial infection transmitted via a patent umbilical vein during infancy. In spite of this common cause, a history of neonatal omphalitis is infrequently obtainable. Bleeding can be massive but is usually self-limited, and esophageal tamponade or vasopressin is usually not necessary. Elective portal-systemic decompression is recommended for recurrent bleeding .episodes
68
321. Intragastric pressure remains steady near 2–5 mm Hg during slow gastric filling, but rises rapidly to high levels after reaching a volume of a. 400–600 mL b. 700–900 mL c. 1000–1200 mL d. 1300–1500 mL 325. Local stimuli that inhibit the release of gastrin from the gastric mucosa include which of the following? a. Small proteins b. 20-proof alcohol c. Caffeine d. Acidic antral contents 327. For a symptomatic partial duodenal obstruction secondary to an annular pancreas, the operative treatment of choice is a. A Whipple procedure b. Gastrojejunostomy d. Partial resection of the annular pancreas e. Duodenojejunostomy 332. Which of the following would be expected to stimulate intestinal motility? a. Fear b. Gastrin c. Secretin d. Acetylcholine e. Cholecystokinin 333. Which of the following statements concerning carcinoma of the esophagus is true? b. Squamous carcinoma is the most common type at the cardioesophageal junction c. It has a higher incidence in males d. It occurs more commonly in patients with corrosive esophagitis e. Surgical excision is the only effective treatment Carcinoma of the esophagus occurs primarily in the sixth and seventh decades of life in a male-to-female ratio of 3:1. Although the cause is unknown, alcohol, tobacco, and dietary factors have been implicated as causative agents. A high incidence is reported in patients with corrosive esophagitis. Malignant tumors arising in the esophagus are usually squamous cell carcinomas, except those involving the esophagogastric junction, which are usually adenocarcinomas. Even though squamous cell carcinomas are weakly radiosensitive, surgical extirpation affords reasonable, if short-term, palliation. Some authorities recommend radiotherapy for palliation alone or in combination with surgery to treat this lesion. Adenocarcinomas are not particularly radiosensitive, and surgical treatment is generally employed. Following resection for esophageal carcinoma among the highly select group of patients whose tumors are still resectable when the diagnosis is made, survival is only about 14% at 5 years. The overall 5-year survival is under 5%.
69
336. The most common clinical presentation of idiopathic retroperitoneal fibrosis is a. Ureteral obstruction b. Leg edema d. Jaundice e. Intestinal obstruction 338. In planning the management of a 2.8-cm epidermoid carcinoma of the anus, the first therapeutic approach should be e. Combined radiation therapy and chemotherapy 340. Indications for operation in Crohn’s disease include which of the following? a. Intestinal obstruction b. Enterovesical fistula c. Ileum–ascending colon fistula e. Free perforation 346. Which of the following is most likely to require surgical correction? a. Large sliding esophageal hiatal hernia b. Paraesophageal hiatal hernia c. Traction diverticulum of esophagus d. Schatzki’s ring of distal esophagus 353. Which statement regarding adenocarcinoma of the pancreas is true? a. It occurs most frequently in the body of the gland b. It carries a 1–2% 5-year survival rate c. It is nonresectable if it presents as painless jaundice d. It can usually be resected if it presents in the body or tail of the pancreas and does not involve the common bile duct 356. Which of the following statements regarding direct inguinal hernias is true? a. They are the most common inguinal hernias in women b. They protrude medially to the inferior epigastric vessels c. They should be opened and ligated at the internal ring d. They commonly protrude into the scrotal sac in men 358. Which of the following statements regarding stress ulceration is true? a. It is true ulceration, extending into and through the muscularis mucosa b. It classically involves the antrum c. Increased secretion of gastric acid has been shown to play a causative role d. It frequently involves multiple sites
70
359. Which statement concerning cholangitis is correct? a. The most common infecting organism is Staphylococcus aureus b. The diagnosis is suggested by the Charcot triad (fever, jaundice, pain) c. The disease occurs primarily in young, immunocompromised patients d. Cholecystectomy is the procedure of choice in affected patients 365. Indications for surgical removal of polypoid lesions of the gallbladder include a. Size greater than 0.5 cm b. Presence of clinical symptoms c. Patient age of over 25 years d. Presence of multiple small lesions v The cholecystectomy should only be undertaken in cases where there are clinical signs of gallbladder polyps, polyps with diameters greater than 10 mm, fast-growing polyps, sessile polyps or wide-based polyps, polyps with long pedicles, patient aged over 50, concurrent gallstones, polyps of the gallbladder in fundibulum or abnormal gallbladder wall ultrasound.[1] The surgery of choice is laparoscopic cholecystectomy. A gallbladder polyp greater than 18 mm in size has a high likelihood of being an advanced cancer; it should be removed with open cholecystectomy, partial liver resection, and possible lymph node dissection 366. A patient who has a total pancreatectomy might be expected to develop which of the following complications? a. Diabetes mellitus b. Hypercalcemia c. Hyperphosphatemia d. Constipation e. Weight gain 368. True statements regarding cavernous hemangiomata of the liver in adults include a. The majority become symptomatic b. They may undergo malignant transformation c. They enlarge under hormonal stimulation d. They should be resected to avoid spontaneous rupture and lifethreatening hemorrhage
1-A 35-year-old male is struck on the lateral aspect of his right knee by the bumper of a car travelling at low velocity. On examination he is unable to dorsiflex the ankle, evert the foot and extend the toes. There is loss of sensation of the dorsum of the foot. He is most likely to have damaged which structure? a. Common peroneal nerve b. Deep peroneal nerve
71
c. Saphenous nerve d. Sural nerve e. Tibial nerve
2-Which one of the following suggest a diagnosis of Hirschsprung's disease? a. b. c. d. e.
A contrast-study showing dilatation of the aganglionic bowel segment. Early presentation with vomiting. neonatal large bowel obstruction. Presentation after 1 year of age. Red current jelly stools.
3-A 46-year-old man presents after penetrating injuries to his arm and forearm. He is unable to extend his fingers. There is no sensory disturbance and there is no vascular injury. Which nerve has been damaged? a. b. c. d. e.
anterior interosseous nerve median nerve posterior interosseous nerve radial nerve ulna nerve
4-Which of the following regarding the anatomy of the heart is true? a. The aortic valve is tricuspid. b. The ascending aorta is entirely outside the pericardial sac. c. The left atrial appendage is identified readily by transthoracic echocardiography. d. The pulmonary trunk lies anterior to the ascending aorta. e. The right atrium is posterior to the left atrium. 5-Which of the following is not within the carpal tunnel? a. b. c. d. e.
median nerve flexor digitorum profundus flexor digitorum superficialis flexor pollicis longus abductor pollicis longus
6-A 48-year-old woman with chronic pancreatitis due to gallstones is noted to have a macrocytic anaemia. What is the most likely cause of the anaemia? a. b. c. d.
72
Bone marrow dysfunction Folate deficiency Hyposplenism Hypothyroidism
e. Vitamin B12 deficiency 7-Which of the following organs is in direct contact with the anterior surface of the left kidney, without being separated from it by visceral peritoneum? a. b. c. d. e.
Duodenum Jejunum Pancreas Spleen Stomach
8-A 5-year-old boy presents to Accident and Emergency complaining of acute pain over his upper tibia. He is febrile and he refuses to move his leg. A diagnosis of osteomyelitis is suspected. The likely infecting organism is? a. b. c. d. e.
Clostridium difficile Haemophilus influenzae Pseudomonas Salmonella Staphylococcus aureus
9-Which is the principle root inervation for the small muscles of the hand? a. b. c. d. e.
C5 C6 C7 C8 T1
10-Which of the following is true concerning Scaphoid fractures? a. Rarely occur in young adults b. when complicated by avascular necrosis the proximal pole is usually affected c. should be treated by bone grafting and internal fixation even if undisplaced d. wrist fractures are uncommon e. anteriorposterior and lateral radiographs reveal most fractures
11-A 17-year-old girl underwent emergency splenectomy after a domestic accident. Which one of the following organisms is most likely to cause life-threatening infection in the future? a. b. c. d. e.
73
Actinomycosis Haemophilus influenzae Pseudomonas aeruginosa Staphylococcus aureus Streptococcus pneumonia
12-Which vertebral level and corresponding structure is correct? a. b. c. d. e.
C4 and bifurcation of the carotid artery T2 and manubriosternal joint T10 and opening for vena cava in diaphragm T12 and oesophageal opening in the diaphragm T8 and aortic opening in the diaphragm
13-Which of the following statements is true of Compartment syndrome:a. b. c. d. e.
Only occurs following fractures. Loss of distal pulse is an early sign. The presence of pain is unhelpful in diagnosis. Can be treated using a minimally invasive technique. Passive stretch of affected muscles exacerbates pain .
14-The axillary nerve passes directly inferior to which muscle as it leaves the axilla? a. b. c. d. e.
Long head of triceps Long head of biceps Pectoralis major Subscapularis Teres minor
15-Which of the following is not a branch of the pudendal nerve? a. b. c. d. e.
Perineal nerve. Dorsal nerve of the penis. Inferior rectal nerve. Genitofemoral nerve. Posterior scrotal nerve
16-A patient presents with a history of low back pain and sciatica. The pain radiates to the little toe, the ankle reflex is absent and the patient has difficulty in everting the foot. Which nerve root is likely to be trapped? a. b. c. d. e.
L3 L4 L5 S1 S2
17-The commonest clinical manifestation of primary hyperparathyroidism is: a. Renal stone disease
74
b. Bone disease c. Peptic ulceration d. Constipation e. Polyuria Renal stone disease occurs in 50 % of patients 18-Which of the following structures accompany the median nerve in the carpal tunnel? a. b. c. d. e.
Flexor carpi ulnaris. Flexor digitorum profundis. The ulnar artery. All of the above. None of the above.
19-Which of the following is a branch of the LATERAL CORD of the brachial plexus? a. b. c. d. e.
Suprascapular nerve. Lower subscapular nerve. Medial pectoral nerve. Musculo-cutaneous nerve. Upper subscapular nerve.
20-Which of the following vertebrae has the most prominent spinous process? a. b. c. d.
T1. T2. C7. T11.
21-The ulnar nerve all are true except : A. innervates the first dorsal interosseus muscle B. originates from the laterall cord of the brachial plexus C. has no branches above the elbow D. innervates the medial half of the flexor digitorum profundus E. innervates the adductor pollicis muscle F. A 50-year-old female has been referred for investigation of abnormal liver function tests. On examination she is obese with mainly truncal obesity, with a moon face and a buffalo hump shaped deposit of fat across her shoulders. Her face is plethoric and there are numerous telangiectasia. The abdomen is protuberant and there are striae. In this patient: a) The biochemical abnormalities do not return to normal with abstinence b) There is little relationship between the degree of liver damage and the plasma cortisol levels c) Urinary 17 - hydroxycorticosteroid levels are suppressed
75
d) Plasma cortisol levels are suppressed e) Plasma ACTH levels are elevated Answer: b) The patient has alcoholic pseudocushings. There is little relationship between the degree of liver damage and the plasma cortisol levels BOF: 53 A 50-year-old man presents with malaise, weight loss, diarrhoea and pain in the joints. He is pigmented, has clubbing and lymphadenopathy. He has ascites and ophthalmoplegia. Investigations show him to be anaemic. ECG shows a right bundle branch block and paracentesis abdominis reveals chylous ascites. The investigation most likely to give a diagnosis would be: a) Lumbar puncture b) CT head c) Ascitic fluid cytology d) Small bowel biopsy e) Transoesphageal echocardiography Answer: d) 22-A 9 month old boy presents with an acute scrotal swelling. The following diagnoses are likely: A. B. C. D. E.
Epididymitis Orchitis Torsion of the testicular appendage Irreducible inguinal hernia Acute idiopathic scrotal oedema
23-Congenital pyloric stenosis: a. b. c. d. e.
Presents in the first few daies after birth. Can cause hyperchloraemic alkalosis. Is inherited as an autosomal recessive. Typically presents with bile stained vomiting after feeds Is often diagnosed by feeling a mass in the right hypochondrium .
24-The following regarding the anatomy of the heart are true: a. b. c. d. e.
The pulmonary trunk lies anterior to the ascending aorta. The ascending aorta is entirely outside the pericardial sac. The right atrium is posterior to the left atrium. The aortic valve is tricuspid. The left atrial appendage is identified readily by transthoracic echocardiography.
25-Meckel's diverticulum all are true except : a. Can present with chronic anaemia.
76
b. c. d. e.
Is present in 2% of the population. Can be diagnosed by hydrogen breath test. May contain ectopic pancreatic tissue. May be implicated in Littre's hernia.
26-Which of the following statements is true of upper limb nerve injuries? a. Injury to the median nerve results in a wrist drop b. Injury to the radial nerve results in loss of sensation over the palmar aspect of the index finger c. Injury to the median nerve results in loss of sensation in the anatomical snuffbox d. Injury to the ulnar nerve results in a claw hand e. Injury to the ulnar nerve results in loss of sensation over the thumb 27-Concerning the inguinal canal: a. b. c. d. e.
It transmits the ilio-inguinal nerve The deep inguinal ring lies below the mid-point of inguinal ligament The superficial inguinal ring overlies the pubic tubercle Laterally , the anterior wall is made up of the external oblique aponeurosis Laterally, the posterior wall is formed by the conjoint tendon
28-In chest trauma, urgent cardiothoracic surgical referral is necessary for all except: a. b. c. d. e.
Continuing massive air leak following insertion of chest drain Cardiac tamponade Disruption of the great vessels Severe pulmonary contusion Continuing haemorrhage following insertion of chest drain
29-A raised titre of anti-HBs in the bloodallare true except : a. b. c. d.
signifies previous hepatitis B infection is produced after hepatitis B vaccination indicates immunity to hepatitis infection indicates active hepatitis B infection
30-The following statements regarding intravenous solutions is correct: a. Normal saline contains 180mmol/l of sodium b. Ringer's lactate solutions is designed for intracellular fluid replacement c. Sodium bicarbonate 8.4% is a hyperosmolar solution
77
d. Normal saline with added potassium is appropriate therapy to correct a respiratory alkalosis 31-The following is true of the blood supply of the rectum all are true except: a. The principle blood supply is derived from the inferior rectal artery b. The median sacral vessels contribute to the blood supply c. As the inferior mesenteric artery crosses the pelvic brim it becomes known as the superior rectal artery d. The internal venous rectal plexus is continuous with the vascular cushions of the anal canal e. The superior rectal vein drains to the portal system 32-Which of the following concerning the Femoral sheath is false: a. b. c. d.
Contains the femoral artery Contains lymph nodes Contains the femoral canal Contains femoral nerve
33-The following is true of the spleen: a. b. c. d. e.
Is the largest lymphoid organ in the body Lies obliquely between the seventh and tenth rib The lower pole extends beyond the mid-axillary line Is usually palpable when normal Usually measures 16cm in maximum length when healthy
34-A knife stabbed horizontally through the fourth intercostals space to the right of the sternum is likely to damage the: a. b. c. d. e.
Right brachiocephalic veinHemiazygos vein Descending aorta Right pulmonary artery Thoracic duct
35-Breast cancer risk is increased in association with the following factors except: a. b. c. d.
78
Nulliparity Immediately after pregnancy Early menarche Early age at first pregnancy
e. Late menopause 36-The Appendix all true except: a. Is typically less than 10 cm in length in the adult. b. Is located in the retrocaecal recess. c. Macburneys point, lies 2/3 laterally from a line from umbilicus to the anterior superior iliac spine. d. The longitudinal coat of the appendix is derived from the three bands of taenia coli. e. Is supplied by branches of the inferior mesenteric artery 37-In tension pneumothorax the following signs are present except: a. b. c. d. e.
Hypoxia Hyperresonance to percussion on the affected side Tracheal deviation to the ipsilateral side Distended neck veins Tachycardia
38-The following are encountered during inguinal hernia repair : a. b. c. d. e.
Scarpas' fascia Colle's fascia Ilioinguinal nerve Cremaster muscle Genitofemoral nerve
39- Which of the following variables best predicts prognosis for patients with a recent diagnosis of cutaneous melanoma and no clinical evidence of metastatic disease? a. b. c. d. e.
Breslow thickness . Clark's level. Ulceration. Gender. Celtic complexion.
40- The intravenous fluid that a 60 kg., 30-year-old woman with an 80% burn should be given in the first 24 hours following burn injury is: a. b. c. d. e. f.
79
19.2 liters of 5% glucose in lactated Ringer's . 14.4 liters of lactated Ringer's. 9.6 liters of hypertonic salt solution (sodium concentration 200 mEq. per liter). 7.2 liters of 5% albumin solution. 5.5 liters of the pentafraction component of hydroxyethyl starch. DISCUSSION: The consensus range for estimating fluid needs of burn patients in the first 24 hours is 2 to 4 ml. of a physiologic crystalloid solution per kilogram body weight per percent of body surface burned. In this patient that would be 9.6 to 19.2 liters of lactated Ringer's solution. The early elevation of circulating levels
of catecholamines and glucocorticoids following burn injury induces glycogenolysis, which results in elevated circulating blood glucose levels. Glucose should not be administered in the resuscitation fluids, since the resulting exaggeration of hyperglycemia could induce osmotic diuresis. In the first 24 hours colloid-containing solution is not commonly used, but if it is used even the Evans formula recommends only 1 ml. per kg. body weight per percent of body surface burned. Moreover, patients in one study who received colloid-containing fluids continued to gain weight during the first 3 postinjury days, retained more sodium, and had less urine output than patients who received only crystalloid fluids in the first 24 hours. Hypertonic salt solution is also not commonly used for burn patient resuscitation because of the recently described and surprisingly frequent occurrence of acute renal failure and increased mortality associated with its use. If hypertonic salt is used, the amount infused should be less than 9 liters, to avoid excessive elevation of the serum sodium concentration (i.e., above 160 mEq. per liter). The recommended limit of hydroxyethyl starch infusion is currently 1500 ml. per day. Although a 10% pentastarch form of hydroxyethyl starch has been used to expand the plasma volume of burn patients at the end of the first 24 hours, even as little as 500 ml. of that solution has been reported to prolong both prothrombin and plasma thromboplastin time. 41-All venous sinuses of the dura mater ultimately drain into : a. b. c. d. e.
External jugular vein Internal jugular vein Emissary vein Caroted sinus Great cerebral vein
42--The tendon of biceps bronchii muscle inserted into the a. b. c. d. e.
Ulner tuberosity Coronoid tuberosity Radial tuberosity Olecranon process Biceptal groove
43--the saphenous nerve is terminal branch of the : a. b. c. d. e.
80
Ilioinguinal nerve Femoral nerve Genitofemoral nerve Superior glutial nerve popliteal nerve
44- All the following origenat on the scapula except a. b. c. d. e.
Trapezius Teres major Deltoid Brachioradialis Coracobrachialis
45- Total number of human vertebra is: a. b. c. d. e.
36 vertebras. 32 vertebras. 34 vertebras. 33 vertebras. 35 vertebras.
46-Foot droop is usually associated with paralysis of the: a. b. c. d. e.
Tibial nerve Common peroneal Obturater Medial planter Lateral planter
47--The following pass through the aortic hiatus of the diaphragm a. b. c. d. e.
Aorta Azygos vein Thoracic duct All of the above Non of the above
48-At birth the umbilical vein become the: a. b. c. d. e.
Ligamentum arteriosum Round ligament of the liver ligamentum teres hepatis Ligamentum venosum Falciform ligament Non of the above
49- Arrangement of structures at hilum of the right kidney as following from anterior to posterior: a. b. c. d. e.
Vein ,artery , ureter Artery, ureter Vein Artery , Vein , , ureter Ureter Artery , Vein Ureter, Vein, Artery,
50-Shock can best be defined as:
81
a. b. c. d.
Hypotension. Hypoperfusion of tissues. Hypoxemia. All of the above.
51-All of the following are true about neurogenic shock except: e. there is a decrease in systemic vascular resistance and an increase in venous capacitance. f. tachycardia or bradycardia may be observed, along with hypotension. g. the use of an alpha agonist such as phenylephrine is the mainstay of treatm ent. h. Severe head injury, spinal cord injury, and high spinal anesthesia may all cause neurogenic shock. 52-All of the following may be useful in the treatment of cardiogenic shock except: a. b. c. d.
Dobutamine. Sodium nitroprusside. Pneumatic antishock garment. Intra-aortic balloon pump.
53-All of the following statements about hemorrhagic shock are true except: a. Following hemorrhagic shock, there is an initial interstitial fluid volume contraction. b. Dopamine, or a similar inotropic agent, should be given immediately for resuscitation from hemorrhagic shock, to increase cardiac output and improve oxygen delivery to hypoperfused tissues. c. The use of colloid solutions or hypertonic saline solutions is indicated for treatment of hemorrhagic shock. d. In hemorrhagic shock, a narrowed pulse pressure is commonly seen before a fall in systolic blood pressure.
54-Which of the following statements about head injury and concomitant hyponatremia is true? a. b. c. d. e.
There are no primary alterations in cardiovascular signs. Signs of increased intracranial pressure may be masked by the hyponatremia. Oliguric renal failure is an unlikely complication. Rapid correction of the hyponatremia may prevent central pontine injury. This patient is best treated by restriction of water intake.
55-Which of the following statements about extracellular fluid is true? F. The total extracellular fluid volume represents 40% of the body weight. G. The plasma volume constitutes 20%of the total extracellular fluid volume .
82
H. Potassium is the principal cation in extracellular fluid. I. The protein content of the plasma produces a lower concentration of cations than in the interstitial fluid. J. The interstitial fluid equilibrates slowly with the other body compartments.
56-Which of the following statements is true of a patient with hyperglycemia and hyponatremia? a. The sodium concentration must be corrected by 5 mEq. per 100 mg. per 100 ml. elevation in blood glucose. b. With normal renal function, this patient is likely to be volume overloaded. c. Proper fluid therapy would be unlikely to include potassium administration. d. Insulin administration will increase the potassium content of cells . e. Early in treatment adequate urine output is a reliable measure of adequate volume resuscitation. 57-Which of the following is not associated with increased likelihood of infection after major elective surgery? F. G. H. I. J.
Age over 70 years. Chronic malnutrition. Controlled diabetes mellitus. Long-term steroid use. Infection at a remote body site.
58-The transfusion of fresh frozen plasma (FFP) is indicated for which of the following reasons? a. b. c. d.
Volume replacement. As a nutritional supplement. Treatment of prolonged bleeding time. For the correction of abnormal PT secondary to warfarin therapy, vitamin K deficiency, or liver disease.
59-In patients receiving massive blood transfusion for acute blood loss, which of the following is correct? a. Packed red blood cells and crystalloid solution should be infused to restore oxygen-carrying capacity and intravascular volume. b. Two units of FFP should be given with every 5 units of packed red blood cells in most cases. c. A “six pack” of platelets should be administered with every 10 units of packed red blood cells in most cases. d. One to two ampules of sodium bicarbonate should be administered with every 5 units of packed red blood cells to avoid acidosis.
60-Hemostasis and the cessation of bleeding require which of the following processes?
83
a. Adherence of platelets to exposed subendothelial glycoproteins and collagen with subsequent aggregation of platelets and formation of a hemostatic plug. b. Interaction of tissue factor with factor VII circulating in the plasma. c. The production of thrombin via the coagulation cascade with conversion of fibrinogen to fibrin. d. Cross-linking of fibrin by factor XIII. e. All of the above 61- Advantages of epidural analgesia include: a. b. c. d. e.
Earlier mobilization after surgery. Earlier return of bowel function. Shorter hospitalizations. Decreased stress response to surgery. All of the above.
62- Factors that decrease collagen synthesis include all of the following except: a. Protein depletion. b. Infection. c. Anemia. d. Advanced age. e. Hypoxia. f. The characteristic metabolic response to injury includes hypermetabolism, fever, accelerated gluconeogenesis, and increased proteolysis (creating a negative nitrogen balance). Food intake is generally impossible because of abdominal injury or ileus. With time, food intake increases, but the patient generally experiences anorexia, not hyperphagia. g. Cardiogenic shock occurs when the heart fails to generate adequate cardiac output to maintain tissue perfusion. Intrinsic causes such as myocardial dysfunction secondary to coronary artery disease, or extrinsic causes such as pulmonary embolism, tension pneumothorax, and pericardial tamponade, may produce cardiogenic shock. Principles of treatment of cardiogenic shock are aimed at optimizing preload, cardiac contractility, and afterload. Preload is usually adequate or high in cardiogenic shock. Dobutamine is a useful inotropic agent, particularly when filling pressures are high, because of its mild vasodilatory effect, as well as its effect to enhance cardiac contractility. Afterload-reducing agents, such as sodium nitroprusside, may be beneficial in cardiogenic shock in the setting of elevated filling pressures, low cardiac output, and elevated systemic vascular resistance. Cardiac output may improve with use of afterload-reducing agents by decreasing myocardial wall tension and optimizing the myocardial oxygen supply-demand ratio. The intra-aortic balloon pump (IABP), by providing diastolic augmentation, reducing left ventricular
84
afterload, and reducing myocardial oxygen consumption, is sometimes useful in the treatment of cardiogenic shock. The IABP is especially useful in low–cardiac output postcardiotomy patients, in patients awaiting revascularization, and in patients with acute myocardial infarction complicated by mitral insufficiency or ventricular septal defect. The pneumatic antishock garment (PASG), which causes an increase in systemic vascular resistance, is contraindicated in cardiogenic shock. h. The total extracellular fluid volume represents 20% of body weight. The plasma volume is approximately 5% of body weight. Sodium is the principal cation. The Gibbs-Donan equilibrium equation explains the higher total concentration of cations in plasma. Except for joint fluid and cerebrospinal fluid, the majority of the interstitial fluid exists as a rapidly equilibrating component. i. Each 100-mg. per 100 ml. elevation in blood glucose causes a fall in serum sodium concentration of approximately 2 mEq. per liter. Excess serum glucose acts as an osmotic diuretic, producing increased urine flow, which can lead to volume depletion. Insulin therapy and the correction of the patient's associated acidosis produce movement of potassium ions into the intracellular compartment. j. Clinical evidence of congestive heart failure in a patient with 8.5 gm. per dl. hemoglobin concentration is a misleading sign. Evidence of congestive failure is ordinarily a major risk factor, but in this particular patient the anemia lends itself to correction by preoperative transfusion with packed red blood cells, and often it is found that congestive failure and the associated increased risks disappear when the hemoglobin concentration is returned to the 12 gm. per dl. or higher ratio. All other factors are overt signs of increased likelihood of a postoperative cardiac event, the most ominous being a myocardial infarction 4 months preoperatively or the presence of a harsh aortic systolic murmur suggesting the presence of aortic stenosis. Age over 70 years and the presence of premature atrial or ventricular contractions on the electrocardiogram are less strong determinants of a postoperative cardiac complication. k. Patients who are suffering from acute blood loss require crystalloid resuscitation as the initial maneuver to restore intravascular volume and re-establish vital signs. If 2 to 3 liters of crystalloid solution is inadequate to restore intravascular volume status, packed red blood cells should be infused as soon as possible. There is no role for “prophylactic infusion” of FFP, platelets, bicarbonate, or calcium to patients receiving massive blood transfusion. If specific indications exist patients should receive these supplemental components. In particular, patients who have abnormal coagulation tests and have ongoing bleeding should receive FFP. Patients who have depressed platelet counts along
85
with clinical evidence of oozing (microvascular bleeding) benefit from platelet infusion. Sodium bicarbonate is not necessary, since most patients who receive blood transfusion ultimately develop alkalosis from the citrate contained in stored red blood cells. The use of calcium chloride is usually unnecessary unless the patient has depressed liver function, ongoing prolonged shock associated with hypothermia, or, rarely, when the infusion of blood proceeds at a rate exceeding 1 to 2 units every 5 minutes. l. The evaluation of most patients scheduled for elective surgery who do not have a history of significant bleeding disorders is somewhat controversial. An adequate history and physical examination screen out most patients with bleeding problems. For patients who are scheduled to undergo a major surgical procedure, it is advisable to obtain a CBC and platelet count, as well as a PT and APTT level. This detects a large number of bleeding disorders but does not rule out all possible causes of perioperative bleeding. Studies of platelet aggregation are indicated only for patients who are suspected of having qualitative defects of platelet function (e.g., von Willebrand's disease) m. he most common cause of fatalities related to transfusion reactions result from ABO-incompatible transfusion related to clerical error. Most such reactions occur if a type O person receives type A red cells owing to a clerical error that occurs either at the time the blood sample was drawn, during processing in the laboratory, or at the time a unit is administered. The importance of extremely careful labeling, transfer, and handling of specimens and of cross-matched blood products cannot be overemphasized. Allergic and other reactions are common but rarely fatal. The transmission of bacterial organisms (e.g., Staphylococcus aureus) has been reported especially with platelet concentrates maintained at or near room temperature. Fortunately, such reactions are rare n. The role of nutritional support in the cancer patient remains an important component of overall therapy. Preoperative nutritional support should be given only to those patients who do not require an emergency operation and who have severe weight loss (> 15% of pre-illness body weight) and a serum albumen < 2.9 mg%. Preoperative nutrition (enteral or parenteral) should not be given for longer than 7 to 10 days. Enteral nutrition is always the preferred route of feeding cancer patients if the GI tract is functional. There are several benefits of using the bowel lumen for nutrient delivery. The trophic effects of enteral feeding on small bowel mucosa have been well described. The integrity of the mucosal lining is maintained and it may provide an effective barrier to intraluminal enteric organisms which might otherwise translocate into the systemic circulation. Atrophic changes may be seen in the intestinal epithelium after several days of bowel rest;
86
this atrophy is not reversed by currently available total parenteral nutrition solutions. o. Epithelialization is more rapid under moist conditions than dry conditions. Without dressings, a superficial wound, or one with minimal devitalized tissue forms a scab or crust, meaning that the blood and serum will coagulate, dry, and form a protective moisture barrier over the open wound. If a wound is kept moist with an occlusive dressing, then epithelial migration is optimized. In addition, the pain of an open wound is dramatically reduced under an occlusive dressing. The traditional wet-to-dry dressing if truly left to dry, simply produces desiccation and necrosis of the surface layer of the wound which delays epithelialization. Although wet-to-dry dressings can be effective for debridement of wound exudate, they are generally less desirable than a moist healing environment combined with effective cleaning of the wound (i.e. water irrigation). Any open wound will leak plasma. With more inflammation, the plasma capillary permeability is further increased. This exudate of serum proteins and inflammatory cells serves as a rich culture medium. This, in turn, will continue to cycle bacterial proliferation and lead to further exudate formation. The net result of this cycle is delayed or absent wound healing. In addition, the edema that results from capillary dysfunction, increases the distance for diffusion from oxygen and nutrient sources to their metabolic targets. p. Several important systemic factors or conditions influence wound healing. Interestingly, there are no known systemic conditions that lead to enhanced or more rapid wound healing. Overall nutrition as well as adequate vitamins play an important role in wound healing. Vitamin A is involved in the stimulation of fibroplasia, collagen cross-linking, and epithelialization. Although there is no conclusive evidence in humans, vitamin A may be useful clinically for steroiddependent patients who have problematic wounds or who are undergoing extensive surgical procedures. Vitamin C is a necessary cofactor in hydroxylization of lysine and proline in collagen synthesis and cross-linkage. The utility of vitamin C supplementation in patients who otherwise take in a normal diet has not been established. Vitamin E is applied to wounds and incisions empirically by many patients. The evidence to support this practice is entirely anecdotal. In fact, large doses of vitamin E have been found to inhibit wound healing. Zinc and copper are also important cofactors for many enzyme systems that are important to wound healing. Deficiency states are seen with parenteral nutrition but are rare and readily recognized and treated with supplements. Overall, vitamin and mineral deficiency states are extremely rare in the absence of parenteral nutrition or other extreme dietary restrictions. There is no evidence to support the
87
concept that supranormal provision of these factors enhance wound healing in normal patients. q. True keloids are uncommon and occur predominantly in dark skinned people with a genetic predisposition for keloid formation. In most cases, the gene appears to be transmitted as an autosomal dominant pattern. The primary difference between a keloid and a hypertrophic scar is that a keloid extends beyond the boundary of the original tissue injury. It behaves as a tumor and extends into or invades the normal surrounding tissue creating a scar that is larger than the original wound. Histologically, keloids and hypertrophic scars are similar. Both contain an overabundance of collagen. Although the absolute number of fibroblasts is not increased, the production of collagen continually out paces the activity of collagenase, resulting in a scar of ever increasing dimensions. Hypertrophic scars respect the boundaries of the original injury and do not extend into normal unwounded tissue. There is less of a genetic predisposition, but hypertrophic scars also occur more frequently in Orientals and the Black population. They are often seen on the upper torso and across flexor surfaces. Some improvement in a keloid can be obtained with excision followed by intra-lesional steroid injection. However, the resulting scar is unpredictable and potentially worse. Reexcision and closure should, however, be considered for hypertrophic scars, if the condition of closure can be improved. This is especially pertinent for wounds that originally healed by secondary intention or that are complicated by infection. Keloids typically develop several months after the injury and rarely, if ever, subside. Hypertrophic scars usually develop within the first month after wounding and often subside gradually. r. After wounding, there is transient vasoconstriction mediated by catecholamines, thromboxane, and prostaglandin F2 (PGF2a). This period of vasoconstriction lasts for only five to ten minutes. Once a clot has been formed and active bleeding has stopped, vasodilatation occurs in an around the wound. Vasodilatation increases local blood flow to the wounded area, supplying the cells and substrate necessary for further wound repair. The vascular endothelial cells also deform, increasing vascular permeability. The vasodilatation and increased endothelial permeability is mediated by histamine, PGE2, and prostacyclin as well as growth factor VEGF (vascular endothelial cell growth factor). These vasodilatory substances are released by injured endothelial cells and mast cells and enhance the egress of cells and substrate into the wound and tissue. s. Fibrinolytic therapy induces a hemostatic defect through a combination of factors. Hypofibrinogenemia and fibrin degradation products inhibit fibrin polymerization and, in combination with a decrease in the clotting factors V and VIII, prolong the ability of
88
blood to clot. However, coagulation tests in general do not correlate well with bleeding complications. A level of fibrinogen less than 100 mg/dl is associated with an increased risk of bleeding. Absolute contraindications to thrombolytic therapy include active internal bleeding, recent (less than 2 months) cerebral vascular accident, and documented left heart thrombosis. Recent (less than 10 days) major surgery, obstetric delivery, organ biopsy, or major trauma is considered a major relative contraindication to either regional or systemic thrombolytic therapy. t. IL-1 and TNFa share many biologic properties. In addition, each potentiates the effects of the other one when given concurrently. Overall, IL-1 alone probably has weaker effects than TNFa with respect to the induction of shock; its role is likely to be important with respect to its marked potentiating abilities as it relates to TNFa. IL-1 expression is regulated by a host of factors including IL2, granulocyte macrophage colony stimulating factor (GM-CSF), transforming growth factor b (TGF-b), TNFa, all of the interferons, and IL-1 itself. Other endogenous stimuli for IL-1 production include antigen-antibody complex, the Fc region of IgG, and C5a; other nonspecific exogenous stimuli include silica particles and UV irradiation. One of the key proinflammatory features of IL-1-induced inflammation is the stimulation of arachadonic acid metabolism. IL1 stimulates the release of pituitary stress hormones and increases the synthesis of collagenases, resulting in the destruction of cartilage, bone and other collagen-rich structures. IL-1 stimulates prostaglandin production. One of the most important properties of IL-1 involves its interaction with the vascular endothelium. This includes the adherence of neutrophils, basophils, eosinophils, monocytes, and lymphocytes to the vascular endothelium via interaction between adhesion molecules on leukocytes and adhesion-receptor complex on the endothelial cells. By inducing the expression of ICAM-1, E-selectin, and VCAM-1 on endothelial cells, IL-1 provides a key step in the extravasation of leukocytes to sites of local inflammation and injury u. With the empiric use of antibiotics, a diligent search for the septic source should be undertaken and continued until identified v. Intravenous administration of an antibiotic is clearly indicated for patients undergoing clean contaminated operations. These antibiotics should be administered prior to surgery to obtain adequate tissue levels at the time of potential contamination. However, there has been no added benefit demonstrated for the postoperative use of antibiotics with regard to prophylaxis. The choice of antibiotic is a complex issue which remains unresolved largely because both superficial and deep wound infections can
89
occur as a result of either or both skin (superficial wound) flora (e.g., Staphylococcus aureus) and body site (deep wound) infection. For this reason, the administration of agents which possess activity directed against these expected agents is reasonable. Although administration of a first-generation cephalosporin is acceptable, second-generation cephalosporins or extended-spectrum penicillins with gram-positive and gramnegative activity and biliary tract excretion may be more suitable for patients undergoing gastrointestinal or biliary tract procedures. Similarly, the use of agents with additional anaerobic activity for patients undergoing gastrointestinal procedures involving the small bowel or colon should be considered. The administration of broadspectrum agents such as third-generation cephalosporins for prophylaxis does not seem to provide additional benefit in comparison to the above-mentioned type antibiotics and may foster the development of resistant organisms within a given institution or superinfection within a given patient. There is evidence that in some cases the topical use of antimicrobial agents is equivalent to the administration of intravenous antimicrobial agent antibiotics w. The relationships between oxygen consumption and oxygen delivery represent one of the most interesting regulation systems in homeostasis. First of all, if one of the three components of oxygen delivery is abnormal, endogenous mechanisms regulate the other two until normal oxygen delivery has been restored. The various combinations of compensatory mechanisms supply adequate oxygen for systemic metabolism through a wide range of variations in oxygen delivery. When there is a change in oxygen consumption, there is a proportionate change in oxygen delivery, which occurs almost immediately, mediated completely by a change in cardiac output. Conversely, a primary change in oxygen delivery is not followed by any change in oxygen consumption. The normal ratio of oxygen delivery to consumption is approximately 5:1. x. The causes of pulmonary edema are: 1) increased hydrostatic pressure; 2) increased capillary permeability and 3) decreased plasma oncotic pressure. The latter, however, is rarely a problem unless the concentration of plasma protein is very low. When fluid begins to collect in the lung interstitium, it migrates to the loose areolar portion of the lung microanatomy that surround the small bronchioles and pulmonary arteries. The edema in these areas has the effect of narrowing bronchi and increasing resistance in the pulmonary vasculature. This will decrease both ventilation and perfusion in the edematous area, but ventilation is often affected more than blood flow, resulting in a decreased / ratio, with all of its attendant effects on gas exchange. Ventilator treatment of pulmonary edema which increases airway pressure tends to hold
90
the alveoli open, spreading out the space available for water accumulation and overcomes the effect of small bronchial occlusion. Positive pressure ventilation does not, therefore, affect the amount of edema in the lung, only its manifestations. 63-Which of the following statement is true concerning excessive scarring processes? a. b. c. d.
Keloids occur randomly regardless of gender or race Hypertrophic scars and keloid are histologically different Keloids tend to develop early and hypertrophic scars late after the surgical injury Simple reexcision and closure of a hypertrophic scar can be useful in certain situations such as a wound closed by secondary intention
64-Which of the following statement is true concerning the vascular response to injury? a. Vasoconstriction is an early event in the response to injury b. Vasodilatation is a detrimental response to injury with normal body processes working to avoid this process c. Vascular permeability is maintained to prevent further cellular injury d. Histamine, prostaglandin E2 (PGE2) and prostacyclin (PGI2) are important mediators of local vasoconstriction
65-=All are true ligamentous attachments of liver except a. b. c. d.
Falciform ligament Coronary Ligaments Hepatoduodenal ligaments Glisson's capsul
66-Which of the following statements about extracellular fluid are true? a. b. c. d.
The total extracellular fluid volume represents 40% of the body weight. The plasma volume constitutes one fourth of the total extracellular fluid volume . Potassium is the principal cation in extracellular fluid. The protein content of the plasma produces a lower concentration of cations than in the interstitial fluid. e. The interstitial fluid equilibrates slowly with the other body compartments. 67- Nasotracheal intubation a. b. c. d.
Is preferred for the unconscious patient without cervical spine injury. Is preferred for patients with suspected cervical spine injury. Maximizes neck manipulation. Is contraindicated in the patient who is breathing spontaneously.
68-. The radiographic findings indicating a torn thoracic aorta include all except : e. Widened mediastinum.
91
f. Presence of an apical “pleural cap.” g. Tracheal deviation to the right. h. riht hemothorax. 69-Which of the following steps is not a part of the primary survey in a trauma patient? a. b. c. d.
Insuring adequate ventilatory support Measurement of blood pressure and pulse Neurologic evaluation with the Glasgow Coma Scale Examination of the cervical spine
70-Which of the following statement(s) is/are true concerning the Advanced Trauma Life Support (ATLS) classification system of hemorrhagic shock? a. Class I shock is equivalent to voluntary blood donation b. In Class II shock there will be evidence of change in vital signs with tachycardia, tachypnea and a significant decrease in systolic blood pressure c. Class IV hemorrhage can usually be managed by simple administration of crystalloid solution d. Class III hemorrhage involves loss of over 40% of blood volume loss and can be classified as life-threatening 71- The clinical picture of gallstone ileus includes all except ? a. b. c. d. e.
Air in the biliary tree. Small bowel obstruction. A stone at the site of obstruction. Acholic stools. Associated bouts of cholangitis.
72- The evaluation of a comatose patient with a head injury begins with: a. b. c. d.
The cardiovascular system. Pupillary reflexes. Establishment of an airway. Computed tomography (CT) of the brain.
73. Which of the following signs does Horner's syndrome include? a. b. c. d.
Ptosis. Facial hyperhidrosis. Exophthalmos. Mydriasis.
74-. The goals of proper fracture reduction include all except? a. Providing patient comfort and analgesia. b. Allowing for restoration of length of the extremity. c. Correcting angular deformity and rotation.
92
d. Enabling immediate motion of all fractured extremitie 75-. The neurovascular structure most commonly injured as a result of an anterior dislocation of the shoulder is the: a. b. c. d.
Musculocutaneous nerve. Axillary nerve. Axillary artery. Median nerve.
76-. The radial nerve is at greatest risk for injury with which fracture? a. b. c. d.
Fracture of the surgical neck of the humerus. Fracture of the shaft of the humerus . Supracondylar fracture of the humerus. Olecranon fractures.
77- The most consistent sign of a fracture of the carpal scaphoid is: a. b. c. d.
Wrist pain during attempted push-ups. Diffuse swelling on the dorsum of the wrist. Localized tenderness in the anatomic snuffbox . Wrist popping on movement.
78. Hematogenous osteomyelitis most frequently affects: a. b. c. d. e.
The diaphysis of long bones. The epiphysis. The metaphysis of long bones. in 80% of the cases it occurs in children Flat bones. Cuboidal bones.
79- The most common hernia in females is: a. b. c. d. e.
Femoral hernia. Direct inguinal hernia. Indirect inguinal hernia. Obturator hernia. Umbilical hernia.
80- Which of the following most often initiates the development of acute appendicitis? a. b. c. d.
A viral infection. Acute gastroenteritis. Obstruction of the appendiceal lumen. A primary clostridial infection.
81-. The most helpful diagnostic radiographic procedure in small bowel obstruction is: a. CT of the abdomen.
93
b. Contrast study of the intestine. c. Supine and erect x-rays of the abdomen . d. Ultrasonography of the abdomen. 82-. Which of the following treatments should never be recommended to a patient with purely intraductal carcinoma? a. b. c. d.
Modified radical mastectomy. Lumpectomy to clear surgical margins, followed by observation. Incisional biopsy with an involved margin, followed by radiation . Excisional biopsy to clear margins, followed by radiation.
83- In role of nine extent of burn if entire trunk is burned it will be equal to: a. b. c. d. e.
9% body surface area. 18% body surface area. 36% body surface area. 27% body surface area. 45% body surface area.
84-. Which of the following does not describe intermittent claudication? a. b. c. d.
Is elicited by reproducible amount of exercise. Abates promptly with rest. Is often worse at night. May be an indication for bypass surgery.
85- Which of the following statements are true? a. b. c. d. e.
All arterial injuries are associated with absence of a palpable pulse. Preoperative arteriography is required to diagnose an arterial injury. The presence of Doppler signals indicates that an arterial injury has not occurred. Patients with critical limb ischemia have paralysis and paresthesias . In all patients with multiple trauma, arterial injuries should be repaired before other injuries are addressed.
86-. The appropriate surgical treatment for suspected carcinoma of the testis is: a. b. c. d.
Transscrotal percutaneous biopsy. Transscrotal open biopsy. Repeated examinations. Inguinal exploration, control of the spermatic cord, biopsy, and radical orchectomy if tumor is confirmed.
87-. If torsion of the testicle is suspected, surgical exploration: a. Can be delayed 24 hours and limited to the affected side. b. Can be delayed but should include the asymptomatic side. c. Should be immediate and limited to the affected side.
94
d. Should be immediate and include the asymptomatic side. 88- A patient with acute urinary tract infection (UTI) usually presents with: a. b. c. d.
Chills and fever. Flank pain. Nausea and vomiting. 5 to 10 white blood cells per high-power field (hpf) in the uncentrifuged urine specimen. e. Painful urination. Upper UTI (more common flank pain ' fever ) 89-. The most precise diagnostic screening procedure for differentiating benign thyroid nodules from malignant ones is: a. b. c. d.
Thyroid ultrasonography. Thyroid scintiscan. Fine-needle-aspiration biopsy (FNAB). Thyroid hormone suppression.
90- Hyperthyroidism can be caused by all of the following except: a. b. c. d. e.
Graves' disease. Plummer's disease. Struma ovarii. Hashimoto's disease. Medullary carcinoma of the thyroid.
Choose only the ONE most Appropriate Answer 6.Which of the following is not true about neurogenic shock? a. It may follow spinal fractures. b. There is tachycardia. c. The extremities are warm. d. There is bradycardia and hypotension. e. Vasopressors may be useful. 7.All the following are complications of massive blood transfusion except: a. Hypothermia. b. Hypocalcaemia. c. Hypokalaemia. d. Acidosis. e. DIC. 8.In idiopathic thrombocytopenic purpura all the following are present except: a. Megakaryocytes are present in the bone marrow. b. Cortisone therapy may improve the condition. c. Marked enlargement of the spleen is present. d. Haemarthrosis is not a complication.
95
e. f.
The INR is normal. The disease is due to the presence of immunoglobulins.
9.Regarding hyperkalemia all are true EXCEPT: a. Is mostly the result of renal failure. b. Is common with massive tissue destruction. c. Muscle rigidity is a manifestation of severe hyperkalemia . d. Causes a peaked T-wave on the electrocardiogram. e. Urgent treatment is by Ca gluconate and Na bicarbonate. 10.
One of the following is an anaerobic organism: a. Staphylococcus aureus. b. E. coli. c. Klebsiella. d. Pseudomonas aeroginosa. e. Bacteriodes.
11.
Which of the following tests will be affected after intake of non-steroidal antiinflammatory drugs: a. Coagulation time. b. PTT. c. INR. d. Bleeding time. e. Thrombin time. All of the following can be treated conservatively in a stable trauma patient except: a. Lung contusion. b. Liver laceration. c. Kidney laceration. d. Splenic hematoma. e. Perforation of the small intestine. The initial maneuver to establish an airway in a patient with multiple injuries is: a. Oropharyngeal airway. b. Uncuffed endo-tracheal tube. c. Suctioning foreign debris and lifting up the mandible. d. Cuffed endo-tracheal tube. e. Tracheostomy. Which is the most commonly injured intra-abdominal organ in blunt trauma? a. Pancreas. b. Kidney. c. Spleen. d. Stomach. e. Colon.
12.
13.
14.
15.
In malignant melanoma which of the following correlates best with the prognosis? a. Age of the patient. b. Gender. c. Clark’s level. d. Site of the lesion. e. Breslew thickness of the lesion.
16.
Risk factors for breast cancer include the following except:
96
a. b. c. d. e.
Prior breast cancer. Mammary duct ectasia. Breast cancer has affected a sister. Early menarche and late menopause. In situ lobular or duct carcinoma.
17.
The commonest histological type of breast cancer is: a. Duct carcinoma in situ. b. Lobular carcinoma in situ. c. Invasive duct carcinoma. d. Invasive lobular carcinoma. e. Paget’s disease of nipple.
18.
What is the commonest presentation of a nephroblastoma? a. Abdominal pain. b. Haematuria. c. Fever. d. Abdominal mass. e. Loss of weight.
19.
All the following statements about follicular thyroid cancer are true except: a. It present at a later age than papillary carcinoma. b. It disseminates via hematogenous route. c. It is less common than papillary carcinoma. d. It is frequently multicentric. e. Bone is a favorite metastatic site.
20.
To prevent foot infection in a diabetic patient the following recommendations are all correct except: a. Careful trimming of toe nails. b. Monthly injection of the long-acting benzathene penicillin. c. Regular washing and daily inspection of the feet by the patient. d. Early treatment of taenia pedis infection. e. Avoidance of walking bare-footed.
21.
Which of the following tests is used to monitor the adequacy of anticoagulation with warfarin? a. Bleeding time. b. Clotting time. c. Activated partial thromboplastin time (APTT). d. Fibrinogen level. e. International normalized ratio (INR). f. Marjolin’s ulcer.
22.
Severe limb pain of sudden onset can be caused by all the following conditions except: a. Acute ischaemia. b. Deep venous thrombosis. c. Muscle tear. d. Sciatica. e. Bone fracture.
97
23.
The following statements about gall bladder stones are all correct except: a. Most of these stones are radio-opaque. b. In many cases these stones produce no symptoms. c. Gall stones are present in most cases of gall bladder cancer. d. Ultrasonography is the preferred imaging study. e. Removal of the gall balder is the standard treatment of symptomatic cases.
24.
The following statements about acute cholecystitis are all correct except: a. The great majority of cases are caused by stones. b. The condition is more serious in diabetics. c. Differential diagnosis includes acute pancreatitis. d. The main presentation is jaundice. e. Initial treatment may be conservative.
25.
Which of the following is a common cause for massive lower gastrointestinal bleeding? a. Crohn’s colitis. b. Rectal polyp. c. Angiodysplasia of the colon. d. Carcinoma of the caecum. e. Solitary rectal ulcer.
26.
Which of the following is not true about achalasia of the cardia? a. It usually occurs in middle age. b. There are hyyperperistlatic waves in the body of the esophagus . c. It predisposes to squamous cell carcinoma of the esophagus. d. Manometric study is diagnostic. e. Pneumatic dilatation is a line of treatment. True regarding Achalasia Cardia is all of the following except
27. 28.
A Dysphagia B Odynophagia C Regurgitation D Premalignant
29.
Organisms found in the large bowel are mostly: a. Gram-positive cocci and anaerobes. b. Gram-negative cocci and anaerobes. c. Gram-negative bacilli and anaerobes. d. Gram-positive bacilli and anaerobes. e. Gram-negative cocci only.
30.
Which of the following operations is considered as clean contaminated operation? a. Herniorrhophy. b. Axillary block dissection. c. Ureterolithotomy. d. Urgent colectomy. e. Appendicular abscess drainage. o
98
kinds of surgical operations class I : clean surgery no inflammation or infection encountered
non-traumatic wound : trauma cause retention, hematoma, and tissue necrosis, an ideal pabulum for germ proliferation no breaks in sterile technique no drainage : in clean surgery blockage of secretion drainage is the leading cause of sepsis no opening of secretory channels potentially causing sepsis (respiratory, alimentary (gastrointestinal, biliary or oropharyngeal) and urogenital tracts : the urogenital tract can normally be sterile)
Wound infection rates without prophylaxis are usually < 2%. Airborne, exogenous micro-organisms are usually involved. Prophylaxis should be given only when: (a) infection would have catastrophic results (e.g. heart valve or joint replacements); or (b) a risk index or some other information indicates an increased probability of postoperative wound infection. Examples of clean surgical procedures include, but are not limited to, the following: adrenalectomy op.muscle, fascia, tendons cardiac orchiopexy cesarean section-elective orthopedics (reconstructive) ear surgery pancreatic surgery eye surgery plastic surgery herniorrhaphy splenectomy laparotomy (bowel not entered) thyroid/parathyroid surgery mastectomy (uninflammed) tubal/ovarian neurosurgery vascular surgery
99
Placement : patients in this category may room with each other may be placed with patients at high risk for acquiring infection and other types of non-surgical patients. A post-op hernia patient could be placed with a brittle diabetic may be placed with clean-contaminated (Class II) patients if necessary. For example: A patient s/p Lap Chole could be placed with a mastectomy patient do not place with contaminated or dirty surgery cases class II : clean-contaminated surgery : clean, but endogenous flora is involved gastrointestinal, genitourinary, and/or respiratory tracts entered under controlled conditions; no significant spillage or unusual contamination occurs. a non-traumatic wound; no inflammation encountered. a minor break in sterile technique occurred re-operation of clean surgery within 7 days procedures following blunt trauma
Postoperative infection rates without prophylaxis range from 5 to 25%, and endogenous flora are commonly involved Examples of clean-contaminated surgical procedures: appendectomy (incidental) -no inflammation or pus seen hysterectomy or hysterotomy bowel resection -(no spillage into peritoneal cavity) lung surgery - (no infection seen) bladder and ureteral surgery nose/throat/laryngectomy cholecystectomy /lap chole (elective) suprapubic prostatectomy cesarean section-after trial labor vaginal hysterectomy vag delivery-PROM< 24hr. hemorrhoidectomy therapeutic abortion dental surgery gastrectomy D and C TraCh TURP
Placement : patients in this category may be placed together. may be placed with patients at high risk for acquiring infection and other types of non-surgical patients. may be placed with clean surgery patients. do not place with contaminated or dirty patients. class III : contaminated surgery : has increasing chance for postoperative infectious complications. The operatory field contains potentially septic tissues (expecially due to the basal disease for which the patient undergoes surgery). Anaerobic bacteria (Bacteroides spp., Clostridium spp., ...). acute nonpurulent inflammation seen operations with major breaks in sterile techniques. gross spillage from GI tract occurs (gastrointestinal content or bile) the GU or biliary tracts are entered in the presence of infected bile or urine traumatic wound < 4-8 hours old from a relatively clean source. Infection rates without prophylaxis range from 15 to 40%. Examples of Class III surgical procedures include, but are not limited to: appendicectomy in acute appendicitis opening of biliary tract in acute cholecystitis incision and drainage of abscess gun shot wound (fresh, without perforated bowel) rectal surgery TUR or suprapubic prostatectomy with positive pre-op urine culture. vaginal or cesarean section deliveries with PROM 24 hours. Placement : patients in this category may be placed together. Prevention : mechanical cleaning, washing, antibiotic therapy
100
class IV : dirty surgery : existing acute bacterial infection or a perforated viscera is encountered (clean tissue is transected to gain access to pus = higher bacterial load). traumatic wound-old (4 < hours < 24) with retained devitalized tissue fecal contamination, foreign body or retained, devitalized tissue is present. Postoperative infection rates in this group exceed 40%. The use of antimicrobials in dirty and some contaminated procedures is not classified as prophylaxis but as treatment for a presumed infection. Examples include but are not limited to : incision and drainage of abscess or debridement of wound reduction of compound fractures visceral perforation ruptured appendix amputation of gangrenous extremity exploratory lap with peritonitis found penetrating trauma Placement : patients in this category may be placed together, unless otherwise indicated as in the disease-specific precautions table. do not place with surgical patients in any other category. do not place with immunocompromised patients. may place with non-surgical patients who are not immunocompromised, if there is no drainage. Prevention : antibiotics after surgery, reduce the time lapse between onset of inflammation and surgery
31.
32.
During conservative treatment of a patient with adhesive intestinal obstruction, which of the following is an indication to proceed to surgery? a. Nasogastric suction of more than 2 L/24 hours. b. Exaggerated bowel sounds. c. Abdominal rigidity. d. Abdominal distension. e. Serum K level below 2.5 mEq/L. The most important prognostic index in breast cancer is: a. Age of the patient. b. Site of the lesion. c. Presence of lymph node metastases. d. Level of CEA. e. Presence of micro-calcification.
11. Which of the following are not found in peritonitis? f. Patient is lying still g. Guarding h. Rebound tenderness i. Hyperactive bowel sounds j. Rigid abdomen
101
,The following is an indication for thoracotomy in chest injury .28 a. b. c. d. e.
Cardiac tamponade Uncontrolled pulmonary air leakage Perforation of thoracic esophagus Blood loss of 200ml/hr for 2-3 hrs via chest tube All of the above
54- Gallstones: (all correct except one) A. Have an incidence which increases with age. B. Are more frequent in females. C. Usually contain a predominance of cholesterol. D. Are formed in bile which is supersaturated with bile acids. * E. Are formed in bile which is supersaturated with cholesterol. 56- stones in the common bile duct: A. Are present in nearly 50 per cent of cases of cholecystitis. B. Often give rise to jaundice, fever and biliary colic. * C. Are usually accompanied by progressive jaundice. D. Are usually associated with a distended gallbladder. E. A&D only. A 40-year-old woman presents to the office for evaluation of yellowish skin. She states that over the past few weeks, she has noticed that her eyes and skin have developed a yellow tint. She also reports that she has dark urine and pale-colored stools. Further history elicits periodic bouts of right upper quadrant pain after eating. She is otherwise healthy. She denies using any medications. On physical examination, a yellowish tint is observed on the patient's skin, sclera, and mucous membranes. On the basis of this patient's history and clinical examination, which type of bilirubin would you expect to predominate? Choose one answer. a. Mixed b. Conjugated c. Unconjugated d. Indirect
39. A 56-year-old woman has been experiencing abdominal pain for 4 hours. The pain is in right upper quadrant and radiates into the scapular region. She has had multiple episodes of vomiting. For this patient, which of the following signs on physical examination is associated with acute cholecystitis? Choose one answer. a. Carnett sign
102
b. Kehr sign c. Murphy sign d. Rovsing sign Management of cholangitis may include all of the following EXCEPT: Choose one answer. a. decomperession of the common bile duct. b. cholecystostomy c. IV antibiotics. d. correct underlying cause. e. percutaneous transhepatic cholangiography. A 48-year-old woman presents to the emergency department complaining of right upper quadrant pain, which began 4 hours ago. She reports the pain as being spasmodic and sharp and that it radiates to her right shoulder blade. She says that she has had similar episodes over the past few months, especially after eating large meals. Associated with the pain is nausea and vomiting. Her blood pressure is 120/85 mm Hg, and her pulse is 100 beats/min. On physical examination, the patient is found to have a nontender abdomen with no palpable masses. Her chest and cardiovascular examinations are normal. The nurse notices that her sclerae are slightly icteric. On subsequent laboratory studies, her serum bilirubin level is found to be 10 mg/dl. What imaging study should be performed next for this patient with presumed posthepatic jaundice? Choose one answer. a. Percutaneous transhepatic cholangiography (PTC) b. Ultrasonography c. Endoscopic retrograde cholangiopancreatography (ERCP) d. Magnetic resonance imaging Gallstones are characterized by all the following EXCEPT: Choose one answer. a. cause mucocoele of the gall bladder b. are present in the common bile duct in 40% of patients with stones in the gall bladder c. are becoming common in post-partum primipara who were pre pregnancy ‘Pill’ takers d. are frequently the cause of flatulent dyspepsia e. may be present in the newborn found in 6 to 12% of patients with stones in the gallbladder. In the treatment of acute cholecystitis, most patient are BEST served with: Choose one answer. a. endoscopic sphincterotomy. b. cholecystostomy. c. percutaneous drainage of the gallbladder. d. early cholecystectomy (within 3 days of onset of symptoms). e. IV antibiotics and cholecystectomy in 6 to 8 weeks.
103
Advantages of laparoscopic versus open cholecystectomy include all of the following EXCEPT: Choose one answer. a. decreased pain. b. less risk of bile duct injury. c. reduced hospitalization. d. improved cosmetic. e. reduced ileus. Carcinoma of the gallbladder is Choose one answer. a. rarely associated with jaundice. b. associated with a good prognosis. c. usually not diagnosed preoperatively. d. most commonly metastatic to the lung. e. best treated with radiation and chemotherapy
1. Which of the following do you consider to be the most important clinical sign in acute appendicitis, 1. 2. 3. 4. 5.
Abdominal tenderness around the umbilicus Abdominal tenderness in the RIF Tenderness over McBurney’s point Rovsing’s sign positive Suprapubic tenderness
2. All of the following are mechanisms of urinary calculi formation except, 1. 2. 3. 4. 5.
Hypoparathyroidism Prolonged recumbency Infection with urea-splitting organisms Foreign body Urinary tract obstruction
3. Organisms most commonly isolated in UTIs, 1. 2. 3. 4. 5.
Kleb E. Coli Proteus Acinetobacter All of the above
4-The ideal fluid therapy in a patient with pyloric stenosis and repeated vomiting is: a. Isotonic crystaloid containing sodium chloride b. Hypertonic crystaloid containing dextrose – saline
104
c. Isotonic solution containing dextrose d. Large molecular weight colloid containing dextran 5. Excessive saliva in a newborn is due to 1. 2. 3. 4. 5.
Esophageal atresia Salivary gland tumour Primary Hypertrophic Pyloric Stenosis Midgut volvulus Hirschprung’s disease
6. The evaluation of a comatose patient with a head injury begins with: a.The cardiovascular system. b. Pupillary reflexes. c. Establishment of an airway. d. Computed tomography (CT) of the brain e .insertion of Intravenous canula 7. The following is an indication for thoracotomy in chest injury, f. g. h. i. j.
Cardiac tamponade Uncontrolled pulmonary air leakage Perforation of thoracic esophagus Blood loss of 200ml/hr for 2-3 hrs via chest tube All of the above
8. In an upper gastrointestinal bleed, in some cases a Sengstaken-Blakemore tube is incerted. What is the primary purpose of the tube? a. b. c. d. e.
Aspirate blood from the stomach Tube feeding Tamponade for varices To decompress bowel All of the above
9. Which of the following is not found in peritonitis? a. b. c. d. e.
105
Patient is lying still Guarding Rebound tenderness Hyperactive bowel sounds Rigid abdomen
10. Commonest site for Cancer breast a. b. c. d. e.
Upper outer quadrant Upper inner quadrant Lower outer quadrant Lower inner quadrant None of the above
11. The back of the medial epicondyle is related to the: a. Radial nerve. b. Axillary nerve. c. Ulnar nerve. d. Median nerve. e. None of the above. 12. Regarding Gallstones all of the following are true except:a. b. c. d.
Prevalence increases with advancing age 30% of gallstones are radio-opaque Cholesterol stones result from a change in solubility of bile constituents Biliary infection, stasis and changes in gallbladder function can precipitate stone formation e. Gram-negative organisms are the most common isolated 13.Clinical features of limb ischaemia includes a. b. c. d. e.
Paraesthesia Pallor Pulselessness Paralysis All of the above
14. Regarding Hydatid disease all of the following are true except:a. b. c. d. e.
106
Due to infection with the helminth Ecchinococcus granulosa Man is an accidental intermediate host Lunges is the commonest organ involved Diagnosis can be confirmed by indirect haemagglutinin assay Aspiration should not be performed if hydatid disease is suspected
15.Which hypersensitivity reaction is associated with serum sickness? a. Type I: immediate b. Type II: cytotoxic c. Type III: immune complex d. Type IV: cell mediated e. None of the above 16. The most common cause of mass in the posterior mediastinum is a. Vascular b. Esophageal diverticula c. Neurogenic tumors d. Lymphomas e. Bronchogenic cysts 17.In obstructive jaundice: a. Urinary conjugated bilirubin is increased b. Serum unconjugated bilirubin is increased c. Urinary urobilinogen increased d. Serum conjugated bilirubin is reduced e. Faecal stercobilinogen is increased 18-Which of the following statements about extracellular fluid is true? K. The total extracellular fluid volume represents 40% of the body weight. L. The plasma volume constitutes 20%of the total extracellular fluid volume . M. Potassium is the principal cation in extracellular fluid. N. The protein content of the plasma produces a lower concentration of cations than in the interstitial fluid. O. The interstitial fluid equilibrates slowly with the other body compartments. 19. The following cause hypercalcaemia except : a. Sarcoidosis b. Primary hyperparathyroidism c. Acute pancreatitis d. Metastatic bronchial carcinoma e. Milk-Alkali syndrome 20.For a 22-kg baby the maintenance daily fluid requirement is approximately which of the following? a. 1100 ml b. 1250 ml c. 1550 ml d. 1700 ml e. 1850 ml 21.The transfusion of fresh frozen plasma (FFP) is indicated for which of the following reasons? a. Volume replacement. b. As a nutritional supplement. c. Treatment of prolonged bleeding time.
107
d. For the correction of abnormal PT secondary to warfarin therapy, vitamin K deficiency, or liver disease. e. All are correct 22.A 9 month old boy presents with an acute scrotal swelling. The following diagnoses are likely: a. b. c. d. e.
Epididymitis Orchitis Torsion of the testicular appendage Irreducible inguinal hernia Acute idiopathic scrotal oedema
23- A 24-year-old man has sustained significant brain injury following an assault. He is showing signs of a raised intra-cranial pressure. Which of the cranial nerves is usally the first to be affected by a raised intracranial pressure? a. The abducent nerve b. The facial nerve c. The hypoglossal nerve d. The vagus nerve e. The vestibulocochlear nerve 24. Anaphylactic shock:xx a. Is an immune-mediated reaction. b. Results in mast cell activation and increased circulating histamine concentrations. c. Produces microcirculatory changes similar to hypovolaemic shock. d. Requires prompt treatment with parenteral adrenaline and hydrocortisone. e. May occur after ingestion of drugs 25. A 33-year-old woman develops a reducible mass of the groin that is inferolateral to the pubic tubercle and medial to the femoral vein. Which of the following is the most likely diagnosis? a. Direct inguinal hernia b. Femoral hernia c. Incisional hernia d. Indirect inguinal hernia e. Umbilical hernia 26- Regarding papillary thyroid cancer a. Account for 40% of thyroid cancer
108
b. Early blood spread c. Commonest thyroid cancer in children d. Common in endemic goiter areas e. Multifocality is rare 27- The most commonly used imaging method for diagnosis of acute cholecystitis is: a. CT of the abdomen. b. Ultrasonography of the gallbladder. c. Oral cholecystogram. d. Radionuclide (HIDA) scan of the gallbladder e. MRI 28. Fibro adenoma a. Is common in females between 15-25 years b. Usually soft indiscreet lump c. Malignant changes in 5 % d. Arise from duct of breast e. none of the above 29-In tension pneumothorax the following signs are present except: f. g. h. i. j.
Hypoxia Hyperresonance to percussion on the affected side Tracheal deviation to the ipsilateral side Distended neck veins Tachycardia
30-Foot droop is usually associated with paralysis of the: f. g. h. i. j.
Tibial nerve Common peroneal Obturater Medial planter Lateral planter
PMC Surgery Exam 2009
109
Paper 1
1. Low molecular weight heparin (LMWH) produces its primary effects because of its inhibition of which factor? A. Ia B. IXa C. Xa D. XIa E. XIIa 2. Which of the following statements about extracellular fluid is true? A.The total extracellular fluid volume represents 40% of the body weight. B.The plasma volume constitutes 20%of the total extracellular fluid volume . C.Potassium is the principal cation in extracellular fluid. D.The protein content of the plasma produces a lower concentration of cations than in the interstitial fluid. E.The interstitial fluid equilibrates slowly with the other body compartments. 3. Which of these is a characteristic common to both meiosis and mitosis? A. final daughter cells produced have half the number of chromosomes B. homolog synapse and crossover recombination create genetic variability C. original parent cells undergo a single round of cytokinesis D. synthesis of new DNA only occurs once during the entire process E. none of the above 4. A 35-year-old trauma victim with malabsorption requires hyperalimentation. The patient’s injuries include a stable nondisplaced fracture of the third thoracic vertebra, a closed head injury, multiple upper and lower extremity fractures and bilateral pulmonary contusions requiring ventilatory support. Which of the following are the most appropriate site and type of venous access in this patient? A. bilateral antecubital fossae, 18G peripheral intravenous catheters B. femoral vein, central venous catheter (CVC) C. dorsum of one foot, single 16G peripheral intravenous catheter D. subclavian vein, CVC 5. A 63-year-old male with end-stage renal disease requiring hemodialysis three times per week presents with bone pain and several pathologic fractures of the extremities. Which is the most likely electrolyte abnormality in this patient? A. hypokalemia B. hypernatremia C. hyperphosphatemia D. hypercalcemia E. hypochloremia 6. An 18-year-old male undergoes extensive small bowel resection after a gunshot wound to the abdomen. Despite his resultant short gut, his remaining colonic function allows him to subsist on an oral high-carbo hydrate diet. What substrate is the preferred fuel for colonocytes? A. short chain fatty acids (SCFAs) B. glutamate C. luminal oligopeptides D. ketones E. fructose 7. The greatest burst strength of the wound will be achieved by: A. 1 week B. 3 weeks
110
C. 6 weeks D. 12 weeks E. 6 months 8. With regards to cytokines, which of the following alternatives is correct? A. IL- is secreted mainly by lymphocytes and mediates inflammation. B. IL- 10 involved in cell division and activation. C. IL-8 is secreted by macrophages and promotes chemotaxis. D. IL-2 is a major inhibitor of cell division. E. TNF-a is produced by T cells and is associated with a rise of immature neutrophils in the blood circulation. 9. Which of the following will immediately delay or cancel an elective surgical case if not obtained appropriately preoperatively? A. CBC B. urinalysis C. CXR D. informed consent E. ECG 10. Preoperative bowel preparation helps reduce which of the following postoperative complications by 40%? A. pneumonia B. urinary tract infection (UTI) C. dehiscence D. wound infection E. intraabdominal abscess 11. Which one of the following substances is not known to directly activate platelets during the process of clot formation? A. thrombin B. adenosine triphosphate (ATP) C. epinephrine D. collagen E. interleukin (TL)-6 12-. The most helpful diagnostic radiographic procedure in small bowel obstruction A.CT of the abdomen. B.Contrast study of the intestine. C.Supine and erect x-rays of the abdomen. D.Ultrasonography of the abdomen 13.Which of the following lab values correspond with acute DIC? A. slowly rising platelet count B. selective deficiency of vitamin K factors C. hypofibrigonemia D. prolonged bleeding time E. presence of fibrin split products )FDP) 14. Factor V Leiden (FVL): A. is the most common inheritable disorder of coagulation B. is associated with a defect in factor S activity C. is caused by a mutation in the gene for factor C D. occurs in approximately 10% of people in the western population E. results in slightly less than two-fold increase in risk thromboembolism 15.Which of the following regarding the anatomy of the heart is true?
111
is:
A.The aortic valve is tricuspid. B.The ascending aorta is entirely outside the pericardial sac. C.The left atrial appendage is identified readily by transthoracic echocardiography. D.The pulmonary trunk lies anterior to the ascending aorta. E.The right atrium is posterior to the left atrium. 16. The cell type most characteristic of chronic inflammation is the: A.macrophage B. B cell C. natural killer cell D. neutrophil E. eosinophil 17. The coagulation cascade is activated as part of the acute inflammatory response by means of: A. factor XIII B. factor VII C. factor VIII D. prekallikrein E. protein 18. A 22-year-old male presents to the emergency depart ment after sustaining a gunshot wound to the right upper quadrant of his abdomen. On arrival, the patient is lethargic. His vital signs are significant for a systolic blood pressure of 85 mmHg with a heart rate of 130 bpm. As part of his therapy, fluid resuscitation is initiated. Which of the following is currently con sidered the best resuscitation fluid? A. 0.9% sodium chloride B. albumin C. dextran D. 5% dextrose in 0.45% sodium chloride E. 7.5% sodium chloride 19.At birth the umbilical vein becomes the: A. Ligamentum arteriosum B. Round ligament of the liver C. Ligamentum venosum D. Falciform ligament E. Non of the above 20. A 23-year-old male presents to the emergency depart ment by ambulance after an altercation in which he sustained trauma to his head and neck. According to witness reports, the patient was dragged on the floor by his head. On arrival, the patient is found to have a of 50 bpm. His Glasgow Coma Scale level is five. A head CT reveals a large epidural hematoma while a cervical spine CT demonstrates bilateral vertebral facet dislocations at the level of C4. What is the likely cause of his hypotension? A. head trauma B. hypovolemia C. massive vasodilatation D. alcohol intoxication E. blunt cardiac injury 21.Which of the following is true of oxygenation in septic shock?
112
A. Oxygen delivery is elevated while oxygen extraction is decreased. B. Oxygen delivery is reduced while oxygen extraction is increased C. Arterial-venous oxygen difference is increased D. Both oxygen delivery and extraction are increased E. Mixed venous oxygen is decreased 22.Which of the following disorders is not associated with hypocalcemia? A. sarcoidosis B. vitamin D deficiency C. renal insufficiency D. hypoparathyroidism E. pancreatitis 23.In patients receiving massive blood transfusion for acute blood loss, which of the following is correct? A. Packed red blood cells and crystalloid solution should be infused to restore oxygen-carrying capacity and intravascular volume. B. Two units of FFP should be given with every 5 units of packed red blood cells in most cases. C. A “six pack” of platelets should be administered with every 10 units of packed red blood cells in most cases. D. One to two ampules of sodium bicarbonate should be administered with every 5 units of packed red blood cells to avoid acidosis. 24. Regarding Meckel's diverticulum all are true except : A. Can present with chronic anaemia. B. Is present in 2% of the population. C. Can be diagnosed by hydrogen breath test. D. May contain ectopic pancreatic tissue. E. May be implicated in Littre's hernia. 25.Which hormone is produced by the kidney A. calcitonin B. erythropoietin C. 25 hydroxyvitamin D D. aldosterone E. antidiuretic hormone 26.What segment of the nephron is responsible for the majority of sodium absorption? A. proximal convoluted tubule B. loop of Henle C. distal convoluted tubule D. collecting tubule 27.Diffuse axonal injury (DAI) results from what type of force acting on the brain? A. direct impact B. axial loading C. linear acceleration D. rotational acceleration 28.A 16-year old unrestrained driver is involved in a rollover head-on collision. Extensive damage to the vehicle and prolonged extrication time was noted. The patient was hemodynamically unstable upon extrication and was transported by air to level I trauma center. While in transport
113
the patient became severely hypotensive, unresponsive, with impending respiratory distress. Which of the following is consid ered an immediate life-threatening injury in this patient? A. pneumothorax secondary to rib fractures B. aortic intimal tear C. diaphragm rupture D. tension pneumothorax E. myocardial contusion 29 A 40-year-old electrical lineman is injured on a electric pole and is brought to the emergency department unconscious. He has an area on his left hand that is charred, evidence of thermal injury to the arm, and an exit wound on his right knee. Which of the following interventions is least appropriate in the emergency department? A. escharotomy of the left arm B. CT scan of the abdomen and pelvis C. ET intubation D. CT scan of the head and neck E. measurement of serum creatinine phosphokinase levels 30.Critical factors in the “take” of a skin autograft include all of the following except: A. thickness of the graft B. presence of elastin fibers in the graft C. anastomosis of capillaries in the graft to capillaries in the bed D. diffusion of metabolites in the graft cells to plasma of the wound bed 31. Fasciotomy for extremity compartment syndrome should be performed at what compartment pressure? A. 20 mmHg B. 30 mmHg C. 40 mmHg D. 50 mmHg E. none of the above 32. The most appropriate management of isolated radial artery injuries is: A. primary repair B. ligation C. repair with vein graft D. repair with PTFE graft E. all of the above 33. A left neck exploration is performed for a stab injury to the neck in an otherwise healthy male. The left internal jugular vein is found to be completely tran sected with significant bleeding and hemodynamic instability. No other vascular injury is identified. Appropriate management of the internal jugular vein is: A. primary repair B. interposition saphenous vein graft C. interposition with 6 mmPTFE D. ligate the left internal jugular vein E. external jugular vein transposition 34. A 22-year-old male presents into the ER after sustain ing a stab wound to the left chest. The injury is 2 cm left of the sternum at the level of his nipple. Initial vital signs include a pulse rate 88 bpm, GCS 15, respiratory rate 12 breaths/mm, and blood pressure 139/74 mmHg. Initial management of this patient should be: A. CXR B. pericardiocentesis C. left chest tube thoracostomy
114
D. esophagogram E. echocardiogram 35. Where does the arterial supply to the superior and inferior parathyroids originate from? A. superior thyroid artery B. inferior thyroid artery C. external carotid artery D. internal carotid artery E. common carotid artery 36. The signs of hypovolemia in children differ from those of an adult because: A. a child will have minimal signs of shock with a blood loss of 20% B. the presence of hypotension in a child suggests a blood loss of 30% C. poor skin perfusion is not a reliable indicator of blood loss in children D. a child’s circulating blood volume is >100 cc/kg E. a systolic blood pressure of 40 mmHg is appropriate in children less than 6 months of age. 37.Overwhelming postsplenectomy sepsis A. occurs in greater than 10% of children who have undergone splenectomy for trauma B. has greater than 60% mortality C. occurs most often greater than 5 years after splenectomy D. can be prevented by the administration of vaccines against pneumococcus and H. influenzae. E. can initially present as a simple febrile illness 38. Which of the following is true regarding pancreatic trauma and its complications ? A. Complications of pancreatic trauma include pancreatic pseudocyst, abscess, hemorrhage, and fistula. B. Asymptomatic pseudocysts may be managed conservatively, as approximately 80% will spontaneously resolve . C. Pseudocysts which do not spontaneously resolve can be managed with percutaneous drainage, if ductal disruption is identified. D. If a pancreatic fistula is secondary to a major ductal disruption it will not respond to sphincterotomy and duct stenting via endoscopic retrograde pancreatic cholangiopancreatography (ERCP) E. Overall mortality from pancreatic injuries is approximately 45%. 39. A 46-year-old patient had respiratory failure from pneumonia and developed acute respiratory distress syndrome (ARDS). He remained on a ventilator for a prolonged period of time and ultimately underwent a tracheostomy at the level of tracheal ring 4. The patient is now in the rehabilitation unit of your hospital 2 weeks after his tracheostomy and you are called to see him about bright red blood around his tracheostomy site. The most common cause of this bleeding is A. granulation tissue B. erosion of the inferior thyroid artery C. erosion of the innominate artery D. tracheal chondritis E. recurrence of pneumonia 40.Which of these histologic findings is considered pathognomonic for Hodgkin’s lymphoma? A.Mikulicz’s cells B. Charcot-Leyden crystals C.Birbeck granules D.Reed-Sternberg cells E.physaliferous cells
115
41. A 41-year-old female presents to the emergency department after sustaining a gunshot wound to the abdomen, with injuries to the liver and large bowel. Despite successful resuscitation and operative inter vention, the patient dies 2 weeks later of multisystem organ failure in the intensive care unit. Which organ most likely first experienced dysfunction? A. liver B. gastrointestinal tract C. lung D. kidney E. heart 42. Preoperative and operative management of pheochro mocytoma include all except which one of the following? A. use of a long-acting alpha-adrenergic antagonist preoperatively B. laparoscopic removal is not acceptable because of the need to adequately explore both adrenal glands C. ligation of the adrenal vein(s) before the adrenal arteryies D. surgical debulking is indicated even if the pheochromocytoma is deemed unresectable on exploration 43. Which of the following is true regarding anatomy of the adrenal glands? A. Venous drainage of the right adrenal gland is predominantly into the right renal vein. B. Both adrenal glands receive significant blood supply from branches of the superior mesenteric artery. C. Venous drainage of both adrenal glands is directly into the inferior vena cava. D. Both adrenal glands receive significant blood supply from their corresponding renal arteries. 44.Which of the following is true concerning primary hyperaldosteronism? A. Bilateral hyperplasia of the zona glomerulosa is about twice likely as either adrenal adenoma or adrenal carcinoma to be responsible for primary hyperaldosteronism. B. Unilateral adrenalectomy is usually curative for bilateral hyperplasia. C. All patients are hypokalemic D. these patients often present with metabolic alkalosis 45. The chromosome carrying the gene associated with MEN 1 is A.chromosome 14 B.chromosome 13 C.chromosome 12 A.chromosome 11 C.chromosome 10 46. Which of the following is true with regard to esophageal Anatomy? A.It consists of a mucosal, submucosal, and serosal layer. B.It lies to the right of the aorta below the carina C.It receives parasympathetic innervation via the cervical and thoracic parasympathetic chains D.It has a segmental arterial supply. E.The lower esophageal sphincter (LES) is a welldefined anatomical sphincter 47.Vomiting is a reflex behavior best characterized by which of the following? A. It is caused by relatively few stimuli. B. The vomiting center, located in the pons, acts to coordinate the vomiting reflex. C. Vomiting is accompanied by reverse peristalsis of the esophagus. D. The chemoreceptor trigger zone is located on the floor of the fourth ventricle and can elicit vomiting following exposure to some blood- borne substances. E. Failure of the LES to relax is the most notable motor difference between retching and vomiting. 48.Cholecystokinin
116
A. relaxes the sphincter of Oddi . B. inhibits gastric acid secretion C. inhibits gallbladder contractions D. causes mesenteric vasodilation E. inhibits pancreatic exocrine secretions 49. Which of the following statements regarding vitamin D is correct ? A.It undergoes 25-hydroxylation in the kidney and 1-hydroxylation in the liver. B.An adequate dietary intake of foods naturally rich in vitamin D is necessary for proper calcium metabolism. C.Vitamin D is water soluble and absorption is enhanced after ionization in an alkaline environment. D.Vitamin D has its most important effect on calcium metabolism by increasing intestinal absorption. E.Vitamin Ji deficiency results in the clinical disease state of beriberi. 50. With regard to the enterohepatic circulation, which of the following is a true statement ? A. The primary means of replenishing the bile acid pool is by enteric absorption from dietary sources. B. The level of bile acids in the serum undergoes a postprandial increase. C. The highest concentration of bile acids in the body is found in hepatic bile. D.Most of the bile acids that leave the body do so in the urine. E.Chenodeoxycholic acid is metabolized to ursodeoxycholic acid in the liver before being secreted. 51 .A 44-year-old female with achalasia is treated with oral nitrates but experiences significant side effects and is switched to calcium antagonists. There is mild improvement in symptoms during the first several months but she now complains of regurgitation and weight loss. What surgical option(s) should be con sidered in this patient? A. fundoplication B. laparoscopic esophagomyotomy C. esophagectomy D. myotomy and fundoplication E. none of the above 52. Which of the following statements concerning gastrin is most accurate? A. Gastrin in high levels has been shown to cause gastric mucosal hypertrophy. B. Glucose in a meal is one of the most potent stimulators of gastrin release. C. Gastrin-secreting cells (G cells) are found only in the gastric antrum and pylorus. D.Regular administration of antacids will suppress gastrin release as long as pH is kept consistently above 5.0. E. Omeprazole, a proton pump inhibitor, can completely suppress gastrin secretion if taken regularly 53. A 31-year-old man presents to the emergency room with nausea and vomiting, with two bouts of hematemesis, of approximately 24-h duration. He is currently in stable condition. His past medical history is significant for a previous admission 2 months ago for hematemesis, which resolved after endoscopic coagulation. He has been diagnosed by endoscopy with a duodenal ulcer twice within the past 2 years, and twice has failed medical therapy because of noncompliance with triple therapy (omeprazole, clar ithromycin, amoxicillin). Endoscopy at this admission reveals no active bleeding and an inability to pass the endoscope past the edematous pylorus. The most appropriate therapy for this patient is as follows: A. urgent surgical exploration of the upper abdomen with vagotomy and pyloroplasty to relieve this patient’s pyloric obstruction B. seven-day trial of nasogastric suction and intravenous H2- receptor antagonists
117
C. supportive care, including intravenous H2- receptor antagonists, for 24 h, then elective vagotomy and antrectomy D. gastric lavage with at least 10 L of ice cold saline\ 54 . Superior mesenteric artery (SMA) syndrome is not associated with which of the following? A. scoliosis B. placement of a body cast C.anorexia nervosa D. Abdominal aortic aneurysm E. rapid weight gain 55. Which of the following is true regarding small bowel carcinoid tumors? A. Most are biochemically atypical tumors lacking the enzyme dopa decarboxylase. B. They are the most common CI carcinoid tumors. C. Regional lymph node involvement is common in tumors less than 1 cm in size. D. Diagnosis is frequently made in patients prior to surgery. E. Among carcinoid tumors, they are associated with the lowest rate of second primary malignancies. 56 .H. pylori produces hypersecretion of gastric acid in patients who develop H. pylori related duodenal ulcers. What is a proposed mechanism for development of increased gastric acid secretion by this bacterium? A. attenuation of somatostatin release from antral D cells B. direct stimulation of gastrin receptor on parietal cells C. direct stimulation of proton pump on parietal cells D. blockage of somatostatin receptors on parietal cells E. bacterial enzymatic degradation of local prostaglandins 57. Which of the following is true concerning the anatomy of the abdominal wall? A. The origin of the external oblique muscle is from the lowest five ribs. B. The embryologic origin of the rectus abdominis muscle is ectoderm. C. The majority of the neurovascular structures that supply the abdominal wall lie in between the external and internal oblique muscles. D. An abdominal wall hernia requires a defect in the transversus abdominis fascia. E. Lymphatic drainage of the abdominal wall above the umbilicus passes through the ipsilateral axillary lymph nodes. 58.Retroperitoneal fibrosis A. is considered idiopathic in about one-third of all cases B. has been associated with hydralazine, ergotamine, methyldopa, and alpha-blocking agents C. is excluded if only one ureter appears to be involved D. can be treated surgically with ureteral transposition, renal autotransplantation, or omental encasement E. cannot be accurately diagnosed with intravenous pyelography
59. The peritoneum A. can absorb isotonic fluids such as saline at a rate of approximately 90—100 cc/h B. contains a mesothelial lining that secretes fluid to lubricate the peritoneal surfaces, and normally 200—300 cc of free intraperitoneal fluid is present in an adult C. can reabsorb approximately 90% of the red blood cells in the peritoneal cavity intact via fenestrated lymphatic channels in the undersurface of the diaphragm D.air can normally be present in the peritoneum after laparotomy for 7—8 days
118
E. chylous ascites predisposes to intraperitoneal infection 60.Mesenteric cysts A. are usually filled with lymph B. are only embryonic or traumatic in origin C. are rarely palpable on physical examination D. usually present as nontender, asymptomatic abdominal mass E. have a characteristic lateral mobility on physical examinatio 61. Which of the following is true about the omentum? A. Omental torsion is usually primary, is frequently seen in athletes and patients with recent dramatic weight loss and is thought to be related to low omental fat content. B. Omental infarction can occur secondary to torsion, but is usually unrelated. C. Collagen vascular disease is not related to omental torsion and/or infarction. D. Omental torsion shows no preference of location, occurring equally on the left and right side of the omentum. E. Patients with omental torsion usually have symptoms mimicking pleurisy. 62. Which of the following is true concerning the location of spigelian hernias? A. It is more likely to occur below the semicircular line of Douglas because the aponeurotic fibers of the internal and external oblique muscles are parallel below the umbilicus. B. They occur just medial to the semilunar line. C. Spigelian hernias may occur inferior to the epigastric vessels. D. AandB E. B and C 63 .Which of the following is true regarding liver regeneration? A.it is essentially hypertrophy of the remaining hepatocytes B. it is a continuous process throughout life C. the liver can regain its normal size after a 75—90% hepatectomy D. hepatocyte cell division after a hepatectomy is complete within 1 week E. the mechanism of liver regeneration has been clearly identified 64. Which is true about the surgical anatomy of the pancreas? A. The head lies over the first lumbar vertebra. B.The accessory duct enters the second part of the duodenum. C. The superior mesenteric vessels pass behind the uncinate process. D. The SMA lies to the right of the superior mesenteric vein. E. The superior mesenteric vein joins the portal vein behind the head of the pancreas. 65. Splenic vein thrombosis A. commonly presents with isolated esophageal varices B. is usually accompanied by cirrhosis and nortaJ venous flypertension C. is usually a consequence of pancreatic pathology D. should be managed initially with a course of thrombolytic therapy followed by systemic anticoagulation E. is usually accompanied by hyperspienism and thrombocytopenia 66.Nasopharyngeal carcinoma is associated most strongly with which virus? A. CMV B. varicella zoster C.herpes simplex. D.Epstein-Barr virus (EBV). E.human papilloma virus.
119
67. Which of the following is true of retroperitoneal soft tissue sarcomas? A. The most common histologic cellular type is malignant fibrous histiosarcoma. B. Because of the risk of tumor seeding, preoperative biopsy is contraindicated. C. Resection of contiguous organs that are not involved with tumor is indicated to obtain adequate margins. D. Death usually results from distant metastases. E. Lung is the most common site of distant metastases. 68. Potential side effect(s) of succinyle choline A. hyperkalemia B. rhabdomyolysis C. malignant hyperthermia D. myalgias E. all of the above 69. The mechanism of action of local anesthetics is best described as A. binding of the cation of the local anesthetic molecule to extracellular local anesthetic receptors B. binding of the neutral (basic) form of the local anesthetic molecule to cytoplasmic receptors C. binding of the cationic form of the local anesthetic molecule to transmembrane sodium receptors in the inactivated-closed state D. binding of the neutral (basic) form of the local anesthetic molecule to transmembrane sodium channels in the activated-open state E. binding of the local anesthetic molecule to transmembrane potassium channels 70. Which of the following is false concerning the physiologic effects of volatile anesthetics? A. decreased tidal volume B. decreased blood pressure C. decreased respiratory rate D. bronchodilation E. decreased sensitivity to the ventilatory stimulant effects of carbon dioxide 71. Pseudomembranous enterocolitis is caused by the following organisms: A) Clostridium sporogenes B) Clostridium defficile C) Streptococcus faecalis D) Penicillin sensitive staphylocci E) Pseudomonas aeruginos 72. A patient receives a spinal anesthetic at the T5 level. Which of the following physiologic effects is not likely to occur? A. urinary retention B. dilated and relaxed small bowel C. decreased venous return D. bradycardia E. decreased ability to cough 73. Which of the following statements regarding transfu sion compatibility testing is true? A. Patients with Rh-negative blood will have anti D antibodies in serum. B. Antibody screening is a check for anti-A and anti-B antibodies in donor serum. C. Rh-negative patients given Rh-positive blood always have hemolytic reactions. D. Cross-matching of blood involves simulation of actual anticipated transfusion by mixing of recipient and donor blood. E. All of the above.
120
74. What is the blood supply for a pedicled transverse rectus abdominus muscle (TRAM) flap? A. superior epigastric artery B. inferior epigastric artery C. rectus abdominis perforators D. internal iliac artery E. intercostal artery 75. A patient sustains a complete cervical cord injury secondary to a cervical burst fracture. The radiographs reveal injury at the C6-C7 level. Assuming a complete cord injury at this level, which of the following find ings on physical examination would not correlate with this level of injury? A. absent patellar tendon reflexes B. weakness with ankle plantar flexion C. weakness with elbow flexion D. weakness with finger abduction/adduction E. weakness with shoulder abduction 76. The following are advantages of bipolar cautery over monopolar cautery except: A. decreased tissue damage B. decreased smoke C. improved hemostasis D. elimination of grounding pad burns E. reduced risk of insulation failure 77. Which of the following statements about patients with abdominal compartment syndrome is NOT true ? A) Abdominal pressure is usually measured indirectly through inferior vena cava B) Multiple contributing factors are commonly responsible C) The chief manifestations are reflected in central venous pressure , ventilatory function, and oliguria D) Decompression of the abdomen is required to resverse the syndrome E) Aggressive hemodynamic monitoring and management is required when the abdomen is opened 78. Which of the following statement characterize the biology of allotransplantation is false? A. The rejection response is systemic. B. The rejection response is learned. C. The rejection response involves a constellation of immunologic and environmental factors. D. Allotransplantation evokes a cellular immune response. E. Allotransplantation evokes a humoral immune response. 79-Sarcomata may show all of the following EXCEPT: A) production of myxomatous tissue B) production of collagen C) spindle shaped cells D) signet ring cells E) blood stream metastasis 80.A 35-year-old male is struck on the lateral aspect of his right knee by the bumper of a car travelling at low velocity. On examination he is unable to dorsiflex the ankle, evert the foot
121
and extend the toes. There is loss of sensation of the dorsum of the foot. He is most likely to have damaged which structure? A. Common peroneal nerve B. Deep peroneal nerve C. Saphenous nerve D. Sural nerve 81.Which of the following is not within the carpal tunnel? A. median nerve B. flexor digitorum profundus C. flexor digitorum superficialis D. flexor pollicis longus? E. abductor pollicis longus 82.A 48-year-old woman with chronic pancreatitis due to gallstones is noted to have a macrocytic anaemia. What is the most likely cause of the anaemia? A. Bone marrow dysfunction B. Folate deficiency C. Hyposplenism D. Hypothyroidism E. Vitamin B12 deficiency 83.A 5-year-old boy presents to Accident and Emergency complaining of acute pain over his upper tibia. He is febrile and he refuses to move his leg. A diagnosis of osteomyelitis is suspected. The likely infecting organism is? A. Clostridium difficile B. Haemophilus influenzae C. Pseudomonas D. Salmonella E. Staphylococcus aureus 84.Which of the following is true concerning Scaphoid fractures? A. Rarely occur in young adults B. when complicated by avascular necrosis the proximal pole is usually affected C. should be treated by bone grafting and internal fixation even if undisplaced D. wrist fractures are uncommon E. anteriorposterior and lateral radiographs reveal most fractures 85.A 17-year-old girl underwent emergency splenectomy after a domestic accident. Which one of the following organisms is most likely to cause life-threatening infection in the future? A. Actinomycosis B. Haemophilus influenzae C. Pseudomonas aeruginosa D. Staphylococcus aureus E. Streptococcus pneumonia
A. B.
122
86.Which vertebral level and corresponding structure is correct? C4 and bifurcation of the carotid artery T2 and manubriosternal joint
C. D. E.
T10 and opening for vena cava in diaphragm T12 and oesophageal opening in the diaphragm T8 and aortic opening in the diaphragm
87.Which of the following statements is true of Compartment syndrome:A. Only occurs following fractures. B. Loss of distal pulse is an early sign. C. The presence of pain is unhelpful in diagnosis. D. Can be treated using a minimally invasive technique. E. Passive stretch of affected muscles exacerbates pain. 88. The rectum: A) is devoid of peritoneum B) is surrounded by peritoneum C) has peritoneum on its lateral surfaces for its upper two- thirds, and on its anterior surface for its upper one-third D) has pritoneum on its anterior surface for its upper two-thirds, and on its lateral surfaces for its upper one-third E) has peritoneum on its anterior surface only 89.Which of the following is not a branch of the pudendal nerve? A. Perineal nerve. B. Dorsal nerve of the penis. C. Inferior rectal nerve. D. Genitofemoral nerve. E. Posterior scrotal nerve 90.A patient presents with a history of low back pain and sciatica. The pain radiates to the little toe, the ankle reflex is absent and the patient has difficulty in everting the foot. Which nerve root is likely to be trapped? A. L3 B. L4 C. L5 D. S1 E. S2 91.The ulnar nerve all are true except : G. innervates the first dorsal interosseus muscle H. originates from the laterall cord of the brachial plexus I. has no branches above the elbow J. innervates the medial half of the flexor digitorum profundus K. innervates the adductor pollicis muscle
92. A) B) C) D)
123
The umbilicus: lies near the to the xiphoid than to the pubis derives its cutaneous innervation from the eleventh thoracic nerve transmits, during development, the umbilical cord two arteries and two veins usually lies at about the level between the third and fourth lumbar vertebra
E)
emberiologicall, may transmit urine but never bowel contents
93.Which of the following statements is true of upper limb nerve injuries? A. Injury to the median nerve results in a wrist drop B. Injury to the radial nerve results in loss of sensation over the palmar aspect of the index finger C. Injury to the median nerve results in loss of sensation in the anatomical snuffbox D. Injury to the ulnar nerve results in a claw hand E. Injury to the ulnar nerve results in loss of sensation over the thumb 94.Concerning the inguinal canal: A. It transmits the ilio-inguinal nerve B. The deep inguinal ring lies below the mid-point of inguinal ligament C. The superficial inguinal ring overlies the pubic tubercle D. Laterally , the anterior wall is made up of the external oblique aponeurosis E. Laterally, the posterior wall is formed by the conjoint tendon 95.A raised titre of anti-HBs in the blood, all are true except : A. signifies previous hepatitis B infection B. is produced after hepatitis B vaccination C. indicates immunity to hepatitis infection D. indicates active hepatitis B infection 96.Which of the following is not an action of angiotensin II? A. aldosterone secretion B. sodium absorption C. efferent arteriolar constriction D. arterial dilation E. nephrosclerosis in the kidney 97.The following statements regarding intravenous solutions is correct: A. Normal saline contains 180mmol/l of sodium B. Ringer's lactate solutions is designed for intracellular fluid replacement C. Sodium bicarbonate 8.4% is a hyperosmolar solution D. Normal saline with added potassium is appropriate therapy to correct a respiratory alkalosis 98.The following is true of the blood supply of the rectum all are true except: A. The principle blood supply is derived from the inferior rectal artery B. The median sacral vessels contribute to the blood supply C. As the inferior mesenteric artery crosses the pelvic brim it becomes known as the superior rectal artery D. The internal venous rectal plexus is continuous with the vascular cushions of the anal canal E. The superior rectal vein drains to the portal system
99.Which of the following concerning the Femoral sheath is false: A. Contains the femoral artery B. Contains lymph nodes
124
C. Contains the femoral canal D. Contains femoral nerve 100.A knife stabbed horizontally through the fourth intercostals space to the right of the sternum is likely to damage the: A. Right brachiocephalic veinB. Hemiazygos vein C. Descending aorta D. Right pulmonary artery
Choose the Best appropriate answerfor each of the following questions: 1.The ilio-inguinal nerve: A. supplies the rectus abdominis muscle B. supplies skin on inner side of thigh C. supplies the cremasteric muscle D. supplies the urethra E. does none of the above 2. The skin of the tip of the index finger is supplied by the: A. Radial nerve only B. Radial & median nerves C. Median & ulnar nerves D. Ulnar nerve only E. Median nerve only 3. Hypovolaemic shock is characterized by: A. A low central venous pressure , high cardiac output, low peripheral resistance B. A high central venous pressure, high cardiac output, high peripheral resistance C. A low central venous pressure , low cardiac output, high peripheral resistance D. A low central venous pressure , high cardiac output, high peripheral resistance E. A high central venous pressure, low cardiac output, low peripheral resistance
125
4. Which of the following in NOT true of Hodgkin's disease? A. Usually starts from several groups of nodes simultaneously B. Usually involves liver & spleen C. Sometimes manifests itself as pyrexia of unknown origin D. Severe pain follows ingestion of alcohol E. Shows increased susceptibility to opportunistic infection 5. Tetanus toxoid: A. Is produced by injecting animals with antititanic serum B. Is administered to previously immunized subjects with potentially infected wounds C. Frequently gives rise to anaphylactic reaction D. Is used to induce active immunity against tetanus 6. The most probable cause of shock in a patient with multiple injuries & craniocerebral trauma is: A. Depression of vital medullary centres B. Hypoperfuion control over subcortical areas C. Loss of cortical control over subcortical areas D. Hypovolaemia E. Inadequate ADH response 7. The most sensitive guide to acute changes in fluid balance in a surgical patient is: A. Accurate daily weight B. Serial serum Na concentration C. Fluid balance sheets recording inputs & outputs D. Daily urine output E. Serial anion gap measurements 8. cellullitis is: A. Inflammation of the bone marrow B. Inflammation of the mastoid cells
126
C. Inflammation of the subcutaneous tissues D. Infiltration of the skin by gaint cells E. A malignant condition 9. secondary haemorrhage occurs: A. Within 6 hours of operation B. 7-14 days after operation C. As a result of violent coughing on recovery from anaesthesia D. Due to a blood transfusion line disconnected E. When a ligature slips 10. the minimum urine output for 24 hours required to excrete end products of protein metabolism is: A. 200 ml B. 300 ml C. 400 ml D. 500 ml E. 600 ml 11. Potassium deficiency is present if the plasma-potassium level is: A. 6.0 mmol/l B. 5.0 mmol/l C. 4.5 mmol/l D. 4.0 mmol/l E. 3.0 mmol/l 12.in health the pH of the blood lies between the range: A. pH 7.05-7.19
127
B.
7.20-7.35
C.
7.36-744
D.
7.45-7.59
E.
7.60-7.80
13. TNM classification of a malignant tumour was designed as: A. An histological staging B. A clinical staging C. A staging carried out at operation D. A staging dependent upon radio scanning & skeletal survey E. A staging dependent upon ultrasound 14. a blue-green discharge from an ulcer will be seen to contain: A. Pseudomonas pyocyaneus B. Streptococcus viridians C. Candida albicans D. Staphylococcus aureus E. Haemophilius influenzae 15. a rodent ulcer is: A. A squamous cell carcinoma B. A basal cell carcinoma C. Only occur on the face D. Contains epithelial pearls E. A venous ulcer 16. the space of Parona is: A. In the wrist between the deep flexor tendons & the pronator quadratus B. Above the patella between the quadriceps muscle & the femur C. Benath the tendon of the iliopsos D. Between the achills tendon & the posterior aspect of the tibia E. The web space of the palm
128
17. 'rest pain' occurs: A. Anywhere in the body at rest B. In the thigh of a patient with Buerger's disease C. In the calf of a patient with intermittent claudicating D. In the foot of a patient with severe vascular disease E. In the back 18. ischaemia means: A. Pain in the ischial tuberosities B. Anaemia due to malignant seconderies in the ischial part of the pelvis C. Lack of blood flow D. Increased blood flow E. Polycythaemia 19. Colles' fracture is: A. A common in adolescence B. A fracture about the ankle joint C. Common in elderly women D. A fracture of the head of the radius E. A fracture of scaphoid 20. Pott's disease is; A. A fracture dislocation about the ankle B. A neuropathic joint C. Traumatic ostechondritis of the spine
129
D. Tuberculosis of the spine E. A secondary tumour in the skull 21. Vincent's angina is a form of angina associated with: A. Spasm of the oesophagus B. Diphtheria C. An infection of the mouth D. Coronary artery spasm E. Carcinoma of the bronchus Vincent's angina is caused by : Mycoplasma Fusiformis Microaerophilic streptococci Treponema pallidum 22. Ludwig's angina is due to : A. A type of coronary artery spasm B. Oesophageal spasm C. Retropharyngeal infection D. A virulent infection of the cellular tissue around the submandibular salivary gland E. Infection with candida 23. in simple nodular goiter: A. Carcinoma occurs in 30% of cases B. The nodular stage is irreversible C. Operation is contraindicated D. The patient does not develop hyperthyroidism E. Cretinism is the presenting feature 24. Hashimoto's disease is: A. A granulomatous thyroiditis B. An auto-immune thyroiditis
130
C. An infiltrating fibrosis of the thyrois & the adjacent muscles D. Focal thyroiditis E. A parathyroid tumour 25. A thyroglossal fistula: A. Is never congenital B. Follows inadequate removal of a thyroglossal cyst C. Has a hood of skin with its concavity upwards D. Is lined throughout by squamous epithelium E. Occurs in carcinoma of the tongue 26. The following are clinical signs supporting an early diagnosis of carcinoma of the breast: A. A prickling sensation in a breast lump B. Peau d'ornge C. Brawny arm D. Cancer en cuirasse E. A krukenderg tumour 27.The gastroduodenal artery is a branch of the: A. Celiac axis B. Hepatic artery C. Superior mesenteric artery D. Gastroepiploic artery E. Splenic artery 28.Chronic gastric ulcers most often occur in patients with: A. Blood groub A B. Tend to occur in alkaline mucosa C. Muscularis mucosae is separated from the muscularis at the edge of the ulcer D. Are malignant when there is epithelial proliferation & downgrowths
131
E. Are never large 29. Meckel's diverticulum: A. Is present in 20% of the human race B. Arises from the mesenteric border of the jejunum C. May contain heterotopic pancreas D. Is only present in the male sex E. Is a diverticulum of the bladder
30. Intussusception is related to: A. Mucoviscidosis B. Swollen Peyer's patches C. Volvulus D. A littre's hernia E. A patent vitello intestinal duct 31. The site of the neck of a femoral hernia is the: A. Transversalia fascia B. Iliopectineal ligament C. Femoral ring D. Cribriform fascia E. Obturator foramen 32. Regarding operation for an indirect inguinal hernia: A. It should not be performed on patients who have chronic bronchitis B. General anaesthesia has to be used C. In infants the posterior inguinal wall should be repaired D. In adults the internal inguinal ring usually needs to be strengthened E. Mesh implants are mandator
132
SELECT THE BEST SINGLE ANSWER : 1-A benign vascular tumor arising in the liver is most likely to be a: A. hemangioma B. lymphangioma C. pericytoma D. glomangioma E. granuloma pyogenicum 2-A neoplasm frequently associated with AIDS is: A. Kaposi’s sarcoma B. carcinoma of lung C. malignant melanoma D. telangiectasia E. Follicular carcinoma of thyroid 3-Petechiae are most suggestive of: A. acute rheumatic heart disease B. bacterial endocarditis C. Libman-Sack’s endocarditis D. Marfan’s disease E. carcinoid syndrome 4-When a person dies suddenly from a “heart attack”, the most likely event that led to the sudden death is: A. rupture of the heart B. congestive heart failure C. angina pectoris D. coronary artery embolism E. cardiac arrhythmia 5-The most frequent cause of pure right-sided heart failure is: A. congenital heart disease B. ischemic heart disease C. pulmonary disease D. liver disease E. renal disease 6-In the uncomplicated patent ductus arteriosus, as might be seen in an infant 6 months of age, blood flows from the aorta to the: A. pulmonary vein B. pulmonary artery C. right ventricle D. right atrium 7- In a moderate-sized myocardial infarct it would take approximately how long to replace the necrotic muscle with fibrous tissue? A. 2 days B. 2 weeks C. 2 months D. 2 years E. 20 years 8-The most common primary tumor of the heart is: A. rhabdomyoma B. myxoma C. angioma D. mesothelioma E. fibroma 9. Ionizing radiation to the neck during childhood was shown to result in significant increase in: A. Leukemia B.
133
Lymphoma Thymoma Laryngeal carcinoma .Carcinoma of the thyroid
C. D. E
10. Barrette esophagus is an example of: A . Cell hypertrophy B .Epithelial hyperplasia C .Metastatic carcinoma D .Ischemic injury E .Epithelial metaplasia : The most common cause of intestinal obstruction is .11 A. volvulus B. neoplasm C. intussusception D. hernia E. adhesion Each of the following applies to Hirschsprung’s disease except .12 A. intestinal obstruction B. absence of ganglion cells in myenteric plexus C. treated by removal of distended segment of bowel D. toxic megacolon as complication E. narrow, more distal segment exhibits characteristic deficiency of development In which of the following segments of the gastrointestinal tract is primary carcinoma least .13 ? common A. esophagus B. stomach C. small intestine D. colon E. mouth Factors associated with an increased risk for development of carcinoma of the stomach .14 include each of the following except A. family history of gastric carcinoma B. acute stress ulcers C. pernicious anemia D. chronic atrophic gastritis with achlorhydria E. high dietary intake of smoked meat and fish : The best method of demonstrating the presence of celiac disease is .15 A. radiographic study of the upper intestinal tract B. small bowel biopsy C. Schilling test D. history of specific food intolerance E. quantitative stool fat test : in children, the most common site of intussusception -16 A. rectosigmoid colon B. ileocecal valve C. transverse colon D. jejunum E. appendix ? Which neoplasm is most FREQUENTLY found in the appendix .17 A. carcinoid B. villous adenoma C. lymphoma
134
D. adenomatous polyp E. adenocarcinoma Which one of the following lesions usually presents as a discrete, freely movable nodule in .18 ? the breast A. sclerosing adenosis B. cystic disease C. ductal carcinoma D. fibroadenoma E. plasma cell mastitis : Breast cancer is suggested by physical findings which include all of the following except .19 A. skin dimpling B. alterations of breast contour C. edema of the skin D. nipple inversion E. pain on palpation : The most common location for carcinoma of the breast is-20 A. upper inner quadrant B. lower inner quadrant C. upper outer quadrant D. lower outer quadrant E. subareola and nipple : Diabetes insipidus is associated with a lack of .21 A. glucocorticoids B. insulin C. thyroid hormone D. antidiuretic hormone E. growth hormone All of the following are associated with Graves’ disease excep .22 A. elevation of body temperature B. increased heart rate C. intolerance to heat D. marked weight loss E. atrophy of lymphoid tissue : Answers A-1 A-2 B-3 E-4 C-5 B-6 C-7 B-8 E-9 E-10 D-11 C-12 C-13 B-14 B-15 B-16 A-17 D-18 E-19 C-20
135
D-21 E-22
NB: The answer of each question is underlined Select the Best Appropriate Answer : 1. Patient with high risk for DVT : a. Smoker patient with operation more than 40 minutes b. Patient with upper abdominal surgery more than 40 minutes c. Patient with pelvic operation for malignancy d. B and c only e. None of the above
2. a. b. c. d. e.
Patient with obstructive jaundice going for surgery will need : Intravenous dextrose solution INR checked Prophylactic antibiotics A and b only All the above
3. Electrocardiogram findings In hyperkalemia a. Depressed ST segment b. Tall peaked T wave c. Widened QRS d. All the above e. None of the above 4. The following can cause respiratory acidosis : a. Post operative pain from abdominal incisions b. Post operative atelectasis c .Hypoventilation by the anesthetist d. None of the above e. All of the above 5. The following are factors determining severity of burn except : a. Extent of burnt surface area b. Depth of burn & causative agent c. Age of the patient d. Associated injuries e. Site of the burn 6. Acute Epidural haematoma : a. Due to meningeal artery tear b. Can be treated conservatively c. Classically there is contra lateral, dilated, fixed pupil d. Needs operation within 24 hours e. A ad c only 7. Regarding papillary carcinoma of the thyroid :
136
a. b. c. d. e.
Commonest thyroid cancer Previous neck radiation decrease the risk Rare familial form Is the tumor of the middle age A and c only
8. The commonest type of breast carcinoma is : a. Invasive duct carcinoma b. Medullary carcinoma c. Invasive lobular carcinoma d. Mucinous (colloid) carcinoma e. Insitu carcinoma & Pagets disease 9 . The classical picture of Acute arterial embolism include All the following except: a.Pallor b.Pain c.Parasthesia d. Impalpable Peripheral pulses e. Swelling 11) Pulmonary embolism may be a complication of all of the following Except: a. prolonged bed rest b. a surgical operation c. vitamine K deficiency d. oral contraceptive therapy e. Antithrombin III deficiency 15) Pseudomembranous enterocolitis is caused by the following a. Clostridium sporogenes b. Clostridium defficile c. Streptococcus faecalis d. Penicillin sensitive staphylocci e. Pseudomonas aeruginos 21) The breast : A) is a modified apocrine sweat gland B) overlies the third to the sixth rib C) is having usually more than 30 lacteferous ducts D) is drained only by six groups of axillary lymph nodes E) is having no attachment to skin 22) The commonest cause of bloody nipple discharge is : A) mammary ductectasia B) carcinoma of the breast C) lactational mastitis D) duct papilloma E) fibrocystic disease of the breast 23) Lactational (Bacterial) mastitis: A) never occurs in lactating mothers B) is usually caused by streptococcu hemolyticus
137
organisms
C) is mostly caused by staphylococcus aureus D) is treated only by incision and drainage E) is the same as mastitis of puberty 24) On a mammogram, Signs of malignancy may include all of the following Except : A) mass lesions with poorly defined irregular margins B) fine stipped soft tissue with microcalcifications C) thickening and retraction of the overlying skin D) dysplastic ductal pattern E) well circumscribed , homogenous,and often surrounded by a zone of fatty tissue 25) Regarding Paget's disease of the nipple : A) It is a benign condition B) It is simply an eczematous lesion of the nipple C) It is treated by excision of the nipple D) It is usually diagnosed by biopsy of the suspected lesion E) the areola and the surrounding skin are never involved 26) Regarding carcinoma of the breast : A) invasive intraductal carcinoma is the commonest form B) lobular carcinoma may present bilateral C) clinical staging is always correct and definit D) the medullary (anaplastic) type feels hard E) A, and B, are correct 27) Hashimoto's thyroiditis : A) is of viral origin B) presents usually by nodular goiter C) can be diagnosed by thyroid antibodies and fine needle aspiration of D) is often premalignant E) must be treated always by surgery 28) the MOST important finding in the diagnosis of acute appendicitis is: a. vomiting b. Fever c. leukocytosis d. right lower quadrant pain and tenderness e. referred rebound tenderness[ Rovsing sign ] 29) Common characteristics of small bowel obstruction include all of the following EXCEPT: a. ascites b. frequent progression to strangulation c. failure to pass flatus d. distention e. vomiting 30) Achalasia is associated with all of the following EXCEPT: A. chagas ’ disease in South America B. dysphasia. C. weight loss. D. relaxation of the lower esophageal sphincter with swallowing .
138
thyroid
E. aspiration pneumonia, which may cause lung abscesses. 31) Most important steps in management of head injury include: A. Prevent hypoxia B. Prevent Dehydration C. Assure Brain Metabolism D. Prevent secondary brain injury E. All the above 32) Follicular carcinoma of thyroid gland: A. Is the commonest thyroid tumor B. Can be diagnosed by F.N.A.B C. Usually multifocal D. Commonest tumor of young age E. Prognosis is worse than papillary carcinoma
33) Tension pneumothorax A. is the commonest type of chest injuries B. Needs urgent X-Ray chest C. Is a clinical Diagnosis D. Causes flat neek viens E. Treated by thoracotomy tube after chest X-ray. 33) Lymphedema: A) may be Congenital B) should be bilateral C) may be pitting in early stage D) A & C only E) None of the above 35) Glasgow coma scale all the following are true except: a)
Used for evaluation of comatose patient.
b)
It ranges from ( 3 to 15).
c)
Useful for neurological follow up.
d)
Useful for pupils evaluation.
e)
Best motor response given 6 point.
36) Calcitonin hormon is secreted to the blood circulations from: a)
Parathyroid gland.
b)
Parafollicular cells of thyroid gland.
c)
Supra renal gland.
d)
Pituitary gland.
e)
Gonads.
37) The following are features of thyrotoxicosis except: a)Weight gain. b)
139
Palpitations.
c)Proximal myopathy. d)
Increased skin pigmentation.
e)
Pretibial myxoedema
38) In obstructive jaundice : a)Urinary conjugated bilirubin is increased. b)
Serum unconjugated bilirubin is increased.
c)Urinary urobilinogen is increased. d)
Serum conjugated bilirubin is reduced.
e)Faecal stercobilinogen is increased.
39)Regarding Hydatid disease: a) Is due to Ecchinococcus granulosa. b) Man is an accidental intermediate host. c) The liver is the commonest site of infection. d) Can be diagnosed by the Casoni test. e) All are true. 40) The first aid of treatment in fracture of cervical spine should be: a)
Cervical spine x-ray.
b)
Analgesia.
c)
Neck immoblization.
d)
Cervical traction.
e)
Non of the above
41) Small bowel obstruction often results in: (all correct except one) a)
Hyperkalaemia.
b)
Metabolic alkalosis.
c)
Oliguria.
d)
Hypovolaemia.
e)
Severe dehydration.
42) In acute appendicitis all of the following are true except: a)Anorexia. b)
Abdominal pain usually precedes vomiting.
c)Pain after begins in the paraumbilical region. d) e)
140
Constipation diarrhea may occur. Dysuria excludes the diagnosis.
43) The most common cause of massive haemorrhage in the lower gastroinfestinal tract is : a)
Carcinoma.
b)
Diverticulosis
c)Diverticulitis d)
Polyp.
e)Ulcerative colitis. 44) The recurrent laryngoeal nerve is branch of : a) Facial nerve. b) Glosso-pharyngeal nerve. c) Cervical plexus. d) Vagus nerve. e) Brachial plexus. 45)Regarding the management of polytrauma: a) b) c) d)
Death follow a trimodal distribution. X-ray after primary survey should be AP cervical spine, chest and pelvis. Cardiac tamponade is characterized by raised B.p, a low JUP. Assessment of uncomplicated limb fractures should occur during the primary survey. e) A and B only. 46) Basal cell carcinomas: a) Usually metastasise to regional lymph nodes. b) Are less common than squamous cell carcinomas. c) Are characterised histologically by epithelial pearls. d) Are particularly common in oriental races. e) Non of the above is correct. 47. Hidradenitis suppurativa a. inflammation of the apocrine sweat glands b. common site is the axilla c. commoner in women d. all of the above are correct e. none of the above 48. Sebaceous cysts a. Characterized by the a punctum. b.Usually subdermal c.Infection is not common. d.Palms and soles may be affected. e.all are true. 49. Inguinal hernia (a) A direct hernia passes through the deep inguinal ring into the inguinal canal. (b) A femoral hernia is more common than an inguinal hernia in females. (c) An inguinal hernia can not be distinguished from a femoral hernia by its relationship to the inguinal ligament.
141
(d) The inferior epigastric vessels lie medial to the deep inguinal ring. (e) The floor of the inguinal canal is formed by the conjoint tendon. 50) Regarding acute abdomen: (a) Mesenteric adenitis is a common cause of abdominal pain in children. (b) The absence of free air on an erect chest x-ray excludes an intra-abdominal perforation. (c) Free air under the right hemi-diaphragm can be mistaken for gas within the stomach. (d) A raised serum amylase is diagnostic of acute pancreatitis. (e) Diverticulosis is a common cause of acute left iliac fossa tenderness associated with pyrexia and a raised white cell count. 51) Medullary carcinoma of the thyroid a. Is a tumour of the parafollicular C cells b. Produce thyroxine as the principle hormone c. 10% of cases are sporadic d. Often occur as part of the MEN type II syndrome e. subtotal thyroidectomy is the surgical treatment of choice
52) ulcerative colitis a. Shows full thickness inflammation b. The rectum is almost always involved c. 50% patients have terminal ileal disease d. Enterocutaneous or intestinal fistulae are common e. The serosa is usually affected 53) Hodgkin's disease a. Most commonly occurs in patients over 60 years of age b. may presents as painless lymphadenopathy c. The Pel-Epstein fever is a characteristic feature d. Stage III disease is confined to one side of the body. e. Reed-Sternberg cells are not always present 54) Intussusception a. Is most common in children from 6 to 12 years b. Presents with colicky abdominal pain, rectal bleeding and an abdominal mass c.50% present with diarrhoea and vomiting suggestive of gastroenteritis d. If shock or peritonitis hydrostatic reduction can be attempted e. A Meckel's diverticulum can never induce an intussusception 1. Hypovolaemic shock is characterized by:
F.
A low central venous pressure , high cardiac output, low peripheral resistance
G. A high central venous pressure, high cardiac output, high peripheral resistance H. A low central venous pressure , low cardiac output, high peripheral resistance I.
142
A low central venous pressure , high cardiac output, high peripheral resistance
J.
A high central venous pressure, low cardiac output, low peripheral resistance
6. The most probable cause of shock in a patient with multiple injuries & craniocerebral trauma is: F.
Depression of vital medullary centres
G.
Hypoperfuion control over subcortical areas
H.
Loss of cortical control over subcortical areas
I.
Hypovolaemia
J.
Inadequate ADH response
7. The most sensitive guide to acute changes in fluid balance in a surgical patient is:
F.
Accurate daily weight
G. Serial serum Na concentration H. Fluid balance sheets recording inputs & outputs I.
Daily urine output
J.
Serial anion gap measurements
6-All of the following are true about neurogenic shock except:
f) g)
There is a decrease in systemic vascular resistance and an increase in venous capacitance.
h)
The use of an alpha agonist such as phenylephrine is the mainstay of treatment.
i)
Severe head injury, spinal cord injury, and high spinal anesthesia may all cause neurogenic shock.
j)
A and B
Tachycardia or bradycardia may be observed, along with hypotension.
18-Which of the following statement(s) is/are true concerning septic shock? e) f)
The clinical picture of gram negative septic shock is specifically different than shock associated with other infectious agents The circulatory derangements of septic shock precede the development of metabolic abnormalities
g)
Splanchnic vascular resistance falls in similar fashion to overall systemic vascular resistance
h)
Despite normal mechanisms of intrinsic expansion of the circulating blood volume, exogenous volume resuscitation is necessary
:In all forms of shock there is.40 a) b) c) d) e)
143
An impairment of cellular oxygenation. * A decreased cardiac output. An increased effective circulating fluid volume. A low central venous pressure (CVP). An increased pulse rate.
:The metabolic acidosis of shock can be effectively treated by .41 f) Warming the patient. g) Administering ammonia chloride. h) Artificial ventilation. i) Restoring normal tissue perfusion. * (Nacl intravenous infusion. septic shock is associated with a hypodynamic cardiovascular state: ( all correct except one .42 j) if preceded by existing hypovolaemia. k) In generalized peritonitis. l) When there is a gram-positive bacteraemia. * m) In elderly patients. n) In late gram negative septicemia. :In cardiogenic shock .43 o) p) q) r) s)
The central venous pressure is low. * The difference in the arteriovenous oxygen tension is increased. The haematocrit is raised. The blood pressure is unaffected. C&D only.
(A blood transfusion reaction: (all correct except one .44 t) u) v) w) x)
May be due to incompatibility of the recipient serum and donor cells. Is manifest by thrombophlebitis of the infusion site. * Occurs within the first 30 minutes of transfusion. May produce renal damage. May produce anaphylactic shock.
Massive blood transfusions may be complicated by:except .45 A. B. C. D. E.
Hyperkalaemia. Hypocalcaemia. Coagulopathy. Leucopenia. * DIC
. 9. secondary haemorrhage occurs:
F.
Within 6 hours of operation
G. 7-14 days after operation H. As a result of violent coughing on recovery from anaesthesia I.
Due to a blood transfusion line disconnected
J.
When a ligature slips
:Platelets in the wound form a hemostatic clot and release clotting factors to produce .Choose one answer a. thrombin b. Fibrin c. Fibrinogen d. Fibroblasts e. thromboplastin
Final surgery exam-4th year medical students 1. Which of the following is not true about Pneumatosis intestinalis of small bowel? a) It is seen equaly and males and females b) Most common location is subserosa in the jejunum c) Operative Procedures are required in most of the cases d) It is associated with COPD and immunodeficiency states
144
2. Which of the following is not true for ectopic pancreas a) stomach and duodenum are the most common site b) Ectopic pancreas appears as a submucosal irregular nodule in the wall c) Islet tissue is present in all the organs where ectopic pancreas is present. d) Ulceration, bleeding and obstruction are the most common symptoms 3. The most common cystic disease of the spleen is a) Hydatic cyst of spleen b) Pseudo cyst of the spleen c) Columnar lined cyst of spleen d) Cystic lymphangioma 4 ) True regarding splenic abscess are all except: a) Majority of splenic abscess result from hematogenous spread from other sites b) Both Gram positive and gram negative organisms are responsible for splenic abscess c) Splenomegaly is present in most of the patients. d) 2/3rd of the splenic abscess are solitary in adults 5. Which of the following serum gastrin values is not indicative of a) 5000pg/ml b)1000pg/ml c) 500pg/ml d) 100 pg/ml
gastrinoma
6. Which does not predispose to Carcinoma stomach? a) Low fat and protein diet b) Salted meat and fish c) Low Nitrate consumption d) HIgh Complex carbohydrate consumption 7. Which of the following is not true about gastric lymphoma? a) Stomach is the most common organ in the gi system which is involved in Lymphoma b) Peak incidence of lymphomas is seen in 6th-7th decade c) Endoscopy usually reveals gastritis like picture or gastric ulcer. d) MALT lymphoma is the commonest variety. 8. Double bubble sign is seen in a) Pyloric stenosis b) Esophageal atresia c) Duodenal Atresia d) Ileal atresia 9. Most common cause of Budd Chiari syndrome in Asia is? a) Polycythemia b) IVC obstruction c) Myeloproilferative disorders d) Bechet’s disease 10. Which does not predispose to Carcinoma stomach? a) H. Pylori infection b) Salted meat and fish c) Low Nitrate consumption d) High Complex carbohydrate consumption
145
11. Regarding the location of gastric ulcers the most common affected part of the stomach is: a) Lesser curvature b) Antrum c) Pyloric channel d) Proximal stomach 12. A 59 year-old man, underwent partial gastrectomy with Billroth ll reconstruction for intractable peptic ulcer disease which of the following metabolic disturbances is not a potential consequences of this procedure: a) Megaloblastic anemia b) Iron deficiency anemia c) Osteoporosis d) Osteitis fibrosa cystica e) Statorrhea 13. A 20 year-old male is examined in ER following motor vehicle accident. His Rt leg is injured as are his elbow and clavicle. Which of the following fractures or dislocations is most likely to result in an associated vascular injury? a) Knee dislocation b) Closed posterior elbow dislocation c) Midclavicular fracture d) Supracondylar femur fracture e) Tibial plateau fracture 14. A 53 year-old man presents with constipation and 10 kgrs loss of weight over the course of 6 months. Colonoscopy revealed a fungating mass in the sigmoid colon. Biopsy is consistent with adenocarcinoma. His metastatic workup is negative. A CEA level is obtained and is fourfold greater than normal. Which of the following is the appropriate use of this test? a) As an indication for neoadjuvant therapy b) As an indication for postop radiation therapy c) As an indication for preoperative PET scanning d) As an indication for a more aggressive sigmoid resection e) As a baseline measurement prior to monitoring postoperatively for recurrence. 15. How much bile is produced by liver /day? a) 100-300 ml b) 300-500 ml c) 500-1000ml d) 500-1200 ml 16. In which of the following conditions anti thyroid antibody may be elevated? a) Hashimoto thyroiditis b) Grave's disease c) Multinodular goitre d) All of the above 17. A 45 year-old man came to ER c/o fever, tachycardia, confusion and vomiting. Investigations were done and reveals markedly elevated T3 and T4. He is diagnosed as having a thyroid strom. Which of the following is the most appropriate next step in the management of this patient. a) Urgent subtotal thyroidectomy b) Urgent total thyroidectomy
146
c) Urgent hemodialysis d) Administration of fluids, antithyroid drugs, B-blockers, iodine solution and steroids. e) Urgent radiation therapy to the neck 18. Which of the following pathologies is thought to have no malignant potential? a) Ulcerative colitis b) Villous adenoma c) Familial polyposis d) Peutz-Jeghers syndrome e) Crohn’s disease 19. The X-Ray showed at the Right is: a. b. c. d.
PTC (percutanoeus transhepatic cholangiography) T-tube cholangiography ERCP (Endoscopic retrograde cholangio pancreatgraphy) Operative cholangiogram through cystic duct
20. The CT scan showed at the Right most probably is: a. Hemangioma of the liver b. Cystic lesions in the liver most probably hydatid cysts c. Hepatoma d. Cholangiocarcinoma
21. The cystic lesion in the floor of the mouth like the one in the picture is: a. Retention cyst b. Hematoma c. Ranula d. Molar abscess 22. Mirizzi's syndrome: – a. Multiple cystic dilatation of CBD, causing obstruction b. Obstruction of biliary system due to malformation in the common bile duct (CBD) c. Large stones within the gallbladder may cause biliary obstruction by external compression. d. Obstruction of biliary system caused by Klatskin tumor 23. Most major bile duct injuries during llap cholecystectomy occur in patients under which one of the following cirucumstances:
147
a. Acute cholecystitis b. Gallstone pancreatitis c. Choledocholithiasis d. Elective cholecystectomy e. Laparoscopic procedure converted to open
24. The picture at the Rt is: a. Retracted lid b. Exophtalmus c. Ptosis d. lid lag
25. complications of blood transfusion include all except: a. urticaria b. Hypokalemia c.Hepatitis c d. ARDS e. Jaundice 26. parameters used to assess nutritional status include all except: a. Serum albumin b. Triceps skin fold thickness c.white cell account d. Hand grip strength e. Chest wall circumference 27. Deep Venous thrombosis is best diagnosed by : a. 121 fibrinogen b. CT scan c.Venogran d. Physical examination e. Doppler ultrasonography
28. Which of the following polyps is considered premalignant : a. adenomatous b. hamartomatous c. inflammatory d. hyperplastic 29. Six months child presented with reducible inguinal hernia, the best choice for him is: a. admit to hospital urgently and operate on b. advice his family to wait for possible disappearance of hernia at age of year c. wait until the age of year then have surgery d. have an appointment for surgery as soon as possible 30. Paget’s disease of the breast presents usually by : a. multiple lumps on the same side b. single big lump in upper outer quadrant
148
c. scaly, itchy, erosive area around the nipple d. milky discharge from both nipples 31. The commonest presentation of Meckel's diverticulum in an adult is: a- Gastroitestinal bleeding B- Gastrointestinal obstruction C- Intussuception D- Litter,s hernia E- Diverticulitis 32. The following are Complications of shock: a. Acute Respiratory failure b. Acute myocardial infarction c. Acute renal failure d. All of the above e. A&C only 33. The following are manifestations of hyperparathyroidism EXCPET: A. Recurrent pancreatitis B. Psychological disturbance C. Renal stones D. Gall stones 34. Complication of ERCP include A. Perforation B. Pancreatitis C. Hemorrhage D. All the Above E. A&C only 35. Complication of Crohn’s disease are: a. Intestinal obstruction b.Fistula c. Anal disease d. All of the above e. A&B only 36. Hashimoto’s thyroiditis : A) is of viral origin B) presents usually by nodular goiter C) can be diagnosed by thyroid antibodies and fine needle aspiration of thyroid D) is often pre malignant E) must be treated always by surgery 37. All of the following are signs of raised intracranial pressure Except: a) Headache. b) Vomiting. c) Papilledema. d) Aphasia. e) Bradycardia.
149
38. All the followings are Indications for central line insertion EXCPET: A. Massive fluid replacement B. Massive blood replacement *** C. Measurement of central venous pressure D. Prolonged Intervenes fluid therapy 39. Most common early complication of central venous line is: a) Sepsis b) Pneumothorax c) Thoracic duct injury d) Thrombosis 40. Small bowel obstruction often results in: (all correct except one) f) Hyperkalaemia. g)
Metabolic alkalosis.
h)
Oliguria.
i) Hypovolaemia. j) Severe dehydration. 41. Regarding Acute pancreatitis typically all of the following are correct except a)
Is accompanied by hypocalcaemia.
b)
Produces paralytic ileus.
c)
Is associated with a pleural effusion.
d)
Produces pyloric obstruction.
e)
Upper abdominal pain and vomiting.
42. Painless haematuria is the leading presentation of : a)
Renal cell carcinoma.
b)
Transitional cell carcinoma of the bladder .
c)
Ureteric stone.
d)
Pelvi-ureteric obstruction.
e)
Ureterocele.
43. The thyroid tumor which may be associated with pheochromocytoma is: a)
Papillary carcinoma.
b)
Medullary carcinoma.
c)
Follicular carcinoma.
d)
Anaplastic carcinoma.
e)
Malignant lymphoma.
44. ) Patients with major burns: a) Are in a negative nitrogen balance. b) Have normal calorie requirements. c) Do not generally become anaemic. d) Are resistant to septicaemia. e) All of the above. 45. Fiboadenomata of the breast:
150
a) Are commonest in early adult life. b) Are indiscrete and difficult to distinguish. c) Are usually painless. d) Resolve without treatment. e) A&C only. 46. Colonic polyps: all of the following are correct except : a) Are associated with colonic cancer. b) May be hereditary. c) Should not be removed if they are asymptomatic. d) May be hyperplastic. e) Are commonly adenomatous. 47. Lymphedema: a. May be Congenital b. Should be bilateral c. May be pitting in early stage d. A & C only e. None of the above 48. In acute appendicitis appendicitis: a. The risk of developing the illness is greatest in childhood b. Mortality increases with age and is greatest in the elderly c. Faecoliths are present in 75-80% of resected specimens d. Appendicitis is a possible diagnosis in the absence of abdominal tenderness e. all are true 49. About pyogenic liver abscess, all the following are TRUE Except: a. the etiology is unexplained in the majority of patients. b. the diagnosis is confirmed by aspiration for culture and sensitivity. c. treatment consists with antibiotics and aspiration. d. atypical clinical or radiological findings should raise the possibility of a necrotic neoplasm. e. the most common organism is Staphylococcus Aureus. 50. About hepatic adenomas, all the following are TRUE Except: A-females are more affected than males. B-incidence is increased by oral contraceptive pills. C-spontaneous regression is well recognized. D-usually they are asymptomatic and are discovered incidentally. E-they can rupture and as many as 25% are identified after an acute episode of hemorrhage. 51. which one of the following clinical signs or symptoms is not associated with serum sodium concentrations below 125 mEq/L? A.Headache B.Hallucinations C.Bradycardia D.Hypoventilation E.Hyperthermia 52) the metabolic rate increases by what percentage for each 1ºC elevation in body temperature? a. 1%
151
b. c. d. e.
5% 10% 15% 20%
53) Chronic adrenal insufficiency is characterized by which of the following? a. Hypothermia b.Hypertension c.Hyperkalemia d.Hypernatremia e.Hyperglycemia 54. Regarding peritonitis which of the following statements is not true? a. Primary peritonitis is commoner in children with nephrosis and adults with cirrhosis than in patients without such conditions b. Primary peritonitis is usually monomicrobial c. Chemical peritonitis often precedes bacterial contamination d. Multiple organisms are commonly cultured from peritoneal dialysis catheters e. Tuberculosis peritonitis has an insidious onset 55) Regarding metastatic cancer, which of the following statements is true? a. Axillary lymph node dissection is essential for staging a sarcoma of the breast b. Melanoma tends to metastasize first to the lung, brain and gas trointestinal tract c. Bone is frequently the site of metastasis for cancer of the breast and prostate d. Primary brain cancers have a predilection for metastasis to the lung e. None of the above 56) regarding acute supurative parotitis, which one of the following statements is not true? a. It usually occurs in elderly or debilitated patients b. Dehydration is a major contributing factor c. Immediate surgical drainage is mandatory d. The numerous vertically oriented fascial septa of the parotid space lead to multiloculated abscesses when infection e. S. aureas is the most frequent causative organism 57) which of the following studies are useful for the diagnosis of a pheochromocytoma? a. Clonidine suppression test b. 24-hr urine metanephrine levels c. 24-hr urine vanillylmandelic acid levels d. plasma catecholamine levels e. none of the above 58) Cushing's disease is caused by which of the following? a. Adrenal adenoma b. Adrenal carcinoma c. Pituitary adenoma d. Ectopic ACTH production e. none of the above 59) With Regard to primary gastric lymphoma, which of the following statements is true?
152
a. b. c. d. e.
G1 bleeding is the most common symptom Mucosal biopsy can establish the diagnosis in nearly all cases Primary therapy is surgical resection Primary therapy irradiation The long-term survival rate is equivalent to that for adenocarcinoma
60) which of the following statements regarding the pathogenesis of appendicitis is false? a. Luminal obstruction is always the cause of acute appendicitis b. Luminal obstruction leads to increased pressure and distention of the appendix c. Obstruction of venous outflow and then arterial inflow results in gangrene d. Obstruction of the lumen may occur from lymphoid hyperplasia, inspissated stool, or a foreign body e. Viral or bacterial infections can precede an episode of appendicitis 61) In Budd-chiari syndrome portal hypertension results from? a. Massive splenomegaly b. Cavernous hemangioma of the liver c. Hepatic vein obstruction d. Arterial venous fistula e. None of the above 62) laparoscopic cholecystectomy is most strongly contra indicated in? a. pregnancy b. prior upper abdominal surgery c. known common bile duct stones d. chronic obstructive pulmonary disease e. gallbladder cancer 63) which of the following is the best indication for preoperative ERCP in a patient with gallstones? a. Obstructive jaundice b. Gallstone pancreatitis c. History of jaundice d. Elevated alkaline phosphatase to twice normal e. A 10 mm common bile duct seen on ultrasonography 64) For which of the following congenital hemolytic anemias is splenectomy primarily indicated? i. Thalassemia ii. Hereditary Spherocytosis iii. Pyruvate kinase deficiency iv. Glucose-6-phosphate dehydrogenase deficiency v. Sickle cell anemia 65) Gastric acid secretion is stimulated by all of the following EXCEPT A. acetylcholine B. gastrin C. secretin D. antral distention E. gastric intraluminal protein 66) .For patients receiving total parenteral nutrition (TPN) as their only source of nutrition A. mortality is a direct result from complications of TPN
153
B. C. D. E.
fever should be treated with immediate removal of the intravenous access renal function and creatinine clearance remain stable calculous and acalculous cholecystitis is a long-term risk bone loss can be controlled with careful supplementation of vitamin D and calcium
67) All the following statements about paraoesophageal hernia are true except: A. The herniated portion of the stomach may become gangrenous. B. Surgical repair generally is indicated C. Heart burn is the usual chief complaint D. It can be life threatening. 68) The most common complication in a patient with aneurysm of the ascending thoracic
aorta is
A. Aortic valve insufficiency. B. Cardiac tamponade. C. Distal dissection. D. Superior vena cava obstruction 69) An 8-year-old boy has had recurrent painful swelling of a 2-cm mass in the midline of his neck just inferior to the hyoid bone. Which of the following statements best describes this lesion? A. Ectopic thyroid is present in approximately half of cases B. Surgical excision includes the pyramidal lobe of the thyroid C. The structure originates at the foramen cecum D. Fistula tracts drain laterally at the anterior border of the sternocleidomastoid muscle E. Simple excision may be accomplished with local anesthesia
70) Acute compartment syndrome is characterized by all of the following EXCEPT: A. pain on stretching the muscles. B. absent arterial pulsation’s. C. the presence of paresthesia or anesthesia. D. the presence of paresis or paralysis. E. can result after revascularisation of an acutely ischemic limb 71) Factors important in the formation of gallstones include a. cholesterol saturation of bile. b. gallbladder dysmotility. c. nucleating agents. d. obesity. e. All of the above
154
72) . Complications of untreated pancreatic pseudocysts include all of the following EXCEPT: a. gastrointestinal obstruction. b. pancreatic necrosis. c. free rupture. d. abscess. e. intracystic hemorrhage 73) .Clinical features of anal fissure include a. sever pain b. bleeding c. sentinel skin tag d. mucus disharge e. All of the above 74) Which one of the following causes elevation in serum amylase a. Perforated. duodenal ulcer b. Rupture Ectopic pregnancy c. Acute pancreatitis d. All of the above e. Only A&C 75) Primary varicose viens a. due to congenital valve incompetence b. due to competent perforators c. has higher complication rate than the secondary varicose d. usually lead to venous ulcer e. none of the above 76) What is the most common organism isolated from bile and blood cultures in patients with acute cholangitis? A. Enterobacter species B. Bacteroides species C. Escherichia coli D. Enterococcus species E. Candida albiccins 77) Von Willebrand’s disease: A. is an autosomal dominant disorder B. results in prolonged prothrombin time C. is associated with normal bleeding time D. is due to decreased hepatic synthesis of von Willebrand’s factor E. is typically associated with joint bleeding 78) The circulating level of which cytokine can be used as an indicator in response to surgical trauma: A. Interleukin-1 B. Interleukin-2 C. Interleukin-6
155
D. interleukin-8 E. tumor necrosis factor 79) M.E a 34 year-old underwent left modified radical mastectomy because of ductal carcinoma in the central retro areolar area. Pathology study result was: infiltrating duct carcinoma. Tumor of 3.2 X 2 X 1.8 cm. in the retro areolar area. The borders were free of tumor. There was 4 out of 18 lymph nodes with metastasis. ES & PR receptors were negative and Her 2 was positive +3. Pre op CT of the chest and abdomen were free as well as bone scan was with no evidence of metastasis. In which stage (TNM) you classified this tumor a. T2 N1 Mx b. T2 N2 M0 c. T3 N1 M0 d. T2 N3 M0 80) For the same patient in Question 79. Which protolo of treatment you advise: a. Chemotherapy + Radiotherapy + Herceptin b. Chemotherapy + Herceptin c. Chemotherapy + Tamoxifen d. Herceptin + Radiotherapy 1- False abut burns : sepsis due to catheterization is greater than wound infection 2-SIRS except: infection 3-Hypothermic coagulopathy: Normal PT &PTT – elevated PT & normal PTT – normal PT & elevated PTT – factor 7 deficiency –factor 8 deficiency 4- Patient with intermittent cardiac risk , going to intermediate or high risk procedure , the way to decrease preoperative morbidity & mortality : Beta blockers 5- False About BRCA : BRCA1 is higher grade than BRCA2 – BRCA1 is aneuploidy كل اسئلة البريست مش ذاكرين الخيارات 6-can cause DVT : Addison's – Pheocromocytoma – Cushing 7- False about breast cancer: Recurrence after breast conserving surgery is not a negative predictor factor ! يمكن 8- Minimal number of lymph nodes should be biopsied in breast conserving surgery: 1–2–3–4–5–6 9-Best to detect T & N in esophageal cancer: EUS 10- True about calculus cholycystitis: absent pain almost exclude the diagnosis 11-least cancer Metz to thyroid: RCC – melanoma –colorectal cancer – lung cancer 12- True about papillary thyroid carcinoma: neck radiation is a risk factor 13- Common in endemic goiters area: follicular thyroid cancer 14- Umbilical hernia: infantile type can wait 4 years 15- unable to pass NGT + gasless abdomen: proximal atresia without distal TEF 16- D-J flexure in the right (case of malrotation ) : need urgent laparotomy – Upper endoscopy – barium enema . 17- True about spleen anatomy : tail of pancreas is close to the spleen hilum 18- true about extra hepatic bile duct cancer : site of cancer determine the procedure 19- surgery of the most common type of choledical cyst : Roux-en-Y hepaticojejunostomy 20-management of wilms’ tumor: Resection followed by chemotherapy – chemotherapy followed by resection 21- Most common cause of massive Lower GI bleeding in elderly : diverticulosis 22-false about direct inguinal hernia: strangulation is common
156
23- All true about gastric lymphoma except: endoscopy used to relieve gastric ulcer 24- A 24 year old male patient who sustained a stab to the left arm, three hours later he was presented to the E&A with a tourniquet over his arm that had been placed by his family, his BP was 100/60 , tachycardic ,and oliguric, no external bleeding or hematomas, he was unable to move his cold limb, ipsilateral radial pulse was not felt, the best management would rather be: -
Left above elbow amputation.
-
brachial artery exploration.
-
Urgent angiogram.
-
Urgent arterial duplex.
-
Anticoagulation.
25- Cardiac cath- through brachial artery , after the procedure there was no radial pulse , what will you do : Brachial artery exploration and repair the injury
26- Elderly woman with prominent varicose veins in all of her leg, all true except : Stripping from groin to ankle has no risk to injure the saphenous nerve 27- A 65 year old male patient,diabetic,hypertensive,heavy smoker,on statins,he is being prepared for CABG,on preoperative investigation carotid duplex showed left internal carotid stenosis more than 75%,the patient has no history of TIAs or previous stroke,which statement is true: CAS (carotid artery stenting) is superior than CEA (carotid artery endarterectomy) regarding postoperative morbidity. In this case CEA or CAS should be done after CABG. In this case no need for CEA because the patient is neurologically asymptomatic. Carotid duplex is the golden standard investigation for carotid artery atherosclerotic diseases and it is superior than arch aortography. Dizziness can be a manifestation of carotid artery disease. 28- Not true about leg ulcers : ischemic ulcer has hyperkeratotic edge 29- Patient with leg pain for month , ulcer in the medial aspect of medial malleolus : Initially wrapping (stocking) & lift his leg 30- High risk of OPSI : 60 years old with hairy cell leukemia 31- Case of NEC (pneumatosis intestinalis ) : NGT, AVF, Antibiotics – 10% risk of perforation – 80% mortality rate
157
32- Increase Risk of malignancy in a nodule : male 33- True about cytokines : stored in vesicles – not present in normal 34- Good for wound epithelization: Occlusive Dressing – tight approximation of wound 35- Most common metz of colon cancer : Lymph nodes – peritoneal – vascular 36- Increase risk of malignancy in incidentaloma : size > 6 cm 37- 55 years with vague abdominal pain , Ct found 2.5 cm incidentaloma located right to adrenal gland : Observation – biopsy - biochemical profile 38- Orchiopexy to unilateral undescended testis: after 1 year. 39- About intermittent claudication : In rest pain and gangrene you should assess for revascularization 40- True about gastroschesis : 80% survival rate 41- Diaphragmatic hernia : mortality due to pulmonary hypoplasia
42- Most common cause of post operation pain : inadequate analgesia 43- All true about pressure ulcers except : stage 1 need debridement 44- Most reliable & cost effective in pressure ulcers : Clinical assessment – duplex 45- one unit of FFP represent about: 1:1,5 46- decrease wound healing except : obstructive jaundice 47- prognostic factors of papillary thyroid cancer except : AGE – Grade – thyroglobulin
158
48- patient with hypertension , AF , can't feel his leg or move it : acute embolic ischemia 49- Harsh sound after thyroidectomy : superior laryngeal N injury Choose THE BEST APPROPRIATE Answer in Each of the following Questions: 1. the MOST important finding in the diagnosis of acute appendicitis is: a. vomiting b. fever c. leukocytosis d. right lower quadrant tenderness e . positive Rovsing sign 2. The commonest cause of bloody nipple discharge is a. mammary ductectasia b. carcinoma of the breast c. lactational mastitis d. duct papilloma e. fibrocystic disease of the breast 3. Second degree burn: a. Heals in 10-21 days b. Painful c. Good capillary refill d. All the above
4. The following can cause Respiratory acidosis except: a. pre existing lung disease b. inadequate ventillation c. long upper abdominal incision d. all the above e. repeated vomiting
5. The classical picture of Acute arterial embolism include All the following except: a. Pallor b. Pain c. Parasthesia d. Impalpable Peripheral pulses e. Swelling 6. Severity of burns depends on: a. Depth of burn b. Total Body Surface Area (TBSA) c. Associated disease or injury d. All the above 7. Prophylactic antibiotics a. should be given one day before the operation
159
b. should be bacteriostatic c. usually narrow spectrum d. usually stopped 24hours after the operation e. none of the above 8. Clinical picture of varicose veins includes all the following except: a. Difficulty in standing b. Heaviness & fullness in legs c. Ankle swelling at the end of day d. Itching e. leg cramps at night 9 . Dialy requrment per kg for the adult includes all the following except a. Water 30-50ml b. Calories 30- 50 kcal c. Nitrogen 0.20-0.35g. d. Sodium 2-3mmol e. Potassium 0.7-0.9mmol 10. Patients on long term steroid treatment Going for surgery will need to: a. Stop treatment , on the morning of surgery b. Stop treatment three days before surgery c.Continue same treatment d. Increase the dose before surgery e. decrease the dose before surgery 11.Causes of metabolic acidosis include all the following Except : a. small bowel fistula b. shock c. severe anemia d. CO poisoning e. all of the above 12. Regarding obstructive jaundice a.caused by hemolytic anaemia b. urobilinogen is absent in the urine c. carcinoma of the head of pancreas is the commonest cause d. normal color stool e. b and c only 13. Carbuncle: a. is a confluence of several boils b. usually cause minimal tissue destruction, c. has one opening d. streptococcus is the commonest organism e. commonest site is the abdominal wall 14. Pilonidal sinus: a. less common in females b. Can occur in the umbilicus c. May affect barbers d. Sometimes accidentally discovered e. All the above 15. Hidradenitis suppurativa:
160
a. inflammation of the apocrine sweat glands b. common site is the axilla c. commoner in women d. all of the above e. none of the above 16. To obtain fine line scar: a.The incision should fall in the natural skin lines. b. Keep sutures for at least 14days c. Use absorbable sutures d. All the above 17. All the following are causes of post operative jaundice Except: a. Massive blood transfusion b. sepsis c. hyperperfusion d. .residual haematoma e. viral hepatitis 18. A perforated duodenal ulcer a.Usually lies on the anterior or superior surface of the duodenum. b.Usually presents with the acute onset of severe back pain. c.Produces radiological evidence of free gas in the peritoneum in over 90 percent of the patients. d.Is usually treated by vagotomy and pyloroplasty. e.Is usually treated conservatively. 19. The acute phase response includes: a. Hypothermia. b. Decreased plasma albumin. c. Hepatic sequestration of copper. d. Increased C-reactive protein. e. Neutrophil leucocytosis.
20.All regarding the flow phase after injury is true Except : a. Glucagon breaks down glycogen stores . b..hepatocytes produce glucose (gluconeogenesis). c. fat breakdown occurs as a result of catecholamine stimulation . d. growth hormone levels are elevated . e. hypoglycemia result as tissues deplete blood glucose stores. 21. levels of the following substances are elevated during the acute response to injury Except a.glucagon . b.insulin. c.catecholamine. d.glucocorticoids. e.growth hormone. 22. What is the commonest cause of low grade fever period immediately following abdominal surgery ? a. Atelectasis.
161
37.2-38.3c in the
48 hrours
b.Pulmonary embolism. c.Wound infection. d.Pseudomonas colitis. 23. Anaphylaxis is characterized by all of the following EXCEPT: F) is a reaction either local or general , frequently occurs within five minutes G) causes an urticarial eruption H) is produced by IgA antibody I) causes eosinophilia J) causes degranulation of basophils and mast cells 24. Shock can best be defined as: e. Hypotension. f. Hypoperfusion of tissues to meet the minimal requirements of cells g. Hypoxemia. h. All of the above. 25. Which of the following statements about extracellular fluid are true? f. The total extracellular fluid volume represents 40% of the body weight. g. The plasma volume constitutes one fourth of the total extracellular fluid volume . h. Potassium is the principal cation in extracellular fluid. i. The protein content of the plasma produces a lower concentration of cations than in the interstitial fluid. j. The interstitial fluid equilibrates slowly with the other body compartments. 26. In acute cholecystitis: a. Commonest bacteria is E.coli b. Gall bladder wall thickness more than 3 mm c. WBC between 10_15,000 cell/mm3 d. mildly elevated bilirubin level e. all of the above.
27. All of the following are true about neurogenic shock EXCEPT: a. There is a decrease in systemic vascular resistance and an increase in venous capacitance. b. Tachycardia or bradycardia may be observed, along with hypotension. c. The use of an alpha agonist such as phenylephrine is the mainstay of treatment . d. Severe head injury, spinal cord injury, and high spinal anesthesia may all cause neurogenic shock. e. A and B 28. Which of the following is not an action of angiotensin II? a. aldosterone secretion b. sodium absorption c. efferent arteriolar constriction d. arterial dilation e. nephrosclerosis in the kidney 29. Hemostasis and the cessation of bleeding require which of the following processes?
162
f. Adherence of platelets to exposed subendothelial glycoproteins and collagen with subsequent aggregation of platelets and formation of a hemostatic plug. g. Interaction of tissue factor with factor VII circulating in the plasma. h. The production of thrombin via the coagulation cascade with conversion of fibrinogen to fibrin. i. Cross-linking of fibrin by factor XIII. j. All of the above 30. Regarding polytrauma: a. Resuscitation in primary survey must start with airway care, breathing then circulation,.. b. X-ray after primary survey should be AP cervical spine, chest and pelvis. c. Cardiac tamponade is characterized by raised blood pressure and a low JVP. d. Assessment of uncomplicated limb fractures should occur during the primary survey. e. a and b only. 31. Hemostasis and the cessation of bleeding require which of the following processes? a. Adherence of platelets to exposed subendothelial glycoproteins and collagen with subsequent aggregation of platelets and formation of a hemostatic plug. b. Interaction of tissue factor with factor VII circulating in the plasma. c. The production of thrombin via the coagulation cascade with conversion of fibrinogen to fibrin. d. Cross-linking of fibrin by factor XIII. e. All of the above 32. Massive blood transfusion: a. is defined as replacement of at least one’s blood volume within the first 12 hours of resuscitation. b. can cause change in acid base balance c. DIC and coagulation defect is the most serious sequalae d. all are true 33. Regarding direct inguinal hernia: a. Common in young age b. Lies medial to inferior epigastric artery c. Internal ring test control it d. Complication is more than indirect hernia e. Bilateral in 20% 34. All of the following are functions of the gallbladder Except: a. absorption b. motor activity c. mucus secretion d. storage of bile e. formation of bile 35. Hypovolaemic shock is characterized by: a. A low central venous pressure , low cardiac output , low peripheral resistance b. A high central venous pressure , high cardiac output , low peripheral resistance c. A low central venous pressure , low cardiac output , high peripheral resistance d. A low central venous pressure , high cardiac output , high peripheral resistance e. A high central venous pressure , low cardiac output , low peripheral resistance
163
36. The Appendix all true except: f. Is typically less than 10 cm in length in the adult. g. Is located in the retrocaecal recess. h. Macburneys point, lies 2/3 laterally from a line from umbilicus to the anterior superior iliac spine. i. The longitudinal coat of the appendix is derived from the three bands of taenia coli. j. Is supplied by branches of the inferior mesenteric artery 37. Which of the following most often initiates the development of acute appendicitis? e. A viral infection. f. Acute gastroenteritis. g. Obstruction of the appendiceal lumen. h. A primary clostridial infection. 38. The inguinal canal: a. Has a deep ring, which is a defect in the transversalis fascia. b. Is bounded posteriorly by the inguinal ligament. c. Has the internal oblique as part of its posterior wall throughout. d. Has the conjoint tendon superiorly. e. Transmits the ilioinguinal nerve, which enters the canal through the deep ring. 39. Which of these statements regarding Meckle’s diverticulum is correct? a. It is found in about 4% of the population. b. Is always found on the antimesenteric border of the ileum. c. Is usually about 2 cm in length. d. Is usually located about 20 cm from the ileocaecal junction. e. Is usually attached to the umbilicus. 40. The most common type of congenital diaphragmatic hernia is caused by: a. A defect in the central tendon. b. Eventration of the diaphragm in the fetus. c. A defect through the space of Larrey. d. An abnormally wide esophageal hiatus. e. A defect through the pleuroperitoneal fold.
- Which statement best describes the planes of the abdomen? a. The transpyloric plane lies halfway between the xiphoid and the symphysis pubis. b. The transpyloric plane passes through the hilar of the kidneys. c. The subcostal plane is at the level of the body of L2. d. The iliac crests lie at the level of L5. e. The umbilicus usually lies at the level of the L4/L5 disc. Answer : b. The transpyloric plane also passes through the pylorus, the pancreatic neck, the duodenojejunal flexure and the fundus of the gall bladder. The spinal cord also ends at the level of the transpyloric plane. 2- Which statement about the regions of the abdomen is correct? a. The epigastrium lies medial to the mid-clavicular line and above the transpyloric plane. b. The suprapubic region lies between the midclavicular lines, the transpyloric plane and the intertubercular line. c. The iliac fossa lies lateral to the mid-clavicular line and above the intertubercular line.
164
d. The hypochondrium lies below the transpyloric plane and medial to the mid-clavicular line. e. The mid-clavicular line crosses the midpoint of the inguinal ligament. Answer : a. The abdomen is conventionally divided into nine regions by the two mid-clavicular lines running vertically, and the transpyloric plane and intertubercular plane running horizontally. The intertubercular plane joins the tubercles of the iliac crests. The epigastrium is in the position described relative to these lines. 3- Which statement correctly describes the abdominal wall? a. The superficial fascia of the abdominal wall contains Camper’s fascia. b. The deep fascia of the abdominal wall is known as Scarpa’s fascia. c. Scarpa’s fascia adheres to the superficial fascia of the thigh. d. The umbilicus receives sensory fibres from T8. e. The groin is innervated by T12. Answer : a. Camper’s fascia is the superficial fatty layer of the fascia of the abdominal wall. This fatty layer is continuous with the fat of the rest of the body. 4- Which statement about the rectus abdominis is correct? a. It has the transversalis fascia posteriorly throughout its length. b. It has three tendinous intersections which are visible posteriorly. c. It has the aponeuroses of the three oblique abdominal muscles anterior to it below the arcuate line. d. The lower free border of the anterior rectus sheath is called the arcuate line. e. The linea alba is a highly vascular structure. Answer : c. Below the arcuate line the aponeuroses of all three oblique muscles pass in front of the rectus abdominis. Above the arcuate line the external oblique aponeurosis passes anterior, the transverse oblique aponeurosis passes behind, and the internal oblique aponeurosis splits to enclose the rectus muscle. 5- Which statement about the oblique abdominal muscles is incorrect? a. The fibres of external oblique pass antero-inferiorly. b. The lower fibres of internal oblique form the inguinal ligament. c. The external oblique arises from the lower eight ribs. d. Internal oblique arises from the lumbar fascia. e. The internal oblique has a free lower border. Answer :b f. The lower border of the external oblique aponeurosis forms the inguinal ligament. 6- Which statement best completes this sentence? The inguinal canal: a. Has a deep ring, which is a defect in the transversus abdominis muscle. b. Is bounded posteriorly by the inguinal ligament. c. Has the internal oblique as part of its posterior wall throughout. d. Has the conjoint tendon superiorly. e. Transmits the ilioinguinal nerve, which enters the canal through the deep ring. Answer:d The conjoint tendon, comprising of the fused fibres of the internal and transverse oblique muscles, arches over the inguinal canal to attach to the pubic crest. 7- Which of the following statements about the peritoneum is correct?
165
a. The median umbilical fold contains the obliterated remnant of the umbilical artery. b. The greater omentum consists of four layers of peritoneum. c. The gastrosplenic ligament contains the splenic vessels. d. The lienorenal ligament contains the short gastric vessels. e. The lesser omentum connects the liver to the transverse colon. Answer : b. The greater omentum is formed by two double layers of peritoneum. The anterior two layers are continuous with the peritoneal layers enclosing the stomach. They then turn, double over and blend with the peritoneum of the transverse colon and mesocolon. 8- Which statement about the lesser sac is not correct? a. It is connected to the greater sac via the epiploic foramen. b. The stomach is related anteriorly. c. The pancreas is a posterior relation. d. The greater omentum is an anterior relation. e. The right border is formed by the lienorenal and gastrosplenic ligaments. Answer: e. These ligamentous structures form the left border of the lesser sac. 9- Which statement about the borders of the epiploic foramen is correct? a. The second part of the duodenum forms the inferior border. b. The quadrate process of the liver forms the superior border. c. The hepatic vein forms the posterior border. d. The free edge of the greater omentum forms the anterior border. e. The common bile duct is contained within the anterior border. Answer: e. The free border of the lesser omentum, which forms the anterior border, contains the hepatic artery and common bile duct anteriorly and the portal vein posteriorly. The hepatic artery is found on the left of the common bile duct. This arrangement of structures allows compression of the hepatic artery between finger and thumb (Pringle’s manoeuvre) to control bleeding from the cystic artery and the liver.
10- Which of the following statements regarding peritoneal compartments is correct? a. The infracolic compartment lies below the lesser omentum. b. The right and left subphrenic spaces are separated by the coronary ligament. c. The right subhepatic space lies between the right lobe of the liver and the right kidney. d. The left subhepatic space is also known as the hepatorenal pouch. e. The right paracolic gutter lies medial to the colon. Answer: c. This space communicates medially with the lesser sac via the epiploic foramen. The lateral border is the diaphragm. The superior border is the inferior border of the coronary ligament and the triangular ligament. 11- Which of these statements about the coeliac trunk is not correct? a. It supplies the foregut and its derivatives.
166
b. It leaves the aorta at the level of L1. c. It gives a left gastric branch that supplies the oesophagus. d. It gives a splenic branch. e. It gives rise to the gastroduodenal artery via its hepatic branch. Answer: b. The coeliac trunk arises from the aorta at the level of T12, a little below the median arcuate ligament. 12- Which statement best describes the blood supply of the stomach? a. It is derived entirely from the superior mesenteric artery. b. The gastroepiploic arteries supply the lesser curvature. c. The right gastric artery is a direct branch of the coeliac axis. d. The left gastroepiploic artery arises directly from the coeliac trunk. e. The short gastric arteries arise from the splenic artery. Answer: e. The short gastric arteries are variable in number. Most commonly six are present. They run in the gastrosplenic ligament to supply the lateral surface of the stomach. 13- Which statement best describes the venous drainage of the alimentary tract? a. All blood drains into the portal system. b. The portal vein is formed from the union of the inferior mesenteric and splenic veins. c. The superior mesenteric vein crosses the uncinate process of the pancreas. d. The inferior mesenteric vein passes behind the left renal vein. e. The prepyloric vein is variable in position. Answer: c. The superior mesenteric vein passes over the uncinate process of the pancreas before joining the splenic vein behind its neck. It is trapped in this position by the fusion of the dorsal and ventral pancreatic diverticulae during development of the pancreas.
14- Which statement best completes this sentence? The superior mesenteric artery: a. Supplies the gut from the pylorus to the terminal ileum. b. Arises from the aorta at the level of L1. c. Runs in front of the body of the pancreas. d. Crosses the second part of the duodenum. e. Supplies the appendix via its right colic branch. Answer: b. The superior mesenteric artery arises from the aorta at the level of L1 and then descends. It then passes behind the splenic vein and the body of the pancreas. 15- Which statement best describes the inferior mesenteric artery? a. It arises from the aorta at the level of the transpyloric plane. b. It supplies the mucus membrane of the gut as far as the mid-rectum.
167
c. It gives off a left colic branch. d. It crosses the pelvic brim at the point of bifurcation of the right common iliac vessels. e. It anastomoses with the superior mesenteric artery via its sigmoid branch. Answer: c. The left colic artery has ascending and descending branches. In a sigmoid colectomy the ascending branch is preserved to maintain the blood supply of the proximal descending colon. 16- Which statement best describes the lymphatics of the gastrointestinal tract? a. They generally follow routes which are distinct from those taken by the venous drainage of the bowel. b. Peyer’s patches are found on the mesenteric surface of the large bowel. c. Lymphoid follicles become less numerous in the distal part of the gut. d. Preaortic nodes lie at the origins of major blood vessels. e. Lymph from the alimentary tract eventually passes into the portal system of veins. Answer: d. There are coeliac, superior mesenteric and inferior mesenteric groups of lymph nodes which lie around the origins of the major blood vessels and drain lymph from their territories of supply. 17- Which statement best describes the stomach? a. Lymph from the superior 2/3 of the stomach drains into the suprapancreatic nodes. b. All lymph from the stomach drains through the coeliac nodes. c. The gastric branches of the vagi are given of in the greater curve. d. The lower oesophageal sphinchter is supplied by the nerves of Latarjet. e. The antral part of the stomach secretes an acid solution. Answer: b. All the lymph drainage from the stomach eventually drains into the coeliac nodes and then into the cisterna chyli.
18- Which of the following statements about the duodenum is incorrect? a. The second part overlies the right kidney. b. The transverse mesocolon attaches over the second part. c. The ampulla of Vater lies in the third part. d. The inferior vena cava and aorta lie directly behind the third part. e. The gall bladder overlies the first part. Answer: c. The ampulla of Vater opens into the second part of the duodenum. It opens onto an eminence called the duodenal papilla which is formed by the union of the common bile duct and pancreatic duct. 19- Which statement regarding the jejunum and ileum is incorrect? a. The ileum has thicker walls than the jejunum. b. The proximal small intestine is of greater diameter than the distal.
168
c. The mesentery of the small intestine is thicker distally. d. The jejunum lies mainly in the umbilical region. e. The mesenteric vessels form more numerous arcades in the ileum. Answer: a. The jejunum has thicker walls than the ileum as the valvulae conniventes are larger proximally. 20- Which of these statements regarding Meckel’s diverticulum is correct? a. It is found in about 4% of the population. b. Is always found on the antimesenteric border of the ileum. c. Is usually about 2 cm in length. d. Is usually located about 20 cm from the ileocaecal junction. e. Is usually attached to the umbilicus. Answer: b. Meckel’s diverticulum represents the proximal remnant of the embryonic yolk stalk, and as such is found at the site of attachment of the yolk stalk at the border of the intestine opposite its mesenteric attachment.
74- Acute arterial occlusion: A. Should be treated conservatively if the site of the occlusion is above the inguinal ligament. B. Demands the urgent use of vasodilator drugs. C. Of a limb is usually painless due to the anoxic damage produced in the peripheral nerves. D. May produce irreversible muscle necrosis after 6 hours. * E. B&C only. 75- Common sites for atheromatous arterial aneurysms are: A. The femoral artery. B. The middle cerebral artery. C. The abdominal aorta. D. Intrarenal. E. A&C only * 76-The long saphenous vein: A. Arises on the medial aspect of the sole of the foot. B. Passes 1cm in front of the medial malleolus. * C. Passes in front the knee joint. D. Enters the femoral sheath by piercing the fascia lata. E. Related to the sural nerve 78- In deep venous thrombosis of the lower limb: A. One of the most common sites of origin is the short saphenous vein. B. One of the common sites of origin is in the iliofemoral segment. * C. The diagnosis can usually be made by clinical examination. D. Tender swollen thrombosed veins are usually palpable. E. Usually associated with varicose vein.
169
17. 'rest pain' occurs: F. Anywhere in the body at rest G. In the thigh of a patient with Buerger's disease H. In the calf of a patient with intermittent claudicating I.
In the foot of a patient with severe vascular disease
J. In the back 18. ischaemia means: F. Pain in the ischial tuberosities G. Anaemia due to malignant seconderies in the ischial part of the pelvis H. Lack of blood flow I.
Increased blood flow
J. Polycythaemia K. The best initial therapy for deep venous thrombosis of the common femoral vein is: Choose one answer. a. warfarin b. streptokinase c. Heparin d. venous thrombectomy e. placement of a vena caval filter The first-choice diagnostic study for suspected deep venous thrombosis of the lower extremity is: Choose one answer. a. b. c. d. e.
real-time Doppler imaging contrast sonography radioactive labeled fibrinogen uptake impedance plethysmography isotope injection with gamma scintillation scanning
Regarding veins of lower limbs all are true except: a. Valves allow flow from deep to superficial system Venous return from lower limbs is aided by respiratory movements b. Superficial veins lie in subcutaneous tissue c. The pressure in veins of the foot while standing is 100mmHg d. a) Stasis of blood is important factor in developing varicose veins Most common complication of central venous access is: Choose one answer.
170
a. b. c. d. e.
major artery damage. Catheter problems. Thrombosis of central vein Catheter related sepsis Pleural space damage, pneumothorax
The classical picture of Acute arterial embolism include all the following except: Choose one answer. a. peripheral pulses b. Parasthesia c. All the above d. Pain e. Pallor 2. Which hypersensitivity reaction is associated with a tuberculin reaction? a. Type I: immediate b. Type II: cytotoxic c. Type III: immune complex d. Type IV: cell mediated 3. The most common location for a gastric ulcer is a. Fundus b. Greater curvature c. Cardia d. Body e. Antrum 4. Regarding the management of major trauma a. Deaths follow a trimodal distribution b. Cardiac tamponade is characterised by raised BP, low JVP and muffled heart sounds c.
Assessment of uncomplicated limb fractures should occur during the primary survey
d.
Deterioration of the casualty during the primary survey should lead to the secondary survey
e. All are false 5. All of the following are true about neurogenic shock except: a. There is a decrease in systemic vascular resistance and an increase in venous capacitance. b. Tachycardia or bradycardia may be observed, along with hypotension. c. The use of an alpha agonist such as phenylephrine is the mainstay of treatment. d. Severe head injury, spinal cord injury, and high spinal anesthesia may all cause neurogenic shock. 6. The following cause hypercalcaemia except :
171
a.
Sarcoidosis
b.
Primary hyperparao]thyroidism
c.
Acute pancreatitis
d.
Metastatic bronchial carcinoma
e.
Milk-Alkali syndrome
7. For a 40-kg baby the maintenance daily fluid requirement is approximately which of the following? a. 1100 ml b. 1250 ml c. 1550 ml d. 1700 ml e. 2000 ml 8. . Infantile hypertrophic pyloric stenosis a.
Occurs with a male: female ratio of 4:1.
b. Presents between six and eight months of age c.
Typically presents with bile stained projectile vomiting
d.
Surgical treatment is by Heller's Cardiomyotomy
e. all are false 8. Which of the following do you consider to be the most important clinical sign in acute appendicitis, 1. 2. 3. 4. 5.
Abdominal tenderness around the umbilicus Abdominal tenderness in the RIF Tenderness over McBurney’s point Rovsing’s sign positive Suprapubic tenderness
9.Heparin 1. 2. 3. 4. 5.
Acts as an anti-platelet Acts as an anti-thromboplastin Acts as an antithrombin All of the above None of the above
10. All of the following are mechanisms of urinary calculi formation except, 1. 2. 3. 4. 5.
Hypoparathyroidism Prolonged recumbency Infection with urea-splitting organisms Foreign body Urinary tract obstruction
11. Which of the following are not found in peritonitis?
172
1. 2. 3. 4. 5.
Patient is lying still Guarding Rebound tenderness Hyperactive bowel sounds Rigid abdomen
12-Which one of the following suggest a diagnosis of Hirschsprung's disease? f. g. h. i. j.
A contrast-study showing dilatation of the aganglionic bowel segment. Early presentation with vomiting. Neonatal large bowel obstruction. Presentation after 1 year of age. Red current jelly stools.
13-Which of the following regarding the anatomy of the heart is true? f. The aortic valve is tricuspid. g. The ascending aorta is entirely outside the pericardial sac. h. The left atrial appendage is identified readily by transthoracic echocardiography. i. The pulmonary trunk lies anterior to the ascending aorta. j. The right atrium is posterior to the left atrium. 14-Which of the following is true concerning Scaphoid fractures? f. Rarely occur in young adults g. when complicated by avascular necrosis the proximal pole is usually affected h. should be treated by bone grafting and internal fixation even if undisplaced i. j.
wrist fractures are uncommon anteriorposterior and lateral radiographs reveal most fractures
15-Which of the following statements is true of upper limb nerve injuries? f. Injury to the median nerve results in a wrist drop g. Injury to the radial nerve results in loss of sensation over the palmar aspect of the index finger h. Injury to the median nerve results in loss of sensation in the anatomical snuffbox i. Injury to the ulnar nerve results in a claw hand j. Injury to the ulnar nerve results in loss of sensation over the thumb 16- Regarding intravenous solutions:
173
e. f. g. h. i.
Normal saline contains 180mmol/l of sodium Ringer's lactate solutions is designed for intracellular fluid replacement Sodium bicarbonate 8.4% is a hyperosmolar solution Normal saline with added potassium is appropriate therapy to correct a respiratory alkalosis All are True
17-Which of the following concerning the Femoral sheath is false: e. f. g. h. i.
Contains the femoral artery Contains lymph nodes Contains the femoral canal Contains femoral nerve Contains the femoral vein
18-The following is true of the spleen: f. g. h. i. j.
Is the largest lymphoid organ in the body Lies obliquely between the seventh and tenth rib The lower pole extends beyond the mid-axillary line Is usually palpable when normal Usually measures 16cm in maximum length when healthy
19-Breast cancer risk is increased in association with the following factors except: f. g. h. i. j.
Nulliparity Immediately after pregnancy Early menarche Early age at first pregnancy Late menopause
20-In tension pneumothorax the following signs are present except: k. Hypoxia l. Hyperresonance to percussion on the affected side m. Tracheal deviation to the ipsilateral side n. Distended neck veins o. Tachycardia 21- The most common hernia in females is: f. g. h. i.
174
Femoral hernia. Direct inguinal hernia. Indirect inguinal hernia. Obturator hernia.
j.
Umbilical hernia.
22-. The most helpful diagnostic radiographic procedure in small bowel obstruction is: e. f. g. h. i.
CT of the abdomen. Contrast study of the intestine. Supine and erect x-rays of the abdomen . Ultrasonography of the abdomen. MRI Abdomen
23- In role of nine extent of burn if entire trunk is burned it will be equal to: f. g. h. i. j.
9% body surface area. 18% body surface area. 36% body surface area. 27% body surface area. 45% body surface area.
24-. If torsion of the testicle is suspected, surgical exploration: e. f. g. h.
Can be delayed 24 hours and limited to the affected side. Can be delayed but should include the asymptomatic side. Should be immediate and limited to the affected side. Should be immediate and include the asymptomatic side .
25- Hyperthyroidism can be caused by all of the following except: f. g. h. i. j.
Graves' disease. Plummer's disease. Struma ovarii. Hashimoto's disease. Medullary carcinoma of the thyroid.
26- A 9 month old boy presents with an acute scrotal swelling. The following diagnoses are likely: a. b. c. d. e.
Epididymitis Orchitis Torsion of the testicular appendage Irreducible inguinal hernia Acute idiopathic scrotal oedema
27. The evaluation of a comatose patient with a head injury begins with: a.The cardiovascular system. b. Pupillary reflexes. c. Establishment of an airway. d. Computed tomography (CT) of the brain
175
e .insertion of Intravenous canul 28. The following is an indication for thoracotomy in chest injury, k. Cardiac tamponade l. Uncontrolled pulmonary air leakage m. Perforation of thoracic esophagus n. Blood loss of 200ml/hr for 2-3 hrs via chest tube o. All of the above 29. Regarding Gallstones all of the following are true except:f. g. h. i. j.
Prevalence increases with advancing age 30% of gallstones are radio-opaque Cholesterol stones result from a change in solubility of bile constituents Biliary infection, stasis and changes in gallbladder function can precipitate stone formation Gram-negative organisms are the most common isolated
30.In obstructive jaundice: a. Urinary conjugated bilirubin is increased b. Serum unconjugated bilirubin is increased c. Urinary urobilinogen increased d. Serum conjugated bilirubin is reduced e. Faecal stercobilinogen is increased -
causes of acute pancrititis all except : hyperkalemia (T) - abd.
Trauma 2-
febrile convulsion true is : if reccur give prophylactic
anticonvulsant - common in neonate - occur early in disease at rise of fever 3-
cause of sudden death in MI : ventricular fibrilation (T) - heart
block – 4-
infective endocarditis : forget it but one choice was about
prosthetic valve 5-
criteria for dignosing infective endocarditis include : ????
6-
case : female with hemoglobin 18 gm could be due to any cause
except : COPD - hemochromatosis(T) – 7-
best managment to prevent renal impairment in toxic shock give
: manitol - restore effective blood volume (T) - give antibiotic against the causitive organism – 8-
prolongest duration of action in penicillins is that of : procain
penicillin - penic. G -penic. V - benzathin penic.? – methacillin
176
9-
suspect neural tube defect in fetus , what investigation confirm
your diagnosis : estradiol - alpha feto protein (T) – 10-
ABO incombatiblity will cause (?? or all except ??): hydrops -
severe hemolysis – 11-
70 years old man presented with fatigue & ...Hemoglobin 8gm
he have had suddenly black stool in the day before , what to do : endoscopy - blood transfusion (T) – 12-
causes of lymphadenitis, all except: cat scratch disease - neman
pick disease - leukemia – 13-
Brucella can cause all except : back pain - gastroenteritis -
splenomegaly - hepatomegaly – 14-
contraindication of lumbar puncture in a neonate (or infant ?) :
skin infection – pulging fontanelle – 15-
patient have bradycardia and salivation,you give him: atropin (T)
– 16-
case: old man was found comatosed in his gardin betwenn empty
cans صفايحح,with bradycardia and frothy mouth , suspected to be poisoned by : organophosphorus - pyrethroid..? – 17-
case: boy with machinary murmur and (poited?) apex : VSD - PDA
(T)18-
case: boy apears healthy and have ejection systolic murmur
heared maximumly on the upper left sternal border ,murmur caused by: moderately severe aortic stenosis - severe pulmonary stenosis (T) 19-
case: male pt presented with fracture femur , on hospital he
have dyspnea and deteriorated blood gases,diagnosed to have pulmonary embolism, what you do first : ventilation - heparin - IV fluid – 20-
all are causes of metabolic acidosis except : severe vomiting (T) -
diarrhea – 21-
case: thin pt with Hx of sudden Rt. chest pain, mild dyspnea ,
impaired air entery in the Rt. lung feild, what investigation you ask : expiration erect chest x ray - supine inspiration chest x ray - CT chest – 22-
same case : what is the diagnosis for the previous case : simple
pneumothorax (T) - lung colapse – 23-
case of cushing disease: what radiological imaging you will ask :
MRI brain & pituitary gland (T) - CT brain & pituitary gland –
177
24-
case: child present with cough and dyspnea and Hx of forign
body inhalation ,where do you think the forign body will be: pharynx larynx - trachea - trachiobronchial tree – 25-
In differentiation between IBD ( Crohns dis. & Ulcerative colitis ) :
both can cause oral & gastroduodenal ulcer - crohns dis. invade the full thickness of small & large intestine – 26-
case: boy have asymptomatic amebiasis : give metronedazole -
do nothing – 27-
development: 10 months old baby can do all except : move
hands(bye bye) - says names of 3 colours (T) - pulls himself up to standing position - sit unsupported – 28-
Growth:head circumfrence between 9th and 12th month will
increases every month by: 0.5cm(T) - 1cm - 2cm - 1.5cm 29-
causes of cerebral palsy : brain anoxia - bilirubin encephalopathy
- intrauterine infection - all of above – 30-
most sensetive test to diagnose primary hypothiroidism is: TSH
(T) - Total T4 - Free T3 – 31-
best antibiotic against mycoplasma infection is: Erythromycin (T)
– 32-
commonest organism causes comunity aquired pneumonia is :
strept. pneumoniae - staph - H influenzae - viral – 33-
Effects of hypokalaemia on heart include all except : peaked T
wave (T) – 34-
manifestations of 3rd CN (occulomotor nerve) palsy include all
except: ptosis - absent light reflex - failure of lateral gaze (T) – 35-
all are indications of Renal dialysis except: Hyperkalemia -
pulmonary edema - hypocalcemia(T) – 36-
In asthmatic pt on theophylin ,what drug can decrease?? level of
theophylin in blood: erthromycin – 37-
all manifested in pt with scarlet fever except: white strawperry
tongue - red strawperry tongue - maculopapular? skin rash periauricular lymph nodes enlargement 38-
all are cause of macrosytosis except: iron defficiency anemia(T) -
B12 defficiency anemia – 39-
case: 16 y old boy with minimal change nephrotic, what you
suspect the prognosis: full recovery - persistent hypertension persistent proteinuria - persistent renal impairment –
178
40-
commonest organism causes bronchiolitis is: RSV(respiratory
sinsitial virus)(T) – 41-
in CSF examination of pt with Guillian Barre syndrome you will
find: increased protein and normal WBC (T) - icreased protein & decseased glucose - increased cells mostly lymphocytes?? – 42-
Case: suspected DVT (pain in calf,....) what investigation you ask:
doppler US (T) - MRI – 43-
case: cystic fibrosis and (pancrititis??) complains of loose stool
since months: replase pancreatic amylase(T) - fat free diet - iron supplementation – 44-
spontaneous primary peritoneatis: give gentamycin - culture of
the ascitic fluid - blood culture – 45-
all are causes of clubbing except : cystic fibrosis - bronchial
asthma (T) - bronchiectasis – 46-
all are manifestitions of upper motor neuron lesion except:
hyper.reflexia - hypertonia - positive babiniski sign - weakness – tremors 47-
case:boy with VSD: surgery if cause cyanosis –
48-
all are causes of hypercalcaemia except: milk alkali - sarcoidosis
-? 49-
all are X resessive inherited except: diabetis insepidus(T) -
thalasemia A - thalasemia B - congenital hypophosphatemic rickets – 50-
defficient compliment 1 leads to increased infections by: bacteria
- viruses - protosoa - fungus – 51-
??? months old boy have neck rigidity and fever ,CSF clowdy ???
and sent for culture ,bacterial meningitis was diagnosed ,what you give him: ceftriaxon - ceftriaxon and vancomycin - wait for culture – 52-
case: female presented with progressive dysphagia over years
(for liquids and solid food??) , diagnosis is: Achalasia - cancer oesophagus - esophageal pouch – 53-
absolute contraindications of streptokinase in pt with acute
myocardial infarction include all except:previous ischemic stroke previous haemorrhagic stroke - active internal bleeding – 54-
causes wheesy chest: bronchiolitis - inhaled forign body -
aspiration of gastric secretion - all of above (T) – 55-
what is support diagnosis of DKA (diabetic ketoacidosis): poor
apetite in the previous day - bicarb 10 - shallow respiration hyperglycaemia –
179
56-
oral anticoagulant: action continue 6 hours after stopage - none
of above – 57-
drugs that contraindicated in patient with bronchial asthma: B
blockers(T) – 58-
vaccins that could?? be given to child aged less than 18 months:
hepatitis A - influenza - varicella - polysaccharide pneumococcal vaccin - congugated neumococcal vaccine – 59-
causes of failure to thrive (or short stature???) (?all except?):
dawn syndrome – 60-
complications of measles (??or mumps) include all except:
orchitis – 61-
toxoplasmosis , caused by : eating raw meat (T) (no cats in
choices) 62-
NSAID therapy will prololong: bleeding time (T) - PT - INR -
coagulation time 63-
Umbilical artery carry : Deoxygenated blood to placenta (T)
64-
best prevention of gas gangrene is: clean & Debridement (T) - Ig
– 65-
one of Anaerobes: Bacteroids (T) - Klebsiella - E Coli -
pseudomonas – 66-
3rd cranial nerve palsy causes all except : ptosis - absent light
reflex - lost lateral gaze (T) 67-
case of preeclampsia complicated by eclampsia (tonic clonic
convulsion..), first step: magnesium phosphate (T) - lowering blood presure – 68-
pregnant with cord prolapse present at home,what should do: try
to entroduce cord into vagina - do nothing till reach hospital (T) – 69-
the second commonest cause of bleeding from single duct in the
niple after duct ectasia : ductal carcinoma (T) 70-
recurrent laryngeal nerve is corresponding to: inferior thyroid
artery (T) 71-
Hyperkalemia causing all of the following except: muscle rigidity
(T) 72-
Acute pancrititis manifestations include all the following except:
colicy pain (T) - vomiting – 73-
Incombitant cervix diagnosed in th week of pregnancy: suture
cervix this week - suture it at 14th - 16th week (T) –
180
74-
female with last menarche before 4 weeks need to be sure if she
is pregnant: do US - do pregnancy serum test (T) - she is not pregnant – 75-
labour is : regular contractions causes progressive dilatation of
cervix (T) – 76-
early and constant feature of thyrotoxicosis is: tachycardia (T) –
77-
complications of total thyroidectomy include all of the following
except: hypercalcemia - hypocacemia – 78-
in tetany the first step in management is : calcium gluconate IV
(T) - calcitonin – 79-
most major health problem after menopause : osteuporosis -
depression - cancer endometrium – 80-
nephroblastoma presents mostly by: abdominal mass (T) –
81-
commonest indication of surgery in a case of acute divirticullitis:
hemorrhage - perforation - intestinal obstruction – 82-
commonest cause of massive bleeding per rectum after
diverticulitis: Angiodysplasia 83-
severe amenorrhea ,what should you do: US –
84-
case: bleeding per vagina in a pregnant female with uterine
contractions and 3cm cervical dilatation and intact membrane,US show retroplacental bleeding? what to do: CS (T) - induction of labour – 85-
true about case of endomertiosis: can cause infertility by
blocking falobian tube (T) – 86-
Glasco coma scale except: pupil (T) –
87-
all true about gallstones except: obaque stones are more
common (T) – 88-
ureteric obstruction: commonest cause is renal (stone) (T) -
elevate urea and creatinine – 89-
intermitent claudication all true except: more agressive at night
(T) - caused by ischemia.. – 90-
case: edema in one leg that increase by the end of the day what
to do: doppler US – 91-
causes of epistaxis: tauma - perforated septum - ? - a & c - all of
above (T) 92-
fracture nose before 1 month: rinoplasty me be needed - have to
come back after 6 months – 93-
181
about melanoma prognosis: clark classification is helpful –
94-
helpful drug for infertility of polycystic ovary disease: metformin
(T) – 95-
organism we care about after splenectomy: pneumococci –
96-
trauma of internal organ that could be managed conservatively
include of all following except: kidney laceration - intestine perforation - liver laceration - spleen hematoma – 97-
case of glaucoma all true except: presents with mydriasis? -
abdominal pain - headache - give mitotic eye drops and refer to ophthalmologist – vomiting 98-
true about crohn's disease : improved with smoking - more in
temperate than tropical climate - ?? in jews and asians - tends to runs in famillies - more in female 99-
character of a vessel: compressable - transilumination of light -
fluctuate – reducable 100-
dipstick test for glucosuria is positive in a pregnant woman,
what to do: she have DM - this is normal in a pregnant woman – 101-
cause of secondary amenorrhea all except: PCO - premature
ovarian failure - asherman – 102-
ITP (idiopathic thrombocytopenic purpura), all true except:
marked splenomegaly (T) – 103-
antipartum hemorrhage: increase risk of postpartum
hemorrhage - any bleeding after the 24th week of pregnancy – 104-
most accurate investigation in dignosis of acute cholecystitis:
HIDA - US – 105-
laparoscobic cholycystectomy: offer faster recovery and less
pain – 106-
rupture of uterus manifestation : fetal distress - uterine
contracions - abdominal pain - vaginal bleeding - all of above 107-
a clean contaminated surgery: ureter lethiotomy - appendicular
abscess – 108-
fever in first 2 days after surgery is mostly due to : drug fever -
pulmonary complication - UTI – 109-
a man shoted in his left chest, his pressure is low and have
tachycardia, and engurged neck veins, what to do: thoracotomy needle decompression – peicardiotomy 110-
patient with dysphagia after ingestion of a forign body need to
do : osophageoscopy - pharyngeoscopy - hypopharyngeoscopy bronchoscopy –
182
111-
indication of thoracotomy :
112-
true about postsplenectomy overwhelming sepsis: mortality
rate 50% 113-
comatosed patient after head injury what to do first: airway -
evaluation ? – 114-
complications of massive blood transfusion include all except :
hyperkalemia - hypercalcemia (T) – 115-
severe vomiting in pyloric stenosis is best corrected by :
isotonic fluid with NaCl - isotonic fluid with decstrose - ringer lactate – 116-
placenta previa (diagnosed surely? by ): US - CS –
117-
cataract is caused by all except: US - trauma - radiation –
118-
acute appendisitis is diagnosed by: US - clinical examination -
ESR – 119-
risk of surgical infection increases in all except: controled DM –
120-
most overlooked hernia is : inguinal - femoral - umbilical –
121-
+ve pregnancy test in a woman with IUCD (intrauterine
contraceptive device): keep IUCD if she want to the baby - remove IUCD if she dont want the baby - keep IUCD whatever she want – 122-
ectopic pregnancy: more dangerous if occur in the isthmus -
uterus enlarged – 123-
bad in cancer breast prognosis: age - LN metastasis –
124-
measures differ between thermal burns & high voltage
electrical shock: fasciotomy - cardiac compression (T) - amputation renal failure – 125-
all increase risk of cancer endometrium except: DM - early
menarche & late menopause?? - tamoxifane ?? – 126-
lobular carcinoma in sito is managed by: resection & radiation -
hormonal therapy - total mastectomy - mirpr imaging for the opposite braest – 127-
true about follicular carcinoma ,??: multicentric - send
metastases through blood - prefer metastasis to bone – 128-
DM in pregnant female will increeases risk of all of the
following except: birth canal injury - PPH – 129-
in diabetic mother, effect of DM on fetus include all except:
hypercalcemia (T) - macrosomia? - IUGR ? - hypoglycemia ? – 130-
all increases risk of PPH except: prolonged labour - rapid
labour - polyhydramenous - DM –
183
131-
true about induction of labour: Bishop scores used to
determine favourability of induction of labour - 6 bishop scores mean that cervix is favourable for ?? labour – 132-
all increases risk of showlder dystocia except:
oligohydramenous - obesity - macrosomia?DM – 133-
by PV examination in a pregnant woman in labour, you feel
mouth and nose?? of fetus: brow presentation - omento? transverse position ? - vertical ? – 134-
patient with polytrauma, to maintain air way: suction of debris?
in mouth and elevate mandible - coughed? mask - tracheostomy – 135-
chest tube put to a patient after chest trauma ?,next step will
be: chest x ray? - Ventelation? – 136-
peritonitis ?? ,manifestations are all except: guarding -
rebound tenderness - increased bowel sounds – 137-
4 days ?? urticaria in woman : topical steroids - oral
prednisolone - ??(antihistamine) – 138A) BASIC SURGICAL SCIENCES DIRECTIONS: Each of the following questions contains Five suggested answers. :Select the Best response to each question 2) Abduction of the vocal cords results from contraction of the: A) crico-thyroid muscles B) posterior crico-arytenoid muscles C) vocalis muscles D) thyro-ary-epiglottic muscles E) lateral crico-arytenoid and transverse arytenoids muscles 3) The rectum: A) is devoid of peritoneum B) is surrounded by peritoneum C) has peritoneum on its lateral surfaces for its upper two- thirds, and on its anterior surface for its upper one-third D) has pritoneum on its anterior surface for its upper two-thirds, and on its lateral surfaces for its upper one-third E) has peritoneum on its anterior surface only 4) The umbilicus: A) lies near the to the xiphoid than to the pubis B) derives its cutaneous innervation from the eleventh thoracic nerve C) transmits, during development, the umbilical cord two arteries and two veins D) usually lies at about the level between the third and fourth lumbar vertebra E) emberiologicall, may transmit urine but never bowel contents 5) The superficial perineal pouch:
184
A) is limited inferiorly by the urogenital diaphragm B) is not continuous with the space in the scrotum occupied by the testes C) has a membranous covering which provides a fascial sheath around the penis D) is traversedby the urethera in the male but not the urethera and vagina in the female E) in the female, the greater vestibular glands are situated outside this pouch 6) The tongue: A) has a foramen caecum at the base of the frenulum B) is separated from the epiglottis by the valleculae on each side of the midline C) has 7-12 circumvallate papillae situated behind the sulcus terminalis D) is attached to the hyoid bone by the genioglossus muscle E) is supplied only by hypoglossal nerve 7) Hypovolaemic shock is characterized by: A) a low central venous pressure , low cardiac output , low peripheral resistance B) a high central venous pressure , high cardiac output , low peripheral resistance C) a low central venoys pressure , low cardiac output , high periphera resistance D) a low central venous pressure , high cardiac output , high peripheral resistance E) a high central venous pressure , low cardiac output , low peripheral resistance 8) An oxygen debt is: A) the amount of oxygen in excess of the resting metabolic needs that must be consumed after completion of exercise B) build up because the pulmonary capillaries limit the uptake of Oxygen at high rates of oxygen consumption C) related to the fact that skeletal muscle cannot function temporarily in the absence of oxygen D) associated with a decrease in blood lactate E) associated with alkalosis 9) Pulmonary embolism may be a complication of: A) prolonged bed rest B) a surgical operation C) vitamine K deficiency D) oral contraceptive therapy E) Antithrombin III deficiency 10) Which of the following statements regarding potassium metabolism is NOT :?True A) potassium deficiency commonly results from thiazide diuretic theraoy B) the normal compensation for potassium deficiency is a metabolic extracellular acidosis C) aldosterone increases urinary potassium loss D) hyperkalaemia causes bradycardia and loss of P waves on the ECG E) hypokalaemia aggrevates the cardiac effects of digitalis toxicity 11) Cutaneous pain: A) is due to overstimulation of receptors serving other sensory modalities B) cannot be elicited more readily if the tissue has recently been injured C) is due to exitation of receptors by pain-producing chemical substances in the injured tissue D) shows marked adaptation, i. e. decrease in severity in response to a constant stimulus E) is conducted through the medial spinothalamic tract
185
12) Which of the following is NOT associated with hyperthyroidism?: A) increase size of the thyroid gland B) increased amount of colloid in thyroid follicle C) increased height of epithelium of the thyroid follicle D) increased vascularity of the thyroid gland E) increased uptake of iodine by the thyroid gland 13) Sarcomata may show all of the following EXCEPT: A) production of myxomatous tissue B) production of collagen C) spindle shaped cells D) signet ring cells E) blood stream metastasis 14) Anaphylaxis is characterized by all of the following EXCEPT: A) is a reaction either local or general , frequently occurs within five minutes B) causes an urticarial eruption C) is produced by IgA antibody D) causes eosinophilia E) causes degranulation of basophils and mast cells 15) Autoimmunity is characterized by the following EXCEPT: A) occurs because of a breakdown in the ability of the body to distinguish between self and non self B) is involved in some forms of orchitis C) is involved in formation of cryo globulin D) is important in the pathogenesis of lupus erytheromatosus E) does not result in immune complex disease 16) Pseudomembranous enterocolitis is caused by the following organisms: A) Clostridium sporogenes B) Clostridium defficile C) Streptococcus faecalis D) Penicillin sensitive staphylocci E) Pseudomonas aeruginos B) CLINICAL SUGERY 17) The “ white clot syndrome”: A) is usually characterized with antithrombin III deficiency B) most often present with arterial complicatios of heparin induced throbocytopenin C) is best managed by loe molecular weight dextran D) is best managed by halving the therapeutic dose of heparin sodium E) results from nitric oxide deficiency of endothelial cells
A 21-year-old man who was the driver in a head-on collision has a pulse of (18 140/min , respiratory rate of 36 and blood pressure of 75 palpable. His trachea is deviated to the left, with palpable subcutaneous emphysema and poor breath sounds in the right hemithorax, The most appropriate initial treatment must be A) immediate thoracotomy B) catheter insertion in the subclavian vein for fluid resuscitation C) intubation and ventilation D) tube thoracostomy E) immediate tracheostomy
186
:The best test to monitor the adequacy of levothyroxin therapy is (18 F) radioactive iodine uptake G) thyroglobulin H) free thyroxine index (T4) I) triiodothyronine resin uptake (T3) J) thyroid stimulating hormone (TSH) 19) Which of the following statements about fungal infection is NOT true ?: A) Prior or synchronous culture positive for Candida at another site occurs in few patients with candidimia B) For critically ill patients nonhaematogenous sites of candida are appropriately treated with systemic antifungal therapy C) Mortality rates are similar regardlss of whether C. albicans fungmia is treated with amphotericin B or fluconazole D) Intravenous catheters and the gastrointestinal tract are common portals for Candida to gain blood stream access E) Septic emboli are more common with fungal endocarditis than with bacterial endocarditis 20) The maximum safe dose of local anaesthetic administered subcutaneously in a 70kg man is: A) 10 to 20 ml of 1% lidocaine B) 40 to 50ml oh 2% lidocaine with epinephrine C) 40 to 50 ml of 1% lidcaine with epinephrine D) 40 to 50 ml of 1% bupivacaine (marcaine) E) 40 to 50 ml of 1%lidocaine without epinephrine 21) Two days after right hemicolectomy for a Dukes B caecal carcinoma , the Patient complains of sharp right-sided chest pain and dyspnea. HisPaO2 Is 64mmHg ,his PaCo2 is 32mmHg. CVP is 26 cm water, and the blood ,pressure is 102/78mmHg. A pulmonary embolus is suspected :The next step in management should be A) A ventilation- perfusion lung scan B) A pulmonary arteriogram C) Postrioanterior and lateral chest x-rays D) Heparin sodium ,100 units/kg intravenously E) Immediate duplex scanning of both lower extremities 22) The major cause of graft loss in heart and kidney allograft is: A) acute rejection B) hyperacute rejection C) vascular thrombosis D) chronic rejection E) graft infection 23) All of the following are indicators of tumor aggressiveness and poor outcome for papillary carcinoma of the thyroid gland EXCEPT: A) age over 50 years B) microscopic lymph node metstasis C) tumor larger than 4 cm D) poorly differentiated histological grade E) invasion through capsule to adjacent tissues A 40-year-old woman has extensive microcalcifications involving the (24 entire upper aspect of the right breast. Biopsy shows a commedo pattern of .intraductal carcinoma : The most appropriate treatment is A) wide local excision B) radiation therapy C) wide local excision plus radiation therapy D) right total mastectomy
187
E) right modified radical mastectomy In the conventional ventilator management of acute adult respiratory distress (25 syndrome (ARDS) , arterial O2 saturation is maintained above 90% by all the : following EXCEPT F) increasing the ventilatory rate G) the use of positive end-expiratory pressure (PEEP) H) increasing mean airway pressure I) increasing tidal volumes J) increasing FiO2 26) Which of the following statements about patients with abdominal compartment syndrome is NOT true ? A) Abdominal pressure is usually measured indirectly through inferior vena cava B) Multiple contributing factors are commonly responsible C) The chief manifestations are reflected in central venous pressure , ventilatory function, and oliguria D) Decopression of the abdomen is required to resverse the syndrome E) Aggressive hemodynamic monitoring and management is required when the abdomen is opened 27) The most appropriate treatment for histologically malignant cystadenoma phylloides is : A) total mastectomy without axillary node dissection B) total mastectomy with axillary node dissection C) wide margin (3) cm excision of the lesion D) post operative hormonal manipulation E) postoperative adjuvant chemotherapy 28) Deep venous thrombosis resulting from upper extremity central venous lines: A) should be treated with catheter removal, heparin therapy, and long term anticoagulants B) is best with urokinase through the catheter C) is innocuous and self limiting, and best treated with catheter removal only D) is best treated with low-dose warfarin (coumadin, 1 mg / day) , without catheter removal E) is best managed by single systemic dose of low molecular weight heparin daily and continued catheter use 29) Emergency surgery is indicated for all of the following complications of ulcerative colitis EXCEPT: A) colonic dilatation greater than 12 cm (toxic mega colon ) B) free perforation C) complete intestinal obstruction D) intractable haemorrhage E) abscess formation 30) All the following statements concerning carcinoma of the oesophagus are true EXCEPT that: A) it has a higher incidence in males than females B) alcohol has been implicated as a precipitating factor C) adenocarcinoma is the most common type at the cardio esophageal junction D) it occurs more commonly in patients with corrosive oesophagitis E) surgical excision is the only effective treatment 31-. Which of the following statements about epiphrenic diverticula of the esophagus is/are correct?
188
f) They are traction diverticula that arise close to the tracheobronchial tree. g) They characteristically arise proximal to an esophageal reflux stricture. h) The degree of dysphagia correlates with the size of the pouch. i)
They are best approached surgically through a right thoracotomy.
j)
The operation of choice is a stapled diverticulectomy, long esophagomyotomy, and partial fundoplication.
32- Which of the following statements about Schatzki's ring is correct? f) The ring represents a panmural fibrotic stricture resulting from gastroesophageal reflux. g) Dysphagia occurs when the ring diameter is 13 mm. or less. h) The ring occurs within 1 to 2 cm. of the squamocolumnar epithelial junction. i)
Schatzki's ring indicates reflux esophagitis.
j)
Schatzki's ring signifies the need for an antireflux operation.
33. Which of the following statements about pathology encountered at esophagoscopy is/are correct? f) Reflux esophagitis should be graded as mild, moderate, or severe, to promote consistency among different observers. g) An esophageal reflux stricture with a 2-mm. lumen is not dilatable and is best treated with resection. h) A newly diagnosed radiographic distal esophageal stricture warrants dilation and antireflux medical therapy. i)
In patients with Barrett's mucosa, the squamocolumnar epithelial junction occurs 3 cm. or more proximal to the anatomic esophagogastric junction.
j)
After fasting at least 12 hours, a patient with megaesophagus of achalasia can safely undergo flexible fiberoptic esophagoscopy.
34. Infantile hypertrophic pyloric stenosis f) Occurs with a female : male ratio of 4:1. g) Sons of affected mothers have a 20% risk of developing the lesion h) Invariably presents between six and eights months of age i)
Typically presents with bile stained projectile vomiting
j)
Surgical treatment is by Heller's Cardiomyotomy
35-. Which of the following statements regarding the pathology of esophageal carcinoma is/are correct? f) Worldwide, adenocarcinoma is the most common esophageal malignancy. g) Squamous cell carcinoma is most common in the distal esophagus, whereas adenocarcinoma predominates in the middle third.
189
h) Patients with Barrett's metaplasia are 40 times more likely than the general population to develop adenocarcinoma. i)
Metastases from esophageal carcinoma are characteristically localized to regional mediastinal lymph nodes adjacent to the tumor.
j)
Achalasia, radiation esophagitis, caustic esophageal stricture, Barrett's mucosa, and Plummer-Vinson syndrome are all premalignant esophageal lesions that predispose to the development of squamous cell carcinoma.
36-45-year-old with isolated 6-cm colorectal metastasis in the liver 2 years after colectomy, otherwise healthy pest treatment would be: f) Radiofrequency ablation g) Systemic chemotherapy h) Hepatic lobectomy i)
Liver transplantation
j)
Cryosurgical ablation
37- Oesophageal atresia all true except: f) Is often associated with a distal trachea-oesophageal fistula g) Polyhydramnios is often present late in pregnancy h) 50% have other associated congenital abnormalities i)
Contrast X-ray studies are necessary to confirm the diagnosis
j)
Post-operatively over 30% develop oesophageal strictures
38-All are true about the dumping syndrome except: f) Symptoms can be controlled with a somatostatin analog. g) Diarrhea is always part of the dumping syndrome. h) Flushing and tachycardia are common features of the syndrome. i)
Separating solids and liquids in the patient's oral intake alleviates some of the symptoms of the syndrome.
j)
Early postoperative dumping after vagotomy often resolves spontaneously.
39-Which of the following statements about gastric polyps is/are true? f) Like their colonic counterparts, gastric epithelial polyps are common tumors. g) They are analogous to colorectal polyps in natural history. h) Endoscopy can uniformly predict the histology of a polyp based on location and appearance. i)
190
In a given patient, multiple polyps are generally of a multiple histologic type.
j)
Gastric adenomatous polyps greater than 2 cm. in diameter should be excised because of the risk of malignant transformation.
40-All of the following statements about surgical management of gastric lymphomas are true except: f) Stage I gastric lymphomas (small lesions confined to the stomach wall) can be cured completely with surgical therapy alone. g) Extensive gastric lymphomas that initially are treated with radiation and/or chemotherapy occasionally perforate during treatment and require secondary resection. h) Patients explored with a presumptive diagnosis of gastric lymphoma should undergo an attempt at curative resection when this is safe and feasible. i)
Without a preoperative diagnosis resection for gastric mass should not be attempted unless lymphoma can be excluded.
j)
Appropriate staging for primary gastric lymphoma includes bone marrow biopsy.
41-the most accurate test to confirm diagnosis of infected necrotizing pancreases is: f) Abdominal ultrasound study g) Indium-labeled leeukocte scan h) Cimputed tomographic scan i)
Elevated serum level of interleukain 6 and 8
j)
Percutaneous needle aspiration
42- Which of the following variables best predicts prognosis for patients with a recent diagnosis of cutaneous melanoma and no clinical evidence of metastatic disease? f) Breslow thickness. g) Clark's level. h) Ulceration. i)
Gender.
j)
Celtic complexion.
43-the following are true about intracranial tumors except: f) The most common location of brain tumors of childhood is the posterior cranial fossa. g) With few exceptions, examination of the CSF is of no value in the diagnosis of an intracranial tumor. h) Even the most malignant of primary brain tumors seldom spread outside the confines of the central nervous system (CNS). i)
191
The majority of astrocytomas can be cured surgically.
j)
Primary neoplasms of astrocytic, oligodendroglial, or ependymal origin represent gradations of a spectrum from slowly growing to rapidly growing neoplasms.
44 A right-sided disc herniation at the L5–S1 level typically may cause: f) Low back pain and left sciatica. g) Weakness of dorsiflexion of the right foot. h) A diminished or absent right ankle jerk. i)
Diminution of sensation over the medial aspect of the right foot, including the great toe .
j)
Weakness of dorsiflexion of the left foot.
45-. The preferred operation for initial management of a thyroid nodule that is considered suspicious for malignancy by FNAB is: f) Excision. g) Partial lobectomy. h) Total lobectomy and isthmusectomy. i)
Total thyroidectomy.
j)
All methods are correct
46-the most common presentation of Meckel,s diverticulum in an adult is: f) GIT bleeding g) GIT obstruction h) Intussuception i)
Litter,s hernia
j)
Diverticulitis
47-Optimal front-line treatment of squamous cell carcinoma of the rectum includes: f) Abdominal perineal resection. g) Low anterior resection when technically feasible. h) Radiation therapy. i)
Chemotherapy.
j)
Combined radiation and chemotherapy.
48 -65-year-old man presents with complaints of mucous discharge and perianal discomfort. Physical examination reveals a fistulous opening lateral to the anus. Anoscopic examination permits passage of a probe through the fistula tract. The fistula traverses the internal anal
192
sphincter, the intersphincteric plane, and a portion of the external anal sphincter. The fistula is categorized as which type? a) b)
Intersphincteric Transsphincteric
c)
Suprasphincteric
d)
Extrasphincteric
e) Non of the above 49-Warthin's tumor: f) Is a pleomorphic adenoma of salivary gland g) Should be treated by total paritidectomy h) Is considered a benign salivary gland neoplasia i)
Respond well to preoperative radiotherapy
j)
Often present with facial nerve compression
50-A 38 year old woman presents with right upper quadrant pain and bouts of vomiting. She is known to have gallstones and has had similar episodes in the past. Which of the following might support a diagnosis of acute cholecystitis rather than biliary colic a) duration of symptoms b) Severity of vomiting c) Presence of Murphy's sign d) Presence of gas under right hemidiaphragm on erect CXR. Q1. Which of the following is not true regarding blood supply of pancreas? a) It receives blood supply from coeliac trunk and superior mesenteric artery. b) Body and tail of pancreas is supplied by Splenic artery c) Postero superior pancreaticoduodenal artery is a branch of Superior mesenteric artery. d) All major pancreatic arteries lie posterior to pancreatic ducts. Q2. Most Common Cause of death in early acute Pancreatitis is a) Renal Failure b) Cardiac failure c) Respiratory Failure d) Uncontrolled Coagulopathy Q3. All of the following have been used in management of Acute Pancreatitis except a) Interleukin-10 b) Gabexate c) Somatostatin d) Peritoneal dialysis
193
Q4. Which of the following is not true for ectopic pancreas a) stomach and duodenum are the most common site b) Ectopic pancreas appears as a submucosal irregular nodule in the wall c) Islet tissue is present in all the organs where ectopic pancreas is present. d) Ulceration, bleeding and obstruction are the most common symptoms Q5. Nealon's classification is used for a) Acute pancreatitis b) chronic pancreatitis c) pseudocyst pancreas d) alcoholic pancreatitis >>>>>>>>>>>>>>>>>>>>>>>>>>>>>>>>>>>>>>>>>>>>>>>>>>>>>>>>>>>>>>>> Q1 Which is not an indication of spleenectomy in idiopathic thrombocytopenia ( ITP) A) asymptomatic patients with platelet count between 30000-50000 mm3 b) refractory thrombocytopenia c) relapse after glucocorticoid therapy d) platelet count of 10000 despite management for 6 weeks but no bleeding. Q2. The most common cystic disease of the spleen is a) Hydatic cyst of spleen b) Pseudo cyst of the spleen c) Columnar lined cyst of spleen d) Cystic lymphangioma Q3 ) True regarding abscess of the spleen are all except a) Majority of splenic abscess result from hematogenous spread from other sites b) Both Gram positive and gram negative organisms are responsible for abscess of spleen c) Splenomegaly (enlargement of spleen) is present in most of the patients. d) 2/3rd of the splenic abscess are solitary in adults Q4 Which of the following is not true regarding splenorrhaphy (Repairing of Spleen) a) Use of Argon Beam coagulator is superior to other techniques. b) Grade II and III injuries can be managed by suture repair c) At least One third of the spleen should be preserved to maintain immunological function. d) Mesh wrapping is recommended for Grade IV injuries Q5 OPSI is commonest in splenectomy done for a) Thalassemia b) Trauma c) Hereditary Spherocytosis d)ITP Q6 Which of the following is not true regarding wandering spleen? a) The spleen is attached to a long vascular pedicle without the usual mesenteric attachments. b) Torsion and infarction of the spleen are common complications c) There is congenital atresia of the dorsal mesogastrium in children d) Splenectomy is required in all cases >>>>>>>>>>>>>>>>>>>>>>>>>>>>>>>>>>>>>>>>>>>>>>>>>>>>>>>>>>>>>>>>. Q1. Endoscopic stone extraction from CBD is not possible in all except? a) Multiple CBD stones b) Intrahepatic stones
194
c) Multiple Gallstones d) Prior Gastrectomy Q2. What is the management of choledochal cyst adherent to portal vein? a) Excision and Roux en y hepaticojejunostomy b)Internal drainage into roux en y jejunal limb c) Hepatic lobectomy d) The internal lining of the cyst can be excised, leaving the external portion of the cyst wall intact. Q3. In the classification of cholangiocarcinoma by site, Type II is a) Confined to the common hepatic duct b) Involve the bifurcation without involvement of secondary intrahepatic ducts c) Tumors extend into either the right or left secondary intrahepatic ducts, respectively. d) Involve the secondary intrahepatic ducts on both sides. Q4. What is not true regarding laparoscopic biliary injuries? a) As surgeon experience goes beyond twenty cases rate of bile duct injusry decreases. b) The rate of laparoscopic bile duct injury is approximately 0.8% c) Most of the injuries are due to errors of judgement and skill Q 5. Brown pigment stones, false is a)They are earthy b) stones Seen in Asian population c) Easily breakable d) All the above >>>>>>>….>>>>>>>>>>>>>>>>>>>>>>>>>>>>>>>>>>>>>>>>>>>>>>>>>>>> Q1. Which of the following structures does not form the portal triad a) Portal Vein b) Hepatic vein c) Hepatic Artery d) Bile Duct Q2. Which of the following has the weakest association with Hepatocellular Carcinoma (HCC) a) Hepatitis B b) Hepatitis C c) oral contraceptives d) Smoking Q3. Which is not a paraneoplastic syndrome for Hepatocellular Carcinoma a) Hypercalcemia b) Hypoglycemia c) Erythrocytosis d) Hyperglycemia
195
Q4. Which is the most common cause of Hemobilia a) Blunt Trauma Abdomen b) Iatrogenic injury c) Cholelithiasis d) Hepatic artery aneyrysm Q5. Which is not true regarding Haemangioendothelioma of liver? a) It is a rare tumor b) It is highly vascular c) It involves both lobes of the liver d)) In 50% cases it has extrahepatic spread. >>>>>>>>>>>>>>>>>>>>>>>>>>>>>>>>>>>>>>>>>>>>>>>>>>>>>>>>>>>>>>> Q1. Most common extraintestinal manifestation of Crohn's disease of small intestine is. a) Ankylosing Spondylitis b) Erythema Nodosum c) Iritis d) Ureteral Obstruction Q2. Which of the following is the most common cause of death in Crohn's disease of small intestine a) Malignancy b) Sepsis c) Electrolyte Disorders d) Thromboembolic Phenomenon Q3. Which of the following is not true about Pneumatosis intestinalis of small intestine? a) It is seen equaly and males and females b) Most common location is subserosa in the jejunum c) Operative Procedures are required in most of the cases d) It is associated with COPD and immunodeficiency states Q4. What is not true about blind loop syndrome? a) It manifets as diarrhoea, weight loss and deficiency of fat soluble vitamins. b)Megaloblastic anemia is commonly seen c)Surgery is almost always required to correct small bowel syndrome d) Broad spectrum antibiotics are the treatment of choice Q5. What is not true about the immune mechanism in the small intestine? a) Intestine contains more than 70% of IgA producing cells in the body b)Ig A acts by activating the complement pathway c)Ig A is produced by plasma cells in the lamina propria d)Approximately 60% of the lymphoid cells are T cells Q6. What is not true about malrotation of the intestine a) In Incomplete rotation the rotation is arested at 180 degree b) The small intestine lies on the right with caecum in the midline c) Ladd's band extends from the Right upper quadrant to the caecum d) Hyperrotation is the most common form of intestinal malrotation
196
Q1. Which of the following is not a congenital abnormality associated with Juvenile polyps a) Malrotation b) Meckel's diverticulum c) Macrocephaly d) Mesenteric lymphangioma Q2. In Lynch syndrome which malignancy does not occur? a) Bronchus b) Ovary c) Endometrium d) Sebaceous Carcinoma Q3. Which of the following is not true for malignancy of Familial Adenomatous Polyposis a) Adrenals b) Thyroid c) Astrocytomas d) Hepatoblastomas Q4. What is not true for HNPCC a) It is the most common hereditary colorectal cancer syndrome in USA b) It is associated with MMR c) It is associated with APC mutation d) It is associated with carcinoma colon and extraintestinal cancers Q5. Which of the following is not fermented by colonic bacteria a) Lignin b) Pectin c) Cellulose d) Q6. True about Ulcerative Colitis with malignancy a) It has a better prognosis b) Is related to disease activity c) Is related to duration of ulcerative colitis d) Malignancy is more in anorectal ulcerative colitis Q7. In ulcerative coilitis with toxic megacolon lowest rcurrence is seen in a) Complete proctocolectomy and Brook's ileostomy b) Ileo rectal anastomoses c) kock's pouch d) Ileo anal pull through procedure Q8) All are precancerous for carcinoma colon except a. crohn's disease b. Bile acids c. Fats d. carotene Q1. The usual incision given for Zenker's diverticulum is a) Left Cervical b) Right Cervical c) Suprahyoid d) Midline
197
Q2. In Transhiatal Vs Trans thoracic esophagectomy most common complication associated with THE (Trans Hiatal esophagectomy) is a) Pulmonary b) Anastomotic leak c) Bleeding d) Injury to recurrent laryngeal nerve Q3.Which is the most disabling complication after three field esophagectomy? a) Bronchorrhoea b) Recurrent laryngeal nerve palsy c) Tracheal stenosis d) Q4. What is the most common complication after esophagectomy a) Arrythmia b) Pulmonary Collapse and Consolidation c) Recurrent laryngeal nerve injury d) Massive bleeding Q 5.Most valuable investigation for preoperative evaluation of extensive corrosive stricture is a) Endoscopic ultrasound b) Barium study c) CT Thorax d) Pharyngoscopy Q1. Ideal treatment of alkaline reflux gastritis after after Billroth I and Billroth II gastrectmy is a) Conversion of Billroth I to Billroth II b)Roux en Y gastrojejunostomy c)Tatal gastrectomy d)Conservative management Q2.Surgical Treatment of bleeding Type I gastric ulcer is a) Wedge resection of the gastric ulcer b) Oversewing the vessel at the ulcer base c) Distal gastrectomy d) Distal gastrectomy along with truncal vagotomy Q3.Helicobacter Pylori (H.Pylori) is a known cause of peptic ulcer disease. a) Its infectivity is highest in developed world. b) Person to person transmission is common c) It is seen in populations with low socio economic status d)H. Pylori is a gram negative microaerophilic bacteria Q4. Which of the following hormones are not released in duodenum? a) Gastrin b) Motilin c) Somatostatin d) Pancreatic YY
198
Q5. Treatment for bleeding duodenal diverticulum is a) Diverticulectomy b) Diverticulopexy c) Diverticulization d) Subtotal diverticulectomy Surgical MCQ 1. The radial nerve. a)Is a branch of the anterior cord of the brachial plexus. b)
Is derived from the posterior primary rami of C5 to C7.
c)Supplies the flexors of the arm. d)
Gives rise to the anterior interosseous nerve.
e)Injury above elbow produces a classical wrist drop. 2. Regarding femoral canal all are true except: a)Lies lateral to the femoral vein. b)
Has the inguinal ligament as its anterior border.
c)Has the lacunar ligament as its medial border. d)
Has the pectineal ligament as its posterior border.
e)Contains the lymph node of Cloquet. 3. The following causes hypercalcaemia except: a) Sarcoidosis. b) Primary hyperparathyroidism. c) Acute pancreatitis. d) Metastatic bronchial carcinoma. e) Milk-Alkali syndrome. :Number of human chromosomes in human female are .4 a)
23 pairs +XX.
b)
21 pairs +XY.
c)
22 pairs +XY.
d)
22 pairs +XX.
e)
23 pairs +XY.
5. Potassium deficiency should be suspected in all the following except: a)
In cases of paralytic ileus.
b)
When the patient's reflexes are exaggerated.
c)
If there is a decrease in height and peaking of the T waves of an ECG.
d)
In alkalotic states
6. Active immunization in case of tetanus:
199
a)
Antitetanus human serum.
b)
Gives short period of protection.
c)
Given in case proved tetanus.
d)
Use of immunoglobulin.
e)
None of the above.
All of the following are signs of rised intracranial pressure .7 :except .a) Headache .b) Vomiting .c) Papilledema .d) Aphasia .e) Bradycardia :All of the following are extra cranial hematomas except .8 .a) Subcutaneos haematoma .b) Extra dural haematoma .c) Cephalohaematoma .d) Subgaleal hematoma .e) Subperiostial haematoma :Glasgow coma scale all the following are true except .9 e)
Used for evaluation of comatose patient.
f)
It ranges from ( 3 to 15).
g)
Useful for neurological follow up.
h)
Useful for pupils evaluation.
i)
Best motor response given 6 point.
:All the followings are Indications for central line insertion EXCPET .10 f. Massive fluid replacement g. Massive blood replacement h. Measurement of central venous pressure i. Prolonged Intervenes fluid therapy :Most common early complication of central venous line is .11 a) Sepsis b) Pneumothorax c) Thoracic duct injury d) Thrombosis :The following are Complications of shock .12 a) Acute Respiratory failure b) Acute myocardial infarction c) Acute renal failure d) A&C only e) All the above
Causes of delayed union of fractures includes all the following .13 :EXCPET a) Compound fracture b) Infection c) Adequate immobilization d) Poor blood supply
200
In Head injury C.T. scan is indicated in the following .14 a) Aphasia after the injury b) Deterioration of level of consciousness c) Skull fracture with persistent headache d) A&B only e) All the above :Most important steps in management of head injury include .15 F. Prevent hypoxia G. Prevent Dehydration H. Assure Brain Metabolism I. Prevent secondary brain injury J. All the above Tension pneumothorax .16 F. is the commonest type of chest injuries G. Needs urgent X-Ray chest H. Is a clinical Diagnosis I. Causes flat neek viens J. Treated by thoracotomy tube after chest X-ray. :Calcitonin hormon is secreted to the blood circulations from .17 e)
Parathyroid gland.
f)
Parafollicular cells of thyroid gland.
g)
Supra renal gland.
h)
Pituitary gland.
i)
Gonads.
:Regarding tension pneumothorax, the first step in the management is .18 a)
Obtaining a stat chest x-ray.
b)
Cricothyroidectomy.
c)
Pass an endotracheal tube.
d)
Starting oxygen by a valve-mask device.
e)
Chest decompression needle.
19. The following are features of thyrotoxicosis except: e)Weight gain. f) Palpitations. g)
Proximal myopathy.
h)
Increased skin pigmentation.
i) Pretibial myxoedema.
20. The following is a clinical feature of Horner's syndrome: a)Miosis b)
Failure of abduction of the orbit
c)Increased sweating on the contralateral side of the forehead
201
d) e)
Exophthalmos.
All are true
:In role of nine extent of burn if entire trunk is burned it will be equal to .21 a)
9% body surface area.
b)
18% body surface area.
c)
36% body surface area.
d)
27% body surface area.
e)
45% body surface area.
: Trachlea (4th) cranial nerve supply .22 a)
Lateral rectus muscle of th eye.
b)
Medial rectus muscle of the eye.
c)
Superior obligue rectus muscle of the eye.
d)
Superior oblique muscle of the eye.
e)
Muscles of the upper eye lid.
23. Regarding pathological terms : a)Hypertrophy is an increase in tissue size due to increased cell number. b)
Hyperplasia is an increase in tissue size due to an increase in cell size.
c)Atrophy is an increase in tissue size due to disuse. d)
Metaplasia is a change form one abnormal tissue type to another.
e)A hamartoma is a developmental abnormality. 24. Regarding nephroblastomas: a)They are otherwise known as a Wilm's tumour. b)
Account for 10% of childhood tumours.
c)The commonest presentation is with an abdominal mass. d)
Most commonly present between 2nd and 4th year of life.
e)All are true. 25. Regarding fluid losses in a major burn all are true except: a)Are maximal between 12 and 24 hours after the injury. b)
Are related to the age of the patient.
c)Are not related to the weight of the patient. d)
Are related to the area burnt.
e)Are not related to the burn duration. 26. In obstructive jaundice : f) Urinary conjugated bilirubin is increased. g)
Serum unconjugated bilirubin is increased.
h)
Urinary urobilinogen is increased.
i) Serum conjugated bilirubin is reduced. j) Faecal stercobilinogen is increased. 27. Regarding Hydatid disease: f) Is due to Ecchinococcus granulosa.
202
g) Man is an accidental intermediate host. h) The liver is the commonest site of infection. i)
Can be diagnosed by the Casoni test.
j)
All are true. :The first aid of treatment in fracture of cervical spine should be .28
e)
Cervical spine x-ray.
f) Analgesia. g)
Neck immoblization.
h)
Cervical traction.
i) Non of teh above. :The first aid of treatment in fracture of cervical spine should be .29 .a) Cervical spine x-ray .b) Analgesia .c) Neck immoblization .d ) Cervical traction 30. Regarding local anaesthesia: a)
Local anaesthetics act on small before large nerve fibres
b)
Adrenaline reduces absorption and prolongs the local effects
c)
Preparations containing adrenaline are safe to use on digits and appendages
d)
Lignocaine has a longer duration of action than bupivicaine.
e)
All are false.
(Small bowel obstruction often results in: (all correct except one .31 k)
Hyperkalaemia.
l) Metabolic alkalosis. m)
Oliguria.
n)
Hypovolaemia.
o)
Severe dehydration.
A serious intra-abdominal injury in a comatose patient may be .32 (diagnosed by: (all are correct except one b)
Abdominal paracentesis.
c)
The observation of bruising pattern on the abdominal wall.
d)
Falling of heamoglobin values.
e)
The presence of marked abdominal distetion.
f) The presence of diarrhea.
:,A perforated duodenal ulcer .33
203
b)
Usually lies on the anterior or superior surface of the duodenum.
c)
Usually presents with the acute onset of severe back pain.
d)
Produces radiological evidence of free gas in the peritoneum in over 90 percent of the patients.
e)
Is usually treated by vagotomy and pyloroplasty.
f)
Is usually treated conservatively.
(Acute pancreatitis typically: (all correct except one .34 f)
Is accompanied by hypocalcaemia.
g)
Produces paralytic ileus.
h)
Is associated with a pleural effusion.
i)
Produces pyloric stenosis.
j)
Upper abdominal pain and vomiting.
:Biliary colic typically .35 b)
Occurs 3 to 4 hours after meals.
c)
Lasts 5 to 20 minutes.
d)
Radiates from the upper abdomen to the right subscapular region.
e)
Is made better by deep inspiration.
f) B&C only. :In post operative DVT, the following are true except .36 b)
Clinical DVT occures in the 4th post operative day.
c)
If complicated by pulmonary embolism, it occures usually after the 7th post operative day.
d)
The process of DVT starts preoperatively with the induction of anaesthesia .
e)
When discovered we should start the patient on coumadin "Warfarin anticoagulation".
f) It may lead to chronic venous in suffering as a complication of DVT. :In acute appendicitis all of the following are true except .37 e)
Anorexia.
f) Abdominal pain usually preceedes vomiting. g)
Pain after begins in the paraumbilical region.
h)
Constipation diarrhea may occur.
i) Dysuria excludes the diagnosis. The most common cause of massive haemorrhage in the lower .38 : gastroinfestinal tract is f) Carcinoma. g)
Diverticulosis
h)
Diverticulitis
i) Polyp. j) Ulcerative colitis. : Painless haematuria is the leading presentation of .39 f) Renal cell carcinoma. g)
204
Transitional cell carcinoma of the bladder .
h)
Ureteric stone.
i) Pelvi-ureteric obstruction. j) Ureterocele. All of the following are complications of massive blood transfusion .40 :except a)
Acute congestive heart failure.
b)
Transmission of infection.
c)
Hypercalcaemia.
d)
Hyperkalaemia.
e)
Transfusion reactions.
: Complication of undescended testis include all of the following except .41 a) Malignant degeneration. b) Increased susceptability to trauma. c) Increased spermatogenesis. d) More liable to testiculer torsion. e) Psychological complication. : The recurrent laryngoeal nerve is branch of .42 e) Facial nerve. f)
Glosso-pharyngeal nerve.
g) Cervical plexus. h) Vagus nerve. i)
Brachial plexus.
The thyroid tumor which is may be associated with pheochromocytoma .43 : is f) Papillary carcinoma. g)
Medullary carcinoma.
h)
Follicular carcinoma.
i) Anaplastic carcinoma. j) Malignant lymphoma. : The most common pancreatic cyst is .44 b)
Dermoid cyst of the pancreas.
c)
Hydatid cyst of the pancreas.
d)
Pancreas pseudocyst.
e)
Pancreatic cystadenoma.
f) Congenital cystic disease of the pancreas.
: The anatomical division between the anus and rectum .45
205
a)
Lateral haemorrhoidal groove.
b)
Inter haemorrhoidal groove.
c)
Dentate line.
d)
Arcuate line
e)
Ano-rectal ring.
:The comments type of Anorectal abscess is .46 a) Ischio rectal b) Perianal c) Submucons d) Pelvirectal :Anal Fissure .47 a) Usually anterior b) May be caused by previous anal surgery c) Can cause dark bleeding PR. d) Sometimes is painful e) Treated by steroids :Neonatal duodenal obstruction .48 a) May be associated with down's syndrome. b) Is more frequently found in premature infants. c) Typically presents with gross abdominal distension. d) Usually presents with vomiting of non-bile stained fluid e) B&C only. (acute superior mesenteric artery occlusion: (all correct except one .49 b) Characteristically presents with sudden pain and tenderness of increasing intensity. c) Is frequently accompanied by overt or occult blood loss in the stools. d) Frequently produces peritonitis. e) Can usually be diagnosed on plain abdominal x-rays. f) Can be diagnosed by mesenteric artery ongiography. :Regarding the management of polytrauma .50 f) Death follow a trimodal distribution. g) X-ray after primary survey should be AP cervical spine, chest and pelvis. h) Cardiac tamponade is characterized by raised B.p, a low JUP. i) Assessment of uncomplicated limb fractures should occur during the primary survey. j) A and B only. (Injuries to the urethra (all are correct except one .51 .a) Are more common in male .b) Are often caused by road traffic accidents .c) Are easily diagnosed on intra venous pyelography .d) Require urgent surgical treatment .e) Diagnosed by retrograde urethragraphy Car seat belts when properly adjusted. 52 .a) Prevent injuries to abdominal organs .b) May cause small bowel injuries c) Do not reduce the incidence of head injuries of passengers involving .in RTA . d) Protect the cervical spine during sudden acceleration .e) A & D only :Patients with major burns .53 f) Are in a negative nitrogen balance. g) Have normal calorie requirements. h) Do not generally become anaemic. i) Are resistant to septicaemia.
206
j) All of the above. (In a healing fracture: (All correct except one . 54 .a) The haematoma is initially invaded by osteoblasts .b) The tissue formed by the invading osteoblasts is termed osteoid .c) Calcium salts are laid down in the osteoid tissue .d) The final stage of repair is the remodelling of the callus .e) The callus formation is related to the amount of stress at fracture side :In a colles’ fracture the distal radial fragment .55 a) Is dorsally angulated on the proximal radius. b) Is usually torn from the intra-articular triangular disc. c) Is deviated to the ulnar side. d) Is rarely impacted. e) Is ventrally displaced. :A malignant melanoma .56 .a) Frequently arises from hair-bearing naevi * .b) Frequently arises from junctional naevi .c) Has a worse prognosis when it areses on the leg .d) Should be suspected in any big pigmented lesion .e) Non of the above is correct :Squamous cancer of the lip .57 .a) Is most common in early adult life .b) Is more common in fair skinned subjects .c) Metastasises readily by the blood stream .d) Is preferably treated by radiotherapy once lymph node deposits are present .e) All of the above are correct :Basal cell carcinomas -58 .a) Usually metastasise to regional lymph nodes .b) Are less common than squamous cell carcinomas .c) Are characterised histologically by epithelial pearls .d) Are particularly common in oriental races .e) Non of the above is correct :Fiboadenomata of the breast -59 .a) Are commonest in early adult life .b) Are indiscrete and difficult to distinguish .c) Are usually painless .d) Resolve without treatment .e) A&C only :Paget’s disease of the nipple .60 .a) Usually presents as abilateral eczema of the nipple .b) Is always related to an underlying breast cancer .c) Indicates incurable breast cancer .d) Has non-specific histological characteristics .e) A&C only :stones in the common bile duct -61 .a) Are present in nearly 50 per cent of cases of cholecystitis .b) Often give rise to jaundice, fever and biliary colic .c) Are usually accompanied by progressive jaundice .d) Are usually associated with a distended gallbladder .e) A&D only (Colonic polyps: (all correct except one -62 f) Are associated with colonic cancer. g) May be hereditary. h) Should not be removed if they are asymptomatic. i) May be hyperplastic.
207
j)
Are commonly adenomatous.
Select the Best Appropriate Answer for each of the following questions: 1) the commonest presentation of Meckel's diverticulum in an adult is: k) Gastroitestinal bleeding l) Gastrointestinal obstruction m) Intussuception n) Litter,s hernia o) Diverticulitis 2) The preferred operation for initial management of a thyroid nodule that is considered suspicious for malignancy by FNAB is: k) Excision. l) Partial lobectomy. m) Total lobectomy and isthmusectomy. n) Total thyroidectomy. o) All methods are correct 3) All of the following are true about neurogenic shock Except: k) There is a decrease in systemic vascular resistance and an increase in venous capacitance. l) Tachycardia or bradycardia may be observed, along with hypotension. m) The use of an alpha agonist such as phenylephrine is the mainstay of treatment. n) Severe head injury, spinal cord injury, and high spinal anesthesia may all cause neurogenic shock. o) A and B 4) Which of the following variables best predicts prognosis for patients with a recent diagnosis of cutaneous melanoma and no clinical evidence of metastatic disease? k) Breslow thickness. l) Clark's level. m) Ulceration. n) Gender. o) Celtic complexion. 5) All are true about the dumping syndrome except: k) Symptoms can be controlled with a somatostatin analog. l) Diarrhea is always part of the dumping syndrome. m) Flushing and tachycardia are common features of the syndrome. n) Separating solids and liquids in the patient's oral intake alleviates some of the symptoms of the syndrome. o) Early postoperative dumping after vagotomy often resolves spontaneously. 6) Which of the following statements regarding the pathology of esophageal carcinoma is correct? k) Worldwide, adenocarcinoma is the most common esophageal malignancy.
208
l)
Squamous cell carcinoma is most common in the distal esophagus, whereas adenocarcinoma predominates in the middle third. m) Patients with Barrett's metaplasia are 40 times more likely than the general population to develop adenocarcinoma. n) Metastases from esophageal carcinoma are characteristically localized to regional mediastinal lymph nodes adjacent to the tumor. o) Achalasia, radiation esophagitis, caustic esophageal stricture, Barrett's mucosa, and Plummer-Vinson syndrome are all premalignant esophageal lesions that predispose to the development of squamous cell carcinoma. 7) Which of the following is the most effective definitive therapy for both prevention of recurrent variceal hemorrhage and control of ascites? f) g) h) i) j)
Endoscopic sclerotherapy. Distal splenorenal shunt. Esophagogastric devascularization (Sugiura procedure). Side-to-side portacaval shunt. End-to-side portocaval shunt
8) Regarding hernia repair: f) Simultaneous repair of bilateral direct inguinal hernias is always having significant decreased risk of recurrence g) The preperitoneal approach may be appropriate for repair of a multiple recurrent hernia h) A femoral hernia repair can best be accomplished using a Bassini or Shouldice repair i) Management of an incarcerated inguinal hernia with obstruction is best approached via laparotomy incision j) All are correct. 9) Regarding excessive skin scarring processes? f) g) h) i)
Keloids occur randomly regardless of gender or race Hypertrophic scars and keloid are histologically different Keloids tend to develop early and hypertrophic scars late after the surgical injury Simple re excision and closure of a hypertrophic scar can be useful in certain situations such as a wound closed by secondary intention j) Non of the above 10) A 22-year-old man sustains a single stab wound to the left chest and presents to the emergency room with hypotension. Which of the following statements is true concerning his diagnosis and management? e) The patient likely is suffering from hypovolemic shock and should respond quickly to fluid resuscitation f) Beck’s triad will likely be an obvious indication of compressive cardiogenic shock due to pericardial tamponade g) Echocardiography is the most sensitive noninvasive approach for diagnosis of pericardial tamponade d)The placement of bilateral chest tubes will likely resolve a. j.
209
The following are Complications of shock: Acute Respiratory failure
k. Acute myocardial infarction l. Acute renal failure m. A&C only 12) The following are manifestations of hyperparathyroidism EXCPET: E. Recurrent pancreatitis F. Psychological disturbance G. Renal stones H. Gall stones 13) Retracted nipple can be caused by the following EXCPET: A. Carcinoma of Breast B. Duct ectasia C. Chronic periductal mastitis D. A&B only E. All the above 14) Carcinoma of the breast A. may be familial B. infiltrating duct carcinoma is the commonest C. treated always by lumpectomy and axillary clearance D. A&B only E. None of the above 15) Tension pneumothorax K. is the commonest type of chest injuries L. needs urgent X-Ray chest M. is a clinical Diagnosis N. causes flat neek viens O. treated by thoracotomy tube after chest X-ray 16) The most accurate test for acute non-calculous cholecystitis is A. plain X-ray B. ultra sound C. C.T scan D. Isotope HIDA scan E. MRI scan 17) The commonest type of gall stones is A. cholesterol stones B. pig minted stones C. calcium stones D. Mixed Stones E. Phosphate Stones 18) Complication of ERCP included F. Perforation G. Pancreatitis H. Hemorrhage I. A&C only J. All the Above 19) Toxic Mega Colon: A. May be a complication of inflammatory bowl disease B. May be a complication of acute infective enteritis C. Affect Left colon alone
210
D. A&B only E. Treated by antibiotics for at least seven days 20) Surgical complication of typhoid fever: E. Paralytic ileus F. Intestinal hemorrhage G. Perforation of typhoid ulcer H. A&B only I. All the above 21) Complication of Crohn’s disease are: f. Intestinal obstruction g. Fistula h. Anal disease i. A & B only E. All the above 22) Abduction of the vocal cords results from contraction of the: K) crico-thyroid muscles L) posterior crico-arytenoid muscles M) vocalis muscles N) thyro-ary-epiglottic muscles O) lateral crico-arytenoid and transverse arytenoids muscles 23) Hashimoto’s thyroiditis : A) is of viral origin B) presents usually by nodular goiter C) can be diagnosed by thyroid antibodies and fine needle aspiration of D) is often premalignant E) must be treated always by surgery
thyroid
24) Mammary duct ectasia is characterized by the following EXCEPT : F) is defiened as primary dilatation of major ducts of breast in middle aged women G) may present with nipple retraction and Peau d orange picture H) is treated usually by simple mastectomy I) anaerobic superinfection commonly occurs in this recurrent periductal plasma cell mastitis
25) A blood-stained discharge from the nipple means that the patient may have: A. papillorna of the nipple B. fibroadenoma C. duct papilloma D. duct ectasia E. fat necrosis of the breast
26) The following causes hypercalcaemia except: a) Sarcoidosis. b) Primary hyperparathyroidism.
211
c) Acute pancreatitis. d) Metastatic bronchial carcinoma. e) Milk-Alkali syndrome. 27) Potassium deficiency should be suspected in all the following Except: e)
In cases of paralytic ileus.
f)
When the patient's reflexes are exaggerated.
g)
If there is a decrease in height and peaking of the T waves of an ECG.
h)
In alkalotic states.
28) Active immunization in case of tetanus: f)
Antitetanus human serum.
g)
Gives short period of protection.
h)
Given in case proved tetanus.
i)
Use of immunoglobulin.
j)
None of the above.
29) All of the following are signs of rised intracranial pressure Except: a) Headache. b) Vomiting. c) Papilledema. d) Aphasia. e) Bradycardia. 30) All of the following are extra cranial hematomas except: a) Subcutaneos haematoma. b) Extra dural haematoma. c) Cephalohaematoma. d) Subgaleal hematoma. e) Subperiostial haematoma.
31) All the followings are Indications for central line insertion EXCPET: n. Massive fluid replacement o. Massive blood replacement p. Measurement of central venous pressure q. Prolonged Intervenes fluid therapy 32) Most common early complication of central venous line is: a) Sepsis b) Pneumothorax c) Thoracic duct injury d) Thrombosis
212
33) The following are Complications of shock: a) Acute Respiratory failure b) Acute myocardial infarction c) Acute renal failure d) A&C only e) All the above 34) Causes of delayed union of fractures includes all the following EXCPET: a) Compound fracture b) Infection c) Adequate immobilization d) Poor blood supply 35) In Head injury C.T. scan is indicated in the following f) Aphasia after the injury g) Deterioration of level of consciousness h) Skull fracture with persistent headache i) A&B only j) All the above 36) Most important steps in management of head injury include: K. Prevent hypoxia L. Prevent Dehydration M. Assure Brain Metabolism N. Prevent secondary brain injury O. All the above 37) Tension pneumothorax a) is the commonest type of chest injuries b) Needs urgent X-Ray chest c) Is a clinical Diagnosis d)Causes flat neek viens i) Treated by thoracotomy tube after chest X-ray. 38) Regarding tension pneumothorax, the first step in the management is: f)
Obtaining a stat chest x-ray.
g)
Cricothyroidectomy.
h)
Pass an endotracheal tube.
i)
Starting oxygen by a valve-mask device.
j)
Chest decompression needle.
39) The following is/are a clinical feature of Horner's syndrome: e)Miosis f) Failure of abduction of the orbit
213
g)
Increased sweating on the contralateral side of the forehead
h)
Exophthalmos.
e)
All are true
40) In role of nine extent of burn if entire trunk is burned it will be equal to: f)
9% body surface area.
g)
18% body surface area.
h)
36% body surface area.
i)
27% body surface area.
j)
45% body surface area.
41) Regarding pathological terms : f) Hypertrophy is an increase in tissue size due to increased cell number. g)
Hyperplasia is an increase in tissue size due to an increase in cell size.
h)
Atrophy is an increase in tissue size due to disuse.
i) Metaplasia is a change form one abnormal tissue type to another. j) A hamartoma is a developmental abnormality. 42) Regarding fluid losses in a major burn all are true except: f) Are maximal between 12 and 24 hours after the injury. g)
Are related to the age of the patient.
h)
Are not related to the weight of the patient.
i) Are related to the area burnt. j) Are not related to the burn duration. 43) In obstructive jaundice : k)
Urinary conjugated bilirubin is increased.
l) Serum unconjugated bilirubin is increased. m)
Urinary urobilinogen is increased.
n)
Serum conjugated bilirubin is reduced.
o)
Faecal stercobilinogen is increased.
44) Regarding Hydatid disease: k) Is due to Ecchinococcus granulosa. l)
Man is an accidental intermediate host.
m) The liver is the commonest site of infection. n) Can be diagnosed by the Casoni test. o) All are true. 45) The first aid of treatment in fracture of cervical spine should be: a) Cervical spine x-ray. b) Analgesia.
214
c) Neck immoblization. d ) Cervical traction. 46) Regarding local anaesthesia: f)
Local anaesthetics act on small before large nerve fibres
g)
Adrenaline reduces absorption and prolongs the local effects
h)
Preparations containing adrenaline are safe to use on digits and appendages
i)
Lignocaine has a longer duration of action than bupivicaine.
j)
All are false.
47) Small bowel obstruction often results in: (all correct except one) p)
Hyperkalaemia.
q)
Metabolic alkalosis.
r) Oliguria. s)
Hypovolaemia.
t) Severe dehydration. 48) A serious intra-abdominal injury in a comatose patient may be diagnosed by: (all are correct except one) g)
Abdominal paracentesis.
h)
The observation of bruising pattern on the abdominal wall.
i) Falling of heamoglobin values. j) The presence of marked abdominal distetion. k)
The presence of diarrhea.
49) A perforated duodenal ulcer, all are true except: g)
Usually lies on the anterior or superior surface of the duodenum.
h)
Usually presents with the acute onset of severe back pain.
i)
Produces radiological evidence of free gas in the peritoneum in over 90 percent of the patients.
j)
Is usually treated by vagotomy and pyloroplasty.
k)
Is usually treated conservatively.
50) Acute pancreatitis typically: (all correct except one) k)
Is accompanied by hypocalcaemia.
l)
Produces paralytic ileus.
m)
Is associated with a pleural effusion.
n)
Produces pyloric stenosis.
o)
Upper abdominal pain and vomiting.
51) Biliary colic typically:
215
g)
Occurs 3 to 4 hours after meals.
h)
Lasts 5 to 20 minutes.
i) Radiates from the upper abdomen to the right subscapular region. j) Is made better by deep inspiration. k)
B&C only.
52) In post operative DVT, the following are true except: g)
Clinical DVT occures in the 4th post operative day.
h)
If complicated by pulmonary embolism, it occures usually after the 7th post operative day.
i)
The process of DVT starts preoperatively with the induction of anaesthesia .
j)
When discovered we should start the patient on coumadin "Warfarin anticoagulation".
k)
It may lead to chronic venous in suffering as a complication of DVT.
53) Painless haematuria is the leading presentation of : k)
Renal cell carcinoma.
l) Transitional cell carcinoma of the bladder . m)
Ureteric stone.
n)
Pelvi-ureteric obstruction.
o)
Ureterocele.
54) The thyroid tumor which is may be associated with pheochromocytoma is : k)
Papillary carcinoma.
l) Medullary carcinoma. m)
Follicular carcinoma.
n)
Anaplastic carcinoma.
o)
Malignant lymphoma.
55) The most common pancreatic cyst is : g)
Dermoid cyst of the pancreas.
h)
Hydatid cyst of the pancreas.
i) Pancreas pseudocyst. j) Pancreatic cystadenoma. k)
Congenital cystic disease of the pancreas.
56) The anatomical division between the anus and rectum : f) Lateral haemorrhoidal groove. g)
Inter haemorrhoidal groove.
h)
Dentate line.
i) Arcuate line j) Ano-rectal ring. 57) The comments type of Anorectal abscess is: e) Ischio rectal f) Perianal g) Submucons
216
h) Pelvirectal 58) Anal Fissure: f) g) h) i) j)
Usually anterior May be caused by previous anal surgery Can cause dark bleeding PR. Sometimes is painful Treated by steroids
59) Neonatal duodenal obstruction: f) May be associated with down's syndrome. g) Is more frequently found in premature infants. h) Typically presents with gross abdominal distension. i) Usually presents with vomiting of non-bile stained fluid j) B&C only. 60) acute superior mesenteric artery occlusion: (all correct except one) g) h) i) j) k)
Characteristically presents with sudden pain and tenderness of increasing intensity. Is frequently accompanied by overt or occult blood loss in the stools. Frequently produces peritonitis. Can usually be diagnosed on plain abdominal x-rays. Can be diagnosed by mesenteric artery angiography.
61) Regarding the management of polytrauma: k) Death follow a trimodal distribution. l) X-ray after primary survey should be AP cervical spine, chest and pelvis. m) Cardiac tamponade is characterized by raised B.p, a low JUP. n) Assessment of uncomplicated limb fractures should occur during the primary survey. o) A and B only. 62) Injuries to the urethra (all are correct except one) a) Are more common in male. b) Are often caused by road traffic accidents. c) Are easily diagnosed on intra venous pyelography. d) Require urgent surgical treatment. e) Diagnosed by retrograde urethragraphy. 63) Car seat belts when properly adjusted a) Prevent injuries to abdominal organs. b) May cause small bowel injuries. c) Do not reduce the incidence of head injuries of passengers involving in RTA. d) Protect the cervical spine during sudden acceleration . e) A & D only. 64) Patients with major burns: k) Are in a negative nitrogen balance. l) Have normal calorie requirements. m) Do not generally become anaemic. n) Are resistant to septicaemia.
217
o) All of the above. 65) In a healing fracture: (All correct except one) a) The haematoma is initially invaded by osteoblasts. b) The tissue formed by the invading osteoblasts is termed osteoid. c) Calcium salts are laid down in the osteoid tissue. d) The final stage of repair is the remodelling of the callus. e) The callus formation is related to the amount of stress at fracture side. 66) In a colles’ fracture the distal radial fragment: f) Is dorsally angulated on the proximal radius. g) Is usually torn from the intra-articular triangular disc. h) Is deviated to the ulnar side. i) Is rarely impacted. j) Is ventrally displaced. 67) A malignant melanoma: a) Frequently arises from hair-bearing naevi. b) Frequently arises from junctional naevi. c) Has a worse prognosis when it areses on the leg. d) Should be suspected in any big pigmented lesion. e) Non of the above is correct. 68) Squamous cancer of the lip: a) Is most common in early adult life. b) Is more common in fair skinned subjects. c) Metastasises readily by the blood stream. d) Is preferably treated by radiotherapy once lymph node deposits are present. e) All of the above are correct. 69) Basal cell carcinomas: a) Usually metastasise to regional lymph nodes. b) Are less common than squamous cell carcinomas. c) Are characterised histologically by epithelial pearls. d) Are particularly common in oriental races. e) Non of the above is correct. 70) Fiboadenomata of the breast: a) Are commonest in early adult life. b) Are indiscrete and difficult to distinguish. c) Are usually painless. d) Resolve without treatment. e) A&C only. 71) Paget’s disease of the nipple: a) Usually presents as abilateral eczema of the nipple. b) Is always related to an underlying breast cancer. c) Indicates incurable breast cancer. d) Has non-specific histological characteristics. e) A&C only. 72) stones in the common bile duct: a) Are present in nearly 50 per cent of cases of cholecystitis. b) Often give rise to jaundice, fever and biliary colic. c) Are usually accompanied by progressive jaundice.
218
d) Are usually associated with a distended gallbladder. e) A&D only. 73) Colonic polyps: (all correct except one) k) Are associated with colonic cancer. l) May be hereditary. m) Should not be removed if they are asymptomatic. n) May be hyperplastic. o) Are commonly adenomatous. 74)The following are resident skin bacterial flora except : a. Diphtheroids b. Micrococci c. Staphylococcus spp. d. Clostredial spores e. Bacteroids 75) The following manifestations are found In HIV infections and AIDS : a. Pneumocystitis carinii b. Kaposi sarcoma c. Cytomegalovirus infection d. A and b only e. All the above 76) Patient on long term steroids going for surgery will need to : a. Stop treatment with steroid 24hours before the operation b. Continue treatment until the morning of operation c. Continue the same treatment d. Increase his the dose by the doctor e .Decrease his the dose by the doctor 77) Patient on aspirin going for surgery will need to : a. Stop treatment one week before the operation b. Stop treatment two week before the operation c. Stop treatment three week before the operation d. Stop treatment four week before the operation e. Stop treatment five week before the operation 78) Electrocardiogram findings In hypokalemia : a. Flattening of T wave b .Raised ST segment c. Tall P wave d Widened QRS e. All the above 79) The following can cause metabolic acidosis : a. Short gut syndrome b. Hemorrhagic shock c. Enterocutaneous fistula d. A and c only e. All the above 80) Normal saline solution a. One liter contain 154 mmol of chloride b. One liter contain145 mmol of sodium c. Can cause metabolic alkalosis d . Used to treat lower gastrointestinal tract losses e. Is the most physiological solution 81) In Wolf graft the following are true except: a. Contains dermis in addition to epidermis b. Natural skin colour
219
c. There is contraction of the graft later d. Used to cover face, tendons e. Needs immobilization 82) All the following are causes of graft failure except : a. Haematoma b. Seroma c. Poor immobilization d. Infection e.. Good recipient bed 83) All the following are indications of CT scan in head injuries except: a. Focal neurological deficit b. GCS score > 12 c. Deterioration of level of consciousness d. A and c only e. all the above 84) All the following are indications of skull x-ray in head injuries except a. Loss of consciousness or amnesia b. Focal neurological signs c. Patient on anticoagulant medication d. Scalp bruising or swelling e. Alcohol intoxication 85) Regarding Acute subdural haematoma : a. It is due to meningeal artery tear b. Has 30% mortality c. Cause secondary brain injury d. Treated by early operation e. More urgent than epidural haematoma 86)In Cystic fibrosis all the following are true except: a. Autosomal dominant b. Dysfunction of exocrine glands c. There is steatorrhea d. Recurent. Pulmonary disease, e. Diagnosis by DNA analysis 87) Ranson criteria in acute pancreatitis includes all the following except: a. Hct increase more than 10% b . BUN increase >1.8 mmol/L or 5mg% c. S. Ca less than 2mmol/L or 8mg% d. Po2 <7.89kpa or 60mmHg e. Base deficit >4mEq/L 88) Mortality rate according to Ranson criteria in acute pancreatitis is as follows: a. Patients with less than three 0.9% b. Patients with 3-4 signs 10% c. Patients with 5-6 signs 25 % d. Patients with >5 signs 50% e. none of the above 89) All are bad prognostic signs in pancreatic carcinoma except: a. Back pain b . Rapid wt. Loss c. Ascitis & liver mets d. Low albumin e. Low C reactive protein 90)In insulinoma the following are false except : a. Age above 40 b. Hyperglycemia
220
c. Clinically may simulate duodenal ulcer d .Never change to malignant e. A and c only 91) Which of the following is one of Whipple s triad in insulinoma a. Hypoglycemic attack typical in the evening b. Blood glucose >45mg% during the attack c. Relieved by glucagon d. Hypoglycemic attack typical two hours after exercise e. Relieved by insulin 92) Regarding gastrinoma : the following are true except a. Site usually head of pancreas b. S. Gastrin is elevated c. Two types antral G cell and pancreatic origin d. Clinically Diarrhea , steatorrhea, hypokalemia e. Treated by gastrectomy or excision 93) the following are involved in the embryology of the thyroid except: a .Thyroglossal duct b Fourth pharengeal pouch c. Ultiomobranchial body d. Cells from neural crest e. Second pharyngeal arch 94) Regarding follicular carcinoma of the thyroid a .More common than papillary CA b. Rarely familial c. Vascular invasion is late d. All the above e. none of the above 95) Regarding medullary thyroid carcinoma ,all are true except:: a. Bilateral in familial type b. Amyloid stroma c. Synthesize calciton d. late lymph. & blood stream metastases e .Is solid tumor 96) Breast pain can be caused by all the following except; a. Tietz syndrome b. Ductectasia c. Triggr point d. Idiopathic e. Sclerosig adenosis 97)The following are risk factors In development of breast carcinoma except: a. Gender b. Age c. Diet d. Geography e. Alcohol 98)Varicose ulcer is: f. Usually very painful& shallow g. Deep with much necrotic tissue h. More common with primary varicose vein i. A & C only j. None of the above 99) Abdominal aortic aneurysm a. Is sacular in type b. Usually true aneurysm
221
c. Occur usually above renal arteries d. B & c only e. None of the above 100) Gangrene: a. Is necrosis of tissue b. The cause may be venous occlusion c. Usually painful d. All the above e. None of the above 101) Lymphedema: a. May be Congenital b. Should be bilateral c. May be pitting in early stage d. A & C only e. None of the above 102) Prophylactic antibiotics: a . Should be bacteriostatic b. Must be give for 48hours only c. Must be started 24hours before the operation d. B and c only e. None of the above Regarding peptic ulceration (103 a. H. pylori is a gram-positive bacillus b. Duodenal is more common than gastric ulceration c. Zollinger-Ellison syndrome is associated with gastrin hyposecretion d. H2-blockers will heal 85-95% of duodenal ulcers in 8 weeks e. Triple therapy can eradicate H. pylori in 80% of patients in one week 104) Regarding the management of major trauma: a. Deaths follow a trimodal distribution b. X-rays after the primary survey should be of AP Cervical spine, chest and pelvis c. Cardiac tamponade is characterised by raised BP, low JVP and muffled heart sounds d. Assessment of uncomplicated limb fractures should occur during the primary survey e. Deterioration of the casualty during the primary survey should lead to the secondary survey 105) In acute appendicitis : a. The risk of developing the illness is greatest in childhood b. Mortality increases with age and is greatest in the elderly c. Faecoliths are present in 75-80% of resected specimens d. Appendicitis is a possible diagnosis in the absence of abdominal tenderness e. all are true
106) Stones in the common bile duct: a. Are found in 30% of patients undergoing cholecystectomy (Without pre-op ERCP) b. Can present with Charcot's Triad c. Are suggested by an bile duct diameter >8mm on ultrasound d. ERCP, sphincterotomy and balloon clearance is now the treatment of choice
222
e. all true. 107) Solitary thyroid nodules: a. Are more prevalent in men b. In the adult population less than 10% are malignant c. More than 50% of scintigraphically cold nodules are malignant d. The risk of a hot module being malignant is high e. Should be surgically removed in all patients 108) Regarding colonic polyps: a. Metaplastic polyps are pre-malignant b. Adenomatous polyps are pre-malignant c. Villous adenomas are more common than tubular adenomas d. Genetic mutations can result in epithelial metaplasia e. few carcinomas arise in adenomatous polyps 109) Which one of the following is not a characteristic of Papillary carcinoma of the thyroid? a. Can be reliably diagnosed using fine needle aspiration cytology b. Is almost always unifocal c. Histologically displays Psammoma bodies d. Typically spread to the cervical lymph nodes e. Requires a total thyroidectomy for large tumours 110) Regarding gas gangrene a. It is due to Clostridium botulinum infection b. Clostridial species are gram-negative spore forming anaerobes c. The clinical features are due to the release of protein endotoxin d. Gas is not present in the muscle compartments e. Surgical debridement and antibiotics are an essential part of treatment 111) Medullary carcinoma of the thyroid a. Is a tumour of the parafollicular C cells b. Produce thyroxine as the principle hormone c. 10% of cases are sporadic d. Often occur as part of the MEN type II syndrome e. subtotal thyroidectomy is the surgical treatment of choice 112) Regarding acute abdomen: (a) Mesenteric adenitis is a common cause of abdominal pain in children. (b) The absence of free air on an erect chest x-ray excludes an intra-abdominal perforation. (c) Free air under the right hemi-diaphragm can be mistaken for gas within the stomach. (d) A raised serum amylase is diagnostic of acute pancreatitis. (e) Diverticulosis is a common cause of acute left iliac fossa tenderness assosciated with pyrexia and a raised white cell count. 113) Obstructive jaundice (a) Ultrasound can not readily reveal intrahepatic ductal dilatation. (b) Obstructive jaundice is suggested by raised AST and ALT. (c) Malabsorption of vitamin D in jaundiced patients can affect the synthesis of coagulation factors. (d) Endoscopic retrograde cholangio-pancreatography (ERCP) is of diagnostic and therapeutic value in jaundiced patients. (e) Pale urine and dark stools are suggestive of obstructive jaundice. 114) Inguinal hernia (a) A direct hernia passes through the deep inguinal ring into the inguinal canal. (b) A femoral hernia is more common than an inguinal hernia in females. (c) An inguinal hernia can not be distinguished from a femoral hernia by its
223
relationship to the inguinal ligament. (d) The inferior epigastric vessels lie medial to the deep inguinal ring. (e) The floor of the inguinal canal is formed by the conjoint tendon. 115) Peptic ulcer disease (a) A minority of duodenal ulcers are caused by Helicobacter Pylori infection. (b) A raised serum creatinine is a sign of a significant upper GI bleed. (c) Triple therapy for H. Pylori eradication involves the combination of an H2 receptor antagonist, a proton pump inhibitor and an antibiotic. (d) Following endoscopic therapy for a bleeding ulcer, a rebleed warrants immediate open surgical intervention. (e) none is true 116) The ideal fluid therapy in a patient with pyloric stenosis and repeated vomiting is: a. Isotonic crystaloid containing sodium chloride b. Hypertonic crystaloid containing dextrose – saline c. Isotonic solution containing dextrose d. Large molecular weight colloid containing dextran e. none of above.
1. All of the following statements about keloid are true except: a) b) c) d)
Keloid does not regress spontaneously. A keloid extends beyond the boundaries of the wound. Are best managed by excision and reapproximation. Contains an abnormally large amounts of collagen.
2. A sliding hernia: a) b) c) d)
Is easily reducible. Is apparent only with Valsalva maneuver. Occur in more than one orifice. Has a viscus contained in the wall of the sac.
3. Which of the following is the most sensitive test for pheochromocytoma? a) b) c) d)
24-hour urine VMA. 24-hour urine metanephrine. 24-hour urine epinephrine. 24-hour urine norepinephrine.
4. The most common physical finding associated with Cushing`s syndrome is : a) b) c) d)
Hirsutism. Truncal obesity. Striae. Acne.
5. A 70 year old female with a tender irreducible mass immediately below and lateral to a) b) c) d)
the pubic tubercle. Plain x-ray showed signs of intestinal obstruction. The possible diagnosis is: large bowel carcinoma. Intestinal obstruction due to adhesions. Strangulated inguinal hernia. Strangulated femoral hernia.
6. A 50 year old woman has a 3cm left adrenal mass discovered incidentally during an abdominal CT scan for epigastric pain, has history of lumpectomy for stage I breast cancer 7 years ago, the next step should be:
224
a) b) c) d)
24-hour urine cortisol. 24-hour urine catecholamines and metabolites. FNA of the mass under CT guidance. Operative excision.
7. A 60 year old man has a 4cm left thyroid nodule. FNA reveals a papillary cancer. No a) b) c) d)
lymph nodes are palpable. Management should include all of the following except: Total thyroidectomy. Radioactive iodine ablation. Serum thyroglobulin monitoring. Modified left radical neck dissection.
8. The proliferative phase of wound healing is characterized by which of the following? a) b) c) d)
Entry of monocytes. Absence of basal epithelial cell proliferation. Fibroblast proliferation, release of collagen, and formation of interstitial matrix. Disappearance of the fibroblasts from the wound.
9. Which of the following factors can be associated with impaired wound healing? a) b) c) d)
Chronic steroid use. Chemotherapy. Radiotherapy. All of the above.
10. Concerning incarcerated inguinal hernia, which of the following statements is not true: a) All incarcerated herniae are surgical emergencies and always require urgent surgical intervention. b) Attempt at reduction of incarcerated hernia is considered generally safe. c) Vigorous Attempts at reduction of incarcerated hernia may result in reduction en masse with progression to obstruction and strangulation. d) Incarcerated hernia almost cause small bowel rather than large bowel obstruction.
11. Principal blood supply to the parathyroid glands are derived from: a) b) c) d)
Thyroidea ima artery. Superior thyroid arteries. Inferior thyroid arteries. Highly variable.
12. Radioactive iodine is effective treatment for metastatic lung disease for which of the a) b) c) d)
following thyroid neoplasms? Anaplastic. Papillary. Medullary. Hurthel cell.
13. MEN 2b is characterized by which of the following? a) Medullary carcinoma of the thyroid, pheochromocytoma, mucosal neuroma, marfanoid habitus. b) Parathyroid adenoma, pancreatic islet cell tumor, pituitary tumor. c) Medullary carcinoma of the thyroid, pheochromocytoma and parathyroid hyperplasi. d) Parathyroid carcinoma, pheochromocytoma and chronic pancreatitis.
14. The 5-year survival rate for adrencortical carcinoma is approximately? 225
a) b) c) d)
0% 20-25%. 50-60%. Nearly 100%.
15. Causes of Cushing`s syndrome include all of the following except: a) b) c) d)
Adrenal carcinoma. Adrenal hyperplasia. Lung small cell carcinoma. Posterior pituitary tumor.
16. Which of the following is a positive DPL (Diagnostic Peritoneal Lavage) in cases of a) b) c) d) e)
blunt abdominal trauma? WBC count more than 100/cc. WBC count more than 200/cc. WBC count more than 250/cc. Red blood cell count more than 10.000/ cc. Red blood cell count more than 100.000/cc.
17. One of the following is a crystalloid solution: a) b) c) d)
Dextrose water 5%. Ringers Lactate. Dextran 70. Albumin 5%.
18. Skeletal changes in hyperparathyroidism include all of the following EXCEPT : a) b) c) d) e)
osteitis fibrosa cystica osteoblastic lesions on the x-rays bone cyst pathologic fractures osteoporosis 19. In Meckel’s diverticulum, all the following are true EXCEPT: a) It is usually located on the mesenteric border b) It is commonly located within the terminal 90 cm of the ileum c) It may be connected to the umbilicus by a band d) Technetium scan may demonstrate it e) It is a remnant of the omphalomesentric duct 20. Hyperkalemia may be caused by: a) Pyloric stenosis b) Chronic diarrhea c) Villous adenoma of the rectum d) Conn’s syndrome e) Addison’s disease
21. Concerning Papillary carcinoma of the thyroid all of the following are true EXCEPT a) Can be reliably diagnosed using fine needle aspiration cytology b) Is almost always unifocal c) Histologically may displays Psammoma bodies d) Typically spread to the cervical lymph nodes e) Requires a total thyroidectomy for large tumours
226
22. All of the following statements regarding timing of elective surgery are true EXCEPT: a) Orchidopexy for undescended testicles is performed by the age one year b) Repair of inguinal hernias in children is performed as soon as possible c) Ligation of patent processus vaginalis in congenital hydrocele is performed after the age of one year d) Pediatric umbilical hernia repair is performed as soon as possible e) Hypospadias repair is best performed between six and 18 months of age
23. The commonest complication of total parenteral nutrition is: a) b) c) d) e)
Hyperosmolar,non-ketotic, hyperglycemic coma. Metabolic acidosis. Thrombosis of the central vein used . Catheter related sepsis. Pneumo or Hemothorax.
24. Regarding diverticular disease. One of the following is true: a) b) c) d) e)
Once it causes massive bleeding, surgery is needed. Surgery should include resection of the whole colon containing diverticulae. The ideal diagnostic modality for acute diverticulitis is colonoscopy. The ideal treatment for acute diverticulitis is early surgery. It is a known cause for colo-vesical fistula. 25. Regarding ulcerative colitis, one of the following is true: a) Massive bleeding is common. b) Does not present with colonic perforation. c) The primary treatment is surgical. d) Prophylactic surgery may be needed to prevent carcinoma. e) Surgical treatment consists of excision of the inflamed part of the colon with primary anastomosis. 26. A 60 year old male patient presented with bleeding per rectum. His hemoglobin was 10 g/dl. Examination revealed stage 4 hemorrhoids. The next most important step in the management is: a) Bulk laxatives. b) Hemorrhoidectomy. c) Injection sclerotherapy of the hemorrhoids. d) Colonoscopy e) Rubber band ligation of the hemorrhoids. 27. A 55-year old woman is admitted with colicky left sided abdominal pain ,vomiting and constipation for 4 days. Examination reveals a distended abdomen , diffuse tenderness and no guarding. Initial management includes arranging the following: a) Immediate laparotomy. b) Intravenous fluids and nasogastric aspiration and analgesia. c) Urgent colonoscopy. d) Urgent upper GI endoscopy. e) Double contrast Barium enema. 28. Regarding spontaneous pneumothorax, all are true Except: a) Occurs in two different age groups. b) Can cause mediastinal deviation and compression of the contralateral lung. c) If suspected , a confirmatory chest X-ray is not necessary. d) Is best managed by immediate pleurectomy. e) Is usually associated with structural abnormalities of the lung. 29. A paraesophageal hiatus hernia : a) Is a common cause of gastro-esophageal reflux. b) Is as common as sliding hiatus hernia.
227
c) Is usually repaired through the chest. d) Is lined by Barrett’s epithelium. e) Can cause gastric outlet obstruction. 30. The following statement about peritonitis are all true except: a) Peritonitis is defined as inflammation of the peritoneum. b) Most surgical peritonitis is secondary to bacterial contamination. c) Primary peritonitis has no documented source of contamination and is more common in adults than in children and in men than in women. d) Tuberculous peritonitis can present with or without ascites.
31. In patients who develop a documented episode of deep venous thrombosis (DVT) the most significant long-term sequela is: a) Claudication. b) Recurrent foot infections. c) Development of stasis ulcer. d) Pulmonary embolization. e) Diminished arterial perfusion. 32. Complete mechanical small bowel obstruction can cause dehydration by all the following except: a) Interfering with oral intake of water. b) Inducing vomiting. c) Increasing intestinal absorption of water. d) Causing secretion of water into the intestinal lumen. e) Causing edema of the intestinal wall. 33. Squamous cell cancer of the lip, all true except: a) The most common cancer of the oral cavity. b) Is most commonly found on the lower lip in males with a history of smoking and prolonged sun exposure. c) Treated by V surgical excision and primary closure if the cancer is a primary lesion. d) Frequently associated with lymph node metastasis requiring modified neck dissection. e) Treated with radiation therapy, particularly if the lesion involves the oral commissure. Occasionally patients with follicular or Hürthle cell carcinoma also require modified neck dissections 34. daily Nutritional requirement for a 70 kg man are : a) 35-40 k cal/kg/day b) 1-2 gm nitrogen/ day c) 15 gm protein/ day d) 35 mmol k+ /day e) 1500 ml/IV fluids/ day 35. Hyponatreamia may be due to : a) hypovolaemia b) edema c) renal failure d) All are true
36. The following are indications for laparotomy in abdominal trauma except : a) b) c) d) e)
Intrahepatic heamatoma Persistant shock Eviceration Uncontrolled heamorhage Gun shot wounds 37. In second degree burn all are true except : a) Blistering is diagnostic sign b) Escarotomy is needed
228
c) Usually is very painful d) If more than 20% hospitalization is needed e) Reflex ileus is a known complication 38. Regarding anal fissure, which of the following is false: a) anal fissure causes severe pain on defecation b) anal fissure causes blood streaks on stools c) local injection of botulin is a choice of treatment d) lateral external sphincterotomy is a choice of treatment e) nitroglycerin oint 0.2% can be used in acute cases 39. which statement regarding fistula in ano is false: a) can result from drained perianal abscess b) treatment is by fistulotomy c) presents with fecal soiling sometimes d) the tract is always intersphinecteric e) can be a manifestation of crohn's disease 40. Deep Venous thrombosis is best diagnosed by : a) 121 fibrinogen b) CT scan c) Venogran d) Physical examination e) Doppler ultrasonography 41. About oesophageal varices, all the following are TRUE Except: a) usually present with a large volume hematemesis. b) Vasopressin has no rule in the treatment of bleeding varices. c) liver cirrhosis is the cause in most cases. d) an associated thrombocytopenia is usually secondary to hypersplenism. e) Sengstaken-Blakemore tube may be inserted to provide temporary hemostasis. 42. About ascites in advanced liver disease, all the following are TRUE Except: a) portal vein thrombosis is a common predisposing factor. b) peritoneal fluid aspirate is usually transudate. c) bacterial peritonitis incidence is increased and is secondary type in most cases. d) associated splenomegally is common. e) the initial treatment is restriction of salt intake and with diuretics. 43. About hydatid liver disease, one of the following is TRUE: a) is rare in countries around the Mediterranean. b) the liver is the most common involved organ after the lung. c) dog is an intermediate host. d) Casoni test is diagnostic in 90% of cases. e) the fluid content of the cyst is alkaline.
44. About hepatic adenomas, all the following are TRUE Except: a) females are more affected than males. b) incidence is increased by oral contraceptive pills. c) spontaneous regression is well recognized. d) usually they are asymptomatic and are discovered incidentally. e) they can rupture and as many as 25% are identified after an acute episode of hemorrhage. 45. About Hepatocellular carcinoma, all the following are TRUE Except: a) It is often presented in elderly patients.
229
b) is one of the world's most common solid cancers. c) its incidence is increasing rapidly. d) it is associated with chronic liver disease like (HBV, HCV). e) α-fetoprotein (AFP) is usually elevated 46. About hypersplenism, all the following are TRUE Except: a) bone marrow biopsy is usually normal. b) hypersplenism may be primary or secondary. c) in the secondary type there is increased destruction of abnormal blood cells. d) there is usually associated splenic enlargement. e) combination of anemia, leucopenia and thrombocytopenia is common 47. About splenectomy, all the following are TRUE Except: a) trauma is the most common indication for surgery. b) splenectomy may be required to achieve diagnosis or for staging. c) TTP (Thrombotic Thrombocytopenic Purpura) is the main indication for splenectomy in non traumatic patients. d) splenectomy may be indicated in tropical countries with malaria. e) splenectomy is rarely indicated in Felty syndrome. 48. In the post splenectomy patients, all the following are TRUE Except: a) Thrombocytosis. b) Leukopenia. c) appearance of Howel-Jolly bodies and sideroytes. d) increased hemoglobin level. e) left basal atelectasis and left pleural effusion are common. 49. In critical ischemia there is: a) rest pain b) oedema c) color changes d) hyperesthesia e) all of the above 50. lymphedema tarda: a) usually before the age of 35 years b) affects upper limbs only c) affect lower limbs only d) usually pitting in early stage e) none of the above 51. The most common cause of Haemtemesis & melena is: a) Acute peptic ulcer b) Chronic peptic ulcer c) Oesophageal varices d) Carcinoma of stomach e) Mallory – wises syndrome. 52. The commonest cause of abdominal aortic aneurysm is: a) Congenital b) collagen diseases c) Marfan's syndrome d) atherosclerosis e) syphilis 53. Third degree haemorrhoids : a) Prolapse on defecation b) Prolapse on defecation and returned manually c) Prolapse on exertion d) Permanent prolapse e) cause bleeding on defecation only 54. Hydradenitis suppurativa a) due to inflammation of eccrine sweat glands
230
b) due to abnormal apocrine sweat gland c) antibiotics has no role in treatment d) perianal region is common site e) treatment is always surgical 55. All the following are complications of diverticular disease of the colon except: a) Haemorrhage b) fistula c) perforation d) malignancy e) abscess 56. Duodenal atresia a) Usually in the first part b) 70% have Down's syndrome c) Double bubble sign, is typical radiological sign d) treatment is conservative until 3 months 57. Which one of the following suggest a diagnosis of Hirschsprung's disease? k. A contrast-study showing dilatation of the ganglionic bowel segment. l. Early presentation with vomiting. m. neonatal large bowel obstruction. n. Presentation after 1 year of age. o. Red currant jelly stools 58. Regarding crohn's disease all are true except i. It is full thickness inflammation ii. Cause depletion of goblet cell mucin iii. Commonly affect small bowel iv. Cause non caseating granuloma 59. Regarding Duke’s staging for colon cancer, all of the following are true except: a) A : confined to bowel wall b) B : penetrates up to submucosa c) C : any T but with lymph node metastasis d) D : With distant metastasis 60. Shock can best be defined as: i. Hypotension. j. Hypoperfusion of tissues k. Hypoxemia. l. All of the above. 61. which of the following most often initiates the development of acute appendicitis? a)Viral infection b)Acute gastroenteritis c)Obstruction of the appendiceal lumen d)A primary clostridial infection 62. The most common hernia in females is: a.Femoral hernia. b.Direct inguinal hernia. c.Indirect inguinal hernia d.Obturator hernia. e.Umbilical hernia.
231
63. The size of adenoma of parathyroid gland is correlated with levels of: a) Alkaline phosphatase b) Phosphorus c) Serum calcium and PTH d) All of the above e) Non of the above 64. Regarding the diagnosis of postoperative intraabdominal infection which of the following is correct: a) the initial symptoms of postoperative intraabdominal infection could be fever, abdominal pain, prolonged ileus or diarrhea. b) if fever persists is mandatory to do X-Ray chest, exam the veins and IV lines, urine culture and blood culture. c) if diarrhea persists, is possible to be due to antibiotics treatment. d) exam of the wound should be important as well as the vital signs and lab tests. e) all of the above. 65. ERCP : (endoscopic retrograde cholangiopancreatography) should be done if: a) There is ecographic evidence of choledocholithiasis b) There is choledocholithiasis by MRCP c) Picture of obstructive jaundice should be done as early as possible d) when enzymes of cholestasis are elevated in presence of choledocholithiasis e) all of the above 66. Early diagnosis of amebic abscess is by a) CBC + serology b) Serology and ultrasound c) CT scan and serology d) CBC + CT scan e) Ultrasound + FNA A 47-year-old woman presents with a left breast mass, which she noticed 1 month ago. She is also experiencing nipple discharge from the same breast. She denies having any other medical problems and takes no medications. Which of the following are risk factors for malignant breast cancer? a) Early menarche b) Nulliparity c) Absence of lactation d) Family history of breast cancer e) All of the above For the patient in Question 67, which of the following statements regarding screening mammography is true? a) Women in their 40s should be screened every 5 years with mammography b) Women 50 years of age and older should be screened every 5 years c) Women who are at higher than average risk of breast cancer should seek expert medical advice about whether they should begin screening before 40 years of age; they should also seek advice about the frequency of screening d) A woman should discontinue screening mammography after 65 years of age 69. For the patient in Question 67, which of the following statements regarding investigative studies is true? a) If the patient's diagnostic mammography is negative, no further studies are warranted b) Ultrasonography can determine whether this patient's lesion is cystic and thus potentially eliminate the need for additional workup or treatment c) The main approved use of MRI in breast disease is for differentiating cystic lesions from solid lesions d) None of the above
232
For the patient in Question 67, which of the following statements regarding cystic lesions is true? a) For asymptomatic simple cysts, no further intervention is required b) If the aspirate from fine-needle aspiration (FNA) is not bloody, the fluid should not be sent for cytologic analysis, because it is unlikely to yield a diagnosis of cancer c) If a simple cyst recurs after aspiration, it should be excised d) All of the above For the patient in Question 67, which of the following statements regarding nipple discharge is true? a) If the discharge is bloody, unilateral and spontaneous, a diagnosis of cancer is more likely b) Hyperthyroidism can cause galactorrhea c) Cytology of nipple discharge to detect breast cancer is highly sensitive d) Ultrasonography is the first study to perform for the evaluation of nipple discharge. A 22-year-old man is admitted to the hospital for treatment of a worsening left arm infection. Findings on physical examination are consistent with a deep tissue necrotizing infection. Subsequent MRI confirms the diagnosis. Of the following, which should be the next step in the management of this patient? a) Prompt, adequate operative debridement b) Urgent amputation c) A 3-day trial of intravenous antimicrobial therapy, followed by surgery d) None of the above A 56-year-old woman has been experiencing abdominal pain for 4 hours. The pain is in right upper quadrant and radiates into the scapular region. She has had multiple episodes of vomiting. For this patient, which of the following signs on physical examination is associated with acute cholecystitis? a) Murphy sign b) Rovsing sign c) Kehr sign d) Carnett sign A 48-year-old woman presents to the emergency department complaining of right upper quadrant pain, which began 24 hours ago. She reports the pain as being spasmodic and sharp and that it radiates to her right shoulder. She says that she has had similar episodes over the past few months, especially after eating large meals. Associated with the pain is nausea and vomiting. Her blood pressure is 120/85 mm Hg, and her pulse is 100 beats/min. On physical examination, the patient is found to have a nontender abdomen with no palpable masses. Her chest and cardiovascular examinations are normal. The nurse notices that her sclera are slightly icteric. On subsequent laboratory studies, her serum bilirubin level is found to be 10 mg/dl. What imaging study should be performed next for this patient with presumed posthepatic jaundice? a) Endoscopic retrograde cholangiopancreatography (ERCP) b) Magnetic resonance imaging c) Percutaneous transhepatic cholangiography (PTC) d) Ultrasonography A 50-year-old man presents to your office for evaluation of jaundice, weight loss, and fatigue. Over the past 6 months, the patient has been experiencing generalized, aching pain in his abdomen and back. The pain has worsened over the past few weeks. In addition, the patient has lost his desire to eat; over the past 3 months, he has lost 12 kgs. Physical examination reveals a cachectic man with yellowish skin. Abdominal
233
examination elicits diffuse tenderness along the upper right and left quadrants with palpable mass (pear size) in the Rt upper quadrant. Ultrasonography reveals no evidence of gallstones but dilated gallbladder and the intrahepatic ducts are dilated. Of the following, which is the most likely cause of this patient's posthepatic obstruction? a) Pancreatic cancer b) Choledocholithiasis c) Hepatitis C virus infection d) Diseased gallbladder A 45-year-old woman presents with abdominal pain and vomiting of 1 day's duration. The patient had a history of exploratory laparotomy after a motor vehicle accident 8 years ago. Which of the following statements is true for this patient? a) Pain from mechanical obstruction is usually localized in the middle of the abdomen, whereas pain from ileus or pseudo-obstruction is diffuse b) Pain from mechanical obstruction is usually more severe than pain from ileus c) The severity of pain from mechanical obstruction may decrease over time d) Bowel sounds are exaggerated in mechanical obstruction while the abdomen is silent in ileus e) All of the above For the patient in Question 76, which of the following statements regarding adhesive partial small bowel obstruction is true? a) Exploratory laparotomy is indicated in a patient with partial small bowel obstruction from adhesions if the patient's condition deteriorates or has not significantly improved within 12 hours after nasogastric decompression and resuscitation b) Nonoperative therapy leads to resolution of adhesive partial small bowel obstruction in fewer than 10% of patients c) A previous abdominal operation and the type of adhesion present are not factors affecting the likelihood of response to nonoperative medical therapy for partial small bowel obstruction d) None of the above
A 55-year-old man presents with hematemesis that began 2 hours ago. He is hypotensive and has altered mental status. No medical history is available. For this patient, which of the following statements regarding nasogastric aspiration is true? a) A bloody aspirate is an indication for esophagogastroduodenoscopy (EGD) b) A clear, nonbilious aspirate rules out the need for EGD c) A clear, bilious aspirate rules out the need for EGD d) None of the above For the patient in Question 78, which of the following is an indication for surgery? a) Substantial bleeding occurs from a duodenal ulcer that is not controlled by EGD b) Ongoing hemorrhage occurs from a gastric ulcer in a hemodynamically unstable patient c) Bleeding continues from either a duodenal ulcer or a gastric ulcer despite medical and endoscopic therapy d) All of the above For the patient in Question 78, which of the following statements is
234
true regarding an esophageal varices as the site of bleeding? a) Rubber banding or intravariceal sclerotherapy should be performed first b) Balloon tamponade should be performed first d) I.V. somatostatin should be administered first e) I.V propranolol should be administered firs A 47-year-old man presents with complaints of nonspecific abdominal pain, which he has been experiencing for 2 days. On examination, the patient is found to be hypotensive and anemic. He denies having experienced any weight loss, and he reports that there have been no changes in his stool. For this patient, which of the following statements is true? a) Given the abundance of diagnostic modalities, the source of the hemorrhage is identified in almost 100% of such patients b) A drop in the orthostatic blood pressure greater than 10 mm Hg indicates that more than 800 ml of blood has been lost c) A clear aspirate on gastric lavage almost always signifies a lower GI source of bleeding d) None of the above A 56-year-old woman presents with symptoms of abdominal pain, weight loss, and rectal bleeding. She is anemic and hypotensive, but she is stable. For this patient, which of the following should be done first if the gastric lavage yields copious amounts of bile? a) Colonoscopy b) Esophagogastroduodenoscopy c) Arteriography d) Emergency laparotomy
82.
A 72-year-old white man presents with weight loss and abdominal discomfort. The patient tests positive for occult fecal blood on guaiac testing. His medical history is significant for gastritis. For this patient, which of the following statements regarding gastric cancer is true? a) The distribution of primary lesion sites within the stomach has remained constant over the past century b) The incidence of gastric cancer has increased in the United States over the past 70 years c) The 5-year survival rate for patients with gastric cancer is about 55% d) Worldwide, there is a strong association between Helicobacter pylori infection and gastric cancer 83. A 38-year-old woman is found to have adenocarcinoma of the stomach. Histologic subtyping shows the adenocarcinoma to be of the diffuse variety. For this patient, which of the following statements regarding surgical therapy is true? a) Resection of all gross disease with microscopically negative margins has not been shown to have a clear impact on overall survival c) Subtotal gastrectomy should be the procedure of choice for cancer of the distal half of the stomach, provided that an adequate negative proximal margin can be achieved d) Total gastrectomy should be the procedure of choice for all cancers of the stomach
235
e)
Any surgical therapy for gastric cancer should include routine splenectomy
A 68-year-old man presents to his primary care physician for a routine physical examination. The patient is found to have guaiac-positive stools. Colonoscopy reveals a “golfball”-size, near-obstructing tumor in the descending colon, not admitting the scope. The biopsy is positive for adenocarcinoma of the colon. Which of the following is the next step in the management of this patient? a) Full metastatic workup first, and if negative, then plan for colon resection b) A course of radiation therapy prior to any resection c) Plan for pre-operative chemotherapy d) Do metastatic work up, but plan for colon resection anyway e) Schedule a barium enema to evaluate the proximal colon 85. After the appropriate evaluation of the patient in Question 85, the patient undergoes surgery. No intraoperative evidence of metastases is identified. Postoperatively, the pathology report reveals that the tumor is an adenocarcinoma invading into the pericolonic fat, with 2 involved lymph nodes. After the patient recovers from surgery, which of the following is the most appropriate next step in his management? a) b) c) d) e)
Abdominal CT scan every 6 months No further therapy is indicated, because the involved nodes were removed Chemotherapy Measurement of CEA levels yearly Colonoscopy every 6 months
86.
A definite increased risk of developing colon cancer is associated with all of the following except: Diet low in animal fat & protein Ulcerative colitis Familial polyposis Strong family history of colon cancer in several preceding generations
a) b) c) d)
Select the most common mode of spread of colon cancer a) b) c) d)
Hematogenous Lymphatic Direct extension Implantation
A 60 year-old man has felt upper quadrant pain and had a 15 kgrs weight loss in the last 6 months. An upper GI series showed a 1.5 cm ulcer on the lesser curvature of the stomach. The next step in management is: a) b) c) d) e)
236
Antacids, H2 blocking agents, and repeat evaluation in 6 wks UGI endoscopy with biopsy and cytologic examination. Gastric acid analysis. Testing to obtain fasting serum gastrin levels. Resection of the Ulcer.
60 y/o female with history of pelvic radiation for cervical cancer. She is day 7 Post operative from right hemicolectomy for adenoma of cecum. She had drainage from wound, initially purulent but then continuous brown? Initial treatment a) b) c) d)
Packing wound with wet to dry Placement of rubber sump catheter to suction Protection of skin with paste, powder, zinc oxide Insert NGT and administer IV- Fluids
For the patient in Question 90, what imaging tests to help for diagnosis a) b) c) d) e)
UGI series with SBFT (small bowel follow through) Barium Enema Instillation of contrast material via catheter into fistula CT abdomen / pelvis All of the above
91. Diagnostic workup revealed distal ileal fistula What is appropriate initial therapy? a) Prompt exploration and interruption of fistula b) Exploration and bypass of fistula c) Exploration with resection of portion of ileum with primary reanastomosis d) 4-6 week trial of TPN and/or low residue/elemental enteral alimentation Clinically a saphena varix is most likely to be confused with: a) Baker’s cyst b) Femoral hernia c) Spermatocele d) Soft sore e) Varicocele Regarding Pneumobilia one of the following is correct: a) Is presence of gas in the gallbladder b) Is presence of bile in the lung due to bronchobiliary fistula c) Is presence of gas in the biliary tree due to gas producing microorganism or biliodigestive anastomosis d) Is presence of gas in the portal system A 32 year-old woman underwent lumpectomy with clearance of the axilla for 1.5 cm infiltrating duct carcinoma of the breast, pathology study showed 18 lymph nodes free of metastasis, Estrogen and Progesteron receptors are negative, HER2 is negative, the surgical resection margin’s are free of malignant cells, this patient will need which one of the following post operative therapy: a) b) c) d) e)
Tamoxifen for 5 years Chemotherapy + Tamoxifen Chemotherapy + Radiotherapy Radiotherapy alone Non of the above (f
h)
237
A 60 – year old woman with a 6 – months history of upper abdominal discomfort and 8 kgms weight loss. CA 19-9 markedly raised. She presents with deep jaundice, an ultrasound showed dilated hepatic ducts. ERCP was unsuccessful at passing a tight CBD narrowing. Which is the most appropriate investigation should be performed next: a) Plain X-Ray b) Contrast CT scan c) Percutaneous transhepatic cholangiogram (PTC) d) MRI (MRCP) A 25 – year old woman presents to emergency department with left loin pain and fever. Her urine analysis showed blood, leucocytes and nitrites, which of the following is the most likely diagnosis: i. ii. iii. iv. v.
Ureteric calculus Adult polycystic disease Pyelonephritis Urinary tract infection Acute appendicitis
A 50 – year old woman with a 3 cm periampullary cholangiocarcinoma but no local or distant spread. The most appropriate operation for possible curative treatment is: 1. 2. 3. 4. 5.
Total pancreatectomy Duodenectomy Whipple's procedure (duodenopancreatectomy) A triple bypass procedure Distal pancreatectomy
A 9 – month – old boy presents with irritability and bilious vomiting, Red - currant jelly stool. There is a sausage – shaped mass on the right side of the abdomen. The most likely diagnosis is: a) b) c) d) e)
Meckele's diverticulitis Marotation volvulus Pyloric stenosis Intussusception Hirshprung's disease
100. Pneumoperitoneum is present after perforated duodenal ulcer in the percentage of: a) 15% b) 35% c) 55% d) 80% e) 95% Please read carefully and choose the best answer: 1. The most common cause of upper limb ischemia is: a. Trauma b. Vaso spastic conditions c. atherosclerosis ……….
238
d. Intrinsic trauma of the SA e. Thrombo arteritis obliterans 2. Popleteal artery aneurysm a. accounts for 70% of all peripheral aneurysms b. 25% are bilateral c. 30% develop complication within 5years d. A and c only e. All of the above 3. All the following can cause thoracic outlet compression except a. cervical rib b. abnormal 1st rib c. abnormality of clavicle d. aneurysm of brachial artery e. abnormality of scalene Muscle 4. Regarding Gas all are true except: a. Is seen in the portal vein in necrotizing enterocolitis b. In the biliary tree indicates possible cholangitis c. Gas in the portal system indicates a serious condition d. Is seen in portal vein in hemorrhagic pancreatitis e Is seen in biliary tree in (Roux in Y) hepato-jejunostomy
5. Which of the following is manifestation of Beurger' diseases : a. occlusion of small and medium arteries b. thrombophlebitis c. Reynold's phenomenon d. all the above e. a and b only Recurrence after resection for Ca rectum is related to which one of .6 :the followings a) Tumor Grade b) Number . of lymph nodes c) Lateral Margin Involvement d) Inexperienced surgeon e) All of the above 7. Lymph edema tarda f) usually before the age of 35 years g) affects upper limbs only h) affect lower limbs only i) usually pitting in early stage j) none of the above 8.The main blood supply of the breast comes from a. Internal mammary perforators b. Thoraco-acromial artery, c. Vessels to serratus anterior, d. Lateral thoracic artery,
239
e. Terminal branches of the 3rd – 8th inter costal perforators 9.Fibroadenosis of the breast: a. is due to proliferation of both duct & stroma b. cysts are usually single c. commonly bilateral inner upper Quadrant. d. tender two weeks before menstruation e. treated by surgical excision 10. All the following are causes of nipple retraction except a. duct ectasia b. mammary duct fistula c. periductal mastitis d. malignancy e. physiological retraction 11. Treatment of cyclical pronounced mastalgia includes all the following except a. supportive bra b. reduce caffeine, red ,chocolate intake c. simple analgesi d. Vitamin A, Vitamin C e. evening primrose oil 12. The commonest type of breast carcinoma is a. invasive duct carcinoma b. medullary carcinoma c. invasive lobular carcinoma d. mucinous (colloid) carcinoma e. in situ carcinoma & Paget's dis. 13. Indications of isotope thyroid scan are a. identifying nodules and determining if they are "hot" or "cold". b. measuring the size of the goiter prior to treatment. c. locating thyroid tissue outside the neck, d. all the above e. a and c only 14. Thyroid cyst a. 50% due to degeneration in the colloid b. 30-50% of cold nodules c. Intra cystic malignancy is more in males d. all the above e. none of the above 15. Regarding occult papillary carcinoma of the thyroid all the following are true except a. the lesion is 1cm or less b. usually not clinically obvious c. rarely metastasis d. Infiltrates the capsule e. Surgery is not required in all cases 16. The most common cause of pyrexia in the 5th post operative day is
240
a. deep vein thrombosis b. wound infection c. respiratory tract infection d. urinary tract infection e. pulmonary embolism 17. Post operative wound dehiscence all are true except a. Incidence is 1-2% b. dehiscence may occur without warning. c. serosanguinous drainage precedes wound dehiscence in 75% of patients. (7th day) d. the cause may be iatrogenic d. treated by Sterile saline pack cover ,antibiotics and surgical repair 18. Regarding sodium balance which of the following is true a. total body Na is 7500 mmol b. bones are the main sodium stores c .daily In take is 200-300mmol d. excretion mainly in feces ,and sweat e. controlled mainly by anti diuretic hormone 19. Hypokalemia which of the following is true a. may be caused by GIT fistula b. clinically ,the patient may have abdominal colics , c. ECG shows, high peaked T wave d. all the above e. a and c only 20. Dextrose 5% solution a. is hypotonic solution b. supplies 250 kcal/L 5% Dextrose provides 170 kcal/L. c. useful in trauma patients d. prolonged use causes hyponatremia e. none of the above Hyponatremic solutions (5% dextrose, total parenteral nutrition, enteral formulations) may cause hyponatremia 21. Patient taking antihypertensive drugs going for surgery should a. continue medication till morning of operation b. stop medication one day before the operation c. stop medication three days before the operation d. stop medication one week before the operation e. none of the above 22. Carbuncle a. is a confluence of several boils b. causes minimal tissue destruction, c. usually pointing to the skin d. less common in diabetic patients e. treated by incision and drainage 23. In Hydradenitis suppurativa all the following are true except a. abnormality in eccrine sweat gland b. recurrent Infection is common c. can cause abscess & sinus formation
241
d. can be treated with antibiotics e. surgical treatment is by wide local excision 24. All the following are indications of surgery in small bowel fistula except a. associated peritonitis b. failure to close in three weeks c. malignant disease d. distal obstruction e. substantial bowel discontinuity 25. All the following are indications of total parenteral nutrition in the surgical patient except a. Short gut syndrome b. Sever acute pancreatitis c. Extensive Burns d. Enterocutaneous fistula e. Esophageal carcinoma 26. A 22-year-old male presents into the ER after sustaining stab wound to the left chest. The injury is 2 cm left of the sternum at the level of his nipple. Initial vital signs include a pulse rate 88 bpm, GCS 15, respiratory rate 12 breaths/mm, and blood pressure 139/74 mmHg. Initial management of this patient should be: a. CXR b. pericardiocentesis c. left chest tube thoracostomy d. esophagogram e. echocardiogram 27. Which of the following is true regarding pancreatic trauma and its complications a. Complications of pancreatic trauma include pancreatic pseudocyst, abscess, hemorrhage, and fistula. b. Asymptomatic pseudocysts may be managed conservatively, as approximately 80% will spontaneously resolve . c. Pseudocysts which do not spontaneously resolve can be managed with percutaneous drainage, if ductal disruption is identified. d. If a pancreatic fistula is secondary to a major ductal disruption it will not respond to sphincterotomy and duct stenting via endoscopic retrograde pancreatic cholangiopancreatography (ERCP) e. Overall mortality from pancreatic injuries is approximately 45%. 28. Initial fluid resuscitation of a patient with multiple fractures and hypovolemic shock should be : a. blood transfusion b. hypertonic saline c. fresh frozen plasma d. Ringer’s lactate e. albumin 29.Which of the following is true concerning primary hyperaldosteronism?
242
a. Bilateral hyperplasia of the zona glomerulosa is about twice likely as either adrenal adenoma or adrenal carcinoma to be responsible for primary hyperaldosteronism. b. Unilateral adrenalectomy is usually curative for bilateral hyperplasia. c. All patients are hypokalemic d. these patients often present with metabolic alkalosis Primary hyperaldosteronism is a rare cause of hypokalaemic metabolic alkalosis 30. The chromosome carrying the gene associated with MEN I is: a. chromosome 14 b. chromosome 13 c. chromosome 12 d. chromosome 11 e. chromosome 10 31. Which of the following statements concerning gastrin is most accurate? a. Gastrin in high levels has been shown to cause gastric mucosal hypertrophy. b. Glucose in a meal is one of the most potent stimulators of gastrin release. c. Gastrin-secreting cells (G cells) are found only in the gastric antrum and pylorus. d.Regular administration of antacids will suppress gastrin release as long as pH is kept consistently above 5.0. e. Omeprazole, a proton pump inhibitor, can completely suppress gastrin secretion if taken regularly 32. Nasopharyngeal carcinoma is associated most strongly with which virus? a. CMV b. varicella zoster c.herpes simplex. d.Epstein-Barr virus (EBV). e. human papilloma virus. 33. Which of the following is false concerning the physiologic effects of volatile anesthetics? a. decreased tidal volume b. decreased blood pressure c. decreased respiratory rate d. bronchodilation e. decreased sensitivity to the ventilatory stimulant effects of carbon dioxide 34. Pseudomembranous enterocolitis is caused by the following organisms: a. Clostridium sporogenes b. Clostridium defficile c. Streptococcus faecalis d. Penicillin sensitive staphylocci e. Pseudomonas aeruginos
243
35. Which of the following statement characterize the biology of allotransplantation is false? a. The rejection response is systemic. b. The rejection response is learned. c. The rejection response involves both of immunologic and environmental factors. d. Allotransplantation evokes a cellular immune response. e. Allotransplantation evokes a humoral immune response. 36. A patient presents with a history of low back pain and sciatica. The pain radiates to the little toe, the ankle reflex is absent and the patient has difficulty in everting the foot. Which nerve root is likely to be trapped? a. L3 b. L4 c. L5 d. S1 e. S2 37. The umbilicus: a. Lies near the to the xiphoid than to the pubis b. Derives its cutaneous innervation from the eleventh thoracic nerve c. Transmits, during development, the umbilical cord two arteries and two veins d. Usually lies at about the level between the third and fourth lumbar vertebra e. Embriologically may transmit urine but never bowel contents 38. Regarding intestinal obstruction, all of the following are true Except: a. Its cardinal features are; abdominal pain, distension, vomiting and constipation b. Small bowel obstruction is commonly of adhesive aetiology in adults c. Erect plain abdominal X-Ray is usually requested to localize the site of obstruction d. Intussception is usually treated primarily with hydrostatic pressure e. Superior mesenteric artery embolus the commonest cause of acute mesenteric ischaemia Embolicarterial occlusion is the commonest cause of acute mesenteric ischaemia (45%) 39. Regarding inguinal hernia: a. The internal ring is a triangular aperture in fascia transversalis b. It is commenest cause of intestinal obstruction in infants and younger age group c. Indirect hernial sac lies lateral to pubic tubercle d. Hasselbach's triangle is formed medially by the medial edge of rectus abdominis, laterally by the inferior epigastric vessles and inferiorly by the inguinal ligament e. Direct inguinal hernia lies lateral to inferior epigastric vessles
244
The hernial sac may extend through the superficial inguinal ring above and medial to the pubic tubercle 40. Regarding blood transfusion: a. blood is collected in sterile bags and stored in Blood Banks At 0°C These units were stored in ACD solution at 4° C b. Platelets are nearly lost after 7 days in the stored blood and whole blood has lost most ofitsplatelets after 3 days of storage c. blood component therapy is the gold standard in transfusion therapy d. cross-mached blood and ELISA tested for HIV is one hundred percent safe e. whole blood transfusion is the ideal to treat chronic anaemia in an elderly patient
41. Achalasia is associated with all of the following EXCEPT: a. chagas ’ disease in South America b. dysphasia. c. weight loss. d. relaxation of the lower esophageal sphincter with swallowing . e. aspiration pneumonia, which may cause lung abscesses. 42. Deep venous thrombosis prophylaxis is appropriate for all of the following patients EXCEPT: a. a 67-year-old male undergoing a colectomy. b. a 21-year-old male undergoing an outpatient open inguinal hernia repair. c. a 21-year-old male in the ICU, comatose, with a closed head injury. d. a 60-year-old female undergoing open reduction and internal fixation of a hip fracture. e. a 60-year-old female undergoing reduction of a lung carcinoma. 43. Which of these histological findings is considered pathognomonic for Hodgkin’s lymphoma? a. Mikulicz’s cells b. Charcot-Leyden crystals c. Birbeck granules d. Reed-Sternberg cells e. Langerhan’s cells 44. Concerning the inguinal canal: a. It transmits the ilio-inguinal nerve b. The deep inguinal ring lies below the mid-point of inguinal ligament c. The superficial inguinal ring overlies the pubic tubercle d. Laterally , the anterior wall is made up of the external oblique aponeurosis e. Laterally, the posterior wall is formed by the conjoint tendon
45. Clinical picture of the lung tumor include all the following Except: a. Carcinoid syndrome
245
b. Superior venacava syndrome c. Horner's syndrome d. Superior laryngeal nerve paralysis e. Dyspnoea 46. One is true regarding the hiatus hernia: a. The most common herniation through the diaphragm is throughthe esophageal hiatus Hernias of the stomach through the esophageal hiatus are the most common b. Sliding hernia is one in which the greater curvature of the stomach rotate into the chest. c. The paraesophageal hernia is one in which the esophagus, esophagogastric junction and proximal stomach move into the chest. d. Sliding hernia is usually symptomatic and associated with reflux. e. Surgical repair is rarely indicated in the treatment of paraesophageal hernia. Hiatal hernias are often asymptomaticbut may be associated with reflux esophagitis Regarding obstructive jaundice:47 . a. Caused by hemolytic anaemia b. Urobilinogen is absent in the urine c. Carcinoma of the head of pancreas is the commonest cause d. Normal color stool e. b and c only
:The most important finding in the diagnosis of acute appendicitis is .48 a. vomiting b. fever c. leukocytosis d. right lower quadrant pain and tenderness e. constipation :Gastroesophageal reflux is best diagnosed with .49 a. radiography b. 24-hour pH monitoring of lower esophagus c. esophagoscopy d. documentation of a decrease in esophageal pH after HCI is placed in the stomach . e. acid-clearing swallowing test 50. Acute Epidural haematoma : a. due to middle meningeal artery tear b. can be treated conservatively c. classically there is contra lateral, dilated, fixed pupil d. needs operation within 24 hours e. a ad c only 51. Regarding colorectal carcinomas a. Right-sided tumours often present with a change in bowel habit b.75% of tumours can be reached with a flexible sigmoidoscope c. Over 20% of patients present with more than one primary tumour d. A Duke's B tumour has nodal metastases
246
75% of tumours are situated in the rectum and sigmoid colon (within 52. Regarding colonic polyps a. Metaplastic polyps are pre-malignant b. Adenomatous polyps are pre-malignant c. Villous adenomas are more common than tubular adenomas d. Genetic mutations can result in epithelial metaplasia Tubular adenomas are more common than villous or tubulovillous. 53. With an iron deficiency anaemia a. Both serum iron and total iron binding capacity are reduced b. The blood film shows a hyperchromic microcytic picture c. The reticulocyte count is low in relation to the degree of anaemia d. Serum ferritin levels are increased 54. The X-Ray you see below is : a. PTC (percutaneous trans hepatic cholangiography) b. Operative cholangiogram c. ERCP ( endoscopic retrograde cholangiopancreatography) d. Portography
Which of the following does not appear to cause early dumping .55 a) Serotonin b) Bradykinin c) Neurotensin d) Secretin ?Which does not predispose to Carcinoma of stomach .56 a) Low fat and protein diet b) Salted meat and fish c) Low Nitrate consumption d) High Complex carbohydrate consumption
247
?Which of the following is not true about gastric lymphoma .56 a) Stomach is the most common organ in the gi system which is involved in Lymphoma b) Peak incidence of lymphomas is seen in 6th-7th decade .c) Endoscopy usually reveals gastritis like picture or gastric ulcer .d) MALT lymphoma is the commonest variety Bicuspid aorticvalve is the most commonly associated cardiac congenital anomaly, present in as many as 50% to 80% of patients withcoarctation Which is false for GIST (Gastro intestinal stromal tumor) of .57 stomach a) It is the same as leiomyoma and leomyosarcoma as described .previously b) origin is from mucosa from the interstitial cells of Cajal c) Associated with C-Kit Mutation .d) Imatinab is a new effective drug for adjuvant therapy Most common site for cholangiocarcinoma is .58 a) Intrahepatic liver b) Hepatic duct bifurcation (c) Lower End of CBD (Common Bile Duct (d) Lower 1/3rd of CBD (Commmon Bile duct ?How much bile is produced by liver /day .59 a) 100-300 ml b) 300-500 ml c) 500-1000ml d) 500-1500 ml Which of the following is not a criteria for diagnosing Sphincter of .60 Oddi Dysfunction a) CBD diameter more than 12 mm on USG b) Decrease in CBD pressure after infusion of Cholecystokinin c) Ampullary pressure more than 40 mm Hg d) Delayed emptying of contrast from CBD after ERCP :All of the following increase bile secretion except .61 a) Vagal stimulation b) Secretin c) Cholecystokin d) Splanchnic nerve secretion ?Which is not a type of anal margin tumors .62 a) Basal cell carcinoma b) Epidermoid carcinoma c) Paget's disease (d) Bowen's disease (Anal Intraepithelial Neoplasia (What is not included in the triad of zollinger ellison syndrome (ZES .63 A) Hyperacidity B) Intractable duodenal ulcer disease C) Liver secondaries
248
D) Non beta islet cell tumor of pancreas Which is not a metabolic abnormality after gastrectomy .64 a) Weight loss b) Hypokalemia c) Anemia d) Osteoporosis True about Hydatid cyst of liver is .65 a) E. multilocularis is more common than E.Granulosus b) Extrahepatic 15% in lungs 2% in brain c) Adult worm infests herbivores .d) Indirect Heamagglutination (IHA) is the most specific serological test :All of the following are indications of operative cholangiogram except .66 a) Obstructive jaundice b) Liver metastasis c) Abnormal anatomy d) Recurrent biliary pancreatitis Peritoneo-Venous shunt is contraindicated in .67 a) Uncorrectable coagulopathy b) Spontaneous bacterial peritonitis c) Hepatic encephalopathy d) All 68. In a patient with positive occult blood in stool, the most likely diagnosis is: a. hemorrhoids b. duodenal ulcer c. colon cancer d. gastric ulcer e. esophageal cancer 69. In a 50 year old patient with rectal bleeding and internal piles, the most appropriate next step is: a. colonoscopy or barium enema b. hemorrhoidal ointments c. hemorrhoidectomy d. angiography e. abdominal ultrasound 70. Charactrized by a "bird's beak" appearance on Ba - swallow a. Gastric ulcer b. Colonic volvulus c. Esophageal stricture d. Esophageal achalasia e. Hiatus hernia 71. 16th year old female patient, known to be Situs inversus totale came to our hospital complaining of severe left lower abdominal pain, nausea and vomiting of 7 hours duration. On physical exam tenderness in the LIF, guarding was positive and Blumberg sign was +. CBC showed increase in WBCs with shift to the left . The most possible diagnosis is: a. Gastro enteritis
249
b. Meckel’s diverticulitis c. Acute appendicitis d. Imperforated hymen e. UTI 72. S.E is a 68 year-old female patient underwent inner lower quadrantectomy with clearance of the axilla because of mucinous carcinoma of the breast diagnosed by true cut needle biopsy. The pathology result was : Tumor size was 1,2 X 2 X1.5 Cm. Two out of 13 LNs were with Mets. Estrogen and progesterone receptors were negative and her-2 was positive +3. Chest and abdominal CT were free, Bone scan was with no evidence of mets. For this patient what is the stage of her breast Ca: a) Stage one b) Stage ll c) Stage lll d) Stage lV 73. For the same patient proper Adjuvant therapy include: a. Chemoradiotherapy + Tamoxifen 20 mg x 2 x day x 5 years b. Radiotherapy + Herceptin c. Chemotherapy alone d. Chemoradiotherapy + Herceptin e. Chemotherapy + Tamoxifen + Herceptin Herceptin® (trastuzumab), a targeted therapy for HER2 Positive Metastatic Breast .Cancer and HER2 Positive Gastric Cancer 74. The treatment of choice for a 40-year-old man who is found on endoscopy and biopsy to have a gastric lymphoma would be a. subtotal gastrectomy b. radiotherapy c. subtotal gastrectomy and radiotherapy d. chemotherapy e. wide local excision
75. Correct statements about the characterization and histologic features of Crohn's disease include a. A marked connective tissue reaction is present in the bowel wall and mesentery b. Cicatrization and bowel stenosis are frequent problems c. Exacerbations and remissions are characteristic of the disease d. Patients are more likely to develop fistulas into bowel or bladder than to experience free perforations e. Patients with long-standing Crohn's disease have an increased incidence of small bowel carcinoma
76. A small bowel fistula will not close during a course of bowl rest and Total parenteral nutrition if there is a. active Crohn's disease b. an undrained abscess
250
c. distal obstruction d. epithelialization of the tract e. radiation enteritis What is the principal method of diagnosing acute .77 ?pulmonary embolism a. Magnetic resonance imaging b. Chest X-Ray c. Spiral computed tomography scanning d. Ultrasound 78. A 33-year-old man presents with a complaint of a slow-growing mass over his right pre auricular area. The lesion is fixed to the underlying structures and has recently become painful. Which of the following features strongly suggests that this patient's lesion is a malignancy? a. Overlying skin involvement b. Facial nerve paralysis c. Ipsilateral numbness of the tongue d. All of the above 79. A 45-year-old man presents with a complaint of hoarseness of 2 month's duration. On physical examination, a right-sided cervical lymphadenopathy is noted. The rest of the examination is normal. The surgeon recommends further evaluation using direct laryngoscopy. Which of the following statements regarding direct laryngoscopy is ?true a. It is performed under general anesthesia b. It allows the surgeon to palpate the involved structures c. Simultaneous biopsies can be performed d. It has the advantage of permitting both diagnostic and therapeutic intervention e. All of the above 80. A 28-year-old woman presents with a complaint of a growing, painless mass in her neck. Physical examination reveals a firm, fixed nodule measuring 2 cm on the right lobe of her thyroid. The surgeon recommends fine-needle aspiration (FNA) of the lesion instead of excisional biopsy.
What are the advantages of FNA over excisional biopsy? a. FNA requires only an office visit b. Anesthesia is not necessary c. FNA is associated with a decreased risk of tumor seeding d. All of the above
251
81. A. M a 30-year old underwent total thyroidectomy before 2 years because of papillary carcinoma of thyroid. Which is the most reliable lab test for follow up to the patient R\O recurrence of the disease a. TSH b. T3 c. Thyroglobuline d. gamaglobuline e. CEA 82. A 77-year-old man undergoes endoscopic ultrasonography as part of a workup for jaundice. He is found to have a tumor in the head of the pancreas. For this patient, which of the following findings would indicate that ?the tumor is unresectable a. Invasion of the superior mesenteric artery b. Peritoneal metastasis c. Metastasis to celiac lymph nodes d. All of the above 83.
A 71 year-old man, on non steroidal anti-inflammatory treatment for his back pain has had sudden onset severe epigastric pain while he was walking in his garden. He was transferred to the E.R, Chest X-Ray done and showed the picture in the Rt. What is the possible diagnosis a. Miocardial infarction b. Pulmonary embolism c. Perforated viscus d. Neumothorax
84. Lateral aberrant thyroid a. abnormally descended thyroid tissue in the lateral neck b. associated with thyroglossal duct cyst c. cervical lymph node metastases from papillary cancer d. cervical lymph node enlargement as a result of Hashimoto’s disease e. asymmetric goiter protruding predominantly to one side of the neck Sliding inguinal hernia is .85 .a-a hernia that contains omentum in it,s sac .b-a hernia that is easy reducible .c-a hernia that reaches scrotum .d-a hernia having a viscous part of its sac wall
which of the following is not a choice for immediate management of .86 blunt abdominal trauma patient with unstable vital signs a. IV fluids up to 2liters of RL running
252
b. CT abdomen with IV contrast c. colloid solutions or blood if needed d. Resuscitation at the operating room 87. regarding necrotizing fascitis all true except: a. mortality rate is between 50%-70% b. it is caused by mixed flora c. heamolysis may complicate the picture d. clostidium perferinges is the pathogen Necrotizing fasciitis is most commonly caused by an infection with group A Streptococcus, commonly known as “flesh-eating bacteria. 88. A 35- year old woman experiences acute onset of epigastric and right upper quadrant pain several hours after a large dinner. She has had similar episode in the past that resolved after a few hours. This episode persists, and she has fever and nonbilious vomiting. What is the most likely source of abdominal pain? a. Perforated ulcer b. Acute appendicitis c. Perforation following bowel obstruction d. Acute Cholecystitis e. Diverticulitis 89. Regarding peritonitis, which of the following statements is/are not true? a. Primary peritonitis is usually monomicrobial b. Chemical peritonitis often precedes bacterial contamination c. Multiple organisms are commonly cultured from peritoneal dialysis catheters d. Tuberculosis peritonitis has an insidious onset 90. Which of the following statements regarding the proliferative phase of wound healing is true? a. The formation of granulation tissue is an important component of the proliferative phase of wound healing. b. Granulation tissue includes collagen, macrophages, hyaluronic acid, fibroblasts, and capillary endothelial cells. c. Angiogenesis, fibroplasia, and epithelialization are key components of the proliferation phase of wound healing. d. Both angiogenesis and fibroplasia are stimulated by platelet-and fibroblast-derived growth factors. e. All of the above 91. All of the following are associated with morbid obesity except? a. Coronary artery disease b. Sleep apnea c. Reduction in life expectancy d. Asthma e. None of the above 92. Which condition contraindicates using enteral feedings? a. Recent surgery b. High-output enteric fistula c. Head trauma d. Cachexia e. All of the above
253
93. Regarding post burn sequelae, all of the following statements are correct except? a. All second-and third-degree burns produce permanent scarring b. The incidence of hypertrophic scar formation is less after excision and skin grafting than with wounds that heal spontaneously. c. Hypertrophic scars are best treated by early excision and would closure. d. Burn scar hypopigmentation and irregularities can be signif-icantly improved by demabrasion and thin split-thickness skin grafting e. Squamous cell carcinoma is the most common carcinoma in an old burn scar. 94. All of the following statements regarding diabetic foot infections are true except? a. Foot infection occurs more frequently in diabetic than in nondiabetic patients b. The saline injection-aspiration method is the preferred method for obtaining reliable cultures c. Diabetic foot infections are commonly polymicrobial d. Osteomyelitis of the foot is frequently encountered in patients with a long history of diabetes and neuropathic ulcers e. None of the above 95. Which of the following eponyms is not used to describe a physical finding suggestive of metastatic or locally advanced malignancy? a. Blumer’s shelf b. Virchow’s node c. Grey Turner’s sign d. Krukenberg’s tumor e. Sister Mary Joseph’s sign 96. A Marjolin’s ulcer arises in areas exposed to a. External beam radiation b. Thermal injury c. Pressure d. Lymphedema e. None of the above 97. The appropriate therapy for Paget’s disease of the nipple is: a. Topical steroid cream b. Topical antifungal medication c. Intralesional steroid injection d. Resection 98. The most common location for a primary adenocarcinoma of small bowel is: a. Duodenum b. Jejunum c. Ileum d. None of the above- the distribution is roughly equal 99. The most common complication following hemorrhoidectomy is: a. Fecal impaction
254
b. Bleeding c. Urinary retention d. Infection 100. A patient with a 1.5-cm carcinoid tumor of the mid appendix should undergo. a. An appendectomy only b. Partial cecuectomy AND Lymph node sampling to confirm negative margins c. Resection of the cecum, terminal ileum, and adjacent mesentery (en bloc resection) d. Right hemicolectom 1. The radial nerve. f) Is a branch of the anterior cord of the brachial plexus. g)
Is derived from the posterior primary rami of C5 to C7.
h)
Supplies the flexors of the arm.
i) Gives rise to the anterior interosseous nerve. j) Injury above elbow produces a classical wrist drop. 2. Regarding femoral canal all are true except: f) Lies lateral to the femoral vein. g)
Has the inguinal ligament as its anterior border.
h)
Has the lacunar ligament as its medial border.
i) Has the pectineal ligament as its posterior border. j) Contains the lymph node of Cloquet. 3. The following causes hypercalcaemia except: a) Sarcoidosis. b) Primary hyperparathyroidism. c) Acute pancreatitis. d) Metastatic bronchial carcinoma. e) Milk-Alkali syndrome. 4. Number of human chromosomes in human female are: f)
23 pairs +XX.
g)
21 pairs +XY.
h)
22 pairs +XY.
i)
22 pairs +XX.
j)
23 pairs +XY.
6. Potassium deficiency should be suspected in all the following except: j)
In cases of paralytic ileus.
k)
When the patient's reflexes are exaggerated.
l)
If there is a decrease in height and peaking of the T waves of an ECG.
m)
In alkalotic states.
6. Active immunization in case of tetanus: k)
255
Antitetanus human serum.
l)
Gives short period of protection.
m)
Given in case proved tetanus.
n)
Use of immunoglobulin.
o)
None of the above.
7. All of the following are signs of rised intracranial pressure except: a) Headache. b) Vomiting. c) Papilledema. d) Aphasia. e) Bradycardia. 8. All of the following are extra cranial hematomas except: a) Subcutaneos haematoma. b) Extra dural haematoma. c) Cephalohaematoma. d) Subgaleal hematoma. e) Subperiostial haematoma. 9. Glasgow coma scale all the following are true except: j)
Used for evaluation of comatose patient.
k)
It ranges from ( 3 to 15).
l)
Useful for neurological follow up.
m)
Useful for pupils evaluation.
n)
Best motor response given 6 point.
10. All the followings are Indications for central line insertion EXCPET: r. Massive fluid replacement s. Massive blood replacement t. Measurement of central venous pressure u. Prolonged Intervenes fluid therapy 11. Most common early complication of central venous line is: a) Sepsis b) Pneumothorax c) Thoracic duct injury d) Thrombosis 12. The following are Complications of shock: a) Acute Respiratory failure b) Acute myocardial infarction c) Acute renal failure d) A&C only e) All the above
256
13. Causes of delayed union of fractures includes all the following EXCPET: a) Compound fracture b) Infection c) Adequate immobilization d) Poor blood supply 14. In Head injury C.T. scan is indicated in the following k) Aphasia after the injury l) Deterioration of level of consciousness m) Skull fracture with persistent headache n) A&B only o) All the above 15. Most important steps in management of head injury include: P. Prevent hypoxia Q. Prevent Dehydration R. Assure Brain Metabolism S. Prevent secondary brain injury T. All the above 16. Tension pneumothorax P. is the commonest type of chest injuries Q. Needs urgent X-Ray chest R. Is a clinical Diagnosis S. Causes flat neek viens T. Treated by thoracotomy tube after chest X-ray. Rib fractures are the most common type of chest trauma, occurring in more than 60% of patients admitted with blunt chest 17. Calcitonin hormon is secreted to the blood circulations from: j)
Parathyroid gland.
k)
Parafollicular cells of thyroid gland.
l)
Supra renal gland.
m)
Pituitary gland.
n)
Gonads.
18. Regarding tension pneumothorax, the first step in the management is: k)
Obtaining a stat chest x-ray.
l)
Cricothyroidectomy.
m)
Pass an endotracheal tube.
n)
Starting oxygen by a valve-mask device.
o)
Chest decompression needle.
19. The following are features of thyrotoxicosis except: j) Weight gain. k)
Palpitations.
l) Proximal myopathy. m)
Increased skin pigmentation.
n)
Pretibial myxoedema.
20. The following is a clinical feature of Horner's syndrome: i) Miosis j) Failure of abduction of the orbit
257
k)
Increased sweating on the contralateral side of the forehead
l) Exophthalmos. e)
All are true
21. In role of nine extent of burn if entire trunk is burned it will be equal to: k)
9% body surface area.
l)
18% body surface area.
m)
36% body surface area.
n)
27% body surface area.
o)
45% body surface area.
22. Trachlea (4th) cranial nerve supply : f)
Lateral rectus muscle of th eye.
g)
Medial rectus muscle of the eye.
h)
Superior obligue rectus muscle of the eye.
i)
Superior oblique muscle of the eye.
j)
Muscles of the upper eye lid.
23. Regarding pathological terms : k)
Hypertrophy is an increase in tissue size due to increased cell number.
l) Hyperplasia is an increase in tissue size due to an increase in cell size. m)
Atrophy is an increase in tissue size due to disuse.
n)
Metaplasia is a change form one abnormal tissue type to another.
o)
A hamartoma is a developmental abnormality.
24. Regarding nephroblastomas: f) They are otherwise known as a Wilm's tumour. g)
Account for 10% of childhood tumours.
h)
The commonest presentation is with an abdominal mass.
i) Most commonly present between 2nd and 4th year of life. j) All are true. 25. Regarding fluid losses in a major burn all are true except: k)
Are maximal between 12 and 24 hours after the injury.
l) Are related to the age of the patient. m)
Are not related to the weight of the patient.
n)
Are related to the area burnt.
o)
Are not related to the burn duration.
26. In obstructive jaundice : p)
Urinary conjugated bilirubin is increased.
q)
Serum unconjugated bilirubin is increased.
r) Urinary urobilinogen is increased. s)Serum conjugated bilirubin is reduced. t) Faecal stercobilinogen is increased. 27. Regarding Hydatid disease:
258
p) Is due to Ecchinococcus granulosa. q) Man is an accidental intermediate host. r) The liver is the commonest site of infection. s) Can be diagnosed by the Casoni test. t) All are true.
28. The first aid of treatment in fracture of cervical spine should be: j) Cervical spine x-ray. k)
Analgesia.
l) Neck immoblization. m)
Cervical traction.
n)
Non of teh above.
30. Regarding local anaesthesia: a)
Local anaesthetics act on small before large nerve fibres
b)
Adrenaline reduces absorption and prolongs the local effects
c)
Preparations containing adrenaline are safe to use on digits and appendages
d)
Lignocaine has a longer duration of action than bupivicaine.
e)
All are false.
low-dose vasoconstrictor, such as adrenaline (1 in 200,000),reduces local blood flow, slows the rate of absorption and prolongsthe duration of local anaesthetic effect 31. Small bowel obstruction often results in: (all correct except one) u)
Hyperkalaemia.
v)
Metabolic alkalosis.
w)
Oliguria.
x)
Hypovolaemia.
y)
Severe dehydration.
32. A serious intra-abdominal injury in a comatose patient may be diagnosed by: (all are correct except one) l) Abdominal paracentesis. m)
The observation of bruising pattern on the abdominal wall.
n)
Falling of heamoglobin values.
o)
The presence of marked abdominal distetion.
p)
The presence of diarrhea.
33. A perforated duodenal ulcer
259
l)
Usually lies on the anterior or superior surface of the duodenum.
m)
Usually presents with the acute onset of severe back pain.
n)
Produces radiological evidence of free gas in the peritoneum in over 90 percent of the patients.
o)
Is usually treated by vagotomy and pyloroplasty.
p)
Is usually treated conservatively.
34. Acute pancreatitis typically: (all correct except one) p)
Is accompanied by hypocalcaemia.
q)
Produces paralytic ileus.
r)
Is associated with a pleural effusion.
s)
Produces pyloric stenosis.
t)
Upper abdominal pain and vomiting.
35. Biliary colic typically: l) Occurs 3 to 4 hours after meals. m)
Lasts 5 to 20 minutes.
n)
Radiates from the upper abdomen to the right subscapular region.
o)
Is made better by deep inspiration.
p)
B&C only.
36. In post operative DVT, the following are true except: l) Clinical DVT occures in the 4th post operative day. m)
If complicated by pulmonary embolism, it occures usually after the 7 th post operative day.
n)
The process of DVT starts preoperatively with the induction of anaesthesia .
o)
When discovered we should start the patient on coumadin "Warfarin anticoagulation".
p)
It may lead to chronic venous in suffering as a complication of DVT.
37. In acute appendicitis all of the following are true except: j) Anorexia. k)
Abdominal pain usually preceedes vomiting.
l) Pain after begins in the paraumbilical region. m)
Constipation diarrhea may occur.
n)
Dysuria excludes the diagnosis.
38. The most common cause of massive haemorrhage in the lower gastroinfestinal tract is : k)
Carcinoma.
l) Diverticulosis m)
Diverticulitis
n)
Polyp.
o)
Ulcerative colitis.
39. Painless haematuria is the leading presentation of : p)
Renal cell carcinoma.
q)
Transitional cell carcinoma of the bladder .
r) Ureteric stone. s)
260
Pelvi-ureteric obstruction.
t) Ureterocele. 40. All of the following are complications of massive blood transfusion except: f) Acute congestive heart failure. g)
Transmission of infection.
h)
Hypercalcaemia.
i) Hyperkalaemia. j) Transfusion reactions. 41. Complication of undescended testis include all of the following except : f)
Malignant degeneration.
g) Increased susceptability to trauma. h) Increased spermatogenesis. i)
More liable to testiculer torsion.
j)
Psychological complication.
42. The recurrent laryngoeal nerve is branch of : j)
Facial nerve.
k) Glosso-pharyngeal nerve. l)
Cervical plexus.
m) Vagus nerve. n) Brachial plexus. 43. The thyroid tumor which is may be associated with pheochromocytoma is : p)
Papillary carcinoma.
q)
Medullary carcinoma.
r) Follicular carcinoma. s)Anaplastic carcinoma. t) Malignant lymphoma. 44. The most common pancreatic cyst is : l)
Dermoid cyst of the pancreas.
m)
Hydatid cyst of the pancreas.
n)
Pancreas pseudocyst.
o)
Pancreatic cystadenoma.
p)
Congenital cystic disease of the pancreas.
45. The anatomical division between the anus and rectum : k)
Lateral haemorrhoidal groove.
l) Inter haemorrhoidal groove.
261
m)
Dentate line.
n)
Arcuate line
o)
Ano-rectal ring.
The demarcation between the rectum above and the anal canal below is the anorectal ring or anorectal flexure 46. The comments type of Anorectal abscess is: i) Ischio rectal j) Perianal k) Submucons l) Pelvirectal 47. Anal Fissure: k) Usually anterior l) May be caused by previous anal surgery m) Can cause dark bleeding PR. n) Sometimes is painful o) Treated by steroids 48. Neonatal duodenal obstruction: k) May be associated with down's syndrome. l) Is more frequently found in premature infants. m) Typically presents with gross abdominal distension. n) Usually presents with vomiting of non-bile stained fluid o) B&C only. Duodenal atresia is the most common intestinal atresia Patients are frequently premature infants Prenatal diagnosis Ultrasonography: double bubble 49. acute superior mesenteric artery occlusion: (all correct except one) l) Characteristically presents with sudden pain and tenderness of increasing intensity. m) Is frequently accompanied by overt or occult blood loss in the stools. n) Frequently produces peritonitis. o) Can usually be diagnosed on plain abdominal x-rays. p) Can be diagnosed by mesenteric artery ongiography. 50. Regarding the management of polytrauma: p) Death follow a trimodal distribution. q) X-ray after primary survey should be AP cervical spine, chest and pelvis. r) Cardiac tamponade is characterized by raised B.p, a low JUP. s) Assessment of uncomplicated limb fractures should occur during the primary survey. t) A and B only. 51. Injuries to the urethra (all are correct except one) a) Are more common in male. b) Are often caused by road traffic accidents. c) Are easily diagnosed on intra venous pyelography. d) Require urgent surgical treatment. e) Diagnosed by retrograde urethragraphy. 52 .Car seat belts when properly adjusted a) Prevent injuries to abdominal organs. b) May cause small bowel injuries. c) Do not reduce the incidence of head injuries of passengers involving in RTA. d) Protect the cervical spine during sudden acceleration . e) A & D only. 53. Patients with major burns: p) Are in a negative nitrogen balance. q) Have normal calorie requirements. r) Do not generally become anaemic. s) Are resistant to septicaemia. t) All of the above. 54 . In a healing fracture: (All correct except one)
262
a) The haematoma is initially invaded by osteoblasts. b) The tissue formed by the invading osteoblasts is termed osteoid. c) Calcium salts are laid down in the osteoid tissue. d) The final stage of repair is the remodelling of the callus. e) The callus formation is related to the amount of stress at fracture side. 55. In a colles’ fracture the distal radial fragment: k) Is dorsally angulated on the proximal radius. l) Is usually torn from the intra-articular triangular disc. m) Is deviated to the ulnar side. n) Is rarely impacted. o) Is ventrally displaced. 56. A malignant melanoma: a) Frequently arises from hair-bearing naevi. b) Frequently arises from junctional naevi. * c) Has a worse prognosis when it areses on the leg. d) Should be suspected in any big pigmented lesion. e) Non of the above is correct. Melanomas in the head and neck area havethe worst prognosis Poor prognosis is most likely due to delayeddiagnosis 57. Squamous cancer of the lip: a) Is most common in early adult life. b) Is more common in fair skinned subjects. c) Metastasises readily by the blood stream. d) Is preferably treated by radiotherapy once lymph node deposits are present. e) All of the above are correct. 58- Basal cell carcinomas: a) Usually metastasise to regional lymph nodes. b) Are less common than squamous cell carcinomas. c) Are characterised histologically by epithelial pearls. d) Are particularly common in oriental races. e) Non of the above is correct. 59- Fiboadenomata of the breast: a) Are commonest in early adult life. b) Are indiscrete and difficult to distinguish. c) Are usually painless. d) Resolve without treatment. e) A&C only. 60. Paget’s disease of the nipple: a) Usually presents as abilateral eczema of the nipple. b) Is always related to an underlying breast cancer. c) Indicates incurable breast cancer. d) Has non-specific histological characteristics. e) A&C only. 61- stones in the common bile duct: a) Are present in nearly 50 per cent of cases of cholecystitis. b) Often give rise to jaundice, fever and biliary colic. c) Are usually accompanied by progressive jaundice. d) Are usually associated with a distended gallbladder. e) A&D only. 62- Colonic polyps: (all correct except one) p) Are associated with colonic cancer. q) May be hereditary. r) Should not be removed if they are asymptomatic. s) May be hyperplastic.
263
t)
Are commonly adenomatous.
1. The following are absorbable sutures a. Catgut b. Silk c. Polyamide (Nylon) d. Polyglyconate (Maxon) e. Polyglactin (Vicryl)
2. During surgery on the submandibular gland a. An incision on the lower border of the mandible is safe b. The submandibular gland is seen to wrap around the posterior border of mylohyoid c. The facial artery and vein are divided as they course through the deep part of the gland d. The hypoglossal nerve is seen to loop under the submandibular duct e. Damage to the lingual nerve will cause loss of sensation to the posterior third of the tongue 3. Regarding pancreatic carcinoma false a. 90% are ductal adenocarcinomas b. Less than 20% occur in the head of the gland c. The usual presentation is with pain, weight loss and obstructive jaundice d. Ultrasound has a sensitivity of 80-90% in the detection of the tumour e. Less than 20% of patients are suitable for curative surgery 4. Regarding the management of major trauma a. Deaths follow a trimodal distribution b. X-rays after the primary survey should be of AP Cervical spine, chest and pelvis c. Cardiac tamponade is characterised by raised BP, low JVP and muffled heart sounds d. Assessment of uncomplicated limb fractures should occur during the primary survey e. Deterioration of the casualty during the primary survey should lead to the secondary survey 5. Regarding appendicitis a. The risk of developing the illness is greatest in childhood b. Mortality decreases with age . c. 20% of appendices are extraperitoneal in a retrocaecal position d. Faecoliths are present in 75-80% of resected specimens e. Appendicitis is a possible diagnosis in the absence of abdominal tenderness 6. Regarding stones in the gallbladder a. Cholesterol stones are the least common b. Pigment stones are due increased excretion of polymerised conjugated bilirubin c. Are not a risk factor for the development of gallbladder carcinoma d. 90% of gallstones are radio-opaque e. A mucocele of the gallbladder is caused by a stone impacted in Hartmann's pouch 7. Stones in the common bile duct the false is a. Are found in 30% of patients undergoing cholecystectomy (Without pre-op ERCP)
264
b. Can present with Charcot's Triad c. Are suggested by an bile duct diameter >8mm on ultrasound d. ERCP, sphincterotomy and balloon clearance is now the treatment of choice e. If removed by exploration of the common bile duct the T-tube can be removed after 3 days 8. Regarding crystalloid solutions the false is a. Normal saline contains 154 mmol sodium and 154 mmol of chloride b. 3 litres of dextrose saline in a day will provide 90 mmol of sodium c. 2 grams of potassium chloride is equal to 57 mmol of the salt d. Hartmann's solution contains calcium and bicarbonate e. The daily maintenance potassium requirement of a 40 Kg woman is about 40 mmol Hartmann's solution contains sodium, potassium, chloride, calcium and lactate (the precursor of bicarbonate) 2 grams of potassium chloride is equal to 27 mmol of the salt 9. Solitary thyroid nodules the false is a. Are more prevalent in women b. In the adult population less than 10% are malignant c. Less than 20% of scintigraphically cold nodules are malignant d. The risk of a hot module being malignant is negligible e. Should be surgically removed in all patients 10. Regarding abdominal wall hernias the false is a. Almost 100,000 hernia operations are performed annually in the United Kingdom b. Over 60% of inguinal hernias are indirect c. In women inguinal hernias are as common as femoral hernias d. The mortality assocaited with strangulation is over 10% e. The mortality has reduced dramatically over the past 30 years 11. The femoral canal the false is a. Lies medial to the femoral vein b. Has the inguinal ligament as its anterior border c. Has the lacunar ligament as its lateral border d. Has the pectineal ligament as its posterior border e. Contains the lymph node of Cloquet 12. Intermittent claudication the false is a. Affects less than 1% of men over the age of 50 years b. At 5 years 10% of claudicants will have progressed to an amputation c. At 5 years 20% of claudicants will have died from ischaemic heart disease d. Is usually associated with an ankle / brachial pressure index (ABPI) 0.3- 0.7 e. Is associated with a fall in the ABPI on exercise with delayed recovery 13. The pathology of ulcerative colitis the false is a. Shows full thickness inflammation b. The rectum is almost always involved c. 10% patients have terminal ileal disease d. Enterocutaneous or intestinal fistulae are less common
265
e. The serosa is usually normal 14. Regarding benign breast disease the false is a. Cyclical mastalgia is the commonest reason for referral to the breast clinic b. Fibroadenomas are derived from the breast lobule c. Lactational breast abscesses are usually due to Staph aureus d. Duct ectasia is less common in smokers e. Atypical lobular hyperplasia is associated with an increased risk of breast cancer
15. Regarding anal fissures the false is a. 10% occur in the posterior midline b. Multiple fissures suggest a diagnosis of tuberculosis or Crohn's Disease c. 50% of acute fissures heal with the use of a bulking agent d. Sphincterotomy has a success rate of over 90% e. Sphincterotomy is associated with minor faecal incontinence in over 15% of patients 16. Fibroadenomas the false is a. Are benign monoclonal neoplasms b. Most commonly present in late adolescence or the early 20s c. Should be diagnosed by triple assessment d. At least 30% reduce in size over a 2 year period e. Can be safely managed conservatively 17. Warfarin the false is a. Reduces the concentration of vitamin A dependent clotting factors b. Has a half life of about 36 hours c. Crosses the placenta and should be avoided in pregnancy d. Doses should be reduced in liver disease e. An INR of between 2.0 and 3.0 is appropriate in the treatment of DVT 18. Heparin the false is a. Is a heterogeneous mixture of sulphated polysaccharides b. Potentiates the actions of antithrombin III c. Has a half life of 12 hours d. Can be reversed by protamine sulphate e. Can induce an idiosyncratic thrombocytopenia Low molecular weight heparins have a slow onset (90 minutes) and a half- life of 4 hours. Their effects last 12 hours he half-life of intravenous heparin is 1 to 2hours and the duration of its effects are short 19. The following cause hypercalcaemia except a. Sarcoidosis b. Primary hyperparathyroidism c. Acute pancreatitis d. Metastatic bronchial carcinoma
266
e. Milk-Alkali syndrome 20. Regarding oesophageal cancer the false is a. Squamous carcinomas predominate in the upper 2/3 of the oesophagus b. Overall 5 year survival is greater than 50% c. Tylosis genetically predisposes to the disease d. 15% of adenocarcinomas are associated with Barrett's oesophagus e. For palliation an Atkinson tube can be inserted endoscopically Although the overall 5-year survival rate for esophageal cancer is low, 15.8% 21. Infantile hypertrophic pyloric stenosis the false is a. Occurs with a male : female ratio of 4:1. b. Sons of affected mothers have a 20% risk of developing the lesion c. Invariably presents between three and four weeks of age d. Typically presents with nonbilious vomiting e. Surgical treatment is by Heller's Cardiomyotomy 22. Oesophageal atresia the false is a. Is often associated with a distal trachea-oesophageal fistula b. Polyhydramnios is often present late in pregnancy c. 50% have other associated congenital abnormalities d. Contrast X-ray studies are necessary to confirm the diagnosis e. Post-operatively over 30% develop oesophageal strictures 23. The following are features of Fallot's Tetralogy except a. An atrial septal defect b. Pulmonary stenosis c. Right ventricular hypertrophy d. A right to left cardiac shunt e. Cyanotic attacks during feeding and crying 24. The functions of a tracheostomy are to except a. Bypass an upper airway obstruction b. Reduce the anatomical dead space c. Increase airway resistance d. Protect against aspiration e. Allow frequent airway suction 25. Medullary carcinoma of the thyroid the false is a. Is a tumour of the parafollicular C cells b. Produce thyroxine as the principle hormone c. 90% of cases are sporadic d. Can occur as part of the MEN type II syndrome e. Total thyroidectomy is the surgical treatment of choice 26. The following are features of thyrotoxicosis except a. Weight gain
267
b. Palpitations c. Proximal myopathy d. Increased skin pigmentation e. Pretibial myxoedema 27. Regarding abdominal aortic aneurysms the false is a. They commonly remain symptomless until they rupture b. The risk of rupture increases with aortic diameter c. Elective repair should have a 30-day mortality of less than 5% d. Emergency repair has a 30-day mortality of less than 10% e. The benefits of surgery for small (4.0-5.5 cm) is unproven However, urgent aortic repair is still associated with substantial mortality and morbidity in the range of 30–40 % 28. In patients with carotid artery disease a. A bruit is a reliable sign of the degree of stenosis b. Atheroma is most commonly seen in the external carotid artery c. An embolic event often results in an ipsilateral hemiplegia d. Prophylactic aspirin reduces the risk of a stroke e. Surgery is of proven benefit in those with asymptomatic stenoses 29. Hepatocellular carcinoma the false is a. Has a high incidence in East Africa and South-east Asia b. Its worldwide incidence parallels the prevalence of Hepatitis B c. Mycotoxins (e.g. aflatoxin) are an important aetiological factor d. Over 80% of tumours are surgically resectable e. Liver transplantation offers the only chance of cure in those with irresectable disease 30. Serum alpha fetoprotein the false is a. Is increased in testicular tumours b. In testicular tumours is produced by trophoblastic elements c. Is increased in over 70% patients with hepatocellular carcinoma d. In patients with hepatocellular carcinoma levels correlate well with size of the tumour e. In patients with hepatocellular carcinoma levels fall following resection of the tumour Alpha-fetoprotein is produced by yolk sac elements and is notproduced by seminomas 31. Regarding testicular tumours a. They are the commonest malignancy in young men b. Peak incidence for teratomas is seen at the age of 25 years c. Seminomas are radiosensitive d. Over 75% of Stage I teratomas are cured by surgery alone e. Chemotherapy rarely produces a cure in those with metastatic disease 32. In patients with ascites the false is a. A exudate has a protein content of greater than 30 g per litre b. Both malignancy and infection result in a transudate c. Ovarian carcinoma is the commonest malignant cause of ascites d. Cirrhosis results in marked sodium retention
268
e. Tumour cells increase the permeability of the peritoneum 33. Regarding opiate analgesia the false is a. Patient controlled analgesia (PCA) is more effective than intermittent parenteral dosing b. The total opiate dose is usually reduced with a PCA c. Fentanyl is more lipid soluble than morphine d. Epidural morphine can result in late respiratory depression e. Epidural and parenteral morphine can be safely administered together
34. Intussusception the false is a. Is most common in children from 6 to 12 years b. Presents with colicky abdominal pain, rectal bleeding and an abdominal mass c. 10% present with diarrhoea and vomiting suggestive of gastroenteritis d. If no shock or peritonitis hydrostatic reduction can be attempted e. A Meckel's diverticulum can induce an intussusception 35. Hirschsprung's disease the false is a. Is due to absent ganglion cells in Auerbach's plexus b. 10% cases have involvement of the recto-sigmoid segment c. 80% cases present with delayed passage of meconium in the first 24 hours after birth d. The affected segment of bowel appears as cone-shaped contracted zone on barium enema e. On rectal biopsy there increased acetylcholinesterase containing cells in the muscularis 36. Laparoscopic cholecystectomy the false is a. Is usually performed using a four port technique b. The Veress needle is an 'open' technique for inducing the pneumoperitoneum c. A supraumbilical abdominal scar is a contraindication to laparoscopic cholecystectomy d. Dissection of Calot's triangle should be performed before the cystic duct is clipped e. Most series report a conversion rate of less than 5% Induction of Pneumoperitoneum Access to the abdominal cavity is achieved by either a closed or open technique superior to the umbilicus. With the closed method, the pneumoperitoneum isinduced by means of a Veress needle placed 37. In obstructive jaundice the false is a. Urinary conjugated bilirubin is increased b. Serum unconjugated bilirubin is increased c. Urinary urobilinogen is reduced d. Serum conjugated bilirubin is increased e. Faecal stercobilinogen is reduced 38. Regarding the anatomy of the inguinal canal the false is a. The internal ring lies midway between the symphysis pubis and anterior superior iliac spine b. The internal ring lies medial to the inferior epigastric vessels c. The external oblique aponeurosis forms the anterior boundary
269
d. The inguinal ligament forms the inferior boundary e. The conjoint tendon forms the medial part of the posterior wall Its entrance, the internal inguinal ring, lies just lateral to the inferior epigastric vessels 39. Papillary carcinoma of the thyroid the false is a. Can be reliably diagnosed using fine needle aspiration cytology b. Is almost always unifocal c. Histologically displays Psammoma bodies d. Typically spread to the cervical lymph nodes e. Requires a total thyroidectomy for large tumours
40. Regarding bladder tumours the false is a. 90% are squamous carcinomas b. Painless haematuria is the commonest presentation c. Cigarette smoking is an important aetiological factor d. 80% of tumours are superficial (i.e. no muscle invasion) e. Superficial tumours are often well controlled by transurethral resection 41. Regarding ureteric calculi the false is a. Are most often composed of calcium oxalate or phosphate b. Most stones of those less than 5 mm in diameter pass spontaneously c. Extracorporeal lithotripsy is useful for stones in the upper third of the ureter d. About 30% of patients require open surgery to remove the stone e. An obstructed ureter in the presence of infection is a surgical emergency 42. Regarding bladder calculi the false is a. The incidence has fallen markedly in this country since the late 19th century b. They may be totally asymptomatic c. They are more prevalent in patients with chronic urinary sepsis d. They are associated with squamous metaplasia of the bladder mucosa e. They increase the risk of transitional cell carcinoma 43. Ectopic pregnancy the false is a. Occurs in about 1% of pregnancies b. The risk is increased in those with a history or pelvic inflammatory disease c. Usually presents between 2 and 4 months of gestation d. Patients usually have a positive pregnancy test e. if shocked early laparotomy is essential 44. Tetanus the false is a. Is due to an infection with a gram-positive spore forming rod b. The organism produces a powerful endotoxin c. The toxin prevents the release of inhibitory neurotransmitter d. Clostridium tetani is sensitive to penicillin e. Risus sardonicus is the typical facial spasm
270
This organism lives on dead tissue in the wound of an animal but produces a powerful exotoxin that causes the disease tetanus 45. Regarding pathological terms a. Hypertrophy is an increase in tissue size due to increased cell number b. Hyperplasia is an increase in tissue size due to an increase in cell size c. Atrophy is an increase in tissue size to disuse d. Metaplasia is a change form one abnormal tissue type to another e. A hamartoma is a developmental abnormality
33.
The initial maneuver to establish an airway in a patient with multiple injuries is: a. Oropharyngeal airway. b. Uncuffed endo-tracheal tube. c. Suctioning foreign debris and lifting up the mandible. d. Cuffed endo-tracheal tube. e. Tracheostomy. 34. Which is the most commonly injured intra-abdominal organ in blunt trauma? f. Pancreas. g. Kidney. h. Spleen. i. Stomach. j. Colon. 35. The most important principle in the management of severe hemorrhagic shock is to: a) Obtain blood for possible type-specific transfusion. b) Place CVP lines early for fluid resuscitation and monitoring. c) Rapidly infuse colloid fluids. d) Apply MAST garment. e) Secure the airway and adequate ventilation. 36. A 6-year-old child sustained the following injuries in an accident. Which one of the followings should be managed first? a. Extradural hematoma. b. Pneumothorax. c. Hollow viscus injury. d. Renal injury. e. Liver laceration. 37. All the following are complications of massive blood transfusion except: a. Hypothermia. b. Hypocalcaemia. c. Hypokalaemia. d. Acidosis. e. DIC. 6.Severe limb pain of sudden onset can be caused by all the following conditions except: f. Acute ischaemia. g. Deep venous thrombosis. h. Muscle tear. i. Sciatica. j. Bone fracture.
271
?Which of the following is not a classic sign of a basal skull fracture .7 a. Battle sign b. racoon eyes c. hemotympanum d. Gray –Turner sign e. CSF rhinnorhea/ottorrhe .e. thiamine :Tension pneumothorax is best diagnosed with.8 A, stat CT scan b. chest x-ray c. watch and wait d. clinical exam .e. none of the above :While doing thoracocentesis, it is advisable to introduce needle along .9 .a. Upper border of the rib .b. lower border of the rib .c. In the center of the inter-costal space .d. In anterior part of inter-costal space .e. any where 10. The most significant immediate complication associated with pelvic fracture is: a. Hemorrhage. b. Rectal or vaginal lacerations. c. Sciatic nerve injury. d. Infection. e. Myositis ossificans. 11 - Which is not contributory to Glasgow Coma Scale? f. obey commands g. localizes painful stimuli h. open eyes to calling i. incomprehensible sounds j. 5mm pupils 12. In role of nine extent of burn if entire trunk is burned it will be equal to: f. 9% body surface area. g. 18% body surface area. h. 36% body surface area. i. 27% body surface area. j. 45% body surface area. 13. A 54-year-old man presents with two episodes of hematemesis since yesterday. The most likely cause of this patient’s upper gastrointestinal bleeding is: a. Gastritis. b. Esophagitis. c. Esophageal varices. d. Peptic ulcer disease. e. Mallory-Weiss tear. 14. Adequate minimum urine output in a 70kg man during resuscitation is f) 35 ml/hr g) 20 ml/hr h) 50 ml/hr
272
i) 45 ml /hr j) 60 ml/hr 15. A 28 year old lady complains of painful defecation associated with fresh per-rectal bleed. Possible diagnosis to consider: f) g) h) i) j)
hiradenitis suppurativa dermoid cyst pilonidal sinus anal fissure pruritis ani
16. Which is not a cause of pancreatitis? f) hypercalcaemia g) hypokalaemia h) hyperlipidaemia i) obstruction at ampulla of Vater j) thiazide 17-All of the following can be treated conservatively in a stable trauma patient except: f. Lung contusion. g. Liver laceration. h. Kidney laceration. i. Splenic hematoma. j. Perforation of the small intestine. 18.What is the commonest presentation of a nephroblastoma? f) Abdominal pain. g) Haematuria. h) Fever. i) Abdominal mass. j) Loss of weight. 19. Which of the following are not found in peritonitis? k. Patient is lying still l. Guarding m. Rebound tenderness n. Hyperactive bowel sounds o. Rigid abdomen The following is an indication for thoracotomy in chest injur .20 p. q. r. s. t.
Cardiac tamponade Uncontrolled pulmonary air leakage Perforation of thoracic esophagus Blood loss of 200ml/hr for 2-3 hrs via chest tube All of the above
THYROID GLAND & NECK SWELLINGS 1. The best test to monitor adequacy of L-Thyroxine therapy is : A. radioactive isotope thyroid scan
273
B. thyroglobulin blood level C. total T4 blood level D. free T3 blood level E. blood level of TSH 2. Hashimotos thyroiditis : A. is of viral origin B. presents usually by nodular goiter C. can be diagnosed by thyroid antibodies and fine needle aspiration of thyroid D. is often premalignant E. must be treated always by surgery 3. Thyrotoxicosis usually manifests by all of the following EXCEPT: A. loss of weight inspite of good appetite B. a recent preference to cold C. palpitation D. more common in male than in female E. excitability of the patient 4. Exophthalmos : A. is always present in thyrotoxicosis B. is always bilateral C. can cause diplopia in severe cases D. decreases by hypothyroidism E. is cured by surgery to the thyrotoxic goiter 5. Complications after thyroidectomy include all the following EXCEPT: A. parathyroid insufficiency B. recurrent laryngeal nerve paralysis C. tracheomalacia D. thyrotoxic crisis(storm) on operating on inadequitly prepared thyrotoxic patient E. hypercalcemia 6. Regardig Carcinoma of the thyroid gland: A. common diffrenciated types are the papillary and the follicular carcinoma B. medullary carcinoma of the thyroid is treated only by resection of the involved lobe C. papillary carcinoma never metastasize to cervical lymph nodes D. anaplastic carcinoma is more common in the young age group 7. Tuberculous cervical lymphadenitis: A. is caused by bovine tuberculous bacillus rather than the human bacillus B. usually occurs through the tonsil of the corresponding side C. collar-stud abscess is never a feature of T. B adenitis D. is treated always by surgery E. biopsy is not necessary for diagnosis 8. Branchial cyst: A. develops from the third branchial cleft B. presence of cholesterol crystals in aspirated fluid from the cyst is pathognomonic to it C. should be treated with repeated aspiration D. is not related to carotid vessles,hypoglossal and spinal accessory nerves E. is never inflamed 9. Cystic hygroma: A. develops as a sequestration of a portion of a jugular lymph sac B. typically occupies the upper one third of the neck C. occasionally may occur in cheek ,axilla and groin but exceptionally in tongue and mediastinum D. repeated aspiration is the only treatment E. A and C are correc 10. Carotid body tumour: A. is chromffin paragranuloma
274
B. is usually bilateral C. is called chemodactoma or potato tumour D. is situated ot the bifurcation of the carotid artery E. C and D are correct 11. Differential diagnosis of lateral sided neck swelling: A. cannot be diagnosed by fine needle asoiration B. can be thyroglssal cyst C. can be a lymphoma when having other characteristic features D. cannot be diffrenciated into solid or cystic by ultrasound E. is never a reactive lymphadenitis or secondary malignant disease
THE BREAST 12. The breast : A. is a modified apocrine sweat gland B. overlies the third to the sixth rib C. is having usually more than 30 lacteferous ducts D. is drained only by six groups of axillary lymph nodes E. is having no attachment to skin 13. The commonest cause of bloody nipple discharge is : A. mammary ductectasia B. carcinoma of the breast C. lactational mastitis D. duct papilloma E. fibrocystic disease of the breast 14. All of the following may be manifestations of breast carcinoma EXCEPT A. peau d orange of covering skin B. no mass may be felt clinically C. enlargement of the ipsilateral axillary lymph nodes D. cervical spine metastasis E. none of the above 15. Bacterial mastitis: A. never occurs in lactating mothers B. is usually caused by streptococcus hemolyticus C. is mostly caused by staphylococcus aureus D. is treated only by incision and drainage E. is the same as mastitis of puberty 16. Signs of malignancy on mammogram may include all of the following EXCEPT : A. mass lesions with poorly defined irregular margins B. fine stipped soft tissue with periductal and not vascular microcalcifications C. thickening and retraction of the overlying skin D. dysplastic ductal pattern E. well circumscribed , homogenous,and often surrounded by a zone of fatty tissue 17. Early detection of carcinoma of the breast includes all of the following EXCEPT: A. self-examination just after menstruation B. screening mammography C. frequent consultations among the high risk group patients D. presence of fixed breast mass to chest wall with skin changes on clinical examination E. A, B, C, are correct 18. Regarding Paget s disease of the nipple : A. It is a benign condition
275
B. It is simply an eczematous lesion of the nipple C. It is treated by excision of the nipple D. It is usually diagnosed by biopsy of the suspected lesion E. the areola and the surrounding skin are never involved 19. in a patient with fibroadenosis of the breast ; A. cyst formation, adenosis, fibrosis,epitheliosis and papillomatosis are invariably present B. is premalignant C. pregnancy usually produces relief D. presentation may include pain, nipple discharge, and/ or breast lump E. all of the above are usually characteristics 20. Regarding gynaecomastia : A. it is hypertrophy of female breast B. it is hypertrophy of male breast C. it may associate leprosy and liver failure D. it may associate cimetidine, spironolactone, INH, or digitalis thrapy E. B, C, and D are correct 21. Regarding modified radical mastectomy all are correct EXCEPT : A. it is indicated recently for T1N0M0 B. pectoralis major muscle is excised C. axillary clearance is mandatory D. adjuvant radiotherapy must be given for all patients post-operatively E. the long thoracic nerve of Bell (nerve to serratous anterior) must be preserved but nerve to latismus dorsi might be sacrified 22. Mammary duct ectasia is characterized by the following EXCEPT : A. is defiened as primary dilatation of major ducts of breast in middle aged women B. may present with nipple retraction and Peau d orange picture C. is treated usually by simple mastectomy D. anaerobic superinfection commonly occurs in this recurrent periductal plasma cell mastitis E. is commonly pre-malignant 23. Regarding carcinoma of the breast : A. invasive intraductal carcinoma is the commonest form B. lobular carcinoma may present bilateral C. clinical staging is always correct and definit D. the medullary (anaplastic) type feels hard E. A, and B, are correct Choose only the ONE most Appropriate Answer 38.
39.
40.
276
Which of the following is not true about neurogenic shock? a. It may follow spinal fractures. b. There is tachycardia. c. The extremities are warm. d. There is bradycardia and hypotension. e. Vasopressors may be useful. All the following are complications of massive blood transfusion except: a. Hypothermia. b. Hypocalcaemia. c. Hypokalaemia. d. Acidosis. e. DIC. In idiopathic thrombocytopenic purpura all the following are present except: a. Megakaryocytes are present in the bone marrow.
41.
42.
43.
44.
45.
46.
47.
48.
277
b. Cortisone therapy may improve the condition. c. Marked enlargement of the spleen is present. d. Haemarthrosis is not a complication. e. The INR is normal. f. The disease is due to the presence of immunoglobulins. Regarding hyperkalemia all are true EXCEPT: a. Is mostly the result of renal failure. b. Is common with massive tissue destruction. c. Muscle rigidity is a manifestation of severe hyperkalemia. d. Causes a peaked T-wave on the electrocardiogram. e. Urgent treatment is by Ca gluconate and Na bicarbonate. One of the following is an anaerobic organism: a. Staphylococcus aureus. b. E. coli. c. Klebsiella. d. Pseudomonas aeroginosa. e. Bacteriodes. Which of the following tests will be affected after intake of non-steroidal antiinflammatory drugs: a. Coagulation time. b. PTT. c. INR. d. Bleeding time. e. Thrombin time. All of the following can be treated conservatively in a stable trauma patient except: a. Lung contusion. b. Liver laceration. c. Kidney laceration. d. Splenic hematoma. e. Perforation of the small intestine. The initial maneuver to establish an airway in a patient with multiple injuries is: a. Oropharyngeal airway. b. Uncuffed endo-tracheal tube. c. Suctioning foreign debris and lifting up the mandible. d. Cuffed endo-tracheal tube. e. Tracheostomy. Which is the most commonly injured intra-abdominal organ in blunt trauma? a. Pancreas. b. Kidney. c. Spleen. d. Stomach. e. Colon. In malignant melanoma which of the following correlates best with the prognosis? a. Age of the patient. b. Gender. c. Clark’s level. d. Site of the lesion. e. Breslew thickness of the lesion. Risk factors for breast cancer include the following except: a. Prior breast cancer. b. Mammary duct ectasia. c. Breast cancer has affected a sister.
49.
50.
51.
52.
53.
54.
55.
56.
278
d. Early menarche and late menopause. e. In situ lobular or duct carcinoma. The commonest histological type of breast cancer is: a. Duct carcinoma in situ. b. Lobular carcinoma in situ. c. Invasive duct carcinoma. d. Invasive lobular carcinoma. e. Paget’s disease of nipple. What is the commonest presentation of a nephroblastoma? a. Abdominal pain. b. Haematuria. c. Fever. d. Abdominal mass. e. Loss of weight.
All the following statements about follicular thyroid cancer are true except: a. It present at a later age than papillary carcinoma. b. It disseminates via hematogenous route. c. It is less common than papillary carcinoma. d. It is frequently multicentric. e. Bone is a favorite metastatic site. To prevent foot infection in a diabetic patient the following recommendations are all correct except: a. Careful trimming of toe nails. b. Monthly injection of the long-acting benzathene penicillin. c. Regular washing and daily inspection of the feet by the patient. d. Early treatment of taenia pedis infection. e. Avoidance of walking bare-footed. Which of the following tests is used to monitor the adequacy of anticoagulation with warfarin? a. Bleeding time. b. Clotting time. c. Activated partial thromboplastin time (APTT). d. Fibrinogen level. e. International normalized ratio (INR). f. Marjolin’s ulcer Severe limb pain of sudden onset can be caused by all the following conditions except: a. Acute ischaemia. b. Deep venous thrombosis. c. Muscle tear. d. Sciatica. e. Bone fracture. The following statements about gall bladder stones are all correct except: a. Most of these stones are radio-opaque. b. In many cases these stones produce no symptoms. c. Gall stones are present in most cases of gall bladder cancer. d. Ultrasonography is the preferred imaging study. e. Removal of the gall balder is the standard treatment of symptomatic cases The following statements about acute cholecystitis are all correct except:
57.
58.
59.
60.
61.
62.
a. The great majority of cases are caused by stones. b. The condition is more serious in diabetics. c. Differential diagnosis includes acute pancreatitis. d. The main presentation is jaundice. e. Initial treatment may be conservative. Which of the following is a common cause for massive lower gastrointestinal bleeding? a. Crohn’s colitis. b. Rectal polyp. c. Angiodysplasia of the colon. d. Carcinoma of the caecum. e. Solitary rectal ulcer. Which of the following is not true about achalasia of the cardia? a. It usually occurs in middle age. b. There are hyyperperistlatic waves in the body of the esophagus. c. It predisposes to squamous cell carcinoma of the esophagus. d. Manometric study is diagnostic. e. Pneumatic dilatation is a line of treatment Organisms found in the large bowel are mostly: a. Gram-positive cocci and anaerobes. b. Gram-negative cocci and anaerobes. c. Gram-negative bacilli and anaerobes. d. Gram-positive bacilli and anaerobes. e. Gram-negative cocci only. Which of the following operations is considered as clean contaminated operation? a. Herniorrhophy. b. Axillary block dissection. c. Ureterolithotomy. d. Urgent colectomy. e. Appendicular abscess drainage. During conservative treatment of a patient with adhesive intestinal obstruction, which of the following is an indication to proceed to surgery? a. Nasogastric suction of more than 2 L/24 hours. b. Exaggerated bowel sounds. c. Abdominal rigidity. d. Abdominal distension. e. Serum K level below 2.5 mEq/L. The most important prognostic index in breast cancer is: a. Age of the patient. b. Site of the lesion. c. Presence of lymph node metastases. d. Level of CEA. e. Presence of micro-calcification.
11. Which of the following are not found in peritonitis? p. Patient is lying still q. Guarding r. Rebound tenderness s. Hyperactive bowel sounds t. Rigid abdomen
279
,The following is an indication for thoracotomy in chest injury .28 u. Cardiac tamponade v. Uncontrolled pulmonary air leakage w. Perforation of thoracic esophagus x. Blood loss of 200ml/hr for 2-3 hrs via chest tube y. All of the above MCQ for PBS 10. Cystic fibrosis of the pancreas f. is inherited as autosomal recessive g. islets of Langerhans are affected h. diagnosis is pssible by DNA analysis i. all the above j. a and c only 11. the following are clinical criteria of Bank ad Wise f. pulmonary g. neurological h. renal i. all the above j. a and c only 12. antibiotics in acute pancreatitis f. must be given to all cases g. should not be given h. given only to modarate & sever cases i. should cover anaerobic & Gam positive bactria j. none of the above 13. pancreatic carcinoma f. occure usually at the age of 50years g. usually is cystadenocarcinoma h. more favaorable in the body and tail i. less favourable if occure with back pain j. all the above 14. infected pancreatic necrosis f. is a collection of pus around the pancreas g. diagnosed by CT scan h. more serious than pancreatic abscess i. all the above j. none of the above 15. secondarey survay in polytrauma f. should be done in the first hour g. done as soon as the patient is stable h. detects life threateing problems i. a and c j. none of the above
280
16. spontaneous ruptur of the spleen f. occure less in tropical countries g. the spleen may be enlarged h. malaria is a common cause i. all the above 17. spleic aretry aneurysm f. incidence is around 5% g. usually symptomless h. more conmmon in males i. all the above 18. wolf skin graft f. used to cover large area of burn g. success rate is less than split thckness graft h. both thighs are common donor areas i. usually contracts 10 . medullary carcinoma of the thyroid e. amyloid stroma is charactrestic a. is a tumor of the G cells c. familial in 70% of the cases d. diagosed by low level of calcitonin 11.solitary thyroid nodule f. 70% are follicular adenoma g. 30% are malignant h. more favourable in young males i. all the above j. none of the above 12. rupture of the diaphragm a. is a life threatenig problem b. usually missed c. poly galctan suture is used for repair d. all the above e. noe of the above 14. Complication of vascular graft includes All the following except: f.Infection g.Aneurysm h.Graft failure i.Hemorrhage j.a & c only 16. Gangrene: a. is necrosis of tissue b. The cause may be venous occlusion c. usually painful d.All the above e.None of the above
281
17. Lynphedema: a. may be Congenital b. should be bilateral c. may be pitting in early stage d.A & C only e. None of the above 18. Suprapubic cystostomy: a. indicated in case of bladder out flow obstruction b. indicated in case of urethral injury c. indicated in case of urethral stricture d. All the above e. B& C only 19.differential diagnosis of Acute scrotal swelling in young adult includes all the follow except: a. Incarcerated inguinal hernia . b. Torsion of testes c. Acute epididymo- orchitis d. Teratoma e.. Seminoma 20. Horizontal spread of infection across the external sphincter can result in which type of anorectal abscess: a. ischiorectal b. perianal c. supralevator d. intersphincteric e. intermuscular 21. All of the following statements are true of diffuse esophageal spasm EXCEPT: a. chest pain is frequently seen b. high amplitude esophageal contraction are present. c. it is best diagnosed with barium esophogram. d. usual surgical treatment is long esohagomyotomy. e. most patients do not have significant coronary artery disease. .24. The treatment of an esophageal burn with a caustic agent may include all of the following EXCEPT: f. expeditious administration of an antidote. g. induction of vomiting. h. steroids and antibiotics. i. bougienage. j. gastrectomy. 25. All of the following substances are irritating to the peritoneum EXCEPT: f. bile. g. meconium. h. blood. i. gastric content. j. pus. 26. Complications of truncal vagotomy and pyloroplasty include all of the following EXCEPT: f. dumping syndrome.
282
g. h. i. j.
recurrent ulcer. diarrhea. alkaline reflux gastritis. steatorrhea.
27. . Gastric polyps: a. are most commonly adenomatous. b. require gastrotomy and removal if greater than 2 cm and are pedunculated. c. are rarely multiple. d. are clearly premalignant. e. are more frequent in achlorhydric patients. 28. . Vascular compression of the duodenum resulting in obstruction: a. is present primarily in patients who are overweight. b. should be given a trial of conservative management. c. is common in pediatric patients. d. is best diagnosed by identifying a "double bubble" sign on abdominal x-ray. e. includes as medical therapy lying in the supine position after meals 29. Conditions associated with gastric cancer include all of the following EXCEPT: f. higher socioeconomic groups. g. pernicious anemia. h. chronic atrophic gastritis. i. adenomatous polyps. j. a high intake of dietary nitrates k. 30. . Patients with morbid obesity have an increased incidence of all of the following EXCEPT: f. gastric carcinoma. g. diabetes. h. stroke i. gallbladder disease. j. joint deterioration. 31. All of the following contribute to malabsorption following truncal vagotomy and antrectomy EXCEPT: f. increased rate of gastric emptying. g. poor mixing of pancreatic secretions and bile salts with food. h. increased release of secretions and bile salts with food. i. decreased small intestinal transit time. j. malabsorption of fat and carbohydrates. 32. All of the following statements are true about patients with carcinoid tumors EXCEPT: f. they often have evidence of serotonin production. g. tumor growth is often slow. h. the majority have carcinoid syndrome. i. they have a much better prognosis if the tumors are less than 2 cm. j. the combination of streptozotocin and 5-fluorouracil can often result in objective response. 9. Which hypersensitivity reaction is associated with a tuberculin reaction? a. Type I: immediate b. Type II: cytotoxic c. Type III: immune complex
283
d. Type IV: cell mediated 10. The most common location for a gastric ulcer is a. Fundus b. Greater curvature c. Cardia d. Body e. Antrum 11. Regarding the management of major trauma a. Deaths follow a trimodal distribution b. Cardiac tamponade is characterised by raised BP, low JVP and muffled heart sounds c.
Assessment of uncomplicated limb fractures should occur during the primary survey
d.
Deterioration of the casualty during the primary survey should lead to the secondary survey
e. All are false 12. All of the following are true about neurogenic shock except: a. There is a decrease in systemic vascular resistance and an increase in venous capacitance. b. Tachycardia or bradycardia may be observed, along with hypotension. c. The use of an alpha agonist such as phenylephrine is the mainstay of treatment . d. Severe head injury, spinal cord injury, and high spinal anesthesia may all cause neurogenic shock. 13. The following cause hypercalcaemia except : a.
Sarcoidosis
b.
Primary hyperparathyroidism
c.
Acute pancreatitis
d.
Metastatic bronchial carcinoma
e.
Milk-Alkali syndrome
14. For a 40-kg baby the maintenance daily fluid requirement is approximately which of the following? a. 1100 ml b. 1250 ml c. 1550 ml d. 1700 ml e. 2000 ml 15. . Infantile hypertrophic pyloric stenosis a.
Occurs with a male: female ratio of 4:1.
b. Presents between six and eight months of age c.
284
Typically presents with bile stained projectile vomiting
d.
Surgical treatment is by Heller's Cardiomyotomy
e. all are false 8. Which of the following do you consider to be the most important clinical sign in acute appendicitis, 6. Abdominal tenderness around the umbilicus 7. Abdominal tenderness in the RIF 8. Tenderness over McBurney’s point 9. Rovsing’s sign positive 10. Suprapubic tenderness 9.Heparin 6. Acts as an anti-platelet 7. Acts as an anti-thromboplastin 8. Acts as an antithrombin 9. All of the above 10. None of the above 10. All of the following are mechanisms of urinary calculi formation except, 6. Hypoparathyroidism 7. Prolonged recumbency 8. Infection with urea-splitting organisms 9. Foreign body 10. Urinary tract obstruction 11. Which of the following are not found in peritonitis? 6. Patient is lying still 7. Guarding 8. Rebound tenderness 9. Hyperactive bowel sounds 10. Rigid abdomen 12-Which one of the following suggest a diagnosis of Hirschsprung's disease? p. q. r. s. t.
A contrast-study showing dilatation of the aganglionic bowel segment. Early presentation with vomiting. Neonatal large bowel obstruction. Presentation after 1 year of age. Red current jelly stools.
13-Which of the following regarding the anatomy of the heart is true? k. The aortic valve is tricuspid.
285
l.
The ascending aorta is entirely outside the pericardial sac.
m. The left atrial appendage is identified readily by transthoracic echocardiography. n. The pulmonary trunk lies anterior to the ascending aorta. o. The right atrium is posterior to the left atrium. 14-Which of the following is true concerning Scaphoid fractures? k. Rarely occur in young adults l. when complicated by avascular necrosis the proximal pole is usually affected m. should be treated by bone grafting and internal fixation even if undisplaced n. wrist fractures are uncommon o. anteriorposterior and lateral radiographs reveal most fractures
15-Which of the following statements is true of upper limb nerve injuries? k. Injury to the median nerve results in a wrist drop l. Injury to the radial nerve results in loss of sensation over the palmar aspect of the index finger m. Injury to the median nerve results in loss of sensation in the anatomical snuffbox n. Injury to the ulnar nerve results in a claw hand o. Injury to the ulnar nerve results in loss of sensation over the thumb 16- Regarding intravenous solutions: j. Normal saline contains 180mmol/l of sodium k. Ringer's lactate solutions is designed for intracellular fluid replacement l. Sodium bicarbonate 8.4% is a hyperosmolar solution m. Normal saline with added potassium is appropriate therapy to correct a respiratory alkalosis n. All are True 17-Which of the following concerning the Femoral sheath is false: j. Contains the femoral artery k. Contains lymph nodes l. Contains the femoral canal m. Contains femoral nerve n. Contains the femoral vein
286
18-The following is true of the spleen: k. Is the largest lymphoid organ in the body l. Lies obliquely between the seventh and tenth rib m. The lower pole extends beyond the mid-axillary line n. Is usually palpable when normal o. Usually measures 16cm in maximum length when healthy
19-Breast cancer risk is increased in association with the following factors except: k. Nulliparity l. Immediately after pregnancy m. Early menarche n. Early age at first pregnancy o. Late menopause
20-In tension pneumothorax the following signs are present except: p. q. r. s. t.
Hypoxia Hyperresonance to percussion on the affected side Tracheal deviation to the ipsilateral side Distended neck veins Tachycardia
21- The most common hernia in females is: k. Femoral hernia. l. Direct inguinal hernia. m. Indirect inguinal hernia. n. Obturator hernia. o. Umbilical hernia 22-. The most helpful diagnostic radiographic procedure in small bowel obstruction is: j. CT of the abdomen. k. Contrast study of the intestine. l. Supine and erect x-rays of the abdomen. m. Ultrasonography of the abdomen. n. MRI Abdomen
23- In role of nine extent of burn if entire trunk is burned it will be equal to: k. 9% body surface area. l. 18% body surface area.
287
m. 36% body surface area. n. 27% body surface area. o. 45% body surface area. 24-. If torsion of the testicle is suspected, surgical exploration: i. j. k. l.
Can be delayed 24 hours and limited to the affected side. Can be delayed but should include the asymptomatic side. Should be immediate and limited to the affected side. Should be immediate and include the asymptomatic side.
25- Hyperthyroidism can be caused by all of the following except: k. Graves' disease. l. Plummer's disease. m. Struma ovarii. n. Hashimoto's disease. o. Medullary carcinoma of the thyroid. 26- A 9 month old boy presents with an acute scrotal swelling. The following diagnoses are likely:
f. g. h. i. j.
Epididymitis Orchitis Torsion of the testicular appendage Irreducible inguinal hernia Acute idiopathic scrotal oedema
27. The evaluation of a comatose patient with a head injury begins with: a.The cardiovascular system. b. Pupillary reflexes. c. Establishment of an airway. d. Computed tomography (CT) of the brain e .insertion of Intravenous canul 28. The following is an indication for thoracotomy in chest injury, z. Cardiac tamponade aa. Uncontrolled pulmonary air leakage bb. Perforation of thoracic esophagus
288
cc. Blood loss of 200ml/hr for 2-3 hrs via chest tube dd. All of the above 29. Regarding Gallstones all of the following are true except:k. Prevalence increases with advancing age l. 30% of gallstones are radio-opaque m. Cholesterol stones result from a change in solubility of bile constituents n. Biliary infection, stasis and changes in gallbladder function can precipitate stone formation o. Gram-negative organisms are the most common isolated 30.In obstructive jaundice: f. Urinary conjugated bilirubin is increased g. Serum unconjugated bilirubin is increased h. Urinary urobilinogen increased i. Serum conjugated bilirubin is reduced j. Faecal stercobilinogen is increased Choose the Best appropriate answerfor each of the following questions: 1.The ilio-inguinal nerve: A. supplies the rectus abdominis muscle B. supplies skin on inner side of thigh C. supplies the cremasteric muscle D. supplies the urethra E. does none of the above
2. The skin of the tip of the index finger is supplied by the: F. Radial nerve only G. Radial & median nerves H. Median & ulnar nerves I.
Ulnar nerve only
J. Median nerve only 3. Hypovolaemic shock is characterized by: K. A low central venous pressure , high cardiac output, low peripheral resistance
289
L. A high central venous pressure, high cardiac output, high peripheral resistance M. A low central venous pressure , low cardiac output, high peripheral resistance N. A low central venous pressure , high cardiac output, high peripheral resistance O. A high central venous pressure, low cardiac output, low peripheral resistance 4. Which of the following in NOT true of Hodgkin's disease? F. Usually starts from several groups of nodes simultaneously G. Usually involves liver & spleen H. Sometimes manifests itself as pyrexia of unknown origin I.
Severe pain follows ingestion of alcohol
J. Shows increased susceptibility to opportunistic infection 5. Tetanus toxoid: E. Is produced by injecting animals with antititanic serum F. Is administered to previously immunized subjects with potentially infected wounds G. Frequently gives rise to anaphylactic reaction H. Is used to induce active immunity against tetanus
6. The most probable cause of shock in a patient with multiple injuries & craniocerebral trauma is: K. Depression of vital medullary centres L. Hypoperfuion control over subcortical areas M. Loss of cortical control over subcortical areas N. Hypovolaemia O. Inadequate ADH response 7. The most sensitive guide to acute changes in fluid balance in a surgical patient is: K. Accurate daily weight
290
L. Serial serum Na concentration M. Fluid balance sheets recording inputs & outputs N. Daily urine output O. Serial anion gap measurements 8. cellullitis is: F. Inflammation of the bone marrow G. Inflammation of the mastoid cells H. Inflammation of the subcutaneous tissues I.
Infiltration of the skin by gaint cells
J. A malignant condition 9. secondary haemorrhage occurs: K. Within 6 hours of operation L. 7-14 days after operation M. As a result of violent coughing on recovery from anaesthesia N. Due to a blood transfusion line disconnected O. When a ligature slips
10. the minimum urine output for 24 hours required to excrete end products of protein metabolism is: F. 200 ml G. 300 ml H. 400 ml I.
500 ml
J. 600 ml 11. Potassium deficiency is present if the plasma-potassium level is:
291
F. 6.0 mmol/l G. 5.0 mmol/l H. 4.5 mmol/l I.
4.0 mmol/l
J. 3.0 mmol/l 12.in health the pH of the blood lies between the range: F. pH 7.05-7.19 G.
7.20-7.35
H.
7.36-744
I.
7.45-7.59
J.
7.60-7.80
13. TNM classification of a malignant tumour was designed as: F. An histological staging G. A clinical staging H. A staging carried out at operation I.
A staging dependent upon radio scanning & skeletal survey
J. A staging dependent upon ultrasound
14. a blue-green discharge from an ulcer will be seen to contain: F. Pseudomonas pyocyaneus G. Streptococcus viridians H. Candida albicans I.
Staphylococcus aureus
J. Haemophilius influenzae
292
15. a rodent ulcer is: F. A squamous cell carcinoma G. A basal cell carcinoma H. Only occur on the face I.
Contains epithelial pearls
J. A venous ulcer
16. the space of Parona is: F. In the wrist between the deep flexor tendons & the pronator quadratus G. Above the patella between the quadriceps muscle & the femur H. Benath the tendon of the iliopsos I.
Between the achills tendon & the posterior aspect of the tibia
J. The web space of the palm
17. 'rest pain' occurs: K. Anywhere in the body at rest L. In the thigh of a patient with Buerger's disease M. In the calf of a patient with intermittent claudicating N. In the foot of a patient with severe vascular disease O. In the back
18. ischaemia means: L. Pain in the ischial tuberosities M. Anaemia due to malignant seconderies in the ischial part of the pelvis N. Lack of blood flow
293
O. Increased blood flow P. Polycythaemia 19. Colles' fracture is: F. A common in adolescence G. A fracture about the ankle joint H. Common in elderly women I.
A fracture of the head of the radius
J. A fracture of scaphoid
20. Pott's disease is; F. A fracture dislocation about the ankle G. A neuropathic joint H. Traumatic ostechondritis of the spine I.
Tuberculosis of the spine
J. A secondary tumour in the skul 21. Vincent's angina is a form of angina associated with: F. Spasm of the oesophagus G. Diphtheria H. An infection of the mouth I.
Coronary artery spasm
J. Carcinoma of the bronchus 22. Ludwig's angina is due to : F. A type of coronary artery spasm G. Oesophageal spasm H. Retropharyngeal infection I.
294
A virulent infection of the cellular tissue around the submandibular salivary gland
J. Infection with candida 23. in simple nodular goiter: F. Carcinoma occurs in 30% of cases G. The nodular stage is irreversible??????? H. Operation is contraindicated I.
The patient does not develop hyperthyroidism
J. Cretinism is the presenting feature 24. Hashimoto's disease is: F. A granulomatous thyroiditis G. An auto-immune thyroiditis H. An infiltrating fibrosis of the thyrois & the adjacent muscles I.
Focal thyroiditis
J. A parathyroid tumour 25. A thyroglossal fistula: F. Is never congenital G. Follows inadequate removal of a thyroglossal cyst H. Has a hood of skin with its concavity upwards I.
Is lined throughout by squamous epithelium
J. Occurs in carcinoma of the tongue 26. The following are clinical signs supporting an early diagnosis of carcinoma of the breast: F. A prickling sensation in a breast lump G. Peau d'ornge H. Brawny arm I.
Cancer en cuirasse
J. A krukenderg tumour
295
27.The gastroduodenal artery is a branch of the: F. Celiac axis G. Hepatic artery H. Superior mesenteric artery I.
Gastroepiploic artery
J. Splenic artery 28.Chronic gastric ulcers most often occur in patients with: F. Blood groub A G. Tend to occur in alkaline mucosa H. Muscularis mucosae is separated from the muscularis at the edge of the ulcer I.
Are malignant when there is epithelial proliferation & downgrowths
J. Are never large 29. Meckel's diverticulum: F. Is present in 20% of the human race G. Arises from the mesenteric border of the jejunum H. May contain heterotopic pancreas I.
Is only present in the male sex
J. Is a diverticulum of the bladder 30. Intussusception is related to: F. Mucoviscidosis G. Swollen Peyer's patches H. Volvulus I.
A littre's hernia
J. A patent vitello intestinal duct 31. The site of the neck of a femoral hernia is the: F. Transversalia fascia
296
G. Iliopectineal ligament H. Femoral ring I.
Cribriform fascia
J. Obturator foramen 32. Regarding operation for an indirect inguinal hernia: F. It should not be performed on patients who have chronic bronchitis G. General anaesthesia has to be used H. In infants the posterior inguinal wall should be repaired I.
In adults the internal inguinal ring usually needs to be strengthened
J. Mesh implants are mandatory 1. The followings are usually associated with acute appendicitis EXCEPT a. Abdominal pain and nausea b. CT scan with water soluble enema is the most helpful imaging study c. Deep right lower abdominal tenderness by rectal examination d. Positive Rovsing sign e. WBCs around 20,000/mm3 2. The least problem that cause right lower abdominal pain in a 18 years female is a. Ectopic pregnancy b. Acute appendicitis c. Ovarian torsion d. Perforated peptic ulcer e. Mittleschmerz 3. A patient with high hichitic fever, severe tenderness and rigidity at the right ileac fossa, WBCs 18000/mm3 and abdominal ultrasound showed a heterogeneous mass in the right iliac fossa with a central fluid collection. Management may include any of the followings EXCEPT a. Open drainage b. I.V antibiotics c. Appendix should be resected in the open drainage d. Percutaneous drainage under U/S or CT guidance e. Oral feeding shouldn’t be delayed in the absence of ileus 4. Regarding the lower esophageal sphincter the following are correct EXCEPT a. It is a physiological sphincter b. Located in the distal 3-5cm of the esophagus c. Has a resting pressure of 20-60 mmHg d. Abdominal pressure play a role e. Its pressure can be estimated by esophageal manometry 5. In esophageal perforation, the most sensitive diagnostic study is a. Plain film abdomen b. Plain film chest and neck
297
c. Esophagogram d. Esophagoscopy e. CT chest and neck Contrast esophagography is the ideal method (90% sensitivity) for evaluating suspected esophageal perforation. 6. Achalasia can be presented with all of the followings EXCEPT a. Recurrent pulmonary infections b. Weight loss c. Regurgitation d. Irregular narrowing of the distal esophagus by Ba. Swallow e. Retrosternal chest pain 7. Complications of reflux esophagitis include the followings EXCEPT a. Dysmotility b. Schatzki's ring c. Barrett's esophagus d. Epiphrenic esophageal diverticulum e. Hemorrhage 8. The pressure in the lower esophageal sphincter decreases by all of the following EXCEPT a. Alcohol b. Nitroglycerin c. Anticholinergics d. Alpha adrenergics e. Cholecystokinin hormone 9. The genetic predisposing factors to gastric cancer include the followings EXCEPT a. Family history of gastric cancer b. Black race c. P 53 mutation d. Germline mutation of e-cadherin e. BRCA2 mutation 10. Regarding the diffuse gastric cancer, the followings are true EXCEPT a. The commonest type of gastric adenocarcinoma b. Not associated with intestinal metaplasia c. More incidence in young ages d. Less related to environmental influences e. Results from single cell mutation 11. The best diagnostic study for gastric adenocarcinoma a. Upper endoscopy b. Endoscopic ultrasound c. Upper gastrointestinal double contrast barium study d. Laparoscopy e. Abdominal CT scanning 12. In primary gastrointestinal stromal tumors (GIST), which is NOT true a. The most common site is the stomach b. Bleeding is the commonest manifestation c. Almost never metastasize to regional lymph nodes d. The traditional cytotoxic chemotherapy greatly suppresses its growth e. Endoscopic ultrasound guided FNAC gives the definitive diagnosis
298
13. Primary gastric lymphoma a. 2% of all hodgkin's lymphoma b. Greatly differs in presentation from gastric adenocarcinoma c. One third of all gastrointestinal lymphomas d. The most common extranodal lymphoma e. Worse prognosis than adenocarcinoma 14. In gastric adenocarcinoma which is NOT true a. Risk increased 3—6 times in patient with gastric H pylori infection b. Blumer's shelf results from omental invasion in pelvic cavity c. Krukenburg's tumor is due to ovarian metastasis d. Weight loss and vague abdominal pain are the commonest presentation e. Endoscopic ultrasound is 90% accurate in determining T stage 15. Mucosa associated lymphoid tissue ( MALT ) is found in all of the followings EXCEPT a. Small bowel ( Peyer's patches ) b. Waldeyer's ring c. Appendix d. Stomach e. Bronchus 16. All of the following are associated with Barrett's esophagus EXCEPT a. GERD b. Squamous carcinoma c. Esophageal mucosal dysplasia d. Increased incidence of p53 mutations e. adenocarcinoma 17. The best test to establish the presence of gastroesophageal reflux (GERD) disease is a. An upper gastrointestinal series b. Bernstein test (acid perfusion) c. 24-hour pH monitoring d. Esophageal manometry e. Endoscopic biopsy 18. The T and N status of esophageal carcinoma is most accurately assessed by a. Upper gastrointestinal series b. Computed tomographic scan of the chest with double contrast c. Endoscopic ultrasound (EUS) d. Positron emission tomography (PET scan) e. Magnetic resonance imaging (MRI)
19. Salivary gland stones a. Most arise in the sublingual gland b. Usually present with persistent pain c. Are composed predominantly of magnesium phosphate d. Predispose to infection of the involved gland
299
e. Never occur in parotid duct 20. 35-year-old alcoholic is admitted with acute pancreatitis. He complains of Numbness of his fingers and toes. On examination he has hyperactive Tendon reflexes. The most likely cause of these symptoms is a. Hyponatremia b. Hypocalcemia c. Hypophosphatemia d. Hypermagnesemia a. Hyperkalemia 21. Regarding benign salivary gland adenomas a. Pleomorphic adenomas are only seen in parotid gland b. Pleomorphic adenomas cannot undergo malignant change c. Warthin's tumor is otherwise known as an adenolymphoma d. Adenolymphomas usually occur in young men e. Adenolymphomas are often bilateral 22. Massive lower gastrointestinal hemorrhage a. Rarely stops after resuscitation b. Is caused by large bowel lesions only c. can be most accurately localized by colonoscopy d. Is frequently related to right colon lesions e. Is most commonly caused by adenocarcinoma of the large intestine 23. Regarding carcinoid tumors all of the following are true EXCEPT a. Liver metastases can result in the carcinoid syndrome b. The appendix is the commonest primary site for gastrointestinal tumors c. Gastric carcinoid tumors produce little 5-hydroxyindoleacetic acid d. If discovered in the appendix right hemicolectomy should always be done e. Octreotide scintigraphy may identify both the primary and secondary lesions 24. A Meckel's diverticulum a. b. c. d. e.
Occurs in 10% of the population Will be found on the mesenteric border of the small intestine Consists of mucosa without a muscle coat Usually found 20 cm from ileocecal valve A fibrous band between the apex and umbilicus can cause intestinal
obstruction
25. In overwhelming post-splenectomy infection, which is NOT true a. b. c. d.
Is usually due to unencapsulated bacterial infection Strep. pneumonia is the commonest etiological agent Despite aggressive therapy it can have a mortality of over 50% The risk of infection can be reduced with pneumococcal and hemophilus vaccination e. Penicillin antibiotic prophylaxis should be considered in all children 26. Clostridium tetani a. Is a gram-negative rod b. Is sensitive to penicillin c. Is available in the expired canned food d. Releases a heat-resistant endotoxin
300
e. The toxin acts on the post-synaptic membrane of inhibitory nerve fibers 27. Grade I hypovolemic shock a. Occurs when more than 40% of the circulating blood volume has been lost b. Tachycardia is a reliable clinical sign c. A fall in pulse pressure is observed d. Urine output is markedly reduced e. Capillary return is delayed 28. Regarding Hydatid disease the followings are true EXCEPT a. Recently surgery is no more the treatment of choice in liver disease b. Man is an accidental intermediate host c. The liver is the commonest site of infection d. Can be diagnosed by the ELISA test e. Treatment by benzimidazoles alone is 30% successful 29. In peptic ulcer disease a. H. pylori is a gram-positive bacillus b. Surgery is always indicated in perforated ulcers c. Sepsis after perforation is the commonest cause of death d. Parietal cell vagotomy carries the highest rate of recurrence e. Gastric drainage should be done in all types of vagotomies 30. Stones in the common bile duct, which is NOT true a. Are mostly secondary stones b. Can present with Charcot's Triad c. Are suggested by a bile duct diameter >8mm on ultrasound d. ERCP, sphincterotomy and balloon clearance is now the treatment of choice e. If removed by exploration of the common bile duct the T-tube can be removed after 3 days 31. Regarding pancreatic carcinoma the followings are true EXCEPT a. 90% are ductal adenocarcinomas b. Less than 20% occur in the head of the gland c. The usual presentation is with pain, weight loss and obstructive jaundice d. Ultrasound has a sensitivity of 80-90% in the detection of the tumor e. Less than 20% of patients are suitable for curative surgery 32. Regarding anal fissures all are true EXCEPT a. 10% occur in the posterior midline b. Multiple fissures suggest a diagnosis of tuberculosis or Crohn's Disease c. 50% of acute fissures heal with the use of a bulking agent d. Sphincterotomy has a success rate of over 90% e. Sphincterotomy is associated with minor fecal incontinence in over 15% of patients
301
33. Medullary carcinoma of the thyroid a. Concentrates Iodine 131 b. Produce thyroxine as the principle hormone c. 10% of cases are sporadic d. 90% can occur in association with MEN type II syndrome e. All patients will need a 24-hour VMA level in urine 34. Which of the following is LEAST likely to be associated with the systemic inflammatory response syndrome (SIRS) a. Infection b. Elevated/depressed temperature c. Elevated heart rate d. Elevated respiratory rate e. Elevated/depressed WBC count 35. In papillary carcinoma of the thyroid the followings are true EXCEPT a. Can be reliably diagnosed using fine needle aspiration cytology b. Is almost always unifocal c. Histologically displays Psammoma bodies d. Typically spread to the cervical lymph nodes e. Requires a total thyroidectomy for large tumors 36. Serum alpha-fetoprotein is increased in the following EXCEPT a. Acute hepatitis b. Hepatocellular carcinoma c. Neuroblastoma d. Teratomas e. Bladder carcinoma 37. The following predispose to wound infection EXCEPT a. Malnutrition b. Hypovolemia c. Malignancy d. Obstructive jaundice e. Steroid therapy 38. Regarding acute respiratory distress syndrome (ARDS) which is NOT true a. Hypoxia in spite of high inspired oxygen b. Increased lung compliance c. Non-cardiac edema d. Diffuse or patchy infiltrates in chest X ray e. Deposition of proteinaceous fluid in the respiratory membrane 39.Which of the following statements about serum thyroglobulin is TRUE? a. Elevation after total thyroidectomy justify the use of iodine131 therapy b. It suppresses thyroid-stimulating hormone (TSH) c. It suppresses thyroid-releasing hormone (TRH) d. It is an effective tumor marker because it is specifically elaborated by the malignant cells of papillary cancer
302
e. It is only about 50% effective in detecting recurrence 40.Women who have hereditary nonpolyposis colorectal cancer (HNPCC) should also be screened for a. Endometrial cancer b. Papillary cancer of thyroid c. Ampullary cancer d. Pheochromocytoma e. Hepatoma 1.e
2.d
3.c
4.c
5.d
6.d
7.d
8.d
9.e
10. a
11.a
12.d
13.d
14.b
15.d
16.b
17.c
18.c
19.d
20.b
21.c
22.d
23.d
24.e
25.a
26.b
27.b
28.a
29.d
30.e
31.b
32.a
33.e
34.a
35.b
36.e
37.d
38.b
39.a
40.a
Pancreas 1. In pancreas divisum all the following are true except A. occur in 5% of the population B. may lead to recurrent pancreatitis C. duct of Wirsung is the main duct D. there is failure of fusion E. diagnosed by ERCP 2. Cystic fibrosis all the following are true except A. transmitted as autosomal dominent B. dysfunction of exocrine glands C. there is poor appetite & growth, D. patient has distension, steatorhea, finger clubbing present E. diagnosis by DNA analysis 3. Sever attack of pancreatitis is A. attack followed by pseudocyst B. has four points on Ranson criteria C. has 5 points on Balthazar grade D. all the above E. none of the above 4. Antibiotics in acute pancreatitis: A. not indicated B. second generations cephalosporins are the best C. indicated in all cases D. b and c only E. none of the above 5. CT in acute pancreatitis : A. should be done on admission B. should be done 48 hours later C. should be done one week later D. not indicated E. none of the above
303
6. Pancreatic pseudocyst A. usually appears after two weeks of the attack B. has one layer of epithelial lining C. may be treated by endoscopy D. all the above E. none of the above 7. Percutaneous drainage of pancreatic cyst is indicated in all the following except : A. after three weeks in order to mature the wall B. rapidly enlarging cyst C. difficult sites D. presence of infection E. unfit patients 8. The following are known complications of acute pancreatitis except A. respiratory failure B. renal failure C. cardiac failure D. coagulopathy E. bleeding gastric erosion 9. The following can cause high serum amylase : A. perf. GB B. rupture ectopic pregnancy C. rupture abdominal aortic aneurysm D. afferent loop obstruction E. all the above 10. Carcinoma of the pancreas: A. 50% adeocarcinoma of duct origin B. slightly more in the head C. 10% cystadenocarcioma D. CIS about 40 % E. none of the above 11. Insolinoma : A. age usually above 40 B. hyper glycemia in the early morning is classical C. clinically may simulate DU D. insulin / glucose ratio should equal one E. all of the above Orthopedic 12. Bennet's fracture is : A. fracture of the base of first meta tarsal bone B. fracture of the base first meta carpal bone C. fracture of the head of first meta tarsal bone D. fracture of the head first meta carpal bone E. fracture of the head second meta carpal bone 13. Thomas test is positive in : A. Paget's disease of the hip B. Perthe's disease of the hip C. osteo arthritis of the knee D. Sudeck's atrophy of the shoulder E. none of the above
304
14. All the following are complications of cast except A. compartment syndrome B. pressure sore C. blisters with 2ry infection D. immobilization of joints E. crush syndrome 15. Volkmann's ischemic contracture : A. may follows fracture of radius and ulna B. fracture neck of femur C. supracondylar fracture of humerus D. all the above E. none of the above 16. Compartment syndrome A. common in fracture of the leg B. passive flexing of the toes increase the pain C. compartment pressure should exceed 50 mmHg D. treated by urgent fasiectomy E. all the above 17. Indication of operative treatment in fractures : A. compound fracture B. intertrochanteric fracture of femur in the elderly C. associated complications D. a and c only E. all the above 18. The following are methods of stabilization of fractures except: A. External splint e.g. POP B. Internal fixation C. Hanging plaster D. Skin traction E. a and b only 19. stress fracture : A. may affect the shaft of tibia B. may affect the second metatarsal bone C. occur in marathon runners D. all the above 20. Mallet finger is due to: D. avulsion fracture of the base of terminal phalanx 21. operative approach to the hip joint may be : A. anterior B. posterior C. lateral }}}. all the above 22. Salter and Harris classification : A. classification for fracture tibial platue B. classification for fracture neck femur C. classification for fracture growth plate VASCULAR 23. In critical ischemia there is: A. rest pain B. oedema C. color changes D. hyperesthesia
305
E. all the above 24. popliteal artery aneurysm A. accounts for 70% of all peripheral aneurysms B. 25% are bilateral C. 30% develop complication within 5years D. All of the above 25. Beurger' diseases has : A. occlusion of small and medium arteries B. thrombophlebitis C. Reynold's phenomenon D. all the above 26. abdominal aortic aneurysm A. usually due to atherosclerosis B. usually at the level of L1 C. incidence of rupture is 50% for 5 cm aneurysms D. elective surgery careies 20% mortality rate E. all the above 27. Berry's aneurysms A. are congenital B. affect medium sized arteries C. fusiform in shape D. all the above 28. Secondary varicose viens A. due to valve destruction B. due to incompetent perforators C. has lower complication rate than the primary varicose D. may lead to venous ulcer E. none of the above 29. Greenfield filter A. reduce incidence of DVT B. inserted in the common iliac vien C. inserted between L1 and L2 levels D. can be inserted percutaneous 30. lymph edema tarda A. usually before the age of 35 years B. affects upper limbs only C. affect lower limbs only D. usually pitting in early stage BASIC SURGICAL SCIENCES 31. Abduction of the vocal cords results from contraction of the: A. crico-thyroid muscles B. posterior crico-arytenoid muscles C. vocalis muscles D. thyro-ary-epiglottic muscles E. lateral crico-arytenoid and transverse arytenoids muscles 32. The rectum: A. is devoid of peritoneum B. is surrounded by peritoneum C. has peritoneum on its lateral surfaces for its upper two- thirds, and on its anterior surface for its upper one-third D. has pritoneum on its anterior surface for its upper two-thirds, and on its lateral surfaces for its upper one-third E. has peritoneum on its anterior surface only
306
33. The umbilicus: A. lies near the to the xiphoid than to the pubis B. derives its cutaneous innervation from the eleventh thoracic nerve C. transmits, during development, the umbilical cord two arteries and two veins D. usually lies at about the level between the third and fourth lumbar vertebra E. emberiologicall, may transmit urine but never bowel content 34. The superficial perineal pouch: A. is limited inferiorly by the urogenital diaphragm B. is not continuous with the space in the scrotum occupied by the testes C. has a membranous covering which provides a fascial sheath around the penis D. is traversedby the urethera in the male but not the urethera and vagina in the female E. in the female, the greater vestibular glands are situated outside this pouch 35. The tongue: A. has a foramen caecum at the base of the frenulum B. is separated from the epiglottis by the valleculae on each side of the midline C. has 7-12 circumvallate papillae situated behind the sulcus terminalis D. is attached to the hyoid bone by the genioglossus muscle E. is supplied only by hypoglossal nerve 36. Hypovolaemic shock is characterized by: A. a low central venous pressure , low cardiac output , low peripheral resistance B. a high central venous pressure , high cardiac output , low peripheral resistance C. a low central venoys pressure , low cardiac output , high periphera resistance D. a low central venous pressure , high cardiac output , high peripheral resistance E. a high central venous pressure , low cardiac output , low peripheral resistance 37. An oxygen debt is: A. the amount of oxygen in excess of the resting metabolic needs that must be consumed after completion of exercise B. build up because the pulmonary capillaries limit the uptake of Oxygen at high rates of oxygen consumption C. related to the fact that skeletal muscle cannot function temporarily in the absence of oxygen D. associated with a decrease in blood lactate E. associated with alkalosis 38. Pulmonary embolism may be a complication of except : A. prolonged bed rest B. a surgical operation C. vitamine K deficiency D. oral contraceptive therapy E. Antithrombin III deficiency 39. Which of the following statements regarding potassium metabolism is NOT True?: A. potassium deficiency commonly results from thiazide diuretic theraoy B. the normal compensation for potassium deficiency is a metabolic extracellular acidosis C. aldosterone increases urinary potassium loss D. hyperkalaemia causes bradycardia and loss of P waves on the ECG E. hypokalaemia aggrevates the cardiac effects of digitalis toxicity 40. Cutaneous pain: A. is due to overstimulation of receptors serving other sensory modalities B. cannot be elicited more readily if the tissue has recently been injured C. is due to exitation of receptors by pain-producing chemical substances in the injured tissue D. shows marked adaptation, i. e. decrease in severity in response to a constant stimulus E. is conducted through the medial spinothalamic tract
307
41. Which of the following is NOT associated with hyperthyroidism?: A. increase size of the thyroid gland B. increased amount of colloid in thyroid follicle C. increased height of epithelium of the thyroid follicle D. increased vascularity of the thyroid gland E. increased uptake of iodine by the thyroid gland 42. Sarcomata may show all of the following EXCEPT: A. production of myxomatous tissue B. production of collagen C. spindle shaped cells D. signet ring cells E. blood stream metastasis 43. Anaphylaxis is characterized by all of the following EXCEPT: A. is a reaction either local or general , frequently occurs within five minutes B. causes an urticarial eruption C. is produced by IgA antibody D. causes eosinophilia E. causes degranulation of basophils and mast cells 44. Autoimmunity is characterized by the following EXCEPT: A. occurs because of a breakdown in the ability of the body to distinguish between self and non self B. is involved in some forms of orchitis C. is involved in formation of cryo globulin D. is important in the pathogenesis of lupus erytheromatosus E. does not result in immune complex disease 45. Pseudomembranous enterocolitis is caused by the following organisms: A. Clostridium sporogenes B. Clostridium defficile C. Streptococcus faecalis D. Penicillin sensitive staphylocci E. Pseudomonas aeruginos CLINICAL SUGERY 46. The “ white clot syndrome”: A. is usually characterized with antithrombin III deficiency B. most often present with arterial complicatios of heparin induced throbocytopenin C. is best managed by loe molecular weight dextran D. is best managed by halving the therapeutic dose of heparin sodium E. results from nitric oxide deficiency of endothelial cells 47. A 21-year-old man who was the driver in a head-on collision has a pulse of 140/min , respiratory rate of 36 and blood pressure of 75 palpable. His trachea is deviated to the left, with palpable subcutaneous emphysema and poor breath sounds in the right hemithorax, The most appropriate initial treatment must be A. immediate thoracotomy B. catheter insertion in the subclavian vein for fluid resuscitation C. intubation and ventilation D. tube thoracostomy E. immediate tracheostomy 48. The best test to monitor the adequacy of levothyroxin therapy is: A. radioactive iodine uptake B. thyroglobulin C. free thyroxine index (T4) D. triiodothyronine resin uptake (T3) E. thyroid stimulating hormone (TSH)
308
49. Which of the following statements about fungal infection is NOT true ?: A. Prior or synchronous culture positive for Candida at another site occurs in few patients with candidimia B. For critically ill patients nonhaematogenous sites of candida are appropriately treated with systemic antifungal therapy C. Mortality rates are similar regardlss of whether C. albicans fungmia is treated with amphotericin B or flconazole D. Intravenous catheters and the gastrointestinal tract are common portals for Candida to gain blood stream access E. Septic emboli are more common with fungal endocarditis than with bacterial endocarditis 50. The maximum safe dose of local anaesthetic administered subcutaneously in a 70-kg man is: A. 10 to 20 ml of 1% lidocaine B. 40 to 50ml oh 2% lidocaine with epinephrine C. 40 to 50 ml of 1% lidcaine with epinephrine D. 40 to 50 ml of 1% bupivacaine (marcaine) E. 40 to 50 ml of 1%lidocaine without epinephrine 51. Two days after right hemicolectomy for a Dukes B caecal carcinoma , the Patient complains of sharp right-sided chest pain and dyspnea. HisPaO2 Is 64mmHg ,his PaCo2 is 32mmHg. CVP is 26 cm water, and the blood pressure is 102/78mmHg. A pulmonary embolus is suspected, The next step in management should be: A. A ventilation- perfusion lung scan B. A pulmonary arteriogram C. Postrioanterior and lateral chest x-rays D. Heparin sodium ,100 units/kg intravenously E. Immediate duplex scanning of both lower extremities 52. The major cause of graft loss in heart and kidney allograft is: A. acute rejection B. hyperacute rejection C. vascular thrombosis D. chronic rejection E. graft infection 53. All of the following are indicators of tumor aggressiveness and poor outcome for papillary carcinoma of the thyroid gland EXCEPT: A. age over 50 years B. microscopic lymph node metstasis C. tumor larger than 4 cm D. poorly differentiated histological grade E. invasion through capsule to adjacent tissues 54. A 40-year-old woman has extensive microcalcifications involving the entire upper aspect of the right breast. Biopsy shows a commedo pattern of intraductal carcinoma.The most appropriate treatment is : A. wide local excision B. radiation therapy C. wide local excision plus radiation therapy D. right total mastectomy E. right modified radical mastectomy 55. In the conventional ventilator management of acute adult respiratory distress syndrome (ARDS) , arterial O2 saturation is maintained above 90% by all the following EXCEPT : A. increasing the ventilatory rate B. the use of positive end-expiratory pressure (PEEP) C. increasing mean airway pressure D. increasing tidal volumes
309
E. increasing FiO2 56. Which of the following statements about patients with abdominal compartment syndrome is NOT true ? A. Abdominal pressure is usually measured indirectly through inferior vena cava B. Multiple contributing factors are commonly responsible C. The chief manifestations are reflected in central venous pressure , ventilatory function, and oliguria D. Decopression of the abdomen is required to resverse the syndrome E. Aggressive hemodynamic monitoring and management is required when the abdomen is opened 57. The most appropriate treatment for histologically malignant cystadenoma phylloides is : A. total mastectomy without axillary node dissection B. total mastectomy with axillary node dissection C. wide margin (3) cm excision of the lesion D. post operative hormonal manipulation E. postoperative adjuvant chemotherapy 58. Deep venous thrombosis resulting from upper extremity central venous lines: A. should be treated with catheter removal, heparin therapy, and long term anticoagulants B. is best with urokinase through the catheter C. is innocuous and self limiting, and best treated with catheter removal only D. is best treated with low-dose warfarin (coumadin, 1 mg / day) , without catheter removal E. is best managed by single systemic dose of low molecular weight heparin daily and continued catheter use 59. Emergency surgery is indicated for all of the following complications of ulcerative colitis EXCEPT: A. colonic dilatation greater than 12 cm (toxic mega colon ) B. free perforation C. complete intestinal obstruction D. intractable haemorrhage E. abscess formation 60. All the following statements concerning carcinoma of the oesophagus are true EXCEPT that: A. it has a higher incidence in males than females B. alcohol has been implicated as a precipitating factor C. adenocarcinoma is the most common type at the cardio esophageal junction D. it occurs more commonly in patients with corrosive oesophagitis E. surgical excision is the only effective treatment 61. Which of the following statements about pathology encountered at esophagoscopy is/are correct? A. Reflux esophagitis should be graded as mild, moderate, or severe, to promote consistency among different observers. B. An esophageal reflux stricture with a 2-mm. lumen is not dilatable and is best treated with resection. C. A newly diagnosed radiographic distal esophageal stricture warrants dilation and antireflux medical therapy. D. In patients with Barrett's mucosa, the squamocolumnar epithelial junction occurs 3 cm. or more proximal to the anatomic esophagogastric junction
310
Fluid And Electrolyte 1. The effective osmotic pressure between the plasma and interstitial fluid compartments is primarily controlled by: A. Bicarbonate. B. Chloride ion. C. Potassium ion. D. Protein. E. Sodium ion. 2. Symptoms and signs of extracellular fluid volume deficit include all of the following except: A. Anorexia. B. Apathy. C. Decreased body temperature. D. High pulse pressure. E. Orthostatic hypotension. 3. The osmolarity of the extracellular fluid space is determined primarily by the concentration of: A. Bicarbonate B. Chloride ion C. Phosphate radicals D. Sodium ion E. Sulfate radicals 4. Potassium deficiency should be suspected in all except: A. In cases of paralytic ileus. B. When the patient's reflexes are exaggerated. C. If there is a decrease in height and peaking of the T waves of an ECG. D. In alkalotic states. 5. The sodium ion: A. Is the principal regulator of the intracellular volume. B. Is the major ionic component of the intracellular fluid volume. C. Is present in greater concentration in intracellular fluid than extracellular fluid. D. Is excreted in larger amounts than normal in the early postoperative period. E. C&D only 6. Acute post traumatic renal failure: A. May be due to hypovolaemia and poor tissue perfusion. B. Is particularly associated with crush injuries. C. May be due to kidney damage following tubular obstruction. D. Should initially be treated by fluid restriction. E. Should be treated initially by fluid restriction. 7. Intravenous parenteral feeding: A. Should deliver at least 2500 calories/day to an adult. B. Should deliver at least 10g of nitrogen (i.e. 66g of protein)/day to an adult. C. Can be effectively achieved with isotonic solutions. D. Is with complications with present day solutions and methods of administration. E. Can be given by central intravenous infusion line (CVL) Acute Abdomen 8. Acute abdominal pain which is (all correct except one) A. Colicky in nature indicates obstruction of hollow viscus. B. In right upper quadrant increased by inspiration is typical of cholecystitis C. Continuous is typical of inflammation D. Maximal in the right loin is typical with duodenal ulcer. E. Maximal in epigastrium and related to meal is typical of gastric problem.
311
9. Faeculent vomiting: A. Is commonly seen after upper gastrointestinal tract. B. Indicates large bowel obstruction. C. Indicates bacterial proliteration in the upper intestinal. D. Suggests a gastro-colic fistula. E. Suggests small bowel fistula 10. Perforated duodenal ulcers: (all correct except one) A. Occur most frequently in female B. Are usually preceded by an dxacerbation of ulcer symptoms. C. Are usually accompanied by a leucocytosis. D. Produce abdominal tenderness which is most marked in the epigastrium. E. May cause septicemia. 11. A perforated duodenal ulcer: A. Usually lies on the anterior or superior surface of the duodenum. B. Usually presents with the acute onset of severe back pain. C. Produces radiological evidence of free gas in the peritoneum in over 90 percent of the patients. D. Is usually treated by vagotomy and pyloroplasty E. Is usually treated conservatively. 12. Appendicitis is all true except: A. More common in females B. Distributed evenly thoughout the world's population C. More likely to occur if the appendix is in the retrocaecal position. D. Commonly the result of appendicular obstruction. 13. Patients with early appendicitis: A. Usually present with central abdominal pain. B. Rarely present with anorexia. C. Have usually vomited many time. D. Usually complain of similar attacks of pain in the previous few weeks. E. B&C only. 14. Obstruction of the lumen of the appendix may lead to: (all correct except one) A. Mucosal ulceration B. Gangrenous appendicitis. C. A perforated appendix. D. Intussusception of the appendix. E. Acute appendicitis. 15. In the differential diagnosis of appendicitis in an infant it's important to consider: (all correct except) A. Ileo-ileal instussusception. B. Basal pneumonia. C. Henoch-schoenlein purpura. D. Torsion of an ovarian cyst. E. Gastroenteritis. 16. Acute non-specific mesenteric lymphadenitis: (all correct except one) A. Is commonest between 5 and 12 years of age. B. Is usually associated with an upper respiratory tract infection. C. Is usually associated with cervical lymphadenopathy D. Is characterized by enlarged mesenteric lymph nodes which are infected by gram-negative organisms. E. Is one of the differential diagnosis of acute appendicitis. 17. The level of intestinal obstruction can be determined by:(all correct except one) A. Questioning the patient. B. Examining the patient. C. Radiological examination of the patient. D. Repeated measurements of the patient's girth.
312
E. Upper & lower contrast studies. 18. Acute small bowel obstruction: (all correct except one) A. Is commonly caused by postoperative adhesions. B. Accompanied by the signs of peritonitis, suggests bowel strangulation. C. Is often associated with a raised serum amylase. D. Generally produces abdominal distension within 2 to 3 hours of onset. E. Can be diagnosed by history and clinical examination 19. Strangulation of the bowel: (all correct except one) A. commonly complicates closed loop obstruction. B. Is difficult to distinguish from simple intestinal obstruction. C. Is accompanied by bleeding into the affected bowel. D. Frequently causes peritonitis. E. Is characterized by severe abdominal pain. 20. Large bowel obstruction: A. Is most commonly caused by colonic cancer B. Has its maximum incidence before the age of 50. C. Frequently presents with nausea and vomiting. D. Usually heralds its onset with constant suprapubic pain. E. Frequently treated conservatively. 21. Acute pancreatitis typically: (all correct except one) A. Is accompanied by hypercalcaemia. B. Produces paralytic ileus. C. Is associated with a pleural effusion. D. Produces pyloric stenosis. E. Upper abdominal pain and vomiting. 22. Patients with acute colonic deveticulitis: A. Often give a history of recent lower abdominal colic. B. Often present with pyrexia. C. Can be frequently diagnosed on sigmoidoscopic appearances. D. Frequently develop faecal peritonitis. E. All the above. 23. Acute pancreatitis: (all correct except one) A. Often simulates a perforated peptic ulcer in its presentation. B. Often presents with the signs of hypovolaemia. C. Can readily be distinguished from other causes of acute abdominal pain by the presence of a raised serum amylase. D. Frequently has a raised concentration of urinary amylase. E. Most commonly caused as a complication of GB stones. 24. Childhood intussusception: (all correct except one) A. Usually presents during the first year of life. B. Is frequently ileocolic. C. Can usually be diagnosed without x-ray examination of the abdomen. D. Rarely requires surgical treatment. E. Can be diagnosed by abdominal US. 25. Neonatal duodenal obstruction: A. May be associated with down's syndrome. B. Is more frequently found in premature infants. C. Typically presents with gross abdominal distension. D. Usually presents with vomiting of non-bile stained fluid E. B&C only.
313
26. acute superior mesenteric artery occlusion: (all correct except one) A. Characteristically presents with sudden pain and tenderness of increasing intensity. B. Is frequently accompanied by overt or occult blood loss in the stools. C. Frequently produces peritonitis. D. Can usually be diagnosed on plain abdominal x-rays. E. Can be diagnosed by mesenteric artery ongiography. 27. Biliary colic typically: A. Occurs 3 to 4 hours after meals. B. Lasts 5 to 20 minutes. C. Radiates from the upper abdomen to the right subscapular region. D. Is made better by deep inspiration. E. B&C only. Polytrauma 28. Regarding tension pneumothorax, the first step in the management is: A. Obtaining a stat chest x-ray. B. Cricothyroidectomy C. Passin on endotracheal tube D. Starting oxygen by a valve-mask device E. Chest decompression needle. 29. Regarding the management of polytrauma: A. Death follow a trimodal distribution. B. X-ray after primary survey should be AP cervical spine, chest and pelvis. C. Cardiac tamponade is characterized by raised B.p, a low JUP. D. Assessment of uncomplicated limb fractures should occur during the primary survey. E. A and B only. 30. Blunt injuries to the abdomen (all are correct except) A. May cause shock B. May cause peritonitis C. May cause acute gastroduodenal ulceration D. May treated conservativly E. Rarely need urgent laparatomy 31.Injuries to the urethra (all are correct except one) A. Are more common in male. B. Are often caused by road traffic accidents. C. Are easily diagnosed on intra venous pyelography. D. Require urgent surgical treatment. E. Diagnosed by antegrade urethragraphy. 32.Car seat belts when properly adjusted A. Prevent injuries to abdominal organs. B. May cause small bowel injuries. C. Do not reduce the incidence of head injuries of passengers involving in RTA. D. Protect the cervical spine during sudden acceleration . E. A & D only. 33. In head trauma patient after control of air way , the first diagnostic study A. X-rays of the skull. B. CT scan of the head. C. X-rays of cervical spine D. Carotid angiography. E. Lumber puncture.
314
Burns 34. Estimation of the area of a burn: ( all correct except one) A. Is of very little clinical significance. B. Provides important prognostic information. C. Is an important factor in the estimation of the fluid required. D. Can be based on a formula which states that the adult trunk is 36 per cent of the whole body surface area. E. Should be recorded in each chart of burned patient. 35. Patients with major burns: A. Are in a negative nitrogen balance. B. Have normal calorie requirements. C. Do not generally become anaemic. D. Are resistant to septicaemia. E. All of the above. 36. Scalds: A. Are more frequent in children. B. Commonly cause full thickness skin loss. C. Should be skin grafted within 48 hours of the injury. D. Need routine antibiotic treatment. E. All of the above. 37. Major burns are sometimes complicated by: (all correct except one) A. Acute gastric and duodenal ulcers. B. Paralytic ileus. C. Cerebral oedema. D. Mesenteric thrombosis. E. Septicemia. Shock and blood transfusion 38.In all forms of shock there is: A. An impairment of cellular oxygenation. B. A decreased cardiac output. C. An increased effective circulating fluid volume. D. A low central venous pressure (CVP). E. An increased pulse rate. 39.The metabolic acidosis of shock can be effectively treated by: A. Warming the patient. B. Administering ammonia chloride. C. Artificial ventilation. D. Restoring normal tissue perfusion. 40. Nacl intravenous infusion. siptic shock is associated with a hypodynamic cardiovascular state: ( all correct except one) A. if preceded by existing hypovolaemia. B. In generalized peritonitis. C. When there is a gram-positive bacteraemia. D. In elderly patients. E. In late gram negative septicemia. 41. In cardiogenic shock: A. The central venous pressure is low. B. The difference in the arteriovenous oxygen tension is increased. C. The haematocrit is raised. D. The blood pressure is unaffected. E. C&D only.
315
42. A blood transfusion reaction: (all correct except one) A. May be due to incompatibility of the recipient serum and donor cells. B. Is manifest by thrombophlebitis of the infusion site. C. Occurs within the first 30 minutes of transfusion. D. May produce renal damage. E. May produce anaphylactic shock. 43. Massive blood transfusions may be complicated by all true except: A. Hyperkalaemia. B. Hypocalcaemia. C. Coagulopathy. D. Leucopenia. E. DIC. Fractures And Dislocations 44. In a healing fracture: (All correct except one) A. The haematoma is initially invaded by osteoblasts. B. The tissue formed by the invading osteoblasts is termed osteoid. C. Calcium salts are laid down in the osteoid tissue. D. The final stage of repair is the remodelling of the callus. E. The callus formation is related to the amount of stress at fracture side. 45. Non-union is often seen in: A. Fractures of the 4th metatarsal. B. Fractures of the neck of the femur. C. Fractures of the condyle of the mandible. D. Colles’ fractures. E. Oblique fracture line of femur. 46. In a colles’ fracture the distal radial fragment: A. Is dorsally angulated on the proximal radius. B. Is usually torn from the intra-articular triangular disc. C. Is deviated to the ulnar side. D. Is rarely impacted. E. Is ventrally displaced. Skin And Breast 47. A malignant melanoma: A. Frequently arises from hair-bearing naevi. B. Frequently arises from junctional naevi. C. Has a worse prognosis when it areses on the leg. D. Should be suspected in any big pigmented lesion. E. Non of the above is correct. 48. Squamous cancer of the lip: A. Is most common in early adult life. B. Is more common in fair skinned subjects. C. Metastasises readily by the blood stream. D. Is preferably treated by radiotherapy once lymph node deposits are present. E. All of the above are correct. 49. Basal cell carcinomas: A. Usually metastasise to regional lymph nodes. B. Are less common than squamous cell carcinomas. C. Are characterised histologically by epithelial pearls. D. Are particularly common in oriental races. E. Non of the above is correct. 50. Fiboadenomata of the breast: A. Are commonest in early adult life. B. Are indiscrete and difficult to distinguish.
316
C. Are usually painless. D. Resolve without treatment. E. A&C only. 51. Paget’s disease of the nipple: A. Usually presents as abilateral eczema of the nipple. B. Is always related to an underlying breast cancer. C. Indicates incurable breast cancer. D. Has non-specific histological characteristics. E. A&C only. Extrahepatic Biliary System 52. Gallstones: (all correct except one) A. Have an incidence which increases with age. B. Are more frequent in females. C. Usually contain a predominance of cholesterol. D. Are formed in bile which is supersaturated with bile acids. E. Are formed in bile which is supersaturated with cholesterol. 53. stones in the common bile duct: A. Are present in nearly 50 per cent of cases of cholecystitis. B. Often give rise to jaundice, fever and biliary colic. C. Are usually accompanied by progressive jaundice. D. Are usually associated with a distended gallbladder. E. A&D only. Colon, Rectum And Anu 54. Ulcerative colitis: A. Is more common in female than males. B. Appears most commonly between the ages of 20 and 30. C. Usually presents with abdominal discomfort and diarrhoea. D. Can usually be diagnosed on sigmoidoscopic examination. E. All of the above correc 55. Diverticular disease of the colon: A. Is usually asymptomatic. B. Often presents with lower abdominal pain. C. May present with severe rectal haemorrhage. D. May present with peritonitis. E. All of the above are correct. 56. Colonic polyps: (all correct except one) A. Are associated with colonic cancer. B. May be hereditary. C. Should not be removed if they are asymptomatic. D. May be hyperplastic. E. Are commonly adenomatous. Thyroid , Parathyroid 57. Signs and symptoms of hyperthytoidism include: (All correct except one) A. Decreased sweating. B. An irregular pulse rate. C. Cardiac failure. D. Diplopia. E. Loss of weight. 58. a multinodular (adenomatous) goitre: A. Is more common in those patients having a deficient iodine intake. B. Is usually preceded by a diffuse goitre in early adult life C. Is rarely a precancerous condition. D. Is effectively treated in early stage with thyroid hormones to prevent further adenomatous changes. E. All of the above.
317
59. Hashimoto’s disease: A. Is often associated with increased levels of circulating thyroid antibodies. B. Is characterized by lymphocytic infiltration and fibrosis of the thyroid gland. C. Is usually treated by sub-total thyroidectomy. D. Should be treated by anti-thyroid drugs. E. A&B only. 60. The surgical treatment of thyroid cancer: A. Should be by total thyroidectomy in the potentially curable patient. B. Should include block dissection of the neighboring lymph nodes whether or not they appear to contain tumour. C. Is most successful in the papillary type of cancer. D. Should be preceded by a therapeutic dose of iodine 131. E. A& C only. 61. The earliest symptoms of hyperparathyroidism include: A. Diarrhea. B. Polydipsia and polyuria. C. Unexplained weight gain. D. Muscle spasm E. C&D only. 62. Phaeochromocytomata: A. Are tumors of the spinal nerve roots. B. Are frequently multiple. C. Characteristically present with a yellow skin discoloration. D. Can be effectively managed by long term medical therapy. E. C&D only. 63. In cushing’s syndrome: A. There is usually an increased deposition of fat over the face and trunk. B. Hypotension and hypokalaemia are frequently present. C. There is an increased production of adrenal hormones. D. The cause is most frequently an adrenal tumor. E. A&C only. Surgical Infection 64. Streptococcal infections: A. Are characterised by abscess formation. B. Rarely produce lymphadenitis. C. Do not produce baceraemia. D. Can produce a gangrenous skin infection. E. All of the above. 65. A pelvic abscess: A. Lies extrapeitoneally. B. May be a complication of abdominal surgery. C. Rarely presents with diarrhoea. D. Should be treated with antibiotics alone. E. A&C only 66. Heavily contaminated and dirty wounds: A. Require surgical toilet and delayed closure. B. Require the administration of systemic antibiotics. C. Can usually be treated by wound toilet and primary closure. D. Should be totally excised. E. A&B only. Hernia
318
67. Inguinal herniae: A. In children are usually of the direct type. B. Of the indirect type are congenital in origin. C. Will regress spontaneously in children. D. In young adults are most commonly of the direct type. E. C&D only. 68. Strangulated contents of hernial sacs: A. Are always accompanied by intestinal obstruction. B. Are more common in direct than indirect inguinal herniae. C. Are usually reducible. D. Produce local pain and tenderness. E. All of the above 69. Incisional herniae are related to:(ِAll correct except one) A. Wound infections. B. Anaemia and malnutrition. C. Obesity. D. The use of absorbable suture materials. E. To the technique of wound closure. 70. True statements concerning a femoral hernia include which of the following? A. It is common in male. B. It is usually results from a defect in lateral part of transversalis fascia. C. It is common in children. D. It is less common in female. E. It may be confused with inguinal lymphadenopathy. Vascular Systems 71. Acute arterial occlusion: A. Should be treated conservatively if the site of the occlusion is above the inguinal ligament. B. Demands the urgent use of vasodilator drugs. C. Of a limb is usually painless due to the anoxic damage produced in the peripheral nerves. D. May produce irreversible muscle necrosis after 6 hours. E. B&C only. 72. Common sites for atheromatous arterial aneurysms are: A. The femoral artery. B. The middle cerebral artery. C. The abdominal aorta. D. Intrarenal. E. A&C only 73. The long saphenous vein: A. Arises on the medial aspect of the sole of the foot. B. Passes 1cm in front of the medial malleolus. C. Passes in front the knee joint. D. Enters the femoral sheath by piercing the fascia lata. E. Related to the sural nerve 74. In deep venous thrombosis of the lower limb: A. One of the most common sites of origin is the short saphenous vein. B. One of the common sites of origin is in the iliofemoral segment. C. The diagnosis can usually be made by clinical examination. D. Tender swollen thrombosed veins are usually palpable. E. Usually associated with varicose vein. Neurosurgery
319
75. In head injuries the causes of a rising intracranial pressure: (all correct except one) A. Intracerebral haemorrhage. B. Cerebral oedema. C. Rhinorrhoea. D. Meningitis. E. Extradural hemorrhage. 76. Birth injuries involving the fifth and sixth cervicaal nerve roots of the brachial plexus: A. Are known as klumpke’s palsy. B. Are rarely followed by full recovery. C. Are characterised by the arm being held in the pronated and internally rotated position. D. Show weakness and wasting of the small muscles of the hand. E. Non of the above correct. 77. Following a peripheral nerve injury: A. Loss of axon continuity is described as neuropraxia. B. Due to gunshot wounds primary nerve repair is desirable. C. Delayed suture is best performed one week after the injury. D. Delayed suture is best performed three months after the injury. E. None of the above is correct. 1 2 3 4 5 6 7 8 9 10 11 12 13 14 15 16 17 18 19 20
D D D B D A E D D A A A A D D D D D B A
21 22 23 24 25 26 27 28 29 30 31 32 33 34 35 36 37 38 39 40
D A+B C D A D C E A E C B C A A A D A D C
41 42 43 44 45 46 47 48 49 50 51 52 53 54 55 56 57 58 59 60
A B D A B A B B E E B D B E E C A E E E
61 62 63 64 65 66 67 68 69 70 71 72 73 74 75 76 77
B B E D B E B D B E D E B B C C E
1-A 35-year-old male is struck on the lateral aspect of his right knee by the bumper of a car travelling at low velocity. On examination he is unable to dorsiflex the ankle, evert the foot and extend the toes. There is loss of sensation of the dorsum of the foot. He is most likely to have damaged which structure? f. Common peroneal nerve g. Deep peroneal nerve h. Saphenous nerve
320
2-Which one of the following suggest a diagnosis of Hirschsprung's disease? u. A contrast-study showing dilatation of the aganglionic bowel segment. v. Early presentation with vomiting. w. neonatal large bowel obstruction. x. Presentation after 1 year of age. y. Red current jelly stools. 3-A 46-year-old man presents after penetrating injuries to his arm and forearm. He is unable to extend his fingers. There is no sensory disturbance and there is no vascular injury. Which nerve has been damaged? f. g. h. i. j.
anterior interosseous nerve median nerve posterior interosseous nerve radial nerve ulna nerve
4-Which of the following regarding the anatomy of the heart is true? p. The aortic valve is tricuspid. q. The ascending aorta is entirely outside the pericardial sac. r.
The left atrial appendage is identified readily by transthoracic echocardiography.
s. The pulmonary trunk lies anterior to the ascending aorta. t.
The right atrium is posterior to the left atrium.
5-Which of the following is not within the carpal tunnel? f. g. h. i. j.
median nerve flexor digitorum profundus flexor digitorum superficialis flexor pollicis longus? abductor pollicis longus
6-A 48-year-old woman with chronic pancreatitis due to gallstones is noted to have a macrocytic anemia. What is the most likely cause of the anemia? f. g. h. i. j.
Bone marrow dysfunction Folate deficiency Hyposplenism Hypothyroidism Vitamin B12 deficiency
7-Which of the following organs is in direct contact with the anterior surface of the left kidney, without being separated from it by visceral peritoneum? f.
321
Duodenum
g. h. i. j.
Jejunum Pancreas Spleen Stomach
8-A 5-year-old boy presents to Accident and Emergency complaining of acute pain over his upper tibia. He is febrile and he refuses to move his leg. A diagnosis of osteomyelitis is suspected. The likely infecting organism is? f. g. h. i. j.
Clostridium difficile Haemophilus influenzae Pseudomonas Salmonella Staphylococcus aureus
9-Which is the principle root inervation for the small muscles of the hand? f. g. h. i. j.
C5 C6 C7 C8 T1
10-Which of the following is true concerning Scaphoid fractures? p. Rarely occur in young adults q. when complicated by avascular necrosis the proximal pole is usually affected r.
should be treated by bone grafting and internal fixation even if undisplaced
11-A 17-year-old girl underwent emergency splenectomy after a domestic accident. Which one of the following organisms is most likely to cause life-threatening infection in the future? f. g. h. i. j.
Actinomycosis Haemophilus influenzae Pseudomonas aeruginosa Staphylococcus aureus Streptococcus pneumonia
12-Which vertebral level and corresponding structure is correct? f. g. h. i. j.
322
C4 and bifurcation of the carotid artery T2 and manubriosternal joint T10 and opening for vena cava in diaphragm T12 and oesophageal opening in the diaphragm T8 and aortic opening in the diaphragm
13-Which of the following statements is true of Compartment syndrome:f. Only occurs following fractures. g. Loss of distal pulse is an early sign. h. The presence of pain is unhelpful in diagnosis. i.
Passive stretch of affected muscles exacerbates pain.
14-The axillary nerve passes directly inferior to which muscle as it leaves the axilla? f. g. h. i. j.
Long head of triceps Long head of biceps Pectoralis major Subscapularis Teres minor
15-Which of the following is not a branch of the pudendal nerve? f. g. h. i. j.
Perineal nerve. Dorsal nerve of the penis. Inferior rectal nerve. Genitofemoral nerve. Posterior scrotal nerve
16-A patient presents with a history of low back pain and sciatica. The pain radiates to the little toe, the ankle reflex is absent and the patient has difficulty in everting the foot. Which nerve root is likely to be trapped? f. g. h. i. j.
L3 L4 L5 S1 S2
17-The commonest clinical manifestation of primary hyperparathyroidism is: f. g. h. i. j.
Renal stone disease Bone disease Peptic ulceration Constipation Polyuria
18-Which of the following structures accompany the median nerve in the carpal tunnel? f.
323
Flexor carpi ulnaris.
g. h. i. j.
Flexor digitorum profundis. The ulnar artery. All of the above. None of the above.
19-Which of the following is a branch of the LATERAL CORD of the brachial plexus? f. g. h. i. j.
Suprascapular nerve. Lower subscapular nerve. Medial pectoral nerve. Musculo-cutaneous nerve. Upper subscapular nerve.
20-Which of the following vertebrae has the most prominent spinous process? e. f. g. h.
T1. T2. C7. T11.
21-The ulnar nerve all are true except : L. innervates the first dorsal interosseus muscle M. originates from the laterall cord of the brachial plexus N. has no branches above the elbow O. innervates the medial half of the flexor digitorum profundus P. innervates the adductor pollicis muscle 22-A 9 month old boy presents with an acute scrotal swelling. The following diagnoses are likely: F. G. H. I. J.
Epididymitis Orchitis Torsion of the testicular appendage Irreducible inguinal hernia Acute idiopathic scrotal oedema
23-Congenital pyloric stenosis: f. g. h. i. j.
Presents in the first few daies after birth. Can cause hyperchloraemic alkalosis. Is inherited as an autosomal recessive. Typically presents with bile stained vomiting after feeds Is often diagnosed by feeling a mass in the right hypochondrium .
25-Meckel's diverticulum all are true except : f. Can present with chronic anaemia. g. Is present in 2% of the population. h. Can be diagnosed by hydrogen breath test.
324
i. j.
May contain ectopic pancreatic tissue. May be implicated in Littre's hernia.
26-Which of the following statements is true of upper limb nerve injuries? p. Injury to the median nerve results in a wrist drop q. Injury to the radial nerve results in loss of sensation over the palmar aspect of the index finger r. Injury to the median nerve results in loss of sensation in the anatomical snuffbox s. Injury to the ulnar nerve results in a claw hand t. Injury to the ulnar nerve results in loss of sensation over the thumb 27-Concerning the inguinal canal: f. g. h. i. j.
It transmits the ilio-inguinal nerve The deep inguinal ring lies below the mid-point of inguinal ligament The superficial inguinal ring overlies the pubic tubercle Laterally , the anterior wall is made up of the external oblique aponeurosis Laterally, the posterior wall is formed by the conjoint tendon
28-In chest trauma, urgent cardiothoracic surgical referral is necessary for all except: f. g. h. i. j.
Continuing massive air leak following insertion of chest drain Cardiac tamponade Disruption of the great vessels Severe pulmonary contusion Continuing haemorrhage following insertion of chest drain
29-A raised titre of anti-HBs in the bloodallare true except : e. f. g. h.
signifies previous hepatitis B infection is produced after hepatitis B vaccination indicates immunity to hepatitis infection indicates active hepatitis B infection
30-The following statements regarding intravenous solutions is correct: o. p. q. r.
Normal saline contains 180mmol/l of sodium Ringer's lactate solutions is designed for intracellular fluid replacement Sodium bicarbonate 8.4% is a hyperosmolar solution Normal saline with added potassium is appropriate therapy to correct a respiratory alkalosis
31-The following is true of the blood supply of the rectum all are true except: f. The principle blood supply is derived from the inferior rectal artery g. The median sacral vessels contribute to the blood supply h. As the inferior mesenteric artery crosses the pelvic brim it becomes known as the superior rectal artery
325
i.
The internal venous rectal plexus is continuous with the vascular cushions of the anal canal The superior rectal vein drains to the portal system
j.
32-Which of the following concerning the Femoral sheath is false: o. p. q. r.
Contains the femoral artery Contains lymph nodes Contains the femoral canal Contains femoral nerve
33-The following is true of the spleen: p. q. r. s. t.
Is the largest lymphoid organ in the body Lies obliquely between the seventh and tenth rib The lower pole extends beyond the mid-axillary line Is usually palpable when normal Usually measures 16cm in maximum length when healthy
34-A knife stabbed horizontally through the fourth intercostals space to the right of the sternum is likely to damage the: f. g. h. i. j.
Right brachiocephalic veinHemiazygos vein Descending aorta Right pulmonary artery Thoracic duct
35-Breast cancer risk is increased in association with the following factors except: p. q. r. s. t.
Nulliparity Immediately after pregnancy Early menarche Early age at first pregnancy Late menopause
36-The Appendix all true except: k. Is typically less than 10 cm in length in the adult. l. Is located in the retrocaecal recess. m. Macburneys point, lies 2/3 laterally from a line from umbilicus to the anterior superior iliac spine. n. The longitudinal coat of the appendix is derived from the three bands of taenia coli. o. Is supplied by branches of the inferior mesenteric artery 37-In tension pneumothorax the following signs are present except:
326
u. v. w. x. y.
Hypoxia Hyper resonance to percussion on the affected side Tracheal deviation to the ipsilateral side Distended neck veins Tachycardia
38-The following are encountered during inguinal hernia repair : f. g. h. i. j.
Scarpas' fascia Colle's fascia Ilioinguinal nerve Cremaster muscle Genitofemoral nerve
39- Which of the following variables best predicts prognosis for patients with a recent diagnosis of cutaneous melanoma and no clinical evidence of metastatic disease? f.
Breslow thickness .
40- The intravenous fluid that a 60 kg., 30-year-old woman with an 80% burn should be given in the first 24 hours following burn injury is: g. 19.2 liters of 5% glucose in lactated Ringer's. h. 14.4 liters of lactated Ringer's. 41-All venous sinuses of the dura mater ultimately drain into : f.
Internal jugular vein
42--The tendon of biceps bronchii muscle inserted into the f.
Radial tuberosity
43--the saphenous nerve is terminal branch of the : f. g. h. i. j.
Ilioinguinal nerve Femoral nerve Genitofemoral nerve Superior glutial nerve popliteal nerve
44- All the following origenat on the scapula except f. g. h. i. j.
327
Trapezius Teres major Deltoid Brachio-radialis Coraco-brachialis
45- Total number of human vertebra is: f.
33 vertebras.
46-Foot droop is usually associated with paralysis of the: k. Common peroneal 47--The following pass through the aortic hiatus of the diaphragm f. g. h. i. j.
Aorta Azygos vein Thoracic duct All of the above Non of the above
48-At birth the umbilical vein become the: f.
Round ligament of the liver
49- Arrangement of structures at hilum of the right kidney as following from anterior to posterior: f.
Vein ,artery , ureter
50-Shock can best be defined as: m. Hypotension. n. Hypo perfusion of tissues. o. Hypoxemia. p. All of the above. 51-All of the following are true about neurogenic shock except: i.
there is a decrease in systemic vascular resistance and an increase in venous capacitance. j. tachycardia or bradycardia may be observed, along with hypotension. k. the use of an alpha agonist such as phenylephrine is the mainstay of treatment . l. Severe head injury, spinal cord injury, and high spinal anesthesia may all cause neurogenic shock.
52-All of the following may be useful in the treatment of cardiogenic shock except: e. f. g. h.
328
Dobutamine. Sodium nitroprusside. Pneumatic antishock garment. Intra-aortic balloon pump.
53-All of the following statements about hemorrhagic shock are true except: e. Following hemorrhagic shock, there is an initial interstitial fluid volume contraction. f. Dopamine, or a similar inotropic agent, should be given immediately for resuscitation from hemorrhagic shock, to increase cardiac output and improve oxygen delivery to hypoperfused tissues. g. The use of colloid solutions or hypertonic saline solutions is indicated for treatment of hemorrhagic shock. h. In hemorrhagic shock, a narrowed pulse pressure is commonly seen before a fall in systolic blood pressure. 54-Which of the following statements about head injury and concomitant hyponatremia is true? f. g. h. i. j.
There are no primary alterations in cardiovascular signs. Signs of increased intracranial pressure may be masked by the hyponatremia. Oliguric renal failure is an unlikely complication. Rapid correction of the hyponatremia may prevent central pontine injury. This patient is best treated by restriction of water intake.
55-Which of the following statements about extracellular fluid is true? P. The total extracellular fluid volume represents 40% of the body weight. Q. The plasma volume constitutes 20%of the total extracellular fluid volume . R. Potassium is the principal cation in extracellular fluid. S. The protein content of the plasma produces a lower concentration of cations than in the interstitial fluid. T. The interstitial fluid equilibrates slowly with the other body compartments.
56-Which of the following statements is true of a patient with hyperglycemia and hyponatremia? f. Insulin administration will increase the potassium content of cells . g. The sodium concentration must be corrected by 5 mEq. per 100 mg. per 100 ml. elevation in blood glucose. h. With normal renal function, this patient is likely to be volume overloaded. i. Proper fluid therapy would be unlikely to include potassium administration. 57-Which of the following is not associated with increased likelihood of infection after major elective surgery? K. Age over 70 years.
329
L. Chronic malnutrition. M. Controlled diabetes mellitus. N. Long-term steroid use. O. Infection at a remote body site. 58-The transfusion of fresh frozen plasma (FFP) is indicated for which of the following reasons? e. For the correction of abnormal PT secondary to warfarin therapy, vitamin K deficiency, or liver disease. f. Volume replacement. g. As a nutritional supplement. h. Treatment of prolonged bleeding time 59-In patients receiving massive blood transfusion for acute blood loss, which of the following is correct? e. Packed red blood cells and crystalloid solution should be infused to restore oxygen-carrying capacity and intravascular volume. f. Two units of FFP should be given with every 5 units of packed red blood cells in most cases. g. A “six pack” of platelets should be administered with every 10 units of packed red blood cells in most cases. h. One to two ampules of sodium bicarbonate should be administered with every 5 units of packed red blood cells to avoid acidosis. 60-Hemostasis and the cessation of bleeding require which of the following processes? k. Adherence of platelets to exposed subendothelial glycoproteins and collagen with subsequent aggregation of platelets and formation of a hemostatic plug. l. Interaction of tissue factor with factor VII circulating in the plasma. m. The production of thrombin via the coagulation cascade with conversion of fibrinogen to fibrin. n. Cross-linking of fibrin by factor XIII. o. All of the above 61- Advantages of epidural analgesia include: f. g. h. i. j.
Earlier mobilization after surgery. Earlier return of bowel function. Shorter hospitalizations. Decreased stress response to surgery. All of the above.
62- Factors that decrease collagen synthesis include all of the following except: y. Protein depletion. z. Infection. aa. Anemia.
330
bb.Advanced age. cc. Hypoxia. 63-Which of the following statement is true concerning excessive scarring processes? e. f. g. h.
Keloids occur randomly regardless of gender or race Hypertrophic scars and keloid are histologically different Keloids tend to develop early and hypertrophic scars late after the surgical injury Simple reexcision and closure of a hypertrophic scar can be useful in certain situations such as a wound closed by secondary intention
64-Which of the following statement is true concerning the vascular response to injury? e. Vasoconstriction is an early event in the response to injury f. Vasodilatation is a detrimental response to injury with normal body processes working to avoid this process g. Vascular permeability is maintained to prevent further cellular injury h. Histamine, prostaglandin E2 (PGE2) and prostacyclin (PGI2) are important mediators of local vasoconstriction 65-=All are true ligamentous attachments of liver except e. f. g. h.
Falciform ligament Coronary Ligaments Hepatoduodenal ligaments Glisson's capsul
67- Nasotracheal intubation all true except : e. f. g. h.
Is preferred for the unconscious patient without cervical spine injury. Is preferred for patients with suspected cervical spine injury . Maximizes neck manipulation. Is contraindicated in the patient who is breathing spontaneously.
68-. The radiographic findings indicating a torn thoracic aorta include all except : i. j. k. l.
Widened mediastinum. Presence of an apical “pleural cap.” Tracheal deviation to the right. right hem thorax.
69-Which of the following steps is not a part of the primary survey in a trauma patient? e. f. g. h.
331
Insuring adequate ventilatory support Measurement of blood pressure and pulse Neurologic evaluation with the Glasgow Coma Scale Examination of the cervical spine
70-Which of the following statement(s) is/are true concerning the Advanced Trauma Life Support (ATLS) classification system of hemorrhagic shock? e. Class I shock is equivalent to voluntary blood donation f. In Class II shock there will be evidence of change in vital signs with tachycardia, tachypnea and a significant decrease in systolic blood pressure g. Class IV hemorrhage can usually be managed by simple administration of crystalloid solution h. Class III hemorrhage involves loss of over 40% of blood volume loss and can be classified as life-threatening 71- The clinical picture of gallstone ileus includes all except ? f. g. h. i. j.
Air in the biliary tree. Small bowel obstruction. A stone at the site of obstruction. Acholic stools. Associated bouts of cholangitis.
72- The evaluation of a comatose patient with a head injury begins with: e. Establishment of an airway. 73. Which of the following signs does Horner's syndrome include? e. f. g. h.
Ptosis. Facial hyperhidrosis. Exophthalmos. Mydriasis.
74-. The goals of proper fracture reduction include all except? e. f. g. h.
Providing patient comfort and analgesia. Allowing for restoration of length of the extremity. Correcting angular deformity and rotation. Enabling immediate motion of all fractured extremities .
75-. The neurovascular structure most commonly injured as a result of an anterior dislocation of the shoulder is the: e. f. g. h.
Musculocutaneous nerve. Axillary nerve. Axillary artery. Median nerve.
76-. The radial nerve is at greatest risk for injury with which fracture? e. Fracture of the surgical neck of the humerus. f. Fracture of the shaft of the humerus .
332
g. Supracondylar fracture of the humerus. 77- The most consistent sign of a fracture of the carpal scaphoid is: e. f. g. h.
Wrist pain during attempted push-ups. Diffuse swelling on the dorsum of the wrist. Localized tenderness in the anatomic snuffbox . Wrist popping on movement.
78. Hematogenous osteomyelitis most frequently affects: f. g. h. i. j.
The diaphysis of long bones. The epiphysis. The metaphysis of long bones. Flat bones. Cuboidal bones.
79- The most common hernia in females is: p. Femoral hernia. q. Direct inguinal hernia. r. Indirect inguinal hernia. 80- Which of the following most often initiates the development of acute appendicitis? i. A viral infection. j. Acute gastroenteritis. k. Obstruction of the appendiceal lumen. 81-. The most helpful diagnostic radiographic procedure in small bowel obstruction is: o. p. q. r.
CT of the abdomen. Contrast study of the intestine. Supine and erect x-rays of the abdomen . Ultrasonography of the abdomen.
82-. Which of the following treatments should never be recommended to a patient with purely intraductal carcinoma? e. f. g. h.
Modified radical mastectomy. Lumpectomy to clear surgical margins, followed by observation. Incisional biopsy with an involved margin, followed by radiation . Excisional biopsy to clear margins, followed by radiation.
83- In role of nine extent of burn if entire trunk is burned it will be equal to: p. 9% body surface area. q. 18% body surface area.
333
r. 36% body surface area. s. 27% body surface area. 84-. Which of the following does not describe intermittent claudication? e. f. g. h.
Is elicited by reproducible amount of exercise. Abates promptly with rest. Is often worse at night. May be an indication for bypass surgery.
85- Which of the following statements are true? f. g. h. i. j.
Patients with critical limb ischemia have paralysis and paresthesias. All arterial injuries are associated with absence of a palpable pulse. Preoperative arteriography is required to diagnose an arterial injury. The presence of Doppler signals indicates that an arterial injury has not occurred. In all patients with multiple trauma, arterial injuries should be repaired before other injuries are addressed.
86-. The appropriate surgical treatment for suspected carcinoma of the testis is: e. Inguinal exploration, control of the spermatic cord, biopsy, and radical orchectomy if tumor is confirmed. 87-. If torsion of the testicle is suspected, surgical exploration: m. Should be immediate and include the asymptomatic side. 88- A patient with acute urinary tract infection (UTI) usually presents with: f. g. h. i.
Chills and fever. Flank pain. Nausea and vomiting. Painful urination.
89-. The most precise diagnostic screening procedure for differentiating benign thyroid nodules from malignant ones Fine-needle-aspiration biopsy (FNAB). 90- Hyperthyroidism can be caused by all of the following except: p. q. r. s. t.
Graves' disease. Plummer's disease. Struma ovarii. Hashimoto's disease. Medullary carcinoma of the thyroid.
1. Which of the following anatomic features of the biliary system are important considerations in operative cholangiography?
334
A. The left hepatic duct comes off farther anterior than the right one. B. At the confluence there may be more than just a right and a left hepatic duct. C. Dissection of the triangle of Calot is more important than cholangiography in preventing bile duct injury. D. Segments V, VII, or VIII sometimes join the biliary system below the confluence. E. All of the above are correct. 2. Which of the following statements characterize amebic abscess? A. Mortality is higher than that for similarly located pyogenic abscesses. B. The diagnosis of amebic abscess may be based on serologic tests and resolution of symptoms. C. In contrast to pyogenic abscess, the treatment of amebic abscess is primarily surgical. D. Patients with amebic abscess tend to be older than those with pyogenic abscess.
3. Which of the following statement(s) is/are true about benign lesions of the liver? A. Adenomas are true neoplasms with a predisposition for complications and should usually be resected. B. Focal nodular hyperplasia (FNH) is a neoplasm related to birth control pills (BCPs) and usually requires resection. C. Hemangiomas are the most common benign lesions of the liver that come to the surgeon's attention. D. Nodular regenerative hyperplasia does not usually accompany cirrhosis. 4. Which of the following statements about hemobilia are true? A. Tumors are the most common cause. B. The primary treatment of severe hemobilia is an operation. C. Percutaneous cholangiographic hemobilia is usually minor. D. Ultrasonography usually reveals a specific diagnosis. 5. Ligation of all of the following arteries usually causes significant hepatic enzyme abnormalities except: A. Ligation of the right hepatic artery. B. Ligation of the left hepatic artery. C. Ligation of the hepatic artery distal to the gastroduodenal branch. D. Ligation of the hepatic artery proximal to the gastroduodenal artery 6. Which of the following is the most common acid-base disturbance in patients with cirrhosis and portal hypertension? C. Metabolic alkalosis. 7. Cytokines are endogenous signals that stimulate: A. Septic shock B. The central nervous system to initiate fever. C. The production of “acute-phase proteins.” D. Hypoferremia. E. All of the above are correct. 8. Which of the following statements about maxillofacial trauma is/are false? A. Asphyxia due to upper airway obstruction is the major cause of death from facial injuries. B. The mandible is the most common site of facial fracture. C. The Le Fort II fracture includes a horizontal fracture of the maxilla along with nasal bone fracture. D. Loss of upward gaze may indicate either an orbital floor or orbital roof fracture. 9. A 28-year-old male was injured in a motorcycle accident in which he was not wearing a helmet. On admission to the emergency room he was in severe respiratory distress and hypotensive (blood pressure 80/40 mm. Hg), and appeared cyanotic . He was bleeding profusely from the nose and had an obviously open femur fracture with exposed bone . Breath
335
sounds were decreased on the right side of the chest. The initial management priority should be: A. Control of hemorrhage with anterior and posterior nasal packing. B. Tube thoracostomy in the right hemithorax. C. Endotracheal intubation with in-line cervical traction. D. Obtain intravenous access and begin emergency type O blood transfusions. E. Obtain cross-table cervical spine film and chest film. 10. Regarding the diagnosis and treatment of cardiac tamponade, which of the following statements is/are true? A. Accumulation of greater than 250 ml. of blood in the pericardial sac is necessary to impair cardiac output. B. Beck's classic triad of signs of cardiac tamponade include distended neck veins, pulsus paradoxicus, and hypotension. C. Approximately 15% of needle pericardiocenteses give a false-negative result. D. Cardiopulmonary bypass is required to repair most penetrating cardiac injuries. 11.Which of the following statement(s) is true concerning the diagnosis of a peripheral vascular injury? A. The presence of a Doppler signal over an artery in an extremity essentially rules out an arterial injury B. Doppler examination is a valuable tool in the diagnosis of venous injuries C. A gunshot wound in the proximity of a major vessel is an absolute indication for arteriography D. Both the sensitivity and specificity of arteriography of the injured extremity approaches 100% 12. An 18-year-old male suffers a gunshot wound to the abdomen, resulting in multiple injuries to the small bowel and colon. Which of the following statement(s) is/are true concerning this patient’s perioperative management? A. A multi-agent antibiotic regimen is indicated B. Antibiotics should be continued postoperatively for at least 7 days C. Laparotomy, as a diagnostic test for postoperative sepsis, should be considered D. The incidence of postoperative wound or intraabdominal infection would be increased in association with a colon injury 13. A 75-year-old man is involved in a motor vehicle accident. Which of the following statement(s) is/are true concerning this patient’s injury and management? A. Acceptable vital sign parameters are similar across all age groups B. Hypertonic solutions should not be used for resuscitation due to concerns for fluid overload C. The patient would be more prone to a subdural hematoma than a younger patient D. There is no role for inotropic agents in the management of this patient 14. Valid points in the management of burns on special areas include: A. The large majority of genital burns are best managed by immediate excision and autografting B. All digits with deep dermal and full-thickness burns should be immobilized with six weeks of axial Kirschner wire fixation C. Deep thermal burns of the central face are best managed with immediate excision and autografting D. Burns of the external ear are commonly complicated by acute suppurative chondritis if topical mafenide acetate is not applied 15. Which of the following statements regarding unusual hernias is incorrect? A. An obturator hernia may produce nerve compression diagnosed by a positive HowshipRomberg sign. B. Grynfeltt's hernia appears through the superior lumbar triangle, whereas Petit's hernia occurs through the inferior lumbar triangle. C. Sciatic hernias usually present with a painful groin mass below the inguinal ligament.
336
D. Littre's hernia is defined by a Meckel's diverticulum presenting as the sole component of the hernia sac. E. Richter's hernia involves the antimesenteric surface of the intestine within the hernia sac and may present with partial intestinal obstruction. 16. The following statement(s) is/are true concerning umbilical hernias in adults. A. Most umbilical hernias in adults are the result of a congenital defect carried into adulthood B. A paraumbilical hernia typically occurs in multiparous females C. The presence of ascites is a contraindication to elective umbilical hernia repair. D. Incarceration is uncommon with umbilical hernias 17. Which of the following statements about esophageal anatomy is correct? A. The esophagus has a poor blood supply, which is segmental in distribution and accounts for the high incidence of anastomotic leakage. B. The esophageal serosa consists of a thin layer of fibroareolar tissue. C. The esophagus has two distinct muscle layers, an outer, longitudinal one and an inner, circular one, which are striated in the upper third and smooth in the distal two thirds. D. Injury to the recurrent laryngeal nerve results in vocal cord dysfunction but does not affect swallowing. E. The lymphatic drainage of the esophagus is relatively sparse, localized primarily to adjacent paraesophageal lymph nodes. 18. Which of the following is most reliable for confirming the occurrence of a significant esophageal caustic injury? A. History of the event. B. Physical examination of the patient. C. Barium esophagraphy. D. Endoscopy. 19. Numerous epidemiologic associations have been made between (1) environmental and dietary factors and (2) the incidence of gastric cancer, including all except: A. Dietary nitrites. B. Dietary salt. C. Helicobacter pylori infection. D. Dietary ascorbic acid. 20. Complete mechanical small bowel obstruction can cause dehydration by: A. Interfering with oral intake of water. B. Inducing vomiting. C. Decreasing intestinal absorption of water. D. Causing secretion of water into the intestinal lumen. E. All of the above 21. Meckel's diverticulum most commonly presents as: A. Gastrointestinal bleeding. B. Obstruction. C. Diverticulitis. D. Intermittent abdominal pain. 22. A 45-year-old man with a history of previous right hemicolectomy for colon cancer presents with colicky abdominal pain which has become constant over the last few hours. He has marked abdominal distension and has had only minimal vomiting of a feculent material. His abdomen is diffusely tender. Abdominal x-ray shows multiple air fluid levels with dilatation of some loops to greater than 3 cm in diameter. The most likely diagnosis is: A. Proximal small bowel obstruction B. Distal small bowel obstruction C. Acute appendicitis D. Closed-loop small bowel obstruction 23. Which of the following statement(s) is true concerning laboratory tests which might be obtained in the patient discussed above?
337
A. The presence of a white blood cell count > 15,000 would be highly suggestive of a closedloop obstruction B. Metabolic acidosis mandates emergency exploration C. An elevation of BUN would suggest underlying renal dysfunction D. There is no rapidly available test to distinguish tissue necrosis from simple bowel obstruction 24. Recurrence after operation for Crohn's disease: A. Occurs after operations for ileal Crohn's but not colonic Crohn's. B. Is usually found just proximal to an enteric anastomosis. C. Rarely requires reoperation. D. Occurs in 1% of patients at risk per year during the first 10 years after the operation. E. Is prevented by maintenance therapy with corticosteroids. 25. The most common indication for surgery secondary to acute diverticulitis is: A. Abscess. 26. Many patients with ulcerative colitis are operated upon electively with total abdominal colectomy, rectal mucosectomy, formation of a small intestinal reservoir, and ileoanal anastomosis. The most common postoperative complication after this operation is which of the following? B. Small bowel obstruction 27. A 30-year-old male two years postoperative after total abdominal colectomy with ileoanal anastomosis reports a sudden increase in stool frequency, nocturnal leakage, and low-grade fevers. Physical examination is unremarkable . Flexible endoscopic examination of the small intestinal pouch reveals a friable erythematous mucosa .Biopsies of the mucosa are obtained. While awaiting biopsy results, which of the following is the most appropriate empiric therapy? C. Oral metronidazole 28. Useful methods for detection of splenic injury, in descending order of sensitivity, are: A. Magnetic resonance imaging (MRI) B. CT. C. Ultrasonography. D. Isotope scan. 29. Which of the following comments does not describe hypersplenism? A. It may occur without underlying disease identification. B. It may be secondary to many hematologic illnesses. C. It is associated with work hypertrophy from immune response. D. It requires evaluation of the myeloproliferation. E. It is associated with antibodies against platelets. 30.You are consulted regarding a 50-year old male with Laennec’s cirrhosis, portal hypertension and hypersplenism. He has no history of gastrointestinal bleeding. You would recommend which of the following? D. Observation 31. Which of the following statements about the differential diagnosis of hypercalcemia is correct. A. Malignant tumors typically cause hypercalcemia by ectopic production of parathyroid hormone (PTH). B. The diagnosis of primary hyperparathyroidism is supported by these serum levels: calcium, 10.8 mg. per dl.; chloride, 104 mmol. per liter; bicarbonate 21 mmol. per liter; phosphorus, 2.4 mg. per dl.; elevated parathyroid hormone. C. Familial hypocalciuric hypercalcemia is distinguished from primary hyperparathyroidism by parathyroid imaging. D. Although serum albumin binds calcium, the measured total calcium value is usually unaffected in patients with severe hypoproteinemia. E. Thiazide diuretics are a good treatment for hypercalcemia and can be given to patients with apparent hypercalcemia of malignancy. 32. The parathyroid glands:
338
A. Develop from the second and third pharyngeal pouches, along with the palatine tonsil and the thymus. B. Migrate caudally in the neck in normal development but can be found anywhere from the pharyngeal mucosa to the deep mediastinum. C. Secrete PTH and calcitonin to manage calcium homeostasis. D. Usually number four, but frequently number only two or three. E. Contain enzymes that catalyze the conversion of 25(OH) vitamin D 3 to 1,25(OH) 2 vitamin D 3. 33. Which of the following breast lesions are noninvasive malignancies? A. Intraductal carcinoma of the comedo type. B. Tubular carcinoma and mucinous carcinoma. C. Infiltrating ductal carcinoma and lobular carcinoma. D. Medullary carcinoma, including atypical medullary lesions. 34. A 45-year-old woman presents with a weeping eczematoid lesion of her nippl E. Which of the following statement(s) is/are true concerning her diagnosis and management? A. Treatment is with warm compresses and oral antibiotics B. Biopsy of the nipple revealing malignant cells within the milk ducts is invariably associated with an underlying invasive carcinoma C. The appropriate treatment is mastectomy D. The lesion always represents a high-risk disease with a significant risk of subsequent metastatic disease 35. The effect of high positive end-expiratory pressures (PEEP) on cardiac output is: A. None. B. Increased cardiac output. C. Decreased cardiac output because of increased afterload to the left ventricle. D. Decreased cardiac output because of decreased effective preload to the left ventricle 36. The diagnosis of myasthenia gravis can be confirmed most reliably using: D. Single-fiber EMG. 37. In an infant with suspected PDA, which of the following would be the optimal method of confirming the diagnosis? D. Two-dimensional echocardiography with continuous-wave and color-flow Doppler echocardiography. 38. A 5-year-old girl is found on routine examination to have a pulmonic flow murmur, fixed splitting of P2 and a right ventricular lift. The following is/are true: A. Cardiac catheterization is indicated if the chest film shows cardiomegaly B. Radiology report of “scimitar syndrome” findings on the chest film would indicate need for an arteriogram C. If the catheterization report is “ostium secondum defect,” at least one pulmonary vein drains anomalously D. Measured pulmonary vascular resistance of 14 Woods units/m2 with an ASD mandates early repair E. An ASD with Qp/Qs of 1.8 can be observed until symptoms occur 39. The single most important indication for emergency pulmonary embolectomy is: C. The presence of persistent and intractable hypotension. 40. A 67-year-old man with acute popliteal arterial embolism has a negative cardiac echo for source of the thrombus. The following is/are true: A. Most likely non-cardiac source is a thoracic aortic aneurysm B. Embolism is more common from femoral than popliteal arterial aneurysms C. Emboli from popliteal aneurysms are often clinically silent D. Embolism is rare from subclavian artery aneurysms 41. Which of the following statements about allograft rejection are true? A. In the absence of immunosuppression, the time and intensity of rejection of transplants between unrelated donors and recipients is highly variable. B. Allograft rejection may be mediated by antibodies or by cells.
339
C. Allograft rejection is thought to be caused by Th2 cells. D. Acute cellular rejection is the major cause for loss of clinical organ transplants. E. An individual with “tolerance” is unable to reject an allograft. 42. Which of the following are contraindications to lung transplantation? A. Age 65 years or older. B. Current corticosteroid therapy. C. History of thoracotomy. D. Ventilator-dependent respiratory failure. 43. Familial hypercholesterolemia has been proposed as a disease to be treated by gene therapy. The molecular basis of familial hypercholesterolemia is which of the following? A. Absence of hepatic low density lipoprotein receptors B. Overproduction of high density lipoprotein C. Absence of lipoprotein lipase D. Overproduction of hepatic ornithine transcarbamylase 44. The pentalogy of Cantrell includes all of the following except: A. Epigastric omphalocele. B. Sternal cleft. C. Intracardiac defect. D. Pericardial cyst. E. Ectopia cordis. 45. Patients with Wilms’ tumors most frequently present with which of the following? E. A unifocal, unilateral lesion 46. Renal adenocarcinomas: A. Are of transitional cell origin. B. Usually are associated with anemia. C. Are difficult to diagnose. D. Are extremely radiosensitive. E. Frequently are signaled by gross hematuria. 47. A 55-year-old male presents with severe flank pain radiating to the groin associated with nausea and vomiting. Urinalysis reveals hematuria .A plain abdominal film reveals a radiopaque 5 mm stone in the area of the ureterovesical junction. Which of the following statement(s) is/are true concerning this patient’s diagnosis and management? A. A likely stone composition for this patient would be uric acid B. The stone will likely pass spontaneously with the aid of increased hydration C. Stone analysis is of relatively little importance D. Patients with a calcium oxalate stone and a normal serum calcium level should undergo further extensive metabolic evaluation 48. The most common physical findings in a patient with median nerve compression at the wrist (carpal tunnel syndrome) are: A. Diminished two-point discrimination and dryness of the index and long fingers. B. Atrophy of the abductor pollicis brevis and opponens pollicis. C. A positive percussion test at the wrist and a positive wrist flexion test producing paresthesias at the thumb, index, and long fingers. D. A weak grip in addition to hand cramping and difficulty writing. 49. Isolated flexor digitorum superficialis tendon function is determined by assessing: A. Flexion of the metacarpophalangeal joint. B. Flexion of the proximal interphalangeal joint with the adjacent digits held in extension. C. Flexion of the distal interphalangeal joint. D. Flexion of the proximal interphalangeal joint. 50. Which of the following statements are true? A. The Hoffmann-Tinel sign localizes the level of a nerve injury. B. Causalgia is a term used to denote the etiology of pain. C. Secondary repair of a lacerated nerve 3 to 8 weeks after injury has several advantages.
340
D. A surgeon who finds at delayed (3 to 8 weeks) exploration that a clinically nonfunctioning nerve is in continuity should resect the injured portion of the nerve and suture together the ends. E. If a nerve is found to be disrupted at delayed (3 to 8 weeks) exploration, the surgeon should find the two ends of the nerve and suture them together. 51. What is the critical difference between frame-based and frameless stereotactic procedures? A. The use of digitized imaging studies such as CT and MRI. B. The use of rendered three-dimensional images and a three-dimensional digitizer. C. Rigid fixation of the patient's head to the operating room table. D. The presence of a lesion in the brain on digitized imaging studies. E. The absence of a lesion in the brain on digitized imaging studies. 52. The following are absorbable sutures A. Catgut B. Silk C. Polyamide (Nylon) D. Polyglyconate (Maxon) E. Polyglactin (Vicryl)
53. During surgery on the submandibular gland A. An incision on the lower border of the mandible is safe B. The submandibular gland is seen to wrap around the posterior border of mylohyoid C. The facial artery and vein are divided as they course through the deep part of the gland D. The hypoglossal nerve is seen to loop under the submandibular duct E. Damage to the lingual nerve will cause loss of sensation to the posterior third of the tongue 54. Regarding pancreatic carcinoma false A. 90% are ductal adenocarcinomas B. Less than 20% occur in the head of the gland C. The usual presentation is with pain, weight loss and obstructive jaundice D. Ultrasound has a sensitivity of 80-90% in the detection of the tumour E. Less than 20% of patients are suitable for curative surgery 55. Regarding the management of major trauma A. Deaths follow a trimodal distribution B. X-rays after the primary survey should be of AP Cervical spine, chest and pelvis C. Cardiac tamponade is characterised by raised BP, low JVP and muffled heart sounds D. Assessment of uncomplicated limb fractures should occur during the primary survey E. Deterioration of the casualty during the primary survey should lead to the secondary survey 56. Regarding appendicitis A. The risk of developing the illness is greatest in childhood B. Mortality decreases with age . C. 20% of appendices are extraperitoneal in a retrocaecal position D. Faecoliths are present in 75-80% of resected specimens E. Appendicitis is a possible diagnosis in the absence of abdominal tenderness 57. Regarding stones in the gallbladder A. Cholesterol stones are the least common B. Pigment stones are due increased excretion of polymerised conjugated bilirubin C. Are not a risk factor for the development of gallbladder carcinoma D. 90% of gallstones are radio-opaque E. A mucocele of the gallbladder is caused by a stone impacted in Hartmann's pouch 58. Stones in the common bile duct the false is A. Are found in 30% of patients undergoing cholecystectomy (Without pre-op ERCP) B. Can present with Charcot's Triad C. Are suggested by an bile duct diameter >8mm on ultrasound D. ERCP, sphincterotomy and balloon clearance is now the treatment of choice
341
E. If removed by exploration of the common bile duct the T-tube can be removed after 3 days 59. Regarding crystalloid solutions the false is A. Normal saline contains 154 mmol sodium and 154 mmol of chloride B. 3 litres of dextrose saline in a day will provide 90 mmol of sodium C. 2 grams of potassium chloride is equal to 57 mmol of the salt D. Hartmann's solution contains calcium and bicarbonate E. The daily maintenance potassium requirement of a 40 Kg woman is about 40 mmol 60. Solitary thyroid nodules the false is A. Are more prevalent in women B. In the adult population less than 10% are malignant C. Less than 20% of scintigraphically cold nodules are malignant D. The risk of a hot module being malignant is negligible E. Should be surgically removed in all patient 61. Regarding abdominal wall hernias the false is A. Almost 100,000 hernia operations are performed annually in the United Kingdom B. Over 60% of inguinal hernias are indirect C. In women inguinal hernias are as common as femoral hernias D. The mortality assocaited with strangulation is over 10% E. The mortality has reduced dramatically over the past 30 years 62. The femoral canal the false is A. Lies medial to the femoral vein B. Has the inguinal ligament as its anterior border C. Has the lacunar ligament as its lateral border D. Has the pectineal ligament as its posterior border E. Contains the lymph node of Cloquet 63. Intermittent claudication the false is A. Affects less than 1% of men over the age of 50 years B. At 5 years 10% of claudicants will have progressed to an amputation C. At 5 years 20% of claudicants will have died from ischaemic heart disease D. Is usually associated with an ankle / brachial pressure index (ABPI) 0.3- 0.7 E. Is associated with a fall in the ABPI on exercise with delayed recovery 64. The pathology of ulcerative colitis the false is A. Shows full thickness inflammation B. The rectum is almost always involved C. 10% patients have terminal ileal disease D. Enterocutaneous or intestinal fistulae are less common E. The serosa is usually normal 65. Regarding benign breast disease the false is A. Cyclical mastalgia is the commonest reason for referral to the breast clinic B. Fibroadenomas are derived from the breast lobule C. Lactational breast abscesses are usually due to Staph aureus D. Duct ectasia is less common in smokers E. Atypical lobular hyperplasia is associated with an increased risk of breast cancer 66. Regarding anal fissures the false is A. 10% occur in the posterior midline B. Multiple fissures suggest a diagnosis of tuberculosis or Crohn's Disease C. 50% of acute fissures heal with the use of a bulking agent D. Sphincterotomy has a success rate of over 90% E. Sphincterotomy is associated with minor faecal incontinence in over 15% of patients 67. Fibroadenomas the false is A. Are benign monoclonal neoplasms B. Most commonly present in late adolescence or the early 20s C. Should be diagnosed by triple assessment D. At least 30% reduce in size over a 2 year period
342
E. Can be safely managed conservatively 68. Warfarin the false is A. Reduces the concentration of vitamin A dependent clotting factors B. Has a half life of about 36 hours C. Crosses the placenta and should be avoided in pregnancy D. Doses should be reduced in liver disease E. An INR of between 2.0 and 3.0 is appropriate in the treatment of DVT 69. Heparin the false is A. Is a heterogeneous mixture of sulphated polysaccharides B. Potentiates the actions of antithrombin III C. Has a half life of 12 hours D. Can be reversed by protamine sulphate E. Can induce an idiosyncratic thrombocytopenia 70. The following cause hypercalcaemia except A. Sarcoidosis B. Primary hyperparathyroidism C. Acute pancreatitis D. Metastatic bronchial carcinoma E. Milk-Alkali syndrome 71. Regarding oesophageal cancer the false is A. Squamous carcinomas predominate in the upper 2/3 of the oesophagus B. Overall 5 year survival is greater than 50% C. Tylosis genetically predisposes to the disease D. 15% of adenocarcinomas are associated with Barrett's oesophagus E. For palliation an Atkinson tube can be inserted endoscopically 72. Infantile hypertrophic pyloric stenosis the false is A. Occurs with a male : female ratio of 4:1. B. Sons of affected mothers have a 20% risk of developing the lesion C. Invariably presents between three and four weeks of age D. Typically presents with nonbilious vomiting E. Surgical treatment is by Heller's Cardiomyotomy 73. Oesophageal atresia the false is A. Is often associated with a distal trachea-oesophageal fistula B. Polyhydramnios is often present late in pregnancy C. 50% have other associated congenital abnormalities D. Contrast X-ray studies are necessary to confirm the diagnosis E. Post-operatively over 30% develop oesophageal strictures 74. The following are features of Fallot's Tetralogy except A. An atrial septal defect B. Pulmonary stenosis C. Right ventricular hypertrophy D. A right to left cardiac shunt E. Cyanotic attacks during feeding and crying 75. The functions of a tracheostomy are to except A. Bypass an upper airway obstruction B. Reduce the anatomical dead space C. Increase airway resistance D. Protect against aspiration E. Allow frequent airway suction 76. Medullary carcinoma of the thyroid the false is A. Is a tumour of the parafollicular C cells B. Produce thyroxine as the principle hormone C. 90% of cases are sporadic D. Can occur as part of the MEN type II syndrome
343
E. Total thyroidectomy is the surgical treatment of choice 77. The following are features of thyrotoxicosis except A. Weight gain B. Palpitations C. Proximal myopathy D. Increased skin pigmentation E. Pretibial myxoedema 78. Regarding abdominal aortic aneurysms the false is A. They commonly remain symptomless until they rupture B. The risk of rupture increases with aortic diameter C. Elective repair should have a 30-day mortality of less than 5% D. Emergency repair has a 30-day mortality of less than 10% E. The benefits of surgery for small (4.0-5.5 cm) is unproven 79. In patients with carotid artery disease A. A bruit is a reliable sign of the degree of stenosis B. Atheroma is most commonly seen in the external carotid artery C. An embolic event often results in an ipsilateral hemiplegia D. Prophylactic aspirin reduces the risk of a stroke E. Surgery is of proven benefit in those with asymptomatic stenoses 80. Hepatocellular carcinoma the false is A. Has a high incidence in East Africa and South-east Asia B. Its worldwide incidence parallels the prevalence of Hepatitis B C. Mycotoxins (e.g. aflatoxin) are an important aetiological factor D. Over 80% of tumours are surgically resectable E. Liver transplantation offers the only chance of cure in those with irresectable disease 81. Serum alpha fetoprotein the false is A. Is increased in testicular tumours B. In testicular tumours is produced by trophoblastic elements C. Is increased in over 70% patients with hepatocellular carcinoma D. In patients with hepatocellular carcinoma levels correlate well with size of the tumour E. In patients with hepatocellular carcinoma levels fall following resection of the tumour 82. Regarding testicular tumours all true except : A. They are the commonest malignancy in young men B. Peak incidence for teratomas is seen at the age of 25 years C. Seminomas are radiosensitive D. Over 75% of Stage I teratomas are cured by surgery alone E. Chemotherapy rarely produces a cure in those with metastatic disease 83. In patients with ascites the false is A. A exudate has a protein content of greater than 30 g per litre B. Both malignancy and infection result in a transudate C. Ovarian carcinoma is the commonest malignant cause of ascites D. Cirrhosis results in marked sodium retention E. Tumour cells increase the permeability of the peritoneum 84. Regarding opiate analgesia the false is A. Patient controlled analgesia (PCA) is more effective than intermittent parenteral dosing B. The total opiate dose is usually reduced with a PCA C. Fentanyl is more lipid soluble than morphine D. Epidural morphine can result in late respiratory depression E. Epidural and parenteral morphine can be safely administered together 85. Intussusception the false is A. Is most common in children from 6 to 12 years B. Presents with colicky abdominal pain, rectal bleeding and an abdominal mass C. 10% present with diarrhoea and vomiting suggestive of gastroenteritis D. If no shock or peritonitis hydrostatic reduction can be attempted
344
E. A Meckel's diverticulum can induce an intussusception 86. Hirschsprung's disease the false is A. Is due to absent ganglion cells in Auerbach's plexus B. 10% cases have involvement of the recto-sigmoid segment C. 80% cases present with delayed passage of meconium in the first 24 hours after birth D. The affected segment of bowel appears as cone-shaped contracted zone on barium enema E. On rectal biopsy there increased acetylcholinesterase containing cells in the muscularis 87. Laparoscopic cholecystectomy the false is A. Is usually performed using a four port technique B. The Veress needle is an 'open' technique for inducing the pneumoperitoneum C. A supraumbilical abdominal scar is a contraindication to laparoscopic cholecystectomy D. Dissection of Calot's triangle should be performed before the cystic duct is clipped E. Most series report a conversion rate of less than 5% 88. In obstructive jaundice the false is A. Urinary conjugated bilirubin is increased B. Serum unconjugated bilirubin is increased C. Urinary urobilinogen is reduced D. Serum conjugated bilirubin is increased E. Faecal stercobilinogen is reduced 89. Regarding the anatomy of the inguinal canal the false is A. The internal ring lies midway between the symphysis pubis and anterior superior iliac spine B. The internal ring lies medial to the inferior epigastric vessels C. The external oblique aponeurosis forms the anterior boundary D. The inguinal ligament forms the inferior boundary E. The conjoint tendon forms the medial part of the posterior wall 90. Papillary carcinoma of the thyroid the false is A. Can be reliably diagnosed using fine needle aspiration cytology B. Is almost always unifocal C. Histologically displays Psammoma bodies D. Typically spread to the cervical lymph nodes E. Requires a total thyroidectomy for large tumours 91. Regarding bladder tumours the false is A. 90% are squamous carcinomas B. Painless haematuria is the commonest presentation C. Cigarette smoking is an important aetiological factor D. 80% of tumours are superficial (i.e no muscle invasion) E. Superficial tumours are often well controlled by transurethral resection 92. Regarding ureteric calculi the false is A. Are most often composed of calcium oxalate or phosphate B. Most stones of those less than 5 mm in diameter pass spontaneously C. Extracorporeal lithotripsy is useful for stones in the upper third of the ureter D. About 30% of patients require open surgery to remove the stone E. An obstructed ureter in the presence of infection is a surgical emergency 93. Regarding bladder calculi the false is A. The incidence has fallen markedly in this country since the late 19th century B. They may be totally asymptomatic C. They are more prevalent in patients with chronic urinary sepsis D. They are associated with squamous metaplasia of the bladder mucosa E. They increase the risk of transitional cell carcinoma 94. Ectopic pregnancy the false is A. Occurs in about 1% of pregnancies B. The risk is increased in those with a history or pelvic inflammatory disease C. Usually presents between 2 and 4 months of gestation
345
D. Patients usually have a positive pregnancy test E. if shocked early laparotomy is essential 95. Tetanus the false is A. Is due to an infection with a gram-positive spore forming rod B. The organism produces a powerful endotoxin C. The toxin prevents the release of inhibitory neurotransmitter D. Clostridium tetani is sensitive to penicillin 96. Regarding pathological terms A. Hypertrophy is an increase in tissue size due to increased cell number B. Hyperplasia is an increase in tissue size due to an increase in cell size C. Atrophy is an increase in tissue size to disuse D. Metaplasia is a change form one abnormal tissue type to another E. A hamartoma is a developmental abnormality 1 2 3 4 5 6 7 8 9 10 11 12 13 14 15
E B A C D C E B C C D D C D C
16 17 18 19 20 21 22 23 24 25 26 27 28 29 30
B C D D E A B D B A B C B E D
31 32 33 34 35 36 37 38 39 40 41 42 43 44 45
B B A C D D D B C C B D A D E
46 47 48 49 50 51 52 53 54 55 56 57 58 59 60
E B C B C B A+D+E B B A B E E C E
61 62 63 64 65 66 67 68 69 70 71 72 73 74 75
E C A A D A A A C C B E D A C
76 77 78 79 80 81 82 83 84 85 86 87 88 89 90
B A E D D B E B E A B B B B B
1. The ilio-inguinal nerve: A. supplies the rectus abdominis muscle B. supplies skin on inner side of thigh C. supplies the cremasteric muscle D. supplies the urethra E. does none of the above 2. The skin of the tip of the index finger is supplied by the: A. Radial nerve only B. Radial & median nerves C. Median & ulnar nerves D. Ulnar nerve only E. Median nerve only 3. Hypovolaemic shock is characterized by: A. A low central venous pressure , high cardiac output, low peripheral resistance B. A high central venous pressure, high cardiac output, high peripheral resistance C. A low central venous pressure , low cardiac output, high peripheral resistance D. A low central venous pressure , high cardiac output, high peripheral resistance E. A high central venous pressure, low cardiac output, low peripheral resistance 4. Which of the following in NOT true of Hodgkin's disease? A. Usually starts from several groups of nodes simultaneously B. Usually involves liver & spleen C. Sometimes manifests itself as pyrexia of unknown origin D. Severe pain follows ingestion of alcohol E. Shows increased susceptibility to opportunistic infection
346
91 92 93 94 95 96
A D E C B E
5. Tetanus toxoid: A. Is produced by injecting animals with antititanic serum B. Is administered to previously immunized subjects with potentially infected wounds C. Frequently gives rise to anaphylactic reaction D. Is used to induce active immunity against tetanus 6. The most probable cause of shock in a patient with multiple injuries & craniocerebral trauma is: A. Depression of vital medullary centres B. Hypoperfuion control over subcortical areas C. Loss of cortical control over subcortical areas D. Hypovolaemia E. Inadequate ADH response 7. The most sensitive guide to acute changes in fluid balance in a surgical patient is: A. Accurate daily weight B. Serial serum Na concentration C. Fluid balance sheets recording inputs & outputs D. Daily urine output E. Serial anion gap measurements 8. cellullitis is: A. Inflammation of the bone marrow B. Inflammation of the mastoid cells C. Inflammation of the subcutaneous tissues D. Infiltration of the skin by gaint cells E. A malignant condition 9. secondary haemorrhage occurs: A. Within 6 hours of operation B. 7-14 days after operation C. As a result of violent coughing on recovery from anaesthesia D. Due to a blood transfusion line disconnected E. When a ligature slips 10. the minimum urine output for 24 hours required to excrete end products of protein metabolism is: A. 200 ml B. 300 ml C. 400 ml D. 500 ml E. 600 ml 11. Potassium deficiency is present if the plasma-potassium level is: A. 6.0 mmol/l B. 5.0 mmol/l C. 4.5 mmol/l D. 4.0 mmol/l E. 3.0 mmol/l 12. in health the pH of the blood lies between the range: A. pH 7.05-7.19 B. pH 7.20-7.35 C. pH 7.36-744 D. pH 7.45-7.59 E. pH 7.60-7.80 13. TNM classification of a malignant tumour was designed as: A. An histological staging B. A clinical staging C. A staging carried out at operation D. A staging dependent upon radio scanning & skeletal survey
347
E. A staging dependent upon ultrasound\ 14. a blue-green discharge from an ulcer will be seen to contain: A. Pseudomonas pyocyaneus B. Streptococcus viridians C. Candida albicans D. Staphylococcus aureus E. Haemophilius influenzae 15. a rodent ulcer is: A. A squamous cell carcinoma B. A basal cell carcinoma C. Only occur on the face D. Contains epithelial pearls E. A venous ulcer 16. the space of Parona is: A. In the wrist between the deep flexor tendons & the pronator quadratus B. Above the patella between the quadriceps muscle & the femur C. Benath the tendon of the iliopsos D. Between the achills tendon & the posterior aspect of the tibia E. The web space of the palm 17. 'rest pain' occurs: A. Anywhere in the body at rest B. In the thigh of a patient with Buerger's disease C. In the calf of a patient with intermittent claudicating D. In the foot of a patient with severe vascular disease E. In the back 18. ischaemia means: A. Pain in the ischial tuberosities B. Anaemia due to malignant seconderies in the ischial part of the pelvis C. Lack of blood flow D. Increased blood flow E. Polycythaemia 19. Colles' fracture is: A. A common in adolescence B. A fracture about the ankle joint C. Common in elderly women D. A fracture of the head of the radius E. A fracture of scaphoid 20. Pott's disease is; A. A fracture dislocation about the ankle B. A neuropathic joint C. Traumatic ostechondritis of the spine D. Tuberculosis of the spine E. A secondary tumour in the skull 21. Vincent's angina is a form of angina associated with: A. Spasm of the oesophagus B. Diphtheria C. An infection of the mouth D. Coronary artery spasm E. Carcinoma of the bronchus 22. Ludwig's angina is due to : A. A type of coronary artery spasm B. Oesophageal spasm C. Retropharyngeal infection D. A virulent infection of the cellular tissue around the submandibular salivary gland E. Infection with candida
348
23. in simple nodular goiter: A. Carcinoma occurs in 30% of cases B. The nodular stage is irreversible C. Operation is contraindicated D. The patient does not develop hyperthyroidism E. Cretinism is the presenting feature 24. Hashimoto's disease is: A. A granulomatous thyroiditis B. An auto-immune thyroiditis C. An infiltrating fibrosis of the thyrois & the adjacent muscles D. Focal thyroiditis E. A parathyroid tumour 25. A thyroglossal fistula: A. Is never congenital B. Follows inadequate removal of a thyroglossal cyst C. Has a hood of skin with its concavity upwards D. Is lined throughout by squamous epithelium E. Occurs in carcinoma of the tongue 26. The following are clinical signs supporting an early diagnosis of carcinoma of the breast: A. A prickling sensation in a breast lump B. Peau d'ornge C. Brawny arm D. Cancer en cuirasse E. A krukenderg tumour 27. The gastroduodenal artery is a branch of the: A. Celiac axis B. Hepatic artery C. Superior mesenteric artery D. Gastroepiploic artery E. Splenic artery 28. Chronic gastric ulcers most often occur in patients with: A. Blood groub A B. Tend to occur in alkaline mucosa C. Muscularis mucosae is separated from the muscularis at the edge of the ulcer D. Are malignant when there is epithelial proliferation & downgrowths E. Are never large 29. Meckel's diverticulum: A. Is present in 20% of the human race B. Arises from the mesenteric border of the jejunum C. May contain heterotopic pancreas D. Is only present in the male sex E. Is a diverticulum of the bladder 30. Intussusception is related to: A. Mucoviscidosis B. Swollen Peyer's patches C. Volvulus D. A littre's hernia E. A patent vitello intestinal duct 31. The site of the neck of a femoral hernia is the: A. Transversalia fascia B. Iliopectineal ligament C. Femoral ring D. Cribriform fascia E. Obturator foramen
349
32. Regarding operation for an indirect inguinal hernia: A. It should not be performed on patients who have chronic bronchitis B. General anaesthesia has to be used C. In infants the posterior inguinal wall should be repaired D. In adults the internal inguinal ring usually needs to be strengthened E. Mesh implants are mandatory 33. Cystic fibrosis of the pancreas A. is inherited as autosomal recessive B. islets of Langerhans are affected C. diagnosis is pssible by DNA analysis D. all the above E. a and c only 34. the following are clinical criteria of Bank ad Wise A. pulmonary B. neurological C. renal D. all the above E. a and c only 35. antibiotics in acute pancreatitis A. must be given to all cases B. should not be given C. given only to modarate & sever cases D. should cover anaerobic & Gam positive bactria E. none of the above 36. pancreatic carcinoma A. occure usually at the age of 50years B. usually is cystadenocarcinoma C. more favaorable in the body and tail D. less favourable if occure with back pain E. all the above 37. infected pancreatic necrosis A. is a collection of pus around the pancreas B. diagnosed by CT scan C. more serious than pancreatic abscess D. all the above E. none of the above 38. secondarey survay in polytrauma A. should be done in the first hour B. done as soon as the patient is stable C. detects life threateing problems D. a and c E. none of the above 39. spontaneous ruptur of the spleen A. occure less in tropical countries B. the spleen may be enlarged C. malaria is a common cause D. all the above 40. spleic aretry aneurysm A. incidence is around 5% B. usually symptomless C. more conmmon in males D. all the above 41. wolf skin graft A. used to cover large area of burn
350
B. success rate is less than split thckness graft C. both thighs are common donor areas D. usually contracts 42. medullary carcinoma of the thyroid A. is a tumor of the G cells B. familial in 70% of the cases C. diagosed by low level of calcitonin D. amyloid stroma is charactrestic 43. solitary thyroid nodule A. 70% are follicular adenoma B. 30% are malignant C. more favourable in young males D. all the above E. none of the above 44. rupture of the diaphragm A. is a life threatenig problem B. usually missed C. poly galctan suture is used for repair D. all the above E. noe of the above 45. Complication of vascular graft includes All the following except: A. Infection B. Aneurysm C. Graft failure D. Hemorrhage E. a & c only 46. Gangrene: A. is necrosis of tissue B. The cause may be venous occlusion C. usually painful D. All the above E. None of the above 47. Lynphedema: A. may be Congenital B. should be bilateral C. may be pitting in early stage D. A & C only E. None of the above 48. Suprapubic cystostomy: A. indicated in case of bladder out flow obstruction B. indicated in case of urethral injury C. indicated in case of urethral stricture D. All the above E. B& C only 49. differential diagnosis of Acute scrotal swelling in young adult includes all the follow except: A. Incarcerated inguinal hernia . B. Torsion of testes C. Acute epididymo- orchitis D. Teratoma E. Seminoma 50. Horizontal spread of infection across the external sphincter can result in which type of anorectal abscess: A. ischiorectal B. perianal
351
C. supralevator D. intersphincteric E. intermuscular 51. All of the following statements are true of diffuse esophageal spasm EXCEPT: A. chest pain is frequently seen B. high amplitude esophageal contraction are present. C. it is best diagnosed with barium esophogram. D. usual surgical treatment is long esohagomyotomy. E. most patients do not have significant coronary artery disease. 52. The treatment of an esophageal burn with a caustic agent may include all of the following EXCEPT: A. expeditious administration of an antidote. B. induction of vomiting. C. steroids and antibiotics. D. bougienage. E. gastrectomy.
53. All of the following substances are irritating to the peritoneum EXCEPT: A. bile. B. meconium. C. blood. D. gastric content. E. pus. 54. Complications of truncal vagotomy and pyloroplasty include all of the following EXCEPT: A. dumping syndrome. B. recurrent ulcer. C. diarrhea. D. alkaline reflux gastritis. E. steatorrhea. 55. Gastric polyps: A. are most commonly adenomatous. B. require gastrotomy and removal if greater than 2 cm and are pedunculated. C. are rarely multiple. D. are clearly premalignant. E. are more frequent in achlorhydric patients. 56. Vascular compression of the duodenum resulting in obstruction: A. is present primarily in patients who are overweight. B. should be given a trial of conservative management. C. is common in pediatric patients. D. is best diagnosed by identifying a "double bubble" sign on abdominal x-ray. E. includes as medical therapy lying in the supine position after meals 57. Conditions associated with gastric cancer include all of the following EXCEPT: A. higher socioeconomic groups. B. pernicious anemia. C. chronic atrophic gastritis. D. adenomatous polyps. E. a high intake of dietary nitrates 58. Patients with morbid obesity have an increased incidence of all of the following EXCEPT: A. gastric carcinoma. B. diabetes. C. stroke D. gallbladder disease. E. joint deterioration.
352
59. All of the following contribute to malabsorption following truncal vagotomy and antrectomy EXCEPT: A. increased rate of gastric emptying. B. poor mixing of pancreatic secretions and bile salts with food. C. increased release of secretions and bile salts with food. D. decreased small intestinal transit time. E. malabsorption of fat and carbohydrates. 60. All of the following statements are true about patients with carcinoid tumors EXCEPT: A. they often have evidence of serotonin production. B. tumor growth is often slow. C. the majority have carcinoid syndrome. D. they have a much better prognosis if the tumors are less than 2 cm. E. the combination of streptozotocin and 5-fluorouracil can often result in objective response. 61. The followings are usually associated with acute appendicitis EXCEPT A. Abdominal pain and nausea B. CT scan with water soluble enema is the most helpful imaging study C. Deep right lower abdominal tenderness by rectal examination D. Positive Rovsing sign E. WBCs around 20,000/mm3 62. The least problem that cause right lower abdominal pain in a 18 years female is A. Ectopic pregnancy B. Acute appendicitis C. Ovarian torsion D. Perforated peptic ulcer E. Mittleschmerz 63. A patient with high hichitic fever, severe tenderness and rigidity at the right ileac fossa, WBCs 18000/mm3 and abdominal ultrasound showed a heterogeneous mass in the right iliac fossa with a central fluid collection. Management may include any of the followings EXCEPT A. Open drainage B. I.V antibiotics C. Appendix should be resected in the open drainage D. Percutaneous drainage under U/S or CT guidance E. Oral feeding shouldn’t be delayed in the absence of ileus 64. Regarding the lower esophageal sphincter the following are correct EXCEPT A. It is a physiological sphincter B. Located in the distal 3-5cm of the esophagus C. Has a resting pressure of 20-60 mmHg D. Abdominal pressure play a role E. Its pressure can be estimated by esophageal manometry 65. In esophageal perforation, the most sensitive diagnostic study is A. Plain film abdomen B. Plain film chest and neck C. Esophagogram D. Esophagoscopy E. CT chest and neck 66. Achalasia can be presented with all of the followings EXCEPT A. Recurrent pulmonary infections B. Weight loss C. Regurgitation D. Irregular narrowing of the distal esophagus by Ba. Swallow E. Retrosternal chest pain 67. Complications of reflux esophagitis include the followings EXCEPT A. Dysmotility B. Schatzki's ring C. Barrett's esophagus
353
D. Epiphrenic esophageal diverticulum E. Hemorrhage 68. The pressure in the lower esophageal sphincter decreases by all of the following EXCEPT A. Alcohol B. Nitroglycerin C. Anticholinergics D. Alpha adrenergics E. Cholecystokinin hormone 69. The genetic predisposing factors to gastric cancer include the followings EXCEPT A. Family history of gastric cancer B. Black race C. P 53 mutation D. Germline mutation of e-cadherin E. BRCA2 mutatio 70. Regarding the diffuse gastric cancer, the followings are true EXCEPT A. The commonest type of gastric adenocarcinoma B. Not associated with intestinal metaplasia C. More incidence in young ages D. Less related to environmental influences E. Results from single cell mutation 71. The best diagnostic study for gastric adenocarcinoma A. Upper endoscopy B. Endoscopic ultrasound C. Upper gastrointestinal double contrast barium study D. Laparoscopy E. Abdominal CT scanning 72. In primary gastrointestinal stromal tumors (GIST), which is NOT true A. The most common site is the stomach B. Bleeding is the commonest manifestation C. Almost never metastasize to regional lymph nodes D. The traditional cytotoxic chemotherapy greatly suppresses its growth E. Endoscopic ultrasound guided FNAC gives the definitive diagnosis 73. Primary gastric lymphoma A. 2% of all hodgkin's lymphoma B. Greatly differs in presentation from gastric adenocarcinoma C. One third of all gastrointestinal lymphomas D. The most common extranodal lymphoma E. Worse prognosis than adenocarcinoma 74. In gastric adenocarcinoma which is NOT true A. Risk increased 3—6 times in patient with gastric H pylori infection B. Blumer's shelf results from omental invasion in pelvic cavity C. Krukenburg's tumor is due to ovarian metastasis D. Weight loss and vague abdominal pain are the commonest presentation E. Endoscopic ultrasound is 90% accurate in determining T stage 75. Mucosa associated lymphoid tissue ( MALT ) is found in all of the followings EXCEPT A. Small bowel ( Peyer's patches ) B. Waldeyer's ring C. Appendix D. Stomach E. Bronchus 76. All of the following are associated with Barrett's esophagus EXCEPT A. GERD B. Squamous carcinoma C. Esophageal mucosal dysplasia D. Increased incidence of p53 mutations
354
E. adenocarcinoma 77. The best test to establish the presence of gastroesophageal reflux (GERD) disease is A. An upper gastrointestinal series B. Bernstein test (acid perfusion) C. 24-hour pH monitoring D. Esophageal manometry E. Endoscopic biopsy 78. The T and N status of esophageal carcinoma is most accurately assessed by A. Upper gastrointestinal series B. Computed tomographic scan of the chest with double contrast C. Endoscopic ultrasound (EUS) D. Positron emission tomography (PET scan) E. Magnetic resonance imaging (MRI) 79. Salivary gland stones A. Most arise in the sublingual gland B. Usually present with persistent pain C. Are composed predominantly of magnesium phosphate D. Predispose to infection of the involved gland E. Never occur in parotid duct 80. 35-year-old alcoholic is admitted with acute pancreatitis. He complains of Numbness of his fingers and toes. On examination he has hyperactive Tendon reflexes. The most likely cause of these symptoms is A. Hyponatremia B. Hypocalcemia C. Hypophosphatemia D. Hypermagnesemia E. Hyperkalemia 81. Regarding benign salivary gland adenomas A. Pleomorphic adenomas are only seen in parotid gland B. Pleomorphic adenomas cannot undergo malignant change C. Warthin's tumor is otherwise known as an adenolymphoma D. Adenolymphomas usually occur in young men E. Adenolymphomas are often bilateral 82. Massive lower gastrointestinal hemorrhage A. Rarely stops after resuscitation B. Is caused by large bowel lesions only C. can be most accurately localized by colonoscopy D. Is frequently related to right colon lesions E. Is most commonly caused by adenocarcinoma of the large intestine 83. Regarding carcinoid tumors all of the following are true EXCEPT A. Liver metastases can result in the carcinoid syndrome B. The appendix is the commonest primary site for gastrointestinal tumors C. Gastric carcinoid tumors produce little 5-hydroxyindoleacetic acid D. If discovered in the appendix right hemicolectomy should always be done E. Octreotide scintigraphy may identify both the primary and secondary lesions 84. A Meckel's diverticulum A. Occurs in 10% of the population B. Will be found on the mesenteric border of the small intestine C. Consists of mucosa without a muscle coat D. Usually found 20 cm from ileocecal valve E. A fibrous band between the apex and umbilicus can cause intestinal obstruction 85. In overwhelming post-splenectomy infection, which is NOT true A. Is usually due to unencapsulated bacterial infection B. Strep. pneumonia is the commonest etiological agent C. Despite aggressive therapy it can have a mortality of over 50%
355
D. The risk of infection can be reduced with pneumococcal and hemophilus vaccination E. Penicillin antibiotic prophylaxis should be considered in all children 86. Clostridium tetani A. Is a gram-negative rod B. Is sensitive to penicillin C. Is available in the expired canned food D. Releases a heat-resistant endotoxin E. The toxin acts on the post-synaptic membrane of inhibitory nerve fibers 87. Grade I hypovolemic shock A. Occurs when more than 40% of the circulating blood volume has been lost B. Tachycardia is a reliable clinical sign C. A fall in pulse pressure is observed D. Urine output is markedly reduced E. Capillary return is delayed
88. Regarding Hydatid disease the followings are true EXCEPT A. Recently surgery is no more the treatment of choice in liver disease B. Man is an accidental intermediate host C. The liver is the commonest site of infection D. Can be diagnosed by the ELISA test E. Treatment by benzimidazoles alone is 30% successful 89. In peptic ulcer disease A. H. pylori is a gram-positive bacillus B. Surgery is always indicated in perforated ulcers C. Sepsis after perforation is the commonest cause of death D. Parietal cell vagotomy carries the highest rate of recurrence E. Gastric drainage should be done in all types of vagotomies 90. Stones in the common bile duct, which is NOT true A. Are mostly secondary stones B. Can present with Charcot's Triad C. Are suggested by a bile duct diameter >8mm on ultrasound D. ERCP, sphincterotomy and balloon clearance is now the treatment of choice E. If removed by exploration of the common bile duct the T-tube can be removed after 3 days 91. Regarding pancreatic carcinoma the followings are true EXCEPT A. 90% are ductal adenocarcinomas B. Less than 20% occur in the head of the gland C. The usual presentation is with pain, weight loss and obstructive jaundice D. Ultrasound has a sensitivity of 80-90% in the detection of the tumor E. Less than 20% of patients are suitable for curative surgery 92. Regarding anal fissures all are true EXCEPT A. 10% occur in the posterior midline B. Multiple fissures suggest a diagnosis of tuberculosis or Crohn's Disease C. 50% of acute fissures heal with the use of a bulking agent D. Sphincterotomy has a success rate of over 90% E. Sphincterotomy is associated with minor fecal incontinence in over 15% of patients 93. Medullary carcinoma of the thyroid A. Concentrates Iodine 131 B. Produce thyroxine as the principle hormone C. 10% of cases are sporadic D. 90% can occur in association with MEN type II syndrome E. All patients will need a 24-hour VMA level in urine
356
94. Which of the following is LEAST likely to be associated with the systemic inflammatory response syndrome (SIRS) A. Infection B. Elevated/depressed temperature C. Elevated heart rate D. Elevated respiratory rate E. Elevated/depressed WBC count 95. In papillary carcinoma of the thyroid the followings are true EXCEPT A. Can be reliably diagnosed using fine needle aspiration cytology B. Is almost always unifocal C. Histologically displays Psammoma bodies D. Typically spread to the cervical lymph nodes E. Requires a total thyroidectomy for large tumors 96. Serum alpha-fetoprotein is increased in the following EXCEPT A. Acute hepatitis B. Hepatocellular carcinoma C. Neuroblastoma D. Teratomas E. Bladder carcinoma 97. The following predispose to wound infection EXCEPT A. Malnutrition B. Hypovolemia C. Malignancy D. Obstructive jaundice E. Steroid therapy 98. Regarding acute respiratory distress syndrome (ARDS) which is NOT true A. Hypoxia in spite of high inspired oxygen B. Increased lung compliance C. Non-cardiac edema D. Diffuse or patchy infiltrates in chest X ray E. Deposition of proteinaceous fluid in the respiratory membrane 99. Which of the following statements about serum thyroglobulin is TRUE? A. Elevation after total thyroidectomy justify the use of iodine131 therapy B. It suppresses thyroid-stimulating hormone (TSH) C. It suppresses thyroid-releasing hormone (TRH) D. It is an effective tumor marker because it is specifically elaborated by the malignant cells of papillary cancer E. It is only about 50% effective in detecting recurrence 100. Women who have hereditary nonpolyposis colorectal cancer (HNPCC) should also be screened for A. Endometrial cancer B. Papillary cancer of thyroid C. Ampullary cancer D. Pheochromocytoma E. Hepatoma 1. Which is the shortest phase of the normal cell cycle? A. G1 phase B. S phase C. G2 phase D. M phase E. All phases approximately equal in length 2. A 67 year-old women with rectal cancer is admitted to gereral surgical floor which of the following laboratory studies should be included in the surgeon’s initial nutritional assessment A. Transferrin B. Prealbumin
357
C. Albumin D. Glutamine E. All of above 3. In which of the following conditions is the entral route appropriate for nutrition : A. Upper gastrointestinal obstruction B. Complete small bowel obstruction C. Acute flare-up of Crohn’s disease D. Low out put colonic fistula E. Non of the above 4. Which is the most commonly cultured hospital acquired organism in critical care with aspiration pneumonia: A. Staphylococcus aureus B. Streptococcus pneumonia C. Anaerobic species D. Pseudomonas aeroginosa E. Haemophlus influenzae
5. Which is the most appropriate single agent for empiric coverage of the above patient : A. Metranidazole B. Clindamycin C. Pipracillin_tazobactam D. Vancomycin E. First generation penicillin 6. All of the following are true about neurogenic shock except: A. There is a decrease in systemic vascular resistance and an increase in venous capacitance. B. Tachycardia or bradycardia may be observed, along with hypotension. C. The use of an alpha agonist such as phenylephrine is the mainstay of treatment. D. Severe head injury, spinal cord injury, and high spinal anesthesia may all cause neurogenic shock. E. A and B 7. Which of the following statements about head injury and concomitant hyponatremia are true? A. There are no primary alterations in cardiovascular signs. B. Signs of increased intracranial pressure may be masked by the hyponatremia. C. Oliguric renal failure is an unlikely complication. D. Rapid correction of the hyponatremia may prevent central pontine injury. E. This patient is best treated by restriction of water intake 8. Regarding Cushing's Syndrome A. 20% of cases are due to pituitary adenomas (Cushing's Disease) B. Most ACTH secreting pituitary adenomas are more than 2 cm in diameter C. Is characterised by loss of the diurnal rhythm of cortisol secretion D. Cortisol production is suppressed by low-dose dexamethasone E. Adrenal carcinomas are more common than adrenal adenomas 9. Which of the following statements about extracellular fluid are true? A. The total extracellular fluid volume represents 40% of the body weight. B. The plasma volume constitutes one fourth of the total extracellular fluid volume. C. Potassium is the principal cation in extracellular fluid. D. The protein content of the plasma produces a lower concentration of cations than in the interstitial fluid. E. The interstitial fluid equilibrates slowly with the other body compartments. 10. In patients receiving massive blood transfusion for acute blood loss, which of the following is/are correct?
358
A. Packed red blood cells and crystalloid solution should be infused to restore oxygen-carrying capacity and intravascular volume. B. Two units of FFP should be given with every 5 units of packed red blood cells in most cases. C. A “six pack” of platelets should be administered with every 10 units of packed red blood cells in most cases. D. One to two ampules of sodium bicarbonate should be administered with every 5 units of packed red blood cells to avoid acidosis. E. One ampule of calcium chloride should be administered with every 5 units of packed red blood cells to avoid hypocalcemia. 11. Which of the following statements about the presence of gallstones in diabetes patients is/are correct? A. Gallstones occur with the same frequency in diabetes patients as in the healthy population. B. The presence of gallstones, regardless of the presence of symptoms, is an indication for cholecystectomy in a diabetes patient. C. Diabetes patients with gallstones and chronic biliary pain should be managed nonoperatively with chemical dissolution and/or lithotripsy because of severe complicating medical conditions and a high operative risk. D. The presence of diabetes and gallstones places the patient at high risk for pancreatic cancer. E. Diabetes patients with symptomatic gallstones should have prompt elective cholecystectomy, to avoid the complications of acute cholecystitis and gallbladder necrosis. 12. concerning Tetanus all true except: A. Is due to an infection with a gram-negative spore forming rod B. The organism produces a powerful exdotoxin C. The toxin prevents the release of inhibitory neurotransmitter D. Clostridium tetani is sensitive to penicillin E. Risus sardonicus is the typical facial spasm 13. when should parentral antibiotics be given perioperatively? A. The night before B. 6 hr prior to surgery C. 30 minutes prior to incision . D. at the time of incision E. 30 minutes after incision 14. Which of the following statements about esophageal anatomy is correct? A. The esophagus has a poor blood supply, which is segmental in distribution and accounts for the high incidence of anastomotic leakage. B. The esophageal serosa consists of a thin layer of fibroareolar tissue. C. The esophagus has two distinct muscle layers, an outer, longitudinal one and an inner, circular one, which are striated in the upper third and smooth in the distal two thirds. D. Injury to the recurrent laryngeal nerve results in vocal cord dysfunction but does not affect swallowing. E. The lymphatic drainage of the esophagus is relatively sparse, localized primarily to adjacent paraesophageal lymph nodes. 15. Wich of the following medication should be given in preparation of a pation with pheochromocytoma? A. Phnoxybenzamine B. Nifedipine C. Linsinopril D. Hydrochlorothiazide E. Propranolol 16. Which of the following statement(s) is true concerning excessive scarring processes? A. Keloids occur randomly regardless of gender or race B. Hypertrophic scars and keloid are histologically different C. Keloids tend to develop early and hypertrophic scars late after the surgical injury D. Simple reexcision and closure of a hypertrophic scar can be useful in certain situations such as a wound closed by secondary intention
359
E. Non of the above 17. A 22-year-old man sustains a single stab wound to the left chest and presents to the emergency room with hypotension. Which of the following statement(s) is true concerning his diagnosis and management? A. The patient likely is suffering from hypovolemic shock and should respond quickly to fluid resuscitation B. Beck’s triad will likely be an obvious indication of compressive cardiogenic shock due to pericardial tamponade C. Echocardiography is the most sensitive noninvasive approach for diagnosis of pericardial tamponade D. The placement of bilateral chest tubes will likely resolve the problem 18. Which of the following statement(s) is/are true concerning septic shock? A. The clinical picture of gram negative septic shock is specifically different than shock associated with other infectious agents B. The circulatory derangements of septic shock precede the development of metabolic abnormalities C. Splanchnic vascular resistance falls in similar fashion to overall systemic vascular resistance D. Despite normal mechanisms of intrinsic expansion of the circulating blood volume, exogenous volume resuscitation is necessary 19. During surgery on the submandibular gland A. An incision on the lower border of the mandible is safe B. The submandibular gland is seen to wrap around the posterior border of mylohyoid C. The facial artery and vein are divided as they course through the deep part of the gland D. The hypoglossal nerve is seen to loop under the submandibular duct E. Damage to the lingual nerve will cause loss of sensation to the posterior third of the tongue 20. Regarding benign breast disease A. Cyclical mastalgia is the commonest reason for referral to the breast clinic B. Fibroadenomas are derived from the breast lobule C. Lactational breast abscesses are usually due to Staph aureus D. Duct ectasia is more common in smokers E. Atypical lobular hyperplasia is associated with an decreased risk of breast cancer 21. Which of the following statements regarding unusual hernias is incorrect? A. An obturator hernia may produce nerve compression diagnosed by a positive HowshipRomberg sign. B. Grynfeltt's hernia appears through the superior lumbar triangle, whereas Petit's hernia occurs through the inferior lumbar triangle. C. Sciatic hernias usually present with a painful groin mass below the inguinal ligament. D. Littre's hernia is defined by a Meckel's diverticulum presenting as the sole component of the hernia sac. E. Richter's hernia involves the antimesenteric surface of the intestine within the hernia sac and may present with partial intestinal obstruction 22. Staples may safely be placed during laparoscopic hernia repair in each of the following structures except: A. Cooper's ligament. B. Tissues superior to the lateral iliopubic tract. C. The transversus abdominis aponeurotic arch. D. Tissues inferior to the lateral iliopubic tract. E. The iliopubic tract at its insertion onto Cooper's ligament. 23. The following statements about the repair of inguinal hernias are true except: A. The conjoined tendon is sutured to Cooper's ligament in the Bassini hernia repair. B. The McVay repair is a suitable option for the repair of femoral hernias. C. The Shouldice repair involves a multilayer, imbricated repair of the floor of the inguinal canal.
360
D. The Lichtenstein repair is accomplished by prosthetic mesh repair of the inguinal canal floor in a tension-free manner. E. The laparoscopic transabdominal preperitoneal (TAPP) and totally extraperitoneal approach (TEPA) repairs are based on the preperitoneal repairs of Cheattle, Henry, Nyhus, and Stoppa. 24. A number of special circumstances exist in the repair of inguinal hernias. The following statement(s) is correct. A. Simultaneous repair of bilateral direct inguinal hernias can be performed with no significant increased risk of recurrence B. The preperitoneal approach may be appropriate for repair of a multiple recurrent hernia C. A femoral hernia repair can best be accomplished using a Bassini or Shouldice repair D. Management of an incarcerated inguinal hernia with obstruction is best approached via laparotomy incision E. All are correct. 5. Which of the following statement(s) is true about benign lesions of the liver? A. Adenomas are true neoplasms with a predisposition for complications and should usually be resected. B. Focal nodular hyperplasia (FNH) is a neoplasm related to birth control pills (BCPs) and usually requires resection. C. Hemangiomas are the most common benign lesions of the liver that come to the surgeon's attention. D. Nodular regenerative hyperplasia does not usually accompany cirrhosi 26. Ligation of all of the following arteries usually causes significant hepatic enzyme abnormalities except: A. Ligation of the right hepatic artery. B. Ligation of the left hepatic artery. C. Ligation of the hepatic artery distal to the gastro-duodenal branch. D. Ligation of the hepatic artery proximal to the gastroduodenal artery. 27. Which of the following is the most effective definitive therapy for both prevention of recurrent variceal hemorrhage and control of ascites? A. Endoscopic sclerotherapy. B. Distal splenorenal shunt. C. Esophagogastric devascularization (Sugiura procedure). D. Side-to-side portacaval shunt. E. End-to-side portacaval shunt. 28. which of the following is associated with best prognosis for patient with breast cancer? A. Male sex B. Estrogen receptor positive C. Patient age <35 years D. Pregnant patient E. Tumor with overexpression of HER/ner. 29. A 49-year-old women has a palpable breast mass in the upper outer quadrant. The size of the mass has increased over the last month . exicisional biopsy reveals cystic carcinoma with invasion .appropriate management now would be : A. Re-excision with wide margins B. Axillary node dissection and hormonal therapy C. Simple mastectomy D. Modified radical mastectomy E. Bilateral mastectomies 30. A contraindication to stereotactic core biopsy of the breast is the mammographic presence of: A. Microcacification B. A radial scar C. A nonpalpable mass lesion D. Lesions<8 mm in diameter E. Mutifocal lesions.
361
31. Which of the following statements about epiphrenic diverticula of the esophagus is/are correct? A. They are traction diverticula that arise close to the tracheobronchial tree. B. They characteristically arise proximal to an esophageal reflux stricture. C. The degree of dysphagia correlates with the size of the pouch. D. They are best approached surgically through a right thoracotomy. E. The operation of choice is a stapled diverticulectomy, long esophagomyotomy, and partial fundoplication. 32. Which of the following statements about Schatzki's ring is correct? A. The ring represents a panmural fibrotic stricture resulting from gastroesophageal reflux. B. Dysphagia occurs when the ring diameter is 13 mm. or less. C. The ring occurs within 1 to 2 cm. of the squamocolumnar epithelial junction. D. Schatzki's ring indicates reflux esophagitis. E. Schatzki's ring signifies the need for an antireflux operation. 33. Which of the following statements about pathology encountered at esophagoscopy is/are correct? A. Reflux esophagitis should be graded as mild, moderate, or severe, to promote consistency among different observers. B. An esophageal reflux stricture with a 2-mm. lumen is not dilatable and is best treated with resection. C. A newly diagnosed radiographic distal esophageal stricture warrants dilation and antireflux medical therapy. D. In patients with Barrett's mucosa, the squamocolumnar epithelial junction occurs 3 cm. or more proximal to the anatomic esophagogastric junction. E. After fasting at least 12 hours, a patient with megaesophagus of achalasia can safely undergo flexible fiberoptic esophagoscopy. 34. Infantile hypertrophic pyloric stenosis A. Occurs with a female : male ratio of 4:1. B. Sons of affected mothers have a 20% risk of developing the lesion C. Invariably presents between six and eights months of age D. Typically presents with bile stained projectile vomiting E. Surgical treatment is by Heller's Cardiomyotomy 35. Which of the following statements regarding the pathology of esophageal carcinoma is/are correct? A. Worldwide, adenocarcinoma is the most common esophageal malignancy. B. Squamous cell carcinoma is most common in the distal esophagus, whereas adenocarcinoma predominates in the middle third. C. Patients with Barrett's metaplasia are 40 times more likely than the general population to develop adenocarcinoma. D. Metastases from esophageal carcinoma are characteristically localized to regional mediastinal lymph nodes adjacent to the tumor. E. Achalasia, radiation esophagitis, caustic esophageal stricture, Barrett's mucosa, and Plummer-Vinson syndrome are all premalignant esophageal lesions that predispose to the development of squamous cell carcinoma. 36. 45-year-old with isolated 6-cm colorectal metastasis in the liver 2 years after colectomy, otherwise healthy pest treatment would be: A. Radiofrequency ablation B. Systemic chemotherapy C. Hepatic lobectomy D. Liver transplantation E. Cryosurgical ablation 37. Oesophageal atresia : A. Is often associated with a distal trachea-oesophageal fistula B. Polyhydramnios is often present late in pregnancy C. 50% have other associated congenital abnormalities
362
D. Contrast X-ray studies are necessary to confirm the diagnosis E. Post-operatively over 30% develop oesophageal strictures 38. All are true about the dumping syndrome except: A. Symptoms can be controlled with a somatostatin analog. B. Diarrhea is always part of the dumping syndrome. C. Flushing and tachycardia are common features of the syndrome. D. Separating solids and liquids in the patient's oral intake alleviates some of the symptoms of the syndrome. E. Early postoperative dumping after vagotomy often resolves spontaneously. 39. Which of the following statements about gastric polyps is/are true? A. Like their colonic counterparts, gastric epithelial polyps are common tumors. B. They are analogous to colorectal polyps in natural history. C. Endoscopy can uniformly predict the histology of a polyp based on location and appearance. D. In a given patient, multiple polyps are generally of a multiple histologic type. E. Gastric adenomatous polyps greater than 2 cm. in diameter should be excised because of the risk of malignant transformation.
40. All of the following statements about surgical management of gastric lymphomas are true except: A. Stage I gastric lymphomas (small lesions confined to the stomach wall) can be cured completely with surgical therapy alone. B. Extensive gastric lymphomas that initially are treated with radiation and/or chemotherapy occasionally perforate during treatment and require secondary resection. C. Patients explored with a presumptive diagnosis of gastric lymphoma should undergo an attempt at curative resection when this is safe and feasible. D. Without a preoperative diagnosis resection for gastric mass should not be attempted unless lymphoma can be excluded. E. Appropriate staging for primary gastric lymphoma includes bone marrow biopsy. 41. the most accurate test to confirm diagnosis of infected necrotizing pancreases is: A. Abdominal ultrasound study B. Indium-labeled leeukocte scan C. Cimputed tomographic scan D. Elevated serum level of interleukain 6 and 8 E. Percutaneous needle aspiration 42. Which of the following variables best predicts prognosis for patients with a recent diagnosis of cutaneous melanoma and no clinical evidence of metastatic disease? A. Breslow thickness. B. Clark's level. C. Ulceration. D. Gender. E. Celtic complexion. 43. the following are true about intracranial tumors except: A. The most common location of brain tumors of childhood is the posterior cranial fossa. B. With few exceptions, examination of the CSF is of no value in the diagnosis of an intracranial tumor. C. Even the most malignant of primary brain tumors seldom spread outside the confines of the central nervous system (CNS). D. The majority of astrocytomas can be cured surgically. E. Primary neoplasms of astrocytic, oligodendroglial, or ependymal origin represent gradations of a spectrum from slowly growing to rapidly growing neoplasms.
363
44. A right-sided disc herniation at the L5–S1 level typically may cause: A. Low back pain and left sciatica. B. Weakness of dorsiflexion of the right foot. C. A diminished or absent right ankle jerk. D. Diminution of sensation over the medial aspect of the right foot, including the great toe. E. Weakness of dorsiflexion of the left foot. 45. The preferred operation for initial management of a thyroid nodule that is considered suspicious for malignancy by FNAB is: A. Excision. B. Partial lobectomy. C. Total lobectomy and isthmusectomy. D. Total t hyroidectomy. E. All methods are correct 46. the most common presentation of Meckel,s diverticulum in an adult is: A. GIT bleeding B. GIT obstruction C. Intussuception D. Litter,s hernia E. Diverticuliti 47. Optimal front-line treatment of squamous cell carcinoma of the rectum includes: A. Abdominal perineal resection. B. Low anterior resection when technically feasible. C. Radiation therapy. D. Chemotherapy. E. Combined radiation and chemotherapy. 48. 65-year-old man presents with complaints of mucous discharge and perianal discomfort. Physical examination reveals a fistulous opening lateral to the anus. Anoscopic examination permits passage of a probe through the fistula tract. The fistula traverses the internal anal sphincter, the intersphincteric plane, and a portion of the external anal sphincter. The fistula is categorized as which type? A. Intersphincteric B. Transsphincteric C. Suprasphincteric D. Extrasphincteric E. Non of the above 49. Warthin's tumor: A. Is a pleomorphic adenoma of salivary gland B. Should be treated by total paritidectomy C. Is considered a benign salivary gland neoplasia D. Respond well to preoperative radiotherapy E. Often present with facial nerve compression 50. A 38 year old woman presents with right upper quadrant pain and bouts of vomiting. She is known to have gallstones and has had similar episodes in the past. Which of the following might support a diagnosis of acute cholecystitis rather than biliary colic A. duration of symptoms B. Severity of vomiting C. Presence of Murphy's sign D. Presence of gas under right hemidiaphragm on erect CXR. 51. Adequate urine out put for adult postoperative surgical patient is greater than: A. 35 ml per hour regardless of body size B. 50 ml per hour regardless of body size C. 0.5 ml per kg. per hour D. 1 ml per kg. per hour E. 1.5 ml per kg. per hour
364
52. the MOST important finding in the diagnosis of acute appendicitis is: A. vomiting B. Fever C. leukocytosis D. right lower quadrant tenderness E. referred rebound tenderness[ Rovsing sign ] 53. the most common site for intestinal obstruction due to cholecystoenteric fistula is the A. pylorus B. duodenum C. jejunum D. ileum E. sigmoid colon. 54. Common characteristics of small bowel obstruction include all of the following EXCEPT: A. ascites B. frequent progression to strangulation C. failure to pass flatus D. distention E. vomiting 55. Gastroesophageal reflux is BEST diagnosed with: A. radiography B. 24-hour pH monitoring of lower esophagus C. esophagoscopy D. documentation of a decrease in esophageal pH after HCI is placed in the stomach . E. acid-clearing swallowing test . 56. The risk of esophageal cancer increases with all of the following EXCEPT: A. alcohol ingestion. B. smoking . C. chronic ingestion of hot beverages. D. aflatoxin.>>(HCC) E. poor oral hygiene. 57. Achalasia is associated with all of the following EXCEPT: A. chagas ’ disease in South America B. dysphasia. C. weight loss. D. relaxation of the lower esophageal sphincter with swallowing . E. aspiration pneumonia, which may cause lung abscesses. 58. Nonsurgical causes of abdominal pain include all of the following EXCEPT: A. pneumonia. B. diabetic ketoacidosis. C. acute salpingitis. D. head trauma. E. myocardial infarction. 59. Gastric acid secretion is stimulated by all of the following EXCEPT: A. sight of food. B. presence of food in the stomach. C. fat in the duodenum. D. gastrin. E. histamine. 60. All the following statementare true of anal fissure EXCEPT: A. is often associated with constipation B. pain during and after defication is typical C. the fissure is usually anterior D. the lesion is commonly seen with crohns disease
365
E. division of the lower half of the internal sphincter is usually curative 61. Factors that are associated with the development of acute pancreatitis include all of the following EXCEPT: A. alcohol. B. gallstones. C. celiac sprue. D. hyperlipidemia. E. pancreatic divisum. 62. Complications of untreated pancreatic pseudocysts include all of the following EXCEPT A. gastrointestinal obstruction. B. pancreatic necrosis. C. free rupture. D. abscess. E. intracystic hemorrhage. 63. Common presenting conditions in patients with pancreatic carcinoma include all of the following EXCEPT: A. esophageal varices. B. jaundice. C. weight loss. D. palpable gallbladder. E. abdominal pain. 64. All of the following are associated with an increased risk of breast cancer EXCEPT: A. dietary consumption of fat. B. history of breast cancer in first-degree maternal relatives C. age over 35. D. early first pregnancy. E. infertility 65. The MOST frequent histologic type of breast carcinoma is: A. infiltrating papillary carcinoma. B. infiltrating ductal carcinoma. C. infiltrating lobular infiltrating. D. colloid infiltrating. E. medullary infiltrating. 66. The MOST frequent variety of Thyroid cancer is: A. follicular carcinoma. B. papillary carcinoma. C. anaplastic carcinoma. D. hashimoto’s associated lymphoma. 67. Acute mastitis MOST commonly occurs at or during: A. birth. B. puberty. C. pregnancy. D. lactation. E. blunt trauma to the breast. 68. All of the following statements concerning nipple discharges are true EXCEPT: A. they may be caused by multiple lesions. B. when bloody , the discharge is due to a malignancy 70% of the time. C. a milky discharge may be due to a pituitary adenoma . D. benign duct papillomas are the most common cause of bloody discharges. E. excision of involved duct may be necessary to determine the etiology 69. The MOST frequent site for breast cancer to develop is the : A. upper inner quadrant. B. lower inner quadrant. C. lower outer quadrant. D. upper outer quadrant.
366
E. subareolar zone. 70. The risk of bilateral breast cancer is HIGHEST if the first breast shows: A. inflammatory carcinoma. B. lobular carcinoma. C. medullary carcinoma. D. infiltrating ductal carcinoma. E. paget’s disease. 71. All of the following are true statements concerning paget’s disease of the nipple EXCEPT. A. it is very uncommon, accounting for only 2% of all breast cancers. B. it is an in situ squamous cell malignancy of the nipple. C. it is an eczematoid lesion. D. it has a better prognosis than the majority of other breast cancers. E. it can be confused with malignant melanoma histologically. 72. Initial fluid resuscitation of a patient with multiple fractures and hypovolemic shock should be : A. blood transfusion B. hypertonic saline C. fresh frozen plasma D. ringer ’s lactate E. albumin 73. Which of the following findings suggests that shock in an injured patient may have a cause other than hypovolemia: A. hypotension/ B. distended neck veins. C. decreased skin temperature. D. diminished pulse pressure. E. falling central venous pressure. 74. All of the following are physical signs of both massive hemothorax and tension pneumothorax EXCEPT: A. tracheal shift. B. decreased breath sounds. C. tachycardia. D. hypotension. E. distended neck veins. 75. What is the MOST common infecting organism in overwhelming postsplenectomy infection: A. Escherichia coli. B. meningococcus. C. streptococcus. D. pneumococcus. E. staphlococcus. 76. Decreased Paco2 levels should be attained in a patient at serious risk for cerebral edema secondary to a head injury in order to :A. prevent neurogenic pulmonary edema B. allow reciprocally high levels of PaO2 in the brain C. prevent increased capillary permeability D. prevent cerebral vasodilation E. prevent metabolic acidosis 77. Brain injury alone A. frequently causes shock B. causes shock that is reversed by very simple measures C. causes shock only if the skull is intact D. rarely causes shock E. causes shock if hypoxia is superimposed
367
78. The level of consciousness for a head injury patient is BEST evaluated by : A. Glasgow coma scale B. response to pain C. CT scan D. papillary responses E. visual evoked potentials 79. All of the following are signs of acute vascular compromise of an extremity EXCEPT: A. diminished sensation B. pallor C. absent pulses D. gangrene E. pain
80. Which of the following should NOT be used to control the pain caused by fractured ribs: A. morphine IV. B. Demerol IM. C. intercostals nerve blocks. D. rib belts. E. muscle relaxants. 81. Postthrombotic varicose veins are due to: A. incompetent communicating veins B. destruction of deep veins. C. destruction of superficial veins. D. iliofemoral incompetence. E. block of the long saphenous vein. 82. Deep venous thrombosis prophylaxis is appropriate for all of the following patients EXCEPT: A. 67-year-old male undergoing a colectomy. B. 21-year-old male undergoing an outpatient open inguinal hernia repair. C. 21-year-old male in the ICU, comatose, with a closed head injury. D. 60-year-old female undergoing open reduction and internal fixation (ORIF) of a hip fracture. C. 60-year-old female undergoing reduction of a lung carcinoma. 83. Causes of delayed union of fractures includes all the following EXCPET: A. Compound fracture B. Infection C. Adequate immobilization D. Poor blood supply 84. Indication for skull X-Ray in head injury are: A. Loss of consciousness B. Focal neurological symptoms C. Difficulty in assessing the patient D. A& B only E. All the above 85. In Head injury C.T. scan is indicated in the following A. Aphasia after the injury B. Deterioration of level of consciousness C. Skull fracture with persistent headache D. A&B only
368
E. All the above 86. Most important steps in management of head injury include: A. Prevent hypoxia B. Prevent Dehydration C. Assure Brain Metabolism D. Prevent secondary brain injury E. All the above 87. Most common cause of solitary thyroid nodule is: A. Malignancy B. Toxic goiter C. Thyroid Adenoma D. Thyroiditis E. Hashimoto’s disease 88. Follicular carcinoma of thyroid gland: A. Is the commonest tumor B. Can be diagnosed by F.N.A.B C. Usually multifocal D. Commonest tumor of young age E. Prognosis is worse than papillary carcinoma 89. Abnormal nipple discharge could be due to: A. Duct papilloma B. Duct ectasia C. Retention cyst of montgomery gland D. A&B only E. All the above 90. Retracted nipple can be caused by the following EXCPET: A. Carcinoma of Breast B. Duct ectasia C. Chronic periductal mastitis D. A&B only E. All the above 91. The commonest type of the breast cyst is: A. Solitary mamary cyst B. Galactocele C. Lymphatic cyst D. Simple multiple cysts. 92. Carcinoma of breast: A. Is the third common malignant tumor of women B. common in Ages between 45-55 years. C. 10% of case are males. D. usually tender rapidly enlarging E. medullay carcinoma is the comment type . 93. Carcinoma of the breast A. may be familial B. infiltrating duct carcinoma is the commonest C. treated by lumpectomy and axillary clearance D. A&B only E. None of the above 94. Fracture of the sternum A. Is the comments fracture of thoracic cage. B. Usually painful. C. can’nt be associated with myocardial damage D. usually treaded by internal fixation . E. B&C only.
369
95. In massive haemothorax. A. about 500ml of blood in pleural cavity B. cause dyspnea & Neck veins distension C. treated by thoracotomy tube only D. All of the above E. None of the above 96. Tension pneumothorax A. is the commonest type of chest injuries B. Needs urgent X-Ray chest C. Is a clinical Diagnosis D. Causes flat neek viens E. Treated by thoracotomy tube after chest X-ray. 97. Varicose view. A. Primary type usually affect one limb , below the knee B. Short saphenous system is more affected C. May be caused by D.VT. D. A&B only E. None of the above. 98. Anal Fissure: A. Usually anterior B. May be caused by previous anal surgery C. Can cause dark bleeding PR. D. Sometimes is painful E. Treated by steroids 99. The comments type of Anorectal abscess is: A. Ischio rectal B. Perianal C. Submucons D. Pelvirectal 100. Varicose ulcer is: A. Usually very painful & shallow B. Deep with much necrotic tissue C. More common with primary varicose vein D. A & C only E. None of the above 1 2 3 4 5 6 7 8 9 10 11 12 13 14 15 16
370
D C D D C C A C B A E A C C A B
21 22 23 24 25 26 27 28 29 30 31 32 33 34 35 36
C D A B A D D B C B E B D B C C
41 42 43 44 45 46 47 48 49 50 51 52 53 54 55 56
E A E C C A E B C C C D D A B D
61 62 63 64 65 66 67 68 69 70 71 72 73 74 75 76
C B A D B B D B D B B D B E D D
81 82 83 84 85 86 87 88 89 90 91 92 93 94 95 96
A B C E E E C E D E D B D B E C
17 18 19 20
C D B E
37 38 39 40
B B E D
57 58 59 60
D D C C
77 78 79 80
D A D D
97 98 99 100
C B B E
31-. Which of the following statements about epiphrenic diverticula of the esophagus is/are correct? k) They are traction diverticula that arise close to the tracheobronchial tree. l) They characteristically arise proximal to an esophageal reflux stricture. m) The degree of dysphagia correlates with the size of the pouch. n) They are best approached surgically through a right thoracotomy. o) The operation of choice is a stapled diverticulectomy, long esophagomyotomy, and partial fundoplication.
32- Which of the following statements about Schatzki's ring is correct? k) The ring represents a panmural fibrotic stricture resulting from gastroesophageal reflux. l) Dysphagia occurs when the ring diameter is 13 mm. or less. m) The ring occurs within 1 to 2 cm. of the squamocolumnar epithelial junction. n) Schatzki's ring indicates reflux esophagitis. o) Schatzki's ring signifies the need for an antireflux operation. 33. Which of the following statements about pathology encountered at esophagoscopy is/are correct? k) Reflux esophagitis should be graded as mild, moderate, or severe, to promote consistency among different observers. l) An esophageal reflux stricture with a 2-mm. lumen is not dilatable and is best treated with resection. m) A newly diagnosed radiographic distal esophageal stricture warrants dilation and antireflux medical therapy. n) In patients with Barrett's mucosa, the squamocolumnar epithelial junction occurs 3 cm. or more proximal to the anatomic esophagogastric junction. o) After fasting at least 12 hours, a patient with megaesophagus of achalasia can safely undergo flexible fiberoptic esophagoscopy. 34. Infantile hypertrophic pyloric stenosis k) Occurs with a female : male ratio of 4:1. l)
Sons of affected mothers have a 20% risk of developing the lesion
m) Invariably presents between six and eights months of age n) Typically presents with bile stained projectile vomiting o) Surgical treatment is by Heller's Cardiomyotomy 35-. Which of the following statements regarding the pathology of esophageal carcinoma is/are correct?
371
p) Worldwide, adenocarcinoma is the most common esophageal malignancy. q) Squamous cell carcinoma is most common in the distal esophagus, whereas adenocarcinoma predominates in the middle third. r) Patients with Barrett's metaplasia are 40 times more likely than the general population to develop adenocarcinoma. s) Metastases from esophageal carcinoma are characteristically localized to regional mediastinal lymph nodes adjacent to the tumor. t) Achalasia, radiation esophagitis, caustic esophageal stricture, Barrett's mucosa, and Plummer-Vinson syndrome are all premalignant esophageal lesions that predispose to the development of squamous cell carcinoma. 36-45-year-old with isolated 6-cm colorectal metastasis in the liver 2 years after colectomy, otherwise healthy pest treatment would be: k) Radiofrequency ablation l) Systemic chemotherapy m) Hepatic lobectomy n) Liver transplantation o) Cryosurgical ablation 37- Oesophageal atresia all true except: k) Is often associated with a distal trachea-oesophageal fistula l)
Polyhydramnios is often present late in pregnancy
m) 50% have other associated congenital abnormalities n) Contrast X-ray studies are necessary to confirm the diagnosis o) Post-operatively over 30% develop oesophageal strictures 38-All are true about the dumping syndrome except: p) q) r) s) t)
Symptoms can be controlled with a somatostatin analog. Diarrhea is always part of the dumping syndrome. Flushing and tachycardia are common features of the syndrome. Separating solids and liquids in the patient's oral intake alleviates some of the symptoms of the syndrome. Early postoperative dumping after vagotomy often resolves spontaneously.
39-Which of the following statements about gastric polyps is/are true? k) Like their colonic counterparts, gastric epithelial polyps are common tumors. l) They are analogous to colorectal polyps in natural history. m) Endoscopy can uniformly predict the histology of a polyp based on location and appearance. n) In a given patient, multiple polyps are generally of a multiple histologic type. o) Gastric adenomatous polyps greater than 2 cm. in diameter should be excised because of the risk of malignant transformation. 40-All of the following statements about surgical management of gastric lymphomas are true except: k) Stage I gastric lymphomas (small lesions confined to the stomach wall) can be cured completely with surgical therapy alone. l) Extensive gastric lymphomas that initially are treated with radiation and/or chemotherapy occasionally perforate during treatment and require secondary resection.
372
m) Patients explored with a presumptive diagnosis of gastric lymphoma should undergo an attempt at curative resection when this is safe and feasible. n) Without a preoperative diagnosis resection for gastric mass should not be attempted unless lymphoma can be excluded. o) Appropriate staging for primary gastric lymphoma includes bone marrow biopsy. 41-the most accurate test to confirm diagnosis of infected necrotizing pancreases is: k) Abdominal ultrasound study l) Indium-labeled leeukocte scan m) Cimputed tomographic scan n) Elevated serum level of interleukain 6 and 8 o) Percutaneous needle aspiration
42- Which of the following variables best predicts prognosis for patients with a recent diagnosis of cutaneous melanoma and no clinical evidence of metastatic disease? p) q) r) s) t)
Breslow thickness. Clark's level. Ulceration. Gender. Celtic complexion.
43-the following are true about intracranial tumors except: k) The most common location of brain tumors of childhood is the posterior cranial fossa. l) With few exceptions, examination of the CSF is of no value in the diagnosis of an intracranial tumor. m) Even the most malignant of primary brain tumors seldom spread outside the confines of the central nervous system (CNS). n) The majority of astrocytomas can be cured surgically. o) Primary neoplasms of astrocytic, oligodendroglial, or ependymal origin represent gradations of a spectrum from slowly growing to rapidly growing neoplasms. 44 A right-sided disc herniation at the L5–S1 level typically may cause: k) Low back pain and left sciatica. l) Weakness of dorsiflexion of the right foot. m) A diminished or absent right ankle jerk. n) Diminution of sensation over the medial aspect of the right foot, including the great toe. o) Weakness of dorsiflexion of the left foot. 45-. The preferred operation for initial management of a thyroid nodule that is considered suspicious for malignancy by FNAB is: p) q) r) s)
373
Excision. Partial lobectomy. Total lobectomy and isthmusectomy. Total thyroidectomy.
t)
All methods are correct
46-the most common presentation of Meckel,s diverticulum in an adult is: p) q) r) s) t)
GIT bleeding GIT obstruction حسب ريكال Intussuception Litter,s hernia Diverticulitis
47-Optimal front-line treatment of squamous cell carcinoma of the rectum includes: k) Abdominal perineal resection. l) Low anterior resection when technically feasible. m) Radiation therapy. n) Chemotherapy. o) Combined radiation and chemotherapy. 48 -65-year-old man presents with complaints of mucous discharge and perianal discomfort. Physical examination reveals a fistulous opening lateral to the anus. Anoscopic examination permits passage of a probe through the fistula tract. The fistula traverses the internal anal sphincter, the intersphincteric plane, and a portion of the external anal sphincter. The fistula is categorized as which type? a) Intersphincteric b) Transsphincteric c) Suprasphincteric d) Extrasphincteric e) Non of the above 49-Warthin's tumor: k) Is a pleomorphic adenoma of salivary gland l) Should be treated by total paritidectomy m) Is considered a benign salivary gland neoplasia n) Respond well to preoperative radiotherapy o) Often present with facial nerve compression 50-A 38 year old woman presents with right upper quadrant pain and bouts of vomiting. She is known to have gallstones and has had similar episodes in the past. Which of the following might support a diagnosis of acute cholecystitis rather than biliary colic a) duration of symptoms b) Severity of vomiting c) Presence of Murphy's sign d) Presence of gas under right hemidiaphragm on erect CXR. THE ANSWER KEY : 1- B 2- C
374
31- E 32- B
3- D 4- C 5- B 6- C 7- A 8- C 9- B 10- C 11- B 12- D 13- C 14- E 15- B 16- A 17- D 18- E 19- A 20- C 21- D 22- D 23- B 24- D 25- A 26- A 27- C 28- A 29- E 30- E
33- D 34- B 35- C 36- C 37- B 38- B 39- E 40- D 41- E 42- A 43- E 44-C 45- C 46- A 47- E 48- B 49- C 50- C
Surgical MCQ
1. The radial nerve. k)
Is a branch of the anterior cord of the brachial plexus.
l) Is derived from the posterior primary rami of C5 to C7. m)
Supplies the flexors of the arm.
n)
Gives rise to the anterior interosseous nerve.
o)
Injury above elbow produces a classical wrist drop.
2. Regarding femoral canal all are true except: k)
Lies lateral to the femoral vein.
l) Has the inguinal ligament as its anterior border. m)
Has the lacunar ligament as its medial border.
n)
Has the pectineal ligament as its posterior border.
o)
Contains the lymph node of Cloquet.
3. The following causes hypercalcaemia except: a) Sarcoidosis. b) Primary hyperparathyroidism. c) Acute pancreatitis.
375
d) Metastatic bronchial carcinoma. e) Milk-Alkali syndrome. 4. Number of human chromosomes in human female are: k)
23 pairs +XX.
l)
21 pairs +XY.
m)
22 pairs +XY.
n)
22 pairs +XX.
o)
23 pairs +XY.
7. Potassium deficiency should be suspected in all the following except: n)
In cases of paralytic ileus.
o)
When the patient's reflexes are exaggerated.
p)
If there is a decrease in height and peaking of the T waves of an ECG.
q)
In alkalotic states.
6. Active immunization in case of tetanus: p)
Antitetanus human serum.
q)
Gives short period of protection.
r)
Given in case proved tetanus.
s)
Use of immunoglobulin.
t)
None of the above.
7. All of the following are signs of rised intracranial pressure except: a) Headache. b) Vomiting. c) Papilledema. d) Aphasia. e) Bradycardia. 8. All of the following are extra cranial hematomas except: a) Subcutaneos haematoma. b) Extra dural haematoma. c) Cephalohaematoma. d) Subgaleal hematoma. e) Subperiostial haematoma. 9. Glasgow coma scale all the following are true except: o)
Used for evaluation of comatose patient.
p)
It ranges from ( 3 to 15).
q)
Useful for neurological follow up.
r)
Useful for pupils evaluation.
s)
Best motor response given 6 point.
10. All the followings are Indications for central line insertion EXCPET:
376
v. w. x. y.
Massive fluid replacement Massive blood replacement Measurement of central venous pressure Prolonged Intervenes fluid therapy
11. Most common early complication of central venous line is: a) Sepsis b) Pneumothorax c) Thoracic duct injury d) Thrombosis 12. The following are Complications of shock: a) Acute Respiratory failure b) Acute myocardial infarction c) Acute renal failure d) A&C only e) All the above 13. Causes of delayed union of fractures includes all the following EXCPET: a) Compound fracture b) Infection c) Adequate immobilization d) Poor blood supply 14. In Head injury C.T. scan is indicated in the following p) Aphasia after the injury q) Deterioration of level of consciousness r) Skull fracture with persistent headache s) A&B only t) All the above 15. Most important steps in management of head injury include: U. Prevent hypoxia V. Prevent Dehydration W. Assure Brain Metabolism X. Prevent secondary brain injury Y. All the above 16. Tension pneumothorax U. is the commonest type of chest injuries V. Needs urgent X-Ray chest W. Is a clinical Diagnosis X. Causes flat neek viens Y. Treated by thoracotomy tube after chest X-ray. 17. Calcitonin hormon is secreted to the blood circulations from:
377
o)
Parathyroid gland.
p)
Parafollicular cells of thyroid gland.
q)
Supra renal gland.
r)
Pituitary gland.
s)
Gonads.
18. Regarding tension pneumothorax, the first step in the management is: p)
Obtaining a stat chest x-ray.
q)
Cricothyroidectomy.
r)
Pass an endotracheal tube.
s)
Starting oxygen by a valve-mask device.
t)
Chest decompression needle.
19. The following are features of thyrotoxicosis except: o)
Weight gain.
p)
Palpitations.
q)
Proximal myopathy.
r) Increased skin pigmentation. s)Pretibial myxoedema.
20. The following is a clinical feature of Horner's syndrome: m)
Miosis
n)
Failure of abduction of the orbit
o)
Increased sweating on the contralateral side of the forehead
p)
Exophthalmos.
e)
All are true
21. In role of nine extent of burn if entire trunk is burned it will be equal to: p)
9% body surface area.
q)
18% body surface area.
r)
36% body surface area.
s)
27% body surface area.
t)
45% body surface area.
22. Trachlea (4th) cranial nerve supply : k)
Lateral rectus muscle of th eye.
l)
Medial rectus muscle of the eye.
m)
Superior obligue rectus muscle of the eye.
n)
Superior oblique muscle of the eye.
o)
Muscles of the upper eye lid.
23. Regarding pathological terms : p)
Hypertrophy is an increase in tissue size due to increased cell number.
q)
Hyperplasia is an increase in tissue size due to an increase in cell size.
r) Atrophy is an increase in tissue size due to disuse. s)Metaplasia is a change form one abnormal tissue type to another.
378
t) A hamartoma is a developmental abnormality. 24. Regarding nephroblastomas: k)
They are otherwise known as a Wilm's tumour.
l) Account for 10% of childhood tumours. m)
The commonest presentation is with an abdominal mass.
n)
Most commonly present between 2nd and 4th year of life.
o)
All are true.
25. Regarding fluid losses in a major burn all are true except: p)
Are maximal between 12 and 24 hours after the injury.???
q)
Are related to the age of the patient.
r) Are not related to the weight of the patient. s)Are related to the area burnt. t) Are not related to the burn duration. 26. In obstructive jaundice : u)
Urinary conjugated bilirubin is increased.
v)
Serum unconjugated bilirubin is increased.
w)
Urinary urobilinogen is increased.
x)
Serum conjugated bilirubin is reduced.
y)
Faecal stercobilinogen is increased.
27. Regarding Hydatid disease: u) Is due to Ecchinococcus granulosa. v) Man is an accidental intermediate host. w) The liver is the commonest site of infection. x) Can be diagnosed by the Casoni test. y) All are true 28. The first aid of treatment in fracture of cervical spine should be: o)
Cervical spine x-ray.
p)
Analgesia.
q)
Neck immoblization.
r)Cervical traction. s)
Non of teh above.
29. The first aid of treatment in fracture of cervical spine should be: a) Cervical spine x-ray. b) Analgesia. c) Neck immoblization. d ) Cervical traction. 30. Regarding local anaesthesia:
379
a)
Local anaesthetics act on small before large nerve fibres
b)
Adrenaline reduces absorption and prolongs the local effects
c)
Preparations containing adrenaline are safe to use on digits and appendages
d)
Lignocaine has a longer duration of action than bupivicaine.
e)
All are false.
31. Small bowel obstruction often results in: (all correct except one) z)
Hyperkalaemia.
aa)
Metabolic alkalosis.
bb)
Oliguria.
cc)
Hypovolaemia.
dd)
Severe dehydration.
32. A serious intra-abdominal injury in a comatose patient may be diagnosed by: (all are correct except one) q)
Abdominal paracentesis.
r) The observation of bruising pattern on the abdominal wall. s)
Falling of heamoglobin values.
t) The presence of marked abdominal distetion. u)
The presence of diarrhea.
33. A perforated duodenal ulcer, all are true except: q)
Usually lies on the anterior or superior surface of the duodenum.
r)
Usually presents with the acute onset of severe back pain.
s)
Produces radiological evidence of free gas in the peritoneum in over 90 percent of the patients.
t)
Is usually treated by vagotomy and pyloroplasty.
u)
Is usually treated conservatively.اعتقد
34. Acute pancreatitis typically: (all correct except one) u)
Is accompanied by hypocalcaemia.
v)
Produces paralytic ileus.
w)
Is associated with a pleural effusion.
x)
Produces pyloric stenosis.
y)
Upper abdominal pain and vomiting.
35. Biliary colic typically: q)
Occurs 3 to 4 hours after meals.
r) Lasts 5 to 20 minutes. s)
Radiates from the upper abdomen to the right subscapular region.
t) Is made better by deep inspiration. u)
B&C only.اعتقد
36. In post operative DVT, the following are true except:????
380
q)
Clinical DVT occures in the 4th post operative day.
r) If complicated by pulmonary embolism, it occures usually after the 7th post operative day. s)
The process of DVT starts preoperatively with the induction of anaesthesia .
t) When discovered we should start the patient on coumadin "Warfarin anticoagulation". u)
It may lead to chronic venous in suffering as a complication of DVT.
37. In acute appendicitis all of the following are true except: o)
Anorexia.
p)
Abdominal pain usually preceedes vomiting.
q)
Pain after begins in the paraumbilical region.
r) Constipation diarrhea may occur. s)
Dysuria excludes the diagnosis.
38. The most common cause of massive haemorrhage in the lower gastroinfestinal tract is : p)
Carcinoma.
q)
Diverticulosis
r) Diverticulitis s)
Polyp.
t) Ulcerative colitis. 39. Painless haematuria is the leading presentation of : u)
Renal cell carcinoma.
v)
Transitional cell carcinoma of the bladder .
w)
Ureteric stone.
x)
Pelvi-ureteric obstruction.
y)
Ureterocele.
40. All of the following are complications of massive blood transfusion except: k)
Acute congestive heart failure.
l) Transmission of infection. m)
Hypercalcaemia.
n)
Hyperkalaemia.
o)
Transfusion reactions.
41. Complication of undescended testis include all of the following except : k) Malignant degeneration. l)
Increased susceptability to trauma.
m) Increased spermatogenesis. n) More liable to testiculer torsion.
381
o) Psychological complication. 42. The recurrent laryngoeal nerve is branch of : o) Facial nerve. p) Glosso-pharyngeal nerve. q) Cervical plexus. r) Vagus nerve. s) Brachial plexus. 43. The thyroid tumor which is may be associated with pheochromocytoma is : u)
Papillary carcinoma.
v)
Medullary carcinoma.
w)
Follicular carcinoma.
x)
Anaplastic carcinoma.
y)
Malignant lymphoma.
44. The most common pancreatic cyst is : q)
Dermoid cyst of the pancreas.
r)Hydatid cyst of the pancreas. s)
Pancreas pseudocyst.
t) Pancreatic cystadenoma. u)
Congenital cystic disease of the pancreas.
45. The anatomical division between the anus and rectum : p)
Lateral haemorrhoidal groove.
q)
Inter haemorrhoidal groove.
r) Dentate line. s)
Arcuate line
t) Ano-rectal ring. 46. The comments type of Anorectal abscess is: m) Ischio rectal n) Perianal o) Submucons p) Pelvirectal 47. Anal Fissure: p) Usually anterior q) May be caused by previous anal surgery r) Can cause dark bleeding PR. s) Sometimes is painful t) Treated by steroids 48. Neonatal duodenal obstruction: p) May be associated with down's syndrome. q) Is more frequently found in premature infants. r) Typically presents with gross abdominal distension.
382
s) Usually presents with vomiting of non-bile stained fluid t) B&C only. 49. acute superior mesenteric artery occlusion: (all correct except one) q) Characteristically presents with sudden pain and tenderness of increasing intensity. r) Is frequently accompanied by overt or occult blood loss in the stools. s) Frequently produces peritonitis. t) Can usually be diagnosed on plain abdominal x-rays. u) Can be diagnosed by mesenteric artery ongiography. 50. Regarding the management of polytrauma: Death follow a trimodal distribution. X-ray after primary survey should be AP cervical spine, chest and pelvis. Cardiac tamponade is characterized by raised B.p, a low JUP. Assessment of uncomplicated limb fractures should occur during the primary survey. y) A and B only. u) v) w) x)
51. Injuries to the urethra (all are correct except one) a) Are more common in male. b) Are often caused by road traffic accidents. c) Are easily diagnosed on intra venous pyelography. d) Require urgent surgical treatment. e) Diagnosed by retrograde urethragraphy. 52 .Car seat belts when properly adjusted a) Prevent injuries to abdominal organs. b) May cause small bowel injuries. c) Do not reduce the incidence of head injuries of passengers involving in RTA. d) Protect the cervical spine during sudden acceleration . e) A & D only. 53. Patients with major burns: u) Are in a negative nitrogen balance. v) Have normal calorie requirements. w) Do not generally become anaemic. x) Are resistant to septicaemia. y) All of the above. 54 . In a healing fracture: (All correct except one) a) The haematoma is initially invaded by osteoblasts. b) The tissue formed by the invading osteoblasts is termed osteoid. c) Calcium salts are laid down in the osteoid tissue. d) The final stage of repair is the remodelling of the callus. e) The callus formation is related to the amount of stress at fracture side. 55. In a colles’ fracture the distal radial fragment: p) Is dorsally angulated on the proximal radius. q) Is usually torn from the intra-articular triangular disc. r) Is deviated to the ulnar side. s) Is rarely impacted. t) Is ventrally displaced. 56. A malignant melanoma: a) Frequently arises from hair-bearing naevi. b) Frequently arises from junctional naevi. * c) Has a worse prognosis when it areses on the leg.
383
d) Should be suspected in any big pigmented lesion. e) Non of the above is correct. 57. Squamous cancer of the lip: a) Is most common in early adult life. b) Is more common in fair skinned subjects. c) Metastasises readily by the blood stream. d) Is preferably treated by radiotherapy once lymph node deposits are present. e) All of the above are correct. 58- Basal cell carcinomas: a) Usually metastasise to regional lymph nodes. b) Are less common than squamous cell carcinomas. c) Are characterised histologically by epithelial pearls.اعتقد d) Are particularly common in oriental races. e) Non of the above is correct. 59- Fiboadenomata of the breast: a) Are commonest in early adult life. b) Are indiscrete and difficult to distinguish. c) Are usually painless. d) Resolve without treatment. e) A&C only. 60. Paget’s disease of the nipple: a) Usually presents as abilateral eczema of the nipple. b) Is always related to an underlying breast cancer. c) Indicates incurable breast cancer. d) Has non-specific histological characteristics. e) A&C only. 61- stones in the common bile duct: a) Are present in nearly 50 per cent of cases of cholecystitis. b) Often give rise to jaundice, fever and biliary colic. c) Are usually accompanied by progressive jaundice. d) Are usually associated with a distended gallbladder. e) A&D only. 62- Colonic polyps: (all correct except one) u) Are associated with colonic cancer. v) May be hereditary. w) Should not be removed if they are asymptomatic. x) May be hyperplastic. y) Are commonly adenomatous. Prepared by: Dr. Spiro A. Tawil FRCSI Dip. Pediatric surgery - London Consultant surgeon Shifa hospital - Gaza Key Answers 1 2 3
384
E A C
16 17 18
C B E
31 32 33
A E A
46 47 48
B B A
61 62
B C
4 5 6 7 8 9 10 11 12 13 14 15
D B E D C D B B E C E E
19 20 21 22 23 24 25 26 27 28 29 30
A A C D E A A A E C C E
34 35 36 37 38 39 40 41 42 43 44 45
D C D E B A C C D B C E
49 50 51 52 53 54 55 56 57 58 59 60
D E C B A A A B B E E B
Abdominal Wall Hernia 1. The diagnosis of an inguinal hernia: A. In infants often depends on the history given by its mother. B. In the adult is most easily made with the patient in the sitting position. C. Depends on the hernial sac or cough impulse being felt below the inguinal ligament. D. Is supported by the presence of a transilluminable scrotal swelling. E. B&C only. 2. Inguinal herniae: A. In children are usually of the direct type. B. Of the indirect type are congenital in origin. C. Will regress spontaneously in children. D. In young adults are most commonly of the direct type. E. C&D only. The majority of indirect inguinal herniae are congenital in origin 3. Strangulated contents of hernial sacs: A. Are always accompanied by intestinal obstruction. B. Are more common in direct than indirect inguinal herniae. C. Are usually reducible. D. Produce local pain and tenderness. E. All of the above Regarding indirect inguinal hernia in children, all of the following are true, except: . Right side is more common than left side . Boys are affected much more than girls . Incarceration occurs more often in small babies . Operative correction is delayed till the child is one year old . Inguinal truss is nowadays not used in the management 4. Incisional herniae are related to:)All correct except one) A. Wound infections. B. Anaemia and malnutrition. C. Obesity. D. The use of absorbable suture materials. E. To the technique of wound closure. 5. herniae in the umbilical region:
385
A. Are always congenital in origin. B. Usually occur in males. C. Usually require surgical repair in infants. D. Rarely strangulate. E. All of the above 6. A discharge from the umbilicus: (all correct except one) A. May indicate a patent vitello-intestinal duct. B. May indicate an anomaly of the urachus. C. At the time of menstration may indicate endometriosis. D. In the neonate is of no immediate clinical significance . E. May indicate infection of the umbilicus (omphalitis) 7. An exomphalos: A. Is a congenital defect of the urethra. B. Is acquired defect of the anterior abdominal wall. C. Is otherwise known as gastroschisis. D. Needs urgent surgical treatment. E. A&B only.
8. The cremaster muscle is derived from: A. External oblique aponeurosis. B. Internal oblique muscle. C. Transversus abdominis muscle D. Transversalis fascia E. Parietal peritoneum. 9. structures lying within the spermatic cord include: ( all correct except one) A. Direct hernia sac B. Testicular artery. C. Indirect hernia sac. D. Proportional fat. E. Vas deferens. 10. True statements concerning a femoral hernia include which of the following? A. It is common in male. B. It is usually results from a defect in lateral part of transversalis fascia. C. It is common in children. D. It is less common in female. E. It may be confused with inguinal lymphadenopathy. Acute Abdomen 1. Acute abdominal pain which is (all correct except on) A. Colicky in nature indicates obstruction of hollow viscus. B. In right upper quadrant increased by inspiration is typical of cholecystitis C. Continuous is typical of inflammation D. Maximal in the right loin is typical with duodenal ulcer. E. Maximal in epigastrium and related to meal is typical of gastric problem. 2. Vomiting in acute abdomen A. Occurring soon after the oncet of colicky pain indicates pathology outside the gastrointestinal tract. B. Of fluid containing no bile is characteristic of small bowl obstruction C. Of faeculent fluid usually indicates gastrointestinal fistula. D. Of bile stain fluid usually indicates stomach outlet obstruction E. B&C only
386
3. Faeculent vomiting: A. Is commonly seen after upper gastrointestinal tract bleeding. B. Indicates large bowel obstruction. C. Indicates bacterial proliteration in the upper intestinal. D. Suggests a gastro-colic fistula. E. Suggests small bowel fistula 4. A patient with generalized peritonitis:(all correct except one) A. Usually has an elevated temperature and pulse rate. B. Characteristically complains of spasmodic severe pain which causes him to be restless. C. Is characteristically vomites. D. Will usually have a rapid and deep respiratory pattern. E. Is Usually has an elevated WBCs. 5. A perforated duodenal ulcer: A. Usually lies on the anterior or superior surface of the duodenum. B. Usually presents with the acute onset of severe back pain. C. Produces radiological evidence of free gas in the peritoneum in over 90 percent of the patients. D. Is usually treated by vagotomy and pyloroplasty E. Is usually treated conservatively. Abdominal radiograph will show free air in 70-90 percent of cases. 6. Congential pyloric stenosis:(all correct except one) A. Occurs more commonly in male children. B. Usually presents in the first few days of life . C. Presents with non bile stained vomiting. D. Is usually diagnosed on clinical examination. E. May cause right upper quadrant tenderness. congenital pyloric stenosis usually presents between the 2nd and 4th weeks of life 7. Appendicitis is: A. More common in females B. Distributed evenly thoughout the world's population C. More likely to occur if the appendix is in the retrocaecal position. D. Commonly the result of appendicular obstruction. E. B&C only. 8. Likely differential diagnoses in a young woman with appendicitis include: A. Ovarian carcinoma B. Ruptured ectopic pregnancy. C. Colonic diverticulitis. D. Caecal carcinoma. E. C&D only. 9. Obstruction of the lumen of the appendix may lead to:(all correct except one) A. Mucosal ulceration B. Gangrenous appendicitis. C. A perforated appendix. D. Intussusception of the appendix. E. Acute appendicitis. 10. Acute non-specific mesenteric lymphadenitis: (all correct except one) A. Is commonest between 5 and 12 years of age. B. Is usually associated with an upper respiratory tract infection. C. Is usually associated with cervical lymphadenopathy D. Is characterized by enlarged mesenteric lymph nodes which are infected by gram-negative organisms. E. Is one of the differential diagnosis of acute appendicitis. 11. The level of intestinal obstruction can be determined by:(all correct except one) A. Questioning the patient.
387
B. Examining the patient. C. Radiological examination of the patient. D. Repeated measurements of the patient's girth. E. Upper & lower contrast studies. 12. Acute small bowel obstruction:(all correct except one) A. Is commonly caused by postoperative adhesions. B. Accompanied by the signs of peritonitis, suggests bowel strangulation. C. Is often associated with a raised serum amylase. D. Generally produces abdominal distension within 2 to 3 hours of onset. E. Can be diagnosed by history and clinical examination. 13. In the treatment of intestinal obstruction: A. Nasogastric suction should be instituted preoperatively. B. Intravenous fluid replacement might be required. C. Immediate surgery is essential. D. Surgery should be restricted to those cases where strangulation is diagnosed. E. C&D only. 14. Strangulation of the bowel:(all correct except one) A. commonly complicates closed loop obstruction. B. Is difficult to distinguish from simple intestinal obstruction. C. Is accompanied by bleeding into the affected bowel. D. Frequently causes peritonitis. E. Is characterized by severe abdominal pain. 15. Large bowel obstruction: A. Is most commonly caused by colonic cancer B. Has its maximum incidence before the age of 50. C. Frequently presents with nausea and vomiting. D. Usually heralds its onset with constant suprapubic pain. E. Frequently treated conservatively. 78% of patients 16. Patients with acute colonic deveticulitis: A. Often give a history of recent lower abdominal colic. B. Often present with pyrexia. C. Can be frequently diagnosed on sigmoidoscopic appearances. D. Frequently develop faecal peritonitis. E. All the above. 17. Acute pancreatitis typically:(all correct except one) A. Is accompanied by hypercalcaemia. B. Produces paralytic ileus. C. Is associated with a pleural effusion. D. Produces pyloric stenosis. E. Upper abdominal pain and vomiting. 18. Acute pancreatitis:(all correct except one) A. Often simulates a perforated peptic ulcer in its presentation. B. Often presents with the signs of hypovolaemia. C. Can readily be distinguished from other causes of acute abdominal pain by the presence of a raised serum amylase. D. Frequently has a raised concentration of urinary amylase. E. Most commonly caused as a complication of GB stones. 19. The treatment of acute pancreatitis: A. Is largely nonspecific and supportive. B. Should include a laparotomy in the majority of cases. C. Should routinely include the administration of calcium. D. Should routinely include the administration of antibiotics. E. All the above.
388
Most Common abdominal emergency of early childhood A - Hydronephrosis B - Intussusception C - neuroblastoma D - adhesion E - acut appendiitis 20. Childhood intussusception:(all correct except one) A. Usually presents during the first year of life. B. Is frequently ileocolic. C. Can usually be diagnosed without x-ray examination of the abdomen. D. Rarely requires surgical treatment. E. Can be diagnosed by abdominal US. 21. Meconium ileus: A. Is the presenting feature in the majority of patients with cystic fibrosis. Meconium ileus is the presenting feature in 10 to 15% of infants with Cystic Fibrosis ( B. Is associated with achlorhydria. C. Presents with a distended abdomen and bilious vomiting. D. Often can be effectively treated with acetyl cysteine. E. All the above. The most common cause of rectal bleeding in infancy and childhood is: A. Anal fissure B. Volvulus C. Foreign body D. Intussusception E. Meckel's diverticulum 22. Neonatal duodenal obstruction: A. May be associated with down's syndrome. B. Is more frequently found in premature infants. C. Typically presents with gross abdominal distension. D. Usually presents with vomiting of non-bile stained fluid E. B&C only. 23. acute superior mesenteric artery occlusion: (all correct except one) A. Characteristically presents with sudden pain and tenderness of increasing intensity. B. Is frequently accompanied by overt or occult blood loss in the stools. C. Frequently produces peritonitis. D. Can usually be diagnosed on plain abdominal x-rays. E. Can be diagnosed by mesenteric artery ongiography. 24. A ruptured ectopic pregnancy:(all correct except one) A. Usually occurs in the first month of pregnancy. ampullary implantations are more likely to rupture at 6 to 8 weeks of gestation B. Usually presents with severe lower abdominal pain. C. Frequently presents with hypovolaemic shock. D. Can usually be diagnosed by pelvic examination. E. Can usually be diagnosed by transvaginal US. 25. Biliary colic typically: A. Occurs 3 to 4 hours after meals. B. Lasts 5 to 20 minutes. C. Radiates from the upper abdomen to the right subscapular region . D. Is made better by deep inspiration. E. B&C only. Biliary colic usually causes severe, steady pain that lasts from 15 to 60 minutes to up to 6 hours
389
peptic ulcer disease occurs usually 60 to 90 minutes after meals (gastric ulcer) or 3 to 4 hours after meals(duodenal ulcer) and is relieved by an antacid Choose THE BEST APPROPRIATE Answer in Each of the following Questions: 1. the MOST important finding in the diagnosis of acute appendicitis is: a. vomiting b. fever c. leukocytosis d. right lower quadrant tenderness e . positive Rovsing sign 2. The commonest cause of bloody nipple discharge is a. mammary ductectasia b. carcinoma of the breast c. lactational mastitis d. duct papilloma e. fibrocystic disease of the breast 3. Second degree burn: a. Heals in 10-21 days b. Painful c. Good capillary refill d. All the above 4. The following can cause Respiratory acidosis except: a. pre existing lung disease b. inadequate ventillation c. long upper abdominal incision d. all the above e. repeated vomiting 5. The classical picture of Acute arterial embolism include All the following except: f. Pallor g. Pain h. Parasthesia i. Impalpable Peripheral pulses j. Swelling 6. Severity of burns depends on: a. Depth of burn b. Total Body Surface Area (TBSA) c. Associated disease or injury d. All the above 7. Prophylactic antibiotics a. should be given one day before the operation b. should be bacteriostatic c. usually narrow spectrum d. usually stopped 24hours after the operation e. none of the above
390
8. Clinical picture of varicose veins includes all the following except: a. Difficulty in standing b. Heaviness & fullness in legs c. Ankle swelling at the end of day d. Itching e. leg cramps at night 9 . Dialy requrment per kg for the adult includes all the following except a. Water 30-50ml b. Calories 30- 50 kcal c. Nitrogen 0.20-0.35g. d. Sodium 2-3mmol e. Potassium 0.7-0.9mmol 10. Patients on long term steroid treatment Going for surgery will need to: a. Stop treatment , on the morning of surgery b. Stop treatment three days before surgery c.Continue same treatment d. Increase the dose before surgery e. decrease the dose before surgery 11.Causes of metabolic acidosis include all the following Except : a. small bowel fistula b. shock c. severe anemia d. CO poisoning e. all of the above 12. Regarding obstructive jaundice a.caused by hemolytic anaemia b. urobilinogen is absent in the urine c. carcinoma of the head of pancreas is the commonest cause d. normal color stool e. b and c only 13. Carbuncle: f. is a confluence of several boils g. usually cause minimal tissue destruction, h. has one opening i. streptococcus is the commonest organism j. commonest site is the abdominal wall 14. Pilonidal sinus: a. less common in females b. Can occur in the umbilicus c. May affect barbers d. Sometimes accidentally discovered e. All the above 15. Hidradenitis suppurativa: a. inflammation of the apocrine sweat glands b. common site is the axilla c. commoner in women d. all of the above
391
e. none of the above 16. To obtain fine line scar: a.The incision should fall in the natural skin lines. b. Keep sutures for at least 14days c. Use absorbable sutures d. All the above 17. All the following are causes of post operative jaundice Except: a. Massive blood transfusion b. sepsis c. hyperperfusion d. .residual haematoma e. viral hepatitis 18. A perforated duodenal ulcer f.Usually lies on the anterior or superior surface of the duodenum . g.Usually presents with the acute onset of severe back pain. h.Produces radiological evidence of free gas in the peritoneum in over 90 percent of the patients. i.Is usually treated by vagotomy and pyloroplasty. j.Is usually treated conservatively. 19. The acute phase response includes: f. Hypothermia. g. Decreased plasma albumin. h. Hepatic sequestration of copper. i. Increased C-reactive protein. j. Neutrophil leucocytosis. 20.All regarding the flow phase after injury is true Except : a. Glucagon breaks down glycogen stores . b..hepatocytes produce glucose (gluconeogenesis). c. fat breakdown occurs as a result of catecholamine stimulation . d. growth hormone levels are elevated . e. hypoglycemia result as tissues deplete blood glucose stores. 21. levels of the following substances are elevated during the acute response to injury Except ? a.glucagon . b.insulin. c.catecholamine. d.glucocorticoids. e.growth hormone. 22. What is the commonest cause of low grade fever period immediately following abdominal surgery ? a. Atelectasis. b.Pulmonary embolism. c.Wound infection. d.Pseudomonas colitis.
37.2-38.3c in the
23. Anaphylaxis is characterized by all of the following EXCEPT: P) is a reaction either local or general , frequently occurs within five minutes
392
48 hrours
Q) causes an urticarial eruption R) is produced by IgA antibody S) causes eosinophilia T) causes degranulation of basophils and mast cells 24. Shock can best be defined as: q. Hypotension. r. Hypoperfusion of tissues to meet the minimal requirements of cells s. Hypoxemia. t. All of the above. 25. Which of the following statements about extracellular fluid are true? k. The total extracellular fluid volume represents 40% of the body weight. l. The plasma volume constitutes one fourth of the total extracellular fluid volume. m. Potassium is the principal cation in extracellular fluid. n. The protein content of the plasma produces a lower concentration of cations than in the interstitial fluid. o. The interstitial fluid equilibrates slowly with the other body compartments. 26. In acute cholecystitis: f. Commonest bacteria is E.coli g. Gall bladder wall thickness more than 3 mm h. WBC between 10_15,000 cell/mm3 i. mildly elevated bilirubin level j. all of the above. 27. All of the following are true about neurogenic shock EXCEPT: a. There is a decrease in systemic vascular resistance and an increase in venous capacitance. b. Tachycardia or bradycardia may be observed, along with hypotension. c. The use of an alpha agonist such as phenylephrine is the mainstay of treatment. d. Severe head injury, spinal cord injury, and high spinal anesthesia may all cause neurogenic shock. e. A and B 28. Which of the following is not an action of angiotensin II? a. aldosterone secretion b. sodium absorption c. efferent arteriolar constriction d. arterial dilation e. nephrosclerosis in the kidney 29. Hemostasis and the cessation of bleeding require which of the following processes? p. Adherence of platelets to exposed subendothelial glycoproteins and collagen with subsequent aggregation of platelets and formation of a hemostatic plug. q. Interaction of tissue factor with factor VII circulating in the plasma. r. The production of thrombin via the coagulation cascade with conversion of fibrinogen to fibrin. s. Cross-linking of fibrin by factor XIII. t. All of the above 30. Regarding polytrauma: f. Resuscitation in primary survey must start with airway care, breathing then circulation,.. g. X-ray after primary survey should be AP cervical spine, chest and pelvis.
393
h. Cardiac tamponade is characterized by raised blood pressure and a low JVP. i. Assessment of uncomplicated limb fractures should occur during the primary survey. j. a and b only. 31. Hemostasis and the cessation of bleeding require which of the following processes? f. Adherence of platelets to exposed subendothelial glycoproteins and collagen with subsequent aggregation of platelets and formation of a hemostatic plug. g. Interaction of tissue factor with factor VII circulating in the plasma. h. The production of thrombin via the coagulation cascade with conversion of fibrinogen to fibrin. i. Cross-linking of fibrin by factor XIII. j. All of the abov 32. Massive blood transfusion: a. is defined as replacement of at least one’s blood volume within the first 12 hours of resuscitation. b. can cause change in acid base balance c. DIC and coagulation defect is the most serious sequalae d. all are true 33. Regarding direct inguinal hernia: f. Common in young age g. Lies medial to inferior epigastric artery h. Internal ring test control it i. Complication is more than indirect hernia j. Bilateral in 20% 34. All of the following are functions of the gallbladder Except: a. absorption b. motor activity c. mucus secretion d. storage of bile e. formation of bile 35. Hypovolaemic shock is characterized by: f. A low central venous pressure , low cardiac output , low peripheral resistance g. A high central venous pressure , high cardiac output , low peripheral resistance h. A low central venous pressure , low cardiac output , high peripheral resistance i. A low central venous pressure , high cardiac output , high peripheral resistance j. A high central venous pressure , low cardiac output , low peripheral resistance 36. The Appendix all true except: p. Is typically less than 10 cm in length in the adult. q. Is located in the retrocaecal recess. r. Macburneys point, lies 2/3 laterally from a line from umbilicus to the anterior superior iliac spine. s. The longitudinal coat of the appendix is derived from the three bands of taenia coli. t. Is supplied by branches of the inferior mesenteric artery 37. Which of the following most often initiates the development of acute appendicitis? l. A viral infection. m. Acute gastroenteritis. n. Obstruction of the appendiceal lumen. o. A primary clostridial infection. 38. The inguinal canal:
394
a. Has a deep ring, which is a defect in the transversalis fascia. b. Is bounded posteriorly by the inguinal ligament. c. Has the internal oblique as part of its posterior wall throughout. d. Has the conjoint tendon superiorly. e. Transmits the ilioinguinal nerve, which enters the canal through the deep ring. 39. Which of these statements regarding Meckle’s diverticulum is correct? a. It is found in about 4% of the population. b. Is always found on the antimesenteric border of the ileum. c. Is usually about 2 cm in length. d. Is usually located about 20 cm from the ileocaecal junction. e. Is usually attached to the umbilicus. 40. The most common type of congenital diaphragmatic hernia is caused by: a. A defect in the central tendon. b. Eventration of the diaphragm in the fetus. c. A defect through the space of Larrey. d. An abnormally wide esophageal hiatus. e. A defect through the pleuroperitoneal fold.
MCQ for the 4th year : Select the Best Appropriate Answer 3. Patient with high risk for DVT : d. Smoker patient with operation more than 40 minutes e. Patient with upper abdominal surgery less than 40 minutes f. Patient with pelvic operation for malignancy d. B and c only e. None of the abo Electrocardiogram findings In hyperkalemia . .2 a. Depressed ST segment b. Tall peaked T wave c. Widened QRS d. All the above e. None of the above : The following can cause respiratory acidosis ..3 a. Post operative pain from abdominal incisions b. Post operative atelectasis c .Hypoventilation by the anesthetist d. None of the above e. All of the above : The following are factors determining severity of burn except .4 a. Extent of burnt surface area b. Depth of burn & causative agent c. Age of the patient ??????d. Associated injuries e. Site of the burn : Acute Epidural haematoma .5
395
a. Due to middle meningeal artery tear b. Can be treated conservatively c. Classically there is contra lateral, dilated, fixed pupil d. Needs operation within 24 hours e. A ad c only : Regarding papillary carcinoma of the thyroid.6 g. Commonest thyroid cancer h. Previous neck radiation decrease the risk i. Rare familial form j. Is the tumor of the middle age k. A and c only : The commonest type of breast carcinoma is .7 f. Invasive duct carcinoma g. Medullary carcinoma h. Invasive lobular carcinoma i. Mucinous (colloid) carcinoma j. Insitu carcinoma & Pagets disease
:The classical picture of Acute arterial embolism include All the following except.8 d.Pallor e.Pain f.Parasthesia d. Impalpable Peripheral pulses e. Swelling : The commonest cause of bloody nipple discharge is a. mammary ductectasia b. carcinoma of the breast c. lactational mastitis d. duct papilloma e. fibrocystic disease of the breast Tension pneumothorax .10 a. is the commonest type of chest injuries b.Needs urgent X-Ray chest c. Is a clinical Diagnosis d.Causes flat neek viens e.Treated by thoracotomy tube after chest X-ray. Ulcerative colitis .11. a. Shows full thickness inflammation b. The rectum is almost always involved c. 50% patients have terminal ileal disease d. Enterocutaneous or intestinal fistulae are common e. The serosa is usually affected The spleen .12 a. develops from ventral mesogastrium b. red pulp has immune function c. blood flow is 600ml/ min d. In contact with Tail of pancreas
396
.9
e. all the above Amoebic liver abscess 13 a. usually in the upper post surface of left lobe b. Solitary in 20% c. has typically golden yellow pus d. causes pyrexia and liver enlargement e. usually treated by surgical drainage .10. Ranson's criteria involves all the following except a. age < 55 years b. WBC > 16,OOO %c. FBG >11.2mmol/L or 200 mg d. LDH > 350 iu/L %e. SGOT >250 SF units The following causes elevation in serum amylase except .11 A .Perforated. duodenal ulcer B .Perforated. gall bladder c. perforated diverticulum d. Rupture Ectopic pregnancy e. Rupture AAA :Classical clinical picture of Crohn's disease . 12 a. colicky abdo pain exacerbated by meals b. rectal bleeding ?????????????c. clinical picture of appendicitis d. sever lower abdominal pain e. repeated vomiting 13. The following are involved in the aetiology of hemorrhoids except a. diet b. straining c. pregnancy d. portal hypertension e. cancer lower rectum Clinical features of anal fissure includes all the following except.14 a. sever pain b. bleeding c. sentinel skin tag d. normal anal sphincter e. mucus disharge In superficial burn all are true except .15 a. caused by direct flam & scalds b. appear moist , red c. usually heals without skin graft d. Pin prick test is negative e, may form blisters .16. Dextrose saline fifth strength solution is: a. hypotonic solution??? b. contains 45 mmol sodium c. supplies 268 kcal, if given to the patient ,
397
d. is similar to plasma e. non of the above 17. Indications for pre operative catheterization : a. patient at risk of renal failure b. patient require prolonged IV therapy c. all pelvic operations d. a & b only e. all the above 18. One of the followings is advantage of Thiersch graft )partial thickness) a. covers large area b. natural skin color c. no contraction d. does not need immobilization e. Used to cover face, tendons 19. All the following are complications of major burn except a. Acute renal failure (b. Gastroduodenal erosion (Curling’s ulcer c. Inhalation injury d. Acute liver failure e Infections Crohn's disease ..20 a. affect any part of GIT b. proximal ileum & caecum is the commonest to be affected c. skip lesions d. A and C only e. all the above 21. Haemangioma of the liver a. Usually Symptom less b. usually capillary type c. main site is the upper surface of Lt lobe d. can cause cardiac failure in children e. a and d only Prophylactic antibiotics.22 . a. should be given one day before the operation b. should be bacteriostatic c. usually narrow spectrum . g. usually stopped 24hours after the operation h. none of the above Regarding annular pancreas all are true except .23 a. more in Downs syndrome b.ring of tissue around the duodenum c.clinical picture of obstruction d.double bubble sign e.treatment usually is conservative Hidradenitis suppurativa .24 a. inflammation of the apocrine sweat glands b. common site is the axilla c. commoner in women d. all of the above e. none of the above
398
25. Regarding post splenectomy sepsis, all are correct except a. incidence is related to the age b. has rapid deteriorating clinical course c. has mortality rate 50 - 80%, death within 48 hrs d .usually due to capsulated bacteria e. pneumovaccione should be given prior to any splenectomy due to traum a 26. Sever attack of pancreatitis is a. attack followed by pseudocyst b. has has four points on Ranson criteria c. has 5 points on Balthazar grade d. all the above e. none of the above 27.Antibiotics in acute pancreatitis: a. not indicated b. second generations cephalosporins are the best c. indicated in all cases d. b and c only e. none of the above 28. Thomas test is positive in : a. Paget's disease of the hip b. Perthe's disease of the hip c. osteo arthritis of the knee d. Sudeck's atrophy of the shoulder e. none of the above 29. All the following are complications of cast except a. compartment syndrome b. pressure sore c. blisters with 2ry infection d. immobilization of joints e. crush syndrome 30. The following are methods of stabilization of fractures except: a. External splint e.g. POP b. Internal fixation c. Hanging plaster d. Skin traction e. a and b only 31. In critical ischemia there is: a. rest pain b. color changes c. oedema d. hyperesthesia e. all the above 32. Beurger's diseases has : a. occlusion of small and medium arteries b. thrombophlebitis c. Reynold's phenomenon d.all the above e. and b only
399
(Surgical Board Promotion Exam ( October 2009 1-
Cytokines are endogenous signals that stimulate the following except a. Local cell proliferation within the wound. b. The central nervous system to initiate fever. c. The production of “acute-phase proteins.” d. Hypoferremia. e. Septic shock.
2- You are carrying out a pleural aspiration for a patient with a persisting post-operative effusion. Which of the following structures do you want the needle to avoid? a. External intercostal muscle b. Internal intercostal muscle c. Parietal pleura d. Tranversus thoracis muscle e. Visceral pleura 3- Which of the following statements is true of upper limb nerve injuries? u. Injury to the median nerve results in a wrist drop v. Injury to the radial nerve results in loss of sensation over the palmar aspect of the index finger w. Injury to the median nerve results in loss of sensation in the anatomical snuffbox x. Injury to the ulnar nerve results in a claw hand y. Injury to the ulnar nerve results in loss of sensation over the thumb 4- A 9 month old boy presents with an acute scrotal swelling. The following diagnoses are likely: k. Epididymitis l. Orchitis m. Torsion of the testicular appendage n. Irreducible inguinal hernia o. Acute idiopathic scrotal oedema 5-
Factors inhibits spontaneous closure of small bowel fistula include all except a. Distal obstruction b.Crohn's disease c. Small bowel radiation d. Foreign body e. Long fistula tract
6- A 58-year-old lady is admitted with abdominal distension. Clinical examination reveals gross ascites. Subsequent CT of her abdomen & pelvis shows an abnormal right ovary and thickened omentum and the liver shows no abnormality. Which of the following investigations would be most useful in this lady’s management? a. αFP b. CA 125
400
c. CA 19-9 d. CEA e. Paraproteins 7-The characteristic changes that follow a major operation or moderate to severe injury do not include the following: a. Hypermetabolism. b. Fever. c. Tachypnea. d. Hyperphagia. e. Negative nitrogen balance. 8.In perforated duodenal ulcer all are true except a. Air under diaphragm is present in 80% of cases b.Best treatment is simple patch closure,HLO eradication and proximal vagotomy in stable patients c. Laparoscopic surgery has no role in surgical management d. There is rise of serum amylase in these patients e. Conservative treatment can be applied in stable patients, with more than 24 hour presentation
9- A 76-year-old man presents with weight loss, dark urine, and pale stools which are difficult to flush away. An excess of which of the following would account for this history? a. Conjugated bilirubin b. Hyperbilirubinaemia c. Stercobilinogen d. Unconjugated bilirubin e. Urobilinogen 10-All of the following may be useful in the treatment of cardiogenic shock except: a. Dobutamine. b. Sodium nitroprusside. c. Pneumatic antishock garment. M d. Intra-aortic balloon pump. 11-
Hypotensive severely injured patient showed not remain in emergency department more than a. 20 minutes b. 30 minutes c. 10 minutes d.40 minutes golden one hour An otherwise well 13-year-old boy is admitted complaining of sudden onset severe left -12 sided testicular pain 2 hours prior to admission. He gives no history of trauma, dysuria or frequency. On examination he is found to have a tender, high-riding testicle.What is the most ?appropriate next step in this young man’s management a. Herniography b. Scrotal Doppler ultrasound on the next available list c. FBC and U&E d. Scrotal Doppler ultrasound as an emergency e. Surgical exploration of his scrotum
401
13- An 18-year-old man shot once in the left chest has a blood pressure of 80/50 mm. Hg, a heart rate of 130 beats per minute, and distended neck veins. Immediate treatments should be: a. Administration of one liter of Ringer's lactate solution. b. Subxiphoid pericardiotomy. c. Needle decompression of the left chest in the second intercostal space. d. Emergency thoracotomy to cross-clamp the aorta e. Thoracostomy tube 14- The most common complication of pancreatic injury is a. pancreatic fistula b.pancreatic abscess c. pancreatic pseudo cyst d. Hemorrhage e. Pancreatic necrosis 15- Which of the following statements about extracellular fluid are true? a. The total extracellular fluid volume represents 40% of the body weight. b. The plasma volume constitutes one fourth of the total extracellular fluid volume. c. Potassium is the principal cation in extracellular fluid. d. The protein content of the plasma produces a lower concentration of cations than in the interstitial fluid. e. The interstitial fluid equilibrates slowly with the other body compartments 16- A 46-year-old man is being investigated in the vascular clinic for thoracic outlet syndrome. Which important structure passes anterior to the scalene tubercle on the first rib? a. Scalenus medius b. Subclavian artery c. Subclavian vein d. Superior intercostal artery e. Sympathetic trunk 17- In damage control surgery DCS all are true except a. Surgeon control bleeding and contamination, but defer definite surgical treatment b. Development of hypothermia less than340c,pH lessthan7.2,base deficit more than 12,all increase coagulopathy c.Celiac artery can be ligated d.Intraabdominal infection occur if abdominal packs are left for more than 24 hours e. Vacuum assisted closure for abdominal cavity i.e left open ,can be applied A 24-year-old man has sustained significant brain injury following an assault. He is -18 showing signs of a raised intra-cranial pressure. Which of the cranial nerves is usally the first ?to be affected by a raised intracranial pressure a. The abducent nerve b. The facial nerve c. The hypoglossal nerve d. The vagus nerve e. The vestibulocochlear nerve 19- Squamous cancer of the lip: a. Is most common in early adult life. b. Is more common in fair skinned subjects. c. Metastasises readily by the blood stream. d. Is preferably treated by radiotherapy once lymph node deposits are present.
402
e. All of the above are correct. 20- Emergency department thoracotomy a. Has better prognosis in abdominal than chest injuries b. has better prognosis in blunt trauma than penetrating trauma c. Left postrolateral thoracotomy is the incision of choice d. The pericardium showed be opened transversely in anterior surface e. The incision showed be placed in the Lt 4th, or 5th intercostals space just below nipple 21- Acute scrotum a. Torsion testis should be operated within 12 hour of presentation b. Epidedimoorchitis pain increase by testicular elevation c. If in doubt scrotum should be explored d.Doppler ultrasound has no role in diagnosis e.None of the above A 29-year-old lady has undergone a partial thyroidectomy for thyrotoxicosis. She has since -22 ?complained of a husky voice. Damage to which nerve has caused the change in her voice a. The external laryngeal nerve b. The glossopharyngeal nerve c. The internal laryngeal nerve d. The lesser petrosal nerve e. The recurrent laryngeal nerve A 28-year-old man is brought to the Accident and Emergency department having -23 sustained a laceration just above the right olecranon. Which of the following structures is ?most likely to be damaged a. Brachial artery b. Median nerve c. Radial nerve d. Triceps tendon c .Ulnar nerve 24- stones in the common bile duct: a. Are present in nearly 50 per cent of cases of cholecystitis. b. Often give rise to jaundice, fever and biliary colic . c. Are usually accompanied by progressive jaundice. d. Are usually associated with a distended gallbladder. e. A&D only. 25- Which of the following are not determinants of a postoperative cardiac complication? a. Myocardial infarct 4 months previously. b. Clinical evidence of congestive heart failure in a patient with 8.5 gm. per dl. hemoglobin. c. Premature atrial or ventricular contractions on electrocardiogram. d. A harsh aortic systolic murmur. e. Age over 70 years A 69-year-old male is due to have a laryngectomy and left radical neck dissection for -26 squamous carcinoma of the larynx. Prior to the operation the surgeon explains that he will have some drooping of the left shoulder with weakness of shoulder elevation as a consequence ?of the surgery. Which nerve will be sacrificed during the operation a. The cranial root of the accessory nerve b. The great auricular nerve c. The spinal root of the accessory nerve d. The supraclavicular nerve
403
e. The transverse cervical nerve 27- Treatment of diaphragmatic injuries a. Small defects less than 2 cm can be left alone b. Absorbable suture (0) is used in repair c. Exploration on left side start by dividing of lienophrenic ligament, and mobilization of splenic flexure down d. video assisted thoracic surgery VATS has no rule in treating diaphragmatic injuries e. None of the above 28- Which of the following statements regarding whole blood transfusion is correct? a. Whole blood is the most commonly used red cell preparation for transfusion in the b. Whole blood is effective in the replacement of acute blood loss. c. Most blood banks have large supplies of whole blood available. d. The use of whole blood produces higher rates of disease transmission than the use of individual component therapies. e. Old Whole blood is effective in the replacement of platelets. 29- Indication for emergency operation in penetrating abdominal injuries include all except a. Hemodynamic liability b. blood from any natural orifice c. Evisceration d. Peritoneal signs e. Knife injury 30- Acute cholecystitis all are true except a. Commonest bacteria is E .coli b. Wall thickness more than 3mm by ultrasound c. WCC is between 10-15 000 cell/mm3 d. Mild elevated bilirubin may accompany it e. HIDA scan has no role in diagnosis of acute cholecystitis 31. Anaphylactic shock: a. is an immune-mediated reaction. b. results in mast cell activation and increased circulating histamine concentrations. c. produces microcirculatory changes similar to hypovolaemic shock. d. requires prompt treatment with parenteral adrenaline and hydrocortisone. e. may occur after ingestion of drugs 32. A 65-year-old female has a positive stool occult blood test on a routine physical examination. A barium enema reveals numerous 1-cm outpouchings of the sigmoid and descending colon. Which of the following complications is most likely to occur from her colonic disease? a. Adenocarcinoma b. Pericolic abscess c. Bowel obstruction d. Malabsorption e. Toxic megacolon 33- Regarding colonic polyps all of the following are true except: a. Are associated with colonic cancer. b. May be hereditary. c. Should not be removed if they are asymptomatic. d. May be hyperplastic. e. Are commonly adenomatous 34. A 51-year-old male experiences the sudden onset of massive emesis of bright red blood. There have been no prior episodes of hematemesis. He is known to be hepatitis B surface antigen positive. His hematemesis is most likely a consequence of which of the following abnormalities of the esophagus? a. Varices
404
b. Barrett esophagus c. Candidiasis d. Reflux esophagitis e. Squamous cell carcinoma 35- Regarding basal cell carcinoma safety free margin is at least in most cases a. 2mm b. 3mm c. 2cm d.3cm 36. A 61-year-old male has had ascites for the past year. After a paracentesis with removal of 1 L of slightly cloudy, serosanguinous fluid, physical examination reveals a firm, nodular liver.Laboratory findings include positive serum HBsAg and presence of hepatitis B core antibody. He has a markedly elevated serum alpha-fetoprotein (AFP) level. Which of the following hepatic lesions is he most likely to have? a. Hepatocellular carcinoma b. Massive hepatocyte necrosis c. Marked steatosis d. Wilson disease e. Autoimmune hepatitis
37. The following statements about the repair of inguinal hernias are true except: a. The conjoined tendon is sutured to Cooper's ligament in the Bassini hernia repair . b. The McVay repair is a suitable option for the repair of femoral hernias. c. The Shouldice repair involves a multilayer, imbricated repair of the floor of the inguinal canal. d. The Lichtenstein repair is accomplished by prosthetic mesh repair of the inguinal canal floor in a tension-free manner. e. The Cooper's ligament repair is a suitable option for the children. A 76-year-old man presents with weight loss, dark urine, and pale stools which are -38 ?difficult to flush away. An excess of which of the following would account for this history a. Conjugated bilirubin b. Hyperbilirubinaemia c. Stercobilinogen d. Unconjugated bilirubin e. Urobilinogen 39- Choose the correct answer: Necrotizing fasciitis all are true except b. The majority is caused by single microbial infection c. Meleleny's gangrene is one of its types d. Treatment is urgent surgical intervention e. There is minor role for IV antibiotics f. Hyperbaric oxygen has little role in management 40- A 33-year-old woman develops a reducible mass of the groin that is inferolateral to the pubic tubercle and medial to the femoral vein. Which of the following is the most likely diagnosis? a. Direct inguinal hernia b. Femoral hernia c. Incisional hernia d. Indirect inguinal hernia e. Umbilical hernia 41- Major burn is defined as one of the following except. a. Second and third degree of greater than 15% in children
405
b. Second and third degree of greater than 20% in adults less than 50 years c. Third degree burn greater than 5% d. Inhalation injury e. Significant electrical injury. 42- A 22-year-old male presents into the ER after sustaining a stab wound to the left chest. The injury is 2 cm left of the sternum at the level of his nipple. Initial vital signs include a pulse rate 88 /min, GCS 15, respiratory rate 12 breaths/min and blood pressure 139/74 mmHg. Initial management of this patient should be: a. Chest X-R b. pericardiocentesis c. left chest tube thoracostomy d. esophagogram e. echocardiogram 43- The following statement is true concerning umbilical hernias in adults.
44-
a. Most umbilical hernias in adults are the result of a congenital defect carried into adulthood b. A paraumbilical hernia typically occurs in multiparous females c. The presence of ascites is a contraindication to elective umbilical hernia repair. d. Incarceration is uncommon with umbilical hernias e. Most umbilical hernias in adults are treated conservatively. A 48-year-old woman maintained on Warfarin for a history of cardiac valvular replacement and a history of recent upper respiratory infection presents with severe abdominal pain exacerbated by movement. Her physical examination shows tenderness in the right paramedian area with voluntary guarding but no peritoneal signs. The a.
Urgent laparotomy should be performed because of concern for arterial mesenteric embolus b. The correct diagnosis could likely be made by CT scan and operation avoided c. The status of her anticoagulation should be checked and if her prothrombin time is excessively prolonged, correction is necessary d. If untreated, hemodynamic instability is uncommon. F. Elective laparotomy should be performed 45- A 22-year-old male presents to the emergency depart ment after sustaining a gunshot wound to the right upper quadrant of his abdomen. On arrival, the patient is lethargic. His vital signs are significant for a systolic blood pressure of 85 mmHg with a heart rate of 130. As part of his therapy, fluid resuscitation is initiated. Which of the following is currently considered the best resuscitation fluid? a. Hartmann's solution b. Albumin c. Dextran d. 5% dextrose in 0.45% sodium chloride e. 7.5% sodium chloride 46-Acute pancreatitis a.Serum calcium start to rise after 48 hours b.Hypoglycaemia is bad prognostic factor c.Age is an important prognostic factor d.Serum amylase is more specific than serum lipase e.Severe pancreatitis compromise around 40% of cases 47- The diagnosis of acute appendicitis is most difficult to establish in: a. Persons aged 60 and older. b. Women aged 18 to 35.
406
c. Infants younger than 1 year. d. Pregnant women. e. Children 48- Extremity compartment syndrome a. Pain out of proportion is the most important symptom b. Pulse disappear early c. Treatment is usually conservative d. There is usually no sensory changes e. None of the above 49- All of the following are potential outcome in laparoscopic surgery EXCEPT : a. Hypercarbia b. Alkalosis c. Decrease urine out put d. Increase intracranial pressure e. Increased cardiac wor 50- The best type of x-ray to locate free abdominal air is: a. A posteroanterior view of the chest. b. A flat and upright view of the abdomen. c. Computed tomograph (CT) of the abdomen. d. A lateral decubitus x-ray, right side up e. MRI 51- Regarding papillary thyroid cancer f. Account for 40% of thyroid cancer g. Early blood spread h. Commonest thyroid cancer in children i. Common in endemic goiter areas j. Multifocality is rare 52- The most commonly used imaging method for diagnosis of acute cholecystitis is: f. CT of the abdomen. g. Ultrasonography of the gallbladder. h. Oral cholecystogram. i. Radionuclide (HIDA) scan of the gallbladder j. MRI 53- All of the following indicates nonresectable lung cancer EXCEPT: a. Isolated cerebral metastasis. b. Tumors of the apex of the lung c. Subcarinal lymph node more than 1cm in PET (positron emission tomography) scan. d. Malignant pleural effusion. e. Pco2> 45 mmHg 54- True statements regarding the pathophysiology of acute appendicitis include which of the following: a. Fecaliths are responsible for the disease process in approximately 30% of adult patients b. Lymphoid hyperplasia is a rare cause of appendicitis in young patients c. Clostridium difficile is implicated as a pathogenic organism d. Carcinoid tumors account for approximately 5% of all cases of acute appendicitis e. Adenocarcinoma account for approximately 20% 5% of all cases of acute appendicitis 55- True statements regarding appendiceal neoplasms include which of the following? a. Carcinoid tumors of the appendix less than 1.5 cm are adequately treated by simple appendectomy b. Appendiceal carcinoma is associated with secondary tumors of the GI tract in up to 60% of patients
407
c. Survival following right colectomy for a Dukes’ stage C appendiceal carcinoma is markedly better than that for a similarly staged colon cancer at 5 years d. Mucinous cystadenocarcinoma of the appendix is adequately treated by simple appendectomy, even in patients with rupture and mucinous ascites e. Up to 50% of patients with appendiceal carcinoma have metastatic disease, with the liver as the most common site of spread 56- Fibro adenoma f. Is common in females between 15-25 years g. Usually soft indiscreet lump h. Malignant changes in 5 % i. Arise from duct of breast j. none of the above 57- In chest injury the following are indications of thoracotomy EXCEPT : a. When cardiac tamponade present. b. Massive air leak c. For all transmediastinal wounds d. Flail chest with sever lung contusion e. Clotted hemothorax
58- Right side colon cancer usually present as all except a. Microcytic hypo chromic anemia b. Large bowel obstruction c. Appendicitis like symptoms d. Mass in right iliac fossa e. Small bowel obstruction 59- Acute Epidural haematoma : a. due to meningeal artery tear b. B. can be treated conservatively c. C. classically there is contra lateral, dilated, fixed pupil d. D. needs operation within 24 hours e. a and c 60- A 12-year-old boy is admitted to the hospital with severe abdominal pain. He is noted to have slight jaundice . His hematocrit is 30, and reticulocytes are evident in a peripheral smear. His father underwent a splenectomy at age 25. Which test would clarify the cause of anemia ? a. Barium enema . b.Hemoglobin electrophoresis . c. Serum iron . d.Coombs' test . e. Red blood cell ( RBC ) osmotic fragility test . 61- Beurger' diseases has : a. occlusion of small and medium arteries b. thrombophlebitis c. Reynold's phenomenon d. all the above 62- IN groin hernias a. Femoral hernia is more common than inguinal hernias in females b. Sliding hernia means part of the sac is formed by a viscus c. Obstruction is more common in direct than indirect hernia d. Deep ring is an opening in fascia transversalis below pubic tubercle e. Posterior wall of the inguinal canal is formed by inguinal ligament
408
63- Pathological breast pain is a. Bilateral b. Symmetrical c. Non cyclic d. Diffuse e. Upper outer quadrant 64- Colon cancer can causes all except a. Change in bowel habbit b. Elevtion of CA 19.9 level c. Fecal occult blood +ve d. Elevation of CA 15.3 level e. Iron deficiency anaemia :Goodsall`s rule is -65 a. showed the location of perianal abscess b. showed the perianal fistulas tract and location c. about the out put of the fistula d. site of gall stone ilieus e. the grading of anal cancer
66- To confirm endotracheal intubations in emergency room all are true except a. Equal bilateral breath sounds. b. Pulse oximeter c. No noise over epigastrium. d. Chest x ray. e. CO 2 detector 67- Fasciotomy for extremity compartment syndrome should be performed at a compartment pressure exceeding : a. 20 mmHg b. 30 mmHg c. 40 mmHg d. 50 mmHg e. none of the above 68- Rectal injuries RI all are true except a. Intraperitoneal rectal injuries can be managed as colon injuries b. Extraperitoneal rectal injuries usually need proximal diversion c. Posterior surface of rectum is Extraperitoneal d. Drainage of presacral space ,distal irrigation is always part of RI treatment e. Defect in Extraperitoneal rectum 69- Causes of Atelctasis in post operative period a. Pain from abdominal wound b. Inadequate analgesia c. No post opetative chest physiotherapy d. Upper abdominal incision e. All of the above 70- Regarding abdominal vascular injuries AVI a. The greater omentum is an important landmark in AVI b. Superior mesenteric artery can be ligated c. Midline hematoma below transverse mesocolon arise from aorta or IVC d. Pelvic hematoma in penetrating abdominal injuries showed not be explored e. None of the abov 71- Regarding femoral hernia a. Difficult to differentiate it from inguinal lymph node
409
b. Femoral vein form medial boundary c. More common than inguinal hernia in female d. Usually is maydal's type e. Represent 30% of all hernias 72- A 48-year-old man presents with an intraa-dominal abscess consequent to leakage form a bowel anastomosis . Where and why does mortality due to postoperative sepsis with intraabdominal abscess increase ? a. In younger patients . b. When a single organ is involved . c. By precutaneous drainage . d. If multiple abscesses occur . e. All of the above . 73- Reggarding massive blood transfusion MBT a. is defined as transfusion of half blood volume in 6 hours b. blood banks usually release for each 10 units of O-ve blood, two units of O FFP and five units o random donor platelets c. Metabolic complication of MBT include hypocalcemia,iron deposition and Hypokalemia d. Direct effect MBT include dilutional coagulopathy, volume over load and hypothermia e. MBT lead to citrate toxicity and acidosis 74- A 23-year-old male presents to the emergency depart ment after being involved in a motor vehicle accident. On physical examination, he opens his eyes spontanuously, he occasionally mumbles incomprehensible sounds, he localizes to painful stimulation with his right upper extremity, His pupils are 4 mm bilaterally and reactive. This patient’s Glasgow Coma Scale (GCS) score: a. 7 b. 9 c. 8 d. 10 e. 12 75- Which of the following findings suggests that shock in an injured patient may have a cause other than hypovolemia: a. hypotension b. distended neck veins. c. decreased skin temperature. d. diminished pulse pressure. e. falling central venous pressure. 76- Ulcerative colitis a. Is confined to small and large bowel b. Sigmoid colon is most area affected c. affect mucosa and sub mucosa d. Anal involvement is common e. Obstructive symptoms is common 77- A 43-year-old man had a previous injury to his wrist .The ulnar nerve was severed, as indicated by which of the following ? a. Claw hand involving the ring and little fingers . b. Claw hand involving the index and middle fingers . c. Atrophy of the thenar muscles . E d. Absent sensation in the index finger . e. Inability to flex the distal phalanx of the index finger . 78- Hypokalemia
410
a. Serum k+ less than 3.8 mmol/ liter b. Hypokalemia causes acidosis c. Alkalosis cause Hypokalemia d. ECG changes include wide QRS complex e. Causes exaggerated tendon reflexes 79- Which of the following is not a bad sign in a patient with abdominal pain a. Pallor b. Hypotension c. Patient lying still d. Jaundice e. Tachycardia 80- Active immunization in case of tetanus: a. Antitetanus human serum. b. Gives short period of protection. c. Given in case proved tetanus. d. Use of immunoglobulin. e. None of the above. 81- A 65 year old man with history of chronic constipation, has had loose bowel motion fever, and left lower abdominal pain for two days, the most likely source of abdominal pain is a. Perforated ulcer b. Diverticulitis c. Cholecystitis d. Perforation following bowel obstruction e. Perforated appendicitis 82- Regarding femoral canal all are true except: a. Lies lateral to the femoral vein. b. Has the inguinal ligament as its anterior border. c. Has the lacunar ligament as its medial border. d. Has the pectineal ligament as its posterior border. e. Contains the lymph node of Cloquet 83- Commonest cause of low grade fever 48 h post operation is a. Urinary tract infection b. Wound infection c. Deep venous thrombosis d. Atelectasis e. Pulmonary embolism 84- Most common early complication of central venous line is: a. Sepsis b. Pneumothorax c. Thoracic duct injury d. Thrombosis e. subcutaneous hematoma 85- How many mil mol of Na+ in a liter of ringer lactate Hartmann's a. 100 mil mol b. 110mil mol c. 154 mil mol d. 30 mil mol
411
e. 130 mil mol 86- 500cc of inter lipid 20% provide a. 450 k cal b. 900 k cal c. 400 k cal d. 450 cal e. 900 cal 87- In tension pneumothorax a. One view chest x-ray is needed b. Causes hypertension c. Causes congested neck veins d. Large sucking chest wound is an important cause e. none of the above 88- Regarding Hydatid disease: a. Is due to Ecchinococcus granulosa. b. Man is an accidental intermediate host. c. The liver is the commonest site of infection. d. Can be diagnosed by the Casoni test. e. All are true. 89- 44 year old male with abdominal gunshot ,he is hemodynamic stable, exploration showed two small bowel injuries 7 cm apart with 70% destruction of bowel wall, and through and through injury to ascending colon with 30% destruction of bowel wall, he should be managed by a. Primary repair of small and large bowel injury b. Primary repair of both injuries +dysfunctiong loop ileostomy c. Resection of small bowel injury, and right hemicolectomy for colon injury d. Resection of small bowel injury +primary repair of colon injury e. Resection of small bowel +exterorisation of colon injury as colostomy 90- Regarding local anaesthesia: a. Local anaesthetics act on small before large nerve fibres b. Adrenaline reduces absorption and prolongs the local effects c. Preparations containing adrenaline are safe to use on digits and appendages d. Lignocaine has a longer duration of action than bupivicaine. e. All are false 91- Regarding water loses in the body per day a. Insensible loss is 600-800 ml b. Average stool loss is 100 ml c. Average urine volume is 2500 ml d. Daily water consumption is 3500 ml e. Oxidation of fat loss is 100 ml 92- In post operative DVT, the following are true except: a. Clinical DVT occures in the 4th post operative day. b. If complicated by pulmonary embolism, it occures usually after the 7 th post operative day. c. The process of DVT starts preoperatively with the induction of anaesthesia .
412
d. When discovered we should start the patient on coumadin "Warfarin anticoagulation". e. It may lead to chronic venous in suffering as a complication of DVT. 93- Regarding asymptomatic gall bladder stones a. Patient showed be advised to have cholecystectomy as soon as possible b. The incidence of developing symptoms is 2% per year c. Patient showed have ultrasound over gallbladder every 3 months d. Increase incidence of gall bladder cancer e. none of the above 94- Complication of undescended testis include all of the following except : a. Malignant degeneration. b. Increased susceptability to trauma. c. Increased spermatogenesis. d. More liable to testiculer torsion. e.
Psychological complication
95- Painless haematuria is the leading presentation of : a. Renal cell carcinoma. b. Transitional cell carcinoma of the bladder . c. Ureteric stone. d. Pelvi-ureteric obstruction. e. Ureterocele 96- Regarding gastric adenocarcinoma a. Represent 60% of gastric neoplasm b. Diffuse type is usually in pyloric region c. Intestinal type has better prognosis d. HLO has no effect on gastric adenocarcinoma e. Diet has no rule in its etiology 97- Neonatal duodenal obstruction: a. May be associated with down's syndrome. b. Is more frequently found in premature infants. c. Typically presents with gross abdominal distension. d. Usually presents with vomiting of non-bile stained fluid e. B&C only. 98- A30 year old male with stab wound to right mid infraclavicular area, had weak pulse in ambulance 10 minute ago, arrived in casualty no pulse, or blood pressure, pupils are reactive, the initial surgical approach involve a. Median sternotomy b. Right side clavicular incision c. Right sided antrolateral thoracotomy d. Right sided postrolateral thoracotomy e. Left sided antrolateral thoracotomy 99- In acute superior mesenteric artery occlusion, all of the following are correct except : a. Characteristically presents with sudden pain and tenderness of increasing intensity. b. Is frequently accompanied by overt or occult blood loss in the stools. c. Frequently produces peritonitis. d. usually diagnosed on plain abdominal x-rays. e. Can be diagnosed by mesenteric artery ongiography. 100- Screening for colorectal cancer include all except a. Fecal occult blood
413
b. c. d. e.
Tumor marker CEA,CA 15.3 Double contrast barium enema Colonoscopy Virtual colonoscopy THE BREAST
1) The breast :
F) is a modified apocrine sweat gland G) overlies the third to the sixth rib H) is having usually more than 30 lacteferous ducts I) is drained only by six groups of axillary lymph nodes J) is having no attachment to skin 2) The commonest cause of bloody nipple discharge is : A) mammary ductectasia B) carcinoma of the breast C) lactational mastitis D) duct papilloma E) fibrocystic disease of the breast 3) All of the following may be manifestations of breast carcinoma EXCEPT A) peau d orange of covering skin B) no mass may be felt clinically C) enlargement of the ipsilateral axillary lymph nodes D) cervical spine metastasis E) none of the above 4) Bacterial mastitis: A) never occurs in lactating mothers B) is usually caused by streptococcu hemolyticus C) is mostly caused by staphylococcus aureus D) is treated only by incision and drainage E) is the same as mastitis of puberty 5) Signs of malignancy on mammogram may include all of the following EXCEPT : A) mass lesions with poorly defined irregular margins B) fine stipped soft tissue with periductal and not vascular microcalcifications C) thickening and retraction of the overlying skin D) dysplastic ductal pattern E) well circumscribed , homogenous,and often surrounded by a zone of fatty tissue 6) Early detection of carcinoma of the breast includes all of the following EXCEPT: A) self-examination just after menstruation B) screening mammography C) frequent consultations among the high risk group patients
414
D) presence of fixed breast mass to chest wall with skin changes on clinical examination E) A, B, C, are correct 6) Regarding Paget s disease of the nipple : A) It is a benign condition B) It is simply an eczematous lesion of the nipple C) It is treated by excision of the nipple D) It is usually diagnosed by biopsy of the suspected lesion E) the areola and the surrounding skin are never involved 7) in a patient with fibroadenosis of the breast ; A) cyst formation, adenosis, fibrosis,epitheliosis and papillomatosis are invariably present B) is premalignant C) pregnancy usually produces relief D) presentation may include pain, nipple discharge, and/ or breast lump E) all of the above are usually characteristics 8) Regarding gynaecomastia : A) it is hypertrophy of female breast B) it is hypertrophy of male breast C) it may associate leprosy and liver failure D) it may associate cimetidine, spironolactone, INH, or digitalis thrapy E) B, C, and D are correct 9) Regarding modified radical mastectomy all are correct EXCEPT : A) it is indicated recently for T1N0M0 B) pectoralis major muscle is excised C) axillary clearance is mandatory D) adjuvant radiotherapy must be given for all patients post-operatively E) the long thoracic nerve of Bell (nerve to serratous anterior) must be prsesrved but nerve to latismus dorsi might be sacrified 10) Mammary duct ectasia is characterized by the following EXCEPT : A) is defiened as primary dilatation of major ducts of breast in middle aged women B) may present with nipple retraction and Peau d orange picture C) is treated usually by simple mastectomy D) anaerobic superinfection cmmmonly occurs in this recurrent periductal plasma cell mastitis E) is commonly pre-malignant 11) Regarding carcinoma of the breast : A) invasive intraductal carcinoma is the commonest form B) lobular carcinoma may present bilateral C) clinical staging is always correct and definit D) the medullary (anaplastic) type feels hard
415
E) A, and B, are correct
Choose the Best appropriate answerfor each of the following questions: 1.The ilio-inguinal nerve: A. supplies the rectus abdominis muscle B. supplies skin on inner side of thigh C. supplies the cremasteric muscle D. supplies the urethra E. does none of the above 2. The skin of the tip of the index finger is supplied by the: K. Radial nerve only L. Radial & median nerves M. Median & ulnar nerves N. Ulnar nerve only O. Median nerve only 3. Hypovolaemic shock is characterized by: P. A low central venous pressure , high cardiac output, low peripheral resistance Q. A high central venous pressure, high cardiac output, high peripheral resistance R. A low central venous pressure , low cardiac output, high peripheral resistance S. A low central venous pressure , high cardiac output, high peripheral resistance T. A high central venous pressure, low cardiac output, low peripheral resistance 4. Which of the following in NOT true of Hodgkin's disease? K. Usually starts from several groups of nodes simultaneously L. Usually involves liver & spleen M. Sometimes manifests itself as pyrexia of unknown origin N. Severe pain follows ingestion of alcohol
416
O. Shows increased susceptibility to opportunistic infection 5. Tetanus toxoid: I.
Is produced by injecting animals with antititanic serum
J. Is administered to previously immunized subjects with potentially infected wounds K. Frequently gives rise to anaphylactic reaction L. Is used to induce active immunity against tetanus 6. The most probable cause of shock in a patient with multiple injuries & craniocerebral trauma is: P. Depression of vital medullary centres Q. Hypoperfuion control over subcortical areas R. Loss of cortical control over subcortical areas S. Hypovolaemia T. Inadequate ADH response
7. The most sensitive guide to acute changes in fluid balance in a surgical patient is: P. Accurate daily weight Q. Serial serum Na concentration R. Fluid balance sheets recording inputs & outputs ??? S. Daily urine output T. Serial anion gap measurements 8. cellullitis is: K. Inflammation of the bone marrow L. Inflammation of the mastoid cells M. Inflammation of the subcutaneous tissues N. Infiltration of the skin by gaint cells O. A malignant condition
417
9. secondary haemorrhage occurs: P. Within 6 hours of operation Q. 7-14 days after operation R. As a result of violent coughing on recovery from anaesthesia S. Due to a blood transfusion line disconnected T. When a ligature slips 10. the minimum urine output for 24 hours required to excrete end products of protein metabolism is: K. 200 ml L. 300 ml M. 400 ml N. 500 ml O. 600 ml 11. Potassium deficiency is present if the plasma-potassium level is: K. 6.0 mmol/l L. 5.0 mmol/l M. 4.5 mmol/l N. 4.0 mmol/l O. 3.0 mmol/l 12.in health the pH of the blood lies between the range: K. pH 7.05-7.19 L.
7.20-7.35
M.
7.36-744
N.
7.45-7.59
O.
7.60-7.80
13. TNM classification of a malignant tumour was designed as: K. An histological staging
418
L. A clinical staging M. A staging carried out at operation N. A staging dependent upon radio scanning & skeletal survey O. A staging dependent upon ultrasound 14. a blue-green discharge from an ulcer will be seen to contain: K. Pseudomonas pyocyaneus L. Streptococcus viridians M. Candida albicans N. Staphylococcus aureus O. Haemophilius influenzae 15. a rodent ulcer is: K. A squamous cell carcinoma L. A basal cell carcinoma M. Only occur on the face N. Contains epithelial pearls O. A venous ulcer
16. the space of Parona is: K. In the wrist between the deep flexor tendons & the pronator quadratus L. Above the patella between the quadriceps muscle & the femur M. Benath the tendon of the iliopsos N. Between the achills tendon & the posterior aspect of the tibia O. The web space of the palm 17. 'rest pain' occurs: P. Anywhere in the body at rest Q. In the thigh of a patient with Buerger's disease
419
R. In the calf of a patient with intermittent claudicating S. In the foot of a patient with severe vascular disease T. In the back 18. ischaemia means: Q. Pain in the ischial tuberosities R. Anaemia due to malignant seconderies in the ischial part of the pelvis S. Lack of blood flow T. Increased blood flow U. Polycythaemia 19. Colles' fracture is: K. A common in adolescence L. A fracture about the ankle joint M. Common in elderly women (( above 60 due to osteporosis)) N. A fracture of the head of the radius O. A fracture of scaphoid 20. Pott's disease is; K. A fracture dislocation about the ankle L. A neuropathic joint M. Traumatic ostechondritis of the spine N. Tuberculosis of the spine O. A secondary tumour in the skull 21. Vincent's angina is a form of angina associated with: K. Spasm of the oesophagus L. Diphtheria M. An infection of the mouth N. Coronary artery spasm
420
O. Carcinoma of the bronchus 22. Ludwig's angina is due to : K. A type of coronary artery spasm L. Oesophageal spasm M. Retropharyngeal infection N. A virulent infection of the cellular tissue around the submandibular salivary gland O. Infection with candida 23. in simple nodular goiter: K. Carcinoma occurs in 30% of cases L. The nodular stage is irreversible (right ,,baily and love mcq) M. Operation is contraindicated N. The patient does not develop hyperthyroidism O. Cretinism is the presenting feature 24. Hashimoto's disease is: K. A granulomatous thyroiditis L. An auto-immune thyroiditis M. An infiltrating fibrosis of the thyrois & the adjacent muscles N. Focal thyroiditis O. A parathyroid tumour
25. A thyroglossal fistula: K. Is never congenital L. Follows inadequate removal of a thyroglossal cyst M. Has a hood of skin with its concavity upwards N. Is lined throughout by squamous epithelium
421
O. Occurs in carcinoma of the tongue 26. The following are clinical signs supporting an early diagnosis of carcinoma of the breast: K. A prickling sensation in a breast lump L. Peau d'ornge M. Brawny arm N. Cancer en cuirasse O. A krukenderg tumour 27.The gastroduodenal artery is a branch of the: K. Celiac axis L. Hepatic artery M. Superior mesenteric artery N. Gastroepiploic artery O. Splenic artery 28.Chronic gastric ulcers most often occur in patients with: K. Blood groub A L. Tend to occur in alkaline mucosa M. Muscularis mucosae is separated from the muscularis at the edge of the ulcer N. Are malignant when there is epithelial proliferation & downgrowths O. Are never large 29. Meckel's diverticulum: K. Is present in 20% of the human race L. Arises from the mesenteric border of the jejunum M. May contain heterotopic pancreas N. Is only present in the male sex O. Is a diverticulum of the bladder 30. Intussusception is related to:
422
K. Mucoviscidosis L. Swollen Peyer's patches M. Volvulus N. A littre's hernia O. A patent vitello intestinal duct 31. The site of the neck of a femoral hernia is the: K. Transversalia fascia L. Iliopectineal ligament M. Femoral ring N. Cribriform fascia O. Obturator forame 32. Regarding operation for an indirect inguinal hernia: K. It should not be performed on patients who have chronic bronchitis L. General anaesthesia has to be used M. In infants the posterior inguinal wall should be repaired N. In adults the internal inguinal ring usually needs to be strengthened O. Mesh implants are mandatory مفيد المخللتاي/أسئلة الدكتور 1. What is the accuracy of peritoneal lavage in determining that in-traperitoneal injury has occurred in blunt trauma patients? a. 15% b. 30% c. 50% d. 70% e. More that 85% 2. which of the following tests is MOST accurate in diagnosing suspected pancreatic injury? a. Elevated serum amylase. b. Computed tomography (CT) scan of the abdomen. c. Elevated white blood count (WBC). d. High urine amylase. e. High serum lipase. 3. A patient has been stabbed in the anterior abdomen. He has no intra-abdominal symptoms or signs. What is the MOST reliable diagnostic maneuver to rule out intra-abdominal injury? a. Peritoneal lavage.
423
b. c. d. e.
CT or the abdomen with contrast. CT of the abdomen without contrast. Wound exploration. Catheter contrast injection of the wound.
4. For which of the following organ injuries is diagnostic peritoneal lavage (DPL) LEAST likely to be helpful? a. Panarceas b. Small intestine c. Spleen d. Sigmoid colon e. Liver 5. All of the following findings suggest urethral injury EXCEPT a. Blood at the external urethral meatus. b. Scrotal hematoma c. Absence of a palpable prostate on rectal examination. d. High-riding prostate on rectal examination. e. Blood in the rectal lumen. 6. In cases of pancreatic injury, the finding listed below which indicates the MOST serious pancreatic injury is a. Serum amylase > 250 units b. Major pancreatic duct injury c. Contusion with major duct intact. d. Serum calcium <8 mg/dL e. Hyperglycemia 7. which is the MOST commonly injured intra-abdominal organ in blunt trauma? a. Liver. b. Kidney c. Spleen d. Stomach e. Colon 8. True statements associated with burns include all of the following EXCEPT a. Cardiac output initially falls in proportion to the area of the burn. b. Mortality is increased when accompanied by inhalation injury. c. Mortality expected from a 50% body surface area (BSA) burn is the same regardless of patient age. d. Pulmonary vascular resistance increases in the immediate postburn period. e. The cellular immune response is depressed after 9. Regarding electrical injury, all of the following statements are true EXCEPT a. Hyperkalemia may result from tissue necrosis. b. Myoglobin and hemoglobin pigment may produce renal failure. c. Heat is the principle mediator of tissue damage in high-volt-age injuries. d. heat production is related to tissue resistance increases in the immediate postburn period. e. The cellular immune response is depressed after burn injury. 10. Deep venous thrombosis prophylaxis is appropriate for all of the following patients EXCEPT a. A 67-year-old male undergoing a colectomy. b. A 21-year-old male undergoing an outpatient open inguinal hernia repair. c. A 21-yar-old male in the ICU, comatose, with a closed head injury.
424
d. A 60-year-old female undergoing open reduction and internal taxation (ORIF) of a hip fracture. e. A 53-year-old female undergoing resection of a lung carcinoma. 11. Techniques for diagnosis of deep venous thrombosis include all of the following EXCEPT. a. Phlebography b. Radiofibrinogen studies. c. Ultrasonic studies. d. Venous flow rate studies e. Electrical impedance test. 12. The definitive diagnosis of PE is best made by a. Ventilation prefusion (VQ) scan b. Depression of arterial O2 c. Pulmonary angiogram d. Duplex scan. 13. Deep venous thrombosis is MOST definitively diagnosed by a. I121 fibrinogen scan. b. CT scan c. Venogram d. Indium-labeled white cell scan. e. Doppler ultrasonography. 14. Complications of chronic varicose vein disease include. EXCEPT a. Eczema b. Lipodermatosclerosis c. Equinus deformity. d. Malignant transformation. e. Deep vein thrombosis 15. Burn injuries:a. If full thickness are normally sensitive to pin prick. b. Affecting a whole lower limb in a child will represent approximately 18% suface area. c. Are associated with Pseudomonas infections. d. Is associated with a low mortality in the elderly e. Are associated with nitrous oxide toxicity Dr.Ashraf Obaid 1. Which hypersensitivity reaction is associated with a tuberculin reaction? A. Type I: immediate B. Type II: cytotoxic C. Type III: immune complex D. Type IV: cell mediated 2. The most common location for a gastric ulcer is A. Fundus B. Greater curvature C. Cardia D. Body E. Antrum 3. Regarding the management of major trauma A. Deaths follow a trimodal distribution B. Cardiac tamponade is characterised by raised BP, low JVP and muffled heart sounds C. Assessment of uncomplicated limb fractures should occur during the primary survey
425
D. Deterioration of the casualty during the primary survey should lead to the secondary survey E. All are false 4. All of the following are true about neurogenic shock except: A. There is a decrease in systemic vascular resistance and an increase in venous capacitance. B. Tachycardia or bradycardia may be observed, along with hypotension. C. The use of an alpha agonist such as phenylephrine is the mainstay of treatment. D. Severe head injury, spinal cord injury, and high spinal anesthesia may all cause neurogenic shock. 5. The following cause hypercalcaemia except : A. Sarcoidosis B. Primary hyperparathyroidism C. Acute pancreatitis D. Metastatic bronchial carcinoma E. Milk-Alkali syndrome 6. For a 40-kg baby the maintenance daily fluid requirement is approximately which of the following? A. 1100 ml B. 1250 ml C. 1550 ml D. 1700 ml E. 2000 ml 7. Infantile hypertrophic pyloric stenosis A. Occurs with a male: female ratio of 4:1. B. Presents between six and eight months of age C. Typically presents with bile stained projectile vomiting D. Surgical treatment is by Heller's Cardiomyotomy E. all are false 8. Which of the following do you consider to be the most important clinical sign in acute appendicitis, A. Abdominal tenderness around the umbilicus B. Abdominal tenderness in the RIF C. Tenderness over McBurney’s point D. Rovsing’s sign positive E. Suprapubic tenderness 9. Heparin A. Acts as an anti-platelet B. Acts as an anti-thromboplastin C. Acts as an antithrombin D. All of the above E. None of the above 10. All of the following are mechanisms of urinary calculi formation except, A. Hypoparathyroidism B. Prolonged recumbency C. Infection with urea-splitting organisms D. Foreign body E. Urinary tract obstruction
11. Which of the following are not found in peritonitis? A. Patient is lying still B. Guarding C. Rebound tenderness
426
D. Hyperactive bowel sounds E. Rigid abdomen 12. Which one of the following suggest a diagnosis of Hirschsprung's disease? A. A contrast-study showing dilatation of the aganglionic bowel segment. B. Early presentation with vomiting. C. Neonatal large bowel obstruction. D. Presentation after 1 year of age. E. Red current jelly stools. 13. Which of the following regarding the anatomy of the heart is true? A. The aortic valve is tricuspid. B. The ascending aorta is entirely outside the pericardial sac. C. The left atrial appendage is identified readily by transthoracic echocardiography. D. The pulmonary trunk lies anterior to the ascending aorta. E. The right atrium is posterior to the left atrium. 14. Which of the following is true concerning Scaphoid fractures? A. Rarely occur in young adults B. when complicated by avascular necrosis the proximal pole is usually affected C. should be treated by bone grafting and internal fixation even if undisplaced D. wrist fractures are uncommon E. anteriorposterior and lateral radiographs reveal most fractures 15. Which of the following statements is true of upper limb nerve injuries? A. Injury to the median nerve results in a wrist drop B. Injury to the radial nerve results in loss of sensation over the palmar aspect of the index finger C. Injury to the median nerve results in loss of sensation in the anatomical snuffbox D. Injury to the ulnar nerve results in a claw hand E. Injury to the ulnar nerve results in loss of sensation over the thumb 16. Regarding intravenous solutions: A. Normal saline contains 180mmol/l of sodium B. Ringer's lactate solutions is designed for intracellular fluid replacement C. Sodium bicarbonate 8.4% is a hyperosmolar solution D. Normal saline with added potassium is appropriate therapy to correct a respiratory alkalosis E. All are True 17. Which of the following concerning the Femoral sheath is false: A. Contains the femoral artery B. Contains lymph nodes C. Contains the femoral canal D. Contains femoral nerve E. Contains the femoral vein 18. The following is true of the spleen: A. Is the largest lymphoid organ in the body B. Lies obliquely between the seventh and tenth rib C. The lower pole extends beyond the mid-axillary line D. Is usually palpable when normal E. Usually measures 16cm in maximum length when healthy 19. Breast cancer risk is increased in association with the following factors except: A. Nulliparity B. Immediately after pregnancy
427
C. Early menarche D. Early age at first pregnancy E. Late menopause 20. In tension pneumothorax the following signs are present except: A. Hypoxia B. Hyperresonance to percussion on the affected side C. Tracheal deviation to the ipsilateral side D. Distended neck veins E. Tachycardia 21. The most common hernia in females is: A. Femoral hernia. B. Direct inguinal hernia. C. Indirect inguinal hernia. D. Obturator hernia. E. Umbilical hernia. 22. The most helpful diagnostic radiographic procedure in small bowel obstruction is: A. CT of the abdomen. B. Contrast study of the intestine. C. Supine and erect x-rays of the abdomen. D. Ultrasonography of the abdomen. E. MRI Abdomen 23. In role of nine extent of burn if entire trunk is burned it will be equal to: A. 9% body surface area. B. 18% body surface area. C. 36% body surface area. D. 27% body surface area. E. 45% body surface area. 24. If torsion of the testicle is suspected, surgical exploration: A. Can be delayed 24 hours and limited to the affected side. B. Can be delayed but should include the asymptomatic side. C. Should be immediate and limited to the affected side. D. Should be immediate and include the asymptomatic side. 25. Hyperthyroidism can be caused by all of the following except: A. Graves' disease. B. Plummer's disease. C. Struma ovarii. D. Hashimoto's disease. E. Medullary carcinoma of the thyroid. 26. A 9 month old boy presents with an acute scrotal swelling. The following diagnoses are likely: A. Epididymitis B. Orchitis C. Torsion of the testicular appendage D. Irreducible inguinal hernia E. Acute idiopathic scrotal oedema 27. The evaluation of a comatose patient with a head injury begins with: A. The cardiovascular system. B. Pupillary reflexes. C. Establishment of an airway.
428
D. Computed tomography (CT) of the brain E. insertion of Intravenous canula 28. The following is an indication for thoracotomy in chest injury, A. Cardiac tamponade B. Uncontrolled pulmonary air leakage C. Perforation of thoracic esophagus D. Blood loss of 200ml/hr for 2-3 hrs via chest tube E. All of the above 29. Regarding Gallstones all of the following are true except:A. Prevalence increases with advancing age B. 30% of gallstones are radio-opaque C. Cholesterol stones result from a change in solubility of bile constituents D. Biliary infection, stasis and changes in gallbladder function can precipitate stone formation E. Gram-negative organisms are the most common isolated 30. In obstructive jaundice: A. Urinary conjugated bilirubin is increased B. Serum unconjugated bilirubin is increased C. Urinary urobilinogen increased D. Serum conjugated bilirubin is reduced E. Faecal stercobilinogen is increased 31. The ideal fluid therapy in a patient with pyloric stenosis and repeated vomiting is: A. Isotonic crystaloid containing sodium chloride B. Hypertonic crystaloid containing dextrose – saline C. Isotonic solution containing dextrose D. Large molecular weight colloid containing dextran 32. Which of the following do you consider to be the most important clinical sign in acute appendicitis, A. Abdominal tenderness around the umbilicus B. Abdominal tenderness in the RIF C. Tenderness over McBurney’s point D. Rovsing’s sign positive E. Suprapubic tenderness 33. All of the following are mechanisms of urinary calculi formation except, A. Hypoparathyroidism B. Prolonged recumbency C. Infection with urea-splitting organisms D. Foreign body E. Urinary tract obstruction 34. Which of the following is incorrect in Paget’s disease of nipple, A. Presents with eczema and redness of the nipple and areola B. Tumour arises from epidermal layer of nipple and not from epidermal layer of underlying ducts C. Underlying carcinoma may be present beneath the nipple D. Mastectomy is considered the treatment of choice E. It is considered carcinoma-in-situ 35. Organisms most commonly isolated in UTIs, A. Kleb B. E. Coli C. Proteus
429
D. Acinetobacter E. All of the above 36. Excessive saliva in a newborn is due to A. Esophageal atresia B. Salivary gland tumour C. Primary Hypertrophic Pyloric Stenosis D. Midgut volvulus E. Hirschprung’s disease 37. The most frequent congenital diaphragmatic hernia seen in infants is, A. Paraesophageal hernia B. Sliding hernia C. Congenitally short esophagus D. Hernia through the foramen of Bochdalek E. Hernia through the foramen of Morgagni 38. An undescended testis. A. Can be associated with contralateral testicular dysfunction B. Should be treated by orchidopexy by age 2 C. Is associated with inguinal hernia D. Predisposes to malignant change E. All of the above 39. The wound made to remove a perforated appendix is classified as, A. Clean B. Clean, contaminated C. Contaminated D. Dirty E. Doesn’t apply as the wound isn’t a traumatic one 40. Concerning the treatment of haemorrhoids, A. Can be by diet B. By injection sclerotherapy C. By banding D. Is mainly by haemorrhoidectomy E. All of the above 41. The following is an indication for thoracotomy in chest injury, A. Cardiac tamponade B. Uncontrolled pulmonary air leakage C. Perforation of thoracic esophagus D. Blood loss of 200ml/hr for 2-3 hrs via chest tube E. All of the above 42. In an upper GI bleed, in some cases a Sengstaken-Blakemore tube is incerted. What is the primary purpose of the tube? A. Aspirate blood from the stomach B. Tube feeding C. Tamponade for varices D. To decompress bowel E. All of the above 43. Risk factors for pulmonary embolism do not include which of the following? A. DVT B. Recent surgery C. Old age
430
D. Myocardial infarction E. Chest infection 44. Which of the following is not found in peritonitis? A. Patient is lying still B. Guarding C. Rebound tenderness D. Hyperactive bowel sounds E. Rigid abdomen 45. Commonest site for CA breast A. Upper outer quadrant B. Upper inner quadrant C. Lower outer quadrant D. Lower inner quadrant E. None of the above 46. The most common site of fracture of the clavicle is: A. Medial end. B. Lateral end. C. Midpoint of the clavicle. D. Junction of the medial two-thirds and the lateral third. E. Junction of the lateral two-thirds and the medial third. 47. The back of the medial epicondyle is related to the: A. Radial nerve. B. Axillary nerve. C. Ulnar nerve. D. Median nerve. E. None of the above. 48. Regarding Gallstones all of the following are true except:A. Prevalence increases with advancing age B. 30% of gallstones are radio-opaque C. Cholesterol stones result from a change in solubility of bile constituents D. Biliary infection, stasis and changes in gallbladder function can precipitate stone formation E. Gram-negative organisms are the most common isolated 49. Clinical features of limb ischaemia includes A. Paraesthesia B. Pallor C. Pulselessness D. Paralysis E. All of the above 50. Regarding Hydatid disease all of the following are true except:A. Due to infection with the helminth Ecchinococcus granulosa B. Man is an accidental intermediate host C. Lunges is the commonest organ involved D. Diagnosis can be confirmed by indirect haemagglutinin assay E. Aspiration should not be performed if hydatid disease is suspected
431
1 2 3 4 5 6 7 8 9 10
D E A C C E A C C A
11 12 13 14 15 16 17 18 19 20
D C A B D C D A D C
21 22 23 24 25 26 27 28 29 30
C C C D D D C E B A
31 32 33 34 35 36 37 38 39 40
A C A A B A D E B E
41 42 43 44 45 46 47 48 49 50
E C E D A D C B E C
Surgery …….DR.marwan abu sa3da 2010 2- A 40-year-old female has a 4-cm hemangioma in the right lobe of the liver on computed tomography scan. She is asymptomatic. Appropriate action should be: (a) fine-needle biopsy (b) arrangement for elective resection (c) no further action (d) angiographic embolization 3- Regarding an amebic liver abscess: (a) surgical drainage is usually required (b) negative stool testing for amebiasis rules out the disease (c) it should be drained percutaneously under com- puted tomography guidance (d) it is treated with metronidazole 4- A single organism is usually the causative agent in: (a) pelvic inflammatory disease (b) perforated diverticulitis (c) acute cholecystitis (d) primary peritonitis (e) diabetic foot infections 9- Signs and symptoms of hemolytic transfusion reactions include a. Hypothermia b. Hypertension c. Polyuria d. Abnormal bleeding e. Hypesthesia at the transfusion site 10- In a hemolytic reaction caused by an incompatible blood transfusion, the treatment that is most likely to be helpful is: a. Promoting a diuresis with 250 ml of 50% mannitol b. Treating anuria with fluid and potassium replacement c. Acidifying the urine to prevent hemoglobin precipitation in the renal tubules d. Removing foreign bodies, such as Foley catheters, which may cause hemorrhagic complications e. Stopping the transfusion immediately
11- Which statements about extrahepatic bile duct cancer are correct?
432
A. Cholangiography is not essential in evaluating patients for resectability. B. The prognosis is excellent when appropriate surgical and adjuvant therapy are given. C. The location of the tumor determines the type of surgical procedure. D. The disease usually becomes manifest by moderate to severe right-side upper quadrant pain. 12- Which of the following statements about the diagnosis of acute calculous cholecystitis is true? A. Pain is so frequent that its absence almost precludes the diagnosis. B. Jaundice is present in a majority of patients. C. Ultrasonography is the definitive diagnostic test. D. Cholescintigraphy is not definitive diagnostic test. 13- Which statement about acute acalculous cholecystitis is correct? A. The disease is often accompanied by or associated with other conditions. B. The diagnosis is often difficult. C. The mortality rate is higher than that for acute calculous cholecystitis. D. The disease has been treated successfully by percutaneous cholecystostomy E- all are correct 14- Which of the following statements about laparoscopic cholecystectomy are correct? A. The procedure is associated with less postoperative pain and earlier return to normal activity. B. The incidence of bile duct injury is lower than for open cholecystectomy. C. Laparoscopic cholecystectomy should be used in asymptomatic patients because it is safer than open cholecystectomy. D. Pregnancy is a contraindication. 15- A 15-year-old female presents with RUQ abdominal pain. Workup reveals a choledochal cyst. Which of the following statements is TRUE? (A) Choledochal cysts are more common in men. (B) Laparoscopic cholecystectomy is the recommended treatment. (C) Patients with a choledochal cyst have an increased risk of cholangiocarcinoma. (D) All patients with a choledochal cyst have abdominal pain, a RUQ mass, and jaundice. (E) The etiology is infectious. 16- An 85-year-old man is brought to the hospital with a 2-day history of nausea and vomiting. He has not passed gas or moved his bowels for the last 5 days. Abdominal films show dilated small bowel, no air in the rectum and air in the biliary tree. Which of the following statements is TRUE? (A) Air in the biliary tree associated with small-bowel obstruction suggests a diagnosis of gallstone ileus. (B) An enterotomy should be distal to the site of obstruction and the stone should be removed. (C) Gallstone ileus is more common in the young adults. (D) Cholecystectomy is contraindicated. (E) Small-bowel obstruction usually occurs in the distal jejunum. 17- A45-year-old patient with chronic pancreatitis is suffering from malnutrition and weight loss secondary to inadequate pancreatic exocrine secretions. Which is TRUE regarding pancreatic secretions? (A) Secretin releases fluid rich in enzymes. (B) Secretin releases fluid rich mainly in electrolytes and bicarbonate. (C) Cholecystokinin releases fluid,predominantly rich in electrolytes, and bicarbonate. (D) All pancreatic enzymes are secreted in an inactive form. (E) The pancreas produces proteolytic enzymes only
433
18-A 43-year-old woman has gallstone pancreatitis that resolves in 2 days with conservative treatment. She has no abdominal complaints and her liver and pancreatic laboratory values have returned to normal. She is scheduled for laparoscopic cholecystectomy. Which of the following statements is TRUE? (A) Intraoperative cholangiography is associated with a decreased risk of biliary tract injury . (B) The procedure should be scheduled for 6 weeks after resolution of symptoms . (C) Intraoperative cholangiography in this patient will identify choledocholithiasis in 50% of cases . (D) Preoperative endoscopic retrograde cholangiopancreatography (ERCP) should be performed. (E) The sensitivity of magnetic resonance cholangiopancreatography (MRCP) for choledocholithiasis in this patient is less than 50%. 19-A 39-year-old woman is admitted with gallstone pancreatitis and epigastric pain. Pertinent data include amylase, 2000 U/L; bilirubin, 1.2 mg/dL; and WBC count, 15,000/mm3 . After 2 days of medical management, her epigastric pain resolves. Her amylase is 340 U/L and her bilirubin and WBC count have returned to normal. Laparoscopic cholecystectomy should be attempted: (A) after endoscopic retrograde cholangiopancreatography (ERCP) and sphincterotomy (B) prior to discharge (C) once her amylase is normal (D) 4 to 6 weeks later (E) only if the patient develops recurrent pancreatitis 20- A 48-year-old woman is admitted with acute cholecystitis. The bilirubin level is elevated, as are the serum and urinary amylase levels. Which radiologic sign indicates biliary obstruction in pancreatitis? (A) Pancreatic intraductal calcification (B) Smooth narrowing of the distal CBD غريب مش لقية حل غير هادا (C) Stomach displaced anteriorly (D) Calcified gallstone (E) Air in the biliary tree) 21. Following a motor vehicle accident a truck driver complains of severe abdominal pain. Serum amylase level is markedly increased to 800 U. Grey Turner’s sign is seen in the flanks. Pancreatic trauma is suspected. Which statement is true of pancreatic trauma? (A) It is mainly caused by blunt injuries. (B) It is usually an isolated single-organ injury. (C) It often requires a total pancreatectomy. (D) It may easily be overlooked at operation. (E) It is proved by the elevated amylase level. 22. A 73-year-old woman is evaluated for obstructive jaundice after an injury to the CBD, 7 months previously at laparoscopic cholecystectomy. The alkaline phosphatase is elevated. In obstructive jaundice, which of the following statements is true regarding alkaline phosphatase? (A) Its level increases before that of bilirubin. (B) Its level is unlikely to be increased in pancreatic malignancy. (C) Its elevation indicates bone metastasis. (D) Its elevation excludes hepatic metastasis. (E) Its level falls after that of the bilirubin,following surgical intervention.
434
23. A48-year-old female travel agent presents with jaundice. Radiological findings confirm the presence of sclerosing cholangitis. She gives a long history of diarrhea for which she has received steroids on several occasions. She is likely to suffer from which of the following? (A) Pernicious anemia (B) Ulcerative colitis (C) Celiac disease (D) Liver cirrhosis (E) Crohn’s disease 24. A38-year-old male lawyer develops abdominal pain after having a fatty meal. Examination reveals tenderness in the right hypochondrium and a positive Murphy’s sign. Which test is most likely to reveal acute cholecystitis? (A) HIDA scan (B) Oral cholecystogram (C) Intravenous cholangiogram (D) CT scan of the abdomen (E) ERCP 25. A 65-year-old woman is admitted with RUQ pain radiating to the right shoulder, accompanied by nausea and vomiting. Examination reveals tenderness in the RUQ and a positive Murphy’s sign. A diagnosis of acute cholecystitis is made. What is the most likely finding? (A) Serum bilirubin levels may be elevated. (B) Cholelithiasis is present in 40–60%. (C) Bacteria are rarely found at operation. (D) An elevated amylase level excludes this diagnosis. (E) A contracted gallbladder is noted on ultrasound. 26. A 32-year-old diabetic woman who has taken contraceptive pills for 12 years develops RUQ pain. CT scan of the abdomen reveals a 5-cm hypodense lesion in the right lobe of the liver consistent with a hepatic adenoma. What should the patient be advised to do? (A) Undergo excision of the adenoma (B) Stop oral contraceptives only (C) Stop oral hypoglycemic medication (D) Undergo right hepatectomy (E) Have serial CT scans every 6 months 27- A 9 month old boy presents with an acute scrotal swelling. The following diagnoses are likely: p. Epididymitis q. Orchitis r. Torsion of the testicular appendage s. Irreducible inguinal hernia t. Acute idiopathic scrotal oedema 28- A 76-year-old man presents with weight loss, dark urine, and pale stools which are difficult to flush away. An excess of which of the following would account for this history? a. Conjugated bilirubin b. Hyperbilirubinaemia c. Stercobilinogen d. Unconjugated bilirubin e. Urobilinogen
435
28- An otherwise well 13-year-old boy is admitted complaining of sudden onset severe left sided testicular pain 2 hours prior to admission. He gives no history of trauma, dysuria or frequency. On examination he is found to have a tender, high-riding testicle.What is the most appropriate next step in this young man’s management? a. Herniography b. Scrotal Doppler ultrasound on the next available list c. FBC and U&E d. Scrotal Doppler ultrasound as an emergency e. Surgical exploration of his scrotum Acute scrotum -29 f. Torsion testis should be operated within 12 hour of presentation g. Epidedimoorchitis pain increase by testicular elevation h. If in doubt scrotum should be explored i. Doppler ultrasound has no role in diagnosis j.None of the above 30- stones in the common bile duct: f. Are present in nearly 50 per cent of cases of cholecystitis. g. Often give rise to jaundice, fever and biliary colic. h. Are usually accompanied by progressive jaundice. i.
Are usually associated with a distended gallbladder.
j.
A&D only.
31- Which of the following statements regarding whole blood transfusion is correct? a. Whole blood is the most commonly used red cell preparation for transfusion in the b. Whole blood is effective in the replacement of acute blood loss. c. Most blood banks have large supplies of whole blood available. d. The use of whole blood produces higher rates of disease transmission than the use of individual component therapies. e. Old Whole blood is effective in the replacement of platelets. 32- Acute cholecystitis all are true except f. Commonest bacteria is E .coli g. Wall thickness more than 3mm by ultrasound h. WCC is between 10-15 000 cell/mm3
436
i.
Mild elevated bilirubin may accompany it
j.
HIDA scan has no role in diagnosis of acute cholecystitis
33. A 51-year-old male experiences the sudden onset of massive emesis of bright red blood. There have been no prior episodes of hematemesis. He is known to be hepatitis B surface antigen positive. His hematemesis is most likely a consequence of which of the following abnormalities of the esophagus? f. Varices g.
Barrett esophagus
h.
Candidiasis
i.
Reflux esophagitis
j.
Squamous cell carcinoma
34- A 61-year-old male has had ascites for the past year. After a paracentesis with removal of 1 L of slightly cloudy, serosanguinous fluid, physical examination reveals a firm, nodular liver.Laboratory findings include positive serum HBsAg and presence of hepatitis B core antibody. He has a markedly elevated serum alpha-fetoprotein (AFP) level. Which of the following hepatic lesions is he most likely to have? f. Hepatocellular carcinoma g.
Massive hepatocyte necrosis
h.
Marked steatosis
i.
Wilson disease
j.
Autoimmune hepatitis
35- A 76-year-old man presents with weight loss, dark urine, and pale stools which are difficult to flush away. An excess of which of the following would account for this history? f. Conjugated bilirubin g.
Hyperbilirubinaemia
h.
Stercobilinogen
i.
Unconjugated bilirubin
j.
Urobilinogen
36-Acute pancreatitis f.Serum calcium start to rise after 48 hours g.Hypoglycaemia is bad prognostic factor h.Age is an important prognostic factor i.Serum amylase is more specific than serum lipase j.Severe pancreatitis compromise around 40% of cases
437
37- The most commonly used imaging method for diagnosis of acute cholecystitis is: k. CT of the abdomen. l.
Ultrasonography of the gallbladder.
m. Oral cholecystogram. n. Radionuclide (HIDA) scan of the gallbladder o. MRI 38- A 23-year-old male presents to the emergency department after being involved in a motor vehicle accident. On physical examination, he opens his eyes spontanously, he occasionally mumbles incomprehensible sounds, he localizes to painful stimulation with his right upper extremity, His pupils are 4 mm bilaterally and reactive. This patient’s Glasgow Coma Scale (GCS) score: f. 7 g.
9
h.
8
i.
11
j.
12
39- Complication of undescended testis include all of the following except : f.
Malignant degeneration.
g. Increased susceptability to trauma. h. Increased spermatogenesis. i.
More liable to testiculer torsion.
j.
Psychological complication
40- Neonatal duodenal obstruction: f. May be associated with down's syndrome. g. Is more frequently found in premature infants. h. Typically presents with gross abdominal distension. i.
Usually presents with vomiting of non-bile stained fluid
j.
B&C only.
41-Markedly elevated alpha-fetoprotein is diagnostic (A) Hepatic hemangioma (B) Angiosarcoma in the liver (C) Hepatic adenoma (D) Focal nodular hyperplasia (E) Hepatocellular carcinoma
438
2.on clinical examination of tortion testis all are true except: a. testis is tender and swollen. b.testis is elevated and raised. c.loss of cremasteric reflex. d.redness with possible reactive hydrocele. e. pain decrease with elevation of the testis . 3. Regarding cryptorchidism(undesended testis) all are true except : a. refers to the interruption of the normal descent of the testis into the scrotum. b. The testicle may reside in the retroperitoneum, in the internal inguinal ring, in the .inguinal canal, or even at the external ring
C. At birth, approximately 95% of infants have the testicles normally positioned in the .scrotum . D. it’s a common disorder and incidence increased up to 30 % in premature .E. undescended testis is always regarded as an ectopic testis
:Regarding GI(gastrointestinal bleeding ) all are true except .4 a.Lower GI hemorrhage is defined as an abnormal intra luminal blood loss from a source distal to .the Treitz ligament .b.The most cause of massive lower GI bleeding in adults are diverticulosis and angiodysplasia .c.Cancer colon is usually associated with massive lower GI hemorrhage .d.Hemorrhage from diverticular disease stops spontaneously in 80% of patients E. patients with massive upper GI bleeding may present with maroon stools or bright red .blood from the rectum :Signs of severe bood loss include the following except .5 a.Pallor
b.Clammy skin c.bradycardia
d.Tachycardia
e.Hypotension
: regarding stigmata of bleeding in peptic ulcer.6 . a.its associated with increase risks for rebleeding .b.adherent clot is the most significant stigmata . c.arterial spurting hemorrhage associated with low risk of rebleeding d.risk of rebleeding in ulcer with clean base is above 50% . e.nonbleeding visible vessel is not associated with risk of rebleeding
7.Features of inflammatory response syndrome (SIRS) include the following except:
439
a.
Temperature> 38.4C
b.
Temperature <36.C
c.
WCC<4.ooo cells per ml
d.
Respiratory rate >20 per minute
e.
PCO2> 32 mmHg
8.The development of thrombocytopenia and arterial thrombosis with heparin requires: a. Continuation of heparin and platelet transfusion b.
Continuation of heparin and thrombolysis
c.
Doubling the heparin dosage
d.
Changing the route of heparin administration
e.
Discontinuation of heparin
13.Regarding Heparin-induced thrombocytopenia (HIT) all are true except a. Is a special case of drug-induced immune thrombocytopenia. b.
The platelet count typically begins to fall 5 to 14 days after heparin has been started.
c.
Thrombocytopenia is usually severe.
d.
HIT should be suspected if the platelet count falls to less than 100,000 or if it drops by 50% from baseline in a patient receiving heparin
e.
HIT is more common with full-dose unfractionated heparin (1 to 3%
14.Regarding gall bladder and bile secretion all are true except : a.The gallbladder is a pear-shaped, about 7 to 10 cm long with an average capacity of 30 to 50ml.
b.When obstructed, the gallbladder can distend markedly and contain up to 300 mL
c. Anomalies of the hepatic artery and the cystic artery are quite common, occurring in as many as 50% of cases.
d.liver produces 500 to 1000 mL of bile a day e.Vagal stimulation decreases secretion of bile
15.Regarding gall bladder stones all are true except: a. Prevalence increases with advancing age b. Over 10% of those with stones in the gallbladder have stones in the common bile .duct
440
c.10-20% become symptomatic d.cholesterol stones are the most common type. e.pigment stones are associated with secondary common bile duct stones . 16. In acute cholecystitis all true except : a.Most common organisms are E. coli b. 90% cases result from obstruction to the cystic duct by a stone. C .patient present with constant pain usually greater than 6 hours duration in right upper quadrant . d.presence of gall stones and percholecystic fluid on US is diagnostic e.Cholecystectomy is contraindicated in acute stage . 17.complication of acute cholecystitis include all the following except: a.Gangrenous cholecystitis b.Gallbladder perforation c.Cholecystoenteric fistula d.mesnteric ischemia e.Gallstone ileus 18.Regarding choledocholithisis all are true except: a. may be silent and or may cause obstruction, complete or incomplete b. may manifest with cholangitis or gallstone pancreatitis. c.present with severe jaundice and cholangitis in case of stone impaction d.Rt upper quadrant pain ,fever,and jaundice are called charcots triad in cholangitis. e.impaction of small stones has no relation with acute pancreatitis . 19.Regarding acalculous cholecystistis all are true except: a. Acute inflammation of the gallbladder can occur without gallstones b. Acalculous cholecystitis typically develops in critically ill patients in the intensive care unit. c. Patients on parenteral nutrition with extensive burns, sepsis, major operations are at risk for developing acalculous cholecystitis. d.US is not a good diagnostic tool. E.can be managed by cholecystectomy or percutanous cholecyststomy . 20.Regardind gall bladder cancer all are true except: a. Larger stones (>3 cm) are associated with a 10-fold increased risk of cancer. b. up to 95% of patients with carcinoma of the gallbladder have gallstones. c. Polypoid lesions of the gallbladder are not associated with increased risk of cancer d. Patients with choledochal cysts have an increased risk of developing cancer e. Sclerosing cholangitis is risk factor for developing gall bladder cancer .
21.Regarding tumors of the liver all are true except : a.Hemangioma is the most common solid benign lesion . b.Spontaneous rupture in hemangioma (bleeding) is rare. c.Hepatic adenomas carry a significant risk of spontaneous rupture with intraperitoneal bleeding. d. Hepatic adenomas have a risk of malignant transformation to a well-differentiated HCC(hepatocellular carcinoma). e.focal nodular hyperplasia lesions(FNH) lesions usually rupture spontaneously and have significant risk of malignant transformation .
441
22.Regarding pyogenic liver abscess all are true except : a. arise as a result of biliary sepsis. b.associated with high mortality. c .appendicitis is unlikely the cause pyogenic liver abscess. d.30% of patient have pleural effusion on presentation . e.lab.investiation show elevated WBC and abnormal liver function . 23.Regarding acute pancreatitis all are true except: a.Gallstones less than 5mm diameter are more likely to cause pancreatitis than larger ones b.. The mortality associated with infected necrosis is about 40% c.Cullen's sign is a sign of retroperitoneal hemorrhage in severe hemorrhagic pancreatitis . d.elevated serum amylase is a significant predictor of severity . e.. 50% of deaths occur within first week due to multi-organ failure . 24.All occur as a complication of acute pancreatitis except: a.panreatic fluid collection b.colonic necrosis c.coagulopathy d.hypercalcemia e.respiratory failure 25.Regarding head trauma all are true except: a.basal skull fracture regarded when one of the orbital roof ,sphenoidal bone or petromastoid portion are involved . b.epidural hematoma is an Lens shape hematoma between dura and the skull. c.subdural hematoma is crescent shaped hematoma ,between brain and dura d.secondary brain injury is preventable. e.GCS glascow Coma scale 3/15 indicate uncomprehensive sounds . 27.Regarding chronic lower limb ischemia all are true except: a. Claudication distance is distance after which the pain is felt. b. Rest pain is continous severe burning pain in the foot which indicate critical ischemia. c. trophic changes include tapering digits ( loss of S.C fat ) and muscle wasting . d. usuall presentations of patients with lower limb ischemia are pain,trophic changes and gangrene. e. venous filling time more than 2 minutes indicates mild lower limb ischemia. 28.Regarding 4 weeks 4 Kg bodywt. old full term neonate presented with rapidly progressive projectile non bilious vomiting and palpable upper abdominal mass all are true except : a. dehydration and alkalosis are prominent features. b.maintenance fluid therapy is about 4ml /Kg /hour. c.administration of IV fluids with 5% dextrose, 0.5% normal saline, and KCl usually corrects the alkalosis . d.Estimated total blood volume is about 320 cc. E. the most likely diagnosis is high jejunal atresia 29.All are true regarding jejunoileal atresia except: a. present with bile stained vomiting . b. Failure to pass meconium or small amounts of mucus or meconium maybe passed per rectum. c. present with abdominal distention.
442
d. X-ray show double bubble appearance . e. The x-rays usually show multiple air-fluid levels. 30.regarding malrotation of the gut all are true except: a.The patient might be asymptomatic and then develop the symptoms when he is older. b.Commonest abnormality results in caecum lying close to DJ flexure. c.Fibrous bands may be present between caecum and DJ flexure (Ladd's bands). d. the patient is unlikely to have clinical picture of duodenal obstruction . e. In malrotation midgut mesentery is abnormally narrow and liable to volvulus. 1. A 40-year-old female has a 4-cm hemangioma in the right lobe of the liver on computed tomography scan. She is asymptomatic. Appropriate action should be: A. fine-needle biopsy B. arrangement for elective resection C. no further action D. angiographic embolization 2. Regarding an amebic liver abscess: A. surgical drainage is usually required B. negative stool testing for amebiasis rules out the disease C. it should be drained percutaneously under com- puted tomography guidance D. it is treated with metronidazole 3. A single organism is usually the causative agent in: A. pelvic inflammatory disease B. perforated diverticulitis C. acute cholecystitis D. primary peritonitis E. diabetic foot infections 4. Signs and symptoms of hemolytic transfusion reactions include A. Hypothermia B. Hypertension C. Polyuria D. Abnormal bleeding E. Hypesthesia at the transfusion site 5. In a hemolytic reaction caused by an incompatible blood transfusion, the treatment that is most likely to be helpful is: A. Promoting a diuresis with 250 ml of 50% mannitol B. Treating anuria with fluid and potassium replacement C. Acidifying the urine to prevent hemoglobin precipitation in the renal tubules D. Removing foreign bodies, such as Foley catheters, which may cause hemorrhagic complications E. Stopping the transfusion immediately 6. Which statements about extrahepatic bile duct cancer are correct? A. Cholangiography is not essential in evaluating patients for resectability. B. The prognosis is excellent when appropriate surgical and adjuvant therapy are given. C. The location of the tumor determines the type of surgical procedure. D. The disease usually becomes manifest by moderate to severe right-side upper quadrant pain. 7. Which of the following statements about the diagnosis of acute calculous cholecystitis is true? A. Pain is so frequent that its absence almost precludes the diagnosis. B. Jaundice is present in a majority of patients. C. Ultrasonography is the definitive diagnostic test. D. Cholescintigraphy is not definitive diagnostic test.
443
8. Which statement about acute acalculous cholecystitis is correct? A. The disease is often accompanied by or associated with other conditions. B. The diagnosis is often difficult. C. The mortality rate is higher than that for acute calculous cholecystitis. D. The disease has been treated successfully by percutaneous cholecystostomy E. all are correct 9. Which of the following statements about laparoscopic cholecystectomy are correct? A. The procedure is associated with less postoperative pain and earlier return to normal activity. B. The incidence of bile duct injury is lower than for open cholecystectomy. C. Laparoscopic cholecystectomy should be used in asymptomatic patients because it is safer than open cholecystectomy. D. Pregnancy is a contraindication. 10. A 15-year-old female presents with RUQ abdominal pain. Workup reveals a choledochal cyst. Which of the following statements is TRUE? A. Choledochal cysts are more common in men. B. Laparoscopic cholecystectomy is the recommended treatment. C. Patients with a choledochal cyst have an increased risk of cholangiocarcinoma. D. All patients with a choledochal cyst have abdominal pain, a RUQ mass, and jaundice. E. The etiology is infectious. 11. An 85-year-old man is brought to the hospital with a 2-day history of nausea and vomiting. He has not passed gas or moved his bowels for the last 5 days. Abdominal films show dilated small bowel, no air in the rectum and air in the biliary tree. Which of the following statements is TRUE? A. Air in the biliary tree associated with small-bowel obstruction suggests a diagnosis of gallstone ileus. B. An enterotomy should be distal to the site of obstruction and the stone should be removed. C. Gallstone ileus is more common in the young adults. D. Cholecystectomy is contraindicated. E. Small-bowel obstruction usually occurs in the distal jejunum. 12. A 45-year-old patient with chronic pancreatitis is suffering from malnutrition and weight loss secondary to inadequate pancreatic exocrine secretions. Which is TRUE regarding pancreatic secretions? A. Secretin releases fluid rich in enzymes. B. Secretin releases fluid rich mainly in electrolytes and bicarbonate. C. Cholecystokinin releases fluid,predominantly rich in electrolytes, and bicarbonate. D. All pancreatic enzymes are secreted in an inactive form. E. The pancreas produces proteolytic enzymes only. 13. A 43-year-old woman has gallstone pancreatitis that resolves in 2 days with conservative treatment. She has no abdominal complaints and her liver and pancreatic laboratory values have returned to normal. She is scheduled for laparoscopic cholecystectomy. Which of the following statements is TRUE? A. Intraoperative cholangiography is associated with a decreased risk of biliary tract injury . B. The procedure should be scheduled for 6 weeks after resolution of symptoms . C. Intraoperative cholangiography in this patient will identify choledocholithiasis in 50% of cases . D. Preoperative endoscopic retrograde cholangiopancreatography (ERCP) should be performed. E. The sensitivity of magnetic resonance cholangiopancreatography (MRCP) for choledocholithiasis in this patient is less than 50%. 14. A 39-year-old woman is admitted with gallstone pancreatitis and epigastric pain. Pertinent data include amylase, 2000 U/L; bilirubin, 1.2 mg/dL; and WBC count, 15,000/mm3 . After 2 days of medical management, her epigastric pain resolves. Her amylase is 340 U/L and her bilirubin and WBC count have returned to normal. Laparoscopic cholecystectomy should be attempted: A. after endoscopic retrograde cholangiopancreatography (ERCP) and sphincterotomy B. prior to discharge
444
C. once her amylase is normal D. 4 to 6 weeks later E. only if the patient develops recurrent pancreatitis 15. A 48-year-old woman is admitted with acute cholecystitis. The bilirubin level is elevated, as are the serum and urinary amylase levels. Which radiologic sign indicates biliary obstruction in pancreatitis? A. Pancreatic intraductal calcification B. Smooth narrowing of the distal CBD C. Stomach displaced anteriorly D. Calcified gallstone E. Air in the biliary tree) 16. Following a motor vehicle accident a truck driver complains of severe abdominal pain. Serum amylase level is markedly increased to 800 U. Grey Turner’s sign is seen in the flanks. Pancreatic trauma is suspected. Which statement is true of pancreatic trauma? A. It is mainly caused by blunt injuries. B. It is usually an isolated single-organ injury. C. It often requires a total pancreatectomy. D. It may easily be overlooked at operation. E. It is proved by the elevated amylase level. 17. A 73-year-old woman is evaluated for obstructive jaundice after an injury to the CBD, 7 months previously at laparoscopic cholecystectomy. The alkaline phosphatase is elevated. In obstructive jaundice, which of the following statements is true regarding alkaline phosphatase? A. Its level increases before that of bilirubin. B. Its level is unlikely to be increased in pancreatic malignancy. C. Its elevation indicates bone metastasis. D. Its elevation excludes hepatic metastasis. E. Its level falls after that of the bilirubin,following surgical intervention. 18. A48-year-old female travel agent presents with jaundice. Radiological findings confirm the presence of sclerosing cholangitis. She gives a long history of diarrhea for which she has received steroids on several occasions. She is likely to suffer from which of the following? A. Pernicious anemia B. Ulcerative colitis C. Celiac disease D. Liver cirrhosis E. Crohn’s disease 19. A38-year-old male lawyer develops abdominal pain after having a fatty meal. Examination reveals tenderness in the right hypochondrium and a positive Murphy’s sign. Which test is most likely to reveal acute cholecystitis? A. HIDA scan B. Oral cholecystogram C. Intravenous cholangiogram D. CT scan of the abdomen E. ERCP 20. A 65-year-old woman is admitted with RUQ pain radiating to the right shoulder, accompanied by nausea and vomiting. Examination reveals tenderness in the RUQ and a positive Murphy’s sign. A diagnosis of acute cholecystitis is made. What is the most likely finding? A. Serum bilirubin levels may be elevated. B. Cholelithiasis is present in 40–60%. C. Bacteria are rarely found at operation. D. An elevated amylase level excludes this diagnosis. E. A contracted gallbladder is noted on ultrasound.
445
21. A 32-year-old diabetic woman who has taken contraceptive pills for 12 years develops RUQ pain. CT scan of the abdomen reveals a 5-cm hypodense lesion in the right lobe of the liver consistent with a hepatic adenoma. What should the patient be advised to do? A. Undergo excision of the adenoma B. Stop oral contraceptives only C. Stop oral hypoglycemic medication D. Undergo right hepatectomy E. Have serial CT scans every 6 months 22. A 9 month old boy presents with an acute scrotal swelling. The following diagnoses are likely: A. Epididymitis B. Orchitis C. Torsion of the testicular appendage D. Irreducible inguinal hernia E. Acute idiopathic scrotal oedema 23. A 76-year-old man presents with weight loss, dark urine, and pale stools which are difficult to flush away. An excess of which of the following would account for this history? A. Conjugated bilirubin B. Hyperbilirubinaemia C. Stercobilinogen D. Unconjugated bilirubin E. Urobilinogen 24. An otherwise well 13-year-old boy is admitted complaining of sudden onset severe left sided testicular pain 2 hours prior to admission. He gives no history of trauma, dysuria or frequency. On examination he is found to have a tender, high-riding testicle.What is the most appropriate next step in this young man’s management? A. Herniography B. Scrotal Doppler ultrasound on the next available list C. FBC and U&E D. Scrotal Doppler ultrasound as an emergency E. Surgical exploration of his scrotum 25. Acute scrotum A. Torsion testis should be operated within 12 hour of presentation B. Epidedimoorchitis pain increase by testicular elevation C. If in doubt scrotum should be explored D. Doppler ultrasound has no role in diagnosis E. None of the above 26. stones in the common bile duct: A. Are present in nearly 50 per cent of cases of cholecystitis. B. Often give rise to jaundice, fever and biliary colic. C. Are usually accompanied by progressive jaundice. D. Are usually associated with a distended gallbladder. E. A&D only. 27. Which of the following statements regarding whole blood transfusion is correct? A. Whole blood is the most commonly used red cell preparation for transfusion in the B. Whole blood is effective in the replacement of acute blood loss. C. Most blood banks have large supplies of whole blood available. D. The use of whole blood produces higher rates of disease transmission than the use of individual component therapies. E. Old Whole blood is effective in the replacement of platelets. 28. Acute cholecystitis all are true except A. Commonest bacteria is E .coli B. Wall thickness more than 3mm by ultrasound C. WCC is between 10-15 000 cell/mm3 D. Mild elevated bilirubin may accompany it
446
E. HIDA scan has no role in diagnosis of acute cholecystitis 29. A 51-year-old male experiences the sudden onset of massive emesis of bright red blood. There have been no prior episodes of hematemesis. He is known to be hepatitis B surface antigen positive. His hematemesis is most likely a consequence of which of the following abnormalities of the esophagus? A. Varices B. Barrett esophagus C. Candidiasis D. Reflux esophagitis E. Squamous cell carcinoma 30. A 61-year-old male has had ascites for the past year. After a paracentesis with removal of 1 L of slightly cloudy, serosanguinous fluid, physical examination reveals a firm, nodular liver.Laboratory findings include positive serum HBsAg and presence of hepatitis B core antibody. He has a markedly elevated serum alpha-fetoprotein (AFP) level. Which of the following hepatic lesions is he most likely to have? A. Hepatocellular carcinoma B. Massive hepatocyte necrosis C. Marked steatosis D. Wilson disease E. Autoimmune hepatitis 31. A 76-year-old man presents with weight loss, dark urine, and pale stools which are difficult to flush away. An excess of which of the following would account for this history? A. Conjugated bilirubin B. Hyperbilirubinaemia C. Stercobilinogen D. Unconjugated bilirubin E. Urobilinogen 32. Acute pancreatitis A. Serum calcium start to rise after 48 hours B. Hypoglycaemia is bad prognostic factor C. Age is an important prognostic factor D. Serum amylase is more specific than serum lipase E. Severe pancreatitis compromise around 40% of cases 33. The most commonly used imaging method for diagnosis of acute cholecystitis is: A. CT of the abdomen. B. Ultrasonography of the gallbladder. C. Oral cholecystogram. D. Radionuclide (HIDA) scan of the gallbladder E. MRI 34. A 23-year-old male presents to the emergency department after being involved in a motor vehicle accident. On physical examination, he opens his eyes spontanously, he occasionally mumbles incomprehensible sounds, he localizes to painful stimulation with his right upper extremity, His pupils are 4 mm bilaterally and reactive. This patient’s Glasgow Coma Scale (GCS) score: A. 7 B. 9 C. 8 D. 11 E. 12 35. Complication of undescended testis include all of the following except : A. Malignant degeneration. B. Increased susceptability to trauma. C. Increased spermatogenesis. D. More liable to testiculer torsion. E. Psychological complication
447
36. Neonatal duodenal obstruction: A. May be associated with down's syndrome. B. Is more frequently found in premature infants. C. Typically presents with gross abdominal distension. D. Usually presents with vomiting of non-bile stained fluid E. B&C only. 37. Markedly elevated alpha-fetoprotein is diagnostic A. Hepatic hemangioma B. Angiosarcoma in the liver C. Hepatic adenoma D. Focal nodular hyperplasia E. Hepatocellular carcinoma 38. on clinical examination of tortion testis all are true except: A. testis is tender and swollen. B. testis is elevated and raised. C. loss of cremasteric reflex. D. redness with possible reactive hydrocele. E. pain decrease with elevation of the testis . 39. Regarding cryptorchidism(undesended testis) all are true except : A. refers to the interruption of the normal descent of the testis into the scrotum. B. The testicle may reside in the retroperitoneum, in the internal inguinal ring, in the inguinal canal, or even at the external ring. C. At birth, approximately 95% of infants have the testicles normally positioned in the scrotum. D. it’s a common disorder and incidence increased up to 30 % in premature . E. undescended testis is always regarded as an ectopic testis. 40. Regarding GI(gastrointestinal bleeding ) all are true except: A. Lower GI hemorrhage is defined as an abnormal intra luminal blood loss from a source distal to the Treitz ligament. B. The most cause of massive lower GI bleeding in adults are diverticulosis and angiodysplasia. C. Cancer colon is usually associated with massive lower GI hemorrhage. D. Hemorrhage from diverticular disease stops spontaneously in 80% of patients. E. patients with massive upper GI bleeding may present with maroon stools or bright red blood from the rectum. 41. Signs of severe bood loss include the following except: A. Pallor B. Clammy skin C. bradycardia D. Tachycardia E. Hypotension 42. regarding stigmata of bleeding in peptic ulcer : A. its associated with increase risks for rebleeding . B. adherent clot is the most significant stigmata. C. arterial spurting hemorrhage associated with low risk of rebleeding . D. risk of rebleeding in ulcer with clean base is above 50% E. nonbleeding visible vessel is not associated with risk of rebleeding . 43. Features of inflammatory response syndrome (SIRS) include the following except: A. Temperature> 38.4C B. Temperature <36.C C. WCC<4.ooo cells per ml D. Respiratory rate >20 per minute E. PCO2> 32 mmHg 44. The development of thrombocytopenia and arterial thrombosis with heparin requires: A. Continuation of heparin and platelet transfusion B. Continuation of heparin and thrombolysis C. Doubling the heparin dosage
448
D. Changing the route of heparin administration E. Discontinuation of heparin 45. Regarding Heparin-induced thrombocytopenia (HIT) all are true except A. Is a special case of drug-induced immune thrombocytopenia. B. The platelet count typically begins to fall 5 to 14 days after heparin has been started. C. Thrombocytopenia is usually severe. D. HIT should be suspected if the platelet count falls to less than 100,000 or if it drops by 50% from baseline in a patient receiving heparin E. HIT is more common with full-dose unfractionated heparin (1 to 3%) 46. Regarding gall bladder and bile secretion all are true except : A. The gallbladder is a pear-shaped, about 7 to 10 cm long with an average capacity of 30 to 50ml. B. When obstructed, the gallbladder can distend markedly and contain up to 300 mL C. Anomalies of the hepatic artery and the cystic artery are quite common, occurring in as many as 50% of cases. D. liver produces 500 to 1000 mL of bile a day E. Vagal stimulation decreases secretion of bile 47. Regarding gall bladder stones all are true except: A. Prevalence increases with advancing age B. Over 10% of those with stones in the gallbladder have stones in the common bile duct. C. 10-20% become symptomatic D. cholesterol stones are the most common type. E. pigment stones are associated with secondary common bile duct stones . 48. In acute cholecystitis all true except : A. Most common organisms are E. coli B. 90% cases result from obstruction to the cystic duct by a stone. C. patient present with constant pain usually greater than 6 hours duration in right upper quadrant . D. presence of gall stones and percholecystic fluid on US is diagnostic E. Cholecystectomy is contraindicated in acute stage . 49. complication of acute cholecystitis include all the following except: A. Gangrenous cholecystitis B. Gallbladder perforation C. Cholecystoenteric fistula D. mesnteric ischemia E. Gallstone ileus 50. Regarding choledocholithisis all are true except: A. may be silent and or may cause obstruction, complete or incomplete B. may manifest with cholangitis or gallstone pancreatitis. C. present with severe jaundice and cholangitis in case of stone impaction D. Rt upper quadrant pain ,fever,and jaundice are called charcots triad in cholangitis. E. impaction of small stones has no relation with acute pancreatitis . 51. Regarding acalculous cholecystistis all are true except: A. Acute inflammation of the gallbladder can occur without gallstones B. Acalculous cholecystitis typically develops in critically ill patients in the intensive care unit. C. Patients on parenteral nutrition with extensive burns, sepsis, major operations are at risk for developing acalculous cholecystitis. D. US is not a good diagnostic tool. E. can be managed by cholecystectomy or percutanous cholecyststomy . 52. Regardind gall bladder cancer all are true except: A. Larger stones (>3 cm) are associated with a 10-fold increased risk of cancer. B. up to 95% of patients with carcinoma of the gallbladder have gallstones. C. Polypoid lesions of the gallbladder are not associated with increased risk of cancer D. Patients with choledochal cysts have an increased risk of developing cancer E. Sclerosing cholangitis is risk factor for developing gall bladder cancer .
449
53. Regarding tumors of the liver all are true except : A. Hemangioma is the most common solid benign lesion . B. Spontaneous rupture in hemangioma (bleeding) is rare. C. Hepatic adenomas carry a significant risk of spontaneous rupture with intraperitoneal bleeding. D. Hepatic adenomas have a risk of malignant transformation to a well-differentiated HCC(hepatocellular carcinoma). E. focal nodular hyperplasia lesions(FNH) lesions usually rupture spontaneously and have significant risk of malignant transformation . 54. Regarding pyogenic liver abscess all are true except : A. arise as a result of biliary sepsis. B. associated with high mortality. C. appendicitis is unlikely the cause pyogenic liver abscess. D. 30% of patient have pleural effusion on presentation . E. lab.investiation show elevated WBC and abnormal liver function . 55. Regarding acute pancreatitis all are true except: A. Gallstones less than 5mm diameter are more likely to cause pancreatitis than larger ones B. The mortality associated with infected necrosis is about 40% C. Cullen's sign is a sign of retroperitoneal hemorrhage in severe hemorrhagic pancreatitis . D. elevated serum amylase is a significant predictor of severity . E. 50% of deaths occur within first week due to multi-organ failure . 56. All occur as a complication of acute pancreatitis except: A. panreatic fluid collection B. colonic necrosis C. coagulopathy D. hypercalcemia E. respiratory failure 57. Regarding head trauma all are true except: A. basal skull fracture regarded when one of the orbital roof ,sphenoidal bone or petromastoid portion are involved . B. epidural hematoma is an Lens shape hematoma between dura and the skull. C. subdural hematoma is crescent shaped hematoma ,between brain and dura D. secondary brain injury is preventable. E. GCS glascow Coma scale 3/15 indicate uncomprehensive sounds . 58. Regarding chronic lower limb ischemia all are true except: A. Claudication distance is distance after which the pain is felt. B. Rest pain is continous severe burning pain in the foot which indicate critical ischemia. C. trophic changes include tapering digits ( loss of S.C fat ) and muscle wasting . D. usuall presentations of patients with lower limb ischemia are pain,trophic changes and gangrene. E. venous filling time more than 2 minutes indicates mild lower limb ischemia. 59. Regarding 4 weeks 4 Kg bodywt. old full term neonate presented with rapidly progressive projectile non bilious vomiting and palpable upper abdominal mass all are true except : A. dehydration and alkalosis are prominent features. B. maintenance fluid therapy is about 4ml /Kg /hour. C. administration of IV fluids with 5% dextrose, 0.5% normal saline, and KCl usually corrects the alkalosis . D. Estimated total blood volume is about 320 cc. E. the most likely diagnosis is high jejunal atresia 60. All are true regarding jejunoileal atresia except: A. present with bile stained vomiting . B. Failure to pass meconium or small amounts of mucus or meconium maybe passed per rectum. C. present with abdominal distention. D. X-ray show double bubble appearance .
450
E. The x-rays usually show multiple air-fluid levels. 61. regarding malrotation of the gut all are true except: A. The patient might be asymptomatic and then develop the symptoms when he is older. B. Commonest abnormality results in caecum lying close to DJ flexure. C. Fibrous bands may be present between caecum and DJ flexure (Ladd's bands). D. the patient is unlikely to have clinical picture of duodenal obstruction . E. In malrotation midgut mesentery is abnormally narrow and liable to volvulus. 18) Hypovolaemic shock is characterized by: A) a low central venous pressure , low cardiac output , low peripheral resistance B) a high central venous pressure , high cardiac output , low peripheral resistance C) *a low central venous pressure , low cardiac output , high peripheral resistance D) a low central venous pressure , high cardiac output , high peripheral resistance E) a high central venous pressure , low cardiac output , low peripheral resistance 2. the most important finding in the diagnosis of acute appendicitis is: a. vomiting b. Fever c. leukocytosis *d. right lower quadrant pain and tenderness e. referred rebound tenderness ( Rovsing sign ) 3. the most common site for intestinal obstruction due to cholecystoenteric fistula is the a. pylorus b. duodenum c. jejunum *d. ileum e. sigmoid colon. 4. Common characteristics of small bowel obstruction include all of the following EXCEPT: * a. ascites b. frequent progression to strangulation c. failure to pass flatus d. distention e. vomiting 5. Initial fluid resuscitation of a patient with multiple fractures and hypovolemic shock should be : a. blood transfusion b. hypertonic saline c. fresh frozen plasma * d. Ringer ’s lactate e. albumin 6. All of the following findings suggests that shock in an injured patient may be due to hypovolemia Except: a. hypotension b. *distended neck veins c. tachycardia d. Weak feeble pulse e. falling central venous pressure 7. All of the following are physical signs of both massive hemothorax and tension pneumothorax EXCEPT: a. tracheal shift. b. decreased breath sounds. c. *tachycardia. d. hypotension. e. distended neck veins.
451
8. Tension pneumothorax Z. is the commonest type of chest injuries AA.Needs urgent X-Ray chest BB.* Is a clinical Diagnosis CC. Causes flat neck viens DD. Treated by thoracotomy tube after chest X-ray 9. Right side colon cancer usually present as all except a. Microcytic hypo chromic anemia b. *Large bowel obstruction c. Appendicitis like symptoms d. Mass in right iliac fossa e. Small bowel obstruction 10.Screening for colorectal cancer mostly includes : a. occult blood in stool b. Tumor marker CEA c. Double contrast barium enema d. Colonoscopy e. *All of the above 11.Duke's staging for colon cancer includes all of the following Except: a. Duke's C is a tumor with lymph node metastasis b. *Duke's B is a tumor localized to submucosa c. Duke's A is a tumor localized to rectal wall d. Duke's D has the warst prognosis e. Dukes C has 40% five year survival 12.In colorectal cancer a.Villous tumor common site is right colon b. Metachronous lesion is two tumors at the same time c. Another tumor after few years is synchronous tumor d. Large villous tumor cause Hyponatremia e. *Malignant transformation in polyp is related to its size :Gallstones are characterized by all the following EXCEPT .13 a. are frequently the cause of flatulent dyspepsia b. are mostly cholesterol, mixed or combination of bile constituents c. are commonly radiopaque d. can cause mucocele of the gall bladder *e. are a rare cause of acute pancreatitis
in origin
14.Regarding the oesophagus, All of the following is correct Except: a. in adult it is about 25 cm long starting from 6th cervical – 11th thoracic vertebra b.* it has three areas of anatomic Constrictions; cricopharyngeal at (15cm), left main bronchus and aortic arch at (25cm), and at diaphragmatic hiatus at (60cm) from central incisor c. Achalasia is a motility disorder of the oesophagus due to loss of ganglionic cells in Auerbach’s plexus d. Plummer- Vinson syndrome is characterized pathologically with an upper oesophageal web e. Barrett’s esophagus may be a risk factor to develop malignancy 15. Pseudomembranous enterocolitis is caused by which the following organisms: a.Clostridium sporogenes *b.Clostridium defficile c.Streptococcus faecalis d.Penicillin sensitive staphylocci e.Pseudomonas aeruginos
452
:Regarding Rectal bleeding all the following are correct EXCEPT .16 a.which is bright red and occurring on defecation is usually arising from the anal canal b. which is dark and mixed with the stool is arising from the rectum or above c.should be investigated by a sigmoidoscopic examination in all cases in elderly *d.is always caused by piles e.as a rule of thumb,any patient above fourty with rectal bleeding is to be warked up for malignancy ?Which of the following disorders is not associated with hypocalcemia .17 a. sarcoidosis * b. vitamin D deficiency c. renal insufficiency d. hypoparathyroidism e. pancreatitis ?Which of the following is a function of the gallbladder .18 a.absorption b. motor activi c. secretion d. storage of bile *e.) all of the above 19.All of the following are true about neurogenic shock EXCEPT: a.There is a decrease in systemic vascular resistance and an increase in venous capacitance. b.Tachycardia or bradycardia may be observed, along with hypotension. * C.The use of an alpha agonist such as phenylephrine is the mainstay of treatment. d.Severe head injury, spinal cord injury, and high spinal anesthesia may all cause neurogenic shock. e.A and B 20. Common causes of acute mesenteric ischaemia are: a.Mesenteric artery thrombosis b.Mesenteric artery embolism c.Mesenteric venous thrombosis d.Non occlusive mesenteric ischemia * e.All of the above :Regarding intestinal obstruction, all of the following are true Except .21 a. Its cardinal features are; abdominal pain, distension, vomiting and constipation b. small bowel obstruction is commonly of adhesive aetiology in adults *c. Erect plan abdominal X-Ray is usually requested to localize the site of obstruction d. intussception is usually treated primarily with hydrostatic pressure e. superior mesenteric artery embolus the commonest cause of acute mesenteric ischaemia :Regarding inguinal hernia .22 a. the internal ring is a triangular aperture in fascia transversalis
453
b. it is commenest cause of intestinal obstruction in infants and group c. indirect hernial sac lies lateral to pubic tubercle * d. Hasselbach's triangle is formed medially by the medial edge of rectus abdominis, laterally by the inferior epigastric vessles and inferiorly by the inguinal ligament forms the e. direct inguinal hernia lies lateral to inferior epigastric vessles :In response to trauma.23 a. ebb phase is the convalescence stage in trauma recovery b. Main source of energy expenditure in tauma is carbohydrate c. third space shift in trama is due mainly to altered permeability of membrane d. IL-1 is liberated mainly from the liver e. mild elevation of temperature is abnormal in the first post operative :In haemostasis, all of the following are true Except .24 a. primary haemostasis at formation of platelet plug b. extrinsic mechanism of coagulation is initiated by activation of Factor VII, but the intrinsic one by activation of Factor XII c. Vitamin K Dependent Factors are Factors; II,VII,IX and XII d. Tissue Plasminogen activator coverts plasminogen to plasmin e. Dissiminated Intravascular Coagulopathy is characterized by low elevated fibrinogen degredation products,low ..,prolonged PT and PTT
younger age
capillary * day
*
platelets, fibrinogen,and
:Regarding blood transfusion .25 a.blood is collected in sterile bags and stored in Blood Banks At 0°C b. Platelets are nearly lost after 7 days in the stored blood c.blood component therapy is the gold standard in transfusion therapy * d. cross-mached blood and ELISA tested for HIV is one hundred percent safe e. whole blood transfusion is the ideal to treat chronic anaemia in an
elderly patient
الجامعـة السلمية بغــزة Islamic University of Gaza Faculty of Medicine
Fourth year 20/ / : التاريخ...........................: الرقم الجامعي........................................................ : السم رباعي Choose the best single answer and fill the corresponding box in the answer sheet: Time Allowed: minutes. 1-All are true regarding scrotal swelling except a.in infantile hydrocele the swelling is irreducible and the testis is impalpable. b.hernia may be reducible with cough impulse and palpable testis. c. on examination of hydrocele of adults you can not get above it . d.communicating hydrocele means patent processus vaginalis .
454
e.in hydrocele the swelling transluminate. 2.on clinical examination of tortion testis all are true except: a. testis is tender and swollen. b.testis is elevated and raised. c.loss of cremasteric reflex. d.redness with possible reactive hydrocele. e. pain decrease with elevation of the testis . 3. Regarding cryptorchidism(undesended testis) all are true except : a. refers to the interruption of the normal descent of the testis into the scrotum. b. The testicle may reside in the retroperitoneum, in the internal inguinal ring, in the .inguinal canal, or even at the external ring C. At birth, approximately 95% of infants have the testicles normally positioned in the .scrotum . D. it’s a common disorder and incidence increased up to 30 % in premature .E. undescended testis is always regarded as an ectopic testis
.:Regarding GI(gastrointestinal bleeding ) all are true except .4 a.Lower GI hemorrhage is defined as an abnormal intra luminal blood loss from a source distal to .the Treitz ligament .b.The most cause of massive lower GI bleeding in adults are diverticulosis and angiodysplasia .c.Cancer colon is usually associated with massive lower GI hemorrhage .d.Hemorrhage from diverticular disease stops spontaneously in 80% of patients E. patients with massive upper GI bleeding may present with maroon stools or bright red .blood from the rectum :Signs of severe bood loss include the following except .5 a.Pallor b.Clammy skin c.bradycardia d.Tachycardia e.Hypotension : regarding stigmata of bleeding in peptic ulcer .6 . a.its associated with increase risks for rebleeding .b.adherent clot is the most significant stigmata . c.arterial spurting hemorrhage associated with low risk of rebleeding d.risk of rebleeding in ulcer with clean base is above 50% . e.nonbleeding visible vessel is not associated with risk of rebleeding 7.Features of inflammatory response syndrome (SIRS) include the following except: f. Temperature> 38.4C g. Temperature <36.C h. WCC<4.ooo cells per ml i. Respiratory rate >20 per minute j. PCO2> 32 mmHg 8.The development of thrombocytopenia and arterial thrombosis with heparin requires: f. Continuation of heparin and platelet transfusion g. Continuation of heparin and thrombolysis h. Doubling the heparin dosage i. Changing the route of heparin administration j. Discontinuation of heparin 9.The mechanism of action of heparin is: a. Direct inhibition of thrombin b. Prevention of factor II synthesis c. Inhibition of cyclo-oxygenase d. Potentiation of antithrombin III action
455
10.Regarding body response to trauma a. immediately after injury body reacts by decreasing energy requirement, temperature, consumption of O2 b. catabolic phase is associated with increased urinary nitrogen excretion and weight loss c. Cortisol is elevated and kept elevated in burn patient up to 4 weeks d. Growth hormone, IGF(insulin like growth hormone) are secreted immediately in catabolic phase e. Both norepinephrine (NE) and epinephrine (EPI) are increased three- to fourfold in plasma immediately following injury. 11.The following metabolic responses occur in response to trauma except Increased growth hormone Increased anti-diuretic hormone Increased ACTH Increased urine osmolality Decrease glucagon About coagulation all are true except .12 a. PTT tests intrinsic pathway of coagulation b. prothrombin time (PT) is associated with the extrinsic cascade. c. Both hemophilia A and hemophilia B are inherited as sex-linked recessive disorders. d. d- Treatment of patients with hemophilia A or B is with fresh frozen plasma e. patients with hemophilia may not bleed immediately after an injury or minor surgery but will begin to bleed several hours later 13.Regarding Heparin-induced thrombocytopenia (HIT) all are true except f. Is a special case of drug-induced immune thrombocytopenia. g. The platelet count typically begins to fall 5 to 14 days after heparin has been started. h. Thrombocytopenia is usually severe. i. HIT should be suspected if the platelet count falls to less than 100,000 or if it drops by 50% from baseline in a patient receiving heparin j. HIT is more common with full-dose unfractionated heparin (1 to 3%) 14.Regarding gall bladder and bile secretion all are true except : a.The gallbladder is a pear-shaped, about 7 to 10 cm long with an average capacity of 30 to 50ml. b.When obstructed, the gallbladder can distend markedly and contain up to 300 mL c. Anomalies of the hepatic artery and the cystic artery are quite common, occurring in as many as 50% of cases. d.liver produces 500 to 1000 mL of bile a day e.Vagal stimulation decreases secretion of bile, 15.Regarding gall bladder stones all are true except: a. Prevalence increases with advancing age b. Over 10% of those with stones in the gallbladder have stones in the common bile duct. c.10-20% become symptomatic d.cholesterol stones are the most common type. e.pigment stones are associated with secondary common bile duct stones . 16. In acute cholecystitis all true except : a.Most common organisms are E. coli b. 90% cases result from obstruction to the cystic duct by a stone.
456
C .patient present with constant pain usually greater than 6 hours duration in right upper quadrant . d.presence of gall stones and percholecystic fluid on US is diagnostic e.Cholecystectomy is contraindicated in acute stage . 17.complication of acute cholecystitis include all the following except: a.Gangrenous cholecystitis b.Gallbladder perforation c.Cholecystoenteric fistula d.mesnteric ischemia e.Gallstone ileus 18.Regarding choledocholithisis all are true except: a. may be silent and or may cause obstruction, complete or incomplete b. may manifest with cholangitis or gallstone pancreatitis. c.present with severe jaundice and cholangitis in case of stone impaction d.Rt upper quadrant pain ,fever,and jaundice are called charcots triad in cholangitis. e.impaction of small stones has no relation with acute pancreatitis . 19.Regarding acalculous cholecystistis all are true except: a. Acute inflammation of the gallbladder can occur without gallstones b. Acalculous cholecystitis typically develops in critically ill patients in the intensive care unit. c. Patients on parenteral nutrition with extensive burns, sepsis, major operations are at risk for developing acalculous cholecystitis. d.US is not a good diagnostic tool. E.can be managed by cholecystectomy or percutanous cholecyststomy . 20.Regardind gall bladder cancer all are true except: a. Larger stones (>3 cm) are associated with a 10-fold increased risk of cancer. b. up to 95% of patients with carcinoma of the gallbladder have gallstones. c. Polypoid lesions of the gallbladder are not associated with increased risk of cancer d. Patients with choledochal cysts have an increased risk of developing cancer e. Sclerosing cholangitis is risk factor for developing gall bladder cancer . 21.Regarding tumors of the liver all are true except : a.Hemangioma is the most common solid benign lesion . b.Spontaneous rupture in hemangioma (bleeding) is rare. c.Hepatic adenomas carry a significant risk of spontaneous rupture with intraperitoneal bleeding. d. Hepatic adenomas have a risk of malignant transformation to a well-differentiated HCC(hepatocellular carcinoma). e.focal nodular hyperplasia lesions(FNH) lesions usually rupture spontaneously and have significant risk of malignant transformation . 22.Regarding pyogenic liver abscess all are true except : a. arise as a result of biliary sepsis. b.associated with high mortality. c appendicitis is unlikely the cause pyogenic liver abscess . d.30% of patient have pleural effusion on presentation . e.lab.investiation show elevated WBC and abnormal liver function . 23.Regarding acute pancreatitis all are true except: a.Gallstones less than 5mm diameter are more likely to cause pancreatitis than larger ones b.. The mortality associated with infected necrosis is about 40% c.Cullen's sign is a sign of retroperitoneal hemorrhage in severe hemorrhagic pancreatitis . d.elevated serum amylase is a significant predictor of severity . e.. 50% of deaths occur within first week due to multi-organ failure . 24.All occur as a complication of acute pancreatitis except: a.panreatic fluid collection b.colonic necrosis c.coagulopathy d.hypercalcemia
457
e.respiratory failure
25.Regarding head trauma all are true except: a.basal skull fracture regarded when one of the orbital roof ,sphenoidal bone or petromastoid portion are involved . b.epidural hematoma is an Lens shape hematoma between dura and the skull. c.subdural hematoma is crescent shaped hematoma ,between brain and dura d.secondary brain injury is preventable. e.GCS glascow Coma scale 3/15 indicate uncomprehensive sounds . 26.all are true regarding action of calcium metabolism except: a.Decreased Plasma Calcium Causes increase PTH . b.increase PTH result in mobilization of bone Ca & phosphate, c.increased PTH decrease renal phosphate excretion & Ca retention d. increased PTH increase Vitamin D3 synthesis e. PTH & calcitonin release are regulated by plasma Ca levels 27.Regarding chronic lower limb ischemia all are true except: a. Claudication distance is distance after which the pain is felt. b. Rest pain is continous severe burning pain in the foot which indicate critical ischemia. c. trophic changes include tapering digits ( loss of S.C fat ) and muscle wasting . d. usuall presentations of patients with lower limb ischemia are pain,trophic changes and gangrene. e. venous filling time more than 2 minutes indicates mild lower limb ischemia. 28.Regarding 4 weeks 4 Kg bodywt. old full term neonate presented with rapidly progressive projectile non bilious vomiting and palpable upper abdominal mass all are true except : a. dehydration and alkalosis are prominent features. b.maintenance fluid therapy is about 4ml /Kg /hour. c.administration of IV fluids with 5% dextrose, 0.5% normal saline, and KCl usually corrects the alkalosis . d.Estimated total blood volume is about 320 cc. E. the most likely diagnosis is high jejunal atresia 29.All are true regarding jejunoileal atresia except: a. present with bile stained vomiting . b. Failure to pass meconium or small amounts of mucus or meconium maybe passed per rectum. c. present with abdominal distention. e. X-ray show double bubble appearance . e. The x-rays usually show multiple air-fluid levels. 30.regarding malrotation of the gut all are true except: a.The patient might be asymptomatic and then develop the symptoms when he is older. b.Commonest abnormality results in caecum lying close to DJ flexure. c.Fibrous bands may be present between caecum and DJ flexure (Ladd's bands). d. the patient is unlikely to have clinical picture of duodenal obstruction . e. In malrotation midgut mesentery is abnormally narrow and liable to volvulus. Fluid And Electrolyte 1. The effective osmotic pressure between the plasma and interstitial fluid compartments is primarily controlled by: A. Bicarbonate. B. Chloride ion. C. Potassium ion. D. Protein.
458
E. Sodium ion. 2. Symptoms and signs of extracellular fluid volume deficit include all of the following except: A. Anorexia. B. Apathy. C. Decreased body temperature. D. High pulse pressure E. Orthostatic hypotension. 3. The osmolarity of the extracellular fluid space is determined primarily by the concentration of: A. Bicarbonate B. Chloride ion C. Phosphate radicals D. Sodium ion E. Sulfate radicals 4. When lactic acid is produced in response to injury, the body minimizes pH change by: A. Decreasing production of sodium bicarbonate in tissues. B. Excreting carbon dioxide through the lungs. C. Excreting lactic acid through the kidneys D. Lowering renal output of chloride ions E. Metabolizing the lactic acid in the liver 5. The simplest effective method of estimating the degree of acidosis in a patient in shock is the measurement of: A. Arterial pH B. End tidal CO concentration C. pH of mixed venous blood D. serum CO level E. urinary pH 6. A decrease in intracellular water can be precipitated by: A. A decrease in sodium in extracellular fluid B. An increase in sodium in extracellular fluid C. An increase in sodium in intracellular fluid D. An isotonic decrease in extracellular fluid E. An isotonic increase in extracellular fluid 7. The first step in the management of acute hypercalcemia should be: A. Correction of deficit of extracellular fluid volume B. Hemodialysis C. Administration of furosemide D. Administration of mithramycin E. Parathyroidectomy. 8. Postoperative third-space accumulation should be managed by intravenous: A. Albumin B. Dextrose in water C. Fluid restriction D. 1/2 normal saline with potassium supplements E. Normal saline 9. The normal adult value for: A. urine output is 1.5 litre/day. B. Insensible water loss is 200 ml/day. C. Potassium requirement is 150 mEq (150mmol)/day. D. Protein requirement is 120 g/day. E. B&C only
459
The lungs normally eliminate water vapor (insensible loss) at a rate of approximately 400 mL every day The normal adult daily potassium requirement is 0.5 to 0.8 mEq/kg/day . The current RDA for men and women is set at 56g/d for males and 46g/d for female or 0.80g/kg body weight/day of protein 10. Potassium deficiency should be suspected: ( All Correct Except one) A. In cases of paralytic ileus. B. When the patient's reflexes are exaggerated. C. If there is a decrease in height and peaking of the T waves of an ECG. D. In alkalosis states. E. In intestinal obstruction. 11. The sodium ion: A. Is the principal regulator of the intracellular volume. B. Is the major ionic component of the intracellular fluid volume. C. Is present in greater concentration in intracellular fluid than extracellular fluid. D. Is excreted in larger amounts than normal in the early postoperative period. E. C&D only 12. Acute post traumatic renal failure:(All Correct Except One) A. May be due to hypovolaemia and poor tissue perfusion. B. Is particularly associated with crush injuries. C. May be due to kidney damage following tubular obstruction. D. Should initially be treated by fluid restriction. E. Should be treated initially by increase the amount of intravenous fluid. 13. The anuric patient: A. Should have a fluid in take of 1 to 1.5 litres per day. B. Should have no potassium administered. C. Is at risk from metabolic alkalosis. D. Should be on continuous urinary catheter drainage. E. A&D only. 14. Intravenous parenteral feeding: (All Correct Except One) A. Should deliver at least 2500 calories/day to an adult. B. Should deliver at least 10g of nitrogen (i.e 66g of protein)/day to an adult. C. Can be effectively achieved with isotonic solutions. D. Is with complications with present day solutions and methods of administration. E. Can be given by central intravenous infusion line (CVL) PARENTERAL NUTRITION PN is the intravenous administration of a hypertonic solution of carbohydrate, fat, protein, electrolytes, ... concentrations >10%, whereas PN solutions with dextrose concentrations <10% can be administered peripherally 1 2 3 4 5
D D D B A
6 7 8 9 10
B A E A B
11 12 13 14
Genito-Urinary Tract 1. Haematuria: A. At the beginning of micturition is usually indicative of urethral pathology. B. At the end of micturition is usually due to bladder neck pathology. C. Throughout the urinary stream is typical of renal pathology. D. In elderly males is usually related to benign prostatic hypertrophy.
460
A D B C
E. All of the above. 2. An intravenous pyelogram: A. Yields most diagnostic information when performed on a slightly hydrated patient. B. Should be preceded by a plain film of the abdomen. C. Normally shows incomplete filling of the ureter in any one exposure. D. Should provide evidence of the presence, if any of lower urinary tract obstruction. E. All of the above. 3. Wilms’ tumours: A. Metastasise readily to the lungs. B. Metastasise rarely to the bones. C. Are usually bilateral. D. Have the worst prognosis of all childhood abdominal tumours. E. B&C only. most common site of metastasis of wilms tumour? A. lungs B. brain C. liver D. bones 4. In renal transplantation: A. A donor kidney may be used from a patient with malignancy provided there is no abdominal involvement. B. ABO compatibility between donor and recipient does not have to be considered. C. Satisfactory renal function can be expected with a warm ischaemic time of up to 200 minutes. D. The characteristic signs of acute rejection include pyrexia, hypertension and leucocytosis. E. A&B only. 5. An adenocarcinoma of the kidney: A. Usually occurs in the 35 to 45 age group. B. Usually presents with a urinary infection. C. Is often distinguishable from a renal cyst radiologically. D. Frequently invades and grows along the renal vein. E. A&C only. 6. Tumours of the renal pelvis:( All correct except one) A. Rarely present as a mass in the loin. B. Are possibly due to a urinary carcinogen. C. Resemble those of the bladder in their histology. D. Are best treated by a partial or total nephrectomy. E. Usually cause hematuria and clot colic. 7. Ureteric calculi: A. Often result from urinary tract infection. B. Rarely cause haematuria. C. Are not usually radio-opaque. D. producing ureteric colic should be surgically removed. E. B&C only. 8. Cancer of the penis: A. Is more common in the circumcised. B. Commonly arises from the corona of glans penis. C. Is usually an adenocarcinoma. D. Rarely metastasises. E. C&D only. 9. Carcinoma of the prostate: A. Is commonly of squamous cell origin. B. Usually originates in the periphery of the gland.
461
C. Usually presents relatively early with lower urinary tract symptoms. D. Rarely can be diagnosed on rectal examination. E. All other above. 10. Carcinoma of the prostate: A. Does not usually metastasise. B. Usually produces an elevated serum acid phosphatase. C. Can be effectively treated by hormones. D. Is most effectively treated by surgery. E. B&C only. 11. Benign prostatic hypertrophy: ( all correct except one ) A. Is the result of hyperplasia of the fibromuscular capsule of the gland. B. Results in diminished power of urination. C. Results in terminal dribbling of urine. D. Often presents with haematuria. E. Might cause mild elevation of serum PSA. 12. Benign prostatic hypertrophy: A. Can readily be assessed on rectal examination. B. Can be effectively treated with hormones. C. Is most effectively treated by surgery. D. Is a premalignant condition. E. Rarely presented with macroscopic hematuria 13. Acute prostatitis: ( all correct except one ) A. Is most commonly due to coliform organisms. B. Often presents as an ache in the perineum. C. May be diagnosed by rectal examination. D. Requires bladder catherisation as part of the treatment . E. May be lead to a prostate abscess formation. 14. Bladder cancer: ( all correct except one ) A. May follow exposure to beta-naphthylamine. B. Is more common in heavy smokers. C. Is more common in females. D. Is frequently associated with bladder schistosomiasis. E. May caused by chronic urinary bladder inflamation. 15. Bladder cancers: A. Are usually adenocarcinomas. B. Are usually ulcerating. C. Usually present with suprapubic pain radiating to the perineum. D. Are usually diagnosed on cystoscopy. E. All of the above. 16. Undescended testes: A. Are often associated with inguinal herniae. B. Usually descend at puberty. C. Can usually be made to descend by the examiner with warm hands. D. Should be treated by orchidopexy at puberty. E. Can be exist without any complication. 17. The spermatic cord contains: A. The inferior epigastric vein. B. The deep circumflex iliac artery. C. The pudendal nerve. D. The subcostal nerve. E. Vasdeference. 18. Torsion of the spermatic cord: A. Often presents with vomiting and lower abdominal pain. B. Often produces gangrene of the testis. C. May be diagnosed clinically.
462
D. Always requires surgical treatment. E. All of the above. 19. Seminomas of the testis: A. Most commonly occur before the age of 40. B. Are usually sensitive to radiotherapy. C. Rarely metastasise via the blood stream. D. Generally carry a good prognosis. E. All of the above. 20. Hypospadias: A. Is the result of failure of scrotal development. B. Results in the abnormal urethra opening on to the dorsum of the penis. C. Is associated with chordee. D. Is associated with maldescent of the testis. E. C&D only. 21. The most ominous sign or symptom of urinary system disease is: A. Urinary frequency. B. Pyuria. C. Pneumaturia. D. Dysuria. E. Hematuria. 22. A patient with acute urinary tract infection (UTI) usually presents with: A. Chills and fever. B. Flank pain. C. Nausea and vomiting. D. 5 to 10 white blood cells E. Painful urination. 23. Renal adenocarcinomas: A. Are of transitional cell origin. B. Usually are associated with anemia. D. Are extremely radiosensitive. E. Frequently are signaled by gross hematuria. 24. Ureteral obstruction: A. Is associated with hematuria. B. Is associated with deterioration of renal function and rising blood urea nitrogen (BUN) and creatinine values. C. Is commonly caused by a urinary tract calculus. D. Usually requires open surgical relief of the obstruction. E. Is usually associated with infection behind the obstruction. 25. Stress urinary incontinence: A. Is principally a disease of young females. B. Occurs only in males. C. Is associated with urinary frequency and urgency. E. Is a disease of aging produced by shortening of the urethra. 26. The major blood supply to the testes comes through the: A. Hypogastric arteries. B. Pudendal arteries. C. External spermatic arteries. D. Internal spermatic arteries. 27. Patients who have undergone operations for benign prostatic hypertrophy or hyperplasia: A. Require routine rectal examinations to detect the development of carcinoma of the prostate. B. Do not need routine prostate examinations. C. Have a lesser incidence of carcinoma of the prostate. D. Have a greater incidence of carcinoma of the prostate.
463
28. To maximize fertility potential, orchidopexy for cryptorchidism should be done before: A. Age 15 years. B. Age 12 years. C. Marriage. D. Age 2 years. 29. The appropriate surgical treatment for suspected carcinoma of the testis is: A. Transscrotal percutaneous biopsy. B. Transscrotal open biopsy. C. Repeated examinations. D. Inguinal exploration, control of the spermatic cord, biopsy, and radical orchectomy if tumor is confirmed. 30. If torsion of the testicle is suspected, surgical exploration: A. Can be delayed 24 hours and limited to the affected side. B. Can be delayed but should include the asymptomatic side. C. Should be immediate and limited to the affected side. D. Should be immediate and include the asymptomatic side. 31. Epididymitis, either unilateral or bilateral, in a prepubertal male: A. Is a frequent diagnosis. B. Can be dealt with on an outpatient basis. C. Is a major scrotal problem in this age group. D. Is a rare phenomenon. 32. Patients with prostatitis, especially acute suppurative prostatitis: A. Should have residual urine measured by intermittent catheterization. B. Should have bladder decompression by urethral catheter. C. Should have repeated prostatic massage. D. Should have no transurethral instrumentation if possible. 33. Benign prostatic hypertrophy with bladder neck obstruction: A. Is always accompanied by significant symptoms. B. Is best diagnosed by endoscopy and urodynamic studies. C. Is easily diagnosed by the symptoms of frequency, hesitancy, and nocturia. D. Is always accompanied by residual urine volume greater than 100 ml. 34. A 28-year-old white male presents with asymptomatic testicular enlargement. Which of the following statement(s) is/are true concerning his diagnosis and management C. The diagnosis of seminoma should be followed by postoperative radiation therapy 35. Which of the following is/are true of blunt renal trauma? D. Blunt renal trauma requires exploration only when the patient exhibits hemodynamic instability. 36. Carcinoma of the bladder: D. May mimic an acute UTI with irritability and hematuria. 37. Within the age group 10 to 35 years, the incidence of carcinoma of the testis in males with intra-abdominal testes is: D. Twenty times greater than that in the general population. At what age is surgical orchiopexy recommended for a child with a unilateral undescended testis? A. Promptly upon discovery, regardless of age B. 1 year C. 5 to 6 years D. Any time prior to puberty E. 6 to 7 years ANSWER: B.
464
Congenital absence of one of the testes is known as: A. cryptorchidism B. anorchia C. monorchism D. ectopia E. dystopia ANSWER: C. Cryptorchidism is most frequently found: A. on the left side B. bilaterally C. on the right side D. retroperitoneally E. in the abdomen ANSWER: C. All are possible complications of an undescended testis except: A. malignization B. acute scrotum C. torsion and trauma D. hypoplasia E. sterility ANSWER: B. Upon further imaging, what associated finding would be expected? A. posterior urethral valves on a voiding cystourethrogram (VCUG) B. hydrocephalus on head ultrasound C. cardiomegaly on chest x-ray (CXR) D. bilateral adrenal enlargement on abdominal ultrasound E. tracheoesophageal fistula on an upper gastrointestinal (UGI) series ANSWER: A. Which of the following statements is false? A. Each testis descends through the inguinal canal into the scrotum within the processus vaginalis. B. A hydrocele can result from incomplete fusion of the processus vaginalis. C. A scrotal hydrocele, or simple hydrocele, is a type of non-communicating hydrocele. D. A communicating hydrocele can develop into an inguinal-scrotal hernia. Some use the terms interchangeably. E. A hernia sac can contain intestine, omentum, testis/ovary or fallopian tube. ANSWER: A. Which structure passes through the deep inguinal ring? A. Iliohypogastric nerve B. Ilioinguinal nerve C. Inferior epigastric artery D. Medial umbilical ligament E. Round ligament of the uterus ANSWER: E. The superficial inguinal ring is an opening in which structure? A. External abdominal oblique aponeurosis B. Falx inguinalis C. Internal abdominal oblique muscle
465
D. Scarpa's fascia E. Transversalis fascia ANSWER: A. If a hernia enters into the scrotum, it is most likely a: A. Direct inguinal hernia B. Indirect inguinal hernia C. Femoral hernia D. Obdurator hernia E. An incisional hernia ANSWER: B. A hernia containing the vermiform appendix in its sac is known as: A. Richter’s hernia B. Amyand’s hernia C. Littre’s hernia D. Omentocele E. Indirect hernia ANSWER: B. A patent processus vaginalis can lead to all of the following, except: A. Funiculocele B. Omphalocele C. Bubonocele D. Scrotal hernia E. Hydrocele ANSWER: B. Which of the following statements is true of infants with gastroschisis? A. It is not associated with malrotation. B. There is a high incidence of associated anomalies. C. There is not prolonged adynamic ileus following repair. D. It is complicated by intestinal atresia in 10% to 12% of cases. E. It is associated with chromosomal syndromes. ANSWER: D. The pentalogy of Cantrell includes all of the following except: A. Epigastric omphalocele. B. Sternal cleft. C. Intracardiac defect. D. Pericardial cyst. E. Ectopia cordis. ANSWER: D. Which of the following statements regarding gastroschisis are true? A. Primary fascial closure can be achieved in only about 25% of these infants B. These infants have an incidence of approximately 40% to 50% of associated anomalies C. Overall survival is approximately 80% to 90% D. When the diagnosis is known prenatally, planned cesarean section is the safest method of delivery E. It is associated with chromosomal syndromes. ANSWER: C. Which clinical feature is not present in cloacal exstrophy? A. Spinal abnormality B. Hemibladders adjacent to exposed cecum C. Gastroschisis D. Omphalocele E. None of the above ANSWER: C. An exomphalos is: A. A strangulated umbilical hernia
466
B. Synonymous with an omphalocele C. A strangulated femoral hernia D. A strangulated inguinal hernia E. A sliding hernia ANSWER: B. A patent processus vaginalis can lead to all of the following, except: A. Funiculocele B. Omphalocele C. Bubonocele D. Scrotal hernia E. Hydrocele ANSWER: B. The earliest way to diagnose an anterior abdominal wall defect is: A. by physical exam B. by history C. by fetal ultrasound D. by fetal CT scan E. None of the above ANSWER: C. The following are correct regarding omphaloceles except: A. is usually covered by a translucent membrane B. is frequently associated with other congenital malformations C. is lateral to the umbilical stump D. is within the umbilical ring E. It is a congenital disease ANSWER: C. The following are true about gastroschisis: A. occurs lateral to the umbilical stump B. can be diagnosed antenatally C. at birth often have edematous matted intestinal loops D. all of the above E. none of the above ANSWER: D. Treatment of abdominal wall defects includes: A. immediate surgical repair B. pushing the intestines back into the abdominal cavity while still in the delivery room C. provide immediate optimal resuscitation and stabilization first, and then surgery D. always do primary closure in both lesions E. conservative treatment only ANSWER: C. Gastroschisis: A. is usually associated with other anomalies B. is usually associated with chromosomal disorders C. is located on the left of the umbilical cord D. repair is followed by prolonged ileus E. none of the above ANSWER: D. Gastroschisis is associated with an increased risk of: A. hepatomegaly B. intestinal atresia C. microcephaly D. cardiac anomalies E. all of the above ANSWER: B. Which of the following statements is TRUE with respect to neonatal abdominal wall defects?
467
A. The bowel in omphalocele is covered by a sac. B. Gastroschisis is frequently associated with other anomalies. C. A Silastic silo is rarely employed in management of these defects. D. Mortality is higher in gastroschisis. E. Operative management of omphalocele usually requires bowel resection. ANSWER: A. A mother of a newborn complains of her baby's constant belching with undigested milk. Which developmental anomaly is it an evidence of? A. Esophageal atresia B. Faux lupinum C. Labium leporium D. Anal atresia E. Esophageal fistula ANSWER: A. A newborn male infant has a low anorectal anomaly. What imaging modality is currently recommended for initial imaging of his spine? A. MRT B. CT C. Spinal x-ray and ultrasound D. Bone scan E. Contrast myelography ANSWER: C. Which of the following is not a recognized feature of the VACTERL association? A. Vertebral anomalies B. Anorectal anomalies C. Eye anomalies (coloboma) D. Renal anomalies E. Limb anomalies ANSWER: C. Which of the following statements about surgical anatomy of the colon and rectum is/are correct? A. The cecum has the largest inner diameter of all segments of the colon (13 to 15 cm.). B. The rectosigmoid junction is situated at approximately 1,5 to 1,8 cm. from the anus. C. The rectum is entirely an intraperitoneal organ. D. The ileocolic, right colic, and middle colic arteries are branches from the inferior mesenteric artery. E. The arterial arcade created by communicating vessels at 1 to 2 cm. from the mesenteric is called the artery of Drummond. ANSWER: E. The test with the highest diagnostic yield for detecting a colovesical fistula is: A. Barium enema. B. Colonoscopy. C. Computed tomography (CT). D. Cystography.
468
E. Cystoscopy. ANSWER: E. Which of the following statements concerning imperforate anus is true? A. Imperforate anus affects males more frequently than females B. In 90% of males, but only 50% of females, the rectum ends below the level of the levator ani complex C. The rectum usually ends in a blind pouch D. The chance for eventual continence is greater when the rectum has descended to below the levator ani muscles E. Immediate definitive repair of the anatomic defect is required to maximize the chance of eventual continence ANSWER: D. Anal incontinence in a patient with rectal prolapse is primarily due to A. loss of anal rectal angle B. weakness of endopelvic fascia C. stretching of pudendal nerves D. all of the above E. none of the above ANSWER: C. Which type of neonatal obstruction is the most likely to be linked to an associated anomaly? A. Colonic B. Ileal C. Jejunal D. Duodenal E. All of the above ANSWER: D. In infants with duodenal atresia all the following statements are true except: A. There is an increased incidence of Down syndrome. B. Duodenal atresia can be detected by prenatal ultrasound examination. C. It may occur in infants with situs inversus, malrotation, annular pancreas, and anterior portal vein. D. It is best treated by gastroenterostomy. E. There is a high incidence of associated cardiac defects. ANSWER: D. Which of the following statements regarding duodenal atresia are true? A. Reconstruction is best achieved with duodenocolonostomy B. When associated with an annular pancreas, division of the pancreas at the site of obstruction is curative C. Bilious vomiting is typical because the obstruction is usually distal to the ampulla of Vater D. Reconstruction is best achieved with Roux-en-Y duodenojejunostomy E. All of the above ANSWER: C. On a plain abdominal radiograph two liquid levels were visible. This testifies to the child having: A. Pyloric atresia B. Ileal atresia C. Hirschsprung’s disease D. Ladd’s syndrome E. Duodenal atresia ANSWER: E. Which of the following does not cause bilious vomiting in an infant or child? A. Intestinal malrotation and volvulus B. Pyloric stenosis C. Duodenal atresia
469
D. Small bowel obstruction E. Ulcerative colitis with colonic obstruction ANSWER: B. A positive Farber’s test is indicative of which pathological state? A. Ladd’s syndrome B. Ileal stenosis C. Duodenal stenosis D. Pancreatic cystic fibrosis E. Duodenal atresia ANSWER: E. Duodenal atresia: A. is caused by intrauterine mesenteric vascular accident B. commonly exhibits normal muscular wall with a mucosal web C. is seldom associated with normal passage of meconium at birth D. is rarely associated with other congenital anomalies E. is often associated with nonbileus vomiting ANSWER: D. Which of the following statements about Hirschsprung's disease is false? A. There are no ganglion cells seen in Auerbach's plexus. B. There is an increased incidence of Down syndrome. C. It is more common in girls. D. It may be associated with enterocolitis. E. It may involve the small intestine. ANSWER: C. Which of the followings statement regarding Hirschprung’s disease are true? A. Suction rectal biopsy is virtually always diagnostic if the specimen includes submucosa B. Hirschprung’s disease is the result of a sex linked dominant gene C. The endorectal pullthrough is demonstrably superior to other forms of surgical construction D. Ten percent or more of patients have an excellent or good functional result following reconstructive surgery E. The important cause of mortality in contemporary practice is bleeding ANSWER: A. Which of the following are typical causes of neonatal intestinal obstruction? A. Intussusception B. Tumors C. Hirschprung’s disease D. Meckel’s diverticulum E. Incarcerated hernia ANSWER: C. All of the following are associated with an increased incidence of abdominal wound dehiscence EXCEPT A. bringing a stoma through the incision B. closure of the wound with a continuous suture C. presence of ascites D. presence of jaundice E. use of braided sutures ANSWER: B. The most effective method of treating Ladd’s syndrome is: A. Conservative management B. Ladd’s operation
470
C. Soave’s operation D. Colostomy E. Hirschsprung’s operation ANSWER: B. Which of the following is the most informative in diagnosis of HD? A. Biopsy B. Plain radiography C. Contrast enema D. Endoscopy E. Ultrasonography ANSWER: A. Which of the following is not a method of surgical management in HD? A. Duhamel’s operation B. Swenson’s operation C. Soave’s operation D. Ladd’s operation E. Rehbein’s operation ANSWER: D. Which is not indicative of HD? A. Aganglionic bowel on biopsy B. Abnormal recto-anal inhibitory reflex C. Abdominal distention D. Abdominal pain E. Constipation ANSWER: D. What is the most common serious complication of an end colostomy? A. Bleeding B. Skin breakdown C. Parastomal hernia D. Colonic perforation during irrigation E. Stomal prolapsed ANSWER: C. Which of the following statements concerning Hirschsprung’s disease is true? A. It is initially treated by colostomy B. It is best diagnosed in the newborn period by barium enema C. It is characterized by the absence of ganglion cells in the transverse colon D. It is associated with a high incidence of genitourinary tract anomalies E. It is the congenital disease that most commonly leads to subsequent fecal incontinence
471
ANSWER: A. Which of the following is the most informative in diagnosing Ladd’s syndrome? A. Complete blood count B. Plain thoracic radiography C. Bronchoscopy D. Contrast radiography E. Plain abdominal radiography ANSWER: D. On a plain abdominal radiograph two liquid levels were visible. This testifies to the child having: A. Pyloric atresia B. Ileal atresia C. Hirschsprung’s disease D. Ladd’s syndrome E. Duodenal atresia ANSWER: E. The most effective method of treating Ladd’s syndrome is: A. Conservative management B. Ladd’s operation C. Soave’s operation D. Colostomy E. Hirschsprung’s operation ANSWER: B. Which of the following signs and symptoms warrant surgical intervention for patients with small bowel obstruction? A. abdominal tenderness B. air-fluid levels on abdominal x-ray C. worsening abdominal pain D. feculent vomitus E. air in the colon and rectum on abdominal x-ray ANSWER: C. Which of the following does not cause bilious vomiting in an infant or child? A. Intestinal malrotation and volvulus B. Pyloric stenosis C. Duodenal atresia D. Small bowel obstruction E. Ulcerative colitis with colonic obstruction ANSWER: B. A 3-wk-old baby, previously well, presents with sudden onset of bilious vomiting. What study is most appropriate? A. Abdominal X-ray. B. CT scan. C. Upper GI series. D. Barium enema. E. Esophageal pH studies. ANSWER: C. What organ systems are affected by CF and what are the clinical manifestations? A. Pulmonary B. Gastrointestinal C. Hepatic D. Reproductive E. All of the above ANSWER: E.
472
How is CF inherited? A. Autosomal Dominant B. Autosomal Recessive C. Sex-linked Recessive D. Sex-linked Dominant E. None of the above ANSWER: B. The treatment of choice for neonates with uncomplicated meconium ileus is: A. Observation. B. Emergency laparotomy, bowel resection, and Bishop-Koop enterostomy. C. Intravenous hydration and a gastrograffin enema. D. Emergency laparotomy, bowel resection, and anastomosis. E. Sweat chloride test and pancreatic enzyme therapy. ANSWER: C. Regarding meconium ileus A. Is a rare cause of neonatal intestinal obstruction B. Less than 5% of cases are associated with cystic fibrosis C. Obstruction usually occurs in the distal ileum D. Presents with neonatal bile-stained vomiting and abdominal distension E. A plain x-ray may show an intra-luminal 'ground glass' appearance ANSWER: A. All the following statements about cystic fibrosis are true EXCEPT A. the incidence in whites is about 1 in 2000 B. it may present in the newborn period with meconium ileus C. it is an autosomal dominant disease D. hepatic involvement is due to inspissated biliary secretions E. chronic hypoxia, hypercapnia, and acidosis produce pulmonary hypertension ANSWER: C. Persistent vomiting with hypertrophic pyloric stenosis usually causes what electrolyte imbalance? A. Metabolic acidosis B. Metabolic alkalosis C. Respiratory acidosis D. Respiratory alkalosis E. None of the above ANSWER: B. Of the following cystic malformations of the tracheobronchial tree, which is most likely to be asymptomatic when discovered? A. Intralobar pulmonary sequestration B. Extralobar pulmonary sequestration C. Congenital cystic adenomatoid malformation D. Congenital lobar emphysema E. None of the above ANSWER: B. The definitive evaluation of a child with a suspected congenital cystic abnormality of the tracheobronchial tree is best done using which of the following? A. Rigid bronchoscopy B. Computerized tomography or magnetic resonance imaging
473
C. Chest x-ray D. Angiography E. Barium esophagogram ANSWER: B. Which of the following lesions contain no cartilage? A. Bronchogenic cyst B. Congenital cystic adenomatoid malformation C. Congenital lobar emphysema D. Both of the above E. Neither of the above ANSWER: B. Which of the following lesions is a form of foregut cyst? A. Bronchogenic cyst B. Congenital cystic adenomatoid malformation C. Congenital lobar emphysema D. Both of the above E. Neither of the above ANSWER: A. Which of the following lesions frequently cause symptoms by mass effect? A. Bronchogenic cyst B. Congenital cystic adenomatoid malformation C. Congenital lobar emphysema D. Bronchogenic cyst & Congenital cystic adenomatoid malformation E. Neither of the above ANSWER: D. Which type of CCAM has the best prognosis? A. Type 0 B. Type I C. Type II D. Type III E. Type IV ANSWER: B. Which type of CCAM is most common? A. Type 0 B. Type I C. Type II D. Type III E. Type IV ANSWER: B. Which statement is false regarding extrapulmonary sequestration? A. The parenchyma is not connected to the tracheobronchial tree B.
Arterial blood supply is systemic
C.
Venous blood supply is pulmonary
D.
Most frequently in males
E.
Commonly associated with other anomalies
ANSWER: C.
474
In neonates with congenital diaphragmatic hernia, which of the following statements is true? A. The defect is more common on the right side. B. Survival is significantly improved by administration of pulmonary vasodilators. C. An oxygen index of 20 is an indication for extracorporeal membrane oxygenation (ECMO). D. Oligohydramnios is a frequent occurrence. E. Mortality is the result of pulmonary hypoplasia. ANSWER: E. The most common type of congenital diaphragmatic hernia is caused by: A. A defect in the central tendon. B. Eventration of the diaphragm in the fetus. C. A defect through the space of Larrey. D. An abnormally wide esophageal hiatus. E. A defect through the pleuroperitoneal fold. ANSWER: E. Which of the following statements regarding congenital diaphragmatic hernia are true? A. The incidence of right and left-sided lesions is equal B. Malrotation is not to be expected C. Left-to-right shunting via a patent ductus arteriosus is a serious but expected physiologic consequence of pulmonary hypoplasia D. Survival rates of 5% are reported in several contemporary series E. Congenital heart disease is present in approximately 20% of these infants ANSWER: E. There is an emerging consensus that the surgical repair for congenital diaphragmatic hernia is best done: A. Emergently at the bedside, eliminating the risks of transporting an unstable neonate B. While on extracorporeal membrane oxygenation C. When the infant is potentially extubatable D. Within the first 48 to 72 hours of life E. All of the above ANSWER: B. Which of the following hernias is diaphragmatic? A. Richter’s hernia B. Bogdalech’s hernia
475
C. Amyand’s hernia D. Littre’s hernia E. None of the above ANSWER: B. In congenital diaphragmatic hernia all are seen exeptA. Common on left side B. Abdominal distension C. Can be detected antenatally D. Heart beat shifted to right E. Bowel sound in the chest ANSWER: B. The earliest way to diagnose a diaphragmatic hernia is: A. by physical exam B. by history C. by fetal ultrasound D. by fetal CT scan E. by fetal MRI ANSWER: C. The following are correct regarding diaphragmatic hernia except: A. is usually on the left side B. is frequently associated with hypoplastic lungs C. can present similar to a tension pneumothorax D. bowel sound in the chest E. is frequently asymptomatic at birth ANSWER: E. The following are true about diaphragmatic hernias: A. often have scaphoid abdomen on exam B. can be diagnosed antenatally by ultrasound C. at birth often have persistent cyanosis and respiratory distress D. all of the above E. none of the above ANSWER: D.
476
Treatment of diaphragmatic hernia includes: A. immediate surgical repair B. pulling the intestines back into the abdominal cavity while still in the delivery room C. provide immediate optimal resuscitation and stabilization first, and then surgery D. always do primary closure of the diaphragm E. all of the above ANSWER: C. The true statement for diaphragmatic hernia below is: A. The surgeon does not need to worry about medical problems as the neonatologist will already have treated them. B. There are essentially no medical problems after surgical repair. C. Improved ultrasound diagnosis has resulted in some women seeking termination of pregnancy. D. The long term outcome of survivors reveals no significant chronic pulmonary problems. E. All of the above ANSWER: C. The main cause of postoperative death in children with chronic diaphragmatic hernia is: A. increased intra-abdominal pressure B. persistent lung collapse C. patent ductus arteriosus D. abnormal pulmonary microvasculature E. all of the above ANSWER: D. In determining the proper treatment for a sliding hiatal hernia, the most useful step would be A. Barium swallow with cinefluoroscopy during Valsalva maneuver B. Flexible endoscopy C. 24-h monitoring of esophageal pH D. Measuring the size of the hernia E. Assessing the patient’s smoking and drinking history ANSWER: B. Esophageal atresia is diagnosed immediately after birth on the basis of: A. Excessive salivation and foaming at the mouth B. Excessive vomiting on feeding C. Esophageal catheterization D. Respiratory distress E. Abdominal distention
477
ANSWER: C. Which of the following statements about the anatomic course of the esophagus is correct? A. The cervical esophagus passes behind and to the right of the trachea. B. The thoracic esophagus enters the posterior mediastinum anterior to the aortic arch. C. The thoracic esophagus passes behind the right mainstem bronchus and the pericardium. D. The esophagus enters the diaphragmatic hiatus at the level of T8. E. The esophagus deviates anteriorly and to the left as it enters the abdomen. ANSWER: E. Which of the following statements about esophageal anatomy is correct? A. The esophagus has a poor blood supply, which is segmental in distribution and accounts for the high incidence of anastomotic leakage. B. The esophageal serosa consists of a thin layer of fibroareolar tissue. C. The esophagus has two distinct muscle layers, an outer, longitudinal one and an inner, circular one, which are striated in the upper third and smooth in the distal two thirds. D. Injury to the recurrent laryngeal nerve results in vocal cord dysfunction but does not affect swallowing. E. The lymphatic drainage of the esophagus is relatively sparse, localized primarily to adjacent paraesophageal lymph nodes. ANSWER: C. The best management for a 48-hour-old distal esophageal perforation is: A. Antibiotics and drainage. B. Division of the esophagus and exclusion of the perforation. C. Primary repair with buttressing. D. Resection with cervical esophagostomy, gastrostomy, and jejunostomy. E. T-tube fistula and drainage. ANSWER: C. An infant with a history of recurrent pneumonia is diagnosed with TEF at 8 months of age. Which of the following statements is correct? A. The infant most likely has a “H-type” TEF. B. The infant most likely has proximal esophageal atresia with distal fistula. C. The infant likely has a previously undetected, associated finding of imperforate anus. D. The infant is unlikely to have gastroesophageal reflux. E. The infant is likely to have cystic fibrosis. ANSWER: A.
478
A 2-year-old girl with a history of esophageal atresia and a ventricular septal defect is hospitalized with Pneumocystis carinii pneumonia. Her immunodeficiency is likely a result of which of the following? Bruton agammaglobulinemia Chronic granulomatous disease DiGeorge syndrome Hyperimmunoglobulin E syndrome Severe combined immunodeficiency syndrome ANSWER: C. During inguinal herniotomy in a 12 year old the testis was found in the hernia sac. What type of hernia is this? Richter’s hernia Congenital indirect hernia Acquired direct hernia Direct hernia Femoral hernia ANSWER: B. Which of the following statement(s) is/are true concerning the diagnosis and management of epigastric hernias? A. A large peritoneal sac containing abdominal viscera is common B. At the time of surgical repair, a careful search for other defects should be performed C. Recurrent epigastric hernias after simple closure is uncommon D. Patients with symptoms of a painful midline abdominal mass frequently will contain incarcerated small bowel E. Patients with symptoms of a painful midline abdominal mass frequently will contain incarcerated large bowel ANSWER: B. A number of special circumstances exist in the repair of inguinal hernias. The following statement(s) is/are correct. Simultaneous repair of bilateral direct inguinal hernias can be performed with no significant increased risk of recurrence The preperitoneal approach may be appropriate for repair of a multiple recurrent hernia A femoral hernia repair can best be accomplished using a Bassini or Shouldice repair Management of an incarcerated inguinal hernia with obstruction is best approached via laparotomy incision All of the above ANSWER: B. Spontaneous closure of which of the following congenital abnormalities of the abdominal wall generally occurs by the age of 4? A. Umbilical hernia B. Patent urachus C. Patent omphalomesenteric duct D. Omphalocele E. Gastroschisis ANSWER: A.
479
Upon delivery of a baby with a small intact omphalocele and an enlarged tongue, which of the following is foremost in the physician’s mind in the management of this infant? A. Cover the omphalocele membrane with plastic wrap to prevent loss of body warmth and fluid. B. Insert an orogastric tube to decompress the intestinal tract. C. Promptly arrange consultation with a pediatric surgeon to repair the omphalocele and assess the baby's airway. D. Insert an IV to correct fluid and electrolyte losses. E. Insert an IV to provide glucose. ANSWER: E. Whenever the intestinal midgut loop fails to return from the umbilical cord into the abdominal cavity, the defect is known as which of the following? A. gastroschisis B. an omphalocele C. Meckel’s diverticulum D. a vitelline fistula E. a left-sided colon ANSWER: B. A neonate in whom the anus was noted to be absent was observed to be occasionally passing urine with traces of meconium, and clear urine on other occasions. Which type of anomaly is present? A. Anal atresia B. Anal atresia with rectourethral fistula C. Anal atresia with rectovesical fistula D. Rectal atresia E. Cloacal malformation ANSWER: B. A male infant weighing 3 kg is born via spontaneous vaginal delivery at 37 weeks’ gestation. His Apgar score is 6/9 at 1 and 5 minutes. The patient is in no apparent distress. Physical examination reveals no anus. What is the most appropriate initial step in this patient’s management? A. Colostomy B. Continued observation for 24 hours C. Intubation and mechanical ventilation D. Magnetic resonance imaging (MRI) of the abdomen and pelvis E. Posterior sagittal anorectoplasty ANSWER: B. The infant begins to have progressively large amounts of bilious emesis. The infant feeds well and has only a small amount of abdominal distention. What is the most likely diagnosis? A. pyloric stenosis B. Hirschsprung disease C. biliary atresia D. duodenal atresia E. milk protein allergy ANSWER: D.
480
The infant have duodenal atresia. If you were to perform an abdominal x-ray, what is the most likely finding that would be seen? A. “double-bubble” sign B. scimitar sign C. normal gas patterns D. free fluid in the abdomen E. pneumatosis intestinalis ANSWER: A. An infant begins to vomit shortly after birth and his abdomen becomes distended. A radiograph of the abdomen shows a "double-bubble" gas shadow. The infant's problem most likely is A. congenital megacolon B. malrotation C. duodenal atresia D. esophageal atresia E. tracheoesophageal fistula ANSWER: C. A neonate has bilious vomiting and a doublebubble sign on plain x-ray. The most appropriate operation is: A. division of annular pancreas B. gastroenterostomy C. duodenoduodenostomy D. duodenal resection E. duodenojejunostomy ANSWER: C. The nurse points out a two day old healthy term infant who is otherwise ready for discharge who still has not passed meconium. Your next step is: A. Order a suppository prior to discharge. B. Careful physical examination, including digital rectal examination. C. Give a normal saline enema to prep for a barium enema. D. Call radiologist to discuss an unprepped barium enema E. Rectal biopsy. ANSWER: D.
481
482
483
484
485
486
487
488
489
490
Which of the following causes adaynamic intestinal obstruction A . bezoars B . annular pancreas C . peritonitis D . volvulus E . intussusceptions In traumatic rupture of the liver A . the left lobe is more commonly involved than the right B . the spleen escapes the injury C . major hepatic resection is more likely indicated D . packing of the liver should be avoided E . hemobilia is recognized feature In acute pancreatitis all are true except A . commonly associated with gall bladder stones B . has high incidence with alcoholics C . produces exaggerated bowel sounds D . may have a viral origin E . may be complicated be renal failure The most common complication associated with Meckels diverticulum is A . bleeding B . perforation C . obstruction D . carcinoma E . non of the above Useful methods for detection of splenic injury, in descending order of sensitivity, are: A. Diagnostic peritoneal lavage. B. CT. C. Ultrasonography. D. Isotope scan. E. Magnetic resonance imaging (MRI). Which of the following statements about diagnostic peritoneal lavage (DPL) is false? A. DPL is the diagnostic procedure of choice for gunshot wounds to the abdomen with no obvious intra-abdominal injuries. B. The average reported incidence of false-positive DPL in patients with significant pelvic fractures is 20% to 30%. C. Accuracy rates for DPL have generally been reported between 95% and 97%. D. DPL has superior value than computed tomography as the diagnostic procedure of choice following blunt abdominal trauma. The most common indication for surgery secondary to acute diverticulitis is: A. Abscess. B. Colonic obstruction. C. Colovesical fistula. D. Free perforation. E. Hemorrhage.
491
Axial twisting of the right colon or cecal volvulus has been shown to be associated with each of the following except: A. A history of abdominal operation. B. A mobile cecum. C. An obstructing lesion in the transverse or left colon. D. Inflammatory bowel disease Signs of acute diverticulitis include all the following except A . tender lower abdominal mass B . fever C . leukocytosis with right shift in the differential D . high fever E . diarrhea Appropriate management of suspicions acute diverticulitis includes which of the following? a. Barium enema b. Computed tomography of the abdomen c. Immediate laparotomy d. Intravenous pyelogram E . MRI Complete mechanical small bowel obstruction can cause dehydration by except: A. Interfering with oral intake of water. B. Inducing vomiting. C. increased intestinal absorption of water. D. Causing secretion of water into the intestinal lumen. E. Causing edema of the intestinal wall. Which of the following are important for the clinical diagnosis of small bowel obstruction except? A. Crampy abdominal pain. B. Fever. C. vomiting. D. Abdominal distention. E. Leukocyte count above 12,000. Patients with established, complete, simple, distal small bowel obstruction usually have the following findings on plain and upright abdominal radiographs: A. Distended small bowel identifiable by the valvulae conniventes. B. Free air. C. Modest amount of gas in the pelvis. D. Peripheral, rather than central, distribution of gas. E. Prominent haustral markings. For which of the following consequences of radiation injury of the intestine is urgent laparotomy required? A. Small bowel obstruction. B. Colonic perforation. C. Rectovaginal fistula.
492
D. Malabsorption and diarrhea. E. Rectal stenosis. Which of the following statement is true concerning postoperative ileus? a. The use of intravenous patient-controlled analgesia has no effect on return of small bowel motor activity b. The presence of peritonitis at the time of the original operation delays the return of normal bowel function c. The routine use of metoclopramide will hasten the return of small intestinal motor activity d. Contrast radiographic studies have no role in distinguishing early postoperative bowel obstruction from normal ileus The initial management of this patient with postoperative ileus should consist of: a. Fluid resuscitation with D5 half normal saline with 40 mEq of potassium chloride/liter b. The patient should be begun on broad spectrum antibiotics at the time of admission c. Nasogastric decompression with a nasogastric tube d. Immediate surgery Which of the following parameters is/are not included in the Ranson's prognostic signs useful in the early evaluation of a patient with acute pancreatitis? A. Elevated blood glucose. B. Leukocytosis. C. Amylase value greater than 1000 U per dl. D. Serum lactic dehydrogenase (LDH) greater than 350 IU per dl. E. Alanine aminotransferase less than 250 U per dl. Standard supportive measures for patients with mild pancreatitis include the following: A. Intravenous fluid and electrolyte therapy. B. Withholding of analgesics to allow serial abdominal examinations. C. Subcutaneous octreotide therapy. D. Nasogastric decompression. E. Prophylactic antibiotics. With regard to acute pancreatitis: which of the following statement is correct? a. The majority of patients presenting with acute pancreatitis of biliary type are female b. The majority of patients presenting with acute pancreatitis of alcoholic type are female c. The most common cause of acute pancreatitis in our population is alcohol use d. Patients with alcohol-induced pancreatitis tend to be older than those with biliary-induced disease 1.
493
Zollinger Ellison syndrome is NOT characterized by: a. lymphadenopathy b. marked increases in gastric acid secretion c. ulcer disease of the upper gastrointestinal tract
2.
d. non-beta islet cell tumors of the pancreas High plasma insulin levels has the following effects
a. It stimulates breakdown of adipose tissue b. It stimulates the storage of glycogen in the liver c. It enhances protein catabolism d. It increases the absorption of glucose from the intestine 3. People with Type 1 diabetes a. Are old and mostly over 30 years b. Type 1 diabetes can be due to autoimmune disease causing damage to the beta cells c. The disease may be caused by helminthic infestation d. The disease develops gradually in its clinical onset 4. a. b. c. d. e. 5. a. b. c. d.
a. b. c. d.
The most important function of insulin is … To release glucose from the liver To maintain normal blood glucose level To avoid late diabetic complications To utilize fat
6. In Type 1 diabetes Insulin is unable to promote utilization of glucose Pancreas does not produce insulin The cells of the body lack insulin receptors Blood glucose level may be normal 7. a. b. c. d.
a. b. c. d.
In diabetes mellitus People with Type 2 diabetes do easily develop ketoacidosis Type 2 diabetes is a mild disease People with Type 2 diabetes are usually less than 30 year old Obesity is a risk factor
Metformin … is an example of a sulphonylurea. Does not stimulate insulin secretion. Causes weight gain. Has no gastrointestinal side effects
8. What treatment modes should be used in type 2 diabetes in order of importance? Education, diet, exercice, OA’s, insulin . OA’s, exercise, diet, insulin, education. Exercise, diet, OA’s. Education, insulin, OA’s, exercise, diet.
9. The classical endocrine action a. Hormone may be secreted by a neighboring cell. b. Hormone may be secreted by the same cell. c. Hormone carried via blood from a secreting glan d. d. Hormone may be manufactured in situ. 10. Excess growth hormone secretion leads to a. Gigantism if it occurs after puberty.
494
b. Acromegaly if it occurs before puberty. c. Gigantism if it occurs before puberty. d. Diabetes insipidus if it occurs after puberty.
11. a. b. c. d.
a. b. c. d.
The hormone vasopressin (AVP)
Is synthesized in the posterior pituitary. Its secretion is defective in diabetes insipidus. Is important for lactation. Is released by a trophic hormone from hypothalamus. 12. a. b. c. d.
The level of total thyroxin (T4) Will be increased during pregnancy. Is usually normal in thyrotoxicosis. Is usually elevated in hypothyroidism. May be increased due to TBG deficiency.
13. a. b. c. d.
In Graves' disease TSH is usually elevated. T4 elevated and T3 is low. Both FT4 and FT3 are elevated. ACTH is elevated.
14. Congenital adrenal hyperplasia Is due to ACTH deficiency. Is due to pituitary adenoma. Usually caused by 21-hydroxylase deficiency. Estrogen is produced in excess. 1. Cortisol synthesis would be diminished by A. A defect in 11-hydoxysteroid dehydrogenase B. A defect in 21-hydoxylase C. A defect in 18-hydoxylation D. A defect in 5-alpha-reductase E. A defect in aromatase 2. You have a brother who has type 1 diabetes. He has had severe hypoglycemia in the past, without warning. Therefore, you have 1 mg of glucagon available for injection in this situation. When you give the glucagon, which of the following will happen? A.Glycogenolysis and gluconeogenesis will promptly increase. B. The glucose transporter GLUT4 will translocate to the plasma membrane, causing brain glucose uptake to increase. C. The tyrosine kinase activity of the glucagon receptor will be turned on. D. Lipolysis will be suppressed. E. Nothing, because you have to give some form of glucose along with the glucagon in order for it to work. 3. On physical examination a patient is noted to have tendon xanthomata. Which statement regarding this patient is true:
495
A. Lipemia retinalis is often seen in association with this physical exam finding. B. This patients triglyceride levels are usually elevated. C. Usually a genetically determined defect in the LDL receptor causes lipid abnormalities associated with this physical finding. D. The condition is not associated with risk of early coronary artery disease.
4. Thyroid hormone stored in the lumen of follicles is in the form of a. free T3 b. free T4 c. attached to thyroglobulin in the gland d. attached to thyroid binding globulin Choose only the ONE most Appropriate Answer All of the following can be treated conservatively in a stable trauma patient except: a. Lung contusion. b. Liver laceration. c. Kidney laceration. d. Splenic hematoma. e. Perforation of the small intestine.
1.
2.
One of the following is not a consequence of excessive increase in intra-abdominal pressure: a. Increased cardiac output. b. Decrease in preload. c. Increase in intracranial pressure. d. Decrease in renal perfusion. e. Decrease in mesenteric perfusion.
3.
Mediastinal widening after deceleration type chest trauma is a pathognomonic sign of: a. Cardiac injury. b. Aortic rupture. c. Bronchial rupture. d. Thymic injury. e. Sternal frature.
4.
The initial maneuver to establish an airway in a patient with multiple injuries is: a. Oropharyngeal airway. b. Uncuffed endo-tracheal tube. c. Suctioning foreign debris and lifting up the mandible. d. Cuffed endo-tracheal tube. e. Tracheostomy.
5.
For which of the following organ injuries is diagnostic peritoneal lavage (DPL) least likely to be helpful? a. Pancreas b. Small intestine. c. Spleen. d. Sigmoid colon. e. Liver.
496
6.
7.
8.
9.
10.
11.
12.
13.
497
Which is the most commonly injured intra-abdominal organ in blunt trauma? a. Pancreas. b. Kidney. c. Spleen. d. Stomach. e. Colon. The most commonly injured structure in blunt neck trauma is the: a. Vertebral column. b. Right common carotid artery. c. Left internal carotid artery. d. Right internal jugular vein. e. Left internal jugular vein. The factor most commonly associated with central venous catheter related nosocomial infection is: a. Sterility of the insertion technique. b. Type of solution being administered through the line. c. Care of the catheter once inserted. d. Proximity of the catheter to another wound. e. Presence of another infection else where. One is not true regarding post-splenectomy sepsis: a. Fatality reaches 50% of cases. b. The risk is higher for patients who undergo splenectomy for trauma. c. Majority of cases occur within two years following splenectomy. d. The greatest risk is in children under 5 years of age. e. Pneumococcus is the organism responsible for about 50% of septic episodes. Which of the following is not true about (SIRS): a. It may be caused by peritonitis, burns and acute pancreatitis. b. The white leucocytic count may be reduced below 4000. c. There is metabolic acidosis. d. The temp may drop to 36 ºC. e. There is reduced output of catecholamines. Which of the following is an early manifestation of sepsis: a. Decreased cardiac output. b. Hypoglycaemia. c. Cutaneous vasodilatation. d. Increased arteriovenous O2 difference. e. Respiratory acidosis. Which of the following is the most likely cause of pyrexia occurring 48 hours after an abdominal operation? a. Wound infection. b. D.V.T. c. Chest infection. d. Pulmonary embolism. e. Leaking intestinal anastomosis. Which of the following is true about gas gangrene: a. The commonest responsible organism is clostridium difficile. b. The infection is mainly in the subcutaneous tissues.
c.
Anti-gas gangrene serum is the most important item of treatment. d. The disease is characterized by rigidity and muscle spasms. e. The disease usually follows lacerations involving the gluteal area and thigh. 14.
Factors that make lacerated wounds more liable to infection include the following except: a. Presence of foreign bodies. b. Ischaemic or dead tissues. c. Suture of the wound under tension. d. Leaving the wound open after debridement. e. The use of internal fixation for accompanying fractures.
15.
One of the following is an anaerobic organism: a. Staphylococcus aureus. b. E. coli. c. Klebsiella. d. Pseudomonas aeroginosa. e. Bacteriodes.
16.
In tetanus infection the following statements are correct except: a. The disease is caused by the toxin of Clostridium tetani. b. The toxin travels along nerves to the central nervous system. c. The spasm can stop respiration. d. It may affect the neonate. e. Treatment should include blood transfusion and IV aminoglycoside. In pseudomembranous enterocolitis the causing organism is: a. Clostridium perfringens. b. Clostridium septicum. c. Clostridium difficle. d. Clostridium tetani. e. Clostridium oedematien.
17.
18.
19.
20.
498
The advantages of third-generation cephalosporin over aminoglycosides in the treatment of a mixed intra-abdominal infection is: a. Lower incidence of ototoxicity. b. A greater spectrum of coverage. c. Prevention of endocarditis in valvular heart disease. d. Prophylaxis against post-splenectomy sepsis. e. A shorter duration of therapy. All the following may be an indication for liver transplantation except: a. Liver cirrhosis. b. Acute fulminant hepatitis. c. Malignant obstructive jaundice due to periampullary cancer. d. Congenital biliary atresia. e. Wilson’s disease. Which of the following is wrong about hyperacute rejection? a. It is due to the presence of preformed antibodies. b. It occurs within hours of transplantation. c. It can be treated by corticosteroids.
d. It is best treated by removal of the graft. e. Cyclosporin A did not reduce its incidence.
21.
Cellular mediated immunity is achieved principally by: a. Dendritic cells. b. B-lymphocytes. c. Monocytes. d. T-lymphocytes. e. Natural killer cells.
22.
The commonest complication following liver transplantation is: a. Hepatic artery thrombosis. b. Portal vein thrombosis. c. Hepatic artery stenosis. d. Biliary problems. e. Non function of the graft.
23.
The most effective drug used in the treatment of cardiac arrest is: a. Sodium bicarbonate. b. Calcium chloride. c. Atropine sulfate. d. Dopamine. e. Epinephrine. The cardinal feature in all cases of cardiac arrest in the operating room is: a. Hypercapnea. b. Hyperkalemia. c. Respiratory alkalosis. d. Metabolic acidosis. e. Hypoxemia.
24.
25.
26.
499
Regarding septic shock all of the following are true, except: a. Cold shock has a better outcome than warm shock. b. The gastrointestinal tract plays a major role in the multiorgan failure syndrome. c. Is caused by Candida in around 10% of cases. d. Hyperventilation is an early sign of pre-shock septic state. e. The respiratory tract is the primary focus in ICU’s in the majority of cases. Which of the following is the most accurate investigation to diagnose pulmonary embolism? a. Ventilation perfusion scan. b. Pulmonary angiography. c. ECG. d. Blood gases. e. Chest x-ray.
27.
High pulmonary artery wedge pressure is present in: a. Cardiogenic shock. b. ARDS. c. Septic shock. d. Neurogenic shock. e. Anaphylactic shock.
28.
Which of the following is not true about neurogenic shock? a. It may follow spinal fractures. b. There is tachycardia. c. The extremities are warm. d. There is bradycardia and hypotension. e. Vasopressors may be useful.
29.
The most appropriate therapy for metabolic alkalosis is: a. Correct the volume depletion. b. IV HCl. c. Intubation and controlled hypoventilation. d. Administration of acetazolamide to promote renal excretion of bicarbonates. e. Stop nasogastric suction.
30.
In paraplegia of acute onset all of the following is helpful in diagnosing the cause of paralysis except: a. Feeling lower limb pulsations. b. Duplex scan of the arterial system of the lower limbs. c. Duplex scan of the venous system of the lower limbs. d. Magnetic resonance imaging of the dorsolumbar spine. e. Asking for history of trauma to the spine.
31.
In a rural health unit you received a patient with acute ischaemia of 5 hours duration. You decided to send the patient to the nearest specialized hospital. You expected that the patient will reach the hospital within 3 hours. Which one of the following drugs is the most important to give to the patient before sending him to the hospital? a. Warfarin. b. Heparin. c. Aspirin. d. A potent analgesic. e. A vasodilator.
32.
A 50-year-old diabetic patient presented with fever (39 ºC) and anorexia. His foot showed an indurated red tender area extending in the sole from the web between the second and third toes up to the medial arch of the foot. The patient had palpable posterior tibial pulse but the dorsalis pedis pulse was not felt. Immediate surgical drainage was performed through a 2.0 cm incision placed at the center of the indurate area. Pus was evacuated by squeezing from above and below the incision. The patient received a combination of a first generation cephalosporin, a quinolone and metronidazole. Dressing was performed twice daily with povidone-iodine and hydrogen peroxide. However, after one-week the patient’s fever did not subside,
500
he started to vomit and the infection in the foot spreaded up into the leg. The cause of inadequate response to treatment is: a. The patient needed arterial reconstruction in addition to drainage. b. The incision was too small for adequate drainage. c. The antibiotics given were not suitable. d. Dressings were too infrequent. e. Povidone iodine and hydrogen peroxide were not suitable antiseptics. 33.
The optimum management of the previous patient is: a. Antibiotics and intravenous fluids as the patient will not tolerate surgery. b. Multiple incisions in the foot and leg to drain pus + antibiotics. c. Correction of the general condition within few hours followed by below knee amputation. d. Correction of the general condition within few hours followed by above knee amputation. e. Correction of the general condition within few days followed by above knee amputation.
34.
Which of the following ulcers is the least likely to be present in the foot? a. Venous ulcer. b. Ulcer due to arterial insufficiency. c. Neuropathic ulcer. d. Decubitus ulcer (bed sore). e. Traumatic ulcer.
35.
What is the best treatment for a severe diffuse atherosclerotic bilateral aorto-iliac occlusion in a high risk patient? a. Endarterectomy. b. Aortobifemoral bypass. c. Axillo-bifemoral bypass d. Angioplasty. e. Stenting.
36.
The commonest cause of mortality following repair of an abdominal aortic aneurysm is: a. Leakage at the site of repair. b. Cerebrovascular stroke. c. Cardiac complication. d. Distal embolism. e. Acute renal failure. Leriche syndrome includes all of the following except: a. Aortoiliac disease. b. Absent femoral pulses. c. Impotence. d. Intermittent claudication. e. Migratory superficial thrombophlebitis.
37.
501
38.
Which of the following drugs is the drug of choice for the treatment of acute deep venous thrombosis during pregnancy? a. Heparin. b. Warfarin. c. Streptokinase. d. Urokinase. e. Aspirin.
39.
The following measure provides the most rapid reversal of warfarin anticoagulation in cases of bleeding from warfarin over dosage: a. Vitamin K intramuscularly. b. Vitamin K intravenously. c. Protamine sulphate intravenously. d. Fresh frozen plasma. e. Platelet transfusion.
40.
All of the following are characteristic features of an ulcer caused by severe arterial insufficiency except: a. It is painful. b. Its floor is formed of healthy easily bleeding granulation tissue. c. It is located in the toes, foot or lower part of the leg. d. It is associated with symptoms and signs of chronic ischaemia. e. It is resistant to healing.
41.
Which of the following is not true about compartment syndrome? a. It may occur following the application of a tight plaster. b. It may lead to Volkmann's ischaemic contracture. c. It cannot be ruled out, if a good peripheral pulse is present. d. Angiography is important for the diagnosis. e. It is characterized by severe pain in the forearm after extension of the fingers.
42.
One of the following has a comparative poor prognosis in breast cancer patients: a. Cancer in old age after 60. b. Mucoid carcinoma type. c. C-erb2 gene positive tumors. d. Estrogen receptor positive tumors. e. Non-familial breast cancer.
43.
Concerning tamoxifen therapy all are true except: a. Is an estrogen agonist. b. As an adjuvant therapy for breast cancer it should be continued for 5 years. c. May be the sole therapy for breast cancer in elderly patients. d. May produce menopausal symptoms. e. Is associated with an increased risk of endometrial cancer.
44.
In malignant melanoma which of the following correlates best with the prognosis?
502
a. b. c. d. e.
Age of the patient. Gender. Clark’s level. Site of the lesion. Breslew thickness of the lesion.
45.
Which of the following tumors is not highly radio-sensitive? a. Medulloblastoma. b. Nephroblastoma. c. Hodgkin’s lymphoma. d. Malignant melanoma. e. Seminoma.
46.
Which of the following factors is associated with the highest risk of developing breast cancer? a. Age of menarche is below 13 years. b. Apocrine metaplasia. c. Mutation of the BRCAI gene. d. First degree relative with breast cancer. e. Florid hyperplasia.
47.
Which of the following is the most important prognostic factor for soft tissue sarcoma? a. Size of the lesion. b. Site of the lesion. c. Grade of malignancy. d. Age of the patient. e. Tissue of origin. Which of the following is associated with the highest risk of malignancy in a solitary thyroid nodule? a. Age of the patient is above 40 years. b. Prior history of head and neck irradiation. c. The nodule is cold on scintigraphy. d. Family history of thyroid cancer. e. Symptoms of compression on the trachea. The safety margin for excision of a melanoma 0.5 mm in thickness should be: a. 1.0 cm. b. 3.0 cm. c. 3.0 mm. d. 5.0 mm e. 5.0 cm.
48.
49.
50.
All the following are precancerous for tongue cancer except: a. Smoking. b. Leukoplakia. c. Lymphangioma. d. Dental ulcer. e. Alcoholism.
51.
Risk factors for breast cancer include the following except: a. Prior breast cancer.
503
52.
53.
b. Mammary duct ectasia. c. Breast cancer has affected a sister. d. Early menarche and late menopause. e. In situ lobular or duct carcinoma. A 41 year-old woman has a painless dominant breast lump. There are no symptoms or signs of metastases. The following are possible diagnosis except: a. Dominant cyst of a fibrocystic breast. b. Breast cancer. c. Non-cyclic mastalgia. d. Traumatic fat necrosis. e. Sclerosing adenosis. The following statements about mammography are correct except: a. Is indicated for the diagnosis of a dominant breast lump. b. Is indicated for regular screening of high-risk women for breast cancer. c. Is indicated for follow-up contralateral breast after mastectomy for breast cancer. d. Is more informative in young women below the age of 35 years. e. Is highly accurate in the diagnosis of breast cancer.
54.
Haematogenous spread of breast cancer may produce the following except: a. Pathological fractures. b. Brain metastases. c. Pleural effusion. d. Jaundice. e. Hypocalcemia.
55.
Concerning tissue sampling of a breast lump all the following are correct except: a. The simplest is fine needle aspiration cytology. b. Fine needle aspiration cytology requires no anesthesia. c. The least reliable is excision biopsy. d. Frozen section biopsy is sometimes inconclusive. e. Tru-cut biopsy obtains a core of tissue.
56.
The following statements about early breast cancer are all correct except: a. Means a T2N1M0 tumour or less. b. Means that cure is possible. c. Means that microscopic metastases are not present. d. Primary treatment is by surgery. e. Adjuvant chemotherapy is indicated in the presence of axillary node deposits.
57.
The most important prognostic index in breast cancer is: a. Age of the patient. b. Site of the lesion. c. Presence of lymph node metastases. d. Level of CEA. e. Presence of micro-calcificatio Which of the following chemotherapeutic agents may lead to hemorrhagic cystitis?
58.
504
59.
a. 5-flurouracil. b. Cyclophosphamide. c. Adriamycin. d. Cisplastinum. e. Methotrexate. The most common tumour that metastasizes to the adrenal gland is: a. Thyroid cancer. b. Bronchogenic cancer. c. Renal cell carcinoma. d. Melanoma. e. Breast cancer.
60.
All the following statements about follicular thyroid cancer are true except: a. It present at a later age than papillary carcinoma. b. It disseminates via hematogenous route. c. It is less common than papillary carcinoma. d. It is frequently multicentric. e. Bone is a favorite metastatic site.
61.
In basal cell carcinoma only one statement is true: a. Typical presentation of an ulcer that enlarges rapidly. b. Most common site is outer canthus of eye. c. Surgical excision is with 5 cm safety margin. d. Radiotherapy has no place in its management. e. It is a good example of a locally malignant tumor. The following conditions will increase the risk of skin tumors except: a. Xeroderma pigmentosum. b. Albinism. c. Histiocytosis. d. Solar keratosis. e. Chronic cutaneous ulcers. The most reliable parameter of cardiac function affecting the survival in patients with triple vessel coronary artery disease is: a. Ejection fraction. b. Left ventricular end-diastolic pressure. c. Cardiac index. d. Diastolic left ventricular volume. e. Cardiac output. All the following statements concerning aorto-coronary bypass grafting are true, except: a. It is indicated for crescendo (preinfarction) angina. b. It is contraindicated for patients with severe congestive heart failure. c. It is indicated for chronic disabling angina. d. It is associated with a 15 percent operative mortality. e. In left main disease it improves survival. The commonest type of lung cancer world wide is: a. Oat cell carcinoma. b. Adenocarcinoma. c. Squamous cell carcinoma. d. Large cell carcinoma. e. Alveolar carcinoma.
62.
63.
64.
65.
505
66.
Which of the following denotes irrecoverable paralysis of the brachial plexus? a. Complete paralysis of the upper limb. b. Sensory loss of the whole upper limb. c. Horner’s syndrome. d. Dryness of the upper limb. e. Presence of ischaemia in the fingers.
67.
Which of the following is irrelevant as a test for brain death? a. Absent corneal reflex. b. Absent gag reflex. c. Fixed dilated pupils. d. Absent spinal reflexes. e. Absent vestibulo-ocular reflex. Which of the following is not true about Horner’s syndrome? a. Ptosis. b. It may occur in Pancoast’s tumour. c. Enophthalmos. d. Mydriasis. e. Anhidrosis. All the following statements are correct about carpal tunnel syndrome except: a. There is hypothesia along the lateral side of the hand. b. There is weakness of adduction of the thumb. c. There is atrophy of the thenar eminence. d. It may occur in patients with rheumatoid arthritis. e. The symptoms may be exacerbated in pregnancy.
68.
69.
70.
In a transient ischaemic attack, complete recovery of the focal neurologic deficit occurs within: a. One day. b. One week. c. One month. d. Six months. e. One year.
71.
In a patient with head injury, which of the following can be a cause of hypovolaemic shock? a. Extradural haematoma. b. Subdural haematoma. c. Scalp lacerations. d. Subarachnoid haemorrhage. e. Intracerebral haematoma.
72.
The major cause of death in head injury patients is: a. Cerebral coning. b. Cardio-respiratory center arrest. c. Airway obstruction. d. Shock. e. Bleeding.
73.
CSF Rhinorrhea is best managed by:
506
a. b. c. d. e.
Immediate exploration. Nasal packing. Prophylactic antibiotics. Laser coagulation. Lumbar puncture.
74.
In significant arteriosclerotic stenosis of the internal carotid artery the treatment of choice is: a. Antiplatelet agents. b. Bypass grafting. c. Thromboendarterectomy. d. Resection and end to end anastomosis. e. Ligation of internal carotid artery.
75.
Hypophysectomy may occur in cases: a. Uncomplicated juvenile diabetes. b. Early breast cancer c. Ectopic pituitary hormone secretion. d. Pituitary tumors. e. Addison’s disease.
76.
The surest modality of pituitary ablation is by: a. Hypophyseal stalk section. b. Cryousrgical destruction. c. Surgical hypophysectomy. d. Transphenoidal 99 Yttnium implants. e. External proton beam irradiation.
77.
Regarding hyperpituitarism: a. Acromegaly is due to excess growth hormone before puberty. b. Cushing syndrome is always due to excess ACTH. c. Cushing disease is due to excess ACTH. d. Prolactinoma is not a cause of sterility. e. Hyperthyroidism is commonly of pituitary origin.
78.
Addison’s disease may be due to: a. Acute cortical hemorrhage of the adrenals. b. Syphilis. c. Basophil adenoma of pituitary gland. d. Medullary carcinoma of thyroid. e. Rheumatic fever.
79.
Primary hyperparathyroidism: a. Occurs more in males. b. Is most commonly due to multiple parathyroid adenomata. c. Its commonest presentations are due to renal involvement. d. Skeletal manifestations are commoner nowadays. e. Parathyroid masses are usually felt in the neck.
80.
Regarding parathyroid tenay:
507
a. Common complication of subtotal thyroidectomy. b. Symptoms appear immediately after recovering from anesthesia. c. Acrocyanosis occurs. d. Symptoms can be relieved by 10-20% ml of 10% solution of calcium gluconate. e. Symptoms can be treated by calcitonin. 81.
Diagnosis of primary hyperparathyroidism is strongly suggested if there is: a. Rise of serum calcium above 13 mg/l00 ml. b. Simultaneous rise of serum calcium and parathormone. c. Rise of serum chloride-phosphate ratio above 15. d. Rise of serum parathormone with negative squeeze test. e. Rise of serum alkaline phosphatase.
82.
Exploration for parathyroid tumors should start by: a. Retropharyngeal and retroesophageal area. b. Region of recurrent laryngeal nerve and inferior thyroid artery. c. Carotid sheath. d. Superior mediastinum. e. Thyroid lobes.
83.
Secondary hyperparathyroidism is associated with: a. Increased serum calcium. b. Low serum parathormone. c. Increased serum phosphorus in cases of renal osteodystrophy on hemodialysis. d. Increased serum phosphorus in cases of osteomalacia. e. Development of chief cell hyperplasia.
84.
Types of hypercorticism include: a. Waterhouse-Friedrechsen syndrome. b. Simmond’s disease. c. Cushing’s syndrome. d. Addison’s disease. e. Rheinefilter disease.
85.
To differentiate between hypercorticism due to adrenal tumor and pituitary tumor we depend on: a. 1 mg dexamethasone suppression overnight. b. 2 mg dexamethasone suppression. c. Urinary cortisol assay. d. Serum aldosterone assay. e. Plasma ACTH measurement. Paroxysmal hypertension with vasomotor phenomena occurs with: a. Conn’s syndrome. b. Renal artery stenosis. c. Polycystic kidney disease. d. Pheochromocytoma. e. Cushing’s syndrome.
86.
508
87.
Regarding pheochromocytoma: a. It is bilateral in 60% of cases. b. It produces only dopamine. c. Any hypertensive patient under 60 years should be screened for it. d. It is treated by antihypertensive drugs. e. It causes skin pigmentation.
88.
Regarding surgical treatment of pheochromochtoma: a. Preoperative preparation depends only on B-adrenergic blockers. b. The patient should be monitored during operation in some selected cases. c. Severe hypotension may occur following ligation of the suprarenal vein. d. Laparoscopic surgery is contraindicated. e. Postoperative intensive care is seldom needed.
89.
Pancreatitis can be attributed to: a. Gall stones confined to the gall bladder. b. Heavy alcohol consumption. c. Contraceptives with low oestrogen content. d. Billroth-I gastrectomy. e. Hypocalcaemia.
90.
Diagnosis of acute pancreatitis is strongly suggested by: a. Nausea with vomiting. b. Presence of gall stones. c. Late rise of serum amylase. d. Rise of amaylase clearance ratio above 10%. e. Epigastric mass.
91.
Surgical treatment of acute pancreatitis is indicated if: a. There is evidence of small intestinal obstruction. b. Diagnosis of acute pancreatitis is proved. c. There is evidence of biliary obstruction. d. Marked rise of serum lipase. e. Presence of severe hypocalcaemia. Regarding pancreatic pseudocyst: a. The cyst is lined by columnar epithelium. b. It follows 80% of acute pancreatitis. c. Always presents by jaundice. d. There is persistent elevation of serum amylase. e. Only 5% resolve spontaneously.
92.
93.
The pancreatic pseudocyst is best treated by: a. Excision of the cyst. b. Repeated aspiration. c. Cysto-gastrostomy. d. External drainage. e. Antibiotics.
94.
Regarding carcinoma of pancreas:
509
95.
a. The incidence is decreasing. b. It occurs most commonly in third decade. c. In two thirds of cases it affects the head of pancreas. d. It has a high operability rate. e. It is not usually associated with weight loss. The commonest presentation of cancer head of pancreas: a. Back pain. b. Palpable epigastric mass. c. Painless obstructive jaundice. d. Weight loss. e. Migrating thrombophlebitis.
96.
The surgical treatment most commonly performed in cancer head of the pancreas is: a. Distal pancreatectomy. b. Cholecystojejunostomy and gastrojejunostomy. c. Total pancreatectomy. d. Pancreaticoduodenectomy. e. Pancreatectomy and segmental hepatectomy.
97.
Regarding cancer ampulla of Vater: a. Spread is more rapid than cancer head of pancreas. b. Jaundice presents late. c. Jaundice may be fluctuant with melaena. d. It is commonly detected by ultrasonography. e. Results of pancreatoduodenectomy are worse than cancer head of pancreas. Regarding pancreatic islet cell tumors: a. B-cell tumors cause pancreatic cholera. b. lnsulinomas are non B-cell tumors. c. Zollinger Ellison syndrome is caused by gastriinomas of non-B cell origin. d. Most tumors are malignant. e. Most tumors are large and easily detectable.
98.
99.
Epinephrine and norepinephrine production is increased in the following conditions: a. Hyperglycaemia. b. Head injuries. c. Endotoxaemia. d. Dibetes inspidus. e. Addison’s disease.
100. In trauma the endocrinal changes are ment to: a. Decrease energy to brain. b. Decrease energy to muscles. c. Conserve salt and water. d. Increase gonadal functions. e. Increase fat and carbohydrate stores.
510
1. The most common hernia in females is: A. Femoral hernia. B. Direct inguinal hernia. C. Indirect inguinal hernia. D. Obturator hernia. E. Umbilical hernia. Answer: C DISCUSSION: Indirect inguinal hernias are the most common hernia in both females and males. Femoral hernias are more common in females than in males. 2. Which of the following statements regarding unusual hernias is incorrect? A. An obturator hernia may produce nerve compression diagnosed by a positive Howship-Romberg sign. B. Grynfeltt's hernia appears through the superior lumbar triangle, whereas Petit's hernia occurs through the inferior lumbar triangle. C. Sciatic hernias usually present with a painful groin mass below the inguinal ligament. D. Littre's hernia is defined by a Meckel's diverticulum presenting as the sole component of the hernia sac. E. Richter's hernia involves the antimesenteric surface of the intestine within the hernia sac and may present with partial intestinal obstruction. Answer: C DISCUSSION: Sciatic hernias usually present with intestinal obstruction or a mass in the gluteal or infragluteal region. 3. Staples may safely be placed during laparoscopic hernia repair in each of the following structures except: A. Cooper's ligament. B. Tissues superior to the lateral iliopubic tract. C. The transversus abdominis aponeurotic arch. D. Tissues inferior to the lateral iliopubic tract.
511
E. The iliopubic tract at its insertion onto Cooper's ligament. Answer: D DISCUSSION: Placement of staples inferior to (below) the lateral iliopubic tract may result in injury to the lateral femoral cutaneous nerve or the genitofemoral nerve. Staples should also not be placed within the triangle of doom, owing to the risk of major vascular injury. 4. The following Nyhus classification of hernias is correct except for: A. Recurrent direct inguinal hernia—Type IVa. B. Indirect inguinal hernia with a normal internal inguinal ring—Type I. C. Femoral hernia—Type IIIc. D. Direct inguinal hernia—Type IIIa. E. Indirect inguinal hernia with destruction of the transversalis fascia of Hesselbach's triangle— Type II. Answer: E DISCUSSION: An indirect inguinal hernia with destruction of the transversalis fascia of Hesselbach's triangle is classified as a Type IIIb hernia. Also classified as Type IIIb hernias are sliding, pantaloon, and massive scrotal hernias. Type II hernia is an indirect inguinal hernia with a dilated internal ring but without displacement of the inferior deep epigastric vessels or destruction of the transversalis fascia of Hesselbach's triangle. 5. Which of the following statements about the causes of inguinal hernia is correct? A. Excessive hydroxyproline has been demonstrated in the aponeuroses of hernia patients. B. Obliteration of the processus vaginalis is a contributing factor for the development of an indirect inguinal hernia. C. Physical activity and athletics have been shown to have a protective effect toward the development of inguinal hernias. D. Elevated levels of circulating serum elastalytic activity have been demonstrated in patients with direct herniation who smoke. E. The majority of inguinal hernias are acquired. Answer: D DISCUSSION: A correlation between cigarette smoking and an inguinal hernia formation has been demonstrated. Elevated circulating serum elastalytic activity and free active unbound neutrophil elastase has been detected in smokers. 6. The following statements about the repair of inguinal hernias are true except: A. The conjoined tendon is sutured to Cooper's ligament in the Bassini hernia repair.
512
B. The McVay repair is a suitable option for the repair of femoral hernias. C. The Shouldice repair involves a multilayer, imbricated repair of the floor of the inguinal canal. D. The Lichtenstein repair is accomplished by prosthetic mesh repair of the inguinal canal floor in a tension-free manner. E. The laparoscopic transabdominal preperitoneal (TAPP) and totally extraperitoneal approach (TEPA) repairs are based on the preperitoneal repairs of Cheattle, Henry, Nyhus, and Stoppa. Answer: A DISCUSSION: The Bassini repair is accomplished by high ligation of the hernia sac followed by suturing the conjoined tendon and the internal oblique muscle to the inguinal ligament. 7. Which of the following statements concerning the abdominal wall layers are correct? A. Scarpa's fascia affords little strength in wound closure. B. The internal abdominal oblique muscles have fibers that continue into the scrotum as cremasteric muscles. C. The transversalis fascia is the most important layer of the abdominal wall in preventing hernias. D. The lymphatics of the abdominal wall drain into the ipsilateral axillary lymph nodes above the umbilicus and into the ipsilateral superficial inguinal lymph nodes below the umbilicus. Answer: ABCD DISCUSSION: The integrity of the abdominal wall is maintained principally by the transversalis fascia. Scarpia's fascia affords little strength in wound closure, but its approximation contributes considerably to the creation of an aesthetically acceptable scar. The cremasteric muscles of the spermatic cord are a continuation of muscle fibers from the internal abdominal oblique musculature. The lymphatic supply of the abdominal wall follows a simple pattern. These superficial lymphatics run parallel to the superficial veins, which above the umbilicus drain into the ipsilateral axillary vein and below it into the ipsilateral femoral vein. 8. Which of the following congenital abnormalities are correctly defined? A. Omphalocele represents a defect in the abdominal wall lateral to the umbilical cord. B. The herniated viscera associated with omphaloceles are usually covered with a membranous sac. C. An umbilical polyp is a small excrescence of omphalomesenteric duct mucosa that is retained in the umbilicus. D. Meckel's diverticulum results when the intestinal end of the omphalomesenteric duct persists and represents a true diverticulum. Answer: BCD DISCUSSION: Omphalocele may be seen in newborns and represents a defect in the closure of the umbilical ring. The herniated viscera are usually covered with a sac. Gastroschisis, a defect of the
513
abdominal wall lateral to the umbilical cord, is caused by failure of closure of the body wall. The intestines protrude through the defect, and no sac is present to cover the herniated intestine. In the fetus, the omphalomesenteric duct may present as abnormalities related to the abdominal wall when the duct fails to obliterate. Meckel's diverticulum is the result of the failure of obliteration of the intestinal end of the omphalomesenteric duct. This is a true diverticulum with all layers of the intestinal wall represented. An umbilical polyp is a small excrescence of omphalomesenteric duct mucosa retained in the umbilicus. Such polyps resemble umbilical granulomas except that they do not disappear after silver nitrate cauterization. Appropriate treatment is excision of the mucosal remnant. The following statement(s) is/are true concerning the indications for treatment of an inguinal hernia. a. Most adult hernias will remain stable in size, therefore delay seldom affects the technical aspects of a surgical repair b. There is a direct correlation between the length of time that a hernia is present and the risk of major complications c. The morbidity and mortality associated with emergent operation due to hernia complications is significantly greater than for elective repair of the identical hernia d. A truss maintains a hernia in the reduced state, therefore, minimizing the risk of incarceration and strangulation Answer: b, c The indications for hernia repair must be individualized for each patient and the particular situation. In general, the presence of a hernia may be considered an adequate indication for hernia repair. Certainly the presence of complications due to hernia necessitates the correction of those complications and usually the repair of the hernia. As with any treatment, the benefits of operative repair must be weighed against the natural history of the disease, the extent to which the treatment can correct the problem, the possibility of treatment-related injury, and the interference of concomitant disease with the treatment results. With a few exceptions, the natural history of an abdominal wall hernia is that the size of the defect and the sac enlarges over time, and this enlargement increases the difficulty of adequate repair and the chances of recurrence of the hernia. The risk of major complications is greater in an individual patient, the longer the exposure to a hernia and the larger the sac relative to the hernia defect. In addition, major complications necessitate an emergent operation with attended high mortality and morbidity relative to that experienced with an elective repair. The use of a truss, an external support device using a system of straps to exert regional pressure over the hernia defect, should generally be avoided. Trusses do not consistently maintain a hernia in the reduced state, and they may put an unreduced hernia in greater jeopardy of strangulation. The pressure exerted induces edema by decreasing lymphatic and venous flow out of the herniated bowel. Trusses may also lead to injury to the skin overlying the hernia. Which of the following statement(s) is/are true concerning the diagnosis and management of epigastric hernias? e. A large peritoneal sac containing abdominal viscera is common f. At the time of surgical repair, a careful search for other defects should be performed g. Recurrent epigastric hernias after simple closure is uncommon
514
h. Patients with symptoms of a painful midline abdominal mass frequently will contain incarcerated small bowel Answer: b Epigastric hernias are usually small but they vary considerably in size. Most of these defects occur in the midline. The small defects contain only preperitoneal fat with no sac. With increasing size, fat in the falciform ligament and eventually a peritoneal sac and abdominal viscera may be contained within the hernia. The preperitoneal fat in the small defect is usually incarcerated. Multiple defects may be present in up to 20% of patients. Surgical treatment is recommended in all adult patients with symptoms or with a hernia defect greater than 1.5 to 2 cm. in diameter. Methods of repair depend upon the size of the defect. For small defects, simple closure with obliquely placed sutures after reduction or removal of the preperitoneal fat from the defect has been recommended. However recurrent epigastric hernias in up to 10% of the cases have been reported with this method, most likely as a result of additional undetected or unrepaired weaknesses in the epigastric midline. The following statement(s) is/are true concerning neurovascular structures in the inguinal region. i. The inferior epigastric artery and vein run upward in the preperitoneal fat posterior to the transversalis fascia close to the lateral margin of the internal inguinal ring j. The iliohypogastric and ilioinguinal are motor and sensory nerves in the inguinal region which lie beneath the external oblique aponeurosis k. The ilioinguinal nerve runs anterior to the spermatic cord in the inguinal canal and at the superficial inguinal ring, branches into the sensory supply to the pubic region and the upper scrotum or labium majoris l. The genital branch of the genitofemoral nerve is a sensory nerve only to the upper thigh and genital area Answer: b, c Arising anteriorly from the external iliac artery, the inferior epigastric artery with its accompanying vein runs obliquely medially and upward in the preperitoneal fat, posterior to the transversalis fascia and close to the inferior margin of the internal inguinal ring. Inguinal hernias arising superior to the inferior epigastric vessels are indirect inguinal hernias, whereas those arising inferior to the vessels are direct inguinal hernias. The iliohypogastric and ilioinguinal nerves are motor and sensory nerves to the muscles and skin of the inguinal region. The nerves penetrate the transversus abdominis muscle at the point above the middle of the iliac crest, lie below the internal oblique muscle up to the point just medial and superior to the anterior superior iliac spine, and then penetrate the internal oblique muscle and lie below the external oblique aponeurosis. The ilioinguinal nerve runs anterior to the spermatic cord in the inguinal canal and at the superficial inguinal ligament, branches into sensory supply to the pubic region and the upper scrotum or labium majoris. The genital branch of the genitofemoral nerve perforates the transversalis fascia usually just inferior to the internal ring. It courses along the posterior surface of the spermatic cord and supplies motor fibers to the cremaster muscle. At the superficial inguinal ring, it divides to provide sensory innervation to the scrotum and medial aspect of the upper thigh.
515
In advising a patient preoperatively of potential complications of operative treatment of an inguinal hernia, which of the following statement(s) is/are true? m. Severe symptoms due to sensory nerve entrapment or injury can occur n. The most common vascular structure injured during the course of a groin hernia repair is the femoral artery o. Recurrent hernia after primary groin repair should occur in less than 10% of cases p. Wound infection increases the risk of recurrent hernia Answer: a, c, d Many complications can occur with operations to repair an inguinal hernia. Sensory nerve injury may lead to disabling symptoms from neuromas or nerve entrapment during inguinal hernia repair. Although vascular injuries are uncommon in inguinal repair, the proximity of the femoral vein to the structures used in the hernia repair makes injury of this vessel the most frequent vascular injury observed. Hernia recurrence after primary groin hernia repairs should be infrequent and varies in several large series from less than one percent to almost nine percent. The prevalence of recurrent hernia may be higher after repair of recurrent groin hernia. Factors responsible for hernia recurrence include closure under excessive tension, failure to identify and use an adequately strong musculoaponeurotic tissue, and wound infection. Chylous ascites is the accumulation of chyle within the peritoneal cavity. Which of the following statement(s) is/are true concerning chylous ascites? q. The cisterna chyli lies at the anterior surface of the first and second lumbar vertebrae and receives lymphatic fluid from the mesenteric lymphatics r. Chylous ascites is most commonly associated with abdominal lymphoma s. Paracentesis and analysis of chylous fluid typically reveals elevated triglycerides, protein, and leukocyte levels with cytologic analysis reflecting the underlying presence of malignancy t. Treatment of chylous ascites with dietary manipulation will be successful in most cases u. The mortality rate in adults with chylous ascites is in excess of 50% Answer: a, b, e Chylous ascites is accumulation within the peritoneal cavity of chyle, a lymphatic fluid with a high lipid content. Access of intestinal lipids to the circulation is via mesenteric lymphatics that enter the cisterna chyle, which in turn becomes the thoracic duct which eventually enters the venous system at the junction of the left subclavian and internal jugular veins. The cisterna chyli lies at the anterior surface of the first and second lumbar vertebrae slightly to the right of the aorta. Chylous ascites may result from injury to major lymphatic duct or the cisterna. However for lymphatic leakage to persist, widespread occlusion of lymphaticovenous collaterals within the abdomen must be present. Malignancy is the predominant cause (88%) of spontaneous chylous ascites in adults, with
516
lymphoma the most common malignancy. Diagnostic studies must include not only documentation of lymphatic origin of the abdominal fluid but also an attempt to delineate the cause of chylous ascites. Paracentesis and analysis of chylous fluid typically reveals elevated triglycerides, protein, and leukocyte levels, with a predominance of lymphocytes. Unfortunately, cytology is seldom positive despite the presence of malignancy. Lymphangiography may define the site of lymphatic leak for patients in whom the leak is from the cisterna or retroperitoneal lymphatics but not when from the mesenteric or hepatic lymphatics. Of noninvasive studies, CT is the test of choice, with a high diagnostic yield in nontraumatic chylous ascites in adults. Frequently, laparotomy with node biopsy is required for histology and typing in cases suspected to be cancer, particularly for lymphoma. Treatments for chylous ascites have been directed toward decreasing lymph and triglyceride accumulation. Successful resolution of chylous ascites has been achieved using a fatrestricted diet with added medium-chain triglycerides in an attempt to reduce lymphatic transport of triglycerides and perhaps intestinal lymph flow. Although there have been reports of success using such dietary manipulation, many failures have been reported. Therefore, in most patients with chylous ascites, treatment is likely to be successful only when directed toward the underlying cause. For patients with lymphoma, therapy effective against lymphoma is likely to eliminate chylous ascites. The prognosis for patients with chylous ascites is much better in infants and children than in adults, principally because of the differences in causes of the condition. A mortality of 21% is reported in infants and children whereas a mortality of 88% has been noted in adults. Patients with chylous ascites with associated neoplasms typically have the gravest prognosis. Which of the statement(s) is/are true concerning laparoscopic hernia repair? v. General anesthesia is required w. Either an abdominal or preperitoneal approach is possible x. The use of prosthetic mesh is required in all variations y. Long-term results suggest that the laparoscopic approach is equal or better than traditional repairs Answer: a, b, c The laparoscopic approach to the repair of groin hernias has been recently developed. Either a transabdominal approach, wherein the peritoneum in the inguinal area is opened, and the repair is performed in the preperitoneum or an entirely preperitoneal approach can be used. In either technique, which are both performed under general anesthesia, after reducing the visceral contents out of the hernia, the repair is performed by placing a sheet of prosthetic mesh over the internal aspect of the inguinal floor and internal ring. Although early results and short-term benefits appear promising, long-term follow-up data is still not available to compare these techniques with traditional repairs. A 28-year-old woman with a history of an appendectomy presents with a nontender palpable mass in the right lower quadrant abdominal incision. The following statement(s) is/are true concerning the diagnosis and management of this patient. z. The best diagnostic test involves imaging of the abdominal wall by either CT or MRI aa. Resection of the mass with a 2 cm margin is usually adequate bb. Low dose radiation is a suitable alternative to surgery for primary treatment
517
cc. Re-resection for recurrence will likely have a higher rate of recurrence than for primary resection Answer: a Desmoid tumors are fibromatous tumors that may resemble low-grade fibrosarcoma but never metastasize. The tumor often infiltrates adjacent muscle and has a high incidence of recurrence despite seemingly adequate gross resection. The highest frequency is in women of childbearing age of which over 90% of tumors are abdominal in location. For abdominal wall desmoid tumors, approximately one-third are associated with a previous operation at the tumor site. The most frequent presenting symptom is a nontender, palpable abdominal wall mass. Diagnostic imaging is best carried out by CT or MRI, which delineate the extent of involvement of the layers of the abdominal wall and potential intraperitoneal extension. Initial treatment of abdominal wall desmoid tumors is surgical. Because the margins of the tumor are not easily determined and because the tumor often infiltrates muscle and periosteum, limited margins around the gross tumor frequently result in microscopic tumor at the margin. Recurrence rates for abdominal desmoid tumors vary from 9% to 40%, and recurrence is frequent with inadequate margins. A 5-cm margin of resection is considered adequate with mono bloc resection of rib cage, pubic or iliac bone or involved portions of organs such as bladder to achieve these margins. Reconstruction of the abdominal wall with polypropylene mesh is necessary in most cases. In patients in whom adequate margins of resection are achieved, there is no benefit from adjuvant radiotherapy. Second and third resections after recurrence have been associated with no higher rate of recurrence than primary resection. Radiotherapy alone has achieved local control in desmoid tumor in as many as 100% of tumors treated primarily and 75% of recurrent tumors. Radiation doses at least 60 Gy are considered necessary for consistent control. The large radiation dose risks major damage to adjacent bowel and therefore primary radiation treatment of abdominal wall desmoid tumors has a limited role. Which of the following statement(s) is/are true concerning repair of inguinal hernias? dd. The Bassini repair approximates the transversus abdominis aponeurosis and transversalis fascia and the shelving edge of the inguinal ligament. ee. The Bassini repair is an adequate repair for a femoral hernia ff. A relaxing incision is important for repairs of direct and large indirect inguinal hernias to prevent excessive tension in the closure gg. An advantage to the use of prosthetic material is the mesh incites formation of scar tissue to further increase tensile strength provided by the mesh alone Answer: a, c, d The Bassini repair is an inguinal hernia repair used world-wide and has been the standard against which other repairs are judged. The repair involves approximation of the transversus abdominis aponeurosis and transversalis fascia and the lateral edge of the rectus sheath to the shelving edge of the inguinal ligament. A femoral hernia cannot be repaired by the Bassini repair because the orifice to the femoral canal lies deep to the inguinal ligament. A Cooper’s ligament repair does approximate the structures to the transversalis fascia of the pectineal (Cooper’s) ligament between the pubic tubercle and the femoral vein and therefore is appropriate for repair of a femoral hernia. A relaxing incision for repairs of direct and large indirect inguinal hernias prevents excessive tension in the closure. There are an increasing number of proponents for the use of prosthetic material for the routine repair of inguinal hernias. Prosthetic material, such as polypropylene mesh, have been used for years for repair of large or recurrent inguinal and femoral hernias. The prosthetic mesh provides a low-tension repair for such large defects which otherwise could not be closed without
518
excessive tension. In addition, the mesh incites the formation of scar tissue to further increase tensile strength beyond that provided by mesh alone. Results reported for inguinal hernia repairs using mesh have been excellent, although there is a slight risk of infection of the prosthetic material which must be considered. The following statement(s) is/are true concerning the epidemiology of inguinal hernias. hh. Inguinal hernias occur with a male-to-female ratio of about 7:1 ii. Femoral and umbilical hernias are more common in women, with a female-to-male ratio of 4:1 jj. The frequency of inguinal hernias increases with age kk. Almost all umbilical hernias occur in the pediatric age group Answer: a, c Inguinal hernias are the most frequently occurring hernia by a factor of five over other individual types. Umbilical hernias constitute about 14% of hernias, femoral hernias about 5%, and other types are rare. There is a male prevalence in inguinal hernias of about 7:1 (male-to-female), whereas there is a female dominance in femoral and umbilical hernias of 8:1 and 7:1 (female-to-male), respectively. For inguinal hernia, which occurs at all age levels, frequency increases with age. Umbilical hernias have a bimodal distribution, peaking in the pediatric population and then in the 40 to 60 year group, in which the hernias are principally paraumbilical. A 77-year-old multiparous female presents with a bowel obstruction. She has no previous abdominal operations and no abdominal wall hernias can be detected. In addition to her abdominal symptoms, she reports pain in her right medial thigh. The following statement(s) is/are true concerning her diagnosis and management. ll. Expectant management with nasogastric suction and IV fluid replacement is indicated mm. A right groin approach is indicated for exploration and repair of the presumed hernia nn. The use of a polypropylene mesh will likely be necessary for repair oo. A correct diagnosis can usually be made by visualizing an external mass in the upper, medial thigh Answer: c An obturator hernia is a hernia that occurs through the obturator canal, accompanied by the obturator vessels and the obturator nerve. Although rare, most obturator hernias occur in older multiparous women and are predominantly right-sided. Symptoms are frequently intermittent but tend to be acute and become increasingly severe with incarceration of the hernia. Intestinal symptoms predominate, but dysesthesia or pain in the medial thigh with occasional radiation to the hip is often present. Dysesthesia results from compression of either division of the obturator nerve. Although the hernia is never externally visible, in a small percentage of patients a mass can be palpated in the upper, medial thigh. A correct diagnosis of obturator hernia is made in only about one-third of patients presenting with intestinal obstruction. Plain radiographs are seldom helpful, however a CT scan will usually confirm the diagnosis. Treatment is operative. There is no place for expectant therapy, especially in a patient with pain an parasthesias along the inner aspect of the thigh or with clinical or radiographic evidence of bowel obstruction. Many surgical approaches
519
have been promoted, but the transabdominal approach should be used because it has several advantages. It best confirms the diagnosis and exposes the obturator canal, orifice, vessels, and nerve, also permitting bowel resection when required. The sac is dealt with in a standard fashion. The hernia defect should be repaired, but repair usually requires a polypropylene mesh patch because the margin of the defect cannot be approximated primarily. The following statement(s) is/are true concerning umbilical hernias in adults. pp. Most umbilical hernias in adults are the result of a congenital defect carried into adulthood qq. A paraumbilical hernia typically occurs in multiparous females rr. The presence of ascites is a contraindication to elective umbilical hernia repair. ss. Incarceration is uncommon with umbilical hernias Answer: b An umbilical hernia in a child is usually considered to be congenital. Only about 10% of umbilical hernias in adults are thought to be the result of a congenital defect carried into adulthood. Most adult umbilical hernias are acquired and are called paraumbilical hernias. The paraumbilical hernia typically occurs in a multiparous female. Other patients with increased intraabdominal pressure, particularly with concomitant chronic abdominal distension as from ascites, are also at increased risk for the development of paraumbilical hernias. Umbilical and paraumbilical hernias vary from small to extremely large. Incarceration is frequent in the large hernias, which typically have a small neck. Indications for umbilical hernia repair in adults include symptoms, incarceration, large hernia relative to the neck, and trophic changes in the overlying skin. Among adults with associated ascites, repair is advocated to avoid potentially serious complications. The presence of discoloration or ulceration of overlying skin or a rapid increase in size of the hernia herald impending rupture. Spontaneous rupture of the hernia in these patients can be catastrophic and is frequently associated with mortality rates approaching 30%. By comparison, elective umbilical hernia repair can be performed safely in patients with ascites with acceptable morbidity and mortality. Retroperitoneal fibrosis is a fibrosing condition of retroperitoneum, which is of significance as it generally encompasses the ureters and eventually causes hydronephrosis and kidney damage. Which of the following statement(s) is/are true concerning this condition? tt. The majority of cases are idiopathic in nature uu. A history of use of methysergide for treatment of migraine headaches would be significant vv. There is no known association of malignancy with retroperitoneal fibrosis ww. The disease occurs more commonly in women than in men Answer: a, b Retroperitoneal fibrosis is a rare condition in which fibrosis develops in the retroperitoneal space. The ureters frequently will become encompassed by the process eventually causing hydronephrosis and kidney damage. Retroperitoneal fibrosis occurs most commonly in the fifth and sixth decades with a 2:1 male-female predominance. The pathophysiology of retroperitoneal fibrosis remains to
520
be delineated. In fully two-thirds of cases, retroperitoneal fibrosis is idiopathic, however, an autoimmune process has been suggested as a potential cause. About 12% of cases of retroperitoneal fibrosis have been associated with the use of methysergide, a serotonin agonist used for vascular and migraine headache, and in this subgroup females outnumber males 2:1. Primary or metastatic malignancy in the retroperitoneum is found in 8% of patients with retroperitoneal fibrosis. Sarcomas are the most common primary tumors, but non-Hodgkin and Hodgkin lymphomas and ureteral cancer have also been found. Metastases have originated from cancer of the stomach, breast, colon, carcinoid, pancreas, prostate, ovary, and cervix. The focus of tumor may be small but may induce desmoplasia that is grossly indistinguishable from benign variance of retroperitoneal fibrosis. The following statement(s) is/are true concerning the anterior abdominal wall musculature. xx. The lateral musculature of the abdominal wall consists of three muscle layers. These are, from external to internal, the external oblique, the transversus abdominis, and the internal oblique muscles yy. The transversalis fascia lies on the deep side of the transversus muscle and extends to form an essentially complete fascial envelope of the abdominal cavity zz. Above the semicircular line, the internal oblique aponeurosis splits into posterior and anterior laminae aaa. The rectus abdominis muscles originate on the ribs superiorly and on the pubis inferiorly and are clearly distinct throughout their entire length Answer: b, c The anterior abdominal wall consists of a group of lateral sheet-like muscles and paired, longitudinally-oriented flat muscles on either side of the midline. The lateral musculature of the abdominal wall consists of three layers, each of which has its fascicles running in an oblique angle to the others. The most superficial of these lateral muscles is the external oblique muscle. The internal oblique muscle lies deep to the external oblique muscle while the transversus abdominis muscle is the innermost of the lateral abdominal wall musculature. The transversalis fascia lies on the deep side of the transversus muscle and extends to form an essentially complete fascial envelope of the abdominal cavity. The semicircular line is defined by the lower edge of the posterior sheath about 3 to 6 cm below the level of the umbilicus, and its convexity is directed superiorly. Above the semicircular line, the internal oblique aponeurosis splits into posterior and anterior laminae. The posterior lamina joins with the transversus abdominis aponeurosis to form the posterior rectus sheath. The anterior lamina fuses with the external oblique aponeurosis to form the anterior rectus sheath. Below the semicircular line, the internal oblique end transversus abdominis aponeurosis fuse to form an internal lamina of the anterior sheath, with the external oblique aponeurosis forming the external lamina of the anterior sheath. The medial paired rectus abdominis muscles originate on the ribs superiorly and on the pubis inferiorly. Below the semicircular line, the rectus muscles are nearly fused in the midline and indistinct, and their posterior surfaces covered only with the transversalis fascia. A 48-year-old woman maintained on Warfarin for a history of cardiac valvular replacement and a history of recent upper respiratory infection presents with severe abdominal pain exacerbated by movement. Her physical examination shows tenderness in the right paramedian area with voluntary guarding but no peritoneal signs. The following statement(s) is/are true concerning the diagnosis and management of this patient. bbb. Urgent laparotomy should be performed because of concern for arterial mesenteric embolus
521
ccc. The correct diagnosis could likely be made by CT scan and operation avoided ddd. The status of her anticoagulation should be checked and if her prothrombin time is excessively prolonged, correction is necessary eee. If untreated, hemodynamic instability is common Answer: b, c Rectus sheath hematoma results from arterial or venous bleeding into the rectus sheath, most commonly from arterial bleeding. Rectus sheath hematomas predominate in women by a ratio of about 3:1. The mean age of incidence is in the late fifth decade. Although spontaneous formation of a rectus hematoma is rare, it can occur with vasculitis, arterial venous malformations, a severe coagulopathy, or with the administration of anticoagulants. The usual cause is trauma. Events as trivial as sneezing, coughing, or twisting to the side have initiated a rectus hematoma. Abdominal pain is almost always described at presentation. Pain is often described as severe and usually is exacerbated by movements that require muscular contraction of the abdominal wall. On examination, there is tenderness over the rectus sheath, voluntary guarding, and often a diffuse mass sensation in the area of tenderness. Contraction of the rectus muscle exacerbates the pain and tenderness. Peritoneal signs are absent. Ecchymosis may occur but usually appears several days after the onset of pain. In cases where the hematoma dissects or originates inferiorly and expands into the prevessicle and preperitoneal space, the hematocrit may fall significantly; however, hemodynamic instability is distinctly unusual. When the intraabdominal source of pain is unknown, ultrasound and particularly computed tomography can delineate the hematoma and localize it to the abdominal wall in almost all cases. Treatment must take into consideration the cause, if known, and whether the hematoma is stable or progressive. Coagulopathy should be corrected when possible. For patients in whom the hematoma is stable, pain medication and avoidance of muscular stress on the abdominal wall are sufficient. For patients with progressive hematoma, the treatment of choice is evacuation of the hematoma from within the rectus sheath and hemostasis, sometimes requiring ligation of the epigastric vessels above and below the hematoma True statements concerning the diagnosis and management of retroperitoneal fibrosis include: fff. Most patients present with dull, non-colicky back, flank, or abdominal pain ggg. Evidence of impaired renal function with an elevated blood urea nitrogen is common hhh. The diagnosis is most commonly suggested by intravenous pyelography although contrast studies with CT scan or MRI are useful in further defining the disease iii. Most patients can be managed nonoperatively jjj. The prognosis for nonmalignant retroperitoneal fibrosis is grim with progression of disease until death occurring in most patients Answer: a, b, c Ninety percent of patients with retroperitoneal fibrosis present with dull, non-colicky pain in the back, flank, or abdomen. Other symptoms include weight loss, non-specific gastrointestinal complaints, and uncommonly, lower extremity edema, malaise, and dysuria. Laboratory studies may be normal in 25% of patients, but 55% of patients will have an elevated blood urea nitrogen.
522
Diagnosis is most commonly suggested by intravenous pyelography. The combination of medial deviation of the ureter, hydroureteronephrosis, and extrinsic ureteral compression are highly suggestive of retroperitoneal fibrosis. CT scanning or MRI can both define the level of ureteral involvement and depict the mass appearance of the fibrotic process. Exploratory laparotomy with multiple deep biopsies of the retroperitoneal process is an essential part of diagnosis, since foci of carcinoma may be sparse within the predominately sclerotic reaction. Treatment for retroperitoneal fibrosis must identify and deal with potential causative agents, relieve the ureteral obstruction, and reverse the inflammatory-fibrotic process. Renal obstruction may need to be relieved acutely, either by retrograde ureteral stents or by percutaneous nephrostomy tubes. Long-term resolution of ureteral obstruction most frequently has been accomplished by operative freeing of the ureters from the fibrosis and displacing them laterally or within the peritoneal cavity. Although renal function is improved in more than 90% of cases so treated, in as many as one-third of patients, ureteral obstruction recurs on the ipsilateral or contralateral side. Prognosis for patients with nonmalignant retroperitoneal fibrosis is good. Survivals of 86–100% for several years have been reported. The following statement(s) is/are true concerning incarceration of an inguinal hernia. kkk. All incarcerated hernias are surgical emergencies and require prompt surgical intervention lll. Attempt at reduction of an incarcerated symptomatic hernia is generally considered safe mmm. Vigorous attempts at reduction of an incarcerated hernia may result in reduction en masse with continued entrapment and possible progression to obstruction or strangulation nnn. Incarcerated hernias frequently cause both small and large bowel obstruction Answer: b, c Hernia incarceration denotes the condition wherein viscera are contained within a hernia sac and cannot be disgorged from the sac. Patients with an incarcerated hernia may be asymptomatic except for the presence of a bulge. Pain associated with an incarcerated hernia should be interpreted as indicative of strangulation. Many hernias are of such size that they cannot be reduced either spontaneously or manually. If the patient is asymptomatic, elective surgery should be planned. In a patient with pain, attempt at reduction is relatively safe as long as excessive force is not applied. An incarcerated hernia with discomfort or signs of bowel obstruction is best treated with urgent hernia repair, although gentle attempts at reduction may be without consequences. Reduction of a symptomatic hernia may result in reduction of gangrenous bowel into the peritoneal cavity. Reduction of bowel with necrotic areas eventuates in bowel perforation and peritonitis with an associated 10% to 30% mortality and high levels of morbidity. Vigorous attempts at reduction may result in reduction en masse, in which the viscera remain within the peritoneal sac after reduction with the entire sac and its contained viscera forced through the abdominal wall defect into the preperitoneal layer. Reduction en masse usually occurs when a small fibrous neck traps enclosed viscera and is associated with a high risk of continued entrapment and progression to obstruction or strangulation. World-wide hernias are the leading cause of intestinal obstruction. The obstruction is almost exclusively small intestinal with only rarely the colon as the site of obstruction. A careful history is necessary in all patients being considered for inguinal hernia repair. Symptoms which deserve investigation and appropriate treatment prior to proceeding with inguinal hernia repair include:
523
ooo. Chronic cough ppp. Urinary hesitancy and straining qqq. Change in bowel habit rrr. A specific episode of muscular straining with associated discomfort Answer: a, b, c The history and physical examination are almost exclusively the diagnostic modalities used for diagnosis and delineation of hernias. Chronic trauma in the form of overstretching of musculoaponeurotic structures is likely to be the significant factor in spontaneously occurring hernias. Failure to recognize underlying pathology contributing to symptoms of abdominal straining may both increase the risk of recurrent hernia as well as miss significant existing pathology. A chronic cough from chronic obstructive pulmonary disease should be investigated and attempts made to control symptoms. Significant obstructive uropathy may warrant urologic consultation and treatment prior to hernia repair. Such treatment is important both to prevent postoperative urinary retention, as well as persistent straining on the newly-completed repair. Change in bowel habits with constipation or the presence of blood associated with bowel movements may suggest a rectal or left-sided colon cancer. Patients frequently relate a specific episode of muscular straining during which a sudden discomfort occurs followed by hernia symptoms of discomfort or a bulge. There is little evidence to suggest that such a specific acute event can precipitate a hernia. A history of heavy lifting is important, however, in both planning of postoperative disability as well as consideration for long-term recurrence rates. The following statement(s) is/are true concerning abdominal incisional hernias. sss. Large incisional hernias are associated with a high recurrence rate when closed primarily ttt. A large potential space remains anterior to the abdominal wall closure in most patients indicating a need for postoperative wound drainage uuu. The use of prosthetic mesh can often be avoided by employing relaxing incisions in the anterior fascia parallel to the midline vvv. Incisional hernias are frequently associated with a tissue deficit either due to chronic retraction and scarring or the result of tissue necrosis from either infection or tension at the initial closure Answer: a, b, c, d Repair of an incisional hernia can be difficult with several factors making these hernias particularly challenging. First, incisional hernias are often related to a postoperative wound infection, in which case associated fascitis or muscle necrosis may result in loss of tissue. Second, a previous abdominal wall closure under tension or with a technique that resulted in tension on particular sutures may lead to a multifenestrated region of the musculoaponeurotic abdominal wall near or slightly back from its margin. Third, chronic retraction of the abdominal wall muscles result in a larger defect. Fourth, a large potential space remains anterior to the abdominal wall closure in the subcutaneous area; postoperative fluid accumulation in this space contributes to the wound infection rate of 5%. Any such potential space should have operatively placed drains. The key to successful repair involves sufficient dissection and exposure of the true musculoaponeurotic edge and exclusion of adjacent musculoaponeurotic defects and avoidance of
524
closing the wound under tension. Large defects greater than 3 to 4 cm in diameter are seldom able to be closed without excessive tension. The use of relaxing incisions decreases tension and may be particularly useful in midline hernias and therefore may avoid the need for prosthetic mesh. Which of the following structures are derived from the external oblique muscle and its aponeurosis? www. The inguinal or Poupart’s ligament xxx. The lacunar ligament yyy. The superficial inguinal ring zzz. The conjoined tendon Answer: a, b, c The external oblique muscle and its aponeurosis, with its inferiorly and medially-directed fascicles and the overlying innominate fascia lie deep to the subcutaneous tissue. The inguinal ligament (Poupart’s ligament) is the inferior edge of the external oblique aponeurosis and extends from the anterior superior iliac spine to the pubic tubercle, turning under itself posteriorly and then superiorly to form a shelving edge. Medially, the inguinal ligament turns under even further to form the lacunar ligament, as part of its insertion on the pubis. The superficial inguinal ring is a triangular opening in the external aponeurosis, with its apex superiorly in position slightly above and lateral to the pubic tubercle, through which the cord exits the inguinal canal. The conjoined tendon is commonly alluded to in descriptions of inguinal hernia repairs. The conjoined tendon is the fusion of the aponeurosis of the internal oblique and transversus abdominis muscles. A number of special circumstances exist in the repair of inguinal hernias. The following statement(s) is/are correct. aaaa. Simultaneous repair of bilateral direct inguinal hernias can be performed with no significant increased risk of recurrence bbbb. The preperitoneal approach may be appropriate for repair of a multiple recurrent hernia cccc. A femoral hernia repair can best be accomplished using a Bassini or Shouldice repair dddd. Management of an incarcerated inguinal hernia with obstruction is best approached via laparotomy incision Answer: b The approach to bilateral groin hernias is based on the extent of the hernia defect. For hernias for which inguinal floor reconstruction is required (all direct and moderate to large indirect inguinal hernias, all femoral hernias), simultaneous repair of bilateral hernia results in recurrence of one or both of the hernias twice as frequently as if the hernias were repaired sequentially. Repair of recurrent inguinal or much less commonly femoral hernias can be repaired via an anterior approach particularly at the time of first recurrence in most cases. If a deficit of aponeurotic tissue exists, methods such as polypropylene mesh as an overlay or preferably as an underlay, and tailored around the spermatic cord have proved highly successful. The preperitoneal approach also has potential benefits especially in cases of multiple recurrence where the technique allows avoidance of the inevitable scar encountered with the anterior approach, excellent assessment of the defect, and the ease for placement of synthetic mesh. The Bassini and Shouldice repairs involve
525
approximation of the medial tissues of the transversus abdominis aponeurosis and transversalis fascia to the inguinal ligament. These techniques cannot be used to repair a femoral hernia because the femoral canal lies deep to the inguinal ligament. Either the anterior approach of McVay (Cooper’s ligament repair) or a preperitoneal approach is preferred for femoral hernias. In patients with bowel obstruction attributed to a hernia, the primary operative approach is on the hernia. Assessment of bowel viability is possible without laparotomy in most cases, and release of adhesions holding the bowel within the sac is more easily accomplished through direct entry into the hernia sac. Reduction of the herniated and incarcerated bowel may be difficult from the intraabdominal approach necessitating a counter incision over the external presentation of the hernia. 29. Which of the following statements concerning intraperitoneal fluid collections are correct? A. Ascites occurs when either the peritoneal fluid secretion rate increases or the absorption rate decreases. B. Accumulation of lymph within the peritoneal cavity usually results from trauma as tumor involving the intra-abdominal lymphatic structures. C. Choleperitoneum (intraperitoneal bile) generally occurs following biliary surgery, but spontaneous perforation of the bile duct has been reported. D. The most common cause of hemoperitoneum is trauma to the liver or spleen. Answer: ABCD DISCUSSION: Normally, there is a balance between fluid secretion and absorption in the peritoneal cavity. Ascites occurs when either the secretion rate increases or the absorption rate decreases disproportionately. Accumulation of lymph in the peritoneal cavity usually results from trauma or tumor involving lymphatic structures. Proposed treatment regimens range from salt restriction and diuretics to surgical ligation and peritoneovenous shunting. Uninfected bile is a mild irritant to the peritoneal cavity and causes increased production of peritoneal fluid, resulting in bile ascities or choleperitoneum. Most cases of choleperitoneum follow biliary tract surgery, but cases of spontaneous bile duct perforation have been reported in infants and some adults. The most common cause of hemoperitoneum is trauma to the liver or spleen. Less common causes include ruptured ectopic pregnancy, ruptured aortic aneurysms, and other intra-abdominal injuries. 30. The following statement about peritonitis are all true except: A. Peritonitis is defined as inflammation of the peritoneum. B. Most surgical peritonitis is secondary to bacterial contamination. C. Primary peritonitis has no documented source of contamination and is more common in adults than in children and in men than in women. D. Tuberculous peritonitis can present with or without ascites. Answer: C DISCUSSION: Peritonitis is inflammation of the peritoneum and can be septic or aseptic, bacterial or viral, primary or secondary, acute or chronic. Most surgical peritonitis is secondary to bacterial contamination from the gastrointestinal tract. Primary peritonitis refers to inflammation of the
526
peritoneal cavity without a documented source of contamination. It is more common in children than in adults and in women than in men. The female predominance is felt to be explained by entry of organism into the peritoneal cavity through the fallopian tubes. The clinical manifestations of tuberculous peritonitis are of two types. The moist form consists of fever, ascites, abdominal pain, and weakness. The dry form presents in a similar manner but without ascites. 31. True or false? A. Mesenteric cysts are most often due to congenital lymphatic spaces that gradually fill with lymph. B. Mesenteric cysts usually present as abdominal masses accompanied by pain, nausea, or vomiting. C. Mesenteric cysts are best treated by marsupialization. D. Omental cysts are frequently asymptomatic unless they undergo torsion. Answer: A-TRUE, B-TRUE, C-FALSE, D-TRUE DISCUSSION: Mesenteric cysts are most often due to congenital lymphatic spaces that gradually enlarge as they fill with lymph. They generally present as abdominal masses accompanied by pain, nausea, and vomiting. They usually can be diagnosed by physical examination and have characteristic lateral mobility. They are best treated by surgical excision, and intestinal resection may be necessary for complete removal. Omental cysts are frequently asymptomatic but may present with vague discomfort or as a mobile abdominal mass that can cause torsion of the omentum. Torsion generally presents with signs and symptoms compatible with acute cholecystitis, appendicitis, or a twisted ovarian cyst. Treatment entails local resection. 32. Which of the following statements about acute salpingitis are true? A. The disease rarely occurs after menopause. B. Gonococcal infection is most common. C. There is minimal cervical tenderness to palpation. D. Vaginal discharge occurs rarely. Answer: AB 33. Acute appendicitis is most commonly associated with which of the following signs? A. Temperature above 104 ؛F. B. Frequent loose stools. C. Anorexia, abdominal pain, and right lower quadrant tenderness. D. White blood cell count greater than 20,000 per cu. mm. Answer: C
527
34. Which of the following most often initiates the development of acute appendicitis? A. A viral infection. B. Acute gastroenteritis. C. Obstruction of the appendiceal lumen. D. A primary clostridial infection. Answer: C DISCUSSION: The majority of patients with acute appendicitis have an obstructed lumen that is due to either hyperplasia of the lymph follicles in the wall of the appendix or a fecalith. The obstruction creates a site where the bacteria in the lumen multiply rapidly, producing exotoxins and endotoxins that then ulcerate the mucosa, allowing pathogenic organisms to enter the wall of the appendix. An inflammatory process follows that can extend to the serosa, and penetration through the serosal layer causes generalized peritonitis. 35. The diagnosis of acute appendicitis is most difficult to establish in: A. Persons aged 60 and older. B. Women aged 18 to 35. C. Infants younger than 1 year. D. Pregnant women. Answer: C DISCUSSION: It is very difficult to establish a firm diagnosis of acute appendicitis in an infant of 1 year or younger since the patient cannot provide a history or be helpful during the physical examination. It is rare to make a definitive diagnosis preoperatively in such infants, and in such cases the appendix is usually perforated at the time of operation. While appendicitis is somewhat more difficult to diagnose in the elderly because of the reduced response to inflammation; nevertheless, it is usually possible to make the diagnosis. With pregnant women it is wise to remember that the enlarging uterus in the last trimester dislocates the appendix higher in the abdomen and that the signs and symptoms follow this anatomic shift accordingly. 36. Once a diagnosis of acute appendicitis has been made and appendectomy decided upon, which of the following is/are true? A. Prophylactic antibiotics should be administered. B. Prophylactic antibitics are not necessary unless there is evidence of perforation. C. If the appendix is not ruptured and not gangrenous, antibiotics may be discontinued after 24 hours. D. Multiple antibiotics are in all cases preferable to a single agent. Answer: AC
528
DISCUSSION: It is generally held that patients with a diagnosis of acute appendicitis should receive antibiotics such as cefoxitin or cefotetan. Administration can be discontinued after 24 hours if the appendix is not gangrenous or ruptured. Multiple antibiotics are unnecessary in straightforward cases. 37. The best type of x-ray to locate free abdominal air is: A. A posteroanterior view of the chest. B. A flat and upright view of the abdomen. C. Computed tomograph (CT) of the abdomen. D. A lateral decubitus x-ray, right side up. Answer: D 38. The most helpful diagnostic radiographic procedure in small bowel obstruction is: A. CT of the abdomen. B. Contrast study of the intestine. C. Supine and erect x-rays of the abdomen. D. Ultrasonography of the abdomen. Answer: C 39. The most commonly used imaging method for diagnosis of acute cholecystitis is: A. CT of the abdomen. B. Ultrasonography of the gallbladder. C. Oral cholecystogram. D. Radionuclide (HIDA) scan of the gallbladder. Answer: B 40. Acute salpingitis occurs most often: A. After menopause. B. In patients with unilateral lower abdominal pain. C. During the menstrual cycle. D. In patients with cervical tenderness and vaginal discharge. Answer: D
529
41. Meckel's diverticulitis most often occurs in the: A. Proximal jejunum. B. Distal jejunum. C. Proximal ileum. D. Distal ileum. Answer:D A patient is seen in the emergency room with reproducible right lower quadrant tenderness. The approximate incidence of finding a normal appendix on right lower quadrant exploration in similar nonselected patients is which of the following: a. 5% b. 10% c. 20% d. 40% Answer: c Appendectomy is the most common surgical procedure performed on an emergency basis in Western medicine. Appendicitis has a negative appendectomy rate of approximately 22% to 26% in broad based reviews. The perforation rate is as low as 3.6% in a subset of young males, although this rises substantially when the children or the elderly are included. Likewise, young females represent a group at particularly high risk for other intraabdominal pathology.
Of adult patients presenting to the emergency room for evaluation of acute abdominal pain, which one of the following answers includes the most common diagnoses? e. Urologic problems, cholelithiasis, pelvic inflammatory disease f. Mittelschmerz, appendicitis, ureterolithiasis g. Nonspecific abdominal pain, appendicitis, intestinal obstruction h. Appendicitis, pelvic inflammatory disease, perforated ulcer Answer: c Numerous surgical causes exist for the patient presenting with acute abdominal pain. A recent review of nearly 1200 patients presenting for emergency evaluation of abdominal pain affords some interesting findings. The most common diagnosis was nonspecific abdominal pain, occurring in 35% of patients. Appendicitis (17%), intestinal obstruction (15%), urologic problems (6%), and gallstones (5%) were the leading surgical causes. The largest number of admissions occurred in the age groups 10–29 years old (31%) and 60–79 years old (29%). Surgical procedures were required in
530
47% of these patients. Large series of elderly patients presenting with acute abdominal pain have found the leading diagnoses to be cholelithiasis, nonspecific pain, malignancy, incarcerated hernia, ileus, and gastroduodenal ulcer. Nonsurgical causes of acute abdominal pain may include which of the following? i. Hyperthyrodism j. Adrenal insufficiency k. Pneumonia l. Diabetic ketoacidosis Answer: b, c, d Many nonsurgical problems cause acute abdominal pain. A partial listing is provided above. Of the choices in question, the only one that is not associated with acute abdominal pain is hyperthyroidism. The remainder cause abdominal pain through a variety of mechanisms, both direct and indirect. NONSURGICAL CAUSES OF THE ACUTE ABDOMEN METABOLIC Diabetic ketoacidosis Porphyria Adrenal insufficiency Uremia Hypercalcemia TOXIC Insect bites Venoms (scorpion, snake) Lead poisoning Drugs MISCELLANEOUS Hemolytic crises Rectus sheath hematoma NEUROGENIC
531
Herpes zoster Abdominal epilepsy Spinal cord tumor, infection Nerve root compression CARDIOPULMONARY Pneumonia Myocardial infarction Myocarditis Empyema Costochondritis
Which of the following cause visceral pain from the abdominal organs? m. Stretching and contraction n. Traction, compression, torsion o. Cutting p. Certain chemicals Answer: a, b, d Abdominal pain can be divided into three categories; visceral, somatic, and referred. The intramural sensory receptors of the abdominal organs are responsible for visceral pain. A diverse group of destructive stimuli to the abdominal viscera are painless. For example, almost all abdominal organs are insensitive to pinching, burning, stabbing, cutting, and electrical and thermal stimulation. The same is true for the application of acid and alkali to normal mucosa. The general classes of visceral stimulation that result in abdominal pain include: (1) stretching and contraction; (2) traction, compression, and torsion; (3) stretch alone; and (4) certain chemicals. Mediating receptors for these responses are located intramurally in hollow organs, on serosal structures such as the visceral peritoneum and capsule of solid organs, within the mesentery and the mucosa. These receptors are polymodal, or responsive to both mechanical and chemical stimuli. Mucosal receptors respond primarily to chemical stimulation. Visceral pain almost always heralds intra-abdominal disease but may not indicate the need for surgical therapy. When visceral pain is superceded by somatic pain, the need for surgical intervation becomes likely.
Factors which may influence the clinical presentation of intraabdominal pathology include which of the following?
532
q. Pregnancy r. Oral anticoagulants s. Age t. HIV infection Answer: a, b, c, d A variety of conditions influence the presentation of intraabdominal pathology. Pregnancy is among these, principally because of displacement of adjacent normal viscera and therefore a shift in the location of the parietal pain. Oral anticoagulation is associated with the development of spontaneous intramural hematomas of the bowel causing pain but not requiring surgical resection. This pain may be confused with a variety of other intraabdominal emergencies. Age is likewise a confounding factor, generally in infancy and in the elderly. In these age groups, the symptoms may be less pronounced and the presentations occur later in the course of disease. Immunocompromised patients are a heterogenous group that includes those receiving allografts, chemotherapy, immunosuppressive drugs for autoimmune disorders, and individuals with the acquired immunodeficiency syndrome (AIDS). This group has a variety of specific abdominal complications that must be appreciated and suspected by the evaluating physician. ACUTE ABDOMINAL PAIN ASSOCIATIONS IN THE IMMUNOCOMPROMISED PATIENT CYTOMEGALOVIRUS INFECTION Interstitial pneumonitis Mononucleosis Pancreatitis Hepatitis Cholecystitis Gastrointestinal ulceration PANCREATITIS Steroids Azathioprine Cytomegalovirus Pentamidine
533
HEPATITIS Hepatitis A, B, and C Cytomegalovirus Epstein-Barr virus CHOLECYSTITIS Cytomegalovirus Acalculous cholecystitis Campylobacter HEPATOSPLENIC ABSCESS Fungal Mycobacterial Protozoal Splenic rupture BOWEL PERFORATION Lymphoma, leukemia (especially after chemotherapy) Cytomegalovirus Colon ulcers Kaposi sarcoma Pseudomembranous colitis Mycobacteria latrogenic ACUTE GRAFT-VERSUS-HOST DISEASE PSEUDOACUTE ABDOMEN FECAL IMPACTION STANDARD ABDOMINAL PROCESSES Appendicitis
534
Cholecystitis Diverticulitis Bowel obstruction Ulcer disease Pelvic inflammatory disease Perirectal abscess Urinary tract infection Lymphadenitis NEUTROPENIC ENTEROCOLITIS
Prospective studies have shown incidental appendectomy to be advantageous in which of the following patient groups? u. Children undergoing staging laparotomy for malignancy who are then to enter chemotherapy v. HIV infected patients w. Patients over 50 years of age x. Patients with spinal cord injuries y. None of the above Answer: e Several studies have looked at incidental appendectomies in a variety of populations. The deficiency in all past studies of this issue is the lack of prospective long-term trials to assess the true cost and benefit. Incidental appendectomy is clearly not indicated in the elderly and in patients undergoing laparatomy for staging of Hodgkin’s disease. These two specific groups have been shown to have increased perioperative risks with incidental appendectomy. No prospective studies have addressed the issue of HIV infected or spinal cord injured patients. While incidental appendectomies may be performed safely in general, it is difficult to justify any increase in operative risk without demonstrable benefit.
Visceral pain is typically: z. Well localized
535
aa. Sharp bb. Mediated via spinal nerves cc. Perceived to be in the midline Answer: d Peritoneum is a continuous visceral and parietal layer. The nerve supply to each layer is separate. The visceral layer, i.e., the layer surrounding all intraabdominal organs, is supplied by autonomic nerves (sympathetic and parasympathetic) and the parietal peritoneum is supplied by somatic innervation (spinal nerves). The pathways relaying the sensation of pain differ for each layer and differ in quality as well. Visceral pain is characteristically dull, crampy, deep, aching and may involve sweating and nausea. Parietal pain is sharp, severe and persistent. Visceral organs have very little pain sensation, but stretching of the mesentery and stimulation of the parietal peritoneum cause severe pain. Normal embryologic development of the abdominal viscera proceeds with bilateral midline autonomic innervation that results in visceral pain usually being perceived as arising from the midline. Epigastric pain is typical of foregut origin. Periumbilical pain signifies pain emanating from the midgut. Hypogastric or lower abdominal midline pain indicates a hindgut origin.
True statements regarding the pathophysiology of acute appendicitis include which of the following: dd. Fecaliths are responsible for the disease process in approximately 30% of adult patients ee. Lymphoid hyperplasia is a rare cause of appendicitis in young patients ff. Clostridium difficile is implicated as a pathogenic organism gg. Carcinoid tumors account for approximately 5% of all cases of acute appendicitis Answer: a The most common cause of appendicitis is obstruction of the appendiceal lumen. In young children and young adults, the most common cause of lumenal obstruction is lymphoid hyperplasia from the submucosal follicles which are abundant. Lymphoid hyperplasia accounts for 60% of acute appendicitis in the young. In adults, fecalith formation accounts for approximately 30% of acute appendicitis. There is no known causative relationship of Clostridium difficile or other specific organisms with acute appendicitis. The normal flora of the appendix is consistent with that of the adjacent cecum. Neoplasms of the appendix are rare, occurring in 1% to 1.3% of all appendectomy specimens. Carcinoid tumors are the most common, followed in frequency by benign and malignant mucoceles.
A 26-year old woman in her first trimester of pregnancy presents with a 2-day history of right lower quadrant pain and fever. Physical examination reveals a tender, palpable, right lower quadrant mass. There is no evidence of peritonitis or systemic sepsis. Laboratory
536
evaluation is remarkable for mild leukocytosis, and abdominal ultrasound demonstrates an inflammatory mass but no evidence of abscess. As the surgeon on call, your recommendation would be: Intravenous hydration, antibiotic prophylasis, and urgent appendectomy Intravenous hydration, antibiotics, bowel rest, and interval appendectomy in 4 to 6 weeks Intravenous hydration, antibiotics, and appendectomy if no improvement in 12 to 24 hours Intravenous hydration, antibiotics, and interval appendectomy when fever has subsided, leukocyte count has returned to normal, and the patient is pain free Emergent obstetrical consultation for evaluation and treatment of possible ectopic pregnancy Answer: a The patient presented has a perforated appendix with a phlegmon, but no abscess. One must routinely provide resuscitation and broad spectrum antiobiotic coverage in this circumstance. As she is not systemically toxic, it would be rational in a nonpregnant patient to treat this patient nonoperatively initially and follow this with interval appendectomy. However, in this circumstance, the risk of preterm labor associated with anesthesia and pelvic inflammation increases with more advanced gestation, so the best decision is to proceed with intravenous hydration, broad spectrum antibiotic coverage and urgent appendectomy.
True statements regarding appendiceal neoplasms include which of the following? Carcinoid tumors of the appendix less than 1.5 cm are adequately treated by simple appendectomy Appendiceal carcinoma is associated with secondary tumors of the GI tract in up to 60% of patients Survival following right colectomy for a Dukes’ stage C appendiceal carcinoma is markedly better than that for a similarly staged colon cancer at 5 years Mucinous cystadenocarcinoma of the appendix is adequately treated by simple appendectomy, even in patients with rupture and mucinous ascites Up to 50% of patients with appendiceal carcinoma have metastatic disease, with the liver as the most common site of spread Answer: a Carcinoids represent two-thirds of all appendiceal neoplasms. Nearly half of all GI carcinoids arise in the appendix at a mean age of 41 years. Two-thirds of the time the carcinoid is only incidentally detected, only 0.5% have evidence of distant metastatic spread at resection. In one experience, carcinoids between 1.5 and 2.0 cm have had minimal metastatic potential and those smaller than 1.5 cm never metastasized. In the 1% that are larger than 2 cm however, metastases are frequent and 80% recur even after resection at this size. Adenocarcinoma of the appendix is exceedingly rare. These tumors occur in elderly patients at the base of the appendix. Appendicitis often follows and the diagnosis is not made preoperatively and is
537
rarely considered during surgery since the appearance of the tumor may mimic perforated appendicitis. Up to half the patients have metastatic disease at diagnosis and the peritoneum is the most common site of spread. Survival is proportional to tumor stage. Dukes’ Stage A disease may be treated simply with appendectomy if all disease can be removed with reasonable margins. Dukes’ B and C lesions require formal right hemicolectomy for disease control. Survival is, stage for stage, similar to colon cancer after 5 years. Appendiceal adenocarcinomas also appear to have an association with secondary tumors, often of the GI tract, in up to 35% of patients. Patients with mucinous cystadenocarcinoma of the appendix typically are symptomatic, and wide resection of the primary disease, together with debulking of peritoneal implants, is indicated. Indolent progression of metastases commonly results in prolonged survival rates (50% at 5 years) during which patients may require repeated laparatomies for complications of the disease. General Surgery Medical Students Exam (Fourth Year) Choose the correct answer: 1. Maintenance fluid in surgical patients a.
Fluid requirement in patient with a temperature of 40 o C increases by 500 ml per day
b.
Adult require around 200 mmol of Na per day as maintenance
c.
Kidney has the affinity to preserve K level more efficiently than Na level
d.
Sweat is very poor in electrolytes
e.
None of the above
2. in acid base balance a.
Kidney control Hco3 through reabsorbtion of secreted bicarbonate in distal renal tubules.
b.
Respiratory compensation takes minutes to work
c.
Acids are molecules that has the ability to accept H+ ions
d.
Normal PH is about4.35-7.45
e.
None of the above
3. Causes of metabolic acidosis include all except a. Severe hypoxemia
538
b.
Liver failure
c.
Intestinal fistula
d.
Decrease tissue perfusion
e.
Hypokalemia
4. Regarding anion gap a.
Wide gap is caused by repeated vomiting
b.
Biliary fistula has no effect on anion gap
c.
Wide gap is present in severe hemorrhage
d.
Is defined as the difference between measured anions and measured cations
e.
Diarrhea leads to wide anion gap
5 .Prognostic nutritional indices a.
Serum albumin less than 40 gm/l
b.
Decrease total lymphocytic count
c.
Serum transferin more than 220 mg/dl
d.
Wasting over muscles of facial expressions
e.
All of the above
6. Regarding colorectal cancer a. Dukes B staging is with lymph node metastasis b.
T4 tumor is adherent to surrounding organs
c.
Caecal tumors is usually presenting with large bowel obstruction
d.
Change in bowel habit is a late symptom
e.
Sensitivity of screening with fecal occult blood is about 80%
7. Regarding total body water (TBW) all are true except a.
Insensible losses is rich in electrolytes
b.
Insensible losses are about 800 ml daily
c.
Extracellular water is about 14 liter
d.
The daily requirement of water in children is higher than adults
e.
The osmolality of the extracellular fluid is determined primarily by Na+,Cl- concentration
8. Regarding polyps of the colon
539
a.
Hamartomatus polyps has very little malignant transformation
b.
Sessile polyps are usually found in right colon
c.
Hyperplastic polyps are usually precancerous
d.
Tubular polyps are usually multiple and pedunculated
e.
Cancer risk is not related to type of polyp
9. The best method of demonstrating the presence of celiac disease is: a. Radiographic study of the upper intestinal tract b.
Small bowel biopsy
c.
Schilling test
d.
History of specific food intolerance
e.
Quantitative stool fat test
10. Regarding Crohn’s disease a.
Cryptitis is diagnostic.
b.
Loss of goblet cell mucin is common.
c.
Sub mucosal fibrosis.
d.
Deep caseating granulomas
e.
Commonest area affected is small bowel alone
11. Which of these statements regarding Meckel’s diverticulum is correct a.
It is found in about 4% of the population.
b.
Is always found on the antimesenteric border of the ileum.
c.
Is usually about 2 cm in length
d.
Is usually located about 20 cm from the ileocaecal junction.
e.
Is usually attached to the umbilicus
12. Small bowel obstruction
540
a.
The commonest cause is hernias
b.
Post laparotomy adhesions usually occur in first year
c.
Bioresorbable barrier membrane technology has no role in adhesion prevention
d.
CT scan has minimal role in diagnosis small bowel obstruction
e.
None of the above
13. Ulcerative colitis a.
Peak incidence is in middle age females
b.
Usually start in sigmoid colon
c.
Usually runs intermittent course of relapse and remissions
d.
Incidence of dysplasia is related to severity of disease
e.
Stricture formation is a common complication
14. Regarding malignant melanoma all are true except a.
Half of malignant melanoma arise in preexisting nevi
b.
Superficial spreading melanoma is most common type
c.
Nodular melanoma has more vertical growth
d.
The most important prognostic factor is depth of lesion
e.
Clarks level three reaches reticular dermis
15. Regarding skin cancers a.
Seborrheic keratoses is a predisposing factor
b.
Squamous cell carcinoma is the commonest type of skin cancer
c.
Basal cell carcinoma spread usually by lymphatic’s
d.
Liposarcoma is the most common sarcoma of middle age
e.
None of the above
16 .Regarding mammograms a.
Is usually recommended for screening above thirty years of age
b.
Usually two views Crainocaudal and oblique
c.
Macro calcification is an important sign of malignancy
d.
Axillary lymph nodes does not show in mammograms````````````````
e.
Ultrasound is more important in elderly
17. Fibroadenoma
541
a.
Occur in female about age of thirty
b.
Cause periareolar inflammation and fibrosis
c.
Incidence of malignant changes is about 2%
d.
Can undergoes spontaneous regression if large
e.
Usually are mobile,smooth,firm lumps
18. Physiological nipple discharge have all criteria except a. Come from both breasts b.
Occult blood –ve
c.
Single duct
d.
Not spontaneous
e.
No underlying mass
19. Medullary thyroid cancer all are true except a.
Arise from Parafollicular C cells
b.
Account for 5% of thyroid malignancy
c.
Accompany MEN type I
d.
Most are sporadic
e.
Has calcitonin as tumor marke
20. Third space loss is due to all except a.
Burns crush syndrome.
b.
Severe soft tissue infections
c.
Repeated vomiting
d.
Intestinal obstruction
e.
Site of major operative dissectio
21. Regarding follicular thyroid cancer a.
Account for 40% of thyroid cancer
b.
Early lymphatic spread
c.
Commonest thyroid cancer in children
d.
Common in endemic goiter areas
e.
Multifocality is common
22. Pressure symptoms of thyroid cancer include all except
542
a.
Dyspnea
b.
Inferior venacava obstruction
c.
Dysphagia
d.
Orthopnea
e.
Hoarseness of voice
23. Regarding anal fissure a.
Acute anal fissure is usually deep
b.
skin tag is present in acute fissure
c.
Chronic anal fissure is usually anterior in females
d.
All of the above
e.
none of the above
24. Hemorrhoid all are true except a.
Fourth degree hemorrhoids cannot be reduced
b.
external piles arise below dentate line
c.
Straining and constipation is an important cause
d.
Causes fresh bleeding per rectum ,after defecation ,and on surface of stool
e.
First degree hemorrhoid cause pain only
25. Anal fistula all are true except a.
Commonest type is transsphincteric fistula
b.
Goodsall’s rule states that posterior fistula open in mid line posteriorly
c.
Most fistula are creptoglandular in origin
d.
Is an abnormal communication between two epithelial surfaces
e.
Usually start as an abscess in intersphincteric anal glands
26. Inguinal hernia
543
a.
More common than femoral hernia in female
b.
Medial boundary is the dangerous boundary as femoral vein lies
c.
Represent 40% of male hernias
d.
Cause gastroenteritis like symptoms
e.
None of the above
27. Femoral canal a.
Lacunar ligament form lateral boundary
b.
Roof is formed by pectenial ligament
c.
Lateral boundary is safe during low femoral hernia repair
d.
Obstructed femoral hernia is usually Richter's type
e.
Femoral artery form the lateral boundary
28. Regarding desmoid tumors all are true except a.
Is a low grade fibro sarcoma
b.
More common in females
c.
Has high rate of local recurrence
d.
Metastasis is common
e.
All of the above
29. Causes of hypercalcaemia include a.
Osteolytic bone metastasis
b.
hyper parathyroidism
c.
Prolong immobilization in bed
d.
Milk alkali syndrome
e.
All of the above
30. Difference between direct and indirect hernia include a. Direct hernia is common in children
544
b.
Indirect hernia neck lie medial to inferior Epigastric artery
c.
Strangulation is more common in direct hernia
d.
Indirect hernia can reach scrotum
e.
Deep ring test prevent direct inguinal hernia from bulging
SANSWERS: Medical Students Exam ( Fourth Year ) 1.A
2.B
3.E
4.C
5.B
6.B
7.A
8.D
9.B
10.C
11.B
12.B
13.C
14.E
15.D
16.B
17.E
18.C
19.C
20.C
21.D
22.B
23.E
24.E
25.E
26.B
27.D
28.D
29.E
30.D
Neurosurgery 1. Intracranial aneurysms: A. Are the cause of the vast majority of cases of spontaneous subarachnoid haemorrhage. B. Are multiple in 20 per cent of cases. C. Rarely rebleed after an initial haemorrhage. D. Which have ruptured require surgical treatment which involves clipping of the appropriate middle cerebral artery. E. C&D only. 2. In head injuries the causes of a rising intracranial pressure: (all correct except one) A. Intracerebral haemorrhage. B. Cerebral oedema. C. Rhinorrhoea. D. Meningitis. E. Extradural hemorrhage. 3. In head injuries the signs of an expanding intracranial lesion include: A. A falling level of consciousness. B. A rising pulse rate. C. A falling blood pressure. D. Small pupils. E. B&C only. 4. Folloeing head injuries surgical intervention is usually required for: A. Linear skull fractures. B. Cerebral oedema. C. Depressed skull fractures. D. Extradural haemorrhage. E. C&D only.
545
5. Chronic subdural haematomata: (all correct except one) A. Are common in the young. B. Have a characteristic angiographic appearance. C. Should be treated surgically. D. Have a better postoperative prognosis than acute subdural haematomata. E. Are venous in orgin. 6. The characteristic signs of chronically raised intracranial pressure include: (all correct except one) A. A bitemporal hemianopia. B. Papilloedema. C. Epilepsy. D. Bradycardia. E. Six nerve pulsy. 7. In a patient with a lumber disc protrusion: A. Loss of the knee jerk is characteristic of a second lumbar nerve root lesion. B. Loss of dorsiflexion of the great toes indicates a third lumbar nerve root lesion. C. Loss of sensation over the sole indicates a fourth lumbar nerve root lesion. D. Loss of the ankle jerk indicates a first sacral root lesion. E. Loss of planter flexion of great toe indicates fifth lumber nerve root lesion. 8. In lesions affecting the common peroneal (lateral popliteal) nerve: A. Sensory loss is limited to the dorsal aspect of the first interdigital cieft. B. There is weakness of dorsiflexion of the foot. C. There is weakness of eversion of the foot. D. The toes become clawed. E. B&C only. 9. Birth injuries involving the fifth and sixth cervicaal nerve roots of the brachial plexus: A. Are known as klumpke’s palsy. B. Are rarely followed by full recovery. C. Are characterised by the arm being held in the pronated and internally rotated position . D. Show weakness and wasting of the small muscles of the hand. E. Non of the above correct. 10. Following a peripheral nerve injury: A. Loss of axon continuity is described as neuropraxia. B. Due to gunshot wounds primary nerve repair is desirable. C. Delayed suture is best performed one week after the injury. D. Delayed suture is best performed three months after the injury. E. None of the above is correct. Neurosurgery 1. Intracranial aneurysms: A. Are the cause of the vast majority of cases of spontaneous subarachnoid haemorrhage. B. Are multiple in 20 per cent of cases. C. Rarely rebleed after an initial haemorrhage. D. Which have ruptured require surgical treatment which involves clipping of the appropriate middle cerebral artery. E. C&D only.
546
2. In head injuries the causes of a rising intracranial pressure: (all correct except one) A. Intracerebral haemorrhage. B. Cerebral oedema. C. Rhinorrhoea. D. Meningitis. E. Extradural hemorrhage. 3. In head injuries the signs of an expanding intracranial lesion include: A. A falling level of consciousness. B. A rising pulse rate. C. A falling blood pressure. D. Small pupils. E. B&C only. 4. Folloeing head injuries surgical intervention is usually required for: A. Linear skull fractures. B. Cerebral oedema. C. Depressed skull fractures. D. Extradural haemorrhage. E. C&D only. 5. Chronic subdural haematomata: (all correct except one) A. Are common in the young. B. Have a characteristic angiographic appearance. C. Should be treated surgically. D. Have a better postoperative prognosis than acute subdural haematomata. E. Are venous in orgin. 6. The characteristic signs of chronically raised intracranial pressure include: (all correct except one) A. A bitemporal hemianopia. B. Papilloedema. C. Epilepsy. D. Bradycardia. E. Six nerve pulsy. 7. In a patient with a lumber disc protrusion: A. Loss of the knee jerk is characteristic of a second lumbar nerve root lesion. B. Loss of dorsiflexion of the great toes indicates a third lumbar nerve root lesion. C. Loss of sensation over the sole indicates a fourth lumbar nerve root lesion. D. Loss of the ankle jerk indicates a first sacral root lesion. E. Loss of planter flexion of great toe indicates fifth lumber nerve root lesion. 8. In lesions affecting the common peroneal (lateral popliteal) nerve: A. Sensory loss is limited to the dorsal aspect of the first interdigital cieft. B. There is weakness of dorsiflexion of the foot. C. There is weakness of eversion of the foot. D. The toes become clawed. E. B&C only. 9. Birth injuries involving the fifth and sixth cervicaal nerve roots of the brachial plexus: A. Are known as klumpke’s palsy. B. Are rarely followed by full recovery. C. Are characterised by the arm being held in the pronated and internally rotated position . D. Show weakness and wasting of the small muscles of the hand. E. Non of the above correct.
547
10. Following a peripheral nerve injury: A. Loss of axon continuity is described as neuropraxia. B. Due to gunshot wounds primary nerve repair is desirable. C. Delayed suture is best performed one week after the injury. D. Delayed suture is best performed three months after the injury. E. None of the above is correct. Pretest Surgery ( 2 ) 1. Wasting of the intrinsic muscles of the hand can be expected to follow injury of the A. Ulnar nerve B. Radial nerve C. Brachial nerve D. Axillary nerve 2. A 30-year-old man is stabbed in the arm. There is no evidence of vascular injury, but he cannot flex his three radial digits. He has injured the C. Median nerve 3. Although wide surgical excision is the traditional treatment for malignant melanoma, narrow excision of thin (less than 1 mm deep) stage I melanomas has been found to be equally safe and effective when the margin of resection is as small as C. 1 cm 4. With regard to wound healing, which one of the following statements is correct? B. Monocytes are essential for normal wound healing 5. While you are on duty in the emergency room, a 12-year-old boy arrives with pain and inflammation over the ball of his left foot and red streaks extending up the inner aspect of his leg. He remembers removing a wood splinter from the sole of his foot on the previous day. The most likely infecting organism is E. Streptococcus 6. The appropriate antibiotic to prescribe while awaiting specific culture verification is A. Penicillin 7. Proper treatment for frostbite consists of D. Immersion of the affected part in water at 40–44°C (104–111.2°F) 8. The true statement regarding tendon injuries in the hand is E. The process of healing a tendon injury involves formation of a tenoma 9. Which one of the following cases is considered a clean contaminated wound A. Open cholecystectomy for cholelithiasis B. Herniorrhaphy with mesh repair C. Lumpectomy with axillary node dissection D. Appendectomy with walled-off abscess 10. True statements regarding squamous cell carcinoma of the lip include A. The lesion often arises in areas of persistent hyperkeratosis B. More than 90% of cases occur on the upper lip C. The lesion constitutes 30% of all cancers of the oral cavity D. Radiotherapy is considered inappropriate treatment for these lesions
548
11. Which of the following statements regarding carpal tunnel syndrome is correct? B. It may be associated with pregnancy 12. Which of the following is true with regard to wound contraction? D. It is based on specialized fibroblasts that contain actin myofilaments 13. Management of leukoplakia of the oral cavity includes A. Excisional biopsy of all lesions B. Application of topical antibiotics C. Low-dose radiation therapy D. Ascertaining that dentures fit properly 14. An 8-lb infant, born following uncomplicated labor and delivery, is noted to have a unilateral cleft lip and palate .The parents should be advised that A. The child almost certainly has other congenital anomalies B. Rehabilitation requires adjunctive speech therapy C. Lip repair is indicated at 1 year of age D. Palate repair is indicated prior to 6 mo of age 15. A teenage boy falls from his bicycle and is run over by a truck. On arrival in the emergency room, he is awake and alert and appears frightened but in no distress. The chest radiograph suggests an airfluid level in the left lower lung field and the nasogastric tube seems to coil upward into the left chest. The next best step in management is C. Immediate celiotomy 16. Which of the following conditions is most likely to follow a compression-type abdominal injury? A. Renal vascular injury B. Superior mesenteric thrombosis C. Mesenteric vascular injury D. Avulsion of the splenic pedicle E. Diaphragmatic hernia 17. A 65-year-old man who smokes cigarettes and has chronic obstructive pulmonary disease falls and fractures the 7th, 8th, and 9th ribs in the left anterolateral chest. Chest x-ray is otherwise normal. Appropriate treatment might include D. Peritoneal lavage 18. A 27-year-old man sustains a single gunshot wound to the left thigh. In the emergency room he is noted to have a large hematoma of his medial thigh. He complains of paresthesias in his foot. On examination there are weak pulses palpable distal to the injury and the patient is unable to move his foot. The appropriate initial management of this patient would be B. Immediate exploration and repair Items 19–20 A 25-year-old woman arrives in the emergency room following an automobile accident. She is acutely dyspneic with a respiratory rate of 60 breaths/min. Breath sounds are markedly diminished on the right side. 19. The first step in managing the patient should be to C. Decompress the right pleural space 20. A chest x-ray of this woman Before therapy would probably reveal A. Air in the right pleural space
549
21. Among the physiologic responses to acute injury is A. Increased secretion of insulin B. Increased secretion of thyroxine C. Decreased secretion of vasopressin (ADH) D. Decreased secretion of glucagon 22. In a stable patient, the management of a complete transection of the common bile duct distal to the insertion of the cystic duct would be optimally performed with a D. Roux-en-Y choledochojejunostomy 23. Non operative management of penetrating neck injuries has been advocated as an alternative to mandatory exploration in asymptomatic patients. Which of the following findings would constitute a relative, rather than an absolute, indication for formal neck exploration? A. Expanding hematoma B. Dysphagia C. Dysphonia D. Pneumothorax E. Hemoptysis 24. Following blunt abdominal trauma, a 12-year-old girl develops upper abdominal pain, nausea, and vomiting. An upper gastrointestinal series reveals a total obstruction of the duodenum with a “coiled spring” appearance in the second and third portions. Appropriate management is B. Nasogastric suction and observation 25. Following traumatic peripheral nerve transection, regrowth usually occurs at which of the following rates? A. 0.1 mm per day B. 1 mm per day C. 5 mm per day D. 1 cm per day Items 26–27 A 28-year-old man is brought to the emergency room for a severe head injury after a fall. Initially lethargic, he becomes comatose and does not move his right side .His left pupil is dilated and responds only sluggishly. 26. The most common initial manifestation of increasing intracranial pressure in the victim of head trauma is A. Change in level of consciousness 27. Initial emergency reduction of intracranial pressure is most rapidly accomplished by E. Hyperventilation 28. In the patient described, compression of the affected nerve is produced by B. Herniation of the uncal process of the temporal lobe 29. Regarding high-voltage electrical burns to an extremity D. Evaluation for fracture of the other extremities and visceral injury is indicated 30. Which of the following fractures or dislocations of the extremities induced by blunt trauma is associated with significant vascular injuries? A. Knee dislocation
550
31. Protein metabolism after trauma is characterized by A. Decreased liver gluconeogenesis B. Inhibition of skeletal muscle breakdown) C. Decreased urinary nitrogen loss D. Hepatic synthesis of acute-phase reactants 32. The response to shock includes which of the following metabolic effects? A. Increase in sodium and water excretion B. Increase in renal perfusion C. Decrease in cortisol levels D. Hyperkalemia E. Hypoglycemia 33. Appropriate treatment for an acute stable hematoma of the pinna of the ear includes which of the following measures? A. Ice packs and prophylactic antibiotics B. Excision of the hematoma C. Needle aspiration D. Incision, drainage, and pressure bandage 34. Animal and clinical studies have shown that administration of lactated Ringer’s solution to patients with hypovolemic shock may A. Increase serum lactate concentration B. Impair liver function C. Improve hemodynamics by alleviating the deficit in the interstitial fluid compartment 35. Which of the following situations would be an indication for performance of a thoracotomy in the emergency room? C. Rapidly deteriorating patient with cardiac tamponade from penetrating thoracic trauma Items 36–37 An 18-year-old high school football player is kicked in the left flank. Three hours later he develops hematuria.His vital signs are stable. 36. The diagnostic tests performed reveal extravasation of contrast into the renal parenchyma .Treatment should consist of E. Antibiotics and serial monitoring of blood count and vital signs 37. Initial diagnostic tests in the emergency room should include which of the following? D. Intravenous pyelogram 38. True statements concerning penetrating pancreatic trauma include E. The major cause of death is exsanguination from associated vascular injuries 39. A 26-year-old man sustains a gunshot wound to the left thigh. Exploration reveals that a 5cm portion of superficial femoral artery is destroyed .Appropriate management includes A. Debridement and end-to-end anastomosis B. Debridement and repair with an interposition prosthetic graft C. Debridement and repair with an interposition arterial graft D. Debridement and repair with an interposition vein graft
551
Pretest Surgery ( 3 ) 1. Estrogen receptor activity is clinically useful in predicting A. The presence of ovarian cancer B. The presence of metastatic disease C. Response to chemotherapy D. Response to hormonal manipulation 2. When galactorrhea occurs in a high school student, a diagnostic associated finding would be A. Gonadal atrophy B. Bitemporal hemianopia C. Exophthalmos and lid lag E. “Buffalo hump” 3. The diagnosis of primary hyperparathyroidism is most strongly suggested by A. Serum acid phosphatase above 120 IU/L B. Serum alkaline phosphatase above 120 IU/L C. Serum calcium above 11 mg/dL D. Urinary calcium below 100 mg/day 4. Somatostatin contributes to which of the following processes? B. Inhibition of pancreatic cells 5. A 35-year-old woman undergoes her first screening mammogram. Which of the following mammographic findings would require a breast biopsy? A. Breast calcifications larger than 2 mm in diameter B. Five or more clustered breast microcalcifications per square centimeter C. A density that effaces with compression E. Multiple round well-circumscribed breast densities 6. . A 49-year-old man has become irritable, his facies have changed to a round configuration, he is impotent, he has developed purplish lines on his flanks, and he is hypertensive. Which of the following statements concerning his diagnosis is true? a- Adrenal adenomas cause 40 to 60% of all cases of this condition b- Biochemical and x-ray procedures are generally unsuccessful in lateralizing the tumor preoperatively c- Exploration of both adrenal glands is indicated d- For uncomplicated tumors, an open trans-peritoneal surgical approach is usually employed e- Postoperative corticoid therapy is required to prevent hypoadrenalism 7. Which statement concerning radiation-induced thyroid cancer is true? E. The treatment of choice is a neartotal (or total) thyroidectomy 8. Fibrocystic disease of the breast has been associated with elevated blood levels of A. Testosterone B. Progesterone C. Estrogen D. Luteinizing hormone
552
9. As an incidental finding during an upper abdominal CT scan, a 3-cm mass in the adrenal gland is noted. The appropriate next step in analysis and management of this finding would be A. Observation B. CT-guided needle biopsy C. Excision of the mass D. Measurement of urine catecholamine excretion E. Cortisol provocation test 10. The most likely diagnosis in a patient with hypertension, hypokalemia, and a 7-cm suprarenal mass is A. Hypernephroma B. Cushing’s disease C. Adrenocortical carcinoma D. Pheochromocytoma 11. Appropriate treatment of this condition would include which of the following? A. Embolization of the arterial blood supply, B. Metronidazole C. Mitotane E. Phenoxybenzamine 12. For pregnant women who are found to have breast cancer E. Administration of adjuvant chemotherapy is safe for the fetus during the second and third trimesters 13. True statements regarding Paget’s disease of the breast include that it A. Usually precedes development of Paget’s disease of bone B. Presents with nipple-areolar eczematous changes C. Does not involve axillary lymph nodes because it is a manifestation of intraductal carcinoma only D. Accounts for 10–15% of all newly diagnosed breast cancers E. Is adequately treated with wide excision when it presents as a mass 14. Of the common complicationsof thyroidectomy, the one that may be avoided through prophylaxis is A. Injury to the recurrent laryngeal nerve B. Injury to the superior laryngeal nerve D. Thyroid storm E. Postoperative hemorrhage and wound hematoma 15. Following correction of the patient’s hypercalcemia with hydration and gentle diuresis with furosemide, the most likely therapeutic approach would be D. Neck exploration and resection of a parathyroid adenoma 16. This 30-year-old woman presented with weakness, bone pain, an elevated parathormone level, and a serum calcium level of 15.2 mg/dL. Skeletal survey films were taken, including the hand films and chest x-ray shown. The most likely cause of these findings is E. Primary hyperparathyroidism
553
17. A 36-year-old woman, 20 wk pregnant, presents with a 1.5-cm right thyroid mass. Fineneedle aspiration is consistent with a papillary neoplasm. The mass is “cold” by scan and solid by ultrasound. Which method of treatment would be contraindicated? A. Right thyroid lobectomy B. Subtotal thyroidectomy C. Total thyroidectomy D. Total thyroidectomy with lymph node dissection E. 131I radioactive ablation of the thyroid gland 18. Incisional biopsy of a breast mass in a 35-year-old woman demonstrates a hypercellular fibroadenoma (cystosarcoma phylloides) at the time of frozen section. Appropriate management of this lesion could include A. Wide local excision with a rim of normal tissue 19. True statements about discharge from the nipple include A. Intermittent thin or milky discharge can be physiologic B. Expressible nipple discharge is an indication for open biopsy C. Bloody discharge is indicative of an underlying malignancy D. Galactorrhea is indicative of an underlying malignancy 20. The incidence of breast cancer A. Increases with increasing age B. Has declined since the 1940s C. Is related to dietary fat intake D. Is related to coffee intake E. Is related to vitamin C intake 21. A cross-match is performed by incubating B. Donor lymphocytes with recipient serum and complement 22. The primary mechanism of action of cyclosporine A is inhibition of D. Interleukin 2 production 23. After the first postoperative year of cardiac transplantation, the most common cause of death is C. Accelerated graft arteriosclerosis 24. Which of the following precludes cadaveric renal transplantation? A. Positive cross-match 25. In centers with experienced personnel, 1-year liver transplant survival is now approximately B. 80% 26. Graft-versus-host disease has occurred with the transplantation of which of the following? D. Bone marrow 27. The most useful serum marker for detecting recurrent disease after treatment of nonseminomatous testicular cancer is B. alpha-fetoprotein (AFP) 28. For which of the following malignancies does histologic grade best correlate with prognosis? E. Soft tissue sarcoma
554
29. Interferons are correctly characterized by which of the following statements? B. They are produced by virus-infected cells 30. Which of the following potentially operable complications is a common occurrence among patients receiving systemic chemotherapy? B. Perirectal abscess 31. Human immunodeficiency virus (HIV) has been isolated from many body fluids. Which of the following is a major source of transmission? A. Tears B. Sweat C. Semen D. Urine E. Breast milk 32. Which of the following agents causes hemorrhagic cystitis? E. Cyclophosphamide 33. What is the most common serious complication of an end colostomy? A. Bleeding B. Skin breakdown C. Parastomal hernia D. Colonic perforation during irrigation E. Stomal prolapse 34. Which of the following colonic pathologies is thought to have no malignant potential? A. Ulcerative colitis B. Villous adenomas C. Familial polyposis D. Peutz-Jeghers syndrome E. Crohn’s colitis 35. Which of the following hernias follows the path of the spermatic cord within the cremaster muscle? A. Femoral B. Direct inguinal C. Indirect inguinal D. Spigelian 36. Spontaneous closure of which of the following congenital abnormalities of the abdominal wall generally occurs by the age of 4? A. Umbilical hernia B. Patent urachus C. Patent omphalomesenteric duct D. Omphalocele 37. Laparoscopic cholecystectomy is indicated for symptomatic gallstones in which of the following conditions? A. Cirrhosis B. Prior upper abdominal surgery C. Suspected carcinoma of the gallbladder D. Morbid obesity E. Coagulopathy
555
38. In determining the proper treatment for a sliding hiatal hernia, the most useful step would be A. Barium swallow with cinefluoroscopy during Valsalva maneuver B. Flexible endoscopy C. 24-h monitoring of esophageal pH D. Measuring the size of the hernia 39. A previously healthy 9-year old child comes to the emergency room because of fulminant upper gastrointestinal bleeding. The hemorrhage is most likely to be the result of A. Esophageal varices B. Mallory-Weiss syndrome C. Gastritis D. A gastric ulcer 40. Intragastric pressure remains steady near 2–5 mm Hg during slow gastric filling, but rises rapidly to high levels after reaching a volume of A. 400–600 mL B. 700–900 mL C. 1000–1200 mL D. 1300–1500 mL 41. Local stimuli that inhibit the release of gastrin from the gastric mucosa include which of the following? A. Small proteins B. 20-proof alcohol C. Caffeine D. Acidic antral contents 42. For a symptomatic partial duodenal obstruction secondary to an annular pancreas, the operative treatment of choice is A. A Whipple procedure B. Gastrojejunostomy D. Partial resection of the annular pancreas E. Duodenojejunostomy 43. Which of the following would be expected to stimulate intestinal motility? A. Fear B. Gastrin C. Secretin D. Acetylcholine E. Cholecystokinin 44. Which of the following statements concerning carcinoma of the esophagus is true? B. Squamous carcinoma is the most common type at the cardioesophageal junction C. It has a higher incidence in males D. It occurs more commonly in patients with corrosive esophagitis E. Surgical excision is the only effective treatment 45. The most common clinical presentation of idiopathic retroperitoneal fibrosis is A. Ureteral obstruction B. Leg edema D. Jaundice E. Intestinal obstruction
556
46. In planning the management of a 2.8-cm epidermoid carcinoma of the anus, the first therapeutic approach should be E. Combined radiation therapy and chemotherapy 47. Indications for operation in Crohn’s disease include which of the following? A. Intestinal obstruction B. Enterovesical fistula C. Ileum–ascending colon fistula E. Free perforation 48. Which of the following is most likely to require surgical correction? A. Large sliding esophageal hiatal hernia B. Paraesophageal hiatal hernia C. Traction diverticulum of esophagus D. Schatzki’s ring of distal esophagus 49. Which statement regarding adenocarcinoma of the pancreas is true? A. It occurs most frequently in the body of the gland B. It carries a 1–2% 5-year survival rate C. It is nonresectable if it presents as painless jaundice D. It can usually be resected if it presents in the body or tail of the pancreas and does not involve the common bile duct 50. Which of the following statements regarding direct inguinal hernias is true? A. They are the most common inguinal hernias in women B. They protrude medially to the inferior epigastric vessels C. They should be opened and ligated at the internal ring D. They commonly protrude into the scrotal sac in men 51. Which of the following statements regarding stress ulceration is true? A. It is true ulceration, extending into and through the muscularis mucosa B. It classically involves the antrum C. Increased secretion of gastric acid has been shown to play a causative role D. It frequently involves multiple sites 52. Which statement concerning cholangitis is correct? A. The most common infecting organism is Staphylococcus aureus B. The diagnosis is suggested by the Charcot triad (fever, jaundice, pain) C. The disease occurs primarily in young, immunocompromised patients D. Cholecystectomy is the procedure of choice in affected patients 53. Indications for surgical removal of polypoid lesions of the gallbladder include A. Size greater than 0.5 cm B. Presence of clinical symptoms C. Patient age of over 25 years D. Presence of multiple small lesions 54. A patient who has a total pancreatectomy might be expected to develop which of the following complications? A. Diabetes mellitus B. Hypercalcemia C. Hyperphosphatemia D. Constipation E. Weight gain
557
55. True statements regarding cavernous hemangiomata of the liver in adults include A. The majority become symptomatic B. They may undergo malignant transformation C. They enlarge under hormonal stimulation D. They should be resected to avoid spontaneous rupture and lifethreatening hemorrhage 1 2 3 4 5 6 7 8 9 10
D B C B B C E C A C
11 12 13 14 15 16 17 18 19 20
C E B D D E E A A A
21 22 23 24 25 26 27 28 29 30
B D C A B D B E B B
31 32 33 34 35 36 37 38 39 40
C E C D C A D B A C
41 42 43 44 45 46 47 48 49 50
D E D D A E E B B B
51 52 53 54 55
Pretest Surgery ( 4 ) 1. The superior vena cava syndrome is most frequently seen in association with A. Histoplasmosis (sclerosing mediastinitis) C. Thoracic aortic aneurysm D. Constrictive pericarditis E. Bronchogenic carcinoma 2. A 3-year-old child with congenital cyanosis is most probably suffering from A. Tetralogy of Fallot B. Ventricular septal defect C. Tricuspid atresia D. Transposition of the great vessels 3. Superior pulmonary sulcus carcinomas (Pancoast tumors) are bronchogenic carcinomas that typically produce which of the following clinical features? A. Atelectasis of the involved apical segment B. Horner syndrome (miosis, ptosis, anhidrosis) C. Pain in the T4 and T5 dermatomes D. Nonproductive cough 4. Which of the following statements is true concerning aortocoronary bypass grafting? A. It is indicated for crescendo (preinfarction) angina 8. Which of the following statements is true regarding the thoracic outlet syndrome? C. If conservative measures fail, it is best treated by surgical decompression of the brachial plexus 6. A 2-year-old asymptomatic child is noted to have a systolic murmur, hypertension, and diminished femoral pulses. Which of the following is true about this child’s disorder? C. Rib notching is often seen on x-ray (Coarctation of the aorta) 7. A correct statement concerning bronchial carcinoid tumors is that C. They rarely produce the carcinoid syndrome
558
D B B A C
8. Patients with phlebographically confirmed deep vein thrombosis of the calf E. Are at risk for significant pulmonary embolism 9. Symptoms or signs of atherosclerotic occlusive disease of the bifurcation of the abdominal aorta (Leriche syndrome) include A. Claudication of the buttock and thigh 10. Initial management of a patient who has a flaccid neurogenic bladder may include which of the following measures? A. Surgical bladder augmentation B. Self-catheterization C. Supravesical urinary diversion D. Limiting fluid intake to less than 300 mL/day 11. The recommended treatment for stage A (superficial and sub mucosal)transitional cell carcinoma of the bladder is A. Local excision B. Radical cystectomy D. Topical (intravesicular) chemotherapy E. Systemic chemotherapy 12. Seminoma is accurately described by which of the following statements? A. It is the most common type of testicular cancer B. Metastases to liver and bone are frequently found C. It does not respond to radiation E. Common presentation is that of a painful lump that trans illuminates 13. Meniscal tears usually result from which of the following circumstances? A. Hyperextension B. Flexion and rotation C. Simple hyperflexion D. Compression 14. In an uncomplicated dislocation of the glenohumeral joint, the humeral head usually dislocates primarily in which of the following directions? A. Anteriorly B. Superiorly C. Posteriorly D. Laterally 15. Which of the following fractures is most commonly seen in healthy bones subjected to violent falls? A. Colles fracture B. Femoral neck fracture C. Intertrochanteric fracture D. Clavicular fracture 16. Which of the following statements regarding compartment syndromes following orthopedic injuries is true? A. The first sign is usually loss of pulse in the extremity B. Passive flexion of the extremity proximal to the involved compartment will aggravate the pain C. Surgical decompression (fasciectomy) is necessary only as a last resort D. These syndromes are most commonly associated with supracondylar fractures of the humerus and tibial shaft
559
17. In contrast to closed reduction, open reduction of a fracture A. Produces a shorter healing time B. Decreases trauma to the fracture site C. Produces a higher incidence of nonunion D. Reduces the risk of infection 18. In a failed suicide gesture, a depressed student severs her Radial nerve at the wrist. The expected disability is A. Loss of ability to extend the wrist B. Loss of ability to flex the wrist C. Wasting of the intrinsic muscles of the hand D. Sensory loss over the thenar pad and the thumb web 19. The most severe epiphyseal growth disturbance is likely to result from which of the following types of fracture? E. Crushing injury compressing the growth plate 20. Which of the following statements regarding the Glasgow coma scale is true? B. A high score correlates with a high mortality C. It includes measurement of intracranial pressure D. It includes measurement of pupillary reflexes E. It includes measurement of verbal response 21. Which of the following statements regarding glioblastoma multiforme is true? B. It arises from the malignant degeneration of an astrocytoma 22. Which of the following statements regarding skull fractures is true? C. Any bone fragment displaced more than 1 cm inwardly should be elevated surgically 23. An acute increase in intracranial pressure is characterized by which of the following clinical findings? A. Respiratory irregularities B. Decreased blood pressure C. Tachycardia D. Papilledema 24. Which of the following statements about schwannomas is true? B. Treatment is via excision 25. Which of the following statements regarding cerebral contusions is true? B. They may occur opposite the point of skull impact 26. True statements regarding meningiomas include that they a.Are malignant in 50% of cases b.Occur predominantly in men c.Are treated primarily by surgical excision d.Are cured, when properly treated, in nearly 95% of cases e.Arise from the dura 27. Which of the following statements about craniopharyngiomas is true? D. The tumors may cause compression of the optic tracts and visual symptoms
560
28. Which of the following statements regarding symptomatic thyroglossal duct cysts is true? A. Over 90% manifest themselves before age 12 B. Treatment includes resection of the hyoid bone C. They usually present as a painful swelling in the lateral neck Thyroglossal duct cysts result from retention of an epithelial tract between the thyroid and its embryologic origin in the foramen cecum at the base of the tongue. This tract usually penetrates the hyoid bone. There is no sex predilection, and although these cysts are more frequently detected in children, they may not become symptomatic until adulthood. The most common presentation is a painless swelling in the midline of the neck that moves with protrusion of the tongue or swallowing. They should not be confused with midline ectopic thyroid tissue. The cysts are prone to infection and progressive enlargement. Although rare (<1%), epidermoid or papillary carcinomas do occur within thyroglossal duct cysts. Surgical resection is the standard therapy. The Sistrunk procedure, which involves local resection of the cyst and the central portion of the hyoid bone, is the operation of choice. Simple excision of the cyst results in an unacceptably high recurrence rate. 29. Pleomorphic adenomas (mixed tumors) of the salivary glands are characterized by which of the following? A. They occur most commonly on the lips, tongue, and palate 30. Verrucous carcinoma of the buccal mucosa is identified with which of the following characteristics? C. It has a predilection for the gingivobuccal gutter 1 2 3 4 5 6 7 8 9 10
E A B A C C C E A B
11 12 13 14 15 16 17 18 19 20
D A B A D D C D E E
21 22 23 24 25 26 27 28 29 30
B C A B B C D B A C
1. The mortality rate in multi-organ failure is directly related to: a. The number of the failing organs b. The presence of septic shock c. The presence of co-morbidities d. Extremes of age e. The underlying cause 2. Which of the following organs has the earliest involvement in multi-organ failure? a. Kidney b. Brain c. Lung d. Heart e. Adrenal
561
3. Which of the following types of shock is characterized by Bradycardia ? a. Hypovolemic b. Septic c. Neurogenic d. Cardiogenic e. Cardiac compressive 4. After tissue injury, the earliest event is : a. Platelet aggregation b. Monocyte migration c. Vasoactive substance release d. Vasoconstriction e. Extrinsic pathway activation 5. Regarding wound healing, which is false ? a. The tensile strength of the injured tissue equals that of normal after 6 months . b. The capillaries are highly permeable in inflammatory phase . c. Disruption of normal healing may result in malignancy d. Vitamin C deficiency causes decreased proline hydroxylation e. Steriods affect inflammatory phase 6. The most common organism isolated from a localized wound infection is : a. Streptococci b. E. Coli c. Staphylococci d. Pseudomonas e. Bacteriodes 7. Regarding necrotizing fasciitis , which is false ? a. Rapidly progressive course b. Monomicrobial or multimicrobial etiology c. It is a common surgical infection d. Debridement is the mainstay of treatment e. Characterized by intense pain 8. Regarding achalasia which is true ? a. The affected segment is the lower esophagus b. It has the classic triad of dysphagia,vomiting and chest pain c. Aperistalsis is an early finding d. Chronic untreated disease has a malignant potential e. Regurgitation of gastric contents frequently occurs 9. Regarding diffuse esophageal spasm all of the following are true except : a. Chest pain is a frequent symptom b. Sigmiod (S-shaped) esophagus in barium study c. High amplitude contractions in manometry d. Psychiatric disorders are common e. Treated by myotomy 10. The gold standard test to measure acid reflux is : a. Endoscopy b. barium swallow c. Acid perfusion test d. Manometry e. 24-hour pH monitoring
562
11. The risk of esophageal adenocarcinoma is increased by : a. Alcohol abuse b. Metaplasia of the lower esophagus c. Long standing achalasia d. Smoking e. Caustic stricture 12. The most common indication for urgent surgery due to peptic ulcer is : a. Bleeding b. Obstruction c. Perforation d. Intractability e. Penetration 13. Postoperative adhesions in most patients occur following which procedure ? a. Cholecystectomy b. Exploratory laparotomy c. Ventral hernia repair d. Laparoscopic procedures e. Appendectomy 14. In evaluating intestinal obstruction, which of the following suggests strangulation? a. Increased vomiting frequency b. Increased intensity of pain c. Localization of symptoms and signs d. Sudden disappearance of pain e. None of the above 15. The best initial test in a patient with fever, left lower quadrant pain and tenderness is : a. Endoscopy b. Barium enema c. Ultrasound d. CT scan e. Plain x-rays 16. In a pregnant woman with appendicitis, the risk of abortion is significantly increased by: a. High grade fever b. Worsening of pain c. Perforation of the appendix d. Being in the third trimester e. WBC count > 16,000 17. The main advantage of laparoscopic appendectomy compared to open appendectomy is: a. Fewer days of hospital stay b. Less pain c. Less risk of wound infection d. Less need for antibiotics e. Less operative time 18. Regarding gastric cancer which is false ? a. Patients with blood group A are at increased risk b. Most patients suffer from anorexia and weight loss c. Intestinal type has a worse prognosis d. H. pylori infection is a main etiologic factor e. Endoscopy is the diagnostic method of choice
563
19. The main pathway of gastric cancer spread is : a. Hematogenous b. Lymphatic c. Peritoneal seeding d. Direct spread to the adjacent organs e. a & b 20. The most common tumor of the parotid gland is : a. Mixed tumor b. Warthin’s tumor c. Mucoepidermiod carcinoma d. Adeniod cystic carcinoma e. Teratoma 21. Salivary duct stones is mostly found in: a. Parotid gland b. Submandibular gland c. Sublingual gland d. Minor salivary glands e. b & c 22. The most common cause of hyperparathyroidism is : a. Chronic renal failure b. Parathyriod adenoma c. Parathyriod hyperplasia d. Parathyriod carcinoma e. Long standing hypocalcemia 23. The best study /procedure for localizing hyper-functioning parathyroid(s) is : a. Ultrasound b. Calcium to phosphate ratio in blood sample from jugular vein c. Neck CT d. Operative exploration e. Sestamibi scan 24. In chronic renal failure , secondary hyperparathyroidism occurs mainly due to : a. Low calcium to phosphate ratio b. Hypercalciuria c. Lack of conversion of vitamin D to its active form d. Osteopenia e. Parathyriod hormone resistance 25. Regarding cystic hygroma which is false : a. It is a congenital anomaly b. Mostly found in the anterior triangle c. It is a benign lesion d. It has a brilliant transillumination e. It can be associated with fetal hydrops
564
Shock and blood transfusion 1. 'Shock' can be most comprehensively defined as: A. A sudden large volume blood loss. B. A diminished effective circulating fluid volume. C. A hypotensive state with peripheral vasoconstriction. D. An unexpected psychological insult. E. A&C only. 2. In all forms of shock there is: A. An impairment of cellular oxygenation. B. A decreased cardiac output. C. An increased effective circulating fluid volume. D. A low central venous pressure (CVP). E. An increased pulse rate. 3. The metabolic acidosis of shock can be effectively treated by: A. Warming the patient. B. Administering ammonia chloride. C. Artificial ventilation. D. Restoring normal tissue perfusion. E. Nacl intravenous infusion. 4. In hypovolaemic shock: A. The central venous pressure is high. B. The difference in arteriovenous oxygen tension is unaffected. C. The extremities are pale, cold and sweating. D. Urine output is unaffected. E. C&D only. 5. Hypovolaemic shock may result from:(all correct except one) A. A 25 per cent third degree burn. B. Generalized peritonitis. C. Massive pulmonary embolism. D. Intestinal obstruction. E. Massive blood loss. 6. Septic shock is particularly associated with: (all correct except one) A. Thoracic surgical patients. B. Hypovolaemia. C. Indwelling urinary or intravenous catherers. D. Gram-negative bacteraemia. E. Gram-positive. 7. septic shock is associated with a hypodynamic cardiovascular state:( all correct except one) A. if preceded by existing hypovolaemia. B. In generalized peritonitis. C. When there is a gram-positive bacteraemia . D. In elderly patients. E. In late gram negative septicemia. 8. The mortality from septic shock can be effectively reduced by:(all correct except one) A. Surgical drainage of abscesses. B. The administration of appropriate antibiotics. C. The restoration of a normal cardiovascular state.
565
D. Positive pressure respiration via an indotracheal intubation or tracheostomy. E. The administration of corticosteroid. 9. In cardiogenic shock: A. The central venous pressure is hig. B. The difference in the arteriovenous oxygen tension is increased. C. The haematocrit is raised. D. The blood pressure is unaffected. E. C&D only. 10. Prospective blood donors: A. Should be asked about previous attacks of jaundice. B. Should have serological tests for syphilis. C. May transmit glandular fever to a recipient. D. May transmit malaria to a recipient. E. All the above. 11. A blood transfusion reaction:(all correct except one) A. May be due to incompatibility of the recipient serum and donor cells. B. Is manifest by thrombophlebitis of the infusion site. C. Occurs within the first 30 minutes of transfusion. D. May produce renal damage. E. May produce anaphylactic shock. 12. Pyrexial reactions to blood transfusions:(all correct except one) A. Have decreased since the introduction of sterile disposable infusion sets. B. May be caused by allergic reactions. C. May be caused by contaminated blood. D. May be a response to a large transfusion of cooled blood. E. Usually caused by pyrogen reaction. 13. Massive blood transfusions may be complicated by:(all correct except one) A. Hyperkalemia. B. Hypocalcaemia. C. Coagulopathy. D. Leucopenia. E. DIC. Nd 1. A 44-year-old man presents with painless rectal bleeding of 1 month's duration. He reports a history of constipation. He works in heavy labor. For this patient, which of the following statements regarding internal hemorrhoids is true? Choose one answer a. Stapled hemorrhoidectomy should be done for grade 1 and 2 hemorrhoids b. All of the above c. Internal hemorrhoids are located proximal to the dentate line and therefore are usually painless d. A grade 1 internal hemorrhoid represents bleeding with prolapse 2 .39. A 34-year-old woman presents for evaluation of severe and frequent bloody bowel movements, as well as abdominal pain, dehydration, and anemia. She has had these symptoms for 2 days. She has not had any similar symptoms in the past, and she has been in relatively good health. If the patient has toxic megacolon, under what circumstances emergency surgical management is indicated? Choose one answer.
566
a. b. c. d.
There is a perforation Any of the above The patient's clinical or radiographic status worsens There is no improvement in 24 to 36 hours after aggressive medical therapy
3 .Complications of untreated pancreatic pseudocysts include all of the following EXCEPT: Choose one answer. a. intracystic hemorrhage b. abscess c. pancreatic necrosis d. free rupture e. gastrointestinal obstruction 4 Platelets in the wound form a hemostatic clot and release clotting factors to produce: Choose one answer. f. thrombin g. Fibrin h. Fibrinogen i. Fibroblasts j. thromboplastin 5 A 43-year-old man presents to the office for evaluation of recent weight loss and frequent loose stools. He is concerned because his father was diagnosed with colon cancer at the age of 50. Besides family history, what are some other risk factors for colorectal cancer? Choose one answer. a. Hypertension b. Diabetes c. All of the above d. Inflammatory bowel disease 6 A 40-year-old woman presents to the office for evaluation of yellowish skin. She states that over the past few weeks, she has noticed that her eyes and skin have developed a yellow tint. She also reports that she has dark urine and pale-colored stools. Further history elicits periodic bouts of right upper quadrant pain after eating. She is otherwise healthy. She denies using any medications. On physical examination, a yellowish tint is observed on the patient's skin, sclera, and mucous membranes. On the basis of this patient's history and clinical examination, which type of bilirubin would you expect to predominate? Choose one answer. e. Mixed f. Conjugated g. Unconjugated h. Indirect
567
7 39. A 38-year-old man presents with a complaint of a slow-growing mass over his right parotid gland. The lesion is fixed to the underlying structures and has recently become painful. Which of the following features strongly suggests that this patient's lesion is a malignancy? Choose one answer. a. Ipsilateral numbness of the tongue b. .All of the above c. Overlying skin involvement d. Facial nerve paralysis 8 Axillary lymph nodes are classified according to the relationship with the Choose one answer. a. pectoralis minor muscle b. pectoralis major muscle c. axillary vein d. serratus anterior muscle e. latissimus dorsi muscle 9 39. A 78-year-old man is recovering from abdomino-perineal (A-P) resection for Ca rectum, which was performed 3 days ago. The patient is now complaining of mild shortness of breath and chest pain. On physical examination, the patient's right leg is slightly more swollen than his left. The pulse oximetry reading is 90%. What is the principal method of diagnosing acute pulmonary embolism? Choose one answer. a. Magnetic resonance imaging b. Chest x-ray c. Ultrasound d. Spiral computed tomography scanning 10 Compartment syndrome Choose one answer. a. Passive stretch decrease muscle pain b. Due to decrease pressure in muscle compartments c. Pulse is the first thing to disappear d. Cause severe pain in the limb e. Treatment is by delayed fasciotomy 11 The most significant risk factor for the development of adenocarcinoma of the esophagus is: Choose one answer. a. lye stricture b. alcohol abuse c. Barrett's esophagus d. long-standing achalasia e. smoking 12 All of the following statements are true about patients with carcinoid tumors EXCEPT: Choose one answer. a. the combination of streptozotocin and 5-fluorouracil (chemotherapy) can often result in objective response. b. tumor growth is often slow c. they often have evidence of serotonin production d. the majority have carcinoid syndrome e. they have a much better prognosis if the tumors are less than 2 cm.
568
13 39. A 67-year-old man presents with left-lower-quadrant pain and low-grade fever. He has had these symptoms for 1 day. The patient denies experiencing any rectal bleeding, but for the past week, his bowel movements have been irregular. For this patient, which of the following statements is true regarding diverticular disease? Choose one answer. a. The sigmoid colon is the most common site of diverticula b. All of the above c. Most diverticula of the colon involve the muscular layer d. Smoking does not seem to be related to the development of diverticular disease 14 All of the following are components of the MEN type 2B syndrome except: Choose one answer. a. Multiple neuromas on the lips, tongue, and oral mucosa b. Medullary thyroid carcinoma c. Pheochromocytoma d. Hyperparathyroidism. 15 The risk of bilateral breast cancer is HIGHEST if the first breast shows: Choose one answer. a. inflammatory carcinoma b. medullary carcinoma c. infiltrating ductal carcinoma d. paget’s disease e. lobular carcinoma 16 The best initial therapy for deep venous thrombosis of the common femoral vein is: Choose one answer. f. warfarin g. streptokinase h. Heparin i. venous thrombectomy j. placement of a vena caval filter 17 Complications after thyroidectomy include all the following EXCEPT: Choose one answer. a. recurrent laryngeal nerve paralysis b. parathyroid insufficiency c. thyrotoxic crisis(storm) on operating on inadequitly prepared thyrotoxic patient d. tracheomalacia e. hypercalcemia 18 A 55-year-old man presents with hematemesis that began 2 hours ago. He is hypotensive and has altered mental status. No medical history is available. For this patient, which of the following statements regarding nasogastric aspiration is true? Choose one answer. a. None of the above b. A clear, nonbilious aspirate rules out the need for EGD c. A clear, bilious aspirate rules out the need for EGD d. A bloody aspirate is an indication for esophagogastroduodenoscopy (EGD) 19 The major cause of impaired wound healing is: Choose one answer. a. steroid use b. malnutrition c. diabetes mellitus d. local tissue infection
569
e. anemia 20 Common presenting conditions in patients with pancreatic carcinoma include all of the following EXCEPT: Choose one answer. a. esophageal varices. b. palpable gallbladder c. weight loss. d. abdominal pain 21 All of the following statements about keloids are true EXCEPT: Choose one answer. a. Keloid tissue contains an abnormally large amount of collagen b. A keloid does not regress spontaneously c. Keloid tissue contains an unusually large amount of soluble collagen d. A keloid extends beyond the boundaries of the original wound e. Keloids or hypertrophic scars are best managed by excision and careful reapproximation of the wound 22 The treatment of choice for a 40-year-old man who is found on endoscopy and biopsy to have a gastric lymphoma would be: Choose one answer. a. wide local excision b. subtotal gastrectomy c. chemotherapy d. subtotal gastrectomy and radiotherapy e. Radiotherapy 23 The treatment of an esophageal burn with a caustic agent may include all of the following EXCEPT: Choose one answer. a. boogieing. (dilatation) b. induction of vomiting c. gastrectomy d. expeditious administration of an antidote e. steroids and antibiotics. 24 A 32-year-old man with a family history of familial adenomatous polyposis (FAP) presents with hematochezia. He denies having any diarrhea, abdominal pain, or fever. For this patient, which of the following statements regarding FAP is true? Choose one answer. a. CRC does not occur in patients with FAP if they are given adequate medical treatment b. Total proctocolectomy (TPC) is considered the only option for the surgical management of FAP c. For patients with FAP, there is a 10% risk of CRC by age 40 if prophylactic colectomy is not performed d. In the setting of FAP, colorectal cancer (CRC) is more commonly located on the left side 25 When stage I breast cancer is treated by partial mastectomy and axillary dissection, further therapy should include: Choose one answer a. antiestrogen agents. b. radiation of the affected breast. c. oophorectomy if premenopausal.
570
d. nothing e. chemotherapy 26 For the patient in Question 68, which of the following statements is true regarding an esophageal varix as the site of bleeding? Choose one answer. a. I.V. propranolol should be administered first b. Balloon tamponade should be performed first c. Rubber banding or intravariceal sclerotherapy should be performed first d. I.V. somatostatin should be administered first 27 Drugs which may produce gynecomastia include all of the following EXCEPT: Choose one answer. a. furosemide b. cimetidine c. Verapamil d. Diazepam e. Tamoxifen 28 Splenectomy is commonly indicated for the following EXCEPT: Choose one answer. a. hypersplenism associated with cirrhosis b. hereditary spherocytosis c. splenic tumor d. immune thrombocytopenic purpura e. grade four splenic injury in trauma 29 An ischiorectal abscess is characterized by all of the following EXCEPT: Choose one answer. a. Requires deroofing b. Should be treated entirely by antibiotics c. May be tuberculous in origin d. Can be followed by anal fistula e. Is an infective necrosis of the fat of the ischiorectal fossa 30 Marks: 1 The most common presentation of Meckel’s diverticulum in an adult is: Choose one answer. a. intussuception b. Littre’s hernia c. Gastrointestinal bleeding d. diverticulitis 31 For the patient in Question 65, which of the following is an indication for immediate surgery? Choose one answer. a. Closed-loop obstruction b. All of the above c. Complete bowel obstruction d. Incarcerated hernia 32 Choledocholithiasis in a patient who previously had cholecystectomy is BEST treated with: Choose one answer. a. endoscopic sphincterotomy b. choledochoduodenostomy c. choledochojejunostomy. d. dissolution with mono-octanoin e. open common bile duct exploration with stone removal
571
33 39. A 77-year-old man undergoes endoscopic ultrasonography as part of a workup for jaundice. He is found to have a tumor in the head of the pancreas. For this patient, which of the following findings would indicate that the tumor is unresectable? Choose one answer. a. All of the above b. Peritoneal metastases c. Invasion of the superior mesenteric artery d. Metastases to celiac lymph nodes 34 Regarding polyps of the colon Choose one answer. a. Villous polyps are usually pedunculated b. Villous polyps occur more proximal in colon c. Adenomatous polyps are usually solitary d. Cancer risk is not related to size of polyp e. Metaplastic polyps are not precancerous 35 The most likely diagnosis in elderly patient with abdominal pain and colonoscopy finding of patchy mucosal ulceration at the splenic flexure of the colon is : Choose one answer. a. ulcerative colitis b. crohns disease c. ischemic colitis d. diverticulitis e. lymphogranuloma venerum 36 39. A 54-year-old man presents with a neck mass of 2 weeks' duration. He has no significant medical history. He smokes two packs of cigarettes a day and has been doing so since he was 21 years of age. For this patient, which of the following statements is true? Choose one answer. a. Low cervical nodes are more likely to contain metastases from a primary source other than the head and neck, whereas upper cervical nodes are more likely to contain metastases from the head and neck b. Soft or tender nodes are more likely to derive from an inflammatory or infectious condition, whereas hard, fixed, painless nodes are more likely to represent metastatic cancer c. All of the above d. Enlarged lymph nodes are by far the most common neck masses encountered 37 The first-choice diagnostic study for suspected deep venous thrombosis of the lower extremity is: Choose one answer. f. real-time Doppler imaging g. contrast sonography h. radioactive labeled fibrinogen uptake i. impedance plethysmography j. isotope injection with gamma scintillation scanning 38 Decreased PaCO2 levels should be attained in a patient at serious risk for cerebral edema secondary to a head injury in order to : Choose one answer.
572
a. b. c. d. e.
prevent neurogenic pulmonary edema prevent increased capillary permeability prevent metabolic acidosis allow reciprocally high levels of PaO2 in the brain prevent cerebral vasodilation
39 Causes of third space loss include all except: Choose one answer. a. Acute pancreatitis b. Pancreatic fistula c. Necrotizing fasciitis d. Site of major surgery e. Crush syndrome 40 Causes of metabolic acidosis include all except: Choose one answer. a. Small bowel fistula b. Shock c. CO poisoning d. severe anemia e. All of the above 41 A 39-year-old man comes in for evaluation of intermittent anal pain and bleeding after bowel movements. He also has hard stools. He has had these symptoms for over 1 year . For this patient, which of the following is included in the classic triad of signs of chronic anal fissures? Choose one answer. a. An anal fissure b. All of the above c. A sentinel skin tag d. Hypertrophy of the anal papilla 42 All the following statements about achalasia are true EXCEPT: Choose one answer. a. it occurs most commonly in persons between the ages of 30 and 50 years b. in most affected persons, ganglion cells in the body of the esophagus either are absent or have degenerated c. esophageal cancer is seven times as common in affected persons as in the general population d. pressure in the body of the esophagus is lower than normal e. affected persons usually experience more difficulty swallowing cold foods than warm foods ?? 43 A patient with the Zollinger-Ellison syndrome is found to have the multiple endocrine neoplasia type I (MEN-I) syndrome. Appropriate management for the ulcer symptoms should be: Choose one answer. a. Omeprazole b. pancreatic resection c. streptozocin d. cimetidine e. total gastrectomy
573
44 Fat absorption occurs primarily in the: Choose one answer. a. Ileum b. third portion of the duodenum c. Stomach d. Jejunum e. first portion of the duodenum 45 39. A 56-year-old woman has been experiencing abdominal pain for 4 hours. The pain is in right upper quadrant and radiates into the scapular region. She has had multiple episodes of vomiting. For this patient, which of the following signs on physical examination is associated with acute cholecystitis? Choose one answer. e. Carnett sign f. Kehr sign g. Murphy sign h. Rovsing sign 46 Mammary duct ectasia is characterized by the following EXCEPT : Choose one answer. a. Is treated usually by simple mastectomy b. May present with nipple retraction and Peau d'orange picture c. Anaerobic superinfection cmmmonly occurs in this recurrent periductal plasma cell mastitis d. Is defined as primary dilatation of major ducts of breast in middle aged women e. Iscommonly pre-malignant 47 Regarding veins of lower limbs all are true except: Choose one answer. e. Valves allow flow from deep to superficial system Venous return from lower limbs is aided by respiratory movements f. Superficial veins lie in subcutaneous tissue g. The pressure in veins of the foot while standing is 100mmHg h. a) Stasis of blood is important factor in developing varicose veins 48 Most common complication of central venous access is: Choose one answer. f. major artery damage. g. Catheter problems. h. Thrombosis of central vein i. Catheter related sepsis j. Pleural space damage, pneumothorax 49 Factors associated with increase risk of death in acute pancreatitis include all except: Choose one answer. a. Ranson score more than five b. Obesity? c. High APACHE_II score d. Age more than 70 years e. sterile necrosis 50 For the patient in Question 56, which of the following chronic conditions can cause a neck mass? Choose one answer. a. Tuberculosis b. Sarcoidosis c. AIDS
574
d.
All of the above
51 For the patient in Question 56, if metastatic cancer is suspected initially, which of the following would be the most appropriate step to take next in the workup? Choose one answer. a. CT scan b. Empirical therapy with antibiotics c. Fine-needle aspiration (FNA) d. Observation only 52 The largest component of intestinal gas is: Choose one answer. a. Nitrogen b. carbon dioxide c. Oxygen d. ammonia e. hydrogen 53 A 66-year-old woman presents to the office complaining of a sharp, constant pain in her lower abdomen. She has had this pain for the past 2 weeks. Examination of her abdomen is normal. However, subsequent ultrasound reveals a 4.5 cm Aortic abdominal aneurysm (AAA). Spiral computed tomography confirms the ultrasound findings. Aside from rupture, which of the following is a complication associated with (AAA) ? Choose one answer. a. Aortoenteric fistula b. Lower-extremity atheroemboli c. All of the above d. Thrombosis 54 Diagnosis of esophageal perforation is best established by: Choose one answer. a. transesophageal ultrasound study b. esophagoscopy with a flexible esophagoscope c. contrast esophagograms d. esophagoscopy with a rigid esophagoscope e. upright X-rays of the chest including lateral and oblique films 55 For the patient in Question 68, which of the following is an indication for surgery? Choose one answer. a. Ongoing hemorrhage occurs from a gastric ulcer in a hemodynamically unstable patient b. All of the above c. Bleeding continues from either a duodenal ulcer or a gastric ulcer despite medical and endoscopic therapy d. Substantial bleeding occurs from a duodenal ulcer that is not controlled by EGD 56 Yesterday, a 38-year-old woman underwent a laparoscopic cholecystectomy for cholelithiasis and was discharged home 8 hours after surgery. She returns this morning complaining of worsening abdominal pain. The oral narcotics that the patient was prescribed are ineffective in controlling the pain. The patient's temperature is (38.3 C). Laboratory studies reveal an elevated white blood cell count. Abdominal ultrasonography shows a large subhepatic fluid collection. The fluid is percutaneously aspirated and reveals enteric contents.
575
What step should be taken next in the management of this patient? Choose one answer. a. Immediate laparotomy b. I.V. antibiotics and close observation c. None of the above d. Observation in the hospital until pain improves 57 The most common symptom after major pulmonary embolism is: Choose one answer. a. cough b. Dyspnea? c. Hemoptysis d. pleural pain e. fear of death 58 Treatment of paralytic ileus includes all of the following EXCEPT : Choose one answer. a. intravenous fluids b. cessation of oral intake c. correction of electrolyte imbalance d. early operation e. nasogastric suction 59 Lymphedema : Choose one answer a. e) None of the above b. b) should be bilateral c. c) may be pitting in early stage d. d) A & C only e. a) may be Congenital 60 The level of consciousness for a head injury patient is BEST evaluated by : Choose one answer. a. CT scan b. visual evoked potentials c. Glasgow coma scale d. papillary responses e. response to pain 61 A 52-year-old male weighing 70 kg, sustained a 65% total body surface area (TBSA) burn. What are his fluid requirements? Choose one answer. a. 18,200 cc in 16 h b. 9100 cc in the first 8 h c. 8000 cc in 24 h d. 12,800 cc in 24 h 62 Management of cholangitis may include all of the following EXCEPT: Choose one answer. a. decomperession of the common bile duct. b. cholecystostomy c. IV antibiotics.
576
d. correct underlying cause. e. percutaneous transhepatic cholangiography. 63 A 48-year-old woman presents to the emergency department complaining of right upper quadrant pain, which began 4 hours ago. She reports the pain as being spasmodic and sharp and that it radiates to her right shoulder blade. She says that she has had similar episodes over the past few months, especially after eating large meals. Associated with the pain is nausea and vomiting. Her blood pressure is 120/85 mm Hg, and her pulse is 100 beats/min. On physical examination, the patient is found to have a nontender abdomen with no palpable masses. Her chest and cardiovascular examinations are normal. The nurse notices that her sclerae are slightly icteric. On subsequent laboratory studies, her serum bilirubin level is found to be 10 mg/dl. What imaging study should be performed next for this patient with presumed posthepatic jaundice? Choose one answer. a. Percutaneous transhepatic cholangiography (PTC) b. Ultrasonography c. Endoscopic retrograde cholangiopancreatography (ERCP) d. Magnetic resonance imaging 64 39. An HIV-positive man presents for evaluation of new oral cavity lesions he discovered last month. Physical examination reveals purple exophytic masses involving the palate mucosa and gingiva. What is the most likely diagnosis of these oral cavity lesions? Choose one answer. a. Oral hairy leukoplakia b. Non-Hodgkin lymphoma c. Syphilis d. Kaposi sarcoma 65 Brain injury alone Choose one answer. a. causes shock only if the skull is intact b. rarely causes shock c. causes shock that is reversed by very simple measures d. frequently causes shock e. causes shock if hypoxia is superimposed 66 All of the following substances are irritating to the peritoneum EXCEPT: Choose one answer. a. bile. b. blood c. gastric content. d. meconium e. pus 67 Regarding volvulus of the sigmoid colon, each of the following is true except : Choose one answer. a. likely results from redundant sigmoid colon with an elongated narrow mesocolon b. there appears to be a congenital predisposition sigmoid volvulus. c. diagnostic barium enema for sigmoid colon is essential
577
d. diagnostic x-ray for sigmoid volvulus shows a dilated loop of colon which points toward the right upper quadrant. 68 68.Gallstones are characterized by all the following EXCEPT: Choose one answer. a. cause mucocoele of the gall bladder b. are present in the common bile duct in 40% of patients with stones in the gall bladder c. are becoming common in post-partum primipara who were pre pregnancy ‘Pill’ takers d. are frequently the cause of flatulent dyspepsia e. may be present in the newborn 69 Following a burn, the agent responsible for early increased capillary permeability is Choose one answer. a. thromboxane A2 b. serotonin c. histamine? d. prostacyclin PGI2 e. bradykinin 70 Incisonal hernias all are true except: Choose one answer. a. 10 % of all hernias b. Usually easy to reduce c. Multiloculated sac d. Operative technique is most important cause e. Complication is common 71 In the treatment of gastric cancer, all of the following are true EXCEPT: Choose one answer. a. five-year survival rates in the continue to be between 10% and 25%. b. Palliative resection is frequently helpful with advanced disease. c. Lymph node involvement is associated with a poorer prognosis. d. Finding early disease at the time of operation is associated with a better prognosis. e. total gastrectomy is mandated in most patient. *** 72 of the involved area and : Severe cases of hidradenitis suppurativa in the groin area are best managed by excision Choose one answer. a. transfer of a rectus abdominus muscle flap b. split thickness skin grafting c. primary closure d. delayed primary closure e. closure by secondary intention 73 Patients at increased risk for gastric carcinoma include all the following EXCEPT: Choose one answer. a. those who have undergone gastric bypass for morbid obesity b. those who have undergone gastric resection for duodenal ulcer c. those with pernicious anemia d. those with a high consumption of smoked fish e. those with blood group A
578
74 A 56-year-old woman presents with symptoms of abdominal pain, weight loss, and rectal bleeding. She is anemic and hypotensive, but she is stable. For this patient, which of the following should be done first if the gastric lavage yields copious amounts of bile? Choose one answer. a. Arteriography b. Emergency laparotomy c. Colonoscopy d. Esophagogastroduodenoscopy 75 A 39-year-old man presents with lower GI bleeding. He has no abdominal discomfort and has experienced no loss of weight. For this patient, which of the following statements regarding the etiology of lower GI bleeding is true? Choose one answer. a. Lower GI bleeding from diverticulosis often requires surgery b. Arteriovenous malformations (AVMs) are the most common cause of lower GI bleeding c. 6% to 10% of patients with ulcerative colitis have lower GI bleeding severe enough to necessitate emergency surgical resection d. None of the above 76 For the patient in Question 65, which of the following statements regarding abdominal radiographic findings is true? Choose one answer. a. In gastric outlet obstruction, no gastric air will be seen, but large amounts of air will be seen in the small bowel and colon b. Mechanical small bowel obstruction usually shows no air-fluid levels, nor will distended bowel loops of similar sizes be seen c. All of the above d. High-grade obstruction of the colon in association with an incompetent ileocecal valve may mimic small bowel obstruction on x-ray 77 A 41-year-old female presents to the emergency department after sustaining a gunshot wound to the abdomen, with injuries to the liver and large bowel. Despite successful resuscitation and operative intervention, the patient dies 2 weeks later of multisystem organ failure in the intensive care unit. Which organ most likely first experienced dysfunction? Choose one answer. a. lung b. heart c. liver d. gastrointestinal tract e. kidney 78 39. A 35-year-old man presents with severe acute abdominal pain of sudden onset. The pain initially began in the upper abdomen and has now settled in the whole abdomen. On examination, the abdomen was rigid. Chest X-Ray showed free gas under diaphragm. Which of the following is the most appropriate course of management for this patient? Choose one answer. a. Observation b. Urgent laparotomy
579
c. Nonurgent laparotomy d. Laparoscopy
79 All are true about the dumping syndrome except: Choose one answer. a. Symptoms can be controlled with a somatostatin analogue. b. Early postoperative dumping after vagotomy often resolves spontaneously. c. Flushing and tachycardia are common features of the syndrome. d. Diarrhea is always part of the dumping syndrome. e. Separating solids and liquids in the patient's oral intake alleviates some of the symptoms of the syndrome. 80 An anal fissure is characterized by the following EXCEPT: Choose one answer. a. Is more common in women. b. Is an ulcer of the anal mucosa. c. Can be treated coservatively. d. Usually lies anteriorly. 81 All of the following statements are true of esophageal carcinoma EXCEPT Choose one answer. a. squamous cell tumor is the most frequent histology. b. operation is frequently curative. c. patient often require a pyloroplasty with operation. d. patients usually die within one year of diagnosis e. patient can have intestinal continuity reestablished using the stomach after esophageal resection. 82 A serum tumor marker correlated with recurrence after management of colon cancer is: Choose one answer. a. carcinoembryonic antigen (CEA) b. Calcitonin c. 5-hydroxyindoleacetic acid d. CA 15-3 e. Alpha-fetoprotein 83 Conditions associated with gastric cancer include all of the following EXCEPT: Choose one answer. a. adenomatous polyps. b. a high intake of dietary nitrates. c. pernicious anemia. d. higher socioeconomic groups. e. chronic atrophic gastritis. 84 Which of the following is required for addressing any pressure sore? Choose one answer. a. pressure reduction b. hyperbaric oxygen c. antibiotics d. skin flap coverage Treatment of pressure sores requires relief of pressure with special cushions and beds and nutritional support to promote healing.
580
85 The most effective treatment of achalasia is: Choose one answer. a. antireflux surgical procedures b. Esophagomyotomy c. dilation of the lower esophageal sphincter d. antispasmodic medication e. resection of the cardioesophageal junction 86 Regarding desmoid tumors all are true except: Choose one answer. a. Treatment is wide local excision with safety margins b. Has high rate of local recurrence c. Is a low grade fibro sarcoma d. More common in females e. Is radio resistant 87 Which of the following variables best predicts prognosis for patients with a recent diagnosis of cutaneous melanoma and no clinical evidence of metastatic disease? Choose one answer. a. Ulceration. b. Clark's level. c. Gender d. Bleeding e. Breslow thickness. 88 In the treatment of acute cholecystitis, most patient are BEST served with: Choose one answer. a. endoscopic sphincterotomy. b. cholecystostomy. c. percutaneous drainage of the gallbladder. d. early cholecystectomy (within 3 days of onset of symptoms). e. IV antibiotics and cholecystectomy in 6 to 8 weeks. 89 Regarding crohn's disease all are true except Choose one answer. a. Cause non caseating granuloma b. commonly affect small bowel c. Causes sub mucosal fibrosis d. It is full thickness inflammation e. Cause depletion of goblet cell mucin 90 Advantages of laparoscopic versus open cholecystectomy include all of the following EXCEPT: Choose one answer. a. decreased pain. b. less risk of bile duct injury. c. reduced hospitalization. d. improved cosmetic. e. reduced ileus. 91 The most common cause of massive hemorrhage in the lower gastrointestinal tract is: Choose one answer. a. carcinoma b. Diverticulosis c. Diverticulitis
581
d. Polyp e. ulcerative colitis
92 Carcinoma of the gallbladder is Choose one answer. a. rarely associated with jaundice. b. associated with a good prognosis. c. usually not diagnosed preoperatively. d. most commonly metastatic to the lung. e. best treated with radiation and chemotherapy 93 All the following statements concerning nipple discharges are true EXCEPT: Choose one answer. a. benign duct papillomas are the most common cause of bloody discharges. b. when bloody , the discharge is due to a malignancy 70% of the time c. a) excision of involved duct may be necessary to determine the etiology d. they may be caused by multiple lesions. e. a milky discharge may be due to a pituitary adenoma . 94 39. A 28-year-old woman presents with a complaint of a growing, painless mass in her neck. Physical examination reveals a firm, fixed nodule measuring 2 cm on the right lobe of her thyroid. The surgeon recommends fine-needle aspiration (FNA) of the lesion instead of excisional biopsy. What are the advantages of FNA over excisional biopsy? Choose one answer. a. FNA is associated with a decreased risk of tumor seeding b. FNA requires only an office visit c. Anesthesia is not necessary d. All of the above 95 For the patient in Question 74, which of the following is a sign of a colovesical fistula associated with diverticulitis, as seen on computed tomography with contrast? Choose one answer. a. Thickening of the bladder and the colon b. Sigmoid diverticula c. All of the above d. Air in the bladder 96 All of the following are true statements concerning paget ’ s disease of the nipple EXCEPT: Choose one answer. a. it is an eczematoid lesion. b. it is very uncommon, accounting for only 2% of all breast cancers. c. it is an in situ squamous cell malignancy of the nipple. d. it can be confused with malignant melanoma histologically e. it has a better prognosis than the majority of other breast cancers. 97 The classical picture of Acute arterial embolism include all the following except: Choose one answer. a. peripheral pulses b. Parasthesia c. All the above d. Pain e. Pallor
582
98 Extra colonic manifestation of inflammatory bowel disease include all except: Choose one answer. a. Erythema nodosum b. Mixed connective tissue disease c. Primary Sclerosing cholangitis d. Polyarthopathy e. Episcleritis 99 A 45-year-old woman presents with abdominal pain and vomiting of 1 day's duration. The patient underwent an exploratory laparotomy after a motor vehicle accident 8 years ago. Which of the following statements is true for this patient? Choose one answer. a. Pain from mechanical obstruction is usually more severe than pain from ileus b. The severity of pain from mechanical obstruction may decrease over time c. Pain from mechanical obstruction is usually localized in the middle of the abdomen, whereas pain from ileus or pseudo-obstruction is diffuse d. All of the above 100 A 4-year-old girl has been experiencing dyspnea on exertion. Chest x-ray shows a Morgagni hernia. For this patient, which of the following statements is true? Choose one answer. a. Morgagni hernias can be repaired with a subcostal, a paramedian, or a midline incision b. Morgagni hernias are most commonly seen on the right side c. All of the above d. The average age at diagnosis is typically greater for patients with Morgagni hernia than for patients with Bochdalek hernia 1 A 44-year-old man presents with painless rectal bleeding of 1 month's duration. He reports a history of constipation. He works in heavy labor. For this patient, which of the following statements regarding internal hemorrhoids is true? Choose one answer e. Stapled hemorrhoidectomy should be done for grade 1 and 2 hemorrhoids f. All of the above g. Internal hemorrhoids are located proximal to the dentate line and therefore are usually painless h. A grade 1 internal hemorrhoid represents bleeding with prolapse 2 39. A 34-year-old woman presents for evaluation of severe and frequent bloody bowel movements, as well as abdominal pain, dehydration, and anemia. She has had these symptoms for 2 days. She has not had any similar symptoms in the past, and she has been in relatively good health. If the patient has toxic megacolon, under what circumstances emergency surgical management is indicated? Choose one answer. e. There is a perforation f. Any of the above g. The patient's clinical or radiographic status worsens h. There is no improvement in 24 to 36 hours after aggressive medical therapy 3 Complications of untreated pancreatic pseudocysts include all of the following EXCEPT:
583
Choose one answer. f. intracystic hemorrhage g. abscess h. pancreatic necrosis i. free rupture j. gastrointestinal obstruction 4 Platelets in the wound form a hemostatic clot and release clotting factors to produce: Choose one answer. k. thrombin l. Fibrin m. Fibrinogen n. Fibroblasts o. thromboplastin 5 A 43-year-old man presents to the office for evaluation of recent weight loss and frequent loose stools. He is concerned because his father was diagnosed with colon cancer at the age of 50. Besides family history, what are some other risk factors for colorectal cancer? Choose one answer. e. Hypertension f. Diabetes g. All of the above h. Inflammatory bowel disease 6 A 40-year-old woman presents to the office for evaluation of yellowish skin. She states that over the past few weeks, she has noticed that her eyes and skin have developed a yellow tint. She also reports that she has dark urine and pale-colored stools. Further history elicits periodic bouts of right upper quadrant pain after eating. She is otherwise healthy. She denies using any medications. On physical examination, a yellowish tint is observed on the patient's skin, sclera, and mucous membranes. On the basis of this patient's history and clinical examination, which type of bilirubin would you expect to predominate? Choose one answer. i. Mixed j. Conjugated k. Unconjugated l. Indirect 7 39. A 38-year-old man presents with a complaint of a slow-growing mass over his right parotid gland. The lesion is fixed to the underlying structures and has recently become painful. Which of the following features strongly suggests that this patient's lesion is a malignancy? Choose one answer. e. Ipsilateral numbness of the tongue f. .All of the above g. Overlying skin involvement h. Facial nerve paralysis 8 Axillary lymph nodes are classified according to the relationship with the Choose one answer. f. pectoralis minor muscle
584
pectoralis major muscle axillary vein serratus anterior muscle latissimus dorsi muscle 9 39. A 78-year-old man is recovering from abdomino-perineal (A-P) resection for Ca rectum, which was performed 3 days ago. The patient is now complaining of mild shortness of breath and chest pain. On physical examination, the patient's right leg is slightly more swollen than his left. The pulse oximetry reading is 90%. What is the principal method of diagnosing acute pulmonary embolism? Choose one answer. e. Magnetic resonance imaging f. Chest x-ray g. Ultrasound h. Spiral computed tomography scanning g. h. i. j.
10 Compartment syndrome Choose one answer. f. Passive stretch decrease muscle pain g. Due to decrease pressure in muscle compartments h. Pulse is the first thing to disappear i. Cause severe pain in the limb j. Treatment is by delayed fasciotomy 11 The most significant risk factor for the development of adenocarcinoma of the esophagus is: Choose one answer. f. lye stricture g. alcohol abuse h. Barrett's esophagus i. long-standing achalasia j. smoking 12 All of the following statements are true about patients with carcinoid tumors EXCEPT: Choose one answer. f. the combination of streptozotocin and 5-fluorouracil (chemotherapy) can often result in objective response. g. tumor growth is often slow h. they often have evidence of serotonin production i. the majority have carcinoid syndrome j. they have a much better prognosis if the tumors are less than 2 cm. 13 39. A 67-year-old man presents with left-lower-quadrant pain and low-grade fever. He has had these symptoms for 1 day. The patient denies experiencing any rectal bleeding, but for the past week, his bowel movements have been irregular. For this patient, which of the following statements is true regarding diverticular disease? Choose one answer. e. The sigmoid colon is the most common site of diverticula f. All of the above g. Most diverticula of the colon involve the muscular layer h. Smoking does not seem to be related to the development of diverticular disease 14 All of the following are components of the MEN type 2B syndrome except: Choose one answer. e. Multiple neuromas on the lips, tongue, and oral mucosa f. Medullary thyroid carcinoma g. Pheochromocytoma
585
h. Hyperparathyroidism.
15 The risk of bilateral breast cancer is HIGHEST if the first breast shows: Choose one answer. f. inflammatory carcinoma g. medullary carcinoma h. infiltrating ductal carcinoma i. paget’s disease j. lobular carcinoma 16 The best initial therapy for deep venous thrombosis of the common femoral vein is: Choose one answer. k. warfarin l. streptokinase m. Heparin n. venous thrombectomy o. placement of a vena caval filter 17 Complications after thyroidectomy include all the following EXCEPT: Choose one answer. f. recurrent laryngeal nerve paralysis g. parathyroid insufficiency h. thyrotoxic crisis(storm) on operating on inadequitly prepared thyrotoxic patient i. tracheomalacia j. hypercalcemia 18 A 55-year-old man presents with hematemesis that began 2 hours ago. He is hypotensive and has altered mental status. No medical history is available. For this patient, which of the following statements regarding nasogastric aspiration is true? Choose one answer. e. None of the above f. A clear, nonbilious aspirate rules out the need for EGD g. A clear, bilious aspirate rules out the need for EGD h. A bloody aspirate is an indication for esophagogastroduodenoscopy (EGD) 19 The major cause of impaired wound healing is: Choose one answer. f. steroid use g. malnutrition h. diabetes mellitus i. local tissue infection j. anemia 20 Common presenting conditions in patients with pancreatic carcinoma include all of the following EXCEPT: Choose one answer. e. esophageal varices. f. palpable gallbladder g. weight loss. h. abdominal pain 21 All of the following statements about keloids are true EXCEPT: Choose one answer. f. Keloid tissue contains an abnormally large amount of collagen g. A keloid does not regress spontaneously
586
h. Keloid tissue contains an unusually large amount of soluble collagen i. A keloid extends beyond the boundaries of the original wound j. Keloids or hypertrophic scars are best managed by excision and careful reapproximation of the wound 22 The treatment of choice for a 40-year-old man who is found on endoscopy and biopsy to have a gastric lymphoma would be: Choose one answer. f. wide local excision g. subtotal gastrectomy h. chemotherapy i. subtotal gastrectomy and radiotherapy j. Radiotherapy 23 The treatment of an esophageal burn with a caustic agent may include all of the following EXCEPT: Choose one answer. f. boogieing. (dilatation) g. induction of vomiting h. gastrectomy i. expeditious administration of an antidote j. steroids and antibiotics. 24 A 32-year-old man with a family history of familial adenomatous polyposis (FAP) presents with hematochezia. He denies having any diarrhea, abdominal pain, or fever. For this patient, which of the following statements regarding FAP is true? Choose one answer. a. CRC does not occur in patients with FAP if they are given adequate medical treatment b. Total proctocolectomy (TPC) is considered the only option for the surgical management of FAP c. For patients with FAP, there is a 10% risk of CRC by age 40 if prophylactic colectomy is not performed d. In the setting of FAP, colorectal cancer (CRC) is more commonly located on the left side 25 When stage I breast cancer is treated by partial mastectomy and axillary dissection, further therapy should include: Choose one answer a. antiestrogen agents. b. radiation of the affected breast. c. oophorectomy if premenopausal. d. nothing e. chemotherapy 26 For the patient in Question 68, which of the following statements is true regarding an esophageal varix as the site of bleeding? Choose one answer. e. I.V. propranolol should be administered first f. Balloon tamponade should be performed first g. Rubber banding or intravariceal sclerotherapy should be performed first h. I.V. somatostatin should be administered first 27 Drugs which may produce gynecomastia include all of the following EXCEPT: Choose one answer. f. furosemide
587
g. cimetidine h. Verapamil i. Diazepam j. Tamoxifen 28 Splenectomy is commonly indicated for the following EXCEPT: Choose one answer. f. hypersplenism associated with cirrhosis g. hereditary spherocytosis h. splenic tumor i. immune thrombocytopenic purpura j. grade four splenic injury in trauma 29 An ischiorectal abscess is characterized by all of the following EXCEPT: Choose one answer. f. Requires deroofing g. Should be treated entirely by antibiotics h. May be tuberculous in origin i. Can be followed by anal fistula j. Is an infective necrosis of the fat of the ischiorectal fossa 30 Marks: 1 The most common presentation of Meckel’s diverticulum in an adult is: Choose one answer. e. intussuception f. Littre’s hernia g. Gastrointestinal bleeding h. diverticulitis 31 For the patient in Question 65, which of the following is an indication for immediate surgery? Choose one answer. e. Closed-loop obstruction f. All of the above g. Complete bowel obstruction h. Incarcerated hernia 32 Choledocholithiasis in a patient who previously had cholecystectomy is BEST treated with: Choose one answer. f. endoscopic sphincterotomy g. choledochoduodenostomy h. choledochojejunostomy. i. dissolution with mono-octanoin j. open common bile duct exploration with stone removal 33 39. A 77-year-old man undergoes endoscopic ultrasonography as part of a workup for jaundice. He is found to have a tumor in the head of the pancreas. For this patient, which of the following findings would indicate that the tumor is unresectable? Choose one answer. e. All of the above f. Peritoneal metastases g. Invasion of the superior mesenteric artery h. Metastases to celiac lymph nodes 34 Regarding polyps of the colon Choose one answer. f. Villous polyps are usually pedunculated g. Villous polyps occur more proximal in colon
588
h. Adenomatous polyps are usually solitary i. Cancer risk is not related to size of polyp j. Metaplastic polyps are not precancerous 35 The most likely diagnosis in elderly patient with abdominal pain and colonoscopy finding of patchy mucosal ulceration at the splenic flexure of the colon is : Choose one answer. f. ulcerative colitis g. crohns disease h. ischemic colitis i. diverticulitis j. lymphogranuloma venerum 36 39. A 54-year-old man presents with a neck mass of 2 weeks' duration. He has no significant medical history. He smokes two packs of cigarettes a day and has been doing so since he was 21 years of age. For this patient, which of the following statements is true? Choose one answer. e. Low cervical nodes are more likely to contain metastases from a primary source other than the head and neck, whereas upper cervical nodes are more likely to contain metastases from the head and neck f. Soft or tender nodes are more likely to derive from an inflammatory or infectious condition, whereas hard, fixed, painless nodes are more likely to represent metastatic cancer g. All of the above h. Enlarged lymph nodes are by far the most common neck masses encountered 37 The first-choice diagnostic study for suspected deep venous thrombosis of the lower extremity is: Choose one answer. k. real-time Doppler imaging l. contrast sonography m. radioactive labeled fibrinogen uptake n. impedance plethysmography o. isotope injection with gamma scintillation scanning 38 Decreased PaCO2 levels should be attained in a patient at serious risk for cerebral edema secondary to a head injury in order to : Choose one answer. f. prevent neurogenic pulmonary edema g. prevent increased capillary permeability h. prevent metabolic acidosis i. allow reciprocally high levels of PaO2 in the brain j. prevent cerebral vasodilation 39 Causes of third space loss include all except: Choose one answer. f. Acute pancreatitis g. Pancreatic fistula h. Necrotizing fasciitis i. Site of major surgery j. Crush syndrome 40 Causes of metabolic acidosis include all except: Choose one answer. f. Small bowel fistula
589
g. Shock h. CO poisoning i. severe anemia j. All of the above 41 A 39-year-old man comes in for evaluation of intermittent anal pain and bleeding after bowel movements. He also has hard stools. He has had these symptoms for over 1 year . For this patient, which of the following is included in the classic triad of signs of chronic anal fissures? Choose one answer. e. An anal fissure f. All of the above g. A sentinel skin tag h. Hypertrophy of the anal papilla 42 All the following statements about achalasia are true EXCEPT: Choose one answer. f. it occurs most commonly in persons between the ages of 30 and 50 years g. in most affected persons, ganglion cells in the body of the esophagus either are absent or have degenerated h. esophageal cancer is seven times as common in affected persons as in the general population i. pressure in the body of the esophagus is lower than normal j. affected persons usually experience more difficulty swallowing cold foods than warm foods 43 A patient with the Zollinger-Ellison syndrome is found to have the multiple endocrine neoplasia type I (MEN-I) syndrome. Appropriate management for the ulcer symptoms should be: Choose one answer. f. Omeprazole g. pancreatic resection h. streptozocin i. cimetidine j. total gastrectomy 44 Fat absorption occurs primarily in the: Choose one answer. f. Ileum g. third portion of the duodenum h. Stomach i. Jejunum j. first portion of the duodenum 45 39. A 56-year-old woman has been experiencing abdominal pain for 4 hours. The pain is in right upper quadrant and radiates into the scapular region. She has had multiple episodes of vomiting. For this patient, which of the following signs on physical examination is associated with acute cholecystitis? Choose one answer. i. Carnett sign j. Kehr sign k. Murphy sign l. Rovsing sign 46 Mammary duct ectasia is characterized by the following EXCEPT : Choose one answer. f. Is treated usually by simple mastectomy g. May present with nipple retraction and Peau d'orange picture
590
h. Anaerobic superinfection cmmmonly occurs in this recurrent periductal plasma cell mastitis i. Is defined as primary dilatation of major ducts of breast in middle aged women j. Iscommonly pre-malignant 47 Regarding veins of lower limbs all are true except: Choose one answer. i. Valves allow flow from deep to superficial system Venous return from lower limbs is aided by respiratory movements j. Superficial veins lie in subcutaneous tissue k. The pressure in veins of the foot while standing is 100mmHg l. a) Stasis of blood is important factor in developing varicose veins 48 Most common complication of central venous access is: Choose one answer. k. major artery damage. l. Catheter problems. m. Thrombosis of central vein n. Catheter related sepsis o. Pleural space damage, pneumothorax 49 Factors associated with increase risk of death in acute pancreatitis include all except: Choose one answer. f. Ranson score more than five g. Obesity h. High APACHE_II score i. Age more than 70 years j. sterile necrosis 50 For the patient in Question 56, which of the following chronic conditions can cause a neck mass? Choose one answer. e. Tuberculosis f. Sarcoidosis g. AIDS h. All of the above 51 For the patient in Question 56, if metastatic cancer is suspected initially, which of the following would be the most appropriate step to take next in the workup? Choose one answer. e. CT scan f. Empirical therapy with antibiotics g. Fine-needle aspiration (FNA) h. Observation only 52 The largest component of intestinal gas is: Choose one answer. f. Nitrogen g. carbon dioxide h. Oxygen i. ammonia j. hydrogen 53 A 66-year-old woman presents to the office complaining of a sharp, constant pain in her lower abdomen. She has had this pain for the past 2 weeks. Examination of her abdomen is normal. However, subsequent ultrasound reveals a 4.5 cm Aortic abdominal aneurysm (AAA). Spiral computed tomography confirms the ultrasound findings.
591
Aside from rupture, which of the following is a complication associated with (AAA) ? Choose one answer. e. Aortoenteric fistula f. Lower-extremity atheroemboli g. All of the above h. Thrombosis 54 Diagnosis of esophageal perforation is best established by: Choose one answer. f. transesophageal ultrasound study g. esophagoscopy with a flexible esophagoscope h. contrast esophagograms i. esophagoscopy with a rigid esophagoscope j. upright X-rays of the chest including lateral and oblique films 55 For the patient in Question 68, which of the following is an indication for surgery? Choose one answer. e. Ongoing hemorrhage occurs from a gastric ulcer in a hemodynamically unstable patient f. All of the above g. Bleeding continues from either a duodenal ulcer or a gastric ulcer despite medical and endoscopic therapy h. Substantial bleeding occurs from a duodenal ulcer that is not controlled by EGD 56 Yesterday, a 38-year-old woman underwent a laparoscopic cholecystectomy for cholelithiasis and was discharged home 8 hours after surgery. She returns this morning complaining of worsening abdominal pain. The oral narcotics that the patient was prescribed are ineffective in controlling the pain. The patient's temperature is (38.3 C). Laboratory studies reveal an elevated white blood cell count. Abdominal ultrasonography shows a large subhepatic fluid collection. The fluid is percutaneously aspirated and reveals enteric contents. What step should be taken next in the management of this patient? Choose one answer. e. Immediate laparotomy f. I.V. antibiotics and close observation g. None of the above h. Observation in the hospital until pain improves 57 The most common symptom after major pulmonary embolism is: Choose one answer. f. cough g. Dyspnea h. Hemoptysis i. pleural pain j. fear of death 58 Treatment of paralytic ileus includes all of the following EXCEPT : Choose one answer. f. intravenous fluids g. cessation of oral intake h. correction of electrolyte imbalance
592
i. early operation j. nasogastric suction 59 Lymphedema : Choose one answer f. e) None of the above g. b) should be bilateral h. c) may be pitting in early stage i. d) A & C only j. a) may be Congenital 60 The level of consciousness for a head injury patient is BEST evaluated by : Choose one answer. f. CT scan g. visual evoked potentials h. Glasgow coma scale i. papillary responses j. response to pain 61 A 52-year-old male weighing 70 kg, sustained a 65% total body surface area (TBSA) burn. What are his fluid requirements? Choose one answer. a. 18,200 cc in 16 h b. 9100 cc in the first 8 h c. 8000 cc in 24 h d. 12,800 cc in 24 h 62 Management of cholangitis may include all of the following EXCEPT: Choose one answer. a. decomperession of the common bile duct. b. cholecystostomy c. IV antibiotics. d. correct underlying cause. e. percutaneous transhepatic cholangiography. 63 A 48-year-old woman presents to the emergency department complaining of right upper quadrant pain, which began 4 hours ago. She reports the pain as being spasmodic and sharp and that it radiates to her right shoulder blade. She says that she has had similar episodes over the past few months, especially after eating large meals. Associated with the pain is nausea and vomiting. Her blood pressure is 120/85 mm Hg, and her pulse is 100 beats/min. On physical examination, the patient is found to have a nontender abdomen with no palpable masses. Her chest and cardiovascular examinations are normal. The nurse notices that her sclerae are slightly icteric. On subsequent laboratory studies, her serum bilirubin level is found to be 10 mg/dl. What imaging study should be performed next for this patient with presumed posthepatic jaundice? Choose one answer. a. Percutaneous transhepatic cholangiography (PTC) b. Ultrasonography c. Endoscopic retrograde cholangiopancreatography (ERCP) d. Magnetic resonance imaging
593
64 39. An HIV-positive man presents for evaluation of new oral cavity lesions he discovered last month. Physical examination reveals purple exophytic masses involving the palate mucosa and gingiva. What is the most likely diagnosis of these oral cavity lesions? Choose one answer. a. Oral hairy leukoplakia b. Non-Hodgkin lymphoma c. Syphilis d. Kaposi sarcoma 65 Brain injury alone Choose one answer. a. causes shock only if the skull is intact b. rarely causes shock c. causes shock that is reversed by very simple measures d. frequently causes shock e. causes shock if hypoxia is superimposed 66 All of the following substances are irritating to the peritoneum EXCEPT: Choose one answer. a. bile. b. blood c. gastric content. d. meconium e. pus 67 Regarding volvulus of the sigmoid colon, each of the following is true except : Choose one answer. a. likely results from redundant sigmoid colon with an elongated narrow mesocolon b. there appears to be a congenital predisposition sigmoid volvulus. c. diagnostic barium enema for sigmoid colon is essential d. diagnostic x-ray for sigmoid volvulus shows a dilated loop of colon which points toward the right upper quadrant. 68 Gallstones are characterized by all the following EXCEPT: Choose one answer. a. cause mucocoele of the gall bladder b. are present in the common bile duct in 40% of patients with stones in the gall bladder c. are becoming common in post-partum primipara who were pre pregnancy ‘Pill’ takers d. are frequently the cause of flatulent dyspepsia e. may be present in the newborn 69 Following a burn, the agent responsible for early increased capillary permeability is Choose one answer. a. thromboxane A2 b. serotonin c. histamine d. prostacyclin PGI2 e. bradykinin 70 Incisonal hernias all are true except: Choose one answer. a. 10 % of all hernias b. Usually easy to reduce c. Multiloculated sac d. Operative technique is most important cause e. Complication is common
594
71 In the treatment of gastric cancer, all of the following are true EXCEPT: Choose one answer. a. five-year survival rates in the continue to be between 10% and 25%. b. Palliative resection is frequently helpful with advanced disease. c. Lymph node involvement is associated with a poorer prognosis. d. Finding early disease at the time of operation is associated with a better prognosis. e. total gastrectomy is mandated in most patient. *** 72 of the involved area and : Severe cases of hidradenitis suppurativa in the groin area are best managed by excision Choose one answer. a. transfer of a rectus abdominus muscle flap b. split thickness skin grafting c. primary closure d. delayed primary closure e. closure by secondary intention 73 Patients at increased risk for gastric carcinoma include all the following EXCEPT: Choose one answer. a. those who have undergone gastric bypass for morbid obesity b. those who have undergone gastric resection for duodenal ulcer c. those with pernicious anemia d. those with a high consumption of smoked fish e. those with blood group A 74 A 56-year-old woman presents with symptoms of abdominal pain, weight loss, and rectal bleeding. She is anemic and hypotensive, but she is stable. For this patient, which of the following should be done first if the gastric lavage yields copious amounts of bile? Choose one answer. a. Arteriography b. Emergency laparotomy c. Colonoscopy d. Esophagogastroduodenoscopy 75 A 39-year-old man presents with lower GI bleeding. He has no abdominal discomfort and has experienced no loss of weight. For this patient, which of the following statements regarding the etiology of lower GI bleeding is true? Choose one answer. a. Lower GI bleeding from diverticulosis often requires surgery b. Arteriovenous malformations (AVMs) are the most common cause of lower GI bleeding c. 6% to 10% of patients with ulcerative colitis have lower GI bleeding severe enough to necessitate emergency surgical resection d. None of the above 76 For the patient in Question 65, which of the following statements regarding abdominal radiographic findings is true? Choose one answer. a. In gastric outlet obstruction, no gastric air will be seen, but large amounts of air will be seen in the small bowel and colon b. Mechanical small bowel obstruction usually shows no air-fluid levels, nor will distended bowel loops of similar sizes be seen c. All of the above
595
d. High-grade obstruction of the colon in association with an incompetent ileocecal valve may mimic small bowel obstruction on x-ray 77 A 41-year-old female presents to the emergency department after sustaining a gunshot wound to the abdomen, with injuries to the liver and large bowel. Despite successful resuscitation and operative intervention, the patient dies 2 weeks later of multisystem organ failure in the intensive care unit. Which organ most likely first experienced dysfunction? Choose one answer. a. lung b. heart c. liver d. gastrointestinal tract e. kidney 78 39. A 35-year-old man presents with severe acute abdominal pain of sudden onset. The pain initially began in the upper abdomen and has now settled in the whole abdomen. On examination, the abdomen was rigid. Chest X-Ray showed free gas under diaphragm. Which of the following is the most appropriate course of management for this patient? Choose one answer. a. Observation b. Urgent laparotomy c. Nonurgent laparotomy d. Laparoscopy 79 All are true about the dumping syndrome except: Choose one answer. a. Symptoms can be controlled with a somatostatin analogue. b. Early postoperative dumping after vagotomy often resolves spontaneously. c. Flushing and tachycardia are common features of the syndrome. d. Diarrhea is always part of the dumping syndrome. e. Separating solids and liquids in the patient's oral intake alleviates some of the symptoms of the syndrome. 80 An anal fissure is characterized by the following EXCEPT: Choose one answer. a. Is more common in women. b. Is an ulcer of the anal mucosa. c. Can be treated coservatively. d. Usually lies anteriorly. 81 All of the following statements are true of esophageal carcinoma EXCEPT Choose one answer. a. squamous cell tumor is the most frequent histology. b. operation is frequently curative. c. patient often require a pyloroplasty with operation. d. patients usually die within one year of diagnosis e. patient can have intestinal continuity reestablished using the stomach after esophageal resection. 82 A serum tumor marker correlated with recurrence after management of colon cancer is: Choose one answer. a. carcinoembryonic antigen (CEA) b. Calcitonin c. 5-hydroxyindoleacetic acid
596
d. CA 15-3 e. Alpha-fetoprotein 83 Conditions associated with gastric cancer include all of the following EXCEPT: Choose one answer. a. adenomatous polyps. b. a high intake of dietary nitrates. c. pernicious anemia. d. higher socioeconomic groups. e. chronic atrophic gastritis. 84 Which of the following is required for addressing any pressure sore? Choose one answer. a. pressure reduction b. hyperbaric oxygen c. antibiotics d. skin flap coverage 85 The most effective treatment of achalasia is: Choose one answer. a. antireflux surgical procedures b. Esophagomyotomy c. dilation of the lower esophageal sphincter d. antispasmodic medication e. resection of the cardioesophageal junction 86 Regarding desmoid tumors all are true except: Choose one answer. a. Treatment is wide local excision with safety margins b. Has high rate of local recurrence c. Is a low grade fibro sarcoma d. More common in females e. Is radio resistant 87 Which of the following variables best predicts prognosis for patients with a recent diagnosis of cutaneous melanoma and no clinical evidence of metastatic disease? Choose one answer. a. Ulceration. b. Clark's level. c. Gender d. Bleeding e. Breslow thickness. 88 In the treatment of acute cholecystitis, most patient are BEST served with: Choose one answer. a. endoscopic sphincterotomy. b. cholecystostomy. c. percutaneous drainage of the gallbladder. d. early cholecystectomy (within 3 days of onset of symptoms). e. IV antibiotics and cholecystectomy in 6 to 8 weeks. 89 Regarding crohn's disease all are true except Choose one answer. a. Cause non caseating granuloma b. commonly affect small bowel c. Causes sub mucosal fibrosis
597
d. It is full thickness inflammation e. Cause depletion of goblet cell mucin 90 Advantages of laparoscopic versus open cholecystectomy include all of the following EXCEPT: Choose one answer. a. decreased pain. b. less risk of bile duct injury. c. reduced hospitalization. d. improved cosmetic. e. reduced ileus. 91 The most common cause of massive hemorrhage in the lower gastrointestinal tract is: Choose one answer. a. carcinoma b. Diverticulosis c. Diverticulitis d. Polyp e. ulcerative colitis 92 Carcinoma of the gallbladder is Choose one answer. a. rarely associated with jaundice. b. associated with a good prognosis. c. usually not diagnosed preoperatively. d. most commonly metastatic to the lung. e. best treated with radiation and chemotherapy 93 All the following statements concerning nipple discharges are true EXCEPT: Choose one answer. a. benign duct papillomas are the most common cause of bloody discharges. b. when bloody , the discharge is due to a malignancy 70% of the time c. a) excision of involved duct may be necessary to determine the etiology d. they may be caused by multiple lesions. e. a milky discharge may be due to a pituitary adenoma . 94 39. A 28-year-old woman presents with a complaint of a growing, painless mass in her neck. Physical examination reveals a firm, fixed nodule measuring 2 cm on the right lobe of her thyroid. The surgeon recommends fine-needle aspiration (FNA) of the lesion instead of excisional biopsy. What are the advantages of FNA over excisional biopsy? Choose one answer. a. FNA is associated with a decreased risk of tumor seeding b. FNA requires only an office visit c. Anesthesia is not necessary d. All of the above 95 For the patient in Question 74, which of the following is a sign of a colovesical fistula associated with diverticulitis, as seen on computed tomography with contrast? Choose one answer. a. Thickening of the bladder and the colon b. Sigmoid diverticula c. All of the above d. Air in the bladder
598
96 All of the following are true statements concerning paget ’ s disease of the nipple EXCEPT: Choose one answer. a. it is an eczematoid lesion. b. it is very uncommon, accounting for only 2% of all breast cancers. c. it is an in situ squamous cell malignancy of the nipple. d. it can be confused with malignant melanoma histologically e. it has a better prognosis than the majority of other breast cancers. 97 The classical picture of Acute arterial embolism include all the following except: Choose one answer. a. peripheral pulses b. Parasthesia c. All the above d. Pain e. Pallor 98 Extra colonic manifestation of inflammatory bowel disease include all except: Choose one answer. a. Erythema nodosum b. Mixed connective tissue disease c. Primary Sclerosing cholangitis d. Polyarthopathy e. Episcleritis 99 A 45-year-old woman presents with abdominal pain and vomiting of 1 day's duration. The patient underwent an exploratory laparotomy after a motor vehicle accident 8 years ago. Which of the following statements is true for this patient? Choose one answer. a. Pain from mechanical obstruction is usually more severe than pain from ileus b. The severity of pain from mechanical obstruction may decrease over time c. Pain from mechanical obstruction is usually localized in the middle of the abdomen, whereas pain from ileus or pseudo-obstruction is diffuse d. All of the above 100 A 4-year-old girl has been experiencing dyspnea on exertion. Chest x-ray shows a Morgagni hernia. For this patient, which of the following statements is true? Choose one answer. a. Morgagni hernias can be repaired with a subcostal, a paramedian, or a midline incision b. Morgagni hernias are most commonly seen on the right side c. All of the above d. The average age at diagnosis is typically greater for patients with Morgagni hernia than for patients with Bochdalek hernia Stomach , Duodenum and Small intestine 1. Bengin gastric ulcers: A. Occur in the same age group as duodenal ulcers. B. Are more common in females than males. C. Are more common in the upper social classes. D. Produce epigastric pain after eating food. E. All of the above is correct.
599
2. Uncomplicated benign gastric ulcers: (All correct except one) A. Occure most commonly on the greater curve of the stomach. B. Should initially be treated medically. C. Commonly recur after medical treatment. D. Should receive surgical treatment if healing has not occurred after 4 to 6 weeks of medical. E. The surgical treatment of gastric ulceris billoroth one partial gastrectomy. 3. There is an association of gastric cancer with: (All correct except one) A. Achlorhydria of the stomach. B. Atrophic gastritis. C. Adenomatous gastric polyps. D. Duodenal ulceration. E. Chronic non healing gastric ulcer. 4. Surgical treatment of gastric cancer: A. Should preferably be by a total gastrectomy. B. Gives overall 5 years survival figures over 40 per cent. C. Includes excision of the tumour as a palliative measure. D. Should be advised for all diagnosed cases. E. The resectability rate is over 70per cent. Gastric adenocarcinoma is preferably treated by resection, although resection usually proves to be palliative 5. The signs and symptoms of potentially curable gastric cancer: (All correct except one) A. May simulate those of a benign gastric ulcer. B. Include a hard palpable left supraclavicular node. C. Requere investigation by a barium meal examination. D. Requjire investigation by a gastroscopic examination even when the barium meal is normal. E. Include a non palpable abdominal mass. 6. Gastric cancer: A. Is most common in the fundus of the stomach. B. Is most commonly a squamous cell carcinoma. C. Frequently metastasises via the blood stram. D. Is most frequently an ulcerating lesion. E. Non of the above is correct. 7. Treatment of symptomatic duodenal ulcers: A. Should include twice daily antacids. B. Shjould include a strict dietary regime . C. Should include anti-cholinergic drugs. D. Should be by early surgery. E. Non of the above is correct. 8. Duodenal ulcers: A. Have an equal incidence in both sexes. B. Have a clicical course characterised by long periods of remission. C. Are characterised by postprandial pain. D. Occur most commonly ion the duodenal cap. E. B&D only. 9. Acute haemorrhage from a duodenal ulcer: A. Requires no urgent diagnosis of the source of haemorrhage. B. Requires an urgent gastroscopy. C. Should be treated surgically as soon as the diagnosis is made.
600
D. Indicates the presence of a chronic duodenal ulcer. E. C&D only. 10. Post-gastrectomy nutritional disturbances may result in: A. Megaloblastic anaemia. B. Steatorrhoea. C. Iron deficiency anaeimia. D. Osteoporosis. E. All of the above. 11. The incidence of crohn’s disease: A. Is not distributed evenly throughout the world’s population. B. Is similar in the two sexes. C. Is highest amongst young adults. D. Is increased in the close relatives of patients with the disease. E. All the above are correct. 12. Crohn’s disease: A. Has an infective aetiology. B. Is limited to the bowel mucosa. C. Does not produce mucosal ulceration. D. Is characterised by the absence of fibrous tissue in the affected inflamed bowel. E. Spreads through the hole thickness of the bowel wall. 13. Patients with crohn’s disease characteristically present with: A. Colicky abdominal pain. B. Constipation. C. Nutritional deficiencies. D. Rectal bleeding. E. A&C only. 14. In the treatment of patients with crohn’s desease: A. Medical methods have no parts to play. B. Surgery should be the primary method of treatment. C. Steroids may provide a remission in the progress of the disease. D. A high bulk diet should be considered. E. All of the above are correct. 15. Tumours of the small bowel: A. Are common. B. May be an inherited disorder. C. Most commonly present with overt or occult rectal bleeding. D. Most commonly present in childhood. E. C&D only. 16. Carcinoid tumours of the gastrointestinal tract: A. Are most commonly located in the appendix . B. Are usually malignant. C. Arise in the mucosa. D. Are frequently the cause of gastrointestinal haemorrhage. E. Mucosal ulceration is common. 17. The carcinoid syndrome:(all correct except one) A. Characteristically includes abdominal cramps. B. Is produced by the release of vasodilating substances from the tumour. C. Occurs most commonly with metastatic carcinoid tumours. D. Is usually cured by excision of the primary tumour.
601
E. May produce diarrhoea and dyspnoea. 18. Chronic radiation injury to the intestinal tract: (all correct except one) A. Typically presents with mucosal atrophy. B. Often presents with perforation of the bowel. C. Frequently presents with intestinal obstruction. D. May present with malabsorption. E. Often produce progressive fibrosis. 19. The “blind-loop” syndrome often: A. Presents with signs of malnutrition. B. Presents as a complication of crohn’s disease. C. Presents as a complication of multiple deverticuli. D. Presents as a complication of intestinal bypass. E. All of the above are correct. 20. A meckel’s deverticulum of the small intestine: (all correct except one) A. Is situated at the jejuno-ileal junction. B. Contains all coats of intestinal wall. C. May be associated with a fibrous band connecting it to the umbilicus. D. Most commonly presents as lower GI bleeding diverticulitis. بسم الله الرحمن الرحيم General Surgery – Final Exam 6th year (2013) (1-29) QUESTIONS FROM http://www.surgical-tutor.org.uk/default-home.htm? MCQ/mcq5.htm~right 1) Peutz Jeghers Syndrome .. True EXCEPT a. Is an autosomal recessive condition b. Often presents with anaemia in childhood c. Is characterised by circumoral mucocutaneous pigmented lesions e. Malignant change occurs in 2-3% of polyps 2) During surgery on the submandibular gland a. An incision on the lower border of the mandible is safe b. The submandibular gland is seen to wrap around the posterior border of mylohyoid c. The facial artery and vein are divided as they course through the deep part of the gland d. The hypoglossal nerve is seen to loop under the submandibular duct e. Damage to the lingual nerve will cause loss of sensation to the posterior third of the tongue 3) Regarding pancreatic carcinoma … True EXCEPT a. 90% are ductal adenocarcinomas b. Less than 20% occur in the head of the gland c. The usual presentation is with pain, weight loss and obstructive jaundice d. Ultrasound has a sensitivity of 80-90% in the detection of the tumour
602
e. Less than 20% of patients are suitable for curative surgery 4) Regarding the management of major trauma a. Deaths follow a trimodal distribution b. X-rays after the primary survey should be of AP Cervical spine, chest and pelvis c. Cardiac tamponade is characterised by raised BP, low JVP and muffled heart sounds d. Assessment of uncomplicated limb fractures should occur during the primary survey e. Deterioration of the casualty during the primary survey should lead to the secondary survey 5) Regarding appendicitis … True except a. The risk of developing the illness is greatest in childhood b. Mortality increases with age and is greatest in the elderly c. 20% of appendices are extraperitoneal in a retrocaecal position d. Faecoliths are present in 75-80% of resected specimens e. Appendicitis is not a possible diagnosis in the absence of abdominal tenderness 6) Stones in the common bile duct … True Except a. Are found in 30% of patients undergoing cholecystectomy (Without pre-op ERCP) b. Can present with Charcot's Triad c. Are suggested by an bile duct diameter >8mm on ultrasound d. ERCP, sphincterotomy and balloon clearance is now the treatment of choice e. If removed by exploration of the common bile duct the T-tube can be removed after 3 days 7) Solitary thyroid nodules … True Except a. Are more prevalent in women b. In the adult population less than 10% are malignant c. More than 50% of scintigraphically cold nodules are malignant d. The risk of a hot module being malignant is negligible 8) Regarding abdominal wall hernias b. 20% of inguinal hernias are indirect c. In women femoral hernia is more common d. The mortality assocaited with strangulation is over 10% e. The mortality has reduced dramatically over the past 30 years 9) Intermittent claudication … True except a. Affects less than 1% of men over the age of 50 years b. At 5 years 10% of claudicants will have progressed to an amputation c. At 5 years 20% of claudicants will have died from ischaemic heart disease
603
d. Is usually associated with an ankle / brachial pressure index (ABPI) < 0.7 e. Is associated with a fall in the ABPI on exercise with delayed recovery 10) The pathology of crohns … True Except a. Shows full thickness inflammation b. The rectum would be involved c. patients have terminal ileal disease d. Enterocutaneous or intestinal fistulae are common e. The serosa is usually normal 11) Regarding benign breast disease… True EXCEPT a. Cyclical mastalgia is the commonest reason for referral to the breast clinic b. Fibroadenomas are derived from the breast lobule c. Lactational breast abscesses are usually due to Staph aureus d. Duct ectasia is more common in non-smokers e. Atypical lobular hyperplasia is associated with an increased risk of breast cancer 12) Regarding anal fissures … True Except a. 10% occur in the posterior midline b. Multiple fissures suggest a diagnosis of tuberculosis or Crohn's Disease c. 50% of acute fissures heal with the use of a bulking agent d. Sphincterotomy has a success rate of over 90% e. Sphincterotomy is associated with minor faecal incontinence in over 15% of patients 13) Regarding rectal cancer a. It often arises in metaplastic polyps b. The rectum is the least site of colorectal tumours c. A cancer in the upper 1/3 of the rectum is appropriately managed by AP resection d. Local recurrence of rectal cancer can be reduced a Total Mesorectal Excision e. Chemotherapy is of proven benefit in Duke's A tumours 14) Regarding malignant melanoma a. The incidence is halving every 10 years b. Superficial spreading melanoma is the commonest clinico-pathological type c. Tumour thickness is the most important prognostic indicator d. Regional lymphadenectomy provides survival benefit in patients with 'thick' tumours e. Acral lentiginous tumours are usually found on the trunk and proximal limbs 15) Regarding oesophageal cancer … True Except a. Squamous carcinomas predominate in the upper 2/3 of the oesophagus b. Overall 5 year survival is greater than 50% c. Tylosis genetically predisposes to the disease
604
d. 15% of adenocarcinomas are associated with Barrett's oesophagus e. For palliation an Atkinson tube can be inserted endoscopically 16) Infantile hypertrophic pyloric stenosis b. Sons of affected mothers have a 20% risk of developing the lesion c. Invariably presents between six and eights months of age d. Typically presents with bile stained projectile vomiting e. Surgical treatment is by Heller's Cardiomyotomy 17) The functions of a tracheostomy are to … True Except a. Bypass an upper airway obstruction b. Increase the anatomical dead space c. decrease airway resistance d. Protect against aspiration e. Allow frequent airway suction 18) Medullary carcinoma of the thyroid a. Is a tumour of the parafollicular C cells c. 10% of cases are sporadic d. Can occur as part of the MEN type II syndrome e. Total thyroidectomy is the surgical treatment of choice 19) Regarding colorectal carcinomas a. About 40% present as surgical emergencies b. Right-sided tumours often present with a change in bowel habit d. Over 20% of patients present with more than one primary tumour e. A Duke's B tumour has nodal metastases 20) Regarding colonic polyps a. Metaplastic polyps are pre-malignant b. Adenomatous polyps are pre-malignant c. Villous adenomas are more common than tubular adenomas d. Genetic mutations can result in epithelial metaplasia 21) Regarding acute limb ischaemia b. Occurs more often in patients in sinus rhythm c. Objective sensory loss requires urgent treatment d. Fixed skin staining is an early sign e. Intra-arterial thrombolysis is invariably successful with no morbidity 22) Hepatocellular carcinoma … True Except a. Has a high incidence in East Africa and South-east Asia
605
b. Its worldwide incidence parallels the prevalence of Hepatitis B c. Mycotoxins (e.g. aflatoxin) are an important aetiological factor d. Over 80% of tumours are surgically resectable e. Liver transplantation offers the only chance of cure in those with irresectable disease 23) Serum alpha fetoprotein … True Except a. Is increased in testicular tumours b. In testicular tumours is produced by trophoblastic elements c. Is increased in over 70% patients with hepatocellular carcinoma d. In patients with hepatocellular carcinoma levels correlate well with size of the tumour 24) Intussusception … True EXCEPT a. Is most common in children from 6 to 12 years b. Presents with colicky abdominal pain, rectal bleeding and an abdominal mass c. 10% present with diarrhoea and vomiting suggestive of gastroenteritis d. If no shock or peritonitis hydrostatic reduction can be attempted e. A Meckel's diverticulum can induce an intussusceptions 25) Hirschsprung's disease a. Non of the above b. 10% cases have involvement of the recto-sigmoid segment c. 10% cases present with delayed passage of meconium in the first 24 hours after birth d. The affected segment of bowel appears dilated on barium enema e. On rectal biopsy there increased acetylcholinesterase containing cells in the muscularis 26) Most common segment of colon affected in Hirschsprung’s disease is: a- Rectum b- Cecum c-Ascending colon d- Descending colon e- Transverse colon 27) Hirschsprung's disease a- On rectal biopsy there decreased acetylcholinesterase containing cells in the muscularis b- loss of Auerbach's plexus and Meissner's plexus c- loss of Auerbach's plexus only 28) In obstructive jaundice … True Except a. Urinary conjugated bilirubin is increased
606
c. Urinary urobilinogen is reduced d. Serum conjugated bilirubin is reduced e. Faecal stercobilinogen is reduced 29) Papillary carcinoma of the thyroid… True Except a. Can be reliably diagnosed using fine needle aspiration cytology b. Is almost always unifocal c. Histologically displays Psammoma bodies d. Typically spread to the cervical lymph nodes e. Requires a total thyroidectomy for large tumours 30) Regarding small bowel obstruction a. In the United Kingdom is most often due to an obstructed hernia b. Causes colicky abdominal pain and vomiting c. Abdominal distension is seen in all patients d. All cases can be managed conservatively for the first 24 hours e. Rarely requires aggressive fluid resuscitation 31) Regarding gas gangrene a. It is due to Clostridium botulinum infection b. Clostridial species are gram-negative spore forming anaerobes c. The clinical features are due to the release of protein endotoxin d. Gas is invariably present in the muscle compartments e. Surgical debridement and antibiotics are an essential part of treatment 32) Regarding varicose veins b. Over 20% are recurrent varicosities c. The sural nerve is in danger during stripping of the long saphenous vein d. The saphenous nerve is closely associated with the short saphenous vein e. 5% oily phenol is an appropriate sclerosant for venous sclerotherapy 33) according to ZES … True Except a- Liver Metastasis b- hyperacidemia c- intractable peptic ulcer d-
34) Stensen's duct opens to: second upper molar 35) Compartment syndrome
607
Choose one answer. a. Passive stretch decrease muscle pain b. Due to decrease pressure in muscle compartments c.
Pulse is the first thing to disappear
d. Cause severe pain in the limb e. Hyperemia below knee
36) all causing central venous HTN and systemic blood Hypotension and respiratory failure except a- CHF B- TENSION PNEMUTHORAX C- Hemorrhage 37) Widening of the Mediastinum after a decelerating accident typically with : Aortic Rupture 38) A young male patient who had a car accident, was hemodynamicaly unstable, was transferred to the Trauma center, in the ambulance he developed respiratory failure .. a- Tension pnemuthorax b- pnemuthorax secondary to fractured rip 39) Daily requirement of Potassium : 40) complication of gastrectomy, all ex: a- Anemia b- osteoporosis c- weight loss d- all of the above 41) most common site of cholangiocarcinoma : Bifurcation 42) most common site of gastric Ulcer: a- Antrum b- incisura c- great curvature 43) A pregnant lady with uncomplicated Appendecitis , all are accepted Except a- Lap appendectomy depending on the experience of the surgeon and size of the uterous b- antibiotic and variant appendectomy
608
c- Ultrasonography d- preoperative fetal monitoring e- pnemucompressive.. 44) Carcinoid Tumor : if less than 1.5 cm appendectomy is enough 45) Causes of post-op Jaundice a- Massive Blood transfusion b- Hematoma c- Sepsis d- Halothane e- Reperfusion of tissues 46) Indication for TPN a- Severe pancreatitis b- extensive burn c- short bowel syndrome d- enterocutaneous fistula e- ill patient who cant swallow 47) complication of Obesity in operations a- difficult intubation b- DVT and PE c- power wound healing/infection d- risk for cerebrovascular accident 48) Site for TPN : Subclavian, CVC 49) all risk factor for Stomach CA EXCEPT : Low nitrite 50) all risk factors for colon cancer EXCEPT: high fiber diet 51) GCS for a patient who open his eyes and withdraw to pain and moaning but no comprehensive sound : 8 ??
52) Regarding hepatic encephalopathy, the condition is exacerbated by all of the following except: A. Bleeding varices. B. Diazepam. C. Infections.
609
D. Protein diet. E. Neomycin. 53) Amebic abscess .. True except : treatment by drainage 54) amylase : a- half life 10 hrs b- rise after few hours c- high specificity when > 3x normal d- none of the above e- all of the above 55) breast mas 7 cm, mobile LN ipsilateral, no mets T3N1M0 56) DCIS < 0.5 CM … TREATMENT: Lumpectomy +- radiation 57) breast mass +ve osterogen receptor , HER2 –VE, 4 Lymph nodes : Chemotherapy + tamoxifen 58) female with thyrotoxicosis , failed medical treatment … next step : Radiotherapy 59) most common fistula : Intersphernic fistula 60) Nursing lady C/O breast tenderness.. best management is : a- trial of Antibotic and analgesia b- Observation c- Needle Aspiration d- Stop lactation 6th year Surgery final exam 2012 The most important two topics are colorectal-CA & Breast – CA , on every exam you will be given cases to determine the stage & management, so study them well from a good resource (Recall is not enough!). 1) Average Daily K requirement: - 120 meq\d - 1 meq\kg\d 2) hyperkalemia ECG changes: a. wide QRS b. Peaked T wave c. ST-depression d. all of the above 3) Inguinal hernia wound is considered : - clean 4) PNS, all true ex: premalignant condition 5) hypovolemic shock is characterized with: low central venous pressure, high cardiac output, high tissue resistance
610
6) psuedomembranous colitis caused by: C. difficile 7) the gas in the abdomen seen on X-ray is: nitrogen methan oxygen 8) case 9mo old boy, had an acute inguinal swelling that was reducible, what’s the most likely diagnosis: -irreducible inguinal hernia 9) all true re undescended testicles ex.: increased spermatogenesis 10) obstructive jaundice, all ex.: dark stool 11) all are D.Dx of acute pancreatitis ex. : -haiatal hernia - acute appendicitis 12) tumor marker of pancreatic –CA: CA-9-19 13) all are part of Ranson’s criteria (on admission) ex.: base deficit >4 14) re etiology of acute pancreatitis in our region: most common cause is cholethiasis 15) Anatomy of CBD, all true ex: a. it lies anterior to portal vein b. it lies anterior to 1st duodenum c. it may open independently in the duodenum (mish m3 elpancreatic duct) 16) re Abdominal Aortic Aneurysm all true ex.: Most common Cause is inflammatory 17) Tx of rapinychia: lateral incision & drainange (jawab el-majority ) direct incision & drinaige 18) re Lower GI bleeding, all true ex.: m.c.c of massive LGIB is grade 4 hemorrhoids 19) diagnostic method to localize small, intermittent LGIB: radiolabeled RBC 20) when endoscopy fails to localize massive LGIB, the best method is: mesenteric angiography 21) long term complication of gastrectomy, all ex: a. B12 deficiency b. iron dif. Anemia c. osteomalacia d. all of the above 22) indications of thoracotomy :
611
a. bleeding 200ml\hr via a central line \chest tube b. ruptured thoracic esophagus c. tamponade d. uncontrollable air leakage from the lung e. all of the above 23) with a change of 1 gram of serum albumin the Ca level: .8 alteration 24) case an elderly man on chronic NSAIDs for osteoarthritis, presents with epigastric discomfort, endoscopy is most likely to show: a. acute gastritis b. H. pylori c. chronic … hypertrophy (I can’t remember the option ,,, bs some students answered this) 25) photo of a fistula, what’s the type: inter-sphincteric 26) 1st trimester pregnant lady, presents with a clinical picture suggestive of Appendicitis, ur management: - IVF, ATB, observe for 12-24 hrs, if no improvement do appiendectomy - IVF, ATBs, urgent appi - IVF, ATBs, schedule elective appi 4-6 wks later 27) re Crohn’s Dz all true ex: serosa is usually normal 28) re Ulcerative colitis (UC), which is true: a. almost always involves rectum b. 10% terminal ileum involvement 29) all in MEN-1 ex: thyroid –CA 30) xray of double bubble sign, seen in: Duodenal atresia 31) case of stable perforated duodenal ulcer, the surgeon will: a. graham patch b. highly selective vagotomy … (o Kaman similar choices :D) 32) to control active bleeding of an artery in the ER: a. direct compression b. apply a clamp c. electro-cauterization 33) re UC, more severe Dz is with: a. smoking b. altered immunity (or something related to immune) c. 34) re U-GI-Bleeding, all true ex: 80% are due to variceal bleeding 35) surgical intervention in UGIB is indicated if: a. adult needed > 3 units \ day b. shock c. …. 36) best indicator of prognosis in cutaneous melanoma: berslow thickness 37) clear margins needed in a .5 cm melanaoma: a. 1 cm b. 2 cm c. 5cm
612
38) case of a 28 yr old male with RIF pain, nausea, vomiting, Abd x-ray shows dilated right colon, most likely Dx: a. cecal volvulus b. can’t tell yet, needs further investigations 39) the most important clue to acute appendicitis: a. Right iliac fossa pain & tenderness b. rovsing’s sign c. inc. WBC d. fever 40) re pheochromocytoma, all true ex: a. 10% will recur after resection\ surgery b. 10% bilateral c. 10% have HTN d. 10% extra-adrenal 41) re Desmoid, what is true: a. high local recurrence rate (40%) after resection b. pre-malignant c. painful …. 42) re colonic polyps, all true ex: a. majority are premalignant b. 95% of adenomatous polyps are tubular 43) M. C. site of colon –CA: a. sigmoid b. cecum d. splenic flexure 44) FAP a. associated with polyps in stomach & duodenum b. 45) M. C. site of appendix: retrocecal 46) M. C. isolated organism from blood Cx of cholingitis: E.coli 47) all in Charcot’s triad, ex: altered mental status 48) re colorectal-CA in UC: it is related to the dz duration 49) all seen in hyperparathyroidism ex.: chovstek’s sign 50) a case of newly diagnosed IBD, with a history of leg ulcers , Cx were negative, o Kaman investigation –ve , what’s ur management: a. systemic steroids & immunosuppressants b. local sulfadiazine 51) best image study of the adrenals: a. CT b. 131-I-MIBG c. NP-59 scan 52) when is the biopsy not useful\ contraindicated: a. previous b. obesity 53) all recommended for varicose prophylaxsis ex: a. periodic legs elevation during day b. elastic socks at night c. exercise
613
54) re pyloric stenosis, all true ex: usually present at 2-8 months old 55) all complications of gastrectomy ex: (not sure about the choices!) a. diarrhea b. steatorrhea c. dumping d. blind loop syndrome e ?? 56) calcitonin is secreted in: parafollicular cells of thyroid 57) so2alen 3n gas gangrene o necrotizing fasciaitis ?? 58) M.C .Cz of Hepatoma world-wide: HBV 59) case femal was on OCP, but stopped them & got pregnant, comes in with acute abdominal pain ….. , the most likely diagnosis: ruptured hepatic adenoma with bleeding 60) 46. Re colorectal carcinomas: (website: http://www.surgical-tutor.org.uk/defaulthome. htm?MCQ/mcq5.htm~right) a. About 40% present as surgical emergencies b. Right-sided tumours often present with a change in bowel habit c. Over 20% of patients present with more than one primary tumour d. A Duke's B tumour has nodal metastases 61) re amebic abscess, all true ex: A. Mortality is higher than that for similarly located pyogenic abscesses. B. The diagnosis of amebic abscess may be based on serologic tests and resolution of symptoms. C. In contrast to pyogenic abscess, the treatment of amebic abscess is primarily medical. D. Pts with amebic abscess tend to be younger than those with pyogenic abscess. E. it affects males in a 9:1 to 10:1 ratio. 62) The most common indications for splenectomy are, in descending order of frequency: Traumatic injury, immune thrombocytopenia, hypersplenism. 63) re Splenectomy and perioperative therapy for ITP one is true: A. Follow successful steroid therapy. B. Respond permanently to high-dose intravenous gamma globulin. C. Are best preceded by polyvalent vaccines for Pneumococcus, Haemophilus influenzae, and Neisseria meningitidis. D. associated with splenomegaly. 64) re parotid gland, all true ex: diagnosis is confirmed via core Biopsy 65) re hypersplenism, all true ex: bone marrow biopsy is normal (bcz it should be hyperactive)
614
66) post splenectomy, all true ex: thrombocytopenia 67) all causes hypertension post-op ex: a. pain b. morphine epidural can’t remember the rest of the choices 68) so2al 3n el-pseudocyst fistula , m.c to: a. gastric & duodenum b. transverse colon 69) case of DCIS, what’s the Mx: a. mastectomy b. lumpectomy c …….. 70) sentinel lymph node: 1st draining node (not 1st positive malignant) 71) non-proliferative benign breast Dz: a. fibroadenoma b. sclerosing adenosis c…… 72) case 30 yr old Female, treated with medications for hyperthyroid (I think it was single toxic nodule), but still has it, what to do? a. radiotherapy b. give potassium iodide for a week (or so), and do surgery c. lobectomy 73) indications for ICU admission: a. pt on CPAP b. ventilated pt c. pt needs continous monitoring of central venous pressure d. all of the above 74) hirschprung’s disease, all true ex: M:F = 1:4 (it’s supposed to be 4:1) 75) most common benign liver tumor: hemangioma 76) case of a pt w colon-CA resected 2 years ago, comes in w liver mets, what’s his prognosis (5 yr servival rate)? a. 33% b. 50% c. 15% 77) in managament of () all ex. a. oral anti-acids (jawabi) b. IV nexium 78) Re colonic polyps a. Metaplastic polyps are pre-malignant b. Adenomatous polyps are pre-malignant c. Villous adenomas are more common than tubular adenomas d. Genetic mutations can result in epithelial metaplasia Noor Al.huda Sawalha
Islamic University – Gaza
615
الجامعة السلمية – غــزة
كليــة الطــب
Faculty of Medicine
SURGERY – 4TH YEAR 2010-2011 1. Which of the following statements about thrombo-obliterative disease of the aorta and its branches is correct? A. The most common cause of obstructive disease is thrombi. B. Atherosclerosis is the most common pathologic cause of arterial obstruction. C. Lesions occur with greater frequency away from the origin of vessels from the aorta. D. Obstructive lesions are preferentially managed by endarterectomy. 2. In a patient who has chronic, complete occlusion of a common iliac artery, which of the following is not true? A. Symptoms are usually claudication of the thigh and calf. B. The decision as to whether or not to operate can be based on angiographic findings. C. Collateral iliac arterial vessels are prevalent. D. The primary symptom is claudication of the calf muscles. E. Balloon angioplasty is appropriate in some patients. 3.
Which of the following does not describe intermittent claudication? A. Is elicited by reproducible amount of exercise. B. Abates promptly with rest. C. Is often worse at night. D. May be an indication for bypass surgery.
4.
In patients with acute mesenteric ischemia due to mesenteric embolism, which of the following statements is correct? A. Patients often have a history of postprandial pain and weight loss. B. The use of digitalis may be a predisposing factor to the acute event. C. Arteriography usually reveals the embolus lodged at the orifice of the superior mesenteric artery. D. At the time of exploration, the proximal jejunum is often viable and ischemia is most severe in the more distal small bowel and colon.
5.
Which of the following is/are true with respect to ankle blood pressure and ankle brachial index (ABI)? A. An ABI of less than 0.92 almost always indicates hemodynamically significant arterial disease B. Claudicants have a wide range of ABIs with average values of 0.6 +/– 0.15 C. In limbs with rest pain the mean ABI is typically 0.25 +/– 0.13 D. In limbs with impending gangrene ABIs seldom exceed 0.25 and average about 0.05 +/– 0.08 E. All are true. Polyhydramnios is frequently observed in all of the following conditions except: A. Esophageal atresia. B. Duodenal atresia. C. Pyloric atresia.
6.
616
D. Hirschsprung's disease. E. Congenital diaphragmatic hernia. 7. In neonates with congenital diaphragmatic hernia, which of the following statements is true? A. The defect is more common on the right side. B. Survival is significantly improved by administration of pulmonary vasodilators. C. Oligohydramnios is a frequent occurrence. D. Mortality is the result of pulmonary hypoplasia. 8. A 2.8-kg. neonate with excessive salivation develops respiratory distress. Attempts to pass an orogastric catheter fail because the catheter coils in the back of the throat. A chest film is obtained and shows right upper lobe atelectasis and a gasless abdomen. The most likely diagnosis is: A. Proximal esophageal atresia without a fistula. B. Proximal esophageal atresia with a distal tracheoesophageal (TE) fistula. C. “H-type” TE fistula. D. Esophageal atresia with both proximal and distal TE fistula. E. Congenital esophageal stricture. 9. For a 22-kg infant, the maintenance daily fluid requirement is approximately which of the following? A. 1100 ml B. 1250 ml C. 1550 ml D. 1700 ml E. 1850 ml 10. Which of the following statements regarding duodenal atresia is not true? A. 20% to 40% of these infants have Trisomy 21 B. Double bubble appearance on abdominal x ray is characteristic for jejunal atresia C. Bilious vomiting is typical because the obstruction is usually distal to the ampulla of Vater D. Reconstruction is best achieved with Roux-en-Y duodenojejunostomy 11. Which of the following is not typical causes of neonatal intestinal obstruction? A. Intussusception B. Meconium ileus C. Hirschprung’s disease D. Jejunal atresia E. Incarcerated hernia 12. At what age is surgical orchidopexy recommended for a child with a unilateral undescended testis? A. Promptly upon discovery, regardless of age B. 1 year C. 5 to 6 years D. Any time prior to puberty
617
13. Which of the following statement(s) is true about benign lesions of the liver? A. Adenomas are true neoplasms with a predisposition for complications and should usually be resected. B. Focal nodular hyperplasia (FNH) is a neoplasm related to birth control pills (BCPs) and usually requires resection. C. Hemangiomas are the most common benign lesions of the liver that come to the surgeon's attention. D. Nodular regenerative hyperplasia does not usually accompany cirrhosis. 14. Which of the following statement(s) is true concerning focal nodular hyperplasia (FNH)? A. The lesion predominantly affects young women B. The lesion is associated with the use of oral contraceptives and other estrogens C. Radionucleotide scanning is not useful in the specific diagnosis of FNH D. Excisional biopsy is indicated in almost all cases because of the risk of bleeding 15. Which of the following statement(s) is true about bile duct cancers? A. If resected, proximal lesions are usually curable. B. The more proximal the lesion, the more likely is resection to be curative. C. Radiation clearly prolongs survival. D. Transplantation is usually successful if the lesion seems confined to the liver. E. None of the above is true. 16. Which of the following statements about the diagnosis of acute calculous cholecystitis is not true? A. Pain is so frequent that its absence almost precludes the diagnosis. B. Jaundice is present in about 10 % of patients. C. Ultrasonography is the definitive diagnostic test . D. Cholescintigraphy is the definitive diagnostic test. 17. Which statements about acute acalculous cholecystitis is not correct? A. The disease is often accompanied by or associated with other conditions. B. The diagnosis is often difficult. C. The mortality rate is lesser than that for acute calculous cholecystitis. D. The disease has been treated successfully by percutaneous cholecystostomy 18. Which of the following is indication for cholecystectomy? A. The presence of gallstones in a patient with intermittent episodes of right-side upper quadrant pain. B. The presence of gallstones in an asymptomatic patient. C. The presence of symptomatic gallstones in a patient with angina pectoris. D. The presence of asymptomatic gallstones in a patient who has insulin-dependent diabetes. 19. Which of the following statements about cholangitis is not correct? A. Charcot's triad is always present. B. Associated biliary tract disease is always present. C. Chills and fever are due to the presence of bacteria in the bile duct system. D. The most common cause of cholangitis is choledocholithiasis. 20. The clinical picture of gallstone ileus includes all the following except? A. Air in the biliary tree.
618
B. Small bowel obstruction. C. A stone at the site of obstruction. D. Acholic stools. E. Associated bouts of cholangitis. 21. 35-year-old alcoholic is admitted with acute pancreatitis. He complains of Numbness of his fingers and toes. On examination he has hyperactive Tendon reflexes. The most likely cause of these symptoms is A. Hyponatremia B. Hypocalcemia C. Hypophosphatemia D. Hypermagnesemia E. Hyperkalemia 22. Which of the following parameters is not included in the Ranson's prognostic signs useful in the early evaluation of a patient with acute pancreatitis? A. Elevated blood glucose. B. Leukocytosis. C. Amylase value greater than 1000 U per dl. D. Serum lactic dehydrogenase (LDH) greater than 350 IU per dl. E. Random blood sugar greater than 200 mg /dl 23. The evaluation of a comatose patient with a head injury begins with: A. The cardiovascular system. B. Pupillary reflexes. C. Establishment of an airway. D. Computed tomography (CT) of the brain. 24. Regarding An epidural hematoma all are true except : A. Is usually arterial in origin. B. Is usually accompanied by a skull fracture. C. Should be suspected only in comatose patients. D. It is lense shape on a brain CT scan which located between the dura and skull E. The outcome of therapy is directly related to the patient's level of consciousness before surgery. 25. All the following are complications of massive blood transfusion except: A. Hypothermia. B. Hypocalcaemia. C. Hypokalaemia. D. Acidosis. E. DIC. 26. Features of inflammatory response syndrome (SIRS) include the following except : A. tepmerature> 38.4C B. temeperature <34.6C C. WCC<4.ooo cells per ml D. respiratory rate >20 per minute E. PCO2> 32 mmHg 27. The mechanism of action of heparin is:
619
A. direct inhibition of thrombin B. prevention of factor II synthesis C. inhibition of cyclo-oxygenase D. potentiation of antithrombin III action 28. The following metabolic responses occur in response to trauma except A. Increased growth hormone B. Increased anti-diuretic hormone C. Increased ACTH D. Increased urine osmolality E. decrease glucagone 29. Regarding body response to trauma all are true except A. immediately after injury body reacts by decreasing energy requirement,temperature, consumption of O2 B. catabolic phase is associated with increased urinary nitrogen excretion and weight loss C. cortisol is elevated and kept elevated in burn patient up to 4 weeks D. Growth hormone, IGF(insulin like growth hormone) are secreted immediately in catabolic phase E. Both norepinephrine (NE) and epinephrine (EPI) are increased three- to fourfold in plasma immediately following injury. 30. A 78-year-old man with a history of coronary artery disease and an asymptomatic reducible inguinal hernia requests an elective hernia repair. You explain to him that valid reasons for delaying the proposed surgery include A. Coronary artery bypass surgery 3 months earlier B. A history of cigarette smoking C. Jugular venous distension D. Hypertension E. Hyperlipidemia 31. Postoperative ileus may result from all of the following except: A. Narcotic analgesics B. Electrolyte imbalance C. Non-steriodal anti-inflammatory drugs D. Poor pain control E. Organ-space infection 32. Postoperative pain might cause all of the following except: A. Hypercatabolism B. Hypoventilation C. Deep vein thrombosis D. Urine retention E. Diarrhea 33. The most effective method to control pain after major abdominothoracic surgery is: A. IV narcotics B. Epidural analgesia C. Wound infiltration with Bupivacaine
620
D. Combination of narcotics and NSAIDs E. None of the above 34. Which of the following disorders responds best to splenectomy? A. IgG hemolytic anemia B. Hairy cell leukemia C. Hereditary spherocytosis D. Thalassemia E. G6PD deficiency 35. The best time for elective pre-splenectomy vaccination is : A. On induction B. The night before C. Any time D. 2 weeks before E. None of the above 36. Splenic vein thrombosis is mostly associated with : A. Pancreatitis B. Trauma C. Portal hypertension D. Hematological disorders E. Splenic abscess 37. In acute GI Hemorrhage which of the following potentially increases morbidity /mortality? A. Renal failure B. Portal hypertension C. Need for surgery D. a + c E. a + b + c 38. Concerning bleeding in Mallory-Weiss syndrome which is false? A. Associated with sudden increase in intragastric pressure B. Associated with alcohol intoxication C. High recurrence rate D. Mucosal tear occurs E. Self limited 39. In diabetic patients, foot ulceration is mainly: A. Ischemic B. Traumatic C. Neuropathic D. Venous E. None of the above 40. In hemorrhagic shock renin secretion is mainly stimulated by : A. Increased plasma osmolality B. ACTH secretion C. Metabolic acidosis
621
D. Decreased hematocrit E. Decreased renal blood flow 41. A pressure ulcer that is covered with eschar is considered : A. Stage I B. Stage II C. Stage III D. Stage IV E. Unstageable 42. The most sensitive tissue to ischemia due to pressure is : A. Bone B. Muscle C. Skin D. Peripheral nerves E. Adipose tissue 43. Regarding tension pneumothorax , which is false ? A. Causes cardiac compressive shock B. Diagnosis is mainly clinical C. Beck’s triad is pathognomonic D. Mediastinal shift on chest x-ray E. Needs urgent decompression 44. The trauma primary survey includes all of the following except : A. Assessment of ventilation B. Endotracheal intubation C. Full physical examination D. Glasgow coma scale E. Direct hemostatic pressure 45. Regarding total body water (TBW) all are true except A. Insensible losses are about 800 ml daily B. The daily requirement of water in children is lower than adults C. TBW in female is less than males D. Insensible losses is electrolytes free E. Intracellular fluid represent two thirds of TBW 46. Hyperkalemia all are true except A. Is due to Crush syndrome B. Lead to Tall peaked T wave, and wide QRS complex C. Can be due to haemolysis and leukocytosis D. Can result from transfusion of old blood E. Redistributinal in alkalosis 47. Hypercalcaemia A. Is serum calcium more than 10.2 mg/dl B. Most common cause is hyperparathyroidism C. Prolong immobilization in bed can cause it D. Causes diarrhea
622
E. Total calcium level is not affected by serum albumin 48. Causes of hypercalcaemia include A. Osteolytic bone metastasis B. hyper parathyroidism C. Prolong immobilization in bed D. Milk alkali syndrome E. All of the above
49. These blood gases (PH 7.25, CO2 50, bicarb 30) could be due to A. Diarrhea B. Anxiety C. Sepsis D. Thoracic surgery E. Pyloric obstruction 50. Which of the following indicate uncompensated metabolic alkalosis A. PH 7.32,C02 40,bicarb19 B. PH 7.55,C02 37 bicarb 30 C. PH 7.55,C0 2 48,bicarb29 D. PH 7.44,C02 20, bicarb26 E. PH 7.44,C0 2 35 bicarb20 51. Regarding nitrogen balance all are true except A. 20% of nitrogen is lost in stool and urine B. One gram nitrogen is present in 6.25 gm protein C. Nitrogen requirement in 60 kilo lady is around 12 gms D. Calorie requirements for each gram nitrogen is 200 Kilo calorie in non –catabolic patient E. Nitrogen should not be used as energy source 52. Prognostic factors for malignant transformation of a polyp include all except A. Size of polyp B. Type of polyp C. Site of polyp D. Number of polyps E. Raised polyps 53. Regarding colorectal cancer all are true except A. Dukes C staging is with lymph node metastasis B. T4 tumor is adherent to surrounding organs C. Cecal tumors is usually presenting with small bowel obstruction D. None specific abdominal pain may be the first symptom E. Specificity of screening with fecal occult blood is about 50% 54. Cancer colon A. Familial polyposis coli lead to 50% incidence of cancer in affected people B. Synchronous lesions are two primaries happening one after the other with 6 month interval C. Dukes B mean breaching the muscularis propria D. Local recurrence is more in sigmoid cancers
623
E. All of the above 55. Regarding rectal cancer A. Only 25% of rectal cancer can be felt by PR examination B. Represent about 20% of colorectal cancer C. Endoscopic ultrasound has no rule in its staging D. Is roughly tumor within 12 cm from anal verge E. None of the above 56. Regarding tumor markers A. CA15.3 is marker for breast cancer B. CA19.9 is marker for liver primary C. Alpha fetoprotein is marker for renal cell cancer D. Ca 125 is marker for endometrial cancer E. Thyroglobulin is marker for differentiated thyroid cancer after subtotal thyroidectomy 57. Ulcerative colitis A. Peak incidence is in middle age females B. Usually start in sigmoid colon C. Usually runs intermittent course of relapses and remissions D. Incidence of dysplasia is related to severity of disease E. Stricture formation is a common complication 58. In ulcerative colitis all are true except A. Bloody diarrhea is commonest presentation B. There is depletion of goblet cell mucin C. Cancer risk is about 20 % after thirty years of diagnosis D. Extra colonic manifestation is only related to disease activity E. Stricture formation is common 59. In Crohn’s disease A. Weight loss is the second commonest systemic symptom B. Indeterminate colitis is present in one third of cases C. Pattern of behavior in Crohn’s include inflammatory, Stricturing, and fistulating type D. Abdominal pain is the commonest presenting symptom E. Fertility rates are not affected in patient with Crohn’s disease 60. Diverticular disease of colon A. Sigmoid colon is the region affected in about 60% of cases B. Bleeding per rectum is common in presence of diverticulitis C. Massive diverticular bleed is usually from right sided diverticulae D. Most of diverticulitis is complicated accompanied by abscess formation E. Has no cancer risk 61. In necrotizing fasciitis all are true except A. If cellulitis is spreading at a rate of half a cm per hour, diagnosis must be suspected B. Necrosis usually affect, skin, subcutaneous fat, fascia, and muscles C. Most of cases result from poly microbial infections D. Type II disease is due to group A Hemolytic streptococcus alone or with other organisms E. Pain out of proportion at infection site plus dishwater pus help in diagnosis
624
62. Regarding skin cancers A. Solar keratoses is a predisposing factor B. Squamous cell carcinoma is the commonest type of skin cancer C. Basal cell carcinoma spread usually by lymphatic’s D. Superficial basal cell carcinoma is commonest type E. None of the above 63. Bowen’s disease A. Is a sub type of basal cell carcinoma B. Is invasive squamous cell carcinoma C. Is squamous cell carcinoma in situ D. Has bad prognosis E. none of the above 64. Regarding mammograms A. Is usually recommended for screening above thirty years of age B. Usually two views Crainocaudal and oblique C. Macro calcification is an important sign of malignancy D. Axillary lymph nodes does not show in mammograms E. Ultrasound is more important in elderly 65. Fibroadenoma all are true except A. Occur in female about age of twenty B. Can be differentiated from phyllodes tumor by FNA C. Incidence of malignant changes is about 0.1% D. Can undergoes spontaneous regression if small E. Usually are mobile,smooth,firm lumps 66. Pathological nipple discharge A. Come from both breasts B. Occult blood –ve C. Single duct D. Not spontaneous E. No underlying mass 67. Regarding thyroid gland A. Thyroid lump is usually painless, except only if there is hemorrhage in to a cyst B. Thyroid cancer metastasize to bone and liver only C. Recurrent laryngeal nerve supply mucous membranes below vocal cords, and all muscles of larynx except cricothyroid D. Inferior thyroid artery arise from second part of subclavian artery E. All of the above 68. Papillary thyroid cancer all are true except A. Account for 50% of thyroid malignancy B. Spread by lymphatic’s C. Multifocal D. Commonest thyroid cancer in children E. All of the above
625
69. Regarding follicular thyroid cancer A. Account for 40% of thyroid cancer B. Early lymphatic spread C. Common in endemic goiter areas D. Multifocality is common E. Less aggressive than papillary tumors 70. Pressure symptoms of thyroid cancer include all except A. Dyspnea B. Inferior venacava obstruction C. Dysphagia D. Orthopnea E. Hoarseness of voice 71. Regarding anal fissure A. Most common in midline anteriorly B. Sentinle pile is a sign of chronicity C. Lateral fissure occur in constipation D. Deep fissure expose external sphincter E. None of the above 72. Regarding hemorrhoids, all are true except A. Fourth degree hemorrhoids cannot be reduced B. external piles arise below dentate line C. Straining and constipation is an important cause D. Causes fresh bleeding per rectum ,after defecation ,and on surface of stool E. First degree hemorrhoid cause pain only 73. Anal fistula A. Commonest type is intersphincteric fistula B. Goodsall’s rule states that posterior fistula open radially C. Is a normal communication between two epithelial surfaces D. Usually start as an abscess in Ischiorectal fossa E. All of the above 74. Inguinal hernia A. Indirect hernia complication is less than direct ones B. Usually lie above and lateral to inguinal ligament C. Sliding hernia is when hernia sac is formed by a viscus D. Deep ring test is by closing the internal ring above and medial to femoral pulse by one cm E. None of the above 75. Regarding femoral hernia, all are true except A. Lacunar ligament form medial boundary B. Roof is formed by inguinal ligament C. Lateral boundary is safe as it contain femoral vein D. Strangulation of femoral hernia is usually common
626
E. In elderly it can present by gastroenteritis like symptoms 76. Difference between direct and indirect hernia include A. Direct hernia is common in children B. Indirect hernia neck lie medial to inferior Epigastric artery C. Strangulation is more common in direct hernia D. Indirect hernia can reach scrotum E. Deep ring test prevent direct inguinal hernia from bulging 77. In paraumbilical hernia A. Defect is usually above umbilicus B. Neck is usually wide C. More common in males D. Reduction of content is easy E. Is Not considered a ventral hernia 78. For which of the following organ injuries is diagnostic peritoneal lavage (DPL) LEAST likely to be helpful? A. Panarceas B. Small intestine C. Spleen D. Sigmoid colon E. Liver 79. which is the MOST commonly injured intra-abdominal organ in blunt trauma? A. Liver. B. Kidney C. Spleen D. Stomach E. Colon 80. Burn injuries:A. If full thickness are normally sensitive to pin prick. B. Affecting a whole lower limb in a child will represent approximately 18% suface area. C. Are associated with Pseudomonas infections. D. Is associated with a low mortality in the elderly E. Are associated with nitrous oxide toxicity
81. Regarding electrical injury, all of the following statements are true EXCEPT A. Hyperkalemia may result from tissue necrosis. B. Myoglobin and hemoglobin pigment may produce renal failure. C. Heat is the principle mediator of tissue damage in high-volt-age injuries. D. heat production is related to tissue resistance increases in the immediate postburn period. E. The cellular immune response is depressed after burn injury. 82. Deep venous thrombosis prophylaxis is appropriate for all of the following patients EXCEPT A. A 67-year-old male undergoing a colectomy. B. A 21-year-old male undergoing an outpatient open inguinal hernia repair.
627
C. A 21-yar-old male in the ICU, comatose, with a closed head injury. D. A 60-year-old female undergoing open reduction and internal taxation (ORIF) of a hip fracture. E. A 53-year-old female undergoing resection of a lung carcinoma. 83. Complications of chronic varicose vein disease include. EXCEPT A. Eczema B. Lipodermatosclerosis C. Equinus deformity. D. Malignant transformation. E. Deep vein thrombosis 84. The least possible cause of right lower abdominal pain in a 18 years female is A. Ectopic pregnancy B. Acute appendicitis C. Ovarian torsion D. Perforated peptic ulcer E. Mittleschmerz
85. The best initial test in a patient with fever, left lower quadrant pain and tenderness is : A. Endoscopy B. Barium enema C. Ultrasound D. CT scan E. Plain x-rays 86. The best test to establish the presence of gastroesophageal reflux (GERD) disease is A. An upper gastrointestinal series B. Bernstein test (acid perfusion) C. 24-hour pH monitoring D. Esophageal manometry E. Endoscopic biopsy 87. Achalasia can be presented with all of the followings EXCEPT A. Recurrent pulmonary infections B. Weight loss C. Regurgitation D. Irregular narrowing of the distal esophagus by Ba. Swallow E. Retrosternal chest pain 88. Regarding gastric cancer which is false ? A. Patients with blood group A are at increased risk B. Most patients suffer from anorexia and weight loss C. Intestinal type has a worse prognosis D. H. pylori infection is a main etiologic factor E. Endoscopy is the diagnostic method of choice 89. The genetic predisposing factors to gastric cancer include the followings EXCEPT
628
A. Family history of gastric cancer B. Black race C. P 53 mutation D. Germline mutation of e-cadherin E. BRCA2 mutation 90. Primary gastric lymphoma A. 2% of all hodgkin's lymphoma B. Greatly differs in presentation from gastric adenocarcinoma C. One third of all gastrointestinal lymphomas D. The most common extranodal lymphoma E. Worse prognosis than adenocarcinoma 91. Mucosa associated lymphoid tissue ( MALT ) is found in all of the followings EXCEPT A. Small bowel ( Peyer's patches ) B. Waldeyer's ring C. Appendix D. Stomach E. Bronchus 92. In evaluating intestinal obstruction, which of the following suggests strangulation? A. Increased vomiting frequency B. Increased intensity of pain C. Localization of symptoms and signs D. Sudden disappearance of pain E. None of the above 93. A Meckel's diverticulum A. Occurs in 10% of the population B. Will be found on the mesenteric border of the small intestine C. Consists of mucosa without a muscle coat D. Usually found 20 cm from ileocecal valve E. A fibrous band between the apex and umbilicus can cause intestinal
obstruction
94. The mortality rate in multi-organ failure is directly related to: A. The number of the failing organs B. The presence of septic shock C. The presence of co-morbidities D. Extremes of age E. The underlying cause 95. Which of the following types of shock is characterized by Bradycardia ? A. Hypovolemic B. Septic C. Neurogenic D. Cardiogenic E. Cardiac compressive 96. The following predispose to wound infection EXCEPT A. Malnutrition
629
B. Hypovolemia C. Malignancy D. Obstructive jaundice E. Steroid therapy 97. Regarding wound healing, which is false ? A. The tensile strength of the injured tissue equals that of normal after 6 months . B. The capillaries are highly permeable in inflammatory phase . C. Disruption of normal healing may result in malignancy D. Vitamin C deficiency causes decreased proline hydroxylation E. Steriods affect inflammatory phase 98. The most common organism isolated from a localized wound infection is : A. Streptococci B. E. Coli C. Staphylococci D. Pseudomonas E. Bacteriodes 99. Salivary duct stones is mostly found in: A. Parotid gland B. Submandibular gland C. Sublingual gland D. Minor salivary glands E. b & c 100. The most common cause of hyperparathyroidism is : A. Chronic renal failure B. Parathyriod adenoma C. Parathyriod hyperplasia D. Parathyriod carcinoma E. Long standing hypocalcemia
Palestinian Medical Council Final General SurgeryThe Written End Exam (April,2011) Paper ( I ) Candidate’s Name:……………………………………………………………………………………………………………. Choose the best appropriate answer in each of the following questions: 1. Which of the following statements about extracellular fluid is true? A. The total extracellular fluid volume represents 40% of the body weight. B. The plasma volume constitutes 20%of the total extracellular fluid volume. C. Potassium is the principal cation in extracellular fluid.
630
D. The protein content of the plasma produces a lower concentration of cations than in the interstitial fluid. E. The interstitial fluid equilibrates slowly with the other body compartments. 2. Anion gap is: F. The difference between measured anions and measured cations G. Is normal in lactic acidosis H. Is high in shock I. Normally is 20 mmol J. High in pancreatic fistula
3. A 23-year-old female receiving chronic corticosteroid therapy for an autoimmune disease underwent minor surgery for incision and drainage of an abscess on her upper outer right arm. The wound healed poorly over the next month. Which of the following aspects of wound healing is most likely to be deficient? A. Re-epithelization B. Fibroblast growth factor elaboration C. Collagen deposition cvdsawsdsr D.Serine proteinase production E.Neutrophil infiltration 4. Preoperative investigation and preparation of a patient with obstructive jaundice should include all of the following except : A. measurement of coagulation status. B. measurement of 24-hour urinary output. C. measurement of serum urea and electrolytes. D. fluid restriction during the 24 hours preoperatively. E. administration of antibiotics during invasive diagnostic procedures (e.g. PTC, ERCP) 5.Diffuse axonal injury (DAI) results from what type of force acting on the brain? A. direct impact B. axial loading C. linear acceleration D. rotational acceleration E. accceleration deceleration injury 6. A 35-year-old trauma victim requires hyperalimentation. The patient’s injuries include a stable undisplaced fracture of the third thoracic vertebra, a closed head injury, multiple upper and lower extremity fractures and bilateral pulmonary contusions requiring ventilatory support. Which of the following are the most appropriate site and type of venous access in this patient? A. bilateral antecubital fossae, 18G peripheral intravenous catheters B. femoral vein, central venous catheter C. dorsum of one foot, single 16G peripheral intravenous catheter D. subclavian vein, central venous catheter E. long saphenous vein cut-down, long catheter 7.The mesentery of small intestine, along its attachment to the posterior abdominal wall, crosses all of the following structures except
631
F. G. H. I. J.
Left gonadal vessels Third part of duodenum Aorta Right Ureter All the above
8. The intravenous fluid that a 60 kg., 30-year-old woman with an 80% burn should be given in the first 24 hours following burn injury is: F. 19.2 liters of 5% glucose in lactated Ringer's. G. 14.4 liters of lactated Ringer's. H. 9.6 liters of hypertonic salt solution (sodium concentration 200 mEq) per liter.. I.
7.2 liters of 5% albumin solution.
J.
5.5 liters of the pentafraction component of hydroxyethyl starch.
9. All of the following are potential outcome in laparoscopic surgery EXCEPT : F. Hypercarbia G. Alkalosis H. Decrease urine out put I.
Increase intracranial pressure
J.
Increased cardiac work
10. In chest injury the following are indications of thoracotomy EXCEPT : F. When cardiac tamponade present. G. Massive air leak H. For all transmediastinal wounds I.
Flail chest with sever lung contusion
J.
Clotted hemothorax
11. Acute Epidural haematoma : A. due to meningeal artery tear B. can be treated conservatively C. classically there is contra lateral, dilated, fixed pupil D. needs operation within 24 hours E. a and c only 12.
632
All the following about Rectus Sheath is true except : A. Enclose the rectus abdominis and pyramidalis muscles B. Formed by the Aponeuroses of two abdominal muscles
C. Contain anterior rami of lower 6 thoracic nerves D. Contain superior and inferior epigastric vessels E. Separated from its fellow on the opposite side by linea alba
13. Regarding Acute subdural haematoma : A. It is due to meningeal artery tear B. has 30% mortality. C. may cause secondary brain injury D. Treated by early operation E. More urgent than epidural haematoma 14. Causes of hypovolaemic shock include all the following Except: A. burn injury. B. low output intestinal fistula C. spinal cord transaction. D. myocardial infarction. E. endotoxaemia. 15. Management of hypovolaemic shock involves all except: A. achievement of venous access via peripheral vein catheterisation. B. bladder catheterisation. C. immediate administration of whole blood or red cell concentrate. D. effective analgesia. E. oxygen administration. 16. Stored whole blood used for transfusion: A. contains similar amounts of coagulation factors blood. B. contains a concentration of leucocytes similar to normal blood. C. can be stored for up to 100 days at 4±2°C. D. contains normal platelet count. E. is used when rapid volume transfusion is required for a patient who has suffered major trauma. 17. Concerning the use of a central venous catheter: A. administration of TPN is safe and does not require repetitive haematological and biochemical monitoring. B. its insertion should have mandatory electrocardiographic monitoring . C. it provides a convenient portal for blood sampling and antibiotic administration. D. if tunnelled subcutaneously it has a higher incidence of infection with endogenous staphylococcus. E. if infected it is effectively treated by administration of antibiotics via the offending catheter. 18. Which of the following changes would you expect to find in a patient who developed acute renal failure after ingesting poisonous mushrooms that caused renal tubular necrosis? A. Increased plasma bicarbonate concentration B. Metabolic acidosis C. Decreased plasma potassium concentration D. Decreased blood urea nitrogen concentration E. Decreased hydrostatic pressure in Bowman's capsule
633
19 .A 16-year old driver, without seat belt, is involved in a rollover head-on accident. Extensive damage to the vehicle . The patient was pulled out of the car, hemodynamically unstable and was transported to a trauma center. During transportation the patient became severely hypotensive , with distended neck veins, unresponsive, with impending respiratory distress. Which of the following is considered an immediate life-threatening injury in this patient? A. pneumothorax secondary to rib fractures B. aortic intimal tear C. diaphragm rupture D. tension pneumothorax E. myocardial contusion 20.The metabolic response to injury is characterised by a sequence of physiological events aimed at: A. increasing core body temperature. B. conservation of sodium and water. C. mobilising glucose from fat and protein stores. D. maintaining body weight. E. enhancing immune function. 21. Fasciotomy for extremity compartment syndrome should be performed at a compartment pressure exceeding : A. 20 mmHg B. 30 mmHg C. 40 mmHg D. 50 mmHg E. none of the above 22. A 50 year old man present with haematemsis, Endoscopy shows a bleeding peptic ulcer in the posterior wall of the first part of duodenum. Which artery is involved A. Right gastric artery B. Right gastroepiploic artery C. inferior pancreaticoduodenal artery D. gasrtoduodenal artery E. superior pancreaticoduodenal artery 23. All of the following are true about neurogenic shock except: A. B. C. D.
there is a decrease in systemic vascular resistance and an increase in venous capacitance. tachycardia or bradycardia may be observed, along with hypotension. the use of an alpha agonist such as phenylephrine is the mainstay of treatment . Severe head injury, spinal cord injury, and high spinal anesthesia may all cause neurogenic shock.
24. Anaphylactic shock: A. is an immune-mediated reaction. B. results in mast cell activation and increased circulating histamine concentrations. C. produces microcirculatory changes similar to hypovolaemic shock. D. requires prompt treatment with parenteral adrenaline and hydrocortisone. E. may occur after ingestion of drugs 25. Which of the following best characterizes the secretions of the small intestine? A. Hypotonic and slightly acidic
634
B. Hypotonic and slightly alkaline C. Isotonic and slightly alkaline D. Isotonic and slightly acidic 26. Nasotracheal intubation all true except : A. B. C. D.
Is preferred for the unconscious patient without cervical spine injury. Is preferred for patients with suspected cervical spine injury. Maximizes neck manipulation. Is contraindicated in the patient who is breathing spontaneously.
27. The radiographic findings indicating a torn thoracic aorta include all Except : A. B. C. D.
Widened mediastinum. Presence of an apical “pleural cap.” Tracheal deviation to the right. right hemothorax.
28.Pseudomembranous enterocolitis is caused by which the following organisms: A. Clostridium sporogenes B. Clostridium defficile C. Streptococcus faecalis D. Penicillin sensitive staphylocci E. Pseudomonas aeruginos 29.Tuberculous cervical lymphadenitis: A. is caused by bovine tuberculous bacillus rather than the human bacillus B. usually occurs through the tonsil of the corresponding side C. collar-stud abscess is never a feature of T. B adenitis D. is treated always by surgery E. biopsy is not necessary for diagnosis
30. Which of the following structures in the gastrointestinal tract is most at risk in a patient as a consequence of the circulatory shock? (A) Submucosal glands (B) Brunner's glands (C) Tips of the villi
635
(D) Sphincter muscles 31. Carotid body tumour: A. is chromffin paragranuloma B. is usually bilateral C. is called chemodactoma or potato tumour D. is situated at the bifurcation of the carotid artery E. C and D are correct
32. The greatest burst strength of the wound will be achieved by: A. 1 week B. 3 weeks C. 6 weeks D. 12 weeks E. 6 months 33. With regards to cytokines, which of the following alternatives is correct? A. IL- is secreted mainly by lymphocytes and mediates inflammation. B. IL- 10involved in cell division and activation. C. IL-8 is secreted by macrophages and promotes chemotaxis. D. IL-2 is a major inhibitor of cell division. E. TNF-a is produced by T cells and is associated with a rise of immature neutrophils in the blood circulation.
34. A 63-year-old male with end-stage renal disease requiring hemodialysis three times per week presents with bone pain and several pathologic fractures of the extremities. Which is the most likely electrolyte abnormality in this patient? A. hypokalemia B. hypernatremia C. hyperphosphatemia D. hypercalcemia
636
E. hypochloremia 35. Which of the following will immediately delay or cancel an elective surgical case if not obtained appro priately preoperatively? A. CBC B. urinalysis C. CXR D. informed consent E. ECG 36. The cell type most characteristic of chronic inflammation is the: A. macrophage B. B cell C. natural killer cell D. neutrophil E. eosinophil 37.Which of the following is not an action of angiotensin II? A. aldosterone secretion B. sodium absorption C. efferent arteriolar constriction D. arterial dilation E. nephrosclerosis in the kidney
38.Which hormone is produced by the kidney? A. calcitonin B. erythropoietin C. 25 hydroxyvitamin D D. aldosterone E. antidiuretic hormone
637
39.What segment of the nephron is responsible for the majority of sodium absorption? A. proximal convoluted tubule B. loop of Henle C. distal convoluted tubule D. collecting tubule 40. A 46-year-old patient had respiratory failure from pneumonia and developed acute respiratory distress syndrome (ARDS). He remained on a ventilator for a prolonged period of time and ultimately underwent a tracheostomy at the level of tracheal ring 4. The patient is now in the rehabilitation unit of your hospital 2 weeks after his tracheostomy and you are called to see him about bright red blood around his tracheostomy site. The most common cause of this bleeding is A. granulation tissue B. erosion of the inferior thyroid artery C. erosion of the innominate artery D. tracheal chondritis E. recurrence of pneumonia 41. Low molecular weight heparin (LMWH) produces its primary effects because of its inhibition of which factor? A. IIa B. IXa C. Xa D. XIa E. XIIa 42. Regarding Shock, all are correct Except : A. it is defined as inadequate cellular perfusion. B. it may occur in the presence of normotension. C. it may occur following cardiomyopathy. D. it may follow gastrointestinal perforation. E. it invariably results in sympathomimetic activity in circulation. 43. Which of the following is not associated with increased likelihood of infection after major elective surgery? A. B. C. D. E.
638
Age over 70 years. Chronic malnutrition. Controlled diabetes mellitus. Long-term steroid use. Infection at a remote body site.
44. Advantages of epidural analgesia include: A. B. C. D. E.
Earlier mobilization after surgery. Earlier return of bowel function. Shorter hospitalizations. Decreased stress response to surgery. All of the above.
45. The following structures form the walls of the inguinal canal except: A. The conjoint tendon B. The aponeurosis of external oblique muscle C. The internal oblique muscle D. The lacunar ligament E. the fascia transversalis 47. A 60 years old male present with dysphagia. Flexible endoscopy shows a growth at cardia. What is the distance of this growth from the incisor teeth A. 25 cm B. 30 cm C. 35 cm D. 40 cm E. 45 cm 48. Regarding the blood supply of the stomach which of the following is not true A. Left gastric artery arise from hepatic artery B. Right gastric artery the lower right part of stomach C. The short gastric artery supply the fundus D. Left gastroepiploic artery arise from splenic artery at the hilum of stomach E. right gastroepiploic artery arise from gasrtoduodenal artery 49. what is the least common position of appendix A. Preileal B. Postileal C. Pelvic D. Paracaecal E. Subcaecal 50. Which of the following is true regarding the blood supply of appendix: A. The Appendicular artery passes in front the terminal ileum B. Blood from appendix drained by inferior mesenteric vein C. The Appendicular artery is a branch of lower division of ileocolic artery D. In most of the people there is accessory Appendicular artery E. The Appendicular artery run toward the base of appendix F. 51. Recurrent laryngeal nerve
639
F.
Is a pure motor nerve
G.
Supply cricothyroid muscle
H.
Supply mucous membranes of larynx above vocal cords
I.
The rate of injury during thyroid surgery is about 10%
J.
None of the above
K. 51. Which of the following disorders is not associated with hypocalcemia? A. sarcoidosis B. vitamin D deficiency C. renal insufficiency D. hypoparathyroidism E. pancreatitis
53. A. B. C. D. E.
Regarding the anatomy of gallbladder (GB) all the following is true except: GB lies in a fossa separating the right and quadrate lobes of liver The cystic duct is 3 cm length usually The intraduodenal part of common bile duct passes through the second part of duodenum Cystic artery is a branch of left hepatic artery Lymphatic vessels of GB drain into lymph node of Lund
54. Level one Axillary clearance F. Means removing lymph nodes behind pectoralis minor G. Mean dissecting apical lymph nodes H. Include removing anterior and posterior Axillary lymph nodes I. Include removing central Axillary nodes J. None of the above 55. The following statement concerning the anal canal are correct except : A. It encircle the anal canal B. It is not attached to the anococcygeal body C. It is composed of striated muscle fibers D. It is not responsible for causing the anal canal and rectum to join at an acute angle E. It is innervated by the middle rectal nerve
640
56. Which of the following can’t be palpated in the anterior surface while doing PR( digital rectal examination) A. Prostate B. Bulb of penis C. Posterior surface of bladder D. Seminal vesicle E. Ischiorectal fossa 56. Which of the following changes tends to increase urinary Ca ++ excretion? A. Extracellular fluid volume expansion B. Increased plasma parathyroid hormone concentration C. Decreased blood pressure D. Increased plasma phosphate concentration E. Metabolic acidosis 57. 75% of the breast is drained by axillary lymph nodes, regarding the axillary lymph node which of the following is not true F. Anterior group lie indirect contact with the axillary tail of Spence G. Posterior group lie along to lateral thoracic vein H. The intercostobrachial nerve passes between the central lymph nodes I. The apical group receive most of lymph from upper part of breast J. Lateral group lie along upper part of Humerus 58. While performing total thyroidectomy the surgeon ligates the superior pedicle as close to thyroid as possible to avoid injury of: A. External laryngeal nerve B. Recurrent laryngeal nerve C. Superior laryngeal nerve D. Internal laryngeal nerve E. All the above 59. All true about the blood supply of thyroid gland except A. Superior thyroid artery is the first branch of ECA B. Inferior thyroid artery is a branch of thyrocervical trunk C. Thyroida ima arise from subclavian artery D. Superior and middle thyroid vein drain into IJV E. Inferior thyroid vein drain into brachiocephalic veins 60. The following statements concerning the parotid salivary gland are correct except A. The facial nerve passes through it, dividing the gland into superficial and deep parts. B. The secretomotor nerve supply is derived from the facial nerve. C. The parotid duct pierces the buccinator muscle and opens into the mouth. D. The external carotid artery divides within its substance to form the superficial temporal and maxillary arteries.
641
E. The retromandibular vein is formed within it by the union of the superficial temporal vein and the maxillary vein 61. Which of the following changes would you expect to find in a patient consuming a highsodium diet (200 mEq/day) compared with the same patient on a normal-sodium diet (100 mEq/day), assuming steady-state conditions? A. Increased plasma aldosterone concentration B. Increased urinary potassium excretion C. Decreased plasma renin activity D. Decreased plasma atrial natriuretic peptide E. An increase in plasma sodium concentration of at least 5 mmol/L 62. All the following structures lie in the transpyloric plane EXCEPT: A. origin of the inferior mesenteric artery B. fundus of the gall-bladder C. termination of the spinal cord D. pancreatic neck E. duodenojejunal flexure
63. Over a period of several months, a 39-year-old woman has developed hyperpigmentation in association with an increase in blood pressure. Additionally, her blood glucose concentration has increased slightly. Which of the following is the most likely diagnosis? A. Addison's disease B. Conn's syndrome C. Pituitary tumor secreting large amounts of adrenocorticotropic hormone D. Adrenal tumor secreting large amounts of cortisol E. Panhypopituitarism 64. The anatomical left & right lobes of the liver are separated on the diaphragmatic surface of the liver by which of the following structures F. fissure for the round ligament of the liver G. fissure for the ligamentum venosum H. falciform ligament I. porta hepatic J. lesser omentum 65.The structure in the free border of lesser omentum from anterior to posterior are E. Common bile duct (CBD), Hepatic artery (HA), portal Vein ( PV). F. PV, HA, CBD
642
G. HA, PV, CBD D.PV, CBD, HA 66. Reed-Sternberg cells are characteristic of : A. chronic lymphatic leukemia B. cat scratch disease C. Hodgkin’s disease D. histiocytic medullary reticulosis E. large cell lymphoma 67. The primary anatomic site of pressure regulation in the vascular system is A. aorta B. arteries C. arterioles D. capillaries E. heart 68. The most common cause of aortic aneurysms is A. syphilis B. congenital weakness of the aortic wall C. atherosclerosis D. occlusion of the vasa vasorum E. bacterial arteritis 69. The major cause of pulmonary thromboemboli is: A. hypertension B. heart failure C. atherosclerosis D. thrombophlebitis E. varicose veins 70. Brown tumors of bone are the result of A. hyperparathyroidism
643
B. trauma C. metastatic carcinoma D. occlusion of the nutrient artery E. vitamin deficiency 71. Which is the most common primary malignant neoplasm of bone in the age group 15—25 years ? A. giant cell tumor B. Ewing’s sarcoma C. chondroblastoma D. osteosarcoma E. chondrosarcoma 72. The most common breast lesion in women is A. fibroadenoma B. carcinoma of the breast C. fibrocystic disease D. ductal papilloma 73. Bleeding from the nipple in a 45-year-old woman, without a palpable breast mass should suggest A. fibroadenoma B. sclerosing adenosis C. fat necrosis D. intraductal papilloma E. chronic cystic mastitis 74. Which breast lesion is most commonly bilateral A. intraductal carcinoma B. medullary carcinoma C. tubular carcinoma D. in situ lobular carcinoma
644
75. Pathologic fractures most typically occur with which of the following A. pituitary adenoma B. adrenal adenoma C. thyroid adenoma D. parathyroid adenoma E. pheochromocytoma 76. Diabetes insipidus is associated with a lack of A. glucocorticoids B. insulin C. thyroid hormone D. antidiuretic hormone E. growth hormone 77. Third space loss is due to all except F. Burns crush syndrome. G.
Severe soft tissue infections
H.
Repeated vomiting
I.
Intestinal obstruction
J.
Site of major operative dissection
K.
78. Each of the following applies to Hirschsprung’s disease except A. intestinal obstruction B. absence of ganglion cells in myenteric plexus C. treated by removal of distended segment of bowel D. toxic megacolon as complication E. narrow, more distal segment exhibits characteristic deficiency of development 79. Regarding polyps of the colon F.
645
Adenomatous polyps are usually solitary
G.
Villous polyps are usually pedunculated
H.
Metaplastic polyps are not precancerous
I.
Villous polyps occur more proximal in colon
J.
Cancer risk is not related to size of polyp
80. Carcinoid tumors (argentaffinomas) are characterized by all of the following except A. many are multiple especially in the ileum B. can occur in alimentary, biliary and respiratory tracts C. may produce systemic symptoms when accompanied by liver metastases D. appendix is the most common site E. appendiceal types have greatest malignant potential 81.Which of the following laboratory values correspond with acute DIC? A. slowly rising platelet count B. selective deficiency of vitamin K factors C. hypofibrigonemia D. prolonged bleeding time E. presence of fibrin split products 82. Damage to the pelvic nerves does which of the following? A. Has little, if any, effect on the defecation reflex B. Attenuates the defecation reflex C. Increases the strength of the defecation reflex D. Results in a continuous urge to defecate (rectal urgency)
83. The most common site of peptic ulcer formation is shown at which point on the figure below?
646
(A) Point A@ (B) Point B (C) Point C (D) Point D 84. A 45-year-old man is found to have a condition in which the parietal cells of his stomach have been destroyed by an autoimmune mechanism. His diagnosis is chronic autoimmune gastritis. This condition is often associated with which of the following? A. Pernicious anemia B. Gastric ulceration C. Steatorrhea D. Protein deficiency 85.The fundus of gall-bladder is in the angle between lateral border of right recius abdominis and which costal cartilage? G. H. I. J. K.
6th 7th 8th 9th 10th
86. Hypochlorhydria is a condition in which acid secretion by the stomach is greatly reduced. How does this condition affect the digestion and absorption of food? A. Digestion and absorption of all food substances are nearly normal B. Digestion and absorption of all foods substances are markedly reduced C. Digestion and absorption of carbohydrates are greatly affected by this condition D. Digestion and absorption of fats are greatly affected by this condition 87. Psychological stress is often associated with peptic ulcer disease. Which of the following is thought to be a contributing factor in stress-induced ulcer disease?
647
A. Sympathetic stimulation decreases gastric acid secretion B. Sympathetic stimulation increases peristalsis, which decreases the transit time of ingested food C. Psychological stress increases mucous secretion D. Sympathetic stimulation decreases the alkaline mucous secretions of Brunner's glands located in the first few centimeters of the duodenal wall 88. Regarding nitrogen balance F. 60% of nitrogen is lost in urine G.
40% of nitrogen is lost in skin and stool
H.
10 gm nitrogen is present in 62.5 gm proteins
I.
Nitrogen requirement is urinary nitrogen +40%
J.
All of the above
89. Regarding K+ ions: P. Hypokalemia Is defined as K+ level less than 3.9 mmol per liter Q.
Hypokalemia lead to wide QRS complex
R.
More than 80 % of K is present intracellulary
S.
K+ maintenance per day is around 3 mmol/ kg /day
T.
Hypokalemia cause metabolic alkalosis
90. Which of the following is true of mass movements? A. They normally move colonic contents from the cecum to the transverse colon B. They are strong peristaltic contractions of the small intestine in response to mucosal irritation C. They normally occur approximately 9 to 12 times per minute D. They are a modified type of peristalsis that occurs in the large intestine 91. A 60-year-old man is being treated for persistent diverticulitis with a prolonged course of antibiotic therapy. The antibiotic being used has broad-spectrum activity and over time has caused sterilization of the gut in this patient. Which of the following sequelae is likely to be observed in this patient? A. Vitamin B!2 deficiency B. Steatorrhea C. Vitamin K deficiency
648
D. Bloating due to excess gas formatio 92. Which of the following would cause the greatest degree of hyperkalemia? A. Increase in potassium intake from 60 to180mmol/day in a person with normal kidneys and a normal aldosterone system B. Chronic treatment with a diuretic that inhibits the action of aldosterone C. Decrease in sodium intake from 200 to l00mmol/day D. Chronic treatment with a diuretic that inhibits loop of Henle Na +-2 Cl -- K+ co-transport E. Chronic treatment with a diuretic that inhibits sodium reabsorption in the collecting ducts 93. Furosemide (Lasix) is a diuretic that also produces natriuresis. Which of the following is an undesirable side effect of furosemide due to its site of action on the renal tubule? A. Edema B. Hyperkalemia C. Hypercalcemia D. Decreased ability to concentrate the urine E. Heart failure 94. A 26-year-old man develops glomerulonephritis, and his glomerular filtration rate (GFR) decreases by 50 per cent and remains at that level. For which of the following substances would you expect to find the greatest increase in plasma concentration. A. Creatinine B. K+ C. Glucose D. Na+ E. Phosphate F. H+ 95. When compared with the late-evening values typically observed in normal subjects, plasma levels of both adrenocorticoiropic hormone and conisol would be expected to be higher in which of the following individuals? A. Normal subjects after waking in the morning B. Normal subjects administered dexamethasone C. Patients with Cushing's syndrome (adrenal adenoma)
649
D. Patients with Addison's disease E. Patients with Conn's syndrome 96. Where does the arterial supply to the superior and inferior parathyroids originate from? A. superior thyroid artery B. inferior thyroid artery C. external carotid artery D. internal carotid artery E. common carotid artery 97. According to the tumor, node, metastasis (TNM) stag ing for colon cancer, which of the following is true? A. All perforated colon cancers are considered T4. B. N2 refers to involvement of greater than one regional lymph node. C. T5 grade involves direct carcinoma invasion into adjacent solid organs. D. MX indicates metastatic disease involvement of more than one additional organ system (e.g., liver, lung, brain(. E. Five-year survival for stage I colon cancer is approximately 75% 98. To confirm endotracheal intubations in emergency room, all are true except F. Equal bilateral breath sounds. G. Pulse oximetry H. No noise over epigastrium. I. Chest x ray. J.
CO 2 detector
99.A 5-year-old boy presents to Accident and Emergency complaining of acute pain over his upper tibia. He is febrile and he refuses to move his leg. A diagnosis of osteomyelitis is suspected. The likely infecting organism is? F. G. H. I. J.
650
Clostridium difficile Haemophilus influenzae Pseudomonas Salmonella Staphylococcus aureus
100. The rectum: A. is devoid of peritoneum B. is surrounded by peritoneum C. has peritoneum on its lateral surfaces for its upper two- thirds, and on its anterior surface for its upper one-third D. has pritoneum on its anterior surface for its upper two-thirds, and on its lateral surfaces for its upper one-third E. has peritoneum on its anterior surface only
GOOD LUCK Palestinian Medical Council Final General Surgery Written Exam (April,2011) Paper ( II )
Candidate’s Name:……………………………………………………………………………………………….. Choose the best appropriate answer in each of the following questions: 1. Achalasia is associated with all of the following EXCEPT: A. abscence of ganglionic cells in the dilated oesophagus B. dysphasia. C. weight loss. D. relaxation of the lower esophageal sphincter with swallowing . E. aspiration pneumonia, which may cause lung abscesses. 2. All of the following are true statements concerning paget’s disease of the nipple EXCEPT: A. it is very uncommon, accounting for only 2% of all breast cancers. B. it is an in situ squamous cell malignancy of the nipple. C. it is an eczematoid lesion. D. it has a better prognosis than the majority of other breast cancers. E. it can be confused with malignant melanoma histologically.
651
3. Decreased Paco2 levels should be attained in a patient at serious risk for cerebral edema secondary to a head injury in order to :A. prevent neurogenic pulmonary edema B.allow reciprocally high levels of PaO2 in the brain C.prevent increased capillary permeability D. prevent cerebral vasodilation E.prevent metabolic acidosis 4. In massive haemothorax. A. about 500ml of blood in pleural cavity B. cause dyspnea & Neck veins distension C. treated by thoracotomy tube only D. All of the above E. None of the above
5. Axillary lymph nodes are classified according to the relationship with the: A. axillary vein B. pectoralis major muscle C. pectoralis minor muscle D. latissimus dorsi muscle E. serratus anterior muscle
6. A 65-year-old female presents with a large (5 cm) mass in her right breast with overlying ulceration of the skin. She is noted to have several enlarged, mobile axillary lymph nodes on examination. She is subjected to a core biopsy which confirms the impression of invasive ductal carcinoma of the breast. Her management should begin with A. mastectomy B. sentinel lymph node biopsy C. systemic chemo or hormonal therapy D. radiation therapy E. alternative therapy .
652
7. Fine needle aspiration (FNA) cytology of a thyroid Hurthie cell neoplasm will most likely show which of the following reports? A. nondiagnostic/unsatisfactory B. benign nonneoplastic C. inconclusive D. malignant 8. A 35-year-old female involved in a restrained motor vehicle accident presents to your emergency depart ment with suspected multiple organ damage. After the patient is stabilized, a chest radiograph is performed revealing opacification of the left side of the patient’s chest. A traumatic diaphragmatic injury is immedi ately suspected. The next step in management is A. immediate exploratory laparotomy B. delayed thoracotomy C. video-assisted thoracoscopy D. barium swallow E. nasogastric tube insertion 9. A 22-year-old lady with hypertension refractory to medical management is referred to you after a CT scan was obtained that demonstrates a 3 cm mass in the right adrenal gland. The initial diagnostic test of choice is A. measurement of catecholamines, metanephrines, and vanillyhnandelic acid in a 24-h urine collection B. MRI C. measurement of catecholamines and metanephrines in a morning serum sample D. high-dose dexamethasone suppression test 10 . Superior mesenteric artery (SMA) syndrome is not associated with which of the following? A. scoliosis B. placement of a body cast C.anorexia nervosa D. Abdominal aortic aneurysm E. rapid weight gain
653
11. Clinical features of inflammatory breast cancer include all except A. Erythematous Indurated breast B.
Usually underlying mass
C.
Young women30-40 year
D.
Quick onset
E.
No response to antibiotic therapy
12. The most useful radiologic examination for small bowel tumors is: A. upper GI with small bowel follow through B. enterocolysis C. abdomen and pelvis CT scan D. esophagoduodenoscopy 13. A patient undergoes proximal gastric vagotomy (PCV) (highly selective vagotomy) for an intractable duodenal peptic ulcer. Which can be said regarding post vagotomy syndromes following this technique? A. Dumping syndrome occurs with greater incidence in these patients than truncal vagotomy with pyloroplasty.
those who undergo
B. Dumping syndrome can often be managed by diet modification including taking liquids30 mm before meals and adding concentrated carbohydrates to meals. C. If postvagotomy diarrhea occurs, it tends to persist and become more severe. D. Alkaline reflux gastritis can occur in up to 10% of patients. E. The incidence of dysphagia is higher with PCV than truncal vagotomy. 14. Which of the following regarding inflammatory bowel disease is true? A.Bloody diarrhea is more common in Crohn’s disease than ulcerative colitis, B. Associated extraintestinal disease is less common in ulcerative colitis than in Crohn’s disease. C. Crobn’s disease frequently involves the entire colon D. Crohn’s is more commonly associated with primary sclerosing cholangitis than is ulcerative colitis, E. Surgical intervention for ulcerative colitis is limited to management of complications of the primary disease process
654
15. A left neck exploration is performed for a stab injury to the neck in an otherwise healthy male. The left internal jugular vein is found to be completely transected with significant bleeding and hemodynamic instability. No other vascular injury is identified. Appropriate management of the internal jugular vein is: A. primary repair B. interposition saphenous vein graft C. interposition with 6 mmPTFE D. ligate the left internal jugular vein E. external jugular vein transposition E. echocardiogram 16. Regarding acute colonic pseudoobstruction (Ogilvie’s syndrome), each of the following is correct except: A. This condition is defined as the radiographic appearance of a large bowel obstruction without mechanical etiology. B. The best pharmacologic treatment is neostigmine, which leads to rapid decompression in a significant number of adults after a single infusion. C. Ogilvie’s syndrome is associated with a number of neurologic disorders, including Alzheimer’s and Parkinson’s disease and elderly dementia. D. Most cases of this disease are idiopathic in etiology E. The proximal and transverse portions of the colon tend to be more involved than the left or sigmoid colon. 17. Melanoma of the anal canal can be discovered incidentally in hemorrhoid specimens. Which of the following is incorrect regarding melanoma? A. The anal canal is the most common site of the development malignant melanoma of the alimentary tract. B. 0.2% of all melanomas occur in the anal canal. C. All melanoma-in-ani are characteristically pigmented . D. Although abdominoperineal resection has traditionally been the surgical procedure of choice for melanoma of the anal canal, wide local excision is gaining acceptance. E. Supplemental therapy with chemotherapy, radiotherapy, and been of no consistent benefit for survival.
655
immunotherapy has
18 . A 45-year-old man is found to have a 6 cm calcified cystic liver lesion containing daughter cysts in the right lobe of the liver. The initial preferred treatment includes A. right hepatic lobectomy B. pericystectomy C. a 2-week course of metronidazole D. percutaneous catheter drainage E. laparoscopic drainage 19.Most primary and metastatic tumors to the liver derive nearly all their vascular inflow from ches of the: A. portal vein B. collateral circulation C. hepatic artery D. celiac axis E. unnamed branches from the aorta 20. During a laparoscopic cholecystectomy performed on a healthy young man, a 1-cm cavemous hemangioma is noticed on the surface of the left lobe of the liver. The most appropriate treatment for this lesion at this lime is A. biopsy and frozen section analysis B. observation C. resection D. laparoscopic fulguration E. postoperative embolization
21. Which of the following statements is true of infants with gastroschisis? A. It is not associated with malrotation. B. There is a high incidence of associated anomalies. C. There is prolonged adynamic ileus following repair . D. It is complicated by intestinal atresia in 50% to 55% of cases. E. It is associated with chromosomal syndromes.
656
22. What is the most common organism isolated from bile and blood cultures in patients with acute cholangitis? A. Enterobacter spp. B. Bacteroides spp. C. Escherichia coli D. Enterococcus spp. E. Candida albiccins 23. A 67-year-old male presents to your office with painless jaundice. Courvoisiers sign is present. An abdominal ultrasound (US) is performed, followed by an abdominal computed tomography (CT) scan. You suspect that there is a neoplasm in the head of the pan creas. Which of the following CT scan findings does not rule out resectability? A. encasement of the superior mesenteric artery (SMA) or celiac axis B. dilated intra- and extrahepatic biliary ducts, with an engorged gallbladder C. the confluence of the superior mesenteric vein (SMV and portal vein is not patent D. evidence of extrapancreatic disease E. absence of a fat plane between the tumor and the SMA 24. Which of the following is true with regard to palliative strategies for unresectable pancreatic cancer? A. Pancreatic adenocarcinoma is an asymptomatic disease. B. Celiac axis neurolysis is effective in 90% of patients over the short term. C.Gastric outlet obstruction in the presence of unresectable disease is best treated by gastrojejunostomy. D.Presence of biliary obstruction is best treated with biliary-enteric anastomosis. E.Prophylactic gastrojejunal anastomosis is widely recommended. 25. A patient with a history of atrial fibrillation presents with a 6-h history of acute right lower extremity pain. On examination, the lower extremity is cool to the touch and the patient has pain on attempts to move his foot. The femoral pulse is palpable but pulses are absent, as are Doppler signals below this level. The next step in management should be: A. immediate embolectomy B. arteriogram C. anticoagulation and observation
657
D. amputation E. femoral-femoral bypass 26. Which of the following is true regarding carotid artery stenosis? A. The 5-year stroke rate for patients with equal or >60% asymptomatic lesion is 9% with surgery versus 26% with medical treatment. B. The 2-year stroke rate for patients with an equalor >70% symptomatic lesion is 5% with surgery versus 11% with medical treatment. C. The 5-year stroke rate for patients with an equal or >60% asymptomatic lesion is 5% with surgery versus 11% with medical treatment. D. The 5-year stroke rate for patients with an equal or >70% symptomatic lesion is 11% with surgery versus 26% with medical treatment. E. Stroke rates for medical versus surgical treatment in patients with an asymptomatic lesions is equivalent.
equal or >60%
27. Which of the following is false regarding subclavian steal? A. The right subclavian artery is affected significantly more often than the left. B. Clinical symptoms can be ischemic or vertebrobasilar in nature. C. Prosthetic grafts (PTFE or Dacron) are more desirable than saphenous vein for use in bypass. D. Symptomatic lesions can be treated with bypass, transposition, or in some instances stenting. E. Transposition is contraindicated in patients with a prior left internal mammary artery (LIMA) coronary graft in place. 28. Which of the following is true regarding aneurysmal disease of the aorta? A. The aorta is considered aneurysmal when its size is twice that of the normal proximal aorta. B. Women are generally affected more frequently than men. C. Aneurysm walls contain increased levels of collagen and elastin. D. The most common presentation of AAA is back pain. E. The risk of rupture increases with increasing size of the aneurysm. 29. Which of the following is true of retroperitoneal soft tissue sarcomas? A. The most common histologic cellular type is malignant fibrous histiosarcoma. B. Because of the risk of tumor seeding, preoperative biopsy is
658
contraindicated.
C. Resection of contiguous organs that are not involved with tumor is indicated to obtain adequate margins. D. Death usually results from distant metastases. E. Lung is the most common site of distant metastases. 30. What is the blood supply for a pedicled transverse rectus abdominus muscle (TRAM) flap? A. superior epigastric artery B. inferior epigastric artery C. rectus abdominis perforators D. internal iliac artery E. intercostal artery 31.All are true about the dumping syndrome except: A. B. C. D.
Symptoms can be controlled with a somatostatin analog. Diarrhea is always part of the dumping syndrom e. Flushing and tachycardia are common features of the syndrome. Separating solids and liquids in the patient's oral intake alleviates some of the symptoms of the syndrome. E. Early postoperative dumping after vagotomy often resolves spontaneously. 32. A 28-year-old white male presents with asymptomatic testicular enlargement. Which of the following statement is true concerning his diagnosis and management? A. Tumor markers, Alfa-fetoprotein (AFP) and human chorionic gonadotropin (HCG) will both be of value in the patient regardless of his ultimate tissue type B. Orchiectomy should be performed via scrotal approach C. The diagnosis of seminoma should be followed by postoperative radiation therapy D. With current adjuvant chemotherapy regimens, retroperitoneal lymphadenectomy is no longer indicated for non-seminomatous testicular tumors 33. Pancreas divisum results from incomplete fusion of the ventral pancreatic duct with the dorsal pancreatic duct during embryologic development. Which of the following statements correctly describes pancreas divisum? A. The body and tail of the pancreas drain via an accessory ampulla distal to the ampulla of Vater. The uncinate process drains via the ampulla of Vater B. The entire pancreatic ductal system drains via the ampulla of Vater C. The entire pancreatic ductal system drains via an accessory ampulla proximal to the ampulla of Vater D. The body and tail of the pancreas are absent. The uncinate process drains via the ampulla of Vater
659
34. The following statements about the repair of inguinal hernias are true except: A. The conjoined tendon is sutured to Cooper's ligament in the Bassini hernia repair . B. The McVay repair is a suitable option for the repair of femoral hernias. C. The Shouldice repair involves a multilayer, imbricated repair of the floor of the inguinal canal. D. The Lichtenstein repair is accomplished by prosthetic mesh repair of the inguinal canal floor in a tension-free manner. E. The laparoscopic transabdominal preperitoneal (TAPP) and totally extraperitoneal approach (TEPA) repairs are based on the preperitoneal repairs of Cheattle, Henry, Nyhus, and Stoppa. 35 . H. pylon produces hypersecretion of gastric acid in patients who develop H. pylon-related duodenal ulcers. What is a proposed mechanism for development of increased gastric acid secretion by this bacterium? A. attenuation of somatostatin release from antral D cells B. direct stimulation of gastrin receptor on parietal cells C. direct stimulation of proton pump on parietal cells D. blockage of somatostatin receptors on parietal cells E. bacterial enzymatic degradation of local prostaglandins 36. Which of the following is not an absolute contraindication to laparoscopic inguinal hernia repair? A. presence of infection B. previous radical prostatectomy C. strangulated hernia D. cirrhosis with portal hypertension E. history of pelvic irradiation 37. Which of the following statements about familial adenomatous polyposis (FAP) is true? A. Inherited in an autosomal-dominant manner, this genetic defect is of variable penetrance, some patients having only a few polyps whereas others develop thousands. B. The phenotypic expression of the disease depends mostly on the genotype. C. Appropriate surgical therapy includes total abdominal colectomy with ileorectal anastomosis and ileoanal pull-through with rectal mucosectomy. D. Panproctocolectomy with ileostomy is not appropriate therapy for this disease.
660
E. Pharmacologic management of this disease may be appropriate in some instances 38. Which of the following is true about the use of radioiodine to treat hyperthyroidism? A. If hyperthyroidism is secondary to radioiodine use, it will occur within 2 years of treatment. B. There is a markedly increased risk of future thyroid cancer following radioiodine therapy. C. The risk of leukemia following radioiodine therapy is approximately 10%. D. Mutation abnormalities occur in 15% of fetuses in utero following internal treatment of the mother with radioiodine during pregnancy. E. Radioiodine may pass through the placenta and lactating breast to produce hypothyroidism in a fetus or infant. 39. A 45-year-old lady has a biopsy proven,ill-defined morpheaform basal cell carcinoma (BCC) on the tip of the nose (Fig. 33-2).The best therapeutic option for this patient is A. radiation therapy B. surgical excision C. electrodesiccation and curettage D. cryosurgery E. Mohs micrographic surgery 40. Regarding phyllodes tumor of the breast A. All should be treated by simple mastectomy B.
FNA is the best diagnostic option
C.
Axillary lymph node dissection is part of treatment
D.
Small phyllodes are excised with one cm free resection margin
E.
None of the above
41. Malignant thyroid nodule by fine needle aspiration is due to: A. Papillary thyroid cancer
661
B.
Medullary thyroid cancer
C.
Anaplastic thyroid cancer
D.
Lymphoma
E.
All of the above
42. Small bowel fistula A. Type II occur in previously diseased bowel B.
If fistula tract is more than 2.5 cm the fistula is unlikely to close
C.
Epithelisation of fistula tract help fistula closure
D.
Duodenal stump fistula is unlikely to close
E.
Radiation fistula to small bowel is unlikely to close
43. In short bowel syndrome all are true except A. Intact ileocaecal valve is important factor in the determining the outcome of massive small bowel resection B.
Short bowel syndrome increase incidence of gall bladder stones
C.
40% of short bowel syndrome are due mesenteric vascular disease
D.
The function of jejunum is taken over by ileum after massive Jejunal resection
E.
Bianchi procedure increase functional absorptive capacity of bowel.
44. With regards to Crohn’s disease A. The disease is confined to mucosa B.
Submucosal fibrosis occurs secondary to bacterial invasion
C.
Anal involvement is common
D.
Bloody diarrhea is the main complaint
E.
Rectal sparing is rare
45. With regard to Meckel’s diverticulum A. They are true diverticulum B.
Most complication occur in elderly
C.
Is the second commonest diverticulum of small intestine
D.
Diverticulitis is the most common complication
E.
Prophylactic diverticulectomy should be done routinely if found during operation for other cause
46. with regards of ulcerative colitis A. The disease is most commonly chronic relapsing with acute fulminating type seen only in 10% of cases B.
662
Cancer arising in patients with ulcerative colitis is more aggressive than other colon cancers
C.
Crypt abscesses is seen only in ulcerative colitis
D.
Usually there is sparing of goblet cell mucin
E.
All of the above
47. In children, umbilical hernia is best operated after age: A. 9-12 months B. 1-2 years C. 3-4 years D. 10-12 year 48. What percentage of patients with blunt thoracic trauma require thoracotomy? A. 10%–15%. B. 20%–25%. C. 30%–40%. D. 45%–50%. E. 60%-70% 49. The following are true about inhalation injury in burn patients except: A. chest x-ray obtained within 24 hours of injury is an accurate means of diagnosis. B. Its presence characteristically necessitates administration of resuscitation fluids in excess of estimated volume. C. When moderate or severe, it exerts a comorbid effect that is related to both extent of burn and the age of the patient. D. It increases the prevalence of bronchopneumonia. E. Prophylactic high-frequency ventilation reduces the occurrence of pneumonia and the mortality in burn patients with inhalation injury. 50. The treatment of patients with high-voltage electric injury differs from that of patients with conventional thermal injury with respect to the need for except: A. Fasciotomy. B. Hemodialysis. C. Amputation. D. Pulse oximetry. E. Prehospital cardiopulmonary resuscitation. 51. In neonates with necrotizing enterocolitis, which of the following findings is an indication of significant bowel ischemia? A. Increased gastric residuals. B. Septic shock.
663
C. Cardiac failure due to a patent ductus arteriosus. D. Elevated platelet count. E. Erythema of the abdominal wall. 52. Which of the following statements regarding colonic diverticulosis is true? A. Diverticula are primarily located in the transverse colon. B. The rectum is virtually never the site of diverticula. C. The pathogenesis of diverticulosis has been associated with hyperactive colonic motility and high fecal volume. D. No identifiable etiology for diverticulosis has been described at the cellular level. E. In a patient undergoing sigmoid colon resection for apparent isofated sigmoid divertiathtis, he recurrence rate for diverticulitis approaches 90%. 53. What is the most sensitive test for the diagnosis of biliary dyskinesia? A. abdominal CT scan B. ERCP C. cholecystokinin-Tc-HIDA scan D. abdominal US E. magnetic resonance cholangiopancreatography (MRCP) 54. A 2.8-kg. neonate with excessive salivation develops respiratory distress. Attempts to pass an orogastric catheter fail because the catheter coils in the back of the throat. A chest film is obtained and shows right upper lobe atelectasis and a gasless abdomen. The most likely diagnosis is: A. Proximal esophageal atresia without a fistula. B. Proximal esophageal atresia with a distal tracheoesophageal (TE) fistula. C. “H-type” TE fistula. D. Esophageal atresia with both proximal and distal TE fistula. E. Congenital esophageal stricture.
664
55. A 3-week old infant has a barium upper gastrointestinal series to evaluate vomiting. The duodenojejunal flexure is found to be to the right of the midline as well as more caudal and anterior than a normal ligament of Treitz. The child is seen to reflux barium spontaneously to the level of the mid-thoracic esophagus. You would recommend which of the following? A.
Barium enema
B.
Emergency laparotomy
C.
A trial of H2, blockade and cisapride therapy
D.
Upper gastrointestinal endoscopy
E.
Overnight pH probe analysis
56.The treatment of choice for neonates with uncomplicated meconium ileus is: A. Observation. B. Emergency laparotomy, bowel resection, and Bishop-Koop enterostomy. C. Intravenous hydration and a gastrograffin enema. D. Emergency laparotomy, bowel resection, and anastomosis. E. Sweat chloride test and pancreatic enzyme therapy. 57. Breast magnetic resonance imaging (MRI) A. is useful as a screening tool B. accurately differentiates tumor recurrence and postsurgical scar formation C. should not be used if the patient has a breast implant D.allows easy tissue procurement for pathologic diagnosis E. is very specific for detecting malignant lesions 58. In infants with duodenal atresia all the following statements are true except: A. There is an increased incidence of Down syndrome. B. Duodenal atresia can be detected by prenatal ultrasound examination. C. It may occur in infants with situs inversus, malrotation, annular pancreas, and anterior portal vein. D. It is best treated by gastroenterostomy. E. There is a high incidence of associated cardiac defects. 59. The most common type of congenital diaphragmatic hernia is caused by: A. A defect in the central tendon. B. Eventration of the diaphragm in the fetus. C. A defect through the space of Larrey. D. An abnormally wide esophageal hiatus. E. A defect through the pleuroperitoneal fold.
665
60. For a 22-kg infant, the maintenance daily fluid requirement is approximately which of the following? A. 1100 ml B. 1250 ml C. 1550 ml D. 1700 ml E. 1850 ml 61. A 56-year-old female is found to have symptoms of hypothyroidism with a painless, enlarged, firm, rubbery thyroid gland. FNA shows diffuse infiltration of the gland with lymphocytes and plasma cells. Which of the following best describes her condifion? A.
Hashimoto’s thyroiditis
B. acute suppurative thyroiditis C.
Riedel’s thyroiditis
D. painless thyroiditis E. subacute de Quervain thyroiditis 62. Which of the statements regarding cecal volvulus is Definitive treatment for cecal volvulus includes a right hemicolectomy? A. Cecal volvulus accounts for 10% of cases of colonic volvulus. B. Cecal vovulus is thought to have a congenital etiology related to incomplete peritoneal fixation of the right colon. C. Radiographic evidence of a cecal volvulus includes a large, dilated loop of colon with the loop of colon pointing to the left upper quadrant of the abdomen. D. Reduction of the cecal volvulus with fixation of the cecum to the abdominal wall provides a similar outcome to segmental resection not true E. All of the above are correct. 63. The following are accepted adjuncts in the management of hypertrophic scar except: A. Local steroid injection B. Compression garments C. Topical silicone D. Release or excision with autografting E. Topical platelet-derived growth factor 64. Valid points concerning the initial physical examination in a burn patient include which of the following statements except: A. Patients should be examined in a warm environment to prevent hypothermia B. All corneal injuries are obvious on initial physical examination C. Inhalation injury is suggested by the presence of singed facial hair and nasal vibrissae,
666
carbonaceous sputum, and a hoarse voice D. Blistering in or around the mouth may suggest hot liquid aspiration in small children 65. Which of the following statements about pulmonary hamartomas is true? A. Hamartomas are benign chondromas. B. Most are located in the conducting airways. C. Wedge resection is curative. D. A lobectomy is necessary to obtain draining hilar lymph nodes. E. Hemoptysis is common. 66. A solitary pulmonary nodule is discovered in an asymptomatic 55-year-old smoker with no evidence of extrathoracic dissemination. The most appropriate management would be to: A. Obtain serial chest films every 3 months to determine the growth potential of the nodule. B. Perform transthoracic needle aspiration (TTNA) before considering pulmonary resection to confirm malignancy. C. Conduct an extensive systematic evaluation to exclude the possibility that the nodule represents a metastatic lesion. D. Proceed with pulmonary resection after ascertaining that the patient would tolerate removal of the requisite amount of lung. E. Obtain baseline serum levels of carcinoembryonic antigen 67. After thoracotomy, pulmonary resection, and mediastinal lymph node dissection, a patient is determined to have a squamous cell carcinoma 2 cm. in diameter, located 1 cm. from the carina along the right mainstem bronchus. Three peribronchial lymph nodes are positive for cancer, and all other lymph node stations are negative. The correct stage, according to the TNM system, is: A. T1N0M0 Stage I. B. T1N1M0 Stage II. C. T2N1M0 Stage II. D. T3N1M0 Stage IIIa. E. T2N3M0 Stage IIIb. 68. Which of the following variables best predicts prognosis for patients with a recent diagnosis of cutaneous melanoma and no clinical evidence of metastatic disease? A. B. C. D. E.
Breslow thickness . Clark's level. Ulceration. Gender. Celtic complexion.
69. A 40 year old woman develops severe lymphedema involving the right calf and foot after receiving radiation therapy to metastatic lymph nodes in the inguinal region and the femoral triangle. The most appropriate management on this problem is: A. Below-knee amputation and a prosthetic limb.
667
B. Microsurgical lymphaticovenous anastomoses in the affected area. C. Staged excision of subcutaneous tissue and excess skin. D. Subcutaneous placement of an omental flap into the upper calf. E. Use of an individually fitted compressive garment on the calf and foot . 70. All of the following statements about surgical management of gastric lymphomas are true except: A. Stage I gastric lymphomas (small lesions confined to the stomach wall) can be cured completely with surgical therapy alone. B. Extensive gastric lymphomas that initially are treated with radiation and/or chemotherapy occasionally perforate during treatment and require secondary resection. C. Patients explored with a presumptive diagnosis of gastric lymphoma should undergo an attempt at curative resection when this is safe and feasible. D. Without a preoperative diagnosis resection for gastric mass should not be attempted unless lymphoma can be excluded. E. Appropriate staging for primary gastric lymphoma includes bone marrow biopsy. 71. In the treatment of cleft lip and cleft palate A. Both should be repaired at the age of two years. B. The object with cleft lip is to obtain closure regardless shortening the upper lip. C. The object with cleft palate is to achieve adequate speech and dentition. D. The repair in cleft palate means suturing the pharyngeal layer only after suitable tension-releasing procedures. E. If the premaxilla is unfused and juts out it has to excised before repair can be carried out.
72. Correct statements concerning the use of skin grafts include: A. Skin from the inner aspect of the upper arm is appropriate for covering wounds over the anterior aspect of the tibia. B. Skin from the postauricular area is ideal when a graft must be placed on the eyellid C. Skin from the upper eyelid may be harvested with underlying orbicularis muscle for a full thickness graft of the contralateral lid . D. When a skin graft is required for an extensive facial wound the graft should be meshed before placement.
668
E. When skin is removed from the scalp, the donor area heals slowly, and hair follicle infection is common 73. Follicular carcinoma of thyroid gland: F.
Is the commonest tumor
G. Can be diagnosed by F.N.A.B H. Usually multifocal I.
Commonest tumor of young age
J.
Prognosis is worse than papillary carcinoma
74. Which of the following is true regarding pancreatic trauma and its complications ? A. Complications of pancreatic trauma include pancreatic pseudocyst, abscess, hemorrhage, and fistula. B. Asymptomatic pseudocysts may be managed conservatively, as approximately 80% will spontaneously resolve . C. Pseudocysts which do not spontaneously resolve can be managed with percutaneous drainage, if ductal disruption is identified. 75.A 23-year-old male presents to the emergency depart ment after being involved in a road traffic accident. On physical examination, he opens his eyes to painful stimulation, he occasionally mumbles incomprehen sible sounds, he localizes to painful stimulation with his right upper extremity, and he withdraws his left upper extremity to pain. His pupils are 4 mm bilaterally and reactive. What is this patient’s Glasgow Coma Scale (GCS) score? A. 7 B. 9 C. 8 D. 10 76.Management options for suprarenal inferior vena cava (IVC) injuries include all except: A. ligation B. lateral venorraphy C. spiral saphenous vein graft D. extraanatomic bypass E. panel graft using saphenous vein
669
77. A 40-year-old electrical lineman is injured on a electric pole and is brought to the emergency department unconscious. He has an area on his left hand that is charred, evidence of thermal injury to the arm, and an exit wound on his right knee. Which of the following interventions is least appropriate in the emergency department? A. escharotomy of the left arm B. CT scan of the abdomen and pelvis C. Endotracheal Intubation (ET) intubation D. CT scan of the head and neck E. measurement of serum creatinine phosphokinase levels 78. 24 year old woman who has ulcerative colitis admitted because of of fever,bloody diarrhea and abdominal tenderness.medical treatment including parental steroid is started.during the next several days she becomes increasingly ill,an abdominal x ray shows a transverse colon diameter of 12 cm but no free intraperitoneal air.the treatment of choice would be: A. B. C. D.
Cecostomy Transverse colostomy Transverse colectomy Total proctocolectomy
79. Pleomorphic salivary adenoma a. Presents as a slowly growing painless lump b. Never presents with facial nerve weakness c. Treatment includes radiotherapy d. Sentinel lymph node mapping is recommended e. The minimum operation is a superficial parotidectomy 80. A 56-year-old woman has rectal cancer found on colonoscopy performed for evaluation of rectal bleeding. The optimal method for assessing whether preoperative combined modality therapy is indicated in this patient would be A. B. C. D. E.
670
endorectal ultrasound (EUS) physical examination computed tomography (CT) positron-emission tomography (PET) endoscopic biopsy
81. A 23-year-old man is scheduled for splenectomy 1 year after diagnosis of idiopathic thrombocytopenic purpura (ITP). His platelet count is 20,000/mm3. A course of corticosteroid therapy produced no response. He is HIV negative. Which of the following BEST predicts that he will have a favorable response to splenectomy? A. B. C. D. E.
Age Response to glucocorticoids HIV status Platelet count Time since diagnosis
82. Transjugular intrahepatic portosystemic shunt (TIPS) is indicated for A. B. C. D. E.
refractory variceal bleeding Caroli's disease primary prophylaxis of variceal hemorrhage prehepatic portal hypertension correction of hypersplenism and thrombocytopenia
83. all of the following conditions are known to have a familial pattern associated with a predisposition to Cancer except: A. B. C. D.
colonic polyposis breast cancer peutz-jeghers syndrome gardner’s syndrome
84.The intense pain associated with a a felon occurs because of A. B. C. D.
bone involvement a closed space infection digital artry thrombosis nail bed involvement
85. A 47-year-old woman has acid reflux that has not responded to therapy with proton pump inhibitors. Her body mass index (BMI) is 43 and she is hypertensive and diabetic. Upper endoscopy reveals grade 1 esophagitis. Esophageal manometry shows good progression of peristalsis with normal lower esophageal sphincter (LES) tone and relaxation. The procedure MOST likely to help this patient overall would be A. B. C. D. E.
671
total fundoplication partial fundoplication gastric bypass endoscopic antireflux procedure vertical banded gastroplasty
86. the most common early complication of a Roux-en-Y gastric bypass is A. B. C. D. E.
Wound de hiscence Pulmonary embolism Anastomotic stricture Small bowel obstruction due to internal hernia vomiting
87. A 45-year-old woman with active Crohn's proctitis has a symptomatic anterior transphincteric ano-vaginal fistula. The most appropriate management would be A. B. C. D. E.
a draining seton saucerization fibrin glue a cutting seton an endorectal advancement flap
88. Obesity is defined as a body mass index greater than or equal to A. B. C. D. E.
25 30 35 40 45
89. One of the following is NOT seen in phlegmasia cerulean dolens A. B. C. D. E.
Pain Edema Blanching Cyanosis Pallor
90. when gallstone ileus occurs,obstruction is most frequent in A. B. C. D. E.
The duodenum Ileocacal junction The jejunum The proximal ileum The terminal ileum
91. The most common melanoma in dark skinned people is:
672
A. B. C. D.
Acral lentiginous Superficial spreading Nodular Lentigo maligna
92. Initial treatment for sever early dumping after gasstrectomy is A. B. C. D.
Expectant management Oral glucose for symptoms Ocreotide Surgical conversion to ROUX-en-Y drainage
93. Which of the following statements about anal fissures is NOT true? A. Nonoperative treatment with increased fluid intake, fiber ingestion, and stool softeners is the best initial management B. Topical nitrates are only marginally effective at healing anal fissures C. Topical calcium channel blockers are more effective than oral calcium channel blockers at healing anal fissures D. Recurrence is common after initial successful treatment with botulinum toxin E. Lateral internal sphincterotomy should be recommended only after topical nitrates, calcium channel blockers, or botulinum therapy has failed 94. A 51-year-old man has intermittent bloating and epigastric pain associated with eating fried foods. Abdominal examination is unremarkable. Right upper quadrant ultrasound examination reveals a 6-mm fixed, solitary projection in the gallbladder lacking an acoustic shadow. The preferred management at this time would be A. B. C. D. E.
repeat ultrasound study in 3 months biliary excretion, eg, HIDA, scan endoscopic ultrasound study laparoscopic cholecystectomy open cholecystectomy
95. Which o following herniasf develops at the level of the arcuate (semicircular) line? A. B. C. D. E.
Spigelian hernia Lumbar hernia Trocar hernia Obturator hernia Richter's hernia
96. Preferred management of Toxic megacolon is: A. Abdominal colectomy B. Abdominoperineal resection
673
C. Restorative ileoanal anastomosis D. Cecostomy E. Stricturoplasty 97. Carcinoma of the bladder: A. Is primarily of squamous cell origin. B. Is preferentially treated by radiation. C. May be treated conservatively by use of intravesical agents even if it invades the bladder muscle. D. May mimic an acute UTI with irritability and hematuria. E. Is preferentially treated by partial cystectomy. 98. - A 45-year-old man with hepatitis C undergoes an uneventful percutaneous liver biopsy. About 6-weeks later, he complains of RUQ pain, is clinically jaundiced, with a hemoglobin of 9.2 mg/dLand is fecal occult blood positive. Which diagnosis best explains this patient’s symptoms? A. Hepatocellular carcinoma B. Chronic hepatitis C C. Colon carcinoma with liver metastasis D. Hemobilia E. Symptomatic cholelithiasis 99. A66-year-old man with obstructive jaundice is found on ERCP to have periampullary carcinoma. He is otherwise in excellent physical shape and there is no evidence of metastasis. What is the most appropriate treatment? F. G. H. I. J.
A. Radical excision (Whipple procedure) where possible B. Local excision and radiotherapy C. External radiotherapy D. Internal radiation seeds via catheter E Stent and chemotherapy
100. The following statement is true concerning necrotizing fascitis EXCEPT: A. Mortality rates as high as 40% can be expected B. The infection involves only the superficial fascia, sparing the deep muscular fascia C. An impaired immune system is a common factor predisposing to this condition D. The infection is usually polymicrobial E. Necrotizing fascitis is most likely to develop in the face of impaired fascial blood supply
674
GOOD LUCK Surgical Infection Wound Healing , Postoperative Complications , Hernia 1. Staphylococcal infections: A. Do not cause cellutitis. B. Do not produce septicaemia. C. Do not produce fever. D. Produce yellow odourless pus. E. A&C only 2. Streptococcal infections: A. Are characterised by abscess formation. B. Rarely produce lymphadenitis. C. Do not produce baceraemia. D. Can produce a gangrenous skin infection. E. All of the above. 3. Tatanus prophylaxis in a patient with a badly contaminated wound:(all correct except one) A. Depends, even in an actively immunised patient, on meticulous immediate debridement of the wound. B. Should include the administration of tetanus toxoid. C. Is more safely achieved with human antitoxin. D. Is unnecessary in patients who have been recently actively immunised. E. Is necessary for the actively immunised patient to have a booster dose of toxiod. 4. Tetanus: A. May have an incubation period of over 20 days. B. Can be prevented by the immediate administration of tetanus toxoid. C. Is more common after scalp lacerations than wounds of the extremities. D. Is usually associated with stupor or coma. E. B&C only. 5. In clostridial infections: A. A spreading cellutitis may be present. B. Gram-positive cocci can be isolated from the discharge. C. Surgical treatment has a minor part to play. D. Gas production is often present. E. C&D only. spreading cellulitis is usually due to Streptococcus pyogenes 6. Actinomycosis is characterised by: A. Chronic abscesses of the foot. B. A granulomatous abcess. C. Red coloured pus. D. Its resistance to antibiotics. E. Blake coloured pus 7. A subphrenic abscess: A. Is usually accompanied by consederable systemic effects. B. Is associated with local rib tenderness. C. Rarely produces abnormal signs in the chest. D. Usually can be diagnosed by a barium meal examinations. E. A&C only.
675
8. A pelvic abscess: A. Lies extrapeitoneally. B. May be a complication of abdominal surgery. C. Rarely presents with diarrhoea. D. Should be treated with antibiotics alone. E. A&C only 9. Paralytic ileus: A. Is associated with electrolyte imbalance. B. May be associated with mechanical intestinal obstruction. C. Requires treatment with nasogastric suction and intravenous fluids. D. Is associated with retroperitoneal haematoma E. All of the above. 10. Acute postoperative gastric dilatation: A. Usually cause postoperative vascular collapse. B. Can be prevented by regular nasogastric aspiration. C. Characteristically occurs on the first postoperative day. D. Is a relatively common problem after surgery on the gastrointetinal tract. E. A&D only. 11. The appearance of jaundice in the postoperative period: (all correct except one) A. May indicate an intraperitoneal haemorrhage. B. Is usually due to the toxic effects of anaesthetic agents. C. May be due to septicaemia. D. May indicate chronic liver disease. E. May be due acute liver insalt 12. A clean incised skin wound: A. Undergoes an inflammatory phase during the processes of repair. B. Commences epithelialization after 7 to 10 days. C. Regains the full strength of normal skin within 10 days. D. Regains its strength as the result of fibroblast activity. E. A&D only. 13. Wound healing: A. Is impaired in anaemic patients. B. Is impaired by haematoma formation. C. Is not impaired by hypoproteinaemia. D. Is stimulated by steroids. E. A&D only. 14. The principles of wound care include: (All except one) A. Early skin cover. B. Removal of foreign material. C. Routine administration of antibiotics. D. Close apposition of uninfected wounded tissues. E. Wound toilet if possible. 15. Heavily contaminated and dirty wounds: A. Require surgical toilet and delayed closure. B. Require the administration of systemic antibiotics. C. Can usually be treated by wound toilet and primary closure. D. Should be totally excised.
676
E. A&B only. 16. surgical drainage of abscesses: A. Should be via a small incision with minimal disturbance of the adjacent tissues. B. Should be dependent wherever possible. C. Has been outmoded by antibiotic therapy. D. Should be undertaken before the signs of fluctuation appear. E. Usually performed under local anaethesia. 17. In localised surgical infections: A. An elevated leucocyte count is usually present. B. Fever and tenderness are usually present. C. The presence of glycosuria usually indicates metastatic pancreatic abscesses. D. Pus is frequently absent. E. A&B only. 18. A distal small bowel fistula: A. Loses intestinal fluid rich in potassium and sodium. B. May give rise to a metabolic acidosis. C. Rapidly results in dehydration. D. May be managed conservatively by the abandonment of oral feeding. E. All of the above. 19. A pancreatic fistula: A. Loses fluids with high potassium and lower sodium levels than plasma. B. May give rise to a metabolic acidosis. C. Requires long term treatment with intravenous normal saline. D. Will usually close if oral feeding is temporarily suspended. E. All of the above. 20. Bed sores: A. Can be prevented by sheepskin blankets. B. Can be prevented by changing the patient’s position four times each 24 hours. C. Are the consequence of local ischemia. D. Only occur over the sacrum.
E. A&B only. 1 2 3 4 5
D D D A A
6 7 8 9 10
B B B E B
11 12 13 14 15
Surgical MCQs Exam For Internship Students
1.Regarding carpal navicular (scaphoid) fracture, which is false? a. Nonunion is common b. The hairline fracture is seen readily on X-ray c. Pain is over the snuff-box area d. Prolonged immobilization should be tried
677
A E B C E
16 17 18 19 20
B E E B C
e. Fragment excision is the last resort 2. Fever early in post operative period is due to all except a. Deep vein thrombosis b. Atelectasis c. Urinary tract infection d. Wound infection e. Preoperative cause 3. Intussusception: ( the following is not true ) a. hypertrophied Peyer's Patches. b. Invagination of proximal bowel into an adjacent part of bowel. c. Polyp, Meckel's diveticulum or lymphoma present in all cases. d. Most common age is 5 – 10 months. e. Ileocolic type accounts for – 90% of cases. 4. Testicular torsion in a 15-year old boy: a. Presents with dysuria b. Must be confirmed by ultrasound c. Is associated with a vaginal hydrocele d. Must be treated within four hours of onset of symptoms e. Presents with hematuria 5. In radical mastectomy (of Halsted) the following structures are preserved. a. Axillary vessels. b. Nerve to serratus anterior muscle. c. Nerve to latissimus dorsi muscle. d. All of the above. e. None of the above 6. Priorities in trauma management are: a. Good ventilation, patent airway, stop arterial bleeding and look for mental status b. Mental status, patent airway, stop bleeding and good ventilation. c. Stop arterial bleeding, patent airway, good ventilation and mental status. d. Patent airway, good ventilation stop arterial bleeding and mental status. 7. The statement below regarding ileus are all true except a. abdominal distension usually noted on physical exam b. usually responds to supportive measures c. treated with bowel stimulus [ eg. Neostigmine] d. characterized by hypoactive bowel sounds e. characterized by intermittent spasm of pain with relatively pain free periods 8. The following are systemic causes of uveitis except: a. Ankylosing spondylitis b. Sarcoidosis c. Rheumatic heart discose d. Ulcerative colitis e. Behcet's discos : The most frequent complication of amebiasis is .9 a. Lung abscess b. Liver abscess c. Convulsions and coma d. Hemolytic anemia and bone marrow suppression
678
e. Retinal detachment Common reactions to whole blood transfusions include each of the following, .10 :except a. Fever b. Hemolysis c. Urticaria d. Hypercalcemia e. Hypocalcemia 11.Which of the following skull fractures always need surgery? a. Fissure fracture. b. Closed depressed fracture c. Compound depressed fracture d. A and C are true e. A and B are true 12. The major cause of impaired wound healing is a. Anemia b. Diabetes mellitus c. Local tissue infection d. Malnutrition e. Steroid use 13. Complications of plaster of paris are: a. Dermatitis b. Volkmann's contracture c. Compartment syndrome d. A and C only e. All of the above 14. Which of the following statements concerning hormone receptors of the breast is true? a. About 60% of breast cancer have receptors for estrogen. b. Cancer breast can be regress if the patient deprived of estrogen. c. Cancer breast get more active under the influence of estrogen. d. All of the above. e. A and C only 15.The radiolucent stone a. Pure uric acid stone b. Infection stone c. Cystine stone d. Calcium containing stone. e. None of the above. 16. In massive haemothorax. a. about 500ml of blood in pleural cavity b .cause dyspnea & Neck veins distension c. treated by thoracotomy tube only d. All of the above e. None of the above
679
17. The best way to relieve pain for an individual with a large area burn is by: a- intramuscular meperidine b- intravenous morphine c- nitrous oxide inhalation d- oral ibuprofen e- subcutaneous Dilaudid 18.The classical picture of acute arterial embolism include all the following except: a. Pallor b. Pain c. Parasthesia d. Palpable peripheral pulses e. Cold skin 19- Which of the following conditions is associated with a greater than normal incidence of testicular tumor? a. Torsion of the spermatic cord b. Hydrocele c. Mumps orchitis d. Cryptorchidism e. Recurrent inguninal hernia 20.The most important diagnostic modality of undescended testis is: a. Ultrasound. b. CT. Scan c. Magnetic Resonance Image ( MRI ) d. Clinical examination. e. Radio-Isotop Scan. 21 . The line of division between right and left lobes of liver is a. the plane of falciform ligament b. the plane connecting gall bladder fossa and inferior vena cava c. the plane of ligamentum teres toinferior vena cava d. at the caudate lobe e. between segments two and four 22. Major burn is defined as one of the following except. f. Second and third degree of greater than 15% in children g. Second and third degree of greater than 20% in adults less than 50 years h. Third degree burn greater than 5% i. Inhalation injury j. Significant electrical injury. 23. Colle's fracture a. Usually affect young females b. The cause is direct trauma to the wrist region c. Has a dinner-fork like deformity d. Is a fracture of proximal end of radius e. None of the above
680
24. Prophylaxis of deep vein thrombosis (DVT) and pulmonary embolism (PE) includes all of the following EXCEPT one. a. b. c. d. e.
Elevation of the lower limbs and avoid early mobilization from bed. Encourage early mobilization from the bed Use of heparin in prophylactic dose before operation. Use of oral anticoagulants few days before operation. Use of low molecular weight heparin.
25. Corneal reflex is function of: a. trigeminal nerve only. b. Facial nerve only. c. Trigeminal and facial nerves. d. Occulomotor and facial nerves. e. All are false. 26. one of the following answers is false regarding inguinal hernias: a. Indirect inguinal hernias pass through the deep inguinal ring. b. An indirect inguinal hernias passes lateral to the inferior epigastric artery. c. An indirect hernia, if large may reach scrotum. d. A direct hernia lies within the covering of spermatic cord. e. An indirect hernia may be congenital origin. 27.Charateristically in acute appendicitis: a. The tongue is clean. b. Rigidity can be absent if the appendix is pelvic in position c. Vomiting always precedes pain d. The pain begins in the RIF. e. Conservative treatment is the rule. 28.Causes of Atelctasis in post operative period f. Pain from abdominal wound g. Inadequate analgesia h. No post opetative chest physiotherapy i. Upper abdominal incision j. All of the above :Regarding perianal abscess all are true except .29 a. Commence in the anal wall b. Has the same etiology as ischiorectal abscess c. Are effectively treated by antibiotics d. May result in fistula in ano e. May cause necrotizing fascitis 30.The posterior urethra consist of: a. Prostatic urethra. b. Prostatic urethra and membranous urethra. c. Membranous urethra and bulbous urethra. d. Penile urethra e. Penile urethra and bulbous urethra
681
31.The most common type of prostate cancer: a. Adenocarcinoma. b. Sarcoma c. Carcinosarcoma d. Transitional cell carcinoma e. Lymphoma 32. Sutured wounds a. Heals by secondary intention b. Less liable for infection c. Heals with minimal scar d. All the above e. None of the above 33. Which is the most common swelling in the posterior triangle of the neck? a. Enlarges lymph nodes. b. Cervical rib. c. Cystic hygroma. d. Pharyngeal pouch. e. brachial cyst 34.Indication of tracheostomy is: a. To bypass of an upper airway mechanical obstruction. b. The relief of lower secretional air way obstruction. c. Elective in laryngeal surgery. d. All of the above e. None of the above. :Pyloric channel ulcers are considered to be .35 a- Gastric ulcers in behavior b- Duodenal ulcers in behavior c- Malignant ulcers d- A form of gastritis e- None of the above 36. The commonest organism of hand infection is: a. Streptococci. b. Staphylococci. c. Pseudomonus pyocynas organisms. d. Bacillus proteus. e. pneucocci 37. Manifestations of pyloric stenosis is not: a. Hypochloremic metabolic alkalosis. b. Bilious vomiting c. Mass located above and to the right of the umbilicus. d. After vomiting, infant is hungry and wants to feed again e. Jaundice occurs in 5% of infants with pyloric stenosis. 38. Charcot's triad include a. Fever, jaundice, septic shock b. Pain jaundice, rigors c. Rigors, change in level of conciousnes, jaundice
682
d. Pain, jaundice, fever e. Change in level of conciousness, fever, pain 39-In patients with neoplastic obstruction of the common bile duct: a. The gallbladder is never distended b. The gallbladder is rarely distended c. The gallbladder is frequently distended d. Tenderness is rarely present e. The prognosis is generally excellent 40.In myopia the following are correct except one: a. The image of object is refracted behind the retina. b. The eye is usually larger than normal c. The error is corrected by concave lenses. d. Difficulty of vision for near objects less than distant objects e. The most common refractive error. 41. Which is the MOST commonly injured intra- abdominal organ in blunt trauma ? a. liver b. kidney c. spleen d. stomach e. colon 42. regarding cancer colon which of the following is not true : a. ulcerative colitis is a risk factor. b. adenomatous polyps are a risk factor . c. Ducke's A group have 5 year survival rate about 50%. d. about 70%of colorectal cancer are located in the rectum . e. familial polyposis coli patient's are at high risk. 43.A swallowed foreign body is removed by: a. Tracheostomy. b. Tracheoscopy c. Oesophogoscopy d. Bronchoscopy e. All of the above 44.The most common cause of Hyponatremia in surgical patient is a. Fluid over load b. Fluid restriction c. Diarrhea d. Acidosis e. All of the above 45. The treatment of severe hypokalemia i.e. potassium level below 3 meq/l is: a. The patient given food rich in potassium. b. The potassium is given via the i.v. fluid without monitoring. c. The potassium should be replace via a central line and ECG monitoring of the patient is needed. d. The potassium given as tablets orally. e. The potassium given via the the i.v. fluid rapidly
683
46.Complication after spleenectomy include a. b. c. d. e.
Injury of head of panceas Overwhelming post splenic infection Duodenal fistula Injury to long gastric arteries Thrombocytopenia
47. All the following are complications of blood transfusion except : a. congestive heart failure b. bacterial infection c. thrombophlebitis d. coagulopathy e. hypokalemia 48. Glascow comma score (GCS) is not useful for: a. Evaluation of comatose patient b. Evaluation of pupils response c. Follow up of head trauma patients d. Determination of the severity of head injury e. Admission of head truma patients to ICU The risk of developing deep vein thrombosis ( DVT ) after .49 surgery is not altered if the patient has which of the following a. b. c. d. e.
A history of DVT. Varicose veins . Had a cancer operation . Used oral contraceptives. Ulcerative colitis .
50. Complications of truncal vagotomy and pyloroplasty include all of the following EXCEPT: k. dumping syndrome. l. recurrent ulcer. m. diarrhea. n. alkaline reflux gastritis. o. steatorrhea. 51.The following are systemic causes of bilateral diffuse eye lid swellings a. Acute glomorulonephritis b. Congetive heart failure. c. Angroneurotic oedema. d. Chalazion e. Nephritic syndrome 52. Colon cancer can causes all except f. Change in bowel habbit g. Elevtion of CA 19.9 level h. Fecal occult blood +ve i. Elevation of CA 15.3 level j. Iron deficiency anaemia
684
53. Fistula in ano (anal fistula) is usually secondary to: a. Third degree hemorrhoids. b. A complication of fissurectomy operation. c. Almost always after Perianal suppuration and drainage or spontaneous rupture of Perianal abscess. d. All of the above. e. None of the above :The most common cause of colon obstruction is -54 a. Adhesions b. Hernias c. Diverticulitis d. Neoplasm e. Volvulus 55.Colour of tympanic membrane is: a. White b. Gray c. Pearl grey d. Dirty white e. Yellow 56. A 40-year-old man is hit by a car and sustains an injury to the pelvis . Which of the following is most indicative of a urethral injury ? a. Hematuria . b. Scrotal ecchymosis . c. Oliguria . d. High –riding prostate on rectal examination . e. IVP showing dye extravasation in the pelvis . 57. Which of the following findings suggests that shock in an injured patient may have a cause other than hypovolemia: f. hypotension g. distended neck veins. h. decreased skin temperature. i. diminished pulse pressure. j. falling central venous pressure. 58. All of the following are ranson criteria for evaluation of pancreatitis except a. Age b. Glucose level>200 mg/dl c. Fluid sequestration >6liters d. SGOT value above 250IU/L e. Serum amylase >150U/L 59. Which of the following is correct about the inguinal canal. a. The posterior wall is formed from the external oblique muscle. b. It is oblique intramuscular split of about 4 cm in the lower part of the abdominal wall. c. The internal (deep) ring is situated above the pubic tubercle. d. It extent from the anterior superior iliac spine to the pubic tubercle e. The interior wall is formed mainly by internal oblique muscle.
685
60. Thyroid cancer a. follicular tumor is the most common b. thyroid lymphoma is usually treated by total thyroidectomy c. papillary tumor is uncommon thyroid tumor d. Medullary tumors cause raise of thyroglobulin level e. Men type II is associated with Medullary tumor Key Answer for surgical MCQs 1 2 3 4 5 6 7 8 9 10 11 12 13 14 15 16 17 18 19 20
B A C D D D E C B D C C D D A E B D D D
21 22 23 24 25 26 27 28 29 30 31 32 33 34 35 36 37 38 39 40
B C C A C D B E C B A C A D B B B B C A
41 42 43 44 45 46 47 48 49 50 51 52 53 54 55 56 57 58 59 60
C C C A C B E B B E D D C D C D B E B E
Trauma & Burns 1. When determining the depth of a burn: A. A knowledge of the type of injury is important. B. The presence of blisters is of no clinical significance. C. Impairment of sensibility of the burned area denotes full thickness burn. D. The presence of severe pain denotes a full thickness skin loss. E. A&C only. 2. Estimation of the area of a burn: ( all correct except one) A. Is of very little clinical significance. B. Provides important prognostic information. C. Is an important factor in the estimation of the fluid required. D. Can be based on a formula which states that the adult trunk is 36 per cent of the whole body surface area. E. Should be recorded in each chart of burned patient.
686
3. Patients with major burns: A. Are in a negative nitrogen balance. B. Have normal calorie requirements. C. Do not generally become anaemic. D. Are resistant to septicaemia. E. All of the above. 4. The catabolic response to trauma and infection is characterized by: A. An increase in lean body mass. B. A positive nitrogen balance. C. Gluconeogenesis. D. A falling haemoglobin level. E. A&B only. 5. The catabolic response to trauma: A. Is related to the severity of the trauma. B. Is accompanied by decreased urinary losses of potassium and nitrogen. C. Can be prevented by parenteral nutrition. D. Does not occur in the adrenalectomised patient. E. C&D only. 6. Scalds: A. Are more frequent in children. B. Commonly cause full thickness skin loss. C. Should be skin grafted within 48 hours of the injury. D. Need routine antibiotic treatment. E. All of the above. 7. The dressing of a small burn should be: A. Occlusive. B. Non-absorbtive. C. Non-compressive. D. Changed daily as a routine. E. All of the above 8. A partial thickness burn: A. Usually requires grafting. B. May deteriorate into full thickness skin loss. C. Rarely causes severe physiological derangement of the patient. D. Heals within 7 days in the absence of infection. E. C&D only. 9. Fluid losses in a major burn: A. Are maximal between 12 and 24 hours after the injury. B. Are related to the age of the patient. C. Are not related to the weight of the patient. D. Are related to the area burnt. E. Are not related to the burnt duration. 10. The increased fluid requirements of a patient with a full thickness burn are due to: (all correct except one) A. Increased evaporative water loss. B. Sequestration of fluid in the injured tissues. C. Serum exuding from the burned area.
687
D. Destruction of blood in the skin vessels. E. Vomiting & paralytic ileus occasionally 11. 48 hours after a major burn and with satisfactory fluid therapy a patient: A. Has very few abnormal fluid losses. B. May need a blood transfusion. C. Is often hypornatraemic. D. Usually needs skin grafting. E. C&D only. 12. Major burns are sometimes complicated by: (all correct except one) A. Acute gastric and duodenal ulcers. B. Paralytic ileus. C. Cerebral oedema. D. Mesenteric thrombosis. E. Septicemia. 13. If burned patient, associated pulmonary injury: (all correct except one) A. Should be suspected in head and neck burns. B. Should be suspected when the nasal hairs are burnt. C. Does not appear clinically in the first 24 hours. D. Should be suspected if burns occurred in closed space. E. May require ICU care. 14. Secondary infection of burns: A. Is less common in partial than in full thickness skin loss. B. Is relatively more common in burns of more than 20 per cent body area. C. Is avoided by leaving the burn eschar intact. D. Is avoided by the immediate application of a sterile occlusive dressing. E. All of the above. 15. The early management of a burn wound may include: A. Early excision. B. Occlusive dressings. C. Exposure treatment. D. Dressings with local antibiotics. E. All of the above. 16. Skin grafting of a burn wound: A. Should usually be with full thickness skin grafts. B. Is more likely to be successful if undertaken in the first week after injury. C. Will be unsuccessful unless the wound surface is sterile. D. Minimizes scar contracture. E. A&C only. 17. The prognosis of a burned patient is: A. Not related to the patient's age. B. Related to the area burnt. C. Generally better below the age of 10 years. D. Very poor in the patient with burns of over 40 per cent surface area. E. C&D only. 18. Nasotracheal intubation: A. Is preferred for the unconscious patient without cervical spine injury. B. Is preferred for patients with suspected cervical spine injury. C. Maximizes neck manipulation.
688
D. Is contraindicated in the patient who is breathing spontaneously. 19. Which of the following statements about head injuries is/are false? A. The majority of deaths from auto accidents are due to head injuries. B. Head injury alone often produces shock. C. A rapid and complete neurologic examination is part of the initial evaluation of the trauma patient. D. Optimizing arterial oxygenation is part of initial therapy. 20. Which of the following statements about maxillofacial trauma is/are false? A. Asphyxia due to upper airway obstruction is the major cause of death from facial injuries. B. The mandible is the most common site of facial fracture. C. The Le Fort II fracture includes a horizontal fracture of the maxilla along with nasal bone fracture. D. Loss of upward gaze may indicate either an orbital floor or orbital roof fracture. The anatomical location of the nose makes it the most common site of facial trauma 21. What percentage of patients with thoracic trauma require thoracotomy? A. 10%–15%. B. 20%–25%. C. 30%–40%. D. 45%–50%. 22. Hemorrhage initiates a series of compensatory responses. Which of the following statement(s) is/are true concerning the physiologic responses to hemorrhagic shock? A. An immediate response is an increased sympathetic discharge with resultant reflex tachycardia and vasoconstriction B. Trans capillary refill is a response serving to restore circulating volume C. Extracellular fluid becomes increasingly hyperosmolar D. Adrenergic ally mediated vasoconstriction is well maintained at the arteriolar and precapillary sphincters 23. Which of the following steps is/are part of the primary survey in a trauma patient? A. Insuring adequate ventilator support B. Measurement of blood pressure and pulse C. Neurologic evaluation with the Glasgow Coma Scale D. Examination of the cervical spine 24. Immediate life-threatening injuries that preclude air exchange which can be treated in the field include which of the following? A. Tension pneumothorax B. Massive open chest wounds C. Sucking chest wounds D. Tracheal disruption 25. Which of the following statement(s) is/are true concerning the diagnosis of a peripheral vascular injury? A. The presence of a Doppler signal over an artery in an extremity essentially rules out an arterial injury B. Doppler examination is a valuable tool in the diagnosis of venous injuries C. A gunshot wound in the proximity of a major vessel is an absolute indication for arteriography D. Both the sensitivity and specificity of arteriography of the injured extremity approaches 100%
689
26. Which of the following statement(s) is/are true concerning the management of chest trauma? A. The majority of injuries to the chest require surgical intervention B. The posterior lateral thoracotomy is the optimal approach for emergency thoracotomy C. Either computed tomography or angiography are suitable methods for detecting aortic disruption in a patient with an abnormal chest x-ray D. Persistent bleeding associated with a penetrating injury to the chest is often due to injury to an artery of the systemic circulation 27. A 22-year-old male driving a car at a high speed and not wearing a seatbelt, leaves a road and crashes with a full frontal impact into a tree. Which of the following injury patterns may be predictable from this type of motor vehicle accident? A. Orthopedic injuries involving the knees, femurs, or hips B. Laceration to the aorta C. Hyperextension of the neck with cervical spine injury D. Diaphragmatic rupture due to marked increase in intraabdominal pressure 28. Indications for Cesarean section during laparotomy for trauma include:except? A. Maternal shock after 28 weeks gestation B. Unstable thoracolumbar spinal injury C. Mechanical limitation for maternal repair D. Maternal death if estimated gestational age is at least 28 weeks 29. Important physiologic alterations of pregnancy which may alter the injury response include: except? A. Increased cardiac output B. Expanded plasma volume C. Decreased fibrinogen and clotting factors D. Partial obstruction of the inferior vena cava 30. The treatment of patients with high-voltage electric injury differs from that of patients with conventional thermal injury with respect to the need for:? A. Fasciotomy. B. Hemodialysis. C. Amputation. D. Pulse oximetry. E. Prehospital cardiopulmonary resuscitation. 31. Characteristics of the hyper metabolic response to burn injury include: A. Elevation of core temperature, skin temperature, and core-to-skin heat transfer. B. Ambient temperature dependency of metabolic rate. C. A marked increase of blood flow to the burn wound. D. Oxidation of stored lipid as the major source of metabolic energy. 32. Which of the following statement(s) is/are true concerning the initial fluid resuscitation of a burn patient? A. Rigid adherence to the Modified Brooke formula is advised B. In general, children require less fluid than that predicted by standard formulae C. Patients with inhalation injuries require less fluid than predicted by standard formulae D. Dextrose should not be given as the primary resuscitative fluid for any age group E. Most resuscitative formulae withhold colloid solutions until 24 hours post-injury
690
33. Which of the following are accepted adjuncts in the management of hypertrophic scar? Except? A. Local steroid injection B. Compression garments C. Topical silicone D. Release or excision with auto grafting E. Topical platelet-derived growth factor 34. Which of the following statement(s) is/are true concerning carbon monoxide and cyanide exposure? A. A normal oxygen saturation by standard transmission pulse oximetry precludes the possibility of significant carboxyhemoglobinemia B. Most patients with cyanide exposure require administration of sodium thiosulfate C. The half-life of carbon monoxide is reduced by a factor of 5 with ventilation with 100% oxygen D. Even if fire victims are well ventilated with high concentrations of oxygen by emergency response personnel from the time of extrication, carboxyhemoglobin values are frequently greater than 10% on initial evaluation 35. Regarding tension pneumothorax, the first step in the management is: A. Obtaining a stat chest x-ray. B. Cricothyroidotomy C. Pass in an endotracheal tube D. Starting oxygen by a valve-mask device E. Chest decompression needle. 36. Regarding central facial bone injury in polytrauma victim Who has labored breathing the first step in management is: A. Perform tracheostomy B. Perform cricothyroidotomy C. Pass endotracheal tube by oral root. D. Pass endotracheal tube by nasal root E. Obtain a cross table x-ray of cervical spine. 37. A young man is admitted to Emergency department with a stab wound to upper third of his right thigh with severe bleeding,the first thing to do is: A. Apply compression to the bleeding vessel with gloved finger. B. Apply bandage and splint to the right thigh. C. Apply tourniquet on right side above the wound. D. Use clamps to control the bleeding. E. Start blood transfusion. 38. Regarding the management of polytrauma: A. Death follow a trimodal distribution. B. X-ray after primary survey should be AP cervical spine, chest and pelvis. C. Cardiac tamponade is characterized by raised B.p, a low JUP. D. Assessment of uncomplicated limb fractures should occur during the primary survey. E. A and B only. 39. Blunt injuries to the abdomen (all are correct except) A. May cause shock B. May cause peritonitis C. May cause acute gastro duodenal ulceration D. May treated conservativly
691
E. Rarely need urgent laparatomy 40. A serious intra-abdominal injury in a comatose patient may be diagnosed by: (all correct except one) A. Abdominal paracentesis. B. The observation of pattern bruising on the abdominal wall. C. Falling of heamoglobin values. D. The presence of marked abdominal distetion E. The presence of diarrhea. 41. Injuries to the urethra (all are correct except one) A. Are more common in male. B. Are often caused by road traffic accidents. C. Are easily diagnosed on intra venous pyelography. D. Require urgent surgical treatment. E. Diagnosed by retrograde urethragraphy. 42. Regarding injury to the spleen(all are correct except one ) A. Is the commonest organ injured in blunt abdominal trauma. B. Usually continues to bleed once its capsule and its pulp lacerated. C. Usually managed conservatively in polytrauma patient. D. Should be removed if there is severe laceration of its capsule. E. The risk of infection is increase after its removal. 43. Regarding urinary tract injuries(all are correct except one) A. Are usually accompanied by hematuria. B. Require urgent intravenous pyelogram. C. Involving the kidney require urgent surgery. D. May cause by RTA. E. Which demonstrate extravasations of urine from urinary bladder require surgery. 44. Patient with renal trauma A. Usually present with hematuria B. Usually require surgical management. C. May require an intravenous pyelogram. D. Often present with acute renal failure. E. B & C only. 45. Regarding traumatic rupture of the spleen (all are correct except one ) A. Frequently presents more than 7 days after the causative injury. B. Frequently presents with shoulder tip pain C. May be diagnosed by paracentesis. D. May be accompanied by other abdominal organ injuries. E. Usually manage conservatively 46. Car seat belts when properly adjusted A. Prevent injuries to abdominal organs. B. May cause small bowel injuries. C. Do not reduce the incidence of head injuries of passengers involving in RTA. D. Protect the cervical spine during sudden acceleration E. A & D only. 47. Penetrating wounds of the abdomen. A. Can be adequately explored by a probe. B. Frequently result in acquired abdominal wall hernia. C. May be managed by careful observation, laparatomy being indicated if signs of peritonitis occur. D. Can be treated conservatively if cause by gun shot.
692
E. A&D only. 48. In head trauma patient after control of air way , the first diagnostic study A. X-rays of the skull. B. CT scan of the head. C. X-rays of cervical spine D. Carotid angiography. E. Lumber puncture. 49. In abdominal trauma with signs of hypovolamic shock the first thing to do after resuscitation of the patient is A. Obtain lateral x-rays of cervical spine. B. Obtain supine x-rays of the abdomen. C. Obtain abdominal CT scan. D. Obtain an abdominal aortogram. E. Explore the abdomen. 50. An early signs of anterior compartment syndrome in the calf is: A. Absence of pulse in the foot. B. Firm calf muscle C. Foot drop D. Paraethesia between the big and second toes. E. Painful calf muscle. 1 2 3 4 5 6 7 8 9 10
E A A C A A A B D D
11 12 13 14 15 16 17 18 19 20
B D C A E D B B B B
21 22 23 24 25 26 27 28 29 30
A 31 A+C+D A+B+C 32 E A+B+C 33 A+B+C+D A+B+C 34 C D 35 E D 36 B A+B+C 37 A B+C+D 38 A A+B+D 39 E A+B+C+E 40 E TRAUMA 1-Regarding tension pneumothorax, the first step in the management is: A. Obtaining a stat chest x-ray. B. Cricothyroidotomy C. Pass in an endotracheal tube D. Starting oxygen by a valve-mask device E. Chest decompression needle.
41 42 43 44 45 46 47 48 49 50
C C C A E B C C E D
2- Regarding central facial bone injury in polytrauma victim Who has labored breathing the first step in management is: A. Perform tracheostomy B. Perform chricothyroidotomy C. Pass endotracheal tube by oral root. D. Pass endotracheal tube by nasal root E. Obtain a cross table x-ray of cervical spine. 3- A young man is admitted to Emergency department with a stab wound to upper of his right thigh with severe bleeding, the first thing to do is: A. Apply compression to the bleeding vessel with gloved finger. B. Apply bandage and splint to the right thigh. C. Apply tourniguet on right side above the wound. D. Use clamps to control the bleeding.
693
third
E. Start blood transfusion. 4- Regarding the management of polytrauma: A. Death follow a trimodal distribution. B. X-ray after primary survey should be AP cervical spine, chest and pelvis. C. Cardiac tamponade is characterized by raised B.p, a low JUP. D. Assessment of uncomplicated limb fractures should occur during the primary survey. E. A and B only. 5- Blunt injuries to the abdomen (all are correct except) A. May cause shock B. May cause peritonitis C. May cause acute gastroduodenal ulceration D. May treated conservativly E. Rarely need urgent laparatomy 6- A serious intra-abdominal injury in a comatose patient may be diagnosed by: (all are correct except one) A. Abdominal paracentesis. B. The observation of pattern bruising on the abdominal wall. C. Falling of heamoglobin values. D. The presence of marked abdominal distetion E. The presence of diarrhea. 7- Injuries to the urethra (all are correct except one) A. Are more common in male. B. Are often caused by road traffic accidents. C. Are easily diagnosed on intra venous pyelography. D. Require urgent surgical treatment. E. Diagnosed by retrograde urethragraphy. 8- Regarding injury to the spleen (all are correct except one ) A. Is the commonest organ injured in blunt abdominal trauma. B. Usually continues to bleed once its capsule and its pulp lacerated. C. Usually managed conservatively in polytrauma patient. D. Should be removed if there is severe laceration of its capsule. E. The risk of infection is increase after its removal. 9- Regarding urinary tract injuries (all are correct except one) A. Are usually accompanied by haematuria. B. Require urgent intravenous pyelogram. C. Involving the kidney require urgent surgery. D. May cause by RTA. E. Which demonstrate extravasations of urine from urinary bladder require surgery. 10- Patient with renal trauma A. Usually present with haemoturia B. Usually require surgical management. C. May require an intravenous pyelogram. D. Often present with acute renal failure. E. B & C only 11- Regarding traumatic rupture of the spleen (all are correct except one ) A. Frequently presents more than 7 days after the causative injury.
694
B. Frequently presents with shoulder tip pain C. May be diagnosed by paracentesis. D. May be accompanied by other abdominal organ injuries. E. Usually manage conservatively 12- Car seat belts when properly adjusted A. Prevent injuries to abdominal organs. B. May cause small bowel injuries. C. Do not reduce the incidence of head injuries of passengers involving in RTA. D. Protect the cervical spine during sudden acceleration D. A & D only. 13- Penetrating wounds of the abdomen. A. Can be adequately explored by a probe. B. Frequently result in acquired abdominal wall hernia. C. May be managed by careful observation, laparatomy being indicated if signs of peritonitis occur. D. Can be treated conservatively if cause by gun shot. E. A&D only. 14- In head trauma patient after control of air way , the first diagnostic study A. X-rays of the skull. B. CT scan of the head. C. X-rays of cervical spine D. Carotid angiography. E. Lumber puncture. 15- In abdominal trauma with signs of hypovolamic shock the first thing to do after resuscitation of the patient is A. Obtain lateral x-rays of cervical spine. B. Obtain supine x-rays of the abdomen. C. Obtain abdominal CT scan. D. Obtain an abdominal aortogram. E. Explore the abdomen. 16. An early signs of anterior compartment syndrome in the calf is: A. Absence of pulse in the foot. B. Firm calf muscle C. Foot drop D. Paraethesia between the big and second toes. E. Painful calf muscle. Palestinian Medical Council Urology Palestinian Board Exam Oct.,2011 :Choose the best appropriate answer in each of the following questions .…………………………………………………… :Candidate’s Name An otherwise well 13-year-old boy is admitted complaining of sudden onset severe left -12 sided testicular pain 2 hours prior to admission. He gives no history of trauma, dysuria or .frequency. On examination he is found to have a tender, high-riding testicle ?What is the most appropriate next step in this youngman’s management a. Herniography b. Scrotal Doppler ultrasound on the next available list c. FBC and U&E
695
d. Scrotal Doppler ultrasound as an emergency e. Surgical exploration of his scrotum 34. A 63-year-old male with end-stage renal disease requiring hemodialysis three times per week presents with bone pain and several pathologic fractures of the extremities. Which is the most likely electrolyte abnormality in this patient? A. hypokalemia B. hypernatremia C. hyperphosphatemia D. hypercalcemia E. hypochloremia 59. Which of the following is true regarding the Ureter: G. It is lie at the lateral edge of psoas major muscle H. It crosses into the pelvis at the level of L3 I. The internal iliac artery branches supply the middle part J. The middle part of each Ureter drain to lymph nodes associated with internal and external vessels K. The left Ureter is crossed by testicular and left colic vessels
59. Which of the following statements characterize the biology of kidney ransplantation is false? a. The rejection response is systemic. b. The rejection response is learned. c. The rejection response involves a combination of immunologic and environmental factors . d. Allotransplantation evokes a cellular immune response. e. Allotransplantation evokes a humoral immune response. 4 . A 20-year-old male sustains a stab wound to the abdomen. He undergoes an emergency laparotomy and is found to have a complete transection of the left ureter at the pelvic brim. Which technique of repair is most appropriate? A. ureteral reimplantation *B. primary reanastomosis C. transureteroureterostomy D. end cutaneous ureterostomy E. ureteronoecystotomy with Boari flap 6. Which of the following statements concerning urinary tract infections (UTIs) during pregnancy is true? A. The prevalence of bacteruria is increased in pregnant females when compared to nonpregnant females. B. The prevalence of bacteruria is decreased in pregnant females when compared to nonpregnant females. C. Sulfa preparations can be safely used during the third trimester of pregnancy *D. The incidence of acute clinical pyelonephritis is increased in pregnant women with bacteruria. E. The risk of bacteruria decreases with the duration of pregnancy
696
7. Which of the following bacteria is most commonly infected renal calculi? A. Escherichia coli B. Urea plasma urealyticum C. Staphylococcus epidermidis D. Kiebsiella pneumonia * E. Proteus mirabilis 40. Which of the following values on semen analysis is infertility? A. ejaculate volume of 3 mL B. 80% of sperm viable C. pH of 7.2 D. 60% motility * E. 5 x 1000000 sperm per ejaculate
associated with
indicative of male
35- Carcinoma of the bladder: F. Is primarily of squamous cell origin. G. Is preferentially treated by radiation. H. May be treated conservatively by use of intravesical agents even if it invades the bladder muscle. I. May mimic an acute UTI with irritability and hematuria. J. Is preferentially treated by partial cystectomy. Answer: D 36-A 28-year-old white male presents with asymptomatic testicular enlargement. Which of the following statement is true concerning his diagnosis and management? Tumor markers, b-fetoprotein (AFP) and ك-human chorionic gonadotropin (HCG) will both be of value in the patient regardless of his ultimate tissue type F. Orchiectomy should be performed via scrotal approach G. The diagnosis of seminoma should be followed by postoperative radiation therapy H. With current adjuvant chemotherapy regimens, retroperitoneal lymphadenectomy is no longer indicated for non-seminomatous testicular tumors E.
Answer: c :Regarding horse shoe kidney, all of the following are true Except (1 A. it is due to the inutero fusion of the medial ends of mesonephiric buds .B. Usually lower poles fuse in front of L4 C. it has an incidence of one in million population * D. always treated by surgery once diagnosed .E. horseshoe kidney is more prone to infection & stone formation :Microscopic haematuria (2 A. is considered pathological when the number of RBCS is at least 10 per high field in urine analysis B. is always pathological C. can be caused by glomerular or extraglomerular lesions *
697
power
D. never occurs in acute nephritic syndrome E. is insignificant when being painless :Familial benign essential hematuria is characterized by all of the following Except (3 A. occurs in families but without protienuria B. will progress ultimately to renal failure * C. associated with hearing loss D. the hallmark of this condition is usually diffuse attenuation of the glomerula basement membrane E. assurance of the patient and follow up is the main line of treatment A thirty-year old male patient presented to the urology clinic with dark urine and (4 impaired hearing and visual acuity . His BP is 170/110 mmHg, pulse rate 90/min and .++ temperature of 37 C. The urine analysis showed 25 RBCS/HPF, protein :The most probable diagnosis is A. Focal Glomerulosclerosis B. Benign Familial Hematuria :C. Thin basement membrane nephropathy D. Idiopathic Hypercalciuria (E. Hereditary Nephritis ( Alport syndrome * .:Regarding anuria (5 hours A. it is defined as absence of urine for 24 .if due to aminoglycosides It is irreversible B C. it is treated initially by fluid and blood transfusion in haemorrhagic shock * D. its diuretic phase on recovery when treated properly usually started after two weeks E. it treated initially by high fluid intake orally :polycystic kidney disease (6 A. it is always congenital B. the infantile type is autosomal dominant C. the congenital polycystic kidney disease may progress to uraemia * D. always any solitary renal cyst must be treated surgically E. fluid inside the cyst is always sterile urine :The appropriate surgical treatment for suspected carcinoma of the testis is (7 .A. transscrotal percutaneous biopsy .B. transscrotal open biopsy .C. repeated examinations D. inguinal exploration, control of the spermatic cord, biopsy, and radical .orchectomy if tumor is confirmed operatively E. transscrotal radical orchiectomy :Regarding urolithiasis, all of the following are true Except (8 A. hyperparathyroidism is usually the cause inabout 5% of cases B. urate stones are usually radiolucent C.infection and stasis of urine are among the primary causes of stone formation D. it is treated generally by fluid restriction and vitamine D excess * E. clinically can be asymptomatic :In renal stones, all are true Except (9 A .the most common type is the calcium oxalate stones B. can cause hydronephrosis C. may present clinically with pain, haematuria and pyuria D. can be treated percutanuously E. all are true*
698
*
:Ureteric colic (10 A. is typically presented by dull aching pain localized to the loin B. is usually treated by nephrostomy C. is usually caused by a stone in renal pelvis D. might be associated with haematuria * E. urgent intravenous urography is the gold standard investigation in the :Regarding the treatment in urolithiasis,all of the following is true A. High fluid intake at all times regardless the type of stones B. Avoid high Ca- content food staff C. percutanuos nephrostomy may be needed in completely obstructed D. alkalinizing the urine in phosphate containing stones * E. in Cystine stones: restriction of eggs, meet and fish in diet is
emergency situation Except (11
advisable
:Hydronephrosis (12 A. is never a manifestation of vesicoureteric reflux B. is a common presentation of PUJ (pelviureteric junction) stenosis C. can be caused by prostatic enlargement D. A and C are correct E. B and C are correct* Acute Pyelonephritis (13 A. occues more aften on the right side and traditionally used to call Honey moon Pyelonephritis B. clinically usually present as a Prodroma :headache, lassitude, nausea But temperature may reach 39-40 C C. in early cases urine analysis may be clear D. clinically might mimic acute appendicitis E. all are true* :Regarding calculous pyonephrosi, all of the following are true Except (14 A. is diagnosed by the triad of Anaema , fever , loin swelling B. is an emergency situation C. may need a nephrostomy tube as a primary treatment D. is treated by proper antibiotics according to culture and sensitivity only * E. may end with toxaemia and kidney damage :perinephric abscess (15 A. may be of tuberculous origin B. usually characterized by no local signs clinically C.may be treated by drainage under CT scan guidance D. all of the above are correct E. A and C are correct* :Normal urine analysis shows (16 A. 10 RBCS per high power field B. few hyaline cast C. acidic reaction* D. minimal proteinuria E. few calcium oxalate crystals :Regarding prostatitis (17 A. Acute bacterial prostatitis may cause toxaemia and shock B. common causative uropathogen is Escherichia coli C. negative culture in Chronic bacterial prostatitis does rule out other pathogens e.g chlamydia trichomatis D. mostly responds to quinolones E. all are correct *
699
not
:Renal Cell Carcinoma (18 A. may present with painless haematurie, flank pain, and loin mass * B. is a well encapsulated tumour C. does not spread via renal tributaries D.arise from the Cells of distal convoluted tubule E. grossly the tumour is orange in color, solid in conistancy :Transitional Cell Carcinoma of urinary bladder is characterized by the following Except (19 *A .it accounts for 30-40% of bladder malignancy B. presents with painless haematuria C. may cause hydronephrosis D. currently T1 tumours are treated by open surgery E. incidence is more in cigarette smokers, workers with dye production (naphthylamine) :Prostatic carcinoma (20 A. is the least common malignant tumour in men B. is an adenocarcinoma* C. early detection is by monitoring acid phosphatase in blood D. causes osteolytic bony lesions E. is treated always by surgery A 58-year-old woman with recurrent attacks of painless haematuria is found to have (21 an irregular filling defect in the right renal pelvis. CT confirms a solid mass. Correct :treatment of this patient is A. nephrectomy only B. chemotherapy C. radiotherapy D. nephrouretrectomy* E. none of the above :Teratoma of testis (22 A.commonly seen in the 20–30-years age group of patients B. alpha faetoprotein may be helpful in the diagnosis C. trans-scrotal biopsy is indicted to confirm the diagnosis D. A and B are correct* E. B and C are correct :(Prostate-specific antigen (PSA (23 A. is a substance the prostate gland naturally produces to help liquefy semen B. increases in infections of prostate, enlargement, PR examination and cancer of C. PSA test is a good screening test to detect cancer prostate D. A and C are correct E. all of the above are correct * :A patient with acute urinary tract infection (UTI) usually presents with .24 .A. chills and fever .B. flank pain .C. nausea and vomiting .D. 5 to 10 white blood cells per high-power field (hpf) in the uncentrifuged urine .E. painful urination* :Renal adenocarcinomas .25 .A. are of transitional cell origin .B. usually are associated with anemia .C. are difficult to diagnose
700
prostate
specimen
.D. are extremely radiosensitive .E. frequently are signaled by gross hematuria * :Carcinoma of the bladder .26 .A. is primarily of squamous cell origin .B. is preferentially treated by radiation C. may be treated conservatively by use of intravesical agents even if it invades the .muscle .D. may mimic an acute UTI with irritability and hematuria * .E. is preferentially treated by partial cystectomy
bladder
: Regarding intra-abdominal testes within the age group 10 to 35 years of age in males .27 .A. the incidence of carcinoma of the testis is equal to that in the general population B. the incidence of carcinoma of the testis is increased up to twenty times greater that in the * .general population .C. the incidence of carcinoma of the testis is decreased than that in the general population by time D. still can function normally for spermatogenesis E. none of the above is true :Aberrant renal artery .28 A. is more common on the right side B. clinically may cause hydronephrosis * C. can be cut safely without any sequalae on the kidney D. is a branch from the aorta E. is a cause of hypertension :If torsion of the testicle is suspected, surgical exploration .29 .A. can be delayed 24 hours and limited to the affected side .B. can be delayed but should include the asymptomatic side .C. should be immediate and limited to the affected side .D. should be immediate and include the asymptomatic side * E. is indicated if pain is relieved on elevation of the scrotum :Benign prostatic hypertrophy with bladder neck obstruction .30 .A. is always accompanied by significant symptoms .B. is best diagnosed by endoscopy and urodynamic studies .C. is suspected by the symptoms of frequency, hesitancy, and nocturia .D. is always accompanied by residual urine volume greater than 100 ml E. B and C are correct* :Cross mach each of the following .31-34 Adult polycystic kidney A. analgesic abuse .31 Renal papillary necrosis B. human chorionic gonadotrophin .32 testicular choriocarcinoma C.gonorrhea .33 uretheritis D.clear cell renal carcinoma .34 hypernephroma E. autosomal dominant .35 :A Varicocele (36 A. is the varicose dilatation of the veins draining the testis B. is generally asymptomatic C. is best treated by high ligation of the dilated gonadal vessles D. clinically feels like a ‘bag of worms’ and may give an impulse on cough E. all are true*
701
Hydronephrosis (37 A. is dilatation of the renal pelvis and calyces associated with progressive atrophy of the kidney B. may be unilateral or bilateral C. its treatment is treatment of the cause D. renal istope scan can determine the percentage of the remaining functioning kidney tissue *E. all are true : (Intravenous urography (IVU (38 A. currently is the gold standard first investigation for investigating painless macroscopic haematuria B. is recommended for all patients with ureteric colic during the attack C. is done routinely irrespective to the kidney function D. all are correct E. none is correct
*
A 20-year-old man has a 24-hour history of severe left scrotal pain and swelling. (3939 There is frequency of micturition and dysuria for the past few days. He has a temperature .of 39 ºC. There are leukocytes in the urine and the WBC is 50/HPF :The diagnosis is most likely A. prostatitis B. uretheritis C. epididymo-orchitis* D. seminoma of left testis E. torsion of left testis year-old man attends the emergency department with frank painless haematuria -56 (40 .and some vague abdominal pain. On examination there is a swelling in the right loin :The initial diagnosis is likely to be (A. renal cell carcinoma (hyprenephroma * B. Wilm’s tumour C. urinary bladder tumour D. hydronephrosis E, carbuncle of the kidney 41) The following investigations could be done as first step in case of renal colic except: A. Urine analysis. B. Plain abdominal film(KUB) C. Abdominal ultrasound. *D. Intravenous urography(IVU). E. renal isotope scan 42) Chronic bacterial prostatitis is best diagnosed by: A. Histological examination B. History and physical examination * C. Segmented urine and prostatic fluid cultures D. Cystoscopy. E. prostatic biopsy
702
43) The treatment of choice for distal ureteral obstructing stone is: A. Double J stent *B. Ureteroscopic lithotripsy C. ESWL D. open surgery E. conservative 44) 16- The area of the prostate from which most cancers arise is: A. Peripheral zone *B. Central zone C. Transitional zone D. Periurethral stroma E. lateral lobe ::Renal Anuria may be caused by all of the following Except (45 A. obstructive jaundice B. aminoglycosides C. Eclampsia toxcins D. mercury salts E. dehydration* :Hypercalciuria can be all of the following Except (46 A. Milk-alkali syndrome B. Vitamin D excess C. Sarcoidosis D.hypothyroidism* E. Renal osteodystrophy :Ureteric colic (47 A. typically radiates from loin to groin B. may be associated with vomiting and profuse seating c. may be caused by a stone or a clot D. A and C are correct E. all are correct* :Regarding uric acid stones, all are true Except (48 A. are formed in acidic urine B. higher incidence in patients having a diet rich in red meat and fish C. treated with with allopurinol if serum uric acid is high D. usually seen in a plain KUB * E. best detected by renal ultrasound and a contrast study Longstanding uretheral obstruction by enlarged prostate in an elderly (e.g BPH) may (49 :cause A. hypertrophy of bladder muscles B. bilateral hydrourteronephrosis C. reduced GFR and ultimately renal failure D. urine retension with overflow E. all are true* :Wilm’s tumour (50 A. its peak incidence is 10 years of age in children B. histologically composed of renal tubules only C. is a Nephroblastoma* D. treated by chemotherapy only E. does not give distant metastases
703
Palestinian Medical Council Urology Palestinian Board Exam Oct.,2011 Candidate’s Name: ……………………………………………………. Choose the best appropriate answer in each of the following questions: 1. The dorsal Lumbotomy incision to expose the Kidney : a. requires incision of latissims dorsi muscle . b. requires incision of the quadrants lumborum muscle . c. splits the lumbodorsal fascia horizontally posterior to anterior . d. splits the lumbodorsal fascia vertically without incising muscle . e. requires excision of the 12th rib . 2. 20_years_old man is under going retroperitoneal dissection for a testicular germ cell tumor . The inferior mesenteric artery is divided during reflection of the intestines to expose retroperitoneum . This can be expected to result in : a. ischemia of the descending colon . b. ischemia of the sigmoid colon . c. ischemia of the rectum . d. ischemia of the transverse colon . e. non of the above . 3. Disruption of which sympathetic nervous plexus on the anterior abdominal aorta during retroperitoneal dissection will probably cause loss of seminal emission in a male patient ? a. celiac plexus . b. renal plexus . c. superior mesenteric plexus . d. superior hypogastric plexus . e. all of the above . 4. To preserve the vascular supply to the ureter , incision in the peritoneum should be : a. medially in the abdomen and laterally in the pelvis . b. laterally in the abdomen and medially in the pelvis . c. always medial to the ureter . d. always lateral to the ureter . e. directly over the ureter . 5. When the endopelvic fascia lateral to the prostate and paboprostatic ligaments is opened , vessels are encountered that piece the lavatory ani to join the periprostatic plexus laterally . These vessels are communicating branches from : a. the pampiniform plexus of veins . b. the dorsal vein of the penis . c. the internal pudendal veins . d. the external pudendal veins . e. the accessory obturator veins . 6. All of the following features of the ureterovesical junction cooperate to to prevent vesicoureteral reflux except : a. fixation of the ureter to the superficial trigone . b. sphincteric closure of the ureteral orifice . c. destrusor backing . d. telescoping of the bladder outward the ureter . e. passive closure of intramural ureter by bladder filling
704
7. The most common cause of glomerular hematuria is : a. transitional cell carcinoma . b. nephritic syndrome . c. Berger's disease ( IgA nephropathy ) . d. post streptococcal glomerulonephritis . e. Good pasture's disease . 8. Transient proteinuria may be due to all of the following except : a. exercise . b. fever . c. emotional stress . d. congestive heart failure . e. ureteroscopy . 9. Which of the following disorders may commonly lead to irritative voiding syndrome ? a. Parkinson's disease . b. renal cell carcinoma . c. testicular torsion . d. bladder diverticula . e. prostatic cancer . 10. Risk factors associated with nephrotoxicity in patients receiving high osmolar contrast including all of the following except : a. renal insufficiency . b. diabetic nephropathy . c. hypoalbuminemia . d. multiple administrations within a short interval . e. hyperuricemia . 11. the most common cause of an acquired renal arteriovenous fistula is : a. blunt renal trauma . b. renal carcinoma . c. renal surgery . d. closed renal biopsy . e. penetrating renal trauma . 12. When renal impairment is demonstrated in patient . The first therapeutic intervention should be to : a. begin low-dose dopamine . b. administer a cardiac inotropic agent . c. restore adequate circulating blood volume . d. administer a loop diuretic . e. begin a mannitol infusion . 13. The most effective agents for preserving residual renal function in chronic renal insufficiency are : a. a blockers . b. dihydropyridine calcium channel blockers . c. ACE inhibitors . d. B_blockers . e. nitrates .
705
14. Pretransplant nephrectomy is indicated for : a. hypertension controlled with medication . b. prior renal infection . c. renal calculi unsuitable for minimally invasive procedures . d. 10 mg/dl proteiuria . e. most polycystic kidneys . 15. The most common cancer after kidney transplantation is : a. skin . b. lymphoma . c. Kaposi's sarcoma . d. renal cell carcinoma . e. carcinoma of the cervix . 16. A cadaver kidney transplant recipient has serum creatinine level of 1.9 mg/dl . A large asymptomatic perigraft fluid collection is aspirated , and the creatinine level is 2.0 mg/dl , What is the most likely diagnosis : a. hydrocele . b. lymphocele . c. urine leak . d. resolving hematoma . e. perinephric abscess . 17. A successful kidney transplant recipient and his wife desire to have a child . What is the recommended length of time between transplantation and impregnation ? a. 3 months . b. 6 months . c. 12 months . d. 24 months . e. 36 months . 18. Which of the following collagen types is associated with ureteral obstruction ? a. Type I collagen . b. Type II collagen . c. Type III collagen . d. Type IV collagen . e. Type V collagen . 19. A drug that has efficacy in managing ureteral colic is : a. bethanechol . b. prostaglandins . c. phostigmine . d. indomethacin . e. ephedrine . 20. Which of the following is not true of patients with tubo-ovarian abscess ? a. the ureteral dilatation is usually unilateral . b. the ureteral dilatation is usually bilateral . c. the ureters are usually dilated laterally . d. the most common causative organisms are usually Neisseria gonorrhea and Chlamydia trachomatis . e. patients may be asymptomatic . 21. Pathologic post-obstruction diuresis refers to post obstructive diuresis resulting from : a. impaired concentrating ability . b. changes in aquaporin expression .
706
c. increased urea . d. decreased prostaglandins excretion . e. decreased angiotensin II secretion . 22. When a psoas hitch used a structure particularly susceptible to injury the : a. obturator nerve . b. superior vesicle artery . c. psoas minor . d. common iliac vein . e. genitofemoral nerve . 23. When a ureteral substitution is being performed in patient with small intrarenal pelvis surgeon should be prepared to : a. abort the procedure and perform nephroctomy . b. place permanent nephroctomy tube . c. evaluate the patient postoperatively for vitamin B12 deficiency . d. excise the lower pole parenchyma an ileocalicostomy . e. leave a double-J stent in place for 4 months . 24. 24_years_old man is referred with persistent right flack pain . His urological history is significant in that he had undergone an open pyeloplasty 4 years earlier , with recurrence of symptoms shortly there after follow up studies at that time should persistent right UPJ obstruction subsequently managed with an attempt at a Cautery-wire balloon incision . and then percutaneous antegrade endopyelotomy , both of which failed , at this time the left kidney is normal . a CT showed moderate cortical loss on the affected ( Right ) kidney . A renogram is performed and revealed 31 % function on the affected side . and the diuretic study confirms functional obstruction . What is the next step ? a. use of a permanent internal stent changed every 3 to 4 months . b. Retrograde balloon dilatation . c. Another pyeloplasty . d. laproscopic pyeloplasty . e. ureterocalicostomy . 25. The most important bacterial virulence factor is : a. hemolysin . b. K antigen . c. Pili . d. colicin production . e. O serogroup . 26. Fournier's gangrene is not associated with scrotal : a. pain . b. discharge . c. creptation . d. erythemia . e. swelling . 27. The antibiotic of choice in interstitial cystitis : a. Doxycycline . b. Amoxicillin . c. Gentamycin . d. Ciprofloxacin . e. None .
707
28. The disorder most commonly associated with interstitial cystitis is : a. irritable bowel syndrome . b. allergy . c. lupus . d. fibromyalgin . e. incontinence . 29. Post gonococcal urathritis is most often caused by which of the following organisms ? a. Chlamydia trachomatis . b. Ureaplasma hominis . c. Mycoplasma hominis . d. Herpes simplex virus . e. Acinobacter . 30. Zoon's balanitis may clinically resemble : a. squamous cell carcinoma . b. penile melanosis . c. wart . d. hidradenitis suppurativa . e. bug bite . 31. Which of the following statements regarding tuberculosis correct ? a. Humans are not the only reservoir for the organism . b. Renal tuberculosis is usually caused by the activation of a prior-blood borne metastatic renal infection . c. Epididymitis is a rare presenting symptoms of genitourinary tuberculosis . d. Transmission of genital tuberculosis from male to female is common . e. Renal tuberculosis is most common in children younger than 5 years of age . 32. All of the following are major predisposing for renal candidal infection in pediatric except : a. indwelling intravascular catheters . b. broad-spectrum antibiotics . c. prematurity . d. low birth weight . e. maternal diabetes mellitus . 33. During physiologic filling . bladder compliance is most affected by : a. prevesical ganglia activity . b. peripheral nerve activity . c. sacral spinal lord activity . d. suprasacral spinal lord activity . e. bladder wall vesicoelasticity . 34. Which hormone increases α-adrenergic receptors density in the urethra ? a. Luteinizing hormone releasing hormone . b. Dihydrotestosterone . c. Testosterone . d. Estrogen . e. Testosterone and Estrogen . 35. Involuntary bladder contractions are most commonly seen in association with : a. Sacral spinal cord neurologic disease or injury . b. Infrasacral neurologic disease or injury . c. Suprasacral neurologic disease or injury . d. Peripheral nerve neurologic disease or injury . e. Interstitial cystitis .
708
36. A 60_years_old woman requests medical therapy for stress urinary incontinence . She had had an abdominal hysterectomy for fibroids 12 years before. She has no other medical problems, is normotensive , and notes dryness with intercourse . What should the practitioner suggests ? a. the first-line medical therapy for female stress urinary incontinence , phenylpropanolamine , 50 mg twice a day . b. the α-agonist pseudoephedrine in a long-acting formulation , once each morning c. because the patient is postmenopausal , conjugated estrogens at 0.625 mg/day . d. vaginal estrogen cream . e. b and d . 37. When devices for stress incontinence are considered , which of the following statements is most correct ? a. Metal-based products are the most likely to achieve dryness . b. Urethral stents are the easiest to use and the best tolerated . c. Urethral stents are the contraindicated in the sexually active female . d. Bladder neck support devices are ideal for patients with intrinsic sphincter deficiency . e. Most devices are advantageous in that they can be used on as-needed basis . 38. Differentiation between late maturation arrest and normal spermatogenesis on a testicular biopsy sample is best accomplished by : a. spermatid count per tubule . b. sertoli cell count per tubule . c. touch preparation . d. ratio of spermatids to sertoli cells . e. absolute spermatid count . 39. An extra X chromosome in the male : a. leads to higher risk of testicular cancer . b. is the hallmark of Klinefelter's syndrome . c. means that sperms cannot be recovered from the testicles . d. is found in sex reversal syndrome . e. has no effect on spermatogenesis . 40. In newborns with ambiguous genetalia , what does a symmetrical gonadal examination suggest ? a. congenital adrenal hyperplasia or true hermaphroditism . b. mixed gonadal digenesis or androgen insensitivity syndrome . c. congenital adrenal hyperplasia or mixed gonadal dysgenesis . d. congenital adrenal hyperplasia or androgen insensitivity syndrome . e. true hermaphroditism or mixed gonadal dysgenesis . 41. A 63-year-old male with end-stage renal disease requiring hemodialysis three times per week presents with bone pain and several pathologic fractures of the extremities. Which is the most likely electrolyte abnormality in this patient? A. hypokalemia B. hypernatremia C. hyperphosphatemia D. hypercalcemia E. hypochloremia
709
42. Which of the following statements concerning urinary tract infections (UTIs) during pregnancy is true? A. The prevalence of bacteruria is increased in pregnant females when compared to nonpregnant females. B. The prevalence of bacteruria is decreased in pregnant females when compared to nonpregnant females. C. Sulfa preparations can be safely used during the third trimester of pregnancy D. The incidence of acute clinical pyelonephritis is increased in pregnant women with bacteruria. E. The risk of bacteruria decreases with the duration of pregnancy 43. Which of the following bacteria is most commonly associated with infected renal calculi? A. Escherichia coli B. Urea plasma urealyticum C. Staphylococcus epidermidis D. Kiebsiella pneumonia E. Proteus mirabilis 44- Carcinoma of the bladder: K. Is primarily of squamous cell origin. L. Is preferentially treated by radiation. M. May be treated conservatively by use of intravesical agents even if it invades the bladder muscle. N. May mimic an acute UTI with irritability and hematuria . O. Is preferentially treated by partial cystectomy. 45-A 28-year-old male presents with asymptomatic testicular enlargement. Which of the following statement is true concerning his diagnosis and management? Tumor markers, b-fetoprotein (AFP) and human chorionic gonadotropin (HCG) will both be of value in the patient regardless of his ultimate tissue type J. Orchiectomy should be performed via scrotal approach K. The diagnosis of seminoma should be followed by postoperative radiation therapy L. With current adjuvant chemotherapy regimens, retroperitoneal lymphadenectomy is no longer indicated for non-seminomatous testicular tumors M. Regarding horse shoe kidney, all of the following are true Except .46 .N A. it is due to the inutero fusion of the medial ends of mesonephiric buds .O .B. Usually lower poles fuse in front of L4 .P C. it has an incidence of one in million population .Q D. always treated by surgery once diagnosed E. horseshoe kidney is more prone to infection & stone formation .R I.
A thirty-year old male patient presented to the urology clinic with dark urine and .47 impaired hearing and visual acuity . His BP is 170/110 mmHg, pulse rate 90/min and .++ temperature of 37 C. The urine analysis showed 25 RBCS/HPF, protein :The most probable diagnosis is A. Focal Glomerulosclerosis B. Benign Familial Hematuria :C. Thin basement membrane nephropathy D. Idiopathic Hypercalciuria (E. Hereditary Nephritis ( Alport syndrome
710
.:Regarding anuria .48 hours A. it is defined as absence of urine for 24 .if due to aminoglycosides It is irreversible B C. it is treated initially by fluid and blood transfusion in haemorrhagic shock D. its diuretic phase on recovery when treated properly usually started after two weeks E. it treated initially by high fluid intake orally :polycystic kidney disease .49 A. it is always congenital B. the infantile type is autosomal dominant C. the congenital polycystic kidney disease may progress to uraemia D. always any solitary renal cyst must be treated surgically E. fluid inside the cyst is always sterile urine :perinephric abscess .50 A. may be of tuberculous origin B. usually characterized by no local signs clinically C.may be treated by drainage under CT scan guidance D. all of the above are correct E. A and C are correct :Renal Cell Carcinoma .51 A. may present with painless haematurie, flank pain, and loin mass B. is a well encapsulated tumour C. does not spread via renal tributaries D.arise from the Cells of distal convoluted tubule E. grossly the tumour is orange in color, solid in conistancy :Transitional Cell Carcinoma of urinary bladder is characterized by the following Except .52 A .it accounts for 30-40% of bladder malignancy B. presents with painless haematuria C. may cause hydronephrosis D. currently T1 tumours are treated by open surgery E. incidence is more in cigarette smokers, workers with dye production (naphthylamine)
*VASCULAR SURGERY MCQs REVISION NOTES FOR UNDERGRADUATE IUG STUDENTS: 1-Concerning Diabetic feet,the following is true: a-APSV is more sensitive than ABI. b-Neuropathic ulcers are insensate ulcers and present mostly over the Gaiter’s area. c-Chiropody includes revascularization of ischaemic diabetic feet. d-Necrotizing fasciitis can seldom occur in diabetic patients. e-Calcified dorsalis pedis artery is seen in a plain film due to calcium ingestion by macrophages and forming ‘’foam cells’’.
711
2-In a 55-year-old grocery store cashier with an 8-month history of leg edema increasing over the course of a work day, associated with moderate to severe lower leg bursting pain, Patient had a positive Trendelenburg’s test ,Duplex showed an incompetent SFJ,which is true: a-Class One stockings are the best for such patient. b-Ambulatory Claudication occur due to arterial insufficiency. c-Ligation and stripping of the GSV is the best management for such patient. d-Sclerotherapy using Aethoxysklerol can be used efficiently in this patient. e-Aspirin . 3-Regarding Compartment Syndrome,which is not true: a-Can occur initially in a case of acute ischaemia or even after reperfusion. b-Occurs when the intracompartmental pressure is more than 30-40 cm water. c-Absence of pulse occurs initially. d-Urgent fasciotomy is mandatory. e-Amputation is the treatment of choice. 4-Regarding Arterial Embolism,choose the most correct: a-Intracardiac emboli are less common than extracardiac. b-Femoral arery embolism occurs less than brachial artery embolism c-Embolus is rather a soluble material that is attached to the arterial wall and doesn’t travel. d-AF is found in more than 2/3 of cases. e-Thrombus travels. 5-A 38-year-old male smoker with gangrenous changes in the toes of both feet has an arteriogram showing normal vessels to the popliteal trifurcation and multiple occlusions distally in small vessels. The following is not true: a. Hyperlipidemia, diabetes, and autoimmune disease must be ruled out to make the diagnosis of Buerger’s disease b. Plethysmographic is a good vascular tool in diaqgnosing digital ischaemia. c. Patients with Buerger’s disease can benefit from surgical sympathectomy. d. In contrast to the lower extremities, Buerger’s involvement of the upper extremities is more. e.Buergers occurs seldom in females. 6-Which of the following statement about arterial injuries is true? A. All arterial injuries are associated with absence of a palpable pulse. B. Preoperative arteriography is required to diagnose an arterial injury. C. In all patients with multiple trauma, arterial injuries should be repaired before other injuries are
712
addressed D. Patients with Acute limb ischaemia necessitate repair . E.Arterial spasm is a clinical diagnosis. 7- A 60 year old man develops TIA's and has a carotid duplex that shows an 80% stenosis. The most appropriate next step in management is: a. Plavix b. Cilostazol c. Carotid endarterectomy d. Nothing e.Aspirin 8-Concerning investigations of vascular diseases the following is NOT TRUE: A-In limbs with rest pain ABI is below 0.3 . B-Carotid duplex is superior than aortic arch aortography in diagnosing carotid artery disease. C-Angiography is indicated in intermittent claudications . D-Plethysmography and capillaroscopy are used in investigating digital ischaemia. E-Duplex Ultrasound shows a Triphasic flow in normal arterial tree . 9-Concerning AAA, the following is not true: A-Smoking is the strongest predisposing factor. B-Infrarenal AAA is the commonest type that accounts more than 95%. C-Rupture is highr in females. D-Diameter more than 3 cm is an indication for elective repair. E-EVAR has lower mortality rates than open repair. 10-Which of the following statements about Antiplatelet drugs is not true: A- They act effectively in arterial diseases more than venous thrombotic diseases. B- Cilostazol inhibits Phosphodiesterase. C- Clopidogrel is an ADP inhibitor. D- A common side effect is GIT bleeding. E-Pentoxifylline increases RBCs deformability
713
11-One of the following is considered a sure sign of vessel injury: a-Shock. b-Stab medial aspect of thigh. c-History of bleeding. d-Absent pulse in a fair limb circulation e-zone one neck injury. 12-A black big toe in a diabetic foot ,all are true except: a-Angiogram is mandatory if the pedal pulse is absent. b-Amputation of this toe ,with good cover of antibiotic and blood sugar control can solve the problem. c-Chiropody has an essential role in preventing such cases. d-Gangrene may be of infective aetiology . e-Ischaemia should be ruled out . 13-The first tissues to be affected by chronic impairment of blood supply to a limb are: a-Skin b-Nails and hairs. c-Muscles. d-Nerves. e-Bones. 14-Mangeled Extremity Severity Score(MESS),Amputation is indicated when MESS is: a- Less than 7. b- Above 7. c- Above 4. d- Less than 4. e- None of the above.
714
15-The commonest AVF is: a-Congenital. b-Traumatic. c-Iatrogenic for dialysis. d-Aortoduodenal fistula post AAA repair. e- None of the above.
ANOTHER GROUP OF QUESTIONS: 1-The following is NOT TRUE about ''leg ulcers'': a- Neuropathic Ulcers are insensate ulcers. b-Venous Ulcers are located mostly over the medial aspect lower leg. c-''Fibrin Cuff and Leukocyte Trapping '' theories describe the pathology of venous ulcers. d-Ischaemic ulcers may have gangrenous floor. . d-diabetic ulcers are premalignant. 2-Concerning the treatment of thrombotic vascular disease, the following is false: a-Aspirin is successful in preventing arterial more than venous thromboembolism. b-Heparin (UFH) stimulates Antithrombin III. c-Warfarin induced skin necrosis is attributed to the early stimulation of protein C and protein S. d-Hirudins is used as anticoagulants in HIT syndrome. e-Cilostazol inhibits phosphodiesterase ,and thus causing arterial wall dilatation. 3-Concerning investigations of vascular diseases the following is NOT TRUE: a-In limbs with rest pain ABI is below 0.3 . b-Carotid duplex is superior than aortic arch aortography in diagnosing carotid artery disease. c-Angiography is indicated in intermittent claudications . d-Plethysmography and capillaroscopy are used in investigating digital ischaemia. e-Duplex Ultrasound shows a Triphasic flow in normal arterial tree .
715
4-A 45 year old female patient underwent THR surgery ,3rd day postoperative her ipsilateral lower limb is swollen,painful,tender with intact peripheral pulsations,full therapeutic LMWH was initiated,2 days later she developed haematemesis ,LMWH is stopped,the next step would be: a-IVC filter insertion. b-CT Pulmonary Angiography. c-Streptokinase therapy. d-UFH administration. e-Urgent pulmonary thrombectomy. 5-A 24 year old male patient who sustained a stab to the left groin,three hours later he was presented to the E&A with a tourniquet over his groin that had been placed by his family, his BP was 100/60 ,tachycardic ,and oliguric,no external bleeding or haematomas,he was unable to move his cold limb,ipsilateral popliteal pulse was not felt,the best management would rather be: a- Left above knee amputation. b- Femoral artery exploration. c- Urgent angiogram. d- Urgent arterial duplex. e-Anticoagulation. 6-Which of the following statements about Antiplatelet drugs is not true: A- They act effectively in arterial diseases more than venous thrombotic diseases. B- Cilostazol inhibits Phosphodiesterase. C- Clopidogrel is an ADP inhibitor. D- A common side effect is GIT bleeding. E-Pentoxifylline increases RBCs deformability . 7-Concerning AAA, the following is TRUE: A-Smoking is the strongest predisposing factor. B-Suprarenal AAA is the commonest type that accounts more than 95%. C-Rupture is highr in males. D-Diameter more than 3 cm is an indication for elective repair. E-EVAR has higher mortality rates than open repair.
716
8-A 56 year old female cardiac patient underwent catheterization by femoral approach,and after 2 monthes she complained of LL oedema,varicose veins & local audible murmurs,your diagnosis is: a-DVT b-AVF c-Pseudoaneurysm. d-Lymphoedema. e-Postphlebetic syndrome. 9-Regarding venous drainage of the lower limb: a-The short saphenous vein drains to femoral vein. b-The long saphenous vein accompanies the femoral nerve. c-The long saphenous vein passes in front of medial malleolus. d-The sapheno-femoral junction is present 3 cm below and medial to the pubic tubercle. e-The short saphenous vein is accompanied by the femoral nerve. 10-A 75 year old overweight ,smoker person with pain in his legs after walking half a kilometer,which is relieved immediately by rest,ABI is 0.8,the most appropriate management is: a-IV heparin b-Fogarty catheter thrombectomy. c-Conservative management. d-Sympathectomy. e-PTA 11-A 75 year old male patient,he is diabetic and a smoker, presents with severe rest pain in his right leg.O/E there is fever, advanced gangrene and cellulitis of the right foot with absent distal pulses and shooting leukocytosis.Angiography shows totally occluded tibial vessels till the ankle.The best management is: a-IV heparin b-Sympathectomy. c-Thrombolysis. d-Below knee amputation. e-Above knee amputation.
717
12-For a patient with AF and acute ischaemia,treatment of choice is: a-IV heparin b-Embolectomy. c-Fasciotomy. d-Thrombolysis. e-Endovascular surgery. 13-As regards Buerger’s disease,all are true except: a-Exclusively in males. b-Exclusively in smokers. c-Atearlier age than atherosclerosis. d-Excellent distal run off on angiogram. e-Affects veins as well. 14-In ‘’LeRiche’’ the followings are true except: a-Retrograde ejaculation. b-Aortoiliac occlusion. c-Claudications in buttocks and upper thighs. d-Impotence. e-Can be treated by Aortobifemoral bypass. 15-The elevated levels of serum CPK post embolectomy is almost due to: a-Cardiac infarction. b- Volkmann’s muscle contracture. c-Compartment Syndrome. d-Rhabdomyolysis and reperfusion injury. e-Irrelevant.
ANOTHER GROUP:
718
Choose the most correct answer to the following questions: Q1- Regarding ''Acute Arterial Thrombosis'' the following is FALSE: A-It may complicate arterial aneurysm. B-It might occur in gastroenteritis in children. C- As a result to contusion of the vessel wall. D-On top of atherosclerosis. E-Non of the above. Q2-The following is NOT TRUE about ''leg ulcers'': A- Mortonell Ulcer occurs mostly in hypertensive females. B-Venous Ulcers are mostly over the medial aspect of lower leg. C-''Fibrin Cuff and Leukocyte Trapping '' theories describe the pathology of venous ulcer. D-Ischaemic ulcers are invariably insensate. E-Neuropathic ulcers are located over bony prominences. Q3-A 24 year old male patient who sustained a stab to the left groin,three hours later he was presented to the E&A with a tourniquet over his groin that had been placed by his family, his BP was 100/60 ,tachycardic ,and oliguric,no external bleeding or haematomas,he was unable to move his cold limb,ipsilateral popliteal pulse was not felt,the best management would rather be: A- Left above knee amputation. B- Femoral artery exploration. C- Urgent angiogram. D- Urgent arterial duplex. E-Anticoagulation Q4-Which of the following statements about Antiplatelet drugs is not true: A- They act effectively in arterial diseases more than venous thrombotic diseases. B- Cilostazol inhibits Phosphodiesterase. C- Clopidogrel is an ADP inhibitor. D- A common side effect is GIT bleeding. E-Pentoxifylline increases RBCs deformability . Q5-Which of the following statements about arteriovenous fistula are correct
719
A- The local features characteristic of an arteriovenous communication are demonstrated by the presence of a thrill and bruit with aneurysmal dilatation. B- An arteriovenous fistula is considered a ''soft sign'' of vascular injuries. C- The most common type of arteriovenous fistula is iatrogenic, created for dialysis access. D-An AVF is considered the commonest complication of central line insertion. E-A and C. Q6-A 28-year-old woman,non diabetic,presented to her family GP with persistent headache,and lightheadedness ,on examination her BP was 220/100,both costoverteberal angles shows bruit, ACE inhibitor was prescribed without improvement,,,14 days later the patient presents with stroke .The provisional diagnosis is: A-Atherosclerotic bilateral renal artery stenosis. B-Fibromuscular dysplasia. C-Takayasu's arteritis. D-Renal artery aneurysm. E-Scleroderma. Q7-Concerning AAA, the following is not true: A-Smoking is the strongest predisposing factor. B-Infrarenal AAA is the commonest type that accounts more than 95%. C-Rupture is highr in females. D-Diameter more than 3 cm is an indication for elective repair. E-EVAR has lower mortality rates than open repair. Q8-Concerning investigations of vascular diseases the following is NOT TRUE: A-In limbs with rest pain ABI is below 0.3 . B-Carotid duplex is superior than aortic arch aortography in diagnosing carotid artery disease. C-Angiography is indicated in intermittent claudications. D-Plethysmography and capillaroscopy are used in investigating digital ischaemia. E-Duplex Ultrasound shows a Triphasic flow in normal arterial tree . Q9-A 65 year old male patient,diabetic,hypertensive,heavy smoker,on statins,he is being prepared for CABG,on preoperative investigation carotid duplex showed left internal carotid
720
stenosis more than 75%,the patient has no history of TIAs or previous stroke,which statement is true: A-CAS (carotid artery stenting) is superior than CEA (carotid artery endarterectomy) regarding postoperative morbidity . B-In this case CEA or CAS should be done after CABG. C-In this case no need for CEA because the patient is neurologically asymptomatic . D-Carotid duplex is the golden standard investigation for carotid artery atherosclerotic diseases and it is superior than arch aortography. E-Dizziness can be a manifestation of carotid artery disease . Q10-A 45 year old female patient underwent THR surgery ,3rd day postoperative her ipsilateral lower limb is swollen,painful,tender with intact peripheral pulsations,she was put under full anticoagulation by UFH,2 days later the patient began to be dyspnoeic … the next step would be: A-IVC filter. B-LMWH. C-CT Pulmonary angiogram. D-Urgent pulmonary thrombectomy. E-Initiate thrombolytic therapy. GOOD LUCK from DR.ALSERR AYMAN H. K. Palestinian Board Surgical residency Training program Regular assessment Exa 16. Which hypersensitivity reaction is associated with serum sickness? a. Type I: immediate b. Type II: cytotoxic c. Type III: immune complex d. Type IV: cell mediated e. N0ne of the above. 17. The most common cause of mass in the posterior mediastinum is a. Vascular b. Esophageal diverticula c. Neurogenic tumors d. Lymphomas e. Bronchogenic cysts
18. The most common location for a gastric ulcer is
721
a. b. c. d. e.
Fundus Greater curvature Cardia Body Antrum
19. Which statement about the borders of the epiploic foramen is correct? a. The second part of the duodenum forms the inferior border. b. The quadrate process of the liver forms the superior border. c. The hepatic vein forms the posterior border. d. The free edge of the greater omentum forms the anterior border. e. The common bile duct is contained within the anterior border . 20. Regarding the management of major trauma a. Deaths follow a trimodal distribution b. Cardiac tamponade is characterised by raised BP, low JVP and muffled heart sounds c.
Assessment of uncomplicated limb fractures should occur during the primary survey
d.
Deterioration of the casualty during the primary survey should lead to the secondary survey
e. All are false 21. Regarding Warfarin: a. Has a half life of about 36 hours b. Crosses the placenta and should be avoided in pregnancy c. Doses should be reduced in liver disease d. An INR of between 2.0 and 3.0 is appropriate in the treatment of DVT e. All are true 22. Ectopic pregnancy a. Occurs in about 10% of pregnancies b. The risk is decreased in those with a history or pelvic inflammatory disease c. Usually presents between 2 and 4 months of gestation d. Patients usually have a negative pregnancy test e. All are false 23. All of the following are true about neurogenic shock except: a. There is a decrease in systemic vascular resistance and an increase in venous capacitance. b. Tachycardia or bradycardia may be observed, along with hypotension. c. The use of an alpha agonist such as phenylephrine is the mainstay of treatment.
722
Severe head injury, spinal cord injury, and high spinal anesthesia may all cause neurogenic shock. e. All of the above are correct. 24. Which of the following statements about extracellular fluid is true? a. The total extracellular fluid volume represents 40% of the body weight. b. The plasma volume constitutes one fourth of the total extracellular fluid volume. c. Potassium is the principal cation in extracellular fluid. d. The protein content of the plasma produces a lower concentration of cations than in the interstitial fluid. e. The interstitial fluid equilibrates slowly with the other body compartments. d.
25. The following cause hypercalcaemia except : a.
Sarcoidosis
b.
Primary hyperparathyroidism
c.
Acute pancreatitis
d.
Metastatic bronchial carcinoma
e.
Milk-Alkali syndrome
26. Advantages of epidural analgesia include: a. Earlier mobilization after surgery. b. Earlier return of bowel function. c. Shorter hospitalizations. d. Decreased stress response to surgery. e. All of the above. 27. The radiographic findings indicating a torn thoracic aorta include: a. Widened mediastinum. b. Presence of an apical “pleural cap.” c. Tracheal deviation to the right. d. Left hemothorax. e. All are true 28. For a 22-kg baby the maintenance daily fluid requirement is approximately which of the following? a. 1100 ml b. 1250 ml c. 1550 ml d. 1700 ml e. 1850 ml 29. The transfusion of fresh frozen plasma (FFP) is indicated for which of the following reasons? a. Volume replacement. b. As a nutritional supplement.
723
c. Treatment of prolonged bleeding time. d. For the correction of abnormal PT secondary to warfarin therapy, vitamin K deficiency, or liver disease. e. All of the above.
30. A 9 month old boy presents with an acute scrotal swelling. The following diagnoses are likely: a. Epididymitis b. Orchitis c. Torsion of the testicular appendage d. Irreducible inguinal hernia e. Acute idiopathic scrotal oedema
31. Regarding Tetanus all of the following are true except : a.
Is due to an infection with a gram-positive spore forming rod
b.
The organism produces a powerful endotoxin
c.
The toxin prevents the release of inhibitory neurotransmitter
d.
Clostridium tetani is sensitive to penicillin
e.
Risus sardonicus is the typical facial spasm
32. Infantile hypertrophic pyloric stenosis a.
Occurs with a male: female ratio of 4:1.
b.
Invariably presents between six and eight months of age
c.
Typically presents with bile stained projectile vomiting
d.
Surgical treatment is by Heller's Cardiomyotomy
e. all are false 33. In obstructive jaundice: a. Urinary conjugated bilirubin is increased b. Serum unconjugated bilirubin is increased c. Urinary urobilinogen increased d. Serum conjugated bilirubin is reduced e. Faecal stercobilinogen is increased 19. Incisonal hernias all are true except
724
a.
10 % of all hernias
b.
Complication is common
c.
Operative technique is most important cause
d.
Usually easy to reduce
e.
Multiloculated sac
20. Diffuse axonal injury (DAI) results from what type of force acting on the brain? a. direct impact b. axial loading c. linear acceleration d. rotational acceleration e. accceleration deceleration injury PALESTINIAN MEDICAL COUNCIL SURGERY EXAM, April 2009
Paper (1)
Candidate’s Name:……………………………………………………………………………………………… Choose the best answer in each of the following questions: 1. Anaphylactic shock: A. is an immune-mediated reaction. B. results in mast cell activation and increased circulating histamine concentrations. C. produces microcirculatory changes similar to hypovolaemic shock. D. requires prompt treatment with parenteral adrenaline and hydrocortisone. E. may occur after ingestion of drugs 2. A 53-year-old female has had a high fever with cough productive of yellowish sputum for the past 2 days. Aus-cultation of the chest reveals a few crackles in both lung bases. A chest radiograph reveals bilateral patchy pulmonary infiltrates. Which of the following inflammatory cell types will be seen in greatly increased numbers in a sputum specimen? A. Macrophages B. Neutrophils C. Mast cells D. Small lymphocytes E. Langhans giant cells 3. A 23-year-old female receiving chronic corticosteroid therapy for an autoimmune disease underwent minor surgery for incision and drainage of an abscess on her upper outer right arm. The wound healed poorly over the next month. Which of the following aspects of wound healing is most likely to be deficient? A. Re-epithelization B. Fibroblast growth factor elaboration C. Collagen deposition D.Serine proteinase production E.Neutrophil infiltration 4. Estrogen is required for normal reproductive function in the male. Where is the principal site of estrogen synthesis in the male? A. Leydig cells B. Osteoblasts C. Liver cells D. Prostate cells E. Zona gloerulosa cells 5. Concerning fluid and electrolyte imbalance:
725
A. in intestinal obstruction, more fluid is generally lost with lower than with higher obstruction. B. thirst occurs following a reduction of total body water by (1-2% 350-700 rnl), C. sodium depletion is generally treated by intravenous infusion of hypertonic saline. D. administration of diuretics may lead to retention of potassium, E. potassium depletion may result in EGG changes including a decreased QT interval, depressed ST segment and T wave inversion. 6. In premenopausal women inflammatory carcinoma breast with metastasis to axillary lymph nodes ,treatment of choice is : A. Modefied radical mastectomy + chemotherapy B. Modefied radical mastectomy +radiotherapy C. Simple mastectomy + radiotherapy D. Chemotherapy+ radiotherapy E. Simple mastectomy + radiotherapy + hormonal therapy 7. Preoperative investigation and preparation of a patient with obstructive jaundice should include: A. measurement of coagulation status. B. measurement of 24-hour urinary output. C. measurement of serum urea and electrolytes. D. fluid restriction during the 24 hours preoperatively. E. administration of antibiotics during invasive diagnostic procedures (e.g. PTC, ERCP) 8. A laceration to the left hand of an 18-year-old male was sutured, and after the sutures were removed a week later, healing continued. However, the site of the wound became disfigured by a prominent raised, nodular scar that developed over the following 2 months. What process occurred? A. Organization B. Dehiscence C. Resolution D. Keloid formation E.Secondary union 9. While shaving one morning, a 23-year-old male nicks his upper lip with the razor. Within a second after this injury, blood loss from a small dermal arteriole is reduced through A. Activated protein C B. Vasoconstriction C.Platelet aggregation D. Neutrophil chemotaxis E.Fibrin polymerization 10. A surgeon is performing a cholecystectomy. Before removing the gallbladder, the cystic artery must be ligated. Within which of the following peritoneal structures is the cystic artery located? A. Falciform ligament B. Gastrocolic ligament C. Gastrohepatic ligament D. Hepatoduodenal ligament E. Splenorenal ligament 11. Most common complication of Adjustable gastric banding is (LAGB) A. Band slippage B. Erosion in stomach
726
C Anastomotic leak D. Port access problems E. sever osteoporosis 12. In which of the following organs is an arterial thromboembolus least likely to produce an infarct? A. Brain B. Liver C.Kidney D. Heart E. Spleen 13. A 60-year-old female has been hospitalized for 3 weeks after an auto accident in which she sustained fractures to the right femur, pelvis, and left humerus. Physical examination reveals swelling and warmth in her left leg, with local pain and tenderness in her left thigh. She is otherwise asymptomatic. A. Air embolus B. Amniotic fluid embolus C. Chronic passive congestion D. Fat embolus E. Phlebothrombosis 14. The catabolic phase of the metabolic response to injury: A. is independent of the injurious insult. B. is characterised by an increase in ADH secretion. C. is characterised by sympathomimetic overdrive. D. is impaired in malnourished patients. E. is potentiated in multiply traumatised patients. 15. The root of the mesentery crosses all of the following structures EXCEPT A. Ascending and horizontal parts of the duodenum B. Abdominal aorta C. Inferior vena cava D. Right ureter E. Splenic artery 16. Fibroadenomas of the breast all are true except: A. Are benign monoclonal neoplasms B. Most commonly present in late adolescence or the early 20s C. Should be diagnosed by triple assessment D. At least 30% reduce in size over a 2 year period E. Can be safely managed conservatively 17. A 1.2-cm, darkly pigmented skin lesion is excised from the dorsum of the right hand of a 42-year-old male, who notices that the lesion had become larger during the previous month. Microscopically, a malignant melanoma is present. Which of the following factors presents the greatest risk for development of this neoplasm? A. Smoking B. Ultraviolet radiation C. Chemotherapy D. Asbestos exposure E. Allergy to latex
727
18. A 60-year-old male presents with bloody diarrhea, abdominal cramps, and fever. He is admitted to the hospital for workup. Sigmoidoscopic examination reveals ulceration in the cecum and ascending colon. Microscopic examination of a colonic biopsy shows flask-shaped mucosal ulcers with extensive necrosis and a modest, nonspecific inflammatory response. The ulcers do not penetrate the muscularis propria. The most likely agent responsible for this patient's diarrhea and mucosal ulceration is A. Giardia lamblia B. Entamoeba histolytica C. Shigella spp. D. Salmonella spp. E. Vibrio cholera 19. A 43-year-old female has been taking oral contraceptives for many years and has noticed no side effects. She is at a higher risk than nonusers of developing which of the following tumors? A. Endometrial cancer B. Hepatic adenoma C. Ovarian cancer D. Gallbladder cancer E. Pancreatic adenoma 20. A large "port wine stain" involves the left side of the face of a 3-year-old child. This irregular, slightly raised, red-blue area is not painful, but is very disfiguring. Histologically, this lesion is most likely composed of a proliferation of : A. Neuroblasts B. Lymphatics C. Fibroblasts D. Lymphoblasts E. Capillaries 21. A 2-month-old infant has been cyanotic since birth. The baby has worsening congestive heart failure. Based on the gross appearance of the heart shown here. this infant's congenital heart disease is best classified as A. Tetralogy of Fallot B. Pulmonic stenosis C. Truncus arteriosus D. Transposition of the great vessels E. Aortic stenosis 22. A 45-year-old male has experienced a 5-kg weight loss over the past 3 months after the loss of his job. He recently developed a low-grade fever and cough with mucoid sputum production, and after a week, he noticed blood-streaked sputum. A chest radiograph shows bilateral upper lobe consolidations and focal cavitations. Which of the following diagnostic tests on sputum is most warranted in this situation? A. Acid-fast stain B. GMS stain C. Gram stain D. Cytologic smear E. Viral culture 23. A 70-year-old man ingests large quantities of nonsteroidal anti-inflammatory drugs (NSAIDS) for his chronic degenerative arthritis of the hips and knees. He has observed some recent epigastric pain along with nausea and vomiting. He then has an episode of hematemesis. Which of the following lesions will gastric biopsies most probably show?
728
A. Epithelial dysplasia B. Hyperplastic polyp C. Acute gastritis D. Adenocarcinoma E. H. pylori infection 24. A 57-year-old female noticed burning epigastric pain after meals for more than 1 year. An endoscopy reveals an erythematons patch in the lower esophageal mucosa. A biopsy of this lesion shows basal sqnamous epithelial hyperplasia, elongation of lamina propria papillae, and scattered intraepithelial nentrophils with some eosinophils. These findings are most typical for A. Barrett esophagus B. Esophageal varices C. Reflux esophagitis D. Scleroderma E. Iron deficiency 25. A 65-year-old female has a positive stool occult blood test on a routine physical examination. A barium enema reveals numerous 1-cm outpouchings of the sigmoid and descending colon. Which of the following complications is most likely to occur from her colonic disease? A. Adenocarcinoma B. Pericolic abscess C. Bowel obstruction D. Malabsorption E. Toxic megacolon 26. A 51-year-old male experiences the sudden onset of massive emesis of bright red blood. There have been no prior episodes of hematemesis. He is known to be hepatitis B surface antigen positive. His hematemesis is most likely a consequence of which of the following abnormalities of the esophagus? A. Varices B. Barrett esophagus C. Candidiasis D. Reflux esophagitis E. Squamous cell carcinoma 27. Which of the following is greater after birth than before birth? A. Flow through the foramen ovale B. Pressure in the right atrium C. Flow through the ductus arteriosus D. Aortic pressure E. Central venous pressure 28. A 61-year-old male has had ascites for the past year. After a paracentesis with removal of 1 L of slightly cloudy, serosanguinous fluid, physical examination reveals a firm, nodular liver.Laboratory findings include positive serum HBsAg and presence of hepatitis B core antibody. He has a markedly elevated serum alpha-fetoprotein (AFP) level. Which of the following hepatic lesions is he most likely to have? A. Hepatocellular carcinoma B. Massive hepatocyte necrosis C. Marked steatosis D. Wilson disease E. Autoimmune hepatitis
729
29 .A 26-year-old male presents with vague abdominal pain in the emergency room following a high velocity motor vehicle accident. Initially, reported as hemodynamically stable, while in ER, a blood pressure of 65/30 mmHg is recorded with a heart rate of 130/min. FAST examination revealed free fluid in the peritonium. The patient is taken to the operating room for immediate exploratory laparotomy. Upon entry into the peritoneum, massive hemorrhage is noted from the liver. What is the most appropriate next step ? A. partial hepatectomy B. Pringle maneuver C. direct suturing of any noticeable lacerations D. pause to allow anesthesia to provide adequate resuscitation before further exploration E. packing of the liver 30. A 55-year-old hypertensive man develops sudden onset of excruciating pain beginning in the anterior chest, and then radiating to the back. Over the next 2 hours, the pain moves downward toward the abdomen. Which of the following is the most probable diagnosis? A. Aortic dissection B.Aortic valve stenosis C. Atherosclerotic aneurysm D. Myocardial infarction E. Syphilitic aneurysm 31. Precipitating physiological events in the metabolic response to injury include: F. haemorrhage. G. pain. H. ischaemia. I.
Endotoxaemia
J. all the above 32. A patient arrives in the emergency room after having been stabbed. He has suffered a penetrating wound in the left fourth intercostal space immediately lateral to the sternal border. Which of the following thoracic structures is most likely to have been injured? A. Left atrium B. Left ventricle C. Right atrium D. Right ventricle E. Upper lobe of the left lung 33. A medical student inserting an intercostal drain for the first time forgets her anatomy and passes it at the lower border of the left eighth rib. The structure most likely to be damaged is the A. Intercostal artery B. Intercostal nerve C. Intercostal vein D. Internal intercostal muscle E. stomach 34. A left neck exploration is performed for a stab injury to the neck in an otherwise healthy male. The left internal jugular vein is found to be completely transected with significant bleeding and hemodynamic instability. No other vascular injury is identified. Appropriate management of the internal jugular vein is: A. primary repair B. interposition saphenous vein graft
730
C. interposition with 6 mmPTFE D. ligate the left internal jugular vein E. external jugular vein transposition 35. A 6-year-old, who is learning to ride a bike, falls and fractures her clavicle. Which muscle will help prevent the fractured bone from damaging the subclavian vessels or large nerves of the arm? A. Deltoid B. Pectoralis major C. Sternocleidomastoid D. Subclavius E. Trapezius 36. A 43-year-old man presents with complaints of difficulty raising his right arm above his head. He reports that he had an accident at home in which a heavy object had fallen onto his right shoulder. Physical examination reveals that when the patient presses his hands against a wall, the medial border of his right scapula protrudes from his back. Which of the following nerves was most likely injured? A. Axillary nerve B. Dorsal scapular nerve C. Long thoracic nerve D. Suprascapular nerve E. Thoracodorsal nerve 37. A 72-year-old woman complains of constipation and abdominal pain. Over a period of 48 hours, her symptoms worsen, and she is transported to the hospital for laparoscopic evaluation of an acute abdomen. At laparoscopy, a volvulus is noted. Which of the following is the most likely location for the volvulus? A. Appendix and cecum B. Ascending colon C. Descending colon D. Sigmoid colon E. Transverse colon 38. A 22-year-old man is found to have a seminoma of one testicle. If metastasis has occurred, lymph nodes at which of the following sites are most likely to be involved early in the disease? A. Deep inguinal B. External iliac C. Internal iliac D. Para-aortic E. All of the above 39. Cardinal signs of intracranial hypertension include all of the following except: A. flexor (decorticate) posturing B. papilledema C. aphasia D. dilated and nonreactive pupil E. tachycardia 40. A 33-year-old woman develops a reducible mass of the groin that is inferolateral to the pubic tubercle and medial to the femoral vein. Which of the following is the most likely diagnosis? A. Direct inguinal hernia B. Femoral hernia C. Incisional hernia D. Indirect inguinal hernia E. Umbilical hernia
731
41. An angiographic study of the distal part of the circle of Willis requires access to the vertebral artery. Access to this artery can be obtained via the A. Anterior triangle of the neck B. Muscular triangle of the neck C. Posterior triangle of the neck D. Submental triangle E. Suboccipital triangle 42. A 22-year-old male presents into the ER after sustaining a stab wound to the left chest. The injury is 2 cm left of the sternum at the level of his nipple. Initial vital signs include a pulse rate 88 /min, GCS 15, respiratory rate 12 breaths/min and blood pressure 139/74 mmHg. Initial management of this patient should be: A. Chest X-R B. pericardiocentesis C. left chest tube thoracostomy D. esophagogram E. echocardiogram 43. The most common outcome of pulmonary thromboembolism is A. Sudden death B. Cor pulmonale C. Hemoptysis D. Dyspnea E. No symptoms 44. Which of the following tends to decrease potassium secretion by the cortical collecting tubule? A. Increased plasma potassium concentration B. A diuretic that decreases proximal tubule sodium reabsorption C. A diuretic that inhibits the action of aldosterone ( e.g., spironolactone) D. Acute alkalosis E. High sodium intake 45. Which of the following would be expected to cause a decrease in extracellular fluid potassium concentration ( hypokalemia) at least in part by stimulating potassium uptake into the cells? A. 3-Adrenergic (blockade) B. Insulin deficiency C. Strenuous exercise D. Aldosterone deficiency ( Addison's disease) E. Metabolic alkalosis 46. Chronic gastritis is often associated with which of the following? A. Microcytic anemia B. Hyperchlorhydria C. Steatorrhea D. Pernicious anemia E. Prolonged use of PPI 47. Which of the following components of bile is critical for fat digestion? A. Calcium salts B. Lecithin
732
C. Bilirubin D. Bicarbonate E. Cholesterol 48. Absorption of water in the small intestine occurs by which of the following? A. Passive diffusion B. Active transport C. Solvent drag D. Couple transport with glucose 49. Damage to the pelvic nerves does which of the following? A. Has little, if any, effect on the defecation reflex B. Attenuates the defecation reflex C. Increases the strength of the defecation reflex D. Results in a continuous urge to defecate( rectal urgency). 50. Which of the following are paired correctly? A. Cholecystokinin—gallbladder relaxation B. Secretin—relaxation of the sphincter of Oddi C. Secretin—stimulation of enzymatic release from the pancreatic acinar cells D. Cholecystokinin—relaxation of the sphincter of Oddi 51.Diffuse axonal injury (DAI) results from what type of force acting on the brain? A. direct impact B. axial loading C. linear acceleration D. rotational acceleration E. accceleration deceleration injury 52. A 35-year-old trauma victim requires hyperalimentation. The patient’s injuries include a stable undisplaced fracture of the third tho racic vertebra, a closed head injury, multiple upper and lower extremity fractures and bilateral pulmonary contusions requiring ventilatory support. Which of the following are the most appropriate site and type of venous access in this patient? A. bilateral antecubital fossae, 18G peripheral intravenous catheters B. femoral vein, central venous catheter C. dorsum of one foot, single 16G peripheral intravenous catheter D. subclavian vein, central venous catheter E. long saphenous vein cut-down, long catheter 53. Which of the following hormones is largely unbound to plasma proteins'? A. Cortisol B. Thyroxine ( T4) C. Antidiuretic hormone D. Estradiol E. Progesterone 54. A 30-year-old woman is breast-feeding her infant. During suckling, which of the following hormonal responses is expected? A. Increased secretion of antidiuretic hormone (ADH) from the supraoptic nuclei B. Increased secretion of ADH from the paraven-tricular nuclei C. Increased secretion of oxytocin from the par-ventricular nuclei D. Decreased secretion of neurophysin (E) Increased plasma levels of both oxytocin and ADH
733
55. A 22-year-old male presents to the emergency depart ment after sustaining a gunshot wound to the right upper quadrant of his abdomen. On arrival, the patient is lethargic. His vital signs are significant for a systolic blood pressure of 85 mmHg with a heart rate of 130. As part of his therapy, fluid resuscitation is initiated. Which of the following is currently considered the best resuscitation fluid? A. Hartmann's solution B. albumin C. dextran D. 5% dextrose in 0.45% sodium chloride E. 7.5% sodium chloride 56. A 41-year-old female presents to the emergency department after sustaining a gunshot wound to the abdomen, with injuries to the liver and large bowel. Despite successful resuscitation and operative inter- vention, the patient dies 2 weeks later of multisystem organ failure in the intensive care unit. Which organ most likely first experienced dysfunction? A. liver B. gastrointestinal tract C. lung D. kidney E. heart 57. Contrast radiological investigative techniques include: A. a barium meal to investigate the stomach and small bowel. B. a small bowel enema to investigate the colon and rectum. C. a barium swallow to investigate dysphagia. D. a single contrast barium enema to detect small colonic polyps. E. an intravenous cholangiogram to investigate obstructive jaundice 58. A 23-year-old male presents to the emergency department by ambulance after an altercation in which he sustained trauma to his chest ,head and neck. According to witness reports, the patient was dragged on the floor by his head. On arrival, the patient puls was 50/min. His Glasgow Coma score is five. A head CT reveals a large epidural hematoma while a cervical spine CT demonstrates bilateral vertebral facet dislocations at the level of C4. What is the likely cause of his hypotension? A. head trauma B. hypovolemia C. massive vasodilatation D. alcohol intoxication E. blunt cardiac injury 59- A 30-year-old, overweight female is a school teacher who is known to be a strict disciplinarian in the classroom. She presents with angina pectoris, and coronary angiography reveals 75% narrowing of the anterior descending branch of the left coronary artery. Which of the following would be a major risk factor for coronary atherosclerosis in this patient? A. Obesity B. Type A personality C. Diabetes mellitus D. Sedentary life style E. Age 60. A 35-year-old female involved in a motor vehicle accident presents to the emergency department with suspected multiple organ damage. After the patient is stabilized, a chest radiograph is performed revealing opacification of the left side of the patient’s chest. A traumatic diaphragmatic injury is immediately suspected. The next step in management is: A. immediate exploratory laparotomy B. delayed thoracotomy
734
C. video-assisted thoracoscopy D. barium swallow E. nasogastric tube insertion 61. Intravenous administration of normal daily fluid and electrolyte requirements includes: A. provision of sodium as a 1.8% sodium chloride solution. B. provision of a total volume of 3.0 litres. C. administration of an intravenous bolus of 40 mmol of potassium chloride D. administration of the average daily sodium requirement as 2 litres of 0.9% sodium chloride. E. administration of 1 liter of 5% dextrose and 2 litres of 0.9% sodium chloride . 62. A 40-year-old electrical lineman is injured on a electric pole and is brought to the emergency department unconscious. He has an area on his left hand that is charred, evidence of thermal injury to the arm, and an exit wound on his right knee. Which of the following interventions is least appropriate in the emergency department? A. escharotomy of the left arm B. CT scan of the abdomen and pelvis C. ET intubation D. CT scan of the head and neck E. measurement of serum creatinine phosphokinase levels 63. Regarding radial nerve, all are true except: A. Is a branch of the posterior cord of the brachial plexus B. Is derived from the posterior primary rami of C5 to C7 C. Supplies the extensors of the arm D. Gives rise to the posterior interosseous nerve E. Injury produces a classical wrist drop 64. Enteral feeding: A. is contraindicated in patients with pseudobulbar palsy. B. is contraindicated in mechanically ventilated patients. C. has no advantages ove'r parentera! feeding in the multiply traucoatised patient. D. promotes bacterial translocation from the gastrointestinal tract. E. may be complicated by lobar pneumonia. 65. Following a road traffic accident (RTA), a 23-year-old trauma patient undergoes a contrast enhanced CT scan of the abdomen which reveals a large right perinephric hematoma with associated contrast extrava sation, failure of the right kidney to uptake contrast, and a normal appearing left kidney. The patient is hemodynamically stable and with no other intraab dominal injuries. Appropriate management of the right kidney is A. admit for observation of residual renal function B. retroperitoneal exploration C. exploration through Gerota’s fascia to exclude a parenchymal injury D. radiologic vascular stent placement E. nephrostomy tube and n-acetylcystein. 66. Management options for suprarenal inferior vena cava (IVC) injuries include all except: A. ligation B. lateral venorraphy C. spiral saphenous vein graft D. extraanatomic bypass
735
E. panel graft using saphenous vein 67. At abdominal exploration for penetrating trauma, a portal vein laceration is identified. The patient has gross hemoperitoneum, a transverse colostomy with gross spillage, and severe hypotension. Management of the portal vein includes all except: A. Pringle maneuver B. ligation portal vein C. lateral portal vein venorraphy D. end-to-end anastomosis E. portocaval shunt 68. Clinical features of shock include: A. a low cardiac output in the majority of cases. B. a rapid thready pulse. C.an increased respiratory rate. D a fall in urine output, E.warm clammy extremities in the majority of cases. 69. The intravenous fluid that a 60 kg., 30-year-old woman with an 80% burn should be given in the first 24 hours following burn injury is: K. 19.2 liters of 5% glucose in lactated Ringer's. L. 14.4 liters of lactated Ringer's. M. 9.6 liters of hypertonic salt solution (sodium concentration 200 mEq) per liter.. N. 7.2 liters of 5% albumin solution. O. 5.5 liters of the pentafraction component of hydroxyethyl starch. 70. Which of the following is not an indication for surgery in GERD? A. Young Age with symptoms controlled on proton pump inhibitors. B. Progressive respiratory symptoms C. Barrett's oesophagus D. Hiatus hernia with symptoms controlled on medicines E. Noncompliant symptomatic patient 71. which of the following can not prevent gastroesophageal reflux A. looping fibers of right crus of diaphragm B. mucosal folds of gastroesophageal junction C. circular muscle fibers of GE sphincter D. angle made by esophagus with stomach 72. A 23-year-old graduate student comes to the emergency room complaining of sudden onset of shortness of breath while walking home from the library. He denies any significant medical history and infrequently uses an inhaler when his asthma "acts up." He appears to be in moderate distress and is breathing at a rate of 28/min.. On physical examination he is afebrile and his breath sounds are normal on the right and decreased on the left. Percussion of the left chest is hyperresonant. An anterior-posterior chest radiograph of this patient would likely show which of the following? A. An infiltrate in the left lower lobe B. A radiolucency along the left chest wall
736
C. A wedge-shaped opacity in the left lung field D. Fluid along the left costophrenic angle E. Hyperinflation of both lung fields 73. Which of the following surgical treatments for morbid obesity is obsolete: A. Gastric Bypass B. jejuno ileal bypass C. Vertical band gastroplasty D. Bilio Pancreatic Diversion E. Vertical sleeve gastroplasty 74. A 2-month-old infant has been cyanotic since birth. The baby has worsening congestive heart failure. Based on the gross appearance of the heart shown here. this infant's congenital heart disease is best classified as (A) Tetralogy of Fallot (B) Pulmonic stenosis (C) Transposition of the great vessels (D) Truncus arteriosus (E) Aortic stenosis 75. A patient who has suffered severe chest trauma in an automobile accident is found to have fluid in the right pleural space. A thoracentesis reveals the presence of milky fluid in the pleural space, with triglyceride level (127mg/100ml) management include all of the following EXCEPT : A . Thoracostomy tube drainage. B . Total parentral nutrition . C . Radiation therapy . D . Operative management . E . Medium chain triglyceride orally . 76 . Which of the following conditions in GERD have the LEAST response after Surgery, (Fundoplication)? A. a. Atypical GERD symptoms B. Patients who achieve symptomatic pain relief with PPIs C. Classical GERD symptoms D. Positive 24 hr pH study E. Positive lower oesophgeal manometry 77.All of the following are potential outcome in laparoscopic surgery EXCEPT : K. Hypercarbia L. Alkalosis M. Decrease urine out put N. Increase intracranial pressure
737
O. Increased cardiac work 78. All of the following are Contraindications to Conservation surgery EXCEPT: A. High tumour /breast ratio B. Involvement of the upper medial quadrant C. Extensive associated DCIS D. Lymphatic or vascular invasion E. High histological tumour grade 79. All of the following indicates nonresectable lung cancer EXCEPT: B. Isolated cerebral metastasis. C. Tumors of the apex of the lung> D. Subcarinal lymph node more than 1cm in PET (positron emission tomography) scan. E. Malignant pleural effusion. F. Pco2> 45 mmHg 80. In chest injury the following are indications of thoracotomy EXCEPT : A. When cardiac tamponade present. B. Massive air leak C. For all transmediastinal wounds D. Flail chest with sever lung contusion E. Clotted hemothorax 81. A 23-year-old male presents to the emergency depart ment after being involved in a motor vehicle accident. On physical examination, he opens his eyes spontanuously, he occasionally mumbles incomprehensible sounds, he localizes to painful stimulation with his right upper extremity, His pupils are 4 mm bilaterally and reactive. This patient’s Glasgow Coma Scale (GCS) score: A. 7 B. 9 C. 8 D. 10 E. 12 82. A patient sustains a complete cervical cord injury secondary to a cervical burst fracture. The radiographs reveal injury at the C6-C7 level. Assuming a complete cord injury at this level, which of the following findings on physical examination would not correlate with this level of injury? A. absent patellar tendon reflexes B. weakness with ankle plantar flexion C. weakness with elbow flexion D. weakness with finger abduction/adduction E. weakness with shoulder abduction 83. Acute Epidural haematoma :
738
A. due to meningeal artery tear B. can be treated conservatively C. classically there is contra lateral, dilated, fixed pupil D. needs operation within 24 hours E. a and c only 84. Regarding Acute subdural haematoma : A. It is due to meningeal artery tear B. has 30% mortality. C. may cause secondary brain injury D. Treated by early operation E. More urgent than epidural haematoma 85. A 20-year-old male injured his left shoulder in a foot ball-related accident. Physical examination reveals decreased external rotation and inability to abduct his shoulder. A neurovascular examination determines the patient has numbness over the proximal-lateral aspect of his upper arm. What is the explanation for the numbness on the arm? A. injury to the axillary nerve B. injury to the musculocutaneous nerve C. injury to the ulnar nerve D. injury to the medial brachial cutaneous nerve E. injury to the lateral antebrachial cutaneous nerve 86. Concerning normal water and electrolyte balance in an average 70 kg adult: A. approximately 2000-2500 ml of urine is passed in 24 hours. B. the major extracellular cation is potassium. C. approximately 300 ml of fluid is lost in faeces in 24 hours, D. approximately 1200-1500 ml of fluid is lost as water vapour ' via the skin and respiratory tract (insensible water loss). E. aldosterone acts on the kidney to conserve potassium and hydrogen ions. 87. Beurger' diseases has : B. occlusion of small and medium arteries C. thrombophlebitis D. Reynold's phenomenon E. all the above F. a and b only 88. The following physiological events occur following injury: A. urinary output decreases. B. plasma osrnolality decreases. C. basal metabolic rate increases. D. urinary excretion of potassium is increased. E. the patient's blood becomes hypercoagulable. 89. Causes of hypovolaemic shock include all the following except: A. burn injury. B. low out put intestinal fistula C. spinal cord transaction. D. myocardial infarction. E. endotoxaemia. 90. All the following are indications of skull x-ray in head injuries except
739
A. Loss of consciousness or amnesia B. Focal neurological signs C. non penetrating injury D. Scalp bruising or swelling E. Alcohol intoxication 91. Management of hypovolaemic shock involves all except: A. achievement of venous access via peripheral vein catheterisation. B. bladder catheterisation. C. immediate administration of whole blood or red cell concentrate. D. effective analgesia. E. oxygen administration. 92. Stored whole blood used for transfusion: A. contains similar amounts of coagulation factors blood. B. contains a concentration of leucocytes similar to normal blood. C. can be stored for up to 100 days at 4±2°C. D. contains normal platelet count. E. is used when rapid volume transfusion is required for a patient who has suffered major trauma. 93. In Shock all are correct except : A. is defined as inadequate cellular perfusion. B. may occur in the presence of normotension. C. may occur following cardiomyopathy. D. may follow gastrointestinal perforation. E. invariably results in sympathomimetic activity in circulation. 94. Basic nutritional requirements in a 70 kg adult: A. include 300g of carbohydrate per day. B. include 14 g of nitrogen per day. C. decrease in the presence of Crohn's disease with a high enterocutaneous fistula. D. who is starving ,are decreased-as regards total energy requirements. E. decrease if the patient is mechanically ventilated. 95. Concerning the use of a central venous catheter: A. administration of TPN is safe and does not require repetitive haematological and biochemical monitoring. B. its insertion should have mandatory electrocardiographic monitoring. C. it provides a convenient portal for blood sampling and antibiotic administration. D. if tunnelled subcutaneously it has a higher incidence of infection with endogenous staphylococcus. E. if infected it is effectively treated by administration of antibiotics via the offending catheter. 96 .A 16-year old driver,without seat belt, is involved in a rollover head-on accident. Extensive damage to the vehicle . The patient was pulled out of the car, hemodynamically unstable and was transported to a trauma center. During transportation the patient became severely hypotensive , with distended neck veins, unresponsive, with impending respiratory distress. Which of the following is considered an immediate life-threatening injury in this patient? A. pneumothorax secondary to rib fractures B. aortic intimal tear C. diaphragm rupture D. tension pneumothorax E. myocardial contusion 97. All of the following conditions are derived from the primitive embryonic foregut except: A. Bronchogenic cyst. B. Cystic adenomatoid malformation.
740
C. Gastric duplication. D. Mesenteric cyst. E. Pulmonary sequestration. 98.The metabolic response to injury is characterised by a sequence of physiological events aimed at: A. increasing core body temperature. B. conservation of sodium and water. C. mobilising glucose from fat and protein stores. D. maintaining body weight. E. enhancing immune function. 99. Fasciotomy for extremity compartment syndrome should be performed at a compartment pressure exceeding : A. 20 mmHg B. 30 mmHg C. 40 mmHg D. 50 mmHg E. none of the above 100. The following are clinical features of Horner's syndrome A. Mydriasis B. Failure of abduction of the orbit C. Ptosis D. Increased sweating on the contralateral side of the forehead E. Exophthalmos 1-Second degree burn: A-Heals in 10-21 days B-Painful C-Good capillary refill D-All the above 2- The topical treatment of choice for burns of trunk and limbs is: A-Silver nitrate B-Gentamicine C-Silver sulfa diazine D-None of the above 3- In patients with inhalation injury: A-Treat with steroids B-Do early intubation C-Give the patient low potassium diet D-None of the above 4-Severity of burns depends on: A-Depth of burn B-TBSA C-Associated disease or injury D-All the above 5-To obtain fine line scar: A-The incision should fall in the natural skin lines. B-Keep sutures for at least 10days C-Use absorbable sutures D- All the above
741
6- Finger Skin Loss is treated by: A- Split thickness skin graft B- Full thickness skin graft C- Full Thickness homograft D-None of the above 7-In third degree burn the skin is: A- Pink B- Painful C- Wet D- None of the above 8- In major burns urine out put should be: A-1ml/kg/day B-1ml/kg/hour C-10ml/kg/hour D-10ml/kg/day :The best time to do cleft palate repair is at -9 A-1 month old B-5 months old C- 1 year old D- 3 years old 10- Below knee Skin Loss with exposed tibia is treated by: A- Split thickness skingraft B- Full thickness skingraft C- Full Thickness homograft D-None of the above :Answer Key D-1 C-2 B-3 D-4 A-5 B-6 D-7 B-8 C-9 D-10
Surgical MCQ for fourth year 1. One is false regarding the diaphragmatic hernia : a) Bachdalek's hernia occurs on the left in more than 80% of patient. b) Bachdalek's hernia has no peritoneal sac. c) Morgagnis hernia usually is recognized in the middle or later decades of life. d) Traumatic diaphragmatic hernia can occur more in case of peretratic diaphragmatic injuries than in blunt one. e) Paraesophageal hernia is less common than sliding hernia.
742
2. In neonates with congenital diaphragmatic hernia, which of the following statements is true? A. The defect is more common on the right side. B. Survival is significantly improved by administration of pulmonary vasodilators. C. An oxygen index of 20 is an indication for extracorporeal membrane oxygenation (ECMO). D. Oligohydramnios is a frequent occurrence. E. Mortality is the result of pulmonary hypoplasia. 3. One is true regarding the congenital diaphragmatic hernia (CDH): t)
Half of cases have associated major congenital defect.
u)
Rarely can be diagnosed on prenatal ultrasound examination.
v)
CDH is consider a physiological emergency and not a surgical emergency.
w)
50% of CDH infants are diagnosed after the first day of life.
x)
Respiratory distress at birth is uncommon presentation.
4. One is true regarding the hiatas hernia: z.The most common herniation through the diaphragm is throughthe esophageal hiatus aa.Sliding hernia is one in which the greater curvature of the stomach rotate into the chest. bb. The paraesophageal hernia is one in which the esophagus, esophagogastric junction and proximal stomach move into the chest. cc. Sliding hernia is usually symptomatic and associated with reflux. dd. Surgical repair is rarely indicated in the treatment of paraesophageal hernia. 5. The most common type of congenital diaphragmatic hernia is caused by: A. A defect in the central tendon. B. Eventration of the diaphragm in the fetus. C. A defect through the space of Larrey. D. An abnormally wide esophageal hiatus. E. A defect through the pleuroperitoneal fold. 6. Tension pneumothorax EE.is the commonest type of chest injuries FF.Needs urgent X-Ray chest GG. Is a clinical Diagnosis HH. Causes flat neek viens II.Treated by thoracotomy tube after chest X-ray. 7. In massive haemothorax. a) about 500ml of blood in pleural cavity b) cause dyspnea & Neck veins distension. c) treated by thoracotomy tube only d) All of the above e) None of the above
743
8. regarding tension pneumothorax, the first step in the mamagement is: a) b) c) d) e)
Obtaining a stat chest x-ray. Cricothyroidectomy. Pass an endotracheal tube. Starting oxygen by a valve-mask device. Chest decompression needle.
9.The radiographic findings indicating a torn thoracic aorta include: f. Widened mediastinum. g. Presence of an apical “pleural cap.” h. Tracheal deviation to the right. i. Left hemothorax. j. All are true 10. Which of the following confirms the diagnosis of transection of the descending thoracic aorta? A. Widened mediastinum. B. Fractured first rib. C. Left pleural effusion. D. Positive aortogram. E. All of the above. 1.Regarding direct inguinal hernia a. Common in young age b.
Lies medial to inferior epigastric artery
c.
Internal ring test control it
d.
Complication is more than indirect hernia
e.
Bilateral in 20%
2.Hypovolaemic shock is characterized by: a. a low central venous pressure , low cardiac output , low peripheral resistance b. a high central venous pressure , high cardiac output , low peripheral resistance c. a low central venous pressure , low cardiac output , high peripheral resistance d. a low central venous pressure , high cardiac output , high peripheral resistance e. a high central venous pressure , low cardiac output , low peripheral resistance
3. All of the following are functions of the gallbladder except: a. absorption b. motor activity
744
c. secretion d. storage of bile e. formation of bile 4. Concerning the inguinal canal: k. It transmits the ilio-inguinal nerve l. The deep inguinal ring lies below the mid-point of inguinal ligament m. The superficial inguinal ring overlies the pubic tubercle n. Laterally , the anterior wall is made up of the external oblique aponeurosis o. Laterally, the posterior wall is formed by the conjoint tendon 5. Regarding the Appendix, all true except: u. Is typically less than 10 cm in length in the adult. v. Is located in the retrocaecal recess. w. McBurney’s point, lies 2/3 laterally from a line from umbilicus to the anterior superior iliac spine. x. The longitudinal coat of the appendix is derived from the three bands of taenia coli. y. Is supplied by branches of the inferior mesenteric artery 6. Hemostasis and the cessation of bleeding require which of the following processes? u. Adherence of platelets to exposed subendothelial glycoproteins and collagen with subsequent aggregation of platelets and formation of a hemostatic plug. v. Interaction of tissue factor with factor VII circulating in the plasma. w. The production of thrombin via the coagulation cascade with conversion of fibrinogen to fibrin. x. Cross-linking of fibrin by factor XIII. y. All of the above 7. Preoperative investigation and preparation of a patient with obstructive jaundice should include all of the following except : A. measurement of coagulation status. B. measurement of 24-hour urinary output. C. measurement of serum urea and electrolytes. D. fluid restriction during the 24 hours preoperatively. E. administration of antibiotics during invasive diagnostic procedures (e.g. PTC, ERCP) 8. A 33-year-old woman develops a reducible mass of the groin that is inferolateral to the pubic tubercle and medial to the femoral vein. Which of the following is the most likely diagnosis? A. Direct inguinal hernia B. Femoral hernia C. Incisional hernia D. Indirect inguinal hernia E. Umbilical hernia
745
9. With regards to cytokines, which of the following alternatives is correct? A. IL- is secreted mainly by lymphocytes and mediates inflammation. B. IL- 10involved in cell division and activation. C. IL-8 is secreted by macrophages and promotes chemotaxis. D. IL-2 is a major inhibitor of cell division. E. TNF is produced by T cells and is associated with a rise of immature neutrophils in the blood circulation. 10. Low molecular weight heparin (LMWH) produces its primary effects because of its inhibition of which factor? A. ha B. IXa C. Xa D. XIa E. XhIa 11. Which of the following will immediately delay or cancel an elective surgical case if not obtained appro priately preoperatively? A. CBC B. urinalysis C. CXR D. informed consent E. ECG 12. Preoperative bowel preparation helps reduce which of the following postoperative complications by 40%? A. pneumonia B. urinary tract infection (UTI) C. dehiscence D. wound infection E. intraabdominal abscess 13.Which hormone is produced by the kidney A. calcitonin B. erythropoietin C. 25 hydroxyvitamin D D. aldosterone E. antidiuretic hormone 14.Which of the following is true of oxygenation in septic shock? A. Oxygen delivery is elevated while oxygen extraction is decreased. B. Oxygen delivery is reduced while oxygen extraction is increased C. Arterial-venous oxygen difference is increased D. Both oxygen delivery and extraction are increased E. Mixed venous oxygen is decreased 15. Which of the following is the major stimulus for aldosterone secretion?
746
f.
Angiotensin II.
g.
ACTH.
h.
Hypokalaemia.
i.
Hypernatraemia.
j.
Cortisol.
16. All the following complications can occur following blood transfusion except: a. Hyperkalaemia. b. Acidosis. c. DIC. d. Hypercalcaemia. e. Shift of O2 dissociation curve to the left.
17. High cardiac output is present in: a. Cardiogenic shock. b. Anaphylactic shock. c. Early septic shock. d. Neurogenic shock. e. Late septic shock. 18. Which of the following is not true about neurogenic shock? a. It may follow spinal fractures. b. There is tachycardia. c. The extremities are warm. d. There is bradycardia and hypotension. e. Vasopressors may be useful 19. The coagulation cascade is activated as part of the acute inflammatory response by means of: A. factor XIII B. factor VII C. factor VIII D. prekallikrein E. protein 20.Which of the following is not an action of angiotensin II?
747
A. aldosterone secretion B. sodium absorption C. efferent arteriolar constriction D. arterial dilation E. nephrosclerosis in the kidney 21.Cholecystokinin A. relaxes the sphincter of Oddi . B. inhibits gastric acid secretion C. inhibits gallbladder contractions D. causes mesenteric vasodilation E. inhibits pancreatic exocrine secretions 22. Which is the most common swelling in the posterior triangle of the neck? A.Enlarges lymph nodes. B.Cervical rib. C.Ectopic thyroid tissue D.Pharyngeal pouch. E.brachial cyst
30. Regarding body response to trauma all are true except F. immediately after injury body reacts by decreasing energy requirement,temperature, consumption of O2 G. catabolic phase is associated with increased urinary nitrogen excretion and weight loss H. cortisol is elevated and kept elevated in burn patient up to 4 weeks I. Growth hormone, IGF(insulin like growth hormone) are secreted immediately in catabolic phase J. Both norepinephrine (NE) and epinephrine (EPI) are increased three- to fourfold in plasma immediately following injury. 29.The following metabolic responses occur in response to trauma except F. Increased growth hormone G. Increased anti-diuretic hormone H. Increased ACTH I. Increased urine osmolality J. decrease glucagone 31. A 78-year-old man with a history of coronary artery disease and an asymptomatic reducible inguinal hernia requests an elective hernia repair. You explain to him that valid reasons for delaying the proposed surgery include
748
F. Coronary artery bypass surgery 3 months earlier G. A history of cigarette smoking H. Jugular venous distension I. Hypertension J. Hyperlipidemia 37. Splenic vein thrombosis is mostly associated with : A. Pancreatitis B. Trauma C. Portal hypertension D. Hematological disorders E. Splenic abscess 40. In diabetic patients, foot ulceration is mainly: A. Ischemic B. Traumatic C. Neuropathic D. Venous E. None of the above 44. The most sensitive tissue to ischemia due to pressure is : A. Bone B. Muscle C. Skin D. Peripheral nerves Adipose tissue 53. Regarding nitrogen balance all are true except F. 20% of nitrogen is lost in stool and urine G. One gram nitrogen is present in 6.25 gm protein H. Nitrogen requirement in 60 kilo lady is around 12 gms I. Calorie requirements for each gram nitrogen is 200 Kilo calorie in non –catabolic patient J. Nitrogen should not be used as energy source 62. Diverticular disease of colon A. Sigmoid colon is the region affected in about 60% of cases B. Bleeding per rectum is common in presence of diverticulitis C. Massive diverticular bleed is usually from right sided diverticulae D. Most of diverticulitis is complicated accompanied by abscess formation E. Has no cancer risk 23. The best study /procedure for localizing hyper-functioning parathyroid(s) is : a. Ultrasound b. Calcium to phosphate ratio in blood sample from jugular vein c. Neck CT d. Operative exploration e. Sestamibi scan 20. The pressure in the lower esophageal sphincter decreases by all of the following EXCEPT a. Alcohol b. Nitroglycerin c. Anticholinergics d. Alpha adrenergics e. Cholecystokinin hormone Women who have hereditary nonpolyposis colorectal cancer (HNPCC) should also be .40 screened for f. Endometrial cancer g. Papillary cancer of thyroid h. Ampullary cancer i. Pheochromocytoma
749
j.
Hepatoma
16. Techniques for diagnosis of deep venous thrombosis include all of the following EXCEPT. f. Phlebography g. Radiofibrinogen studies. h. Ultrasonic studies. i. Venous flow rate studies j. Electrical impedance test. 17. The definitive diagnosis of PE is best made by e. Ventilation prefusion (VQ) scan f. Depression of arterial O2 g. Pulmonary angiogram h. Duplex scan. 18. Deep venous thrombosis is MOST definitively diagnosed by f. I121 fibrinogen scan. g. CT scan h. Venogram i. Indium-labeled white cell scan. j. Doppler ultrasonography. 20) Which of the following is a precipitant of DKA ? A- Stroke B- Pregnancy C- Infection D- Myocardial infection E- All of the above 23)A 40-year-old previously healthy female presents with sudden onset of a severe occipital and nuchal headache following a coughing fit. Vital signs and physical examination are normal. Which of the following is indicated? (A) Contrast CT scan of the brain followed by lumbar puncture if negative. (B) Noncontrast CT scan of the brain followed by lumbar puncture if negative. (C) MRI scan of the brain. (D) Trial of pain medication and CT scan of the head and lumbar puncture only if headache is unrelieved. (E) None of the above 11-Regarding acute limb ischaemia a. Arterial occlusion has less dramatic effects in those with underling occlusive disease b. Occurs more often in patients in sinus rhythm c. Objective sensory loss does not requires urgent treatment d. Fixed skin staining is an early sign e. Intra-arterial thrombolysis is invariably successful with no morbidity 19) The level of larynx in adults is : A) C2-C3 B) C3- C4 C) C4-C6 D) C5-C7 E) C7-T1 20) Which antidotes combination is FALSE A) Succinycholine -Neostigmine
750
B) Opioids Naloxone C) Carbon monoxide -Oxygen D) Paracetamol - N- acetylcystein E) Benzodiazepines -Flumazenil 21) Ketamine A) Has antanalgesic properties B) Is antisialogogue C) Decrease heart rate D) Must be given intramuscularly E) May cause unpleasant dreams in children :Anion gap is .2 K. The difference between measured anions and measured cations L. Is normal in lactic acidosis M.Is high in shock N. Normally is 20 mmol O. High in pancreatic fistula 9. All of the following are potential outcome in laparoscopic surgery EXCEPT : P. Hypercarbia Q. Alkalosis R. Decrease urine out put S. Increase intracranial pressure T. Increased cardiac work 17. Concerning the use of a central venous catheter: A. administration of TPN is safe and does not require repetitive haematological and biochemical monitoring. B. its insertion should have mandatory electrocardiographic monitoring. C. it provides a convenient portal for blood sampling and antibiotic administration. D. if tunnelled subcutaneously it has a higher incidence of infection with endogenous staphylococcus. E. if infected it is effectively treated by administration of antibiotics via the offending catheter. Which of the following changes would you expect to find in a patient who developed .18 ?acute renal failure after ingesting poisonous mushrooms that caused renal tubular necrosis F. Increased plasma bicarbonate concentration G. Metabolic acidosis H. Decreased plasma potassium concentration I.
Decreased blood urea nitrogen concentration
J.
Decreased hydrostatic pressure in Bowman's capsule
20.The metabolic response to injury is characterised by a sequence of physiological events aimed at: A. increasing core body temperature. B. conservation of sodium and water. C. mobilising glucose from fat and protein stores. D. maintaining body weight. E. enhancing immune function. 57. The following statement concerning the anal canal are correct except : F. It encircle the anal canal
751
G. H. I. J.
It is not attached to the anococcygeal body It is composed of striated muscle fibers It is not responsible for causing the anal canal and rectum to join at an acute angle It is innervated by the middle rectal nerve
Which of the following changes tends to increase urinary Ca++ excret .56 F. Extracellular fluid volume expansion G. Increased plasma parathyroid hormone concentration H. Decreased blood pressure I.
Increased plasma phosphate concentration
J. Metabolic acidosis
65.The structure in the free border of lesser omentum from anterior to posterior are H. Common bile duct (CBD), Hepatic artery (HA), portal Vein (PV ). I. PV, HA, CBD J. HA, PV, CBD D.PV, CBD, HA 8. Which of the following is false concerning the physiologic effects of volatile anesthetics? A. decreased tidal volume B. decreased blood pressure C. decreased respiratory rate D. bronchodilation E. decreased sensitivity to the ventilatory stimulant effects of carbon dioxide 63.
Which of the following is an early manifestation of sepsis: a. Decreased cardiac output. b. Hypoglycaemia. c. Cutaneous vasodilatation. d. Increased arteriovenous O2 difference. e. Respiratory acidosis. 64. The following conditions will increase the risk of skin tumors except: a. Xeroderma pigmentosum. b. Albinism. c. Histiocytosis. d. Solar keratosis. e. Chronic cutaneous ulcers. 65. Secondary hyperparathyroidism is associated with: a. Increased serum calcium. b. Low serum parathormone. c. Increased serum phosphorus in cases of renal osteodystrophy on hemodialysis. d. Increased serum phosphorus in cases of osteomalacia. e. Development of chief cell hyperplasia. :All of the following increase bile secretion except .61 a) Vagal stimulation
752
b) Secretin c) Cholecystokin d) Splanchnic nerve secretion ?Which is not a type of anal margin tumors .62 a) Basal cell carcinoma b) Epidermoid carcinoma c) Paget's disease (d) Bowen's disease (Anal Intraepithelial Neoplasia 68- Rectal injuries RI all are true except f. Intraperitoneal rectal injuries can be managed as colon injuries g. Extraperitoneal rectal injuries usually need proximal diversion h. Posterior surface of rectum is Extraperitoneal i. Drainage of presacral space ,distal irrigation is always part of RI treatment j. Defect in Extraperitoneal rectum
Select the Best Appropriate Answer : 1. The commonest organism causing intra abdominal abscess is : a. Gram –ve Bacilli , b. Gram +ve Bacilli , c. Staph aureus d. Gram +ve cocci e. Gram –ve cocci 2. . Electrocardiogram findings In hyperkalemia a. Depressed ST segment b. Tall peaked T wave c. Widened QRS d. All the above e. None of the above 3. All the following can cause small bowel fistula except a. Small bowel surgery in 80- 90% b. Anastomotic leak c. diverticular disease of the colon d. wound dehiscence e. Radiation injury 4. Acute Epidural haematoma : a. Due to middle meningeal artery tear b. Can be treated conservatively c. Classically there is contra lateral, dilated, fixed pupil d. Needs operation within 24 hours e. A ad c only 5. Papillary carcinoma of the thyroid : l. Is the commonest thyroid cancer m. Previous neck radiation decrease the risk n. Familial form is rare o. Is the tumor of the middle age
753
p. A and c only 6. The commonest cause of bloody nipple discharge is : a. mammary ductectasia b. carcinoma of the breast c. lactational mastitis d. duct papilloma e. fibrocystic disease of the breast 7. Ulcerative colitis a. Shows full thickness inflammation b. The rectum is almost always involved c. 50% patients have terminal ileal disease d. Enterocutaneous or intestinal fistulae are common e. The serosa is usually affected 8. The spleen a. develops from ventral mesogastrium b. red pulp has immune function c. blood flow is 600ml/ min d. In contact with Tail of pancreas e. all the above 9. Amoebic liver abscess a. usually in the upper post surface of left lobe b. Solitary in 20% c. has typically golden yellow pus d. causes pyrexia and liver enlargement e. usually treated by surgical drainage 10. Ranson criteria involves all the following except a. age < 55 years b. WBC > 16,OOO c. FBG >11.2mmol/L or 200 mg% d. LDH > 700 u/L e. SGOT >250 SF units% 11. The following causes elevation in serum amylase except a. .Perforated. duodenal ulcer b. .Perforated. gall bladder c. perforated diverticulum d. Rupture Ectopic pregnancy e. Rupture AAA 12 .Classical clinical picture of Crohn's disease is : a. colicky abdo pain exacerbated by meals b. rectal bleeding c. clinical picture of appendicitis d. sever lower abdominal pain e. repeated vomiting 13. The following are involved in the aetiology of haemorrhoids except
754
a. diet b. straining c. pregnancy d. portal hypertension e. cancer lower rectum 14. In superficial burn all are true except a. caused by direct flam & scalds b. appear moist , red c. usually heals without skin graft d. pin prick test is negative e. may form blisters .15. All the following are complications of major burn except a. acute renal failure b. gastroduodenal erosion (Curling’s ulcer) c. inhalation injury d. acute liver failure e infections 16.. Crohn's disease a. affect any part of GIT b. proximal ileum & cecum is the commonest to be affected c. skip lesions d. a and c only e. all the above 17. Haemangioma of the liver a. usually Symptom less b. usually capillary type c. main site is the upper surface of Lt lobe d. can cause cardiac failure in children e. a and d only 18. Hidradenitis suppurativa a. is inflammation of the apocrine sweat glands b. common site is the axilla c. commoner in women d. all of the above e. none of the above 19. Regarding post splenectomy sepsis, all are correct except a. incidence is related to the age b. has rapid deteriorating clinical course c. has mortality rate 50 - 80%, death within 48 hrs d .usually due to capsulated bacteria e. pneumo vaccine should be given prior to any splenectomy due to trauma 20. Complications of Angioplasty include all the following except a. blockage of the artery b. common complications of general anaesthesia c. intimal dissection d. re stenosis e. A-V fistula
755
21. In acute mesenteric artery occlusion all the following are tru except a. marked leuckocytosis b. metabolic acidosis c. acute abdominal pain d. metabolic alkalosis e. can be treated with early operation 22. The commonest cause of upper limb ischemia is a. trauma b. vaso spastic conditions c. atherosclerosis d. intrinsic trauma of the SA e. thrombo arteritis obliterans 23. Post operative pain relief is essential for a. adequate respiration b. effective coughing c. early mobilization d. all the above e. a and c 24. The commonest cause of pyrexia in the 5th post operative day is a. atelectesis of lung segment b. deep venous thrombosis c. wound infection d. urinary tract infection e. drug fever 25. Healing by first intention a. consists of epithelization and connective tissue formation b. consists of epithelization, contracture and connective tissue formation c. more resistant to for infection d. a and c e. none of the above 26. All the following affects wound healing except a. age of the patient b. type of operation c. nutritional status d. vascularity e. sepsis a Collagen synthesis is impaired by all the following except a. preoperative starvation b. long duration steroids c. long duration antibiotics d. radiation injury e. jaundice 28. All the following are actors affecting rate & extent of scar formation except a. position of the scar b. direction of the scar c. age of the patient d. ethnic group e. diabetes mellitus
756
29. Fibroadenosis of the breast a. affects age group 40th- 50th , b. is due to proliferation of duct system of the breast c. commonly affects bilateral upper outer quadrent. d. tender two weeks before menstruation e. all the above 30. Breast pain can be caused by all the following except a. hormonal effect b. tietz syndrome c. previous trauma d. cancer e. fibroadenoma 31. Regarding obstructive jaundice a. caused by hemolytic anaemia b. urobilinogen is absent in the urine c. carcinoma of the head of pancreas is the commonest cause d. normal color stool e. b and c only 32. The acute phase response includes: k. Hypothermia. l. Decreased plasma albumin. m. Hepatic sequestration of copper. n. Increased C-reactive protein. o. Neutrophil leucocytosis. 33. levels of the following substances are elevated during the acute response to injury Except : a.glucagon . b.insulin. c.catecholamine. d.glucocorticoids. e.growth hormone. 34. What is the commonest cause of low grade fever 37.2-38.3c in the 48 hrours immediately following abdominal surgery ? a. Atelectasis. b.Urinary tract infection. c.Pulmonary embolism. d.Wound infection. e.Pseudomonas colitis. 35. Anaphylaxis is characterized by all of the following EXCEPT: F) is a reaction either local or general , frequently occurs within five minutes G) causes an urticarial eruption H) is produced by IgA antibody I) causes eosinophilia J) causes degranulation of basophils and mast cells 36. Shock can best be defined as: a. Hypotension. b. Hypoperfusion of tissues to meet the minimal requirements of cells c. Hypoxemia. d. Tachycardia
757
period
e. All of the above. 37. Which of the following is not an action of angiotensin II? a. aldosterone secretion b. sodium absorption c. efferent arteriolar constriction d. arterial dilation e. nephrosclerosis in the kidney 38. Haemostasis and the cessation of bleeding require which of the following processes? z. Adherence of platelets to exposed subendothelial glycoproteins and collagen with subsequent aggregation of platelets and formation of a hemostatic plug. aa. Interaction of tissue factor with factor VII circulating in the plasma. bb. The production of thrombin via the coagulation cascade with conversion of fibrinogen to fibrin. cc. Cross-linking of fibrin by factor XIII. dd. All of the above 39. The transfusion of fresh frozen plasma (FFP) is indicated for which of the following reasons? i. Volume replacement. j. As a nutritional supplement. k. Treatment of prolonged bleeding time. l. For the correction of abnormal PT secondary to warfarin therapy . 40. Massive blood transfusion: a. is defined as replacement of at least one’s blood volume within the first 12 hours of resuscitation. b. can cause change in acid base balance c. DIC and coagulation defect is the most serious sequalae d. all are true e. none of the above 41. All of the following are functions of the gallbladder Except: a. absorption b. motor activity c. mucus secretion d. storage of bile e. formation of bile 42. Regarding Appendix all true Except: z. It is typically less than 10 cm in length in the adult. aa. Is located in the retrocaecal recess. bb. Macburneys point, lies 2/3 laterally from a line from umbilicus to the anterior superior iliac spine. cc. The longitudinal coat of the appendix is derived from the three bands of taenia coli. dd. Is supplied by branches of the inferior mesenteric artery 43. Which of the following most often initiates the development of acute appendicitis? p. A viral infection. q. Acute gastroenteritis. r. Obstruction of the appendiceal lumen. s. A primary clostridial infection. t. Typhoid infection
758
44.Regarding direct inguinal hernia f. Common in young age g. Lies medial to inferior epigastric artery h. Internal ring test control it i. Complication is more than indirect hernia j. Bilateral in 20% 45.Screening for colorectal cancer include all Except: f. Fecal occult blood g. Tumor marker ,CA 15.3 h. Double contrast barium enema i. Colonoscopy j. All of the above 46.Tuberculous cervical lymphadenitis: a. is caused by bovine tuberculous bacillus rather than the human bacillus b. usually occurs through the tonsil of the corresponding side c. collar-stud abscess is never a feature of T. B adenitis d. is treated always by surgery e. biopsy is not necessary for diagnosis 47.Concerning the inguinal canal: a. It transmits the ilio-inguinal nerve b. The deep inguinal ring lies below the mid-point of inguinal ligament c. The superficial inguinal ring overlies the pubic tubercle d. Laterally , the anterior wall is made up of the external oblique aponeurosis e. Laterally, the posterior wall is formed by the conjoint tendon 48. The most common presentation of Meckel’s diverticulum in an adult is: a. Gastrointestinal bleeding b. Intussuception c. Littre’s hernia d. Diverticulitis e. Diarrhea 49. Tension pneumothorax a. is the commonest type of chest injuries b. Needs urgent X-Ray chest c. Is a clinical Diagnosis d. Causes flat neek viens e. Treated by thoracotomy tube after chest X-ray. 50. Which of the following signs/symptoms are pathognomonic of hyperparathyroidism? a. Pathologic fractures of the metacarpals b. Calcium oxalate nephrolithiasis c. Hypercalcemia causing mental status changes d. Atrophy of Type II muscle fibers e. Osteitis fibrosa cystica
759
a. b. c. d. e.
51.Ulcerative colitis Is confined to small and large bowel Sigmoid colon is most area affected affect mucosa and sub mucosa Anal involvement is common Obstructive symptoms is common 52. With regards to cytokines, which of the following alternatives is correct? a. IL- is secreted mainly by lymphocytes and mediates inflammation. b. IL- 10 involved in cell division and activation. c. IL-8 is secreted by macrophages and promotes chemotaxis. d. IL-2 is a major inhibitor of cell division. e. TNF-a is produced by T cells and is associated with a rise of immature neutrophil circulation 53. The cell type most characteristic of chronic inflammation is the: a. macrophage b. B cell c. natural killer cell d. neutrophil e. eosinophi 53.Which of the following is true of oxygenation in septic shock? a. Oxygen delivery is elevated while oxygen extraction is decreased. b. Oxygen delivery is reduced while oxygen extraction is increased c. Arterial-venous oxygen difference is increased d. Both oxygen delivery and extraction are increased e. Mixed venous oxygen is decreased
in the blood
54.Which of the following disorders is not associated with hypocalcemia? a. sarcoidosis b. vitamin D deficiency c. renal insufficiency d. hypoparathyroidism e. pancreatitis 55.Cholecystokinin a. relaxes the sphincter of Oddi . b. inhibits gastric acid secretion c. inhibits gallbladder contractions d. causes mesenteric vasodilation e. inhibits pancreatic exocrine secretions 56. One is true regarding the hiatus hernia: a. The most common herniation through the diaphragm is through the esophageal hiatus b. Sliding hernia is one in which the greater curvature of the stomach rotate into the chest. c. The paraesophageal hernia is one in which the esophagus, esophagogastric junction and proximal stomach move into the chest. d. Sliding hernia is usually symptomatic and associated with reflux. e. Surgical repair is rarely indicated in the treatment of paraesophageal hernia.
Choose THE BEST APPROPRIATE Answer in Each of the following Questions: 1. the MOST important finding in the diagnosis of acute appendicitis is: a. vomiting
760
b. fever c. leukocytosis d. right lower quadrant tenderness e . positive Rovsing sign 2. The commonest cause of bloody nipple discharge is a. mammary ductectasia b. carcinoma of the breast c. lactational mastitis d. duct papilloma e. fibrocystic disease of the breast 3. Second degree burn: a. Heals in 10-21 days b. Painful c. Good capillary refill d. All the above
4. The following can cause Respiratory acidosis except: a. pre existing lung disease b. inadequate ventillation c. long upper abdominal incision d. all the above e. repeated vomiting 5. The classical picture of Acute arterial embolism include All the following except: k. Pallor l. Pain m. Parasthesia n. Impalpable Peripheral pulses o. Swelling 6. Severity of burns depends on: a. Depth of burn b. Total Body Surface Area (TBSA) c. Associated disease or injury d. All the above 7. Prophylactic antibiotics a. should be given one day before the operation b. should be bacteriostatic c. usually narrow spectrum d. usually stopped 24hours after the operation e. none of the above 8. Clinical picture of varicose veins includes all the following except: a. Difficulty in standing b. Heaviness & fullness in legs c. Ankle swelling at the end of day d. Itching e. leg cramps at night
761
9 . Dialy requrment per kg for the adult includes all the following except a. Water 30-50ml b. Calories 30- 50 kcal c. Nitrogen 0.20-0.35g. d. Sodium 2-3mmol e. Potassium 0.7-0.9mmol 10. Patients on long term steroid treatment Going for surgery will need to: a. Stop treatment , on the morning of surgery b. Stop treatment three days before surgery c.Continue same treatment d. Increase the dose before surgery e. decrease the dose before surgery 11.Causes of metabolic acidosis include all the following Except : a. small bowel fistula b. shock c. severe anemia d. CO poisoning e. all of the above 12. Regarding obstructive jaundice a.caused by hemolytic anaemia b. urobilinogen is absent in the urine c. carcinoma of the head of pancreas is the commonest cause d. normal color stool e. b and c only 13. Carbuncle: k. is a confluence of several boils l. usually cause minimal tissue destruction, m. has one opening n. streptococcus is the commonest organism o. commonest site is the abdominal wall 14. Pilonidal sinus: a. less common in females b. Can occur in the umbilicus c. May affect barbers d. Sometimes accidentally discovered e. All the above 15. Hidradenitis suppurativa: a. inflammation of the apocrine sweat glands b. common site is the axilla c. commoner in women d. all of the above e. none of the above 16. To obtain fine line scar: a.The incision should fall in the natural skin lines. b. Keep sutures for at least 14days c. Use absorbable sutures d. All the above
762
17. All the following are causes of post operative jaundice Except: a. Massive blood transfusion b. sepsis c. hyperperfusion d. .residual haematoma e. viral hepatitis 18. A perforated duodenal ulcer k.Usually lies on the anterior or superior surface of the duodenum . l.Usually presents with the acute onset of severe back pain. m.Produces radiological evidence of free gas in the peritoneum in over 90 percent of the patients. n.Is usually treated by vagotomy and pyloroplasty. o.Is usually treated conservatively. 19. The acute phase response includes: p. Hypothermia. q. Decreased plasma albumin. r. Hepatic sequestration of copper. s. Increased C-reactive protein. t. Neutrophil leucocytosis. 20.All regarding the flow phase after injury is true Except : a. Glucagon breaks down glycogen stores . b..hepatocytes produce glucose (gluconeogenesis). c. fat breakdown occurs as a result of catecholamine stimulation . d. growth hormone levels are elevated . e. hypoglycemia result as tissues deplete blood glucose stores. 21. levels of the following substances are elevated during the acute response to injury Except ? a.glucagon . b.insulin. c.catecholamine. d.glucocorticoids. e.growth hormone. 22. What is the commonest cause of low grade fever period immediately following abdominal surgery ? a. Atelectasis. b.Pulmonary embolism. c.Wound infection. d.Pseudomonas colitis.
37.2-38.3c in the
23. Anaphylaxis is characterized by all of the following EXCEPT: K) is a reaction either local or general , frequently occurs within five minutes L) causes an urticarial eruption M)is produced by IgA antibody N) causes eosinophilia O) causes degranulation of basophils and mast cells 24. Shock can best be defined as:
763
48 hrours
u. Hypotension. v. Hypoperfusion of tissues to meet the minimal requirements of cells w. Hypoxemia. x. All of the above. 25. Which of the following statements about extracellular fluid are true? p. The total extracellular fluid volume represents 40% of the body weight. q. The plasma volume constitutes one fourth of the total extracellular fluid volume. r. Potassium is the principal cation in extracellular fluid. s. The protein content of the plasma produces a lower concentration of cations than in the interstitial fluid. t. The interstitial fluid equilibrates slowly with the other body compartments. 26. In acute cholecystitis: k. Commonest bacteria is E.coli l. Gall bladder wall thickness more than 3 mm m. WBC between 10_15,000 cell/mm3 n. mildly elevated bilirubin level o. all of the above. 27. All of the following are true about neurogenic shock EXCEPT: a. There is a decrease in systemic vascular resistance and an increase in venous capacitance. b. Tachycardia or bradycardia may be observed, along with hypotension. c. The use of an alpha agonist such as phenylephrine is the mainstay of treatment. d. Severe head injury, spinal cord injury, and high spinal anesthesia may all cause neurogenic shock. e. A and B 28. Which of the following is not an action of angiotensin II? a. aldosterone secretion b. sodium absorption c. efferent arteriolar constriction d. arterial dilation e. nephrosclerosis in the kidney 29. Hemostasis and the cessation of bleeding require which of the following processes? ee.Adherence of platelets to exposed subendothelial glycoproteins and collagen with subsequent aggregation of platelets and formation of a hemostatic plug. ff. Interaction of tissue factor with factor VII circulating in the plasma. gg. The production of thrombin via the coagulation cascade with conversion of fibrinogen to fibrin. hh. Cross-linking of fibrin by factor XIII. ii. All of the above 30. Regarding polytrauma: k. Resuscitation in primary survey must start with airway care, breathing then circulation,.. l. X-ray after primary survey should be AP cervical spine, chest and pelvis. m. Cardiac tamponade is characterized by raised blood pressure and a low JVP. n. Assessment of uncomplicated limb fractures should occur during the primary survey. o. a and b only. 31. Hemostasis and the cessation of bleeding require which of the following processes?
764
k. Adherence of platelets to exposed subendothelial glycoproteins and collagen with subsequent aggregation of platelets and formation of a hemostatic plug. l. Interaction of tissue factor with factor VII circulating in the plasma. m. The production of thrombin via the coagulation cascade with conversion of fibrinogen to fibrin. n. Cross-linking of fibrin by factor XIII. o. All of the above 32. Massive blood transfusion: a. is defined as replacement of at least one’s blood volume within the first 12 hours of resuscitation. b. can cause change in acid base balance c. DIC and coagulation defect is the most serious sequalae d. all are true 33. Regarding direct inguinal hernia: k. Common in young age l. Lies medial to inferior epigastric artery m. Internal ring test control it n. Complication is more than indirect hernia o. Bilateral in 20% 34. All of the following are functions of the gallbladder Except: a. absorption b. motor activity c. mucus secretion d. storage of bile e. formation of bile 35. Hypovolaemic shock is characterized by: k. A low central venous pressure , low cardiac output , low peripheral resistance l. A high central venous pressure , high cardiac output , low peripheral resistance m. A low central venous pressure , low cardiac output , high peripheral resistance n. A low central venous pressure , high cardiac output , high peripheral resistance o. A high central venous pressure , low cardiac output , low peripheral resistance 36. The Appendix all true except: ee. Is typically less than 10 cm in length in the adult. ff. Is located in the retrocaecal recess. gg. Macburneys point, lies 2/3 laterally from a line from umbilicus to the anterior superior iliac spine. hh. The longitudinal coat of the appendix is derived from the three bands of taenia coli. ii. Is supplied by branches of the inferior mesenteric artery 37. Which of the following most often initiates the development of acute appendicitis? u. A viral infection. v. Acute gastroenteritis. w. Obstruction of the appendiceal lumen. x. A primary clostridial infection. 38. The inguinal canal: a. Has a deep ring, which is a defect in the transversalis fascia.
765
b. Is bounded posteriorly by the inguinal ligament. c. Has the internal oblique as part of its posterior wall throughout. d. Has the conjoint tendon superiorly. e. Transmits the ilioinguinal nerve, which enters the canal through the deep ring. 39. Which of these statements regarding Meckle’s diverticulum is correct? a. It is found in about 4% of the population. b. Is always found on the antimesenteric border of the ileum. c. Is usually about 2 cm in length. d. Is usually located about 20 cm from the ileocaecal junction. e. Is usually attached to the umbilicus. 40. The most common type of congenital diaphragmatic hernia is caused by: a. A defect in the central tendon. b. Eventration of the diaphragm in the fetus. c. A defect through the space of Larrey. d. An abnormally wide esophageal hiatus. e. A defect through the pleuroperitoneal fold.
Choose THE BEST APPROPRIATE Answer in Each of the following Questions:
SURGERY IN GENERAL 4. Patient with high risk for deep venous thrombosis (DVT) : i.
Smoker patient with operation more than 40 minutes
j.
Patient with upper abdominal surgery less than 40 minutes
k. Patient with pelvic operation for malignancy d. b and c only d. None of the above 2. ECG changes in hypokalemia : a. prolonged QT b. elevated ST, c. Q wave in lead III d. high peaked P wave e.
a and c only
3. sodium chloride 0.9% contains: q. 145mmol Na
766
r.
145mmol CL
s. Is hypertonic t.
a and b only
u. none of the above 4. Sodium balance k. Total body Na is3 500 mmol l.
Daily In take is 120-180mmol
m. Controlled by ant diuretic hormone n. Mainly stored in bones o. All the above
5. The following can cause Respiratory acidosis except a. pre existing lung disease b. inadequate ventillation c. long upper abdominal incision d. all the above e. repeated vomiting 6.The following are complications of drains Except: a. wound idehiscence b. early mobilization c. bleeding at site of stab d. bowel Herniation e. anastomotic leak 7.Absorbable sutures are: a. polyglycolic acd b..chromic catgut c. polydeanon suture d. all the above
767
e. a and b only 8. Dextrose saline fifth strength solution is: a. hypotonic solution b. contains 45 mmol sodium c. supplies 268 kcal, if given to the patient d. is similar to plasma e. non of the above 9. Prophylactic antibiotics a. should be given one day before the operation b. should be bacteriostatic c. usually narrow spectrum d. usually stopped 24hours after the operation e. none of the above 10. Clinical picture of varicose veins includes all the following except: a. Difficulty in standing b. Heaviness & fullness in legs c. Ankle swelling at the end of day d. Itching e. leg cramps at night 11 . Dialy requrment per kg for the adult includes all the following except a. Water 30-50ml b. Calories 30- 50 kcal c. Nitrogen 0.20-0.35g. d. Sodium 2-3mmol e. Potassium 0.7-0.9mmol 12. Patients on long term steroid treatment Going for surgery will need to: a. Stop treatment , on the morning of surgery b. Stop treatment three days before surgery
768
c.Continue same treatment d. Increase the dose before surgery e. decrease the dose before surgery
13.Causes of metabolic acidosis include all the following Except : a. small bowel fistula b. shock c. severe anemia d. CO poisoning e. all of the above
14. Regarding obstructive jaundice a.caused by hemolytic anaemia tumour b. urobilinogen is absent in the urine c. carcinoma of the head of pancreas is the commonest cause d. normal color stool e. b and c only 15. Carbuncle: p. is a confluence of several boils q. usually cause minimal tissue destruction, r.
has one opening
s. streptococcus is the commonest organism t.
commonest site is the abdominal wall
16. Breast abscess a. usually bilateral b. staph aureus is the commonest organism c. less common in nursing mothers d. all the above e. b and c only 17. Pilonidal sinus: a. less common in females
769
b. Can occur in the umbilicus c. May affect barbers d. Sometimes accidentally discovered e. All the above
18. Billiary fistula: a.. may follow billiary surgery b. may be internal fistula d. can cause metabolic alkalosis c. all of the above e. a and b only 19.The following are indication of surgical treatment in small bowel fistula except a. failed to close in two weeks b. associated peritonitis c. abscess formation d. malignant disease e. distal obstruction 20. All the following are causes of post operative jaundice Except: a. Massive blood transfusion b. sepsis c. hyperperfusion d. .residual haematoma e. viral hepatitis
i) 21.
ENDOCRINE AND METABOLIC RESPONSE TO SURGERY AND TRAUMA Regarding Cytokines, all of the following are true Except: a. Cytokines are low molecular weight, heterogeneous glycoproteins.
770
b. They are synthesized by activated macrophages, fibroblasts, endothelial and glial cells in response to tissue injury from surgery or trauma. c. IL-6, and other cytokines, cause the acute phase response ,which includes the production of acute phase proteins such as fibrinogen, C reactive protein, complement proteins, α2-macroglobulin, amyloid A and ceruloplasmin d. The size of IL-6 response reflects the degree of tissue damage which has occurred during surgery. e.
They produce their actions only systemically .
22. The acute phase response includes: u.
Hypothermia.
v.
Decreased plasma albumin.
w.
Hepatic sequestration of copper.
x.
Increased C-reactive protein.
y.
Neutrophil leucocytosis.
23.All regarding the flow phase after injury is true Except : a. Glucagon breaks down glycogen stores . b..hepatocytes produce glucose (gluconeogenesis). c. fat breakdown occurs as a result of catecholamine stimulation . d. growth hormone levels are elevated . e. hypoglycemia result as tissues deplete blood glucose stores. 24. levels of the following substances are elevated during the acute response to injury Except ? a.glucagon . b.insulin. c.catecholamine. d.glucocorticoids. e.growth hormone. 25. What is the commonest cause of low grade fever 37.2-38.3c in the 48 hrours period immediately following abdominal surgery ? b. Atelectasis. b.Urinary tract infection . c.Pulmonary embolism . d.Wound infection . e.Pseudomonas colitis . SHOCK 26. Anaphylaxis is characterized by all of the following EXCEPT: P) is a reaction either local or general , frequently occurs within five minutes Q) causes an urticarial eruption
771
R) is produced by IgA antibody S) causes eosinophilia T) causes degranulation of basophils and mast cells 27-Shock can best be defined as: y. Hypotension. z. Hypoperfusion of tissues to meet the minimal requirements of cells aa. Hypoxemia. bb. All of the above.
28. Which of the following statements about extracellular fluid are true? u. v. w. x.
The total extracellular fluid volume represents 40% of the body weight. The plasma volume constitutes one fourth of the total extracellular fluid volume . Potassium is the principal cation in extracellular fluid. The protein content of the plasma produces a lower concentration of cations than in the interstitial fluid. y. The interstitial fluid equilibrates slowly with the other body compartments.
29. Which of the following statements is true concerning the clinical classification system of hemorrhagic shock? i. j.
Class I shock is equivalent to voluntary blood donation In Class II shock there will be evidence of change in vital signs with tachycardia, tachypnea and a significant decrease in systolic blood pressure k. Class IV hemorrhage can usually be managed by simple administration of crystalloid solution l. Class III hemorrhage involves loss of over 40% of blood volume loss and can be classified as life-threatening 30. All of the following are true about neurogenic shock EXCEPT: A) There is a decrease in systemic vascular resistance and an increase in venous capacitance. B) Tachycardia or bradycardia may be observed, along with hypotension. C) The use of an alpha agonist such as phenylephrine is the mainstay of treatment. D) Severe head injury, spinal cord injury, and high spinal anesthesia may all cause neurogenic shock. E) A and B 31. Which of the following is not an action of angiotensin II? A. aldosterone secretion
772
B. sodium absorption C. efferent arteriolar constriction D. arterial dilation E. nephrosclerosis in the kidney
HAEMOSTASIS AND BLOOD TRANSFUSION 32. Hemostasis and the cessation of bleeding require which of the following processes? jj. Adherence of platelets to exposed subendothelial glycoproteins and collagen with subsequent aggregation of platelets and formation of a hemostatic plug. kk. Interaction of tissue factor with factor VII circulating in the plasma. ll. The production of thrombin via the coagulation cascade with conversion of fibrinogen to fibrin. mm. Cross-linking of fibrin by factor XIII. nn. All of the above
33. The transfusion of fresh frozen plasma (FFP) is indicated for which of the following reasons? m. Volume replacement. n. As a nutritional supplement. o. Treatment of prolonged bleeding time. p. For the correction of abnormal PT secondary to warfarin therapy, vitamin K deficiency, or liver disease.
34. Hemostasis and the cessation of bleeding require which of the following processes? p. Adherence of platelets to exposed subendothelial glycoproteins and collagen with subsequent aggregation of platelets and formation of a hemostatic plug. q. Interaction of tissue factor with factor VII circulating in the plasma. r. The production of thrombin via the coagulation cascade with conversion of fibrinogen to fibrin. s. Cross-linking of fibrin by factor XIII. t. All of the above
35. Massive blood transfusion: a. is defined as replacement of at least one’s blood volume within the first 12 hours of
773
resuscitation. b. can cause change in acid base balance c. DIC and coagulation defect is the most serious sequalae d. all are true 36.With regard to hemolytic transfusion reaction, which of the following statement is true ? a. They are generally caused by ABO incompatibility b. Urticaria and pruritus are the commonest symptoms. c. Acidification of the urine prevents precipitation of hemoglobin. d. The laboratory finding include a positive direct hemoglobin test result and a free hemoglobin concentration higher than 5 mg/dl in post –transfusion blood sample. E. A and D are correct . CHOLELITIASIS 37. All of the following are functions of the gallbladder Except: a. absorption b. motor activity c. mucus secretion d. storage of bile e. formation of bile 38. The clinical picture of gallstone ileus includes all except ? k. Air in the biliary tree. l. Small bowel obstruction. m. A stone at the site of obstruction. n. Acholic stools. o. Associated bouts of cholangitis.
39. What is the most common organism isolated from bile and blood cultures in patients with acute cholangitis? a. Enterobacter spp. b. Bacteroides spp. c. Escherichia coli d. Enterococcus spp. e. Candida albiccins
774
ACUTE ABDOMEN AND ACUTE APPENDICITIS 40. The Appendix all true except: jj. Is typically less than 10 cm in length in the adult. kk. Is located in the retrocaecal recess. ll. Macburneys point, lies 2/3 laterally from a line from umbilicus to the anterior superior iliac spine. mm. The longitudinal coat of the appendix is derived from the three bands of taenia coli. nn. Is supplied by branches of the inferior mesenteric artery
41. Which of the following most often initiates the development of acute appendicitis? y. A viral infection. z. Acute gastroenteritis. aa. Obstruction of the appendiceal lumen. bb. A primary clostridial infection.
42. The most helpful diagnostic radiographic procedure in small bowel obstruction is: s. t. u. v.
CT of the abdomen. Contrast study of the intestine. Supine and erect x-rays of the abdomen . Ultrasonography of the abdomen.
43. The inguinal canal: a. Has a deep ring, which is a defect in the transversalis fascia. b. Is bounded posteriorly by the inguinal ligament. c. Has the internal oblique as part of its posterior wall throughout. d. Has the conjoint tendon superiorly. e. Transmits the ilioinguinal nerve, which enters the canal through the deep ring.
44. Which of these statements regarding Meckle’s diverticulum is correct? a. It is found in about 4% of the population. b. Is always found on the antimesenteric border of the ileum . c. Is usually about 2 cm in length. d. Is usually located about 20 cm from the ileocaecal junction. e. Is usually attached to the umbilicus. CHEST SURGERY 45. One is true regarding the hiatas hernia:
775
a. The most common herniation through the diaphragm is throughthe esophageal hiatus b. Sliding hernia is one in which the greater curvature of the stomach rotate into the chest. c.The paraesophageal hernia is one in which the esophagus, esophagogastric junction and proximal stomach move into the chest. d.Sliding hernia is usually symptomatic and associated with reflux. e.Surgical repair is rarely indicated in the treatment of paraesophageal hernia.
46. The most common type of congenital diaphragmatic hernia is caused by: a. A defect in the central tendon. b. Eventration of the diaphragm in the fetus. c. A defect through the space of Larrey. d. An abnormally wide esophageal hiatus. e. A defect through the pleuroperitoneal fold. 47. Tension pneumothorax JJ. is the commonest type of chest injuries KK.Needs urgent X-Ray chest c. Is a clinical Diagnosis and can be treated primarily by Chest decompression needle. d. Causes flat neek viens e.Treated by thoracotomy tube after chest X-ray. CARDIAC SURGERY 48. Which coronary arterial vessels are most susceptible to ischaemia? a. Myocardial arterial vessels. b. Epicardial arterial vessels c. Pericardial arterial vessels d. All vessels are equally affected 49. Which of the following is not an indication for Coronary artery Bypass grafting (CABG)? a. Left main coronary artery disease stenosis more than 60%. b. One, two or three vessel disease with proximal left Anterior descending artery (LAD) stenosis c. Three vessel disease with impaired ventricular function (less than 50%) d. Double vessel disease with patent left anterior descending artery (LAD) 50. Which of the following pathological change does not occur in mitral stenosis a. Increased left atrial pressure b. Left atrium dilatation c. Ventricular hypertrophy
776
d. Embolisation of clots 51. Most common congenital anomaly associated with coarctation of aorta is a. Ventricular septal defect (VSD) b. Atrial septal defect ( ASD) c . Bicuspid Aortic valve d. Patent ductus arteriosis (PDA) 52. Structures which may be injured in mitral valve replacement are all except a.Circumflex coronary artery b. Right coronary artery c. Aortic valve d. AV node 1. In the preoperative assessment,which of the following is not suggestive of a platelet disorder? (A) Epistaxis (B) Gingival bleeding (C) Hemarthrosis (D) Easy bruising (E) Petechia 2. Which of the following is a physical finding specific for vascular structures? (A) Reducibility (B) Fluctuation (C) Transillumination (D) Compressibility
(E) Crepitancy 3. Which of the following medications must be stopped preoperatively: (A) Atenolol (B) Thyroxine (C) Albuterol (D) Aspirin (E) Enalapril
777
778
779
780
(A) Air embolism (B) Tension pneumothorax (C) Flail chest (D) Cardiac tamponade
(E) Myocardial contusion 16. The severity of hypovolemic shock has been found to correlate with the (A) hematocrit (B) pulmonary capillary wedge pressure (PCWP) (C) The percentage of volume lost (D) PaO2 (E) white blood cell count 17. A patient is transported to the ER following a fall from height . The Glasgow Coma Scale (GCS) can categorize the patient's neurologic status by assessing all of the following except: (A) Withdrawal to pain (B) Inappropriate words (C) Eye opening response (D) pupil response to light (E) Obeying commands 18. Neurogenic shock can be caused by all of the following except: (A) Spinal anaesthesia (B) Isolated head injury (C) Extremely severe pain (D) Accidental back trauma (E) Unpleasent scenes
781
19. Which of the following types of shock is characterized by a higher incidence of delayed mortality? (A) hypovolemic (B) cardiogenic (C) neurogenic (D) cardiac compressive (E) septic
20.What is the first priority in the management of trauma patients ? (A) continued intravenous hydration (B) Oxygen mask (C) Chest tube (D) Airway & C-spine control (E) Blood transfusion 21. In physical examination if an inguinal hernia is completely irreducible it is called (A) Strangulated (B) obstructed (C) Recurrent (D) Fixed (E) Incarcerated 22. Which of the following is not a characteristic of uncomplicated inguinal hernia (A) Reducible (B) Expansile (C) May be congenital or acquired (D) Pulsatile (E) May be caused by chronic cuogh 23. The extension of the existing pain to a site other than the original one is called:
782
(A) Radiation (B) Shifting (C) migration (D) Exacerbation (E) All of the above 24. All of the following symptoms and signs are suggestive of acute pancreatitis EXCEPT (A) Anorexia (B) Severe epigastric pain (C) Pain is colicky in nature (D) Vomiting (E) Ileus 25. A 42-year-old female comes to the ER complaining of right upper quadrant abdominal pain for the last 36 h, associated with fever up to 39°C, bilious vomiting, and jaundice. Direct bilirubin 2.2, alkaline phosphatase 450, WBC 19,000. What is the best cost- effective imaging study to define the etiology? (A) HIDA scan (B) Ultrasound (C) CT (D) MRI (E) Plain abdominal x-ray 26. The diagnosis of appendicitis depends mainly on : (A) CBC (B) Ultrasound (C) Clinical assessment (D) CT scan (E) None of the above
783
27. The risk of which of the following infections is markedly increased after splenectomy? (A) Candidiasis (B) Pneumococcal pneumonia (C) Cytomegalovirus (D) E.coli
(E) All of them 28. In breast examination dimpling of the overlying skin is most likely due to: (A) Breast abscess (B) Mastitis (C) Cancer (D) Fibroadenoma
(E) None of the above 29. Melena may be caused by which of the following? (A) Diverticular disease (B) Hemorrhiods (C) Peptic ulcer (D) Rectal cancer (E) Anal fissure
30. All of the following increase the risk of breast cancer except? (A) Estrogen containing oral contraceptives (B) Early menarche (C) Prolonged and multiple episodes of lactation (D) Late menopause (E) The presence of family history of breast cancer
784
MCQ in surgery
1. Adequate urine out put for adult postoperative surgical patient is greater than: a.35 ml per hour regardless of body size b. 50 ml per hour regardless of body size c. o.5 ml per kg. per hour d. 1 ml per kg. per hour e.1.5 ml per kg. per hour 2. the MOST important finding in the diagnosis of acute appendicitis is: a. vomiting b. Fever c. leukocytosis d. right lower quadrant tenderness e. referred rebound tenderness[ Rovsing sign ] 3. the most common site for intestinal obstruction due to cholecystoenteric fistula is the a. pylorus b. duodenum c. jejunum d. ileum e. sigmoid colon. 4. Common characteristics of small bowel obstruction include all of the following EXCEPT: a. ascites b. frequent progression to strangulation c. failure to pass flatus d. distention e. vomiting 5. Gastroesophageal reflux is BEST diagnosed with: a. radiography b. 24-hour pH monitoring of lower esophagus c. esophagoscopy d. documentation of a decrease in esophageal pH after HCI is placed in the stomach . e. acid-clearing swallowing test . 6.. The risk of esophageal cancer increases with all of the following EXCEPT: a. alcohol ingestion. b. smoking . c. chronic ingestion of hot beverages. d. aflatoxin. e. poor oral hygiene. 7. Achalasia is associated with all of the following EXCEPT: a. chagas ’ disease in South America b. dysphasia. c. weight loss. d. relaxation of the lower esophageal sphincter with swallowing .
785
e. aspiration pneumonia, which may cause lung abscesses. 8.Nonsurgical causes of abdominal pain include all of the following EXCEPT: a. pneumonia. b. diabetic ketoacidosis. c. acute salpingitis. d. head trauma. e. myocardial infarction. 9. Gastric acid secretion is stimulated by all of the following EXCEPT: a. sight of food. b. presence of food in the stomach. c. fat in the duodenum. d. gastrin. e. histamine. 10. All the following statementare true of anal fissure EXCEPT: a. is often associated with constipation b. pain during and after defication is typical c. the fissure is usually anterior d. the lesion is commonly seen with crohns disease e. division of the lower half of the internal sphincter is usually curative 11.Factors that are associated with the development of acute pancreatitis include all of the following EXCEPT: a. alcohol. b. gallstones. c. celiac sprue. d. hyperlipidemia. e. pancreatic divisum. 12. Complications of untreated pancreatic pseudocysts include all of the following EXCEPT: f. gastrointestinal obstruction. g. pancreatic necrosis. h. free rupture. i. abscess. j. intracystic hemorrhage. 13. Common presenting conditions in patients with pancreatic carcinoma include all of the following EXCEPT: a. esophageal varices. b. jaundice. c. weight loss. d. palpable gallbladder. e. abdominal pain. 14. All of the following are associated with an increased risk of breast cancer EXCEPT: a. dietary consumption of fat. b. history of breast cancer in first-degree maternal relatives c. age over 35.
786
d. early first pregnancy. e. infertility a. infiltrating papillary carcinoma. b. infiltrating ductal carcinoma. c. infiltrating lobular infiltrating. d. colloid infiltrating. e. medullary infiltrating. a. follicular carcinoma. b. papillary carcinoma. c. anaplastic carcinoma. d. hashimoto’s associated lymphoma. 17. Acute mastitis MOST commonly occurs at or during: a. birth. b. puberty. c. pregnancy. d. lactation. e. blunt trauma to the breast. 18. All of the following statements concerning nipple discharges are true EXCEPT: a. they may be caused by multiple lesions. b. when bloody , the discharge is due to a malignancy 70% of the time . c. a milky discharge may be due to a pituitary adenoma . d. benign duct papillomas are the most common cause of bloody discharges. e. excision of involved duct may be necessary to determine the etiology 19. The MOST frequent site for breast cancer to develop is the : a. upper inner quadrant. b. lower inner quadrant. c. lower outer quadrant. d. upper outer quadrant. e. subareolar zone. 20. The risk of bilateral breast cancer is HIGHEST if the first breast shows: a. inflammatory carcinoma. b. lobular carcinoma. c. medullary carcinoma. d. infiltrating ductal carcinoma. e. paget’s disease. 21. All of the following are true statements concerning paget’s disease of the nipple EXCEPT. a. it is very uncommon, accounting for only 2% of all breast cancers. b. it is an in situ squamous cell malignancy of the nipple. c. it is an eczematoid lesion. d. it has a better prognosis than the majority of other breast cancers. e. it can be confused with malignant melanoma histologically. 22. Initial fluid resuscitation of a patient with multiple fractures and hypovolemic shock should be : a. blood transfusion b. hypertonic saline c. fresh frozen plasma
787
d. ringer ’s lactate e. albumin 23. Which of the following findings suggests that shock in an injured patient may have a cause other than hypovolemia: f. hypotension/ g. distended neck veins. h. decreased skin temperature. i. diminished pulse pressure. j. falling central venous pressure. 24. All of the following are physical signs of both massive hemothorax and tension pneumothorax EXCEPT: f. tracheal shift. g. decreased breath sounds. h. tachycardia. i. hypotension. j. distended neck veins. 25. What is the MOST common infecting organism in overwhelming postsplenectomy infection: a. Escherichia coli. b. meningococcus. c. streptococcus. d. pneumococcus. e. staphlococcus. 26. Decreased Paco2 levels should be attained in a patient at serious risk for cerebral edema secondary to a head injury in order to :a. prevent neurogenic pulmonary edema b. allow reciprocally high levels of PaO2 in the brain c. prevent increased capillary permeability d. prevent cerebral vasodilation e. prevent metabolic acidosis 27. Brain injury alone a. frequently causes shock b. causes shock that is reversed by very simple measures c. causes shock only if the skull is intact d. rarely causes shock e. causes shock if hypoxia is superimposed 28. The level of consciousness for a head injury patient is BEST evaluated by : a. Glasgow coma scale b. response to pain c. CT scan d. papillary responses e. visual evoked potentials 29. All of the following are signs of acute vascular compromise of an extremity EXCEPT: a. diminished sensation b. pallor c. absent pulses d. gangrene e. pain
788
30. Which of the following should NOT be used to control the pain caused by fractured ribs: a. morphine IV. b. Demerol IM. c. intercostals nerve blocks. d. rib belts. e. muscle relaxants. 31. Postthrombotic varicose veins are due to: a. incompetent. b. destruction of deep veins. c. destruction incompetence. d. iliofemoral incompetence. e. block of the long saphenous vein. 32. Deep venous thrombosis prophylaxis is appropriate for all of the following patients EXCEPT: a. a 67-year-old male undergoing a colectomy. b. a 21-year-old male undergoing an outpatient open inguinal hernia repair. c. a 21-year-old male in the ICU, comatose, with a closed head injury. d. a. 60-year-old female undergoing open reduction and internal fixation (ORIF) of a hip fracture. c. a. 60-year-old female undergoing reduction of a lung carcinoma. 33.Causes of delayed union of fractures includes all the following EXCPET: ee. Compound fracture ff. Infection gg. Adequate immobilization hh. Poor blood supply ii. 34. Indication for skull X-Ray in head injury are: A. Loss of consciousness B. Focal neurological symptoms C. Difficulty in assessing the patient D. A& B only E. All the above 35. In Head injury C.T. scan is indicated in the following u) Aphasia after the injury v) Deterioration of level of consciousness w) Skull fracture with persistent headache x) A&B only y) All the above 36.Most important steps in management of head injury include: Z. Prevent hypoxia AA. Prevent Dehydration BB.Assure Brain Metabolism CC.Prevent secondary brain injury DD. All the above 37. Most common cause of solitary thyroid nodule is: A. Malignancy B. Toxic goiter C. Thyroid Adenoma D. Thyroiditis E. Hashimoto’s disease
789
38. Follicular carcinoma of thyroid gland: K. Is the commonest tumor L. Can be diagnosed by F.N.A.B M. Usually multifocal N. Commonest tumor of young age O. Prognosis is worse than papillary carcinoma 39. Abnormal nipple discharge could be due to: A. Duct papilloma B. Duct ectasia C. Retention cyst of montgomery gland D. A&B only E. All the above 40. Retracted nipple can be caused by the following EXCPET: F. Carcinoma of Breast G. Duct ectasia H. Chronic periductal mastitis I. A&B only J. All the above 41. The commonest type of the breast cyst is: A. Solitary mamary cyst B. Galactocele C. Lymphatic cyst D. Simple multiple cysts. 42. Carcinoma of breast: A. Is the third common malignant tumor of women B. common in Ages between 45-55 years. C. 10% of case are males. D. usually tender rapidly enlarging E. medullay carcinoma is the comment type . 43. Carcinoma of the breast E. may be familial F. infiltrating duct carcinoma is the commonest G. treated by lumpectomy and axillary clearance H. A&B only I. None of the above 44. Fracture of the sternum A. Is the comments fracture of thoracic cage. B. Usually painful. C. can’nt be associated with myocardial damage D. usually treaded by internal fixation . E. B&C only. 45. In massive haemothorax. F. about 500ml of blood in pleural cavity G. cause dyspnea & Neck veins distension H. treated by thoracotomy tube only I. All of the above
790
J. None of the above 46. Tension pneumothorax LL. is the commonest type of chest injuries MM. Needs urgent X-Ray chest NN. Is a clinical Diagnosis OO. Causes flat neek viens PP. Treated by thoracotomy tube after chest X-ray. Rib fractures are the most common type of chest trauma, occurring in more than 60% of patients admitted with blunt chest 47. Varicose view. A. Primary type usually affect one limb , below the knee B. Short saphenous system is more affected C. May be caused by D.VT. D. A&B only E. None of the above. 48. Anal Fissure: u) Usually anterior v) May be caused by previous anal surgery w) Can cause dark bleeding PR. x) Sometimes is painful y) Treated by steroids 49. The comments type of Anorectal abscess is: q) Ischio rectal r) Perianal s) Submucons D . Pelvirectal 50.Varicose ulcer is: k.Usually very painful & shallow l.Deep with much necrotic tissue m.More common with primary varicose vein n.A & C only e. None of the above
791